Skip to main content

Full text of "The complete mathematical and general navigation tables : including every table necessary to be used with the nautical almanac in finding the latitude and longitude : with their description and use, comprising the principles of their construction, and their direct application to plane and spherical trigonometry, navigation, nautical astronomy, dialling, practical gunnery, mensuration, guaging &c. &c."

See other formats


This  is  a  digital  copy  of  a  book  that  was  preserved  for  generations  on  library  shelves  before  it  was  carefully  scanned  by  Google  as  part  of  a  project 
to  make  the  world's  books  discoverable  online. 

It  has  survived  long  enough  for  the  copyright  to  expire  and  the  book  to  enter  the  public  domain.  A  public  domain  book  is  one  that  was  never  subject 
to  copyright  or  whose  legal  copyright  term  has  expired.  Whether  a  book  is  in  the  public  domain  may  vary  country  to  country.  Public  domain  books 
are  our  gateways  to  the  past,  representing  a  wealth  of  history,  culture  and  knowledge  that's  often  difficult  to  discover. 

Marks,  notations  and  other  marginalia  present  in  the  original  volume  will  appear  in  this  file  -  a  reminder  of  this  book's  long  journey  from  the 
publisher  to  a  library  and  finally  to  you. 

Usage  guidelines 

Google  is  proud  to  partner  with  libraries  to  digitize  public  domain  materials  and  make  them  widely  accessible.  Public  domain  books  belong  to  the 
public  and  we  are  merely  their  custodians.  Nevertheless,  this  work  is  expensive,  so  in  order  to  keep  providing  this  resource,  we  have  taken  steps  to 
prevent  abuse  by  commercial  parties,  including  placing  technical  restrictions  on  automated  querying. 

We  also  ask  that  you: 

+  Make  non-commercial  use  of  the  files  We  designed  Google  Book  Search  for  use  by  individuals,  and  we  request  that  you  use  these  files  for 
personal,  non-commercial  purposes. 

+  Refrain  from  automated  querying  Do  not  send  automated  queries  of  any  sort  to  Google's  system:  If  you  are  conducting  research  on  machine 
translation,  optical  character  recognition  or  other  areas  where  access  to  a  large  amount  of  text  is  helpful,  please  contact  us.  We  encourage  the 
use  of  public  domain  materials  for  these  purposes  and  may  be  able  to  help. 

+  Maintain  attribution  The  Google  "watermark"  you  see  on  each  file  is  essential  for  informing  people  about  this  project  and  helping  them  find 
additional  materials  through  Google  Book  Search.  Please  do  not  remove  it. 

+  Keep  it  legal  Whatever  your  use,  remember  that  you  are  responsible  for  ensuring  that  what  you  are  doing  is  legal.  Do  not  assume  that  just 
because  we  believe  a  book  is  in  the  public  domain  for  users  in  the  United  States,  that  the  work  is  also  in  the  public  domain  for  users  in  other 
countries.  Whether  a  book  is  still  in  copyright  varies  from  country  to  country,  and  we  can't  offer  guidance  on  whether  any  specific  use  of 
any  specific  book  is  allowed.  Please  do  not  assume  that  a  book's  appearance  in  Google  Book  Search  means  it  can  be  used  in  any  manner 
anywhere  in  the  world.  Copyright  infringement  liability  can  be  quite  severe. 

About  Google  Book  Search 

Google's  mission  is  to  organize  the  world's  information  and  to  make  it  universally  accessible  and  useful.  Google  Book  Search  helps  readers 
discover  the  world's  books  while  helping  authors  and  publishers  reach  new  audiences.  You  can  search  through  the  full  text  of  this  book  on  the  web 


at|http  :  //books  .  google  .  com/ 


I 

i 

n 


Digitized  by 


Google 


Digitized  by 


Google 


Digitized  by 


Google 


Digitized  by 


Google 


Digitized  by 


Google 


Digitized  by 


Google 


^i^^irnpht^n   ,md  f,^.   t,,,,,   ^gy 


Digitized  by 


Google 


Digitized  by 


Google 


LONDON  : 
PRINTED  BY  MILLS,  JOWETT  AND  MILLS, 

(late  bensley) 

BOLT-COURT,  FLEET-STREET. 


•  »     • 


Digitized  by 


Google 


TO   THE    RIGHT    HONOURABLE 

THE  (latb)  lords  COMMISSIONERS 

FOR    EXECUTING    THE   OFFICE   OF    LORD   HIGH    ADMIRAL 

OP  THE 

UNITED  KINGDOMS  OF  GREAT  BRITAIN  AND  IRELAND^ 


THE  RIGHT  HONOURABLE  ROBERT,  LORD  VISCOUNT  MELVILLE,  K.T. 
VICE-ADMIRAL  SIR  WILLIAM  JOHNSTONE  HOPE,  G.C.B. 
VICE-ADMIRAL  THE  RIGHT  HON.  SIR  GEORGE  COCKBURM,  G.C.B. 
SIR  GEORGE  CLERK,  BART.^-and 
W1LUAM  ROBERT  KEITH  DOUGLAS,  ESa 


My  Lords, 

In  brin^g  to  a  close  the  following  Treatise,  I  feel  that 
it  cannot  with  so  much  propriety  be  inscribed  tb  any  other 
department  in  the  State,  as  to  that  which  has  so  successfully 
presided  over,  and  so  long  and  judiciously  directed  the  Naval 
operations  of  Great  Britain. 

It  will  ever  be  to  me,  my  Lords,  a  cause  of  the  most  sin- 
cere gratitude,  that  to  the  condescension  of  your  Lordships, 
in  accepting  the  Dedication  of  my  mathematical  labours,  I 
am  principally  indebted  for  the  encouragement  and  support 
which  I  have  received,  in  presentiDg  the  result  of  those 
labours  to  the  Royal  Naval  Service  of  His  Majesty^  and  to 
the  Merchant  Service,  in  general,  of  the  British  Empire. 

I  have  the  honour  to  be, 

My  Lords, 

With  the  utmost  deference. 

Your  Lordships*  most  humble, 

And  most  obedient  Servant, 

THOMAS  KERIGAN. 

PtfrUmouihf 
Dtcember,  1827. 

cyAQA  QQ  Digitized  by  VjOOQ  IC 


Digitized  by 


Google 


LIST  OP   SUBSCRIBERS.  XXI 


Capt  Arthur  Fanshawe,  R.N. 

Capt.  B.  M.  Festiog,  R.N.,  Fareham,  Hants. 

Capt.  Peter  Fisher,  R.N. 

Capt.  Oshome  Foley,  R.N. 

Capt.  Foster,  R.N.,  F.R.S. 

Lieut.  Edmd.  H.  Fitzmaurice,  Scout  Revenue  Cutter. 

Mr.  Thomas  Fairweather,  Purser,  H.M.S.  Wolf. 

J.  M.  French,  esq.,  Royal  Exchange,  London. 


Rear  Admiral  John  Giffard. 

Capt.  Sir  James  A.  Gordon,  K.C.B.,  R.N.,  Resident  Commissioner^  Plymouth 

Hospital. 
Capt.  Henry  Garrett,  Resident  Commissioner,  Haslar  Hospital. 
Capt.  Robert  Gambier,  R.N. 
Capt.  George  C.  Gambier,  R.N. 
Lieut  R.  F.  Gambier,  H.M.S.  Asia. 
Capt.  J.  G.  Garland,  R.N.,  Poole. 
George  Garland ,  esq.,  Poole. 
Capt  Charles  Gordon,  H.M.S.  Cadmus. 
Capt.  Thomas  S.  Griffinhoofe,  H.M.S.  Primrose. 
Lieut.  0.  G.  Sutton  Gunning,  H.M.S.  Wellesley. 
Mr.  Jas.  Geary^  R.N.,  Portsmouth. 
Joseph  Grout,  esq.  Stamford  Hill,  Middlesex. 


•Vice  Admiral  Peter  Halkett,  Uplands,  Fareham,  Hants. 

Rear  Admiral  G.  E.  Hamond,  C.B.,  Yarmouth,  Isle  of  Wight 

Rear  Admiral  Sir  Tliomas  M.  Hardy,  hart.,  K.C.B. 

Capt  H.  C.  Harrison,  R.N.,  Southampton. 

Capt.  Henry  Haynes,  R.N. 

Capt  John  Hayes,  C.B.,  R.N.,  Shallots,  Hants. 

Capt  William  Hendry,  R.N.,  Kingston  Crescent 

Capt.  P.  Heywood,  R.N.,  Highgate,  2  copies. 

Capt.  T.  Huskissou,  R.N.,  Paymaster  of  the  Royal  Navy. 

Lieut.  George  Hales,  R.N. 

Lieut  Frederic  Hutton,  H.M.S.  Dispatch. 

Lieut  Charles  Hopkins,  (b)  R.N. 

George  Hall,  esq.,  Chichester. 

Mr.  Harrison,  Bookseller,  Portsmouth,  12  copies. 

Mr.  Harvey,  Royal  Naval  College,  Portsmouth  Dock^yard. 

Mr.  T.  S.  Herring,  Daniel  Street,  Portsea. 

Edward  James  Hopkins,  M.D.,  Queen-square,  St.  James's  Park* 


Digitized  by 


Google 


XXll  LIST  OF   SUBS^CBIBERS. 


Capt,  the  Hon.  C.  L.  Irby,  H.M.S.  Ariadne* 
Lieut.  R.  Ingram,  H.M.S.  Gloucester. 

Admiral  Jones,  10,  Curzon  Street,  May  Fair. 

Capt.  Theobald  Jones,  R.N.,  Bamibttry  Row^  Islington. 

Mr.  Jeringham,  H.M.S.  Galatea. 

The  Rev.  J.  Kirkby,  Sheemess  Dockyard. 

Capt.  Abraham  Lowe,  R.N. 

Lieut.  Gower  Lowe,  H.M.S.  Valorous. 

Alexander  Lumsdale,  esq..  Master  Attendant,  Plymouth  Dock-yard. 

Mr.  H.  Lawrence,  R.N.,  Kingston,  near  Portsea. 

Mr.  Thomas  Lock,  Weymouth,  2  copies. 

Capt.  The  Hon.  J.  A.  Maude,  H.M.S.  Glasgow. 

Capt.  Jas.  Mangles,  R.N. 

Capt.  Joseph  Maynard,  R.N. 

Capt.  W.  J.  Mingaye,  H.M.S.  Hyperion* 

Capt.  Andrew  Mitchell,  R.N. 

Capt.  John  Molesworth,  R.N.  Clapham. 

Capt.  C.  R.  Moorsom,  R.N. 

Capt.  William  Mudge,  R.N. 

Lieut  S.  Meredith,  H.M.  Cutter  Vigilant 

Capt.  C  Morton,  R.N.,  Lower  Eaton  Street,  Grosvenor  Place. 

J.  M'Crea,  esq..  Surgeon,  R.N.,  Bamsbury  Row,  Cloudesley  Square. 

John  M' Arthur,  esq.^  Hinton  Lodge^  Homdean,  Hants. 

Mr.  George  Miller,  R.N.,  Portsmouth. 

Lieut  Thomas  M'Gowan,  R.N. 

Admiral  the  Right  Hon.  Earl  Northesk,  Commander  in  Chief,  Plymouth. 
Capt.  the  Right  Hon.  Lord  Napier,  R.N. 
Lieut.  H.  Nurse,  R.N.,  Pinner,  Middlesex. 
Mr.  Joseph  Nalder,  Guildhall,  London. 


Rear  Admiral  Sir  E.  W.  C.  R.  Owen,  K.C.B.  and  M.P. 

Rear  Admiral  R.  D.  Oliver,  Dublin. 

Rear  Admiral  the  Right  Hon.  Lord  James  0*Bryen. 

Capt.  Hayes  O'Grady,  R.N. 

Capt  W.  F.  W.  Owen,  H.M.S.  Eden. 

B.  E.  O'Meara,  esq.,  Montague  Square. 

Mr.  Joseph  Oakey,  R.N. 


Digitized  by 


Google 


LIST  OV   SUBSCRIBBRS*  Xxiii 

Vice  Admiral  C.  W.  Patenon,  Coaham,  Hants. 

Capt.  Lord  William  Paget,  WiUiam  and  Mary  Yacht. 

Cape  William  E.  Parry,  F.R.S.,  R.N.,  Hydrographer  td  tlie  Admiralty. 

Capt  Charles  G.  R,  Phillott,  R.N. 

Capt.  W.  H.  Pierson,  R.N.,  Havant. 

Capt  H.  Prescott,  C.B.,  R.N.,  FarDham,  Surrey. 

lient  J,  T.  Paulson,  R.N. 

Mr.  J.  B.  Paddon,  H.M.S.  Galatea. 

George  Peel,  esq.,  George  Yard,  Lombard  Street 

Mr.  Joseph  Pym,  Bartholomew  Close. 

Capt  J.  C.  Ross,  R.N. 
Capt  Edwin  Rkhards,  R.N. 
Lieut  Harry  B.  feichaitls,  R.N. 
Lieut  Curtis  Reid,  R.N.,  Southampton. 
Lieut  Beoj.  Roberts,  H.M.S.  Wolf. 
Mr.  PercETal  Roberts,  H.M.S.  Wolf. 
Lieut  Edward  Rogier,  R.N. 
Mr.  Rolhtnd,  H.M.S.  Galatea. 

The  Right  Hon.  Earl  Spencer,  K.G.,  &c.  &c. 

Admiral  the  Hon.  Sir  R.  Stopford,  K.C.6.,  Commander  in  Chief,  Portsmouth. 

Thomas  Asherton  Smith,  esq.,  M.P.^  Penton  Lodge,  Andorer. 

Capt.  W.  Sanders,  R.N.,  Kingston^  Portsea. 

Capt  Thomas  Sanders,  H.M.S.  Maidstone. 

Capt.  G.  R.  Sartorius,  H.M.S.  Pyramus. 

Capt  G.  F.  Seymour,  C.B.,  R.N.,  Hampton  Court 

Capt.  Charles  Shaw,  R.N. 

Capt  Henry  Shifiher,  R.N.,  Sompting  Abbotts,  Shoreham. 

Capt  Houston  Stewart,  R.N. 

Capt  Charles  Stnmgways,  R.N. 

Capt  C.  B.  Strong,  R.N.,  King's  Terrace,  PorUmouth. 

Capt  H.  E.  P.  Sturt,  R.N. 

Lieut  Archibald  Sinclair,  R.N. 

Lieut.  M.  A.  Slater,  R.N. 

Lieut  Thomas  Spark,  H.M.  Revenue  Cutter  Fancy. 

Lieut  John  Steane,  R.N.,  Ryde. 

Lieut  W.  B.  Stocker,  R.N.,  Poole. 

Lieut  George  F.  Stow,  H.M.S.  Espoir. 

The  Rev.  T.  Surridge,  H.M.S.  Ocean. 

The  Rev.  J.  E.  Surridge,  M.A.,  R.N. 

Mr.  George  Starr,  R.N. 

Mr.  George  Saulez,  Alton,  Hants. 

Mr.  W.  D.  Snooke,  Professor  of  Mathematics,  Ryde,  Isle  of  Wight. 

Mr.  W.  Selby,  Portsmouth. 

c2 


Digitized  by 


Google 


XXiv  LIST  OF   SUBSCRIBERS.' 

Capt.  N.  Thompson,  H.M.S.  Rerenge. 

Capt.  John  Tancock,  R.N. 

Capt.  John  Jervis  Tucker,  R.N.,  Trematon  Castle,  Plymouth. 

Lieut.  John  Thompson  (5),  R.N.,  North  Potherton. 

Mr.  Thomas  P.  Thompson,  H.M.S.  Pyramus. 

llie  Rey.  John  Taylor,  H.M.S.  Ramiliies. 

Mr.  S.  Tuck,  R.N.,  Kingston  Cross,  Portsea. 

Mr.  Joseph  Tizard,  jun.,  Weymouth,  2  copies. 

The  Hon.  G.  Vernon,  Ryde,  Isle  of  Wight. 
Capt.  A.  E.  T.  Vidal,  R.N. 

Commodore  J.  C.  White,  R.N. 

Capt.  James  Wemyss,  R.N.  and  M.P.,  Wemyss. 

Thomas  P.  Williams,  esq.,  M.P.,  Berkeley  Square. 

lieut.  H.  Walker,  R.N.,  Cosham,  Hants. 

Lieut.  William  Wilson,  H.M.S.  Challenger. 

Lieut.  Joseph  C.  Woolnough,  Com.  H.M.  Cutter  Surly. 

Lieut.  J.  L.  Wynn,  R.N. 

Edward  D.  Warrington,  esq.,  Charles  Square,  Hoxton. 

Thomas  S.  Whitney,  esq.,  Newpass,  Rathone,  Ireland. 

Mr.  Thomas  Woore,  H.M.S.  Alligator. 

The  Right  Hon.  Lord  Viscount  Yarhorough,  2  copies. 
Captain  Thomas  Young,  R.N.,  Fareham,  Hants. 


Digitized  by 


Google 


CONTENTS. 


X 


DESCRIPTION  OF  THE,  TABLES. 

Table.  Page. 

I.  To  conyert  longitude,  or  d^;rees  into  time,  and  conversely 1 

II.  Depreflsion  of  the  horizon -. ., 3 

III.  Dip  of  the  horizon  at  different  distances  from  the  obsenrer 6 

IV.  Augmentation  of  the  moon's  semi-diameter 8 

V.      Contraction  of  the  semi-diameters  of  the  sun  and  moon   •  •  11 

VI.  Parallax  of  the  planets  in  altitude 12 

VII.  Parallax  of  the  sun  in  altitude 13 

VIII.  Mean  astronomical  refraction. • w  13 

IX.  Correction  of  the  mean  astronomical  refraction 15 

X.  To  find  the  latitude  by  the  north  polar  star 17 

XI.      Correction  of  the  latitude  deduced  from  the  preceding  table.  • .  •  •  •  20 

XII.  JVIean  right  ascension  of  the  sun 21 

XIII.  Equations  to  equal  altitudes  of  the  sun,  part  First 22 

Xiy.     Equations  to  equal  altitudes  of  the  sun,  part  Second 22 

XV.  To  reduce  the  sun's  longitude,  right  ascension,  and  declination ; 

and,  also  the  equation  of  time,  as  given  in  the  Nautical  Almanac, 

to  any  given  time  under  a  known  meridian 25 

XVI.  To  reduce  the  moon's  longitude,  latitude,  right  ascension,  declin- 

ation, semi-diameter,  and  horizontal  parallax,  as  given  in  the 

Nautical  Almanac,  to  any  given  time  under  a  known  meridian  30 

XVII.  Equation  of  the  second  difference  of  the  moon's  place 33 

XVIIL  Correction  of  the  moon*8  apparent  altitude 38 

XIX.  To  reduce  the  true  altitudes  of  the  sun,  moon,  stars,  and  planets, 

to  their  apparent  altitudes 40 

XX.  Auxiliary  angles • 42 

XXI.  Correction  of  the  auxiliary  angle  when  the  moon's  distance  from  a 

planet  is  observed 45 

XXII.  Error  arising  from  a  deviation  of  one  minute  in  the  parallelism  of 

the  surfaces  of  the  central  mirror  of  the  circular  instrument  of 

reflection 46 

XXIII.  Error  arising  from  an  inclination  of  the  line  of  collimation  to  the 

plane  of  the  sextant,  or  to  that  of  the  circular  instrument  of  re- 
flection  • 47 

XXIV.  I^rithmicdiference .,-,-j,.,<3oOgl^ 


XXvi  CONTENTS. 

Table.  Page. 

XXV.  Correction  of  the  logarithmic  difference  for  the  sun's,  or  star's  appa- 

rent altitude   61 

XXVI.  Coirection  of  the  logarithmic  difference  for  a  planet's  apparent 

altitude 52 

XXVIL  Natural  Tersed  sines,  and  natural  sines    53 

XXVIII.  Logarithms  of  nifmb^rs ^ , 62 

XXIX.  Proportional  logarithms 75 

XXX.  .  Logarithmic  half  elapsed  time   • 84 

XXXI.  Logarithmic  middle  time 86 

XXXII.  Logarithmic  rising 87 

XXXIII.  To  reduce  points.  Qf  the  compass  to  degrees,  and  conversely 89 

XXXIV.  Logarithmic  sines,  tangents,  and  secants  to  every  point  and  quar- 

ter point  of  the  compass    89 

XXXV.  Logarithmic  secants  to  every  second  in  the  semi-circle    ••......     90 

XXXVI.   Logarithmic  sines  to  every  second  in  the  semicircle 93 

XXXVII.  Logarithmic  tangents  to  every  second  in  the  semicircle 97 

XXXVIII.  To  reduce  the  time  of  the  moon's  passage  over  the  meridian  of 

Greenwich  ta  the  time  of  her  passage  over  any  other  meridian  100 
XXXIX.  Correction  to  be  applied  to  the  time  of  the  moon's  reduced  transit 

in  finding  the  time  of  high  water  at  any  given  place 102 

XL.       Reduction  of  the  moon's  horizontal  parallax  on  account  of  tfie 

spheroidal  figure  of  the  earth 104 

XLL      Reduction  of  terrestrial  latitude  on  account  of  the  sphenoidal 

figure  of  the  earth 105 

XLII.     A  genera]  traverse  table,  or  difference  of  latitude  and  departure  106 

XLIIL    Meridional  parts , 113 

XLIV.    The  mean  right  ascensions,  and  declinations  of  the  principal  fixed 

stars 114 

XLV.      Acceleration  of  the  fixed  stars,  or  to  reduce  sidereal  time  into 

mean  solar  time 117 

XLVI.    To  reduce  mean  solar  time  into  sidereal  time 119 

XLVII.  Time  from  noon  when  the  sun's  centre  is  in  the  prime  vertical ; 
being  the  instant  at  which  the  altitude  of  that  object  should  be 
observed,  in  order  to  ascertain  the  apparent  time  with  the  great- 
est accuracy 119 

'  XLVIII.  Altitude  of  a  celestial  object  (when  its  centre  is  in  the  prime  ver- 
tical), most  proper  for  determining  the  apparent  time  with  the 

greatest  accuracy 120 

XLIX.     Amplitudes  of  a  celestial  object  reckoned  from  the  true  east  or 

west  point  of  the  horizon 122 

L.        To  find  the  times  of  the  rising  and  setting  of  a  celestial  object. . . .    123 
LI.        For  computing  the  meridional  altitude  of  a  celestial  object,  the 

latitude  and  the  declination  being  of  the  same  name  138 

LII.      For  computing  the  meridional  altitude  of  a  celestial  object,  the 

latitude  and  the  declination  being  of  contrary  names 138 

LIU.      The  miles  and  parts  of  a  mile  in  a  degree  of  longitude  at  every 

degree  of  laatttde.,.; •. 144 

Digitized  by  VjOOQ IC 


CO|iTKNT$.  XXVU 

Takle.  Page. 

LIV.      IVfifXtflioiial  miles  lor  congtnictiag  MeroaWr*a  chltfW 145 

LV.       Tq  find  the  disUace  of  terre^triftl  objocto  at  tea 147 

LYI.      To  leducse  the  Freach  oentMiioal  d^yiiioa  of  the  cirde  into  the 
Eng^ah  aexafesiiaal  diwioB;  or,  to  reduce  Fiench  degrees 

into  EngUah  degrees,  and  odhversely    • . .  •  ^ 150 

hVlU     A  g«^eral  table  for  gaagiogt  or  finding  the  content  of  all  circular 

headed  casks 1 52 

LVIII.  Latitudes  and  longitttdes  of  the  prinoipal  aea-poits,  islands,  capes, 
shoals,  &c  in  the  luiawn  world ;  with  tbe  time  of  high  water, 
at  the  full  and  ch^Age  of  the  moon,  at  all  plaees  where  it  is 

known ••.... , 154 

Alphabeticai  refeienoe  to.  the  pieoeding  table 155 

Form  of  a  treni^t  table •.. 155 

Miscellaneous  niunbeca  with  their  corresponding  logarithms   ....   155 
A  table  showing  the  tn^e  time  and  degree  at  which  the  hour  and 
minute  hands  of  a  well-regulated  watch,  or  clock,  should 
exactly  meet,  or  be  in  coi\|unction,  &c.  in  every  revolution. ...    155 
A  concise  system  of  decimal  arithmetic   • 156 

Solution  of   Problems  in    Plane,    and   Spherical  Trigono- 

METRT     168 

Plane  trigonometry,  solution  of  right  aiigled  triangles 171 

solution  of  oblique  angled  triangles.  • ' 177 

Sphencal  trigonometry,  solution  of  right  angled  triangles . .    1 81  1 82 

solution  of  quadrantal  triangles   193 

solution  of  oblique  angled  triangles  . . .  •  197 

Nayisation.., ••  ..••.. 211 

Solution  of  problems  relative  to  the  difference  of  latiUide  and  dif- 
ference of  longitude    • 214 

Solution  of  problems  in  parallel  sailing 217 

middle  latitude  sailing ,  221 

Mercator's  sailing 236 

oblique  sailing ^ 255 

windward  sailing 262 

current  sailing 266 

Solution  of  pioUems  relative  to  the  errors  of  the  log  line  and  the 

half  minute  glass    272 

Solution  of  a  very  useful  problem  in  great  circle  sailing 276 

To  find  the  time  of  high  water  at  any  known  place  •  •  •• 103 

.    To  make  out  a  day's  work  at  sea  by  inspection ••••  249 

SoLUTiOK  or  Problsms  jk  Nautical  Astronomy    »... 296 

L         To  convert  longitttde,  or  parts  of  the  equator  into  time 296 

IL        To  conTjert  time  into  longitude  or  parts  of  the  equator  • 296 

HI.        Given  the  time  under  any  known  meridian,  to  find  the  corres* 

pondiof  time  it  Gifsnwicb  •,•••.•««•.«*•  ••«««.f«ft,t  397 

Digitized  by  VjOOQ IC 


XXviii  '  CON TBNTS  • 

Problem.  Page. 

IV.        GiveD  the  time  at  Greenwich,  to  find  the  corresponding  time 

under  a  known  meridian. • 297 

V*  To  reduce  the  sun's  longitude,  right  ascension,  declination,  and, 
also,  the  equation  of  time  as  given  in  the  Nautical  Almanac, 
to  any  other  meridian,  *and  to  any  given  time  under  that 

meridian  ..••••• •  •  • • * 2^98 

VI.  To  reduce  the  mpon*s  longitude,  latitude,  right  ascension,  declin- 
ation, semi-diameter,  and  horizontal  parallax,  as  given  in  the 
Nautical  Almanac,  to  any  other  meridian,  and  to  any  given 
time  under  that  meridian   ••••• 302 

VII.  To  reduce  the  right  ascension  and  declination  of  a  planet,  as 

given  in  the.  Nautical  Almanac,  to  any  given  time  under  a 
known  meridian • ; 307 

VIII.  To  compute  the  apparent  time  of  the  moon's  transit  over  the  me- 

ridian of  Greenwich   '.  •••••• 309 

IX,  Given  the  apparent  time  of  the  moon's  transit  over  the  meridian 

of  Greenwich,  to  find  the  apparent  time  of  her  transit  over  any 
other  meridian  •  •  •  • • • 312 

X.  To  compute  the  apparent  time  of  a  planet's  transit  over  the  meri*    - 

dian  of  Greenwich •• •• .•••  313 

XI.  Given  the  apparent  time  of  a  planet's  transit  over  the  meridian  of 
Greenwich,  to  find  the  apparent  time  of  its  transit  over  any 

other  meridian •••.»«••.. • 315 

^^  XII.     -  To  find  the  apparent  time  of  a  star's  transit  over  the  meridian  of 

any  known  place •••.••••'-....••••..   317 

XIII.  To  find  what  stars  will  be  on,  ov  nearest  to  the  meridian  at  any 

given  time  ..•••• ••••••^•••••«« 319 

XIV.  Given  the  observed  altitude  of  the  lower  or  upper  limb  of  the 

sun,  to  find  the  true  altitude  of  its  centre , 320 

XV.  Given  the  observed  altitude  of  the  lower  or  upper  limb  of  the 

moon,  to  find  the  true  altitude*of  her  centre 323 

XVI.  Given  the  observed  central  altitixde  of  a  planet,  to  find  its  true 

altitude 325 

XVII.  Given  the  obsen'ed  altitude  of  a  fixed  star,  to  find  its  true 

altitude 327 

SoLUTiOK  Of  Problems  relative  to  the  Latitude.^  •••• 328 

I.  Given  the  sun's  meridian  altitude,  to  find  the  latitude  of  the 

place  of  observation ...• 328 

II.  Given  the  moon's  meridian  altitude,  to  find  the  latitude  of  the 

place  of  observation . .  •  •  • .••••••••..   330 

III.  Given  the  meridian  altitude  of  a  planet,  to  find  th*  latitude  of 

the  place  of  observation • t 333 

IV.  Given  the  meridian  altitude  of  a  fixed  stav,  to  find  the  latitude  of 

the  place  of  observation  • •  •  • 335 

V.  Given  the  meridian  altitude  of  a  celestial  object  observed  below 

the  pole,  to  find  the  latitude  of  the  place  of  observation. .  • .  • .  336 


Digitized  by 


Google 


CONTENTS.  XXix 

Problem.  •  Page. 

VI.  '  '  Given  the  altitttde  of  the  north  polar  star^  taken  at  any  hour  of 

the  night ;  to  find  the  latitude  of  the  plape  of  dbeerration  ....   337 

VII.  Oiren  the  latitude  by  account,  the  sun's  declination,  and  two 

observed  altitudes  of  its  lower  or  upper  limb ;  the  elapsed  dme^ 
and  the  course  and  distance  run  between  the  observations ;  to 
find  the  latitude  of  the  ship  at  the  time  of  observation  of  the 
greatest  altitude  ..» « 341 

VIII.  Given  the  altitudes  of  two  known  fixed  stars  observed  at  the  same 

instant,  at  any  time  of  the  night ;  to  find  the  latitude  of  the 
place  of  observation,  independent  of  the  latitude  by  account^ 
the  longitude,  or  the  apparent  time  of  observation 347 

IX.  -      Given  the  latitude  by  account,  the  altitude  of  the  sun's  lower  or 

upper  limb,  observed  within  certain  limits  of  noon,  the 
apparent  time  of  observation,  and  the  longitude ;  to  find  the 
true  latitude  of  the  place  of  observation • 354 

X.  Given  the  latitude  by  account,  the  altitude  of  the  moon's  lower  or 

upper  limb,  observed  within  certain  limits  of  the  meridian,  the 
apparent  time  of  observation,  and  the  longitude' ;  to  find  the 
latitude  of  the  place  of  observation « ••••..    358 

XI.  Given  the  latitude  by  account,  the  altitude  of  a  planet's  centre 

observed  within  certain  hmits  of  the  meridian,  the  apparent 
time  of  observation,  and  the  longitude ;  to  find  the  true  latitude 
of  the  place  of  observation 362 

XII.  Given  the  latitude  by  account,  the  altitude  of  a  fixed  star  observed 
within  certain  limits  of  the  meridian,  the  apparent  time  of 
observation,  and  the  longitude ;  to  find  the  true  latitude  of  the 

place  of  observation •••••••.••••«..    365 

To  find  the  latitude  by  an  altitude  taken  near  the  meridian  below 

thepole 368  369 

Captain  Wilfiam  Fitzwilliam  Owen*^B  general  Problem  for  finding 
the  latitude  r 371 

Xin.      Given  the  interval  of  time  between  the  rising  or  setting  of  the 

sun's  upper  and  lower  limbs ;  to  find  the  latitude • . .   373 

SOLUTIOV  or   PROBI.EHS   RELATIVE  TO   THE  APPARENT  TiME 375 

I.  To  find  the  error  of  a  watch  by  equal  altitudes  of  the  sun 377 

II.  To  find  the  error  of  a  watch  by  equal  altitudes  of  a  fixed  star  ..   380 
^'UI.       Given  the  latitude  of  a  place,  and  the  altitude  and  declination  of 

the  sun ;  to  find  the  apparent  time  of  observation,  and,  thence, 

the  error  of  the  watch.     Method  I ••... 383 

Method  II.     Of  computing  the  horary  distance  of  a  celestial 

object  from  the  meridian  •••••• 388 

Method  III.    Of  computing  the  horary  distance  of  a  celestial 

object  from  the  meridian. • 390 

Method  IV.  Of  computing  the  horary  distance  of  a  celestial 
.    object  from  the  »«ddiaA.t,« 392 

Digitized  by  VjOOQ IC 


inp^  COMTBNTS. 

Problem.  •  J'oQI^- 

)V.       Gifen  the  Mtude  ami  lovyitiide  of  ^  pkce^  tl^  %Ktit«4e»  n^l 
iu8ceiiai(ui,  and  dec^ifAtiaa  of  a  known  fixed  star,  and  the  sun's 
right  ascension ;  to  find  the  apparent  time  •.•.•..«....••...   394 
V.        Qiven  the  latitude  and  longitude  of  a  place,  and  the  a].titude  of  a 

planet ;  to  find  the  apparent  time  of  obserT^tioa. . .  ^ 397 

Vf ,  Given  th^  latitude  and  longitude  of  a  place,  the  estimated  time 
at  that  place,  and  the  altitude  of  the  moon*s  limb ;  to  find  the 
apparent  time  of  observation  •••«••..••  ^«  i^« .,  .^  •  .^  ..«•• .  400 

SoLDTIOir   pF   PaOI^LVMS   E^LATIVE   TO   riKDIVG   THE  AX'TJ^TUDES  OF 

THE  Heavenly  Bqdies.v ••«. .,., ,  ^••,«, , ^..^ 403 

L  Given  the  latitude  and  longitude  of  a  place,  and  the  apparent 
dpie  at  that  place ;  to  find  the  true  a^d  thet  apparent  altitude 
of  the  sun's  centre • • *  404 

II.  Given  the  latitude  and  longitude  of  a  place,  and  the  apparent 

time  at  that  place ;  to  find  the  true  and  the  apparent  altitude 

of  a  fixed  star •«•••... , 406 

III.  Given  the  latitude  and  longitude  of  a  place,  and  the  apparent 

time  at  that  place ;  to  find  the  true  and  the  apparent  altitude 

of  a  planet 408 

ly.  Given  the  latitude  and  longitude  of  a  place,  apd  the  apparent  time 
at  that  place  ;•  to  find  the  true  and  the  apparent  altitude  of  the 
moon's  centre  • « • • 410 

Solution  of  PaoBLEMs  relative  to  the  Lqvg^tude  ••• 413 

I.  To  convert  apparent  Ume  into  mean  time • •  415 

II.  To  convert  mean  time  at  Greenwich  into  apparent  time 416 

III.  Given  the  ktitude  of  a  place,  and  the  observed  altitude  of  the 

sun*s  limb ;  to  find  the  longitude  of  that  place  by  a  chrono- 
meter or  time-keeper 417 

IV.  Given  the  latitude  of  a  place,  and  the.  observed  altitude  of  a 

known  fixed  star ;  to  find  the  longitude  of  that  place  bj  a 
chronometer  or  time-keeper , 420 

V.  Given  the  latitude  of  a  place,  and  the  i>b8erved  altitude  of  a 

planet ;  to  find  the  longitude  of  the  place  of  observation  by  a 
chronometer  or  time-keeper 423 

VI.  Given  the  latitude  of  a  place,  and  the  observed  altttode  of  the 

moon's  limb ;  to  find  the  longitude  of  the  place  of  observation 

by  a  chronometer  or  time-keeper. 426 

VII.  To  find  the  longitude  of  a  ship  or  place  by  celestial  observation, 

commonly  called  a  LuKAR  Observation  ..•••  • 431 

Method    I.     Of  reducing  the  apparent  to  the  true  central 

distance  433 

Method    11.    Of  reducing  the  apparent  to  the  true  central 

distance 436 

Method  IIL  Of  reducing  the  apparent  to  the  true  central 

fliitaooe  ^••«,,.«o*ftff • ft 439 

Digitized  by  VjOOQ IC 


PfMem.  Pmg§. 

Method  IV.  Of  reduciag  the  appiprent  to  the  tnie  central 

difitancse.  •• 439 

Method    V.   Of  reducing  the  apparent  to  the  true  central 

distance  •  ^ 441 

Method .  VI.  Of  reducing  the  apparent  to  the  tr4ie  central 

distance : ; 442 

Method  VII.  Of  reducing  the  apparent  ta  the  true  central 

distance   443 

Method  VIII.  Of  reducing  the  apparent  to  the  true  central 

.distance • 445 

Method  IX.   Of  reducing  the  apparent  to  the  true  central 

distance  447 

Method  X.    Of  reducing  the  apparent  to  the  true  central 

distance .• 448 

Method  XI.   Of  reducing  the  apparent  to  the  trae  central 

distance   • 450 

Method  XII.  Of  reducing  the  apparent  to  the  true  central 

distance • 451 

Method  XIII.  Of.  reduciBg  the  apparent  to  the  true  central 

distance • • 453 

Vill.      Given  the  apparent  time,  and  the  true  central  distance  between 
the  moon  and  sun,  a  fixed  star,  or  planet ;  to  determine  the 

longitude  of  the  place  of  observation  • 454 

IX.        Given  the  latitude  of  a  place,  its  longitude  by  account,  the 
'    observed  distance  between  the  moon  and  sun,  a  fixed  star,  or 
a  planet,  and  the  observed  altitudes  of  these  objects ;  to  find 
the  true  longitude  of  the  place  of  observation  ••••«.•,••••••  456 

X.  Given  the  observed  dbtance  between  the  moon  and  sun,  a  fixed 
star,  or  planet,  the  apparent  time,  with  the  latitude  and  longi- 
tude by  account ;  to  find  the  true  longitude  of  the  place  of 

observation 470 

Xi.       To  find  the  longitude  of  a  place  by  the  eclipses  of  Jupiter's 

.satellites 478 

XII.      To  find  the  longitude  of  a  place  by  the  eclipses  of  the  moon  ....   481 

SOLITTIOV    OF     PrOBLBI^S     RELATIVE     TO     TH*   VaRIATIOK     OP    THE 

Compass • 483 

I.  (Hven  the  latitude  of  a  place,  and  the  sun's  magnetic  amplitude ; 

to  find  the  variation  of  the  compass • 484 

II.  Given  the  latitude  of  a  place,  the  sun's  altitude,  and  his  magnetic 

azimuth ;  to  find  the  variation  of  the  compass 487 

A  new  method  of  computing  the  truo  azimuth  of  a  celestial 
object \ 490 

III.  To  find  the  variation  of  the  compass  by  observations  of  a  circum- 

polar  star 492 

IV,  To^find  ibe  varit^tioQ  of  the  compass  hj  the  maj;netic  bearing  of 


Digitized  by 


Google 


XXxii  CONTENTS. 

Problem.  Page. 

a  fixed  star  or  planet,  taken  at  the  time  of  its  transit  over  the 

meridian  of  any  known  place 494 

V.        Giyen  the  true  course  between  two  places^  and  the  variation  of 

the  compass ;  U>  find  the  magnetic  or  compav  coarse   ••.•••  496 
VI.  '     Given  the  magnetic  course^  or  -that  steered  by  compa»»  and  the 

variationrof  the  compass;  to find*the  true  course.* •'•••••••••  497 

Description  of  an  improved  azimuth  compass  card •  •  •  •  497 


\' 


Solution  of  PaoBLEMs   relative  to  the  Risivg   akd  Setting 
OF  THE  Celestial  Bodies    ..•••••. • ••••«.••  500 

I.  Given  the  latitude  of  a  place,  and  the  height  of  the  eye  above 

the  level  of  the  horizon ;  to  find  the  apparent  times  of  the  sun's 
rising  and  setting • •••:.. •• 500 

II.  Given  the  latitude  of  a  place,  and  the  height  of  the  eye  above     , 

the  level  of  the  horizon ;  to  find  the  apparent  times  of  rising 
and  setting  of  a  fixed  star  .•••••• • 504 

III.  Given  the  latitude  of  .a  place,  and  the  height  of  tbe  eye  above  the 

level  of  the  horizon ;  to  find  tbe  apparent  times  of  a  planet's 
rising  and  setting, • ••  506 

IV.  Given  the  latitude  of  a  place,  and  tbe  height  of  the  eye  above 

the  level  of  the  horizon ;  to  find  the  apparent  times  of  the 
moon's  rising  jind  setting •••••• •  •  5l  1 

V.  Given  the  latitude  and  longitude  of  a  place,  and  the  day  of  the 

month ;  to  find  the  times  of  the  beginning  and  end  of  twilight, 
and  the  length  of  its  dilation  .•..•••...••« 516 

VI.  Given  tbe  latitude  of  a  place ;  to  .find  the  time  of  the  shortest 

twilight,  and  the  length  of  its  duration  • ••••••  519 

VII.  Given  the  httitude  of  a  place  between  48° 32'  and  66'>32'  (the 

limits  of  regular  twilight)';  to  find  when  real  night  or  darkness 

ceases,  and  when  it  commences .••••••••••••  {^20 

VII L  Given  the  latitude,  and  the  sun's  declination ;  to  find  the  interval 
of  time  between  the  rising  or  setting  of  the  upper  and  lower 
limbs  of  that  luminary ., , , ,   620 

Solution  of  Problems  in  Gnomonics  oa  Dialling 522 

L  Given  the  latitude  of  a  place ;  to  find  the  angles  which  the  hour- 
lines  make  with  the  substyle,  or  meridian  line  of  a  horizontal 

•sun-dial *, 523 

II.  To  find  the  angles  on  the  plane  of  an  erect  direct  south  dial  for 
any  proposed  north  latitude,  or  on  that  of  an  erect  direct  north 
dial  for  any  proposed  south  latitude 526 

Solution  of    Problems    relative    to    the   Mensuration    of 
Heights  ani^"  Distances 528 

I.  To  find  the  height  of  an  accessible  object 529 

II.  Given  the  angle  of  elevation,  and  the  height  of  an  object ;  to  find 

the  observer's  horizontal  distance  from  that  object   , .  530 


Digitized  by 


Google 


CONTENTS.  XXXIU 

Problem.  Page. 

III.  To  find  the  height  of  an  ioaccestdble  object 531 

IV.  To  find  the  distance  of  an  inacceesible  object,  which  can  neither 

be    receded   from   nor   approached, -in   its  vertical  line  of 
direction  .  •  • • 532 

V.  To  find  the  distai\ce  between  two  inaccessible  objects •. .  •  •  534 

VI.  Given  the  distances  between  three  objects,  and  the  angular  dis- 

tances between  those  objects  taken  at  any  point  in  the  same 
hoii^ntal  plane ;  to  find  the  distance  between  that  point  and 

each  of  the  objects •.... 53^ 

VIL       Given,  the  distances  between  three  objects,  and  the  angular  dis-  ' 

*  tances  between  those  objects  taken  at  any  point  within  the 

'    triangle  formed    by    the  right   lines  connecting  them;   to 

find,  the  distance  between  that  point  and  each  of -the.  objects.  •  539 

VIII.      Given  the  distances  between  three  objects  situated  in  a  straight 

line,  and  the  angular  distances  between  those  objects  taken  at 

any  point  in  the  same  horizoikal  plane ;  to  find  the  distance 

between  that  point  and  each  of  the  objects,  • 541 

IX.        Given  the  height  oi,  the  eye,  to  find  the  distance  of  the  visible 

horizon ••••.-••••••• ••. 544 

X»        Given  the  measured  length  of  a  base  line,  to  find  the  allowance 

for  the  curvature  or  spherical  figiure  of  the  earth 545 

XL        Given  a  base  line  measured  on  any  elevated  tevel,  to  find  its  true 

measure  at  the  suriace  of  the  sea •  547 

XII.  To  find  the  height  and  distance  of  a  hill  or  mountain 549 

XIII.  To  find  the  height  of  a  mountain,  by  means  of  two  barometers 

and  thermometers  •••••.••••••••• 550 

XIV.  To  find  the  distance  of  an  object  by  observing  the  interval  of  ' 

time  between  seeing  the  flash  and  hearing  the  report  of  a  gun 

or  of  a  thunder  cloud  •  •  ••••.. 552 

XV.  Given  three  bearings  of  a  ship  sailing  upon  a  direct  course,  and 
the  intervals  of  time  between  those  bearings ;  to  find  the  course 
steered  by  that  ship,  and  the  time  of  her  nearest  distance  firom 
the' observer. .  • ••••••••• ••••••..••••••  553 


SOLUTIOH   OV  PkOBLEMS   IN    PfiJiCTICAL   GuNNERY 557 

I.  .      Given  the  diameter  of  an  iron  ball,  to  find  its  weight  ..•..•••  557 

II.  Given  the  weight  of  an  iron  ball,  to  find  its  diameter 558 

III.  Given  the  diameter  of  a  leaden  ball,  to  find  its  weight 558 

IV.  Given  the  weight  of  a  leaden  ball,  to  find  its  diameter  •••.....  559 

V.  Given  the  internal  and  external  diameters  of  an  iron  shell,  to 

find  its  weight 560 

VI.  To  find  how  much  powder  will  fill  a  shell •  •  561 

111.       To  find  the  size  of  a  shell  to  contain  a  given  weight  of  powder.  •  562 

VIII.      To  find  how  much  powder  will  fill  a  rectangular  box 562 

IX.       To  find  the  size  of  a  cubical  box  to  contain  a  given  weight  of 

powder «;••«•••  t  •..•  • •  • ,   563 


Digitized  by 


Google 


X3tkiv 

Problem, 
X. 
XL* 

XII* 

xni. 

XIV. 
XV. 

XVI. 
XVII. 

XVIII. 
XIX. 


XX. 


XXI: 

xxn. 

XXIII. 
XXIV. 

XXV.- 

XXVI. 
XXVII. 
XXVIIL 

XXIX. 

XXX. 

XXXI. 


XXXII. 
XXXIIL 


CONTENTS, 

Pistge. 

Te  find 'how  much  powder  will  fill  a  cylinder ,   564 

To  find  What  length  of  a  cylinder  •will  be  filled  hf  a  given 

weight  of  powder , 565 

To'find  the  number  of  balls  or  shells  in  a  triangular  pile  ..•••.   566 

To  find  the  number  of  balls  or  shells  ifl  a  square  pile 567 

To  find  the  number  of  balls  or  shells  in  a  rectangular  pile. . . . ; .   567 
To  find  the  number  of  balls  or  shells  in  an  incomplete  triangular 

pile ; 4 568 

To  find  the  number  x)f  balls  or  shdk  in  an  incomplete  square  pile  569 
To  find  the  number  of  balls  or  bhells  in  an  incomplete  rectkngnlar 

pile  .; ;i i..., 570 

To  find  the  velocity  of  any  shot  or  shell.  .;.••• '. 57 1 

To  fitid  the  terminal  velocity  of  a  shot  or  shell ;  that  is,  the 
greatest  velocity  it  can  acquire  in  descending  through  the  air 

by  its  bwn  weight 572 

To  find  the  height  from  which  a  body  must  fall,  in  vacuo,  in 

order  to  acquire  a  given  velocity. .' 573 

Concise  Tables  for  determining  the  greatest  horizontsd  range  of 
a  shot  or  shell,  when  projected  in  the  air  with  a  given  velo- 
city ;  with  the  elevation  of  the  piece  to  produce  that  range  574 
To  find  the  greatest  range  of  a  shot  or  shell,  and  the  elevation  of 

the  piece  to  produce  that  range .'...•;••.•....••••  575 

Given  the  range  at  one  elevation,  to  find  the  range  at  another 

elevation .:... 576 

Given  the  elevation  for  one  range,  to  find  the  elevation  for  another 

•    range  ..••••••••,••.  4  • • • 577 

Given  the  charge  for  one  rtinge,  to  find  the  ch&rge  for  anothet 

range  •••.k« 578 

Given  the  range  for  one  charge,  to  6nd  the  range  for  another 

chal-ge » ' i . . .  ^ . . .    579 

Given  the  range  and  the  elevation,  to  find  the  impetus  . .  • .  . .'  579 
Given  the  devation  and  the  range,  to  find  the  time  of  fiight  •  •  •  •  580 
Given  the  range  and  the  elevation,  to  find  the  greatest  altitude 

of  the  shell 581 

Given  the  inclination  of  the  plane,  the  elevation  of  the  piece,  and 

the  impetus ;  to  find  the  radge ....<••••• 58^ 

Given  the  inclinatiob  of  the  plane,  the  eletation  of  the  piece,  and 

the  range ;  to  find  the  impetus.  .••...••«. *  • 583 

Given  the  weight  of  a  ball,  the  charge  of  powder  with  which  it 

is  fired,  and  the  known  yelocity  of  that  ball ;  to  find  the  velo* 

city  of  a  shell,  when  projected  with  a  given  ch&rge  of  powder  584 

A  Table,  showing  the  velocities  of  the  different-siccd  shells, 

when  projected  with  a  given  charge  of  powder  •  •  • <  •   585 

Givbn  the  elevation  and  the  rangej  to  find  the  impetus,  velocity, 

and  charge  of  powder  ••,•••••.•.*.•••••..•• 685 

Given  the  inclination  of  the  plane,  the  elevation  of  the  j^ece,  anit 

the  range  I  tofiad  th«  charge  of  powdtri  «»«•  #  •b«i»«  ••••«•  586 


Digitized  by 


Google 


CONTENDS.  xxxr 

FroMem.  Page. 

XXXIV.  Given  tiiB  inclinatioii  of  the  plane,  the  eleration  of  the  piece,  and 

the  impetus ;  to  find  the  time  of  flight  ••••. 688 

XXXV.  Given  the  impetus  and  the  elevation,  to  find  the  horisontal  range  589 

XXXVI.  Given  the  impetus  and  the  elevation,  to  find  the  time  of  flight  on 

the  horizontal  range 590 

XXXVIL  Given  the  time  of  flight  of  a  shelly  to  find  the  length  of  the  fuse  591 

• 
Solution  of  Paobliems  ik  the  Mensuratiow  of  Planeb,  &c  ..  592 

I.  Given  the  hase,  and  the  perpendicular  height  of  a  plane  triangle ; 

to  find  its  area    • •••••••  i  •••.'•••.  •  592 

IL        Given  two  sides,  and  the  conti^oed  angle  of  a  plane  triangle ;  to 

find  its  area * 592 

III.  Given  the  three  sides  of  a  plane  triangle ;  tor  find  its  area  •  •  •  •  593 

IV.  Given  the  diameter  of.a  circle;  to  find  its  circumference,  and 

conversely   • ••..•...•••.••  594 

V.  Given  the  diameter,  or  the  circumference  of  the  earth ;  to  find 

the  whole  area  of  its  surface  • •• •»..••  594 

VL        To  find  the  length  of  any  arc  of  a4^irele    •  ^ •..••••••  595 

Solution  OF  Paoblems  ik  Gauging.. • 596 

I.        To  reduce  the   old  standard  wine  measure  into  the  Imperial 

standard  measure.  »«••• «•••••*••••••»«•••• 597 

I},        To  reduce  t)ie  Imperial  standard  measure  into  the  old  stand* 

ard  wine  measure    •••^•••••. ••••••••••••«*•••••  597 

III.        To  reduce   the  old  standard  ale  measure    into  the  Imperial 

standard  measure     ••••••»• • .«...    598 

rV.       To  reduce  the  Imperial  standard  measure  into  the  old  standard 

ale  measure • ••••••••••••• 598 

V.  Given  the  dimensions  of  a  circular  headed  cask;  to  find  its  con- 

tents in  ale  and  in  wine,  gallons,  and,  also,  id  gallons  agreeably 

to  the  Imperial  standard  measure  *«••••• • 599 

VI.  To  find  the  ullage  of  a  cask  lying  in  a  horisontal  position 60 1 

VII.  To  find  the  ullage  of  a  cask  standing  in  a  vertical  position  • . .  •  604 
A  general  Table  for  converting  ale  or  wine  measure  into  the  im- 
perial standard  measure^  and  conversely  •  • « • • 606 

Solution  of  Miscellaneous  Problems   • 607 

h        To  find  the  weight  of  a  cable » 607 

II.  To  find  the  circumference  of  a  circle »•!.••••• 608 

III.  To  find  the  area,  or  superficial  content  of  a  circle •  •  •  •  • «  609 

IV.  Given  the  area  of  a  circle,  to  find  its  diameter   « «•.•••«  609 

V.  To  find  the  side  of  a  square  equal  in  area  to  a  given  circle 610 

VI.         To  find  the  side  o(  a  square  inscribed  in  a  given  circle 610 

VIL       Tofindtheareaof  andlipsis  ..•• • 611 

VIII.  To  find  the  diameter  of  a  circle  equal  in  area  to  a  given  ellipsis.  •  61 1 

IX.        To  find  the  circumference  of  an  ellipsis.  •••••  • 612 


Digitized  by 


Google 


XXXvi  CONtENTS, 

Problem.  Page. 

X.        To  find  the  solid  content  of  a  sphere  or  globe 612 

XL        To  find  the  height  from  which  a  person  could  see  the  one  third  of 

the  earth's  surface •  • .  •   Q13 

XII.       To  find  the  distance  of  the  sun  from  the  earth  . .  .^ •  •  614 

'    XIII.       To  find  the  measure  of  the  8un*8  diameter  in  English  miles  .....   614 

XIV.       To  find  the  ratio  of  the  magnitudes  of  the  earth  and  sun 615 

XV.  To  find  the  rate  at  which  the  inhabitants  under  the  equator  are 
carried  in  consequence  of  the  earth's  diurnal  motion  round  its 
axis  ••••. « • 615 

XVI.  To  find  the  rate  at  which  the  inhabitants  under  any  given  parallel 

of  latitude  are  carried,  in  consequence  of  the  earth's  diurnal 
motion  round  its  axis  .« 616 

XVII.  To  find  the  length  of  the  tropical  or  solar  year 616 

XVIII.  To  find  the  rate  at  which  the  earth  moves  in  the  ecliptic •  617 

XIX.     To  find  the  measure  of  the  moon's  diameter  in  English  miles    . .   617 

XX.      To  find  the  ratio  of  the  magnitudes  of  the  earth  and  moon 618 

XXI.  To  find  how  much  lai^er  the  earth  appears  to  the  lunar  inha- 

bitants than  the  moon  fippears  to  the  terrestrial  inhabitants   •  •  618 

XXII.  To  find  the  rate  at  which  the  moon  revolves  round  her  orbit*  •  •  •  619 

XXIII.  To  find  the  true  distance  of  a  planet  from  the  sun  ••••• ••  619 

XXIV.  To  find  the  comparative  heat  and  light  which  the  difiierent  planets 

receive  from  the  sun ., ••  620 

XXV. ' '  To  find  the  measure  of  a  planet's  diameter  in  English  miles  •  •  •  •  621 

XXVI.  To  find  the  time  that  the  sun  takes  to  turn  round  its  axis   ••••••  622 

XXVII.  To  find  the  length  of  a  penduhrai  for  vibrating  seconds 623 

XXVIIL  To  find  the  length  of  a  pendulum  for  vibrating  half  seconds  •  •  •  •  623 

A  compendium  of  Practical  Navigation,  &c.  &c.  &c 624 

To  make  out  a  day's  work  at  sea  by.  calculation 633 

Of  the.Iog  book  . . . , ' * ; 639 

.  Of  the  measure  of  a  knot  on  the  log  line ;  and  of  the  true  figure 

of  the  earth    •• 649 

The  true  method  of  finding  the  index  error  of  a  sextant,  &c.  so  as 

to  guard  against  the  error  arising  from  the  elasticity  or  spring 

of  the  bar,  &c , 653 

Of  taking  altitudes  by  means  of  an  artificial  horizon 655 

A  new  and  correct  method  of  finding  the  longitude  of  places  on 

shore , 661 

Solution  OF  Useful  Astronomical  Problems   , 672 

I.        To  find  the  latitude  and  longitude  of  a  celestial  object  ........   672 

II.  To  find  the  right  ascension  and  declination  of  a  celestial  object.  •  677 

III.  To  compute  the  lunar  distances,  as  given  in  the  Nautical  Almanac  68 1 
Appendix,  showing  the  direct  application  of  logarithms  to  the 

doctrine  of  compound  interest •  •  •  • 687 

Description  and  use  of  the  general  victualling  table 717 


Digitized  by 


Google 


TO 


HIS  ROYAL  HIGHNESS  WILLIAM  HENRY,  DUKE  OF  CLARENCE 
AND  ST.  ANDRE\<^S,  K.Q.,  &c.  &c.  &c. 

« 

LORD  HIGH  ADMIRAL 


UNITED  KINGDOMS  OF  GREAT  BRITAIN  AND  IRELA"ND, 
MAY  IT  PLEASE  YOUR  ROYAL  HIGHNESS; 

This  Treatise,  which  I  am  graciously  permitted  to  lay- 
before  your  Royal  Highness,  is  the  result  of  long  study 
apd  labour ;  the  chief  aim  of  which,  .has  been,  to  can- 
tribute,  in  some  measure,  to  the  benefit  of  the  Naval  Service 
of  His  Majesty.  To  this  end,  I  have  sought  to  combine 
simplicity,  perspicuity,  and  conciseness,  in  trigonometrical 
calculations,  in  a  greater  degree  than  has  hitherto  been 
attempted  by  the  writers  of  nautical  works;  and  to  comprise, 
in  one  book,  a  compendium  of  all  the  sciences  that  may 
be  useful  or  interesting  to  the  practical  navigator. 

Tliat  my  humble  attempt  has  met  with  your  Royal  High- 
nesses approbation  and  high  sanction,  I  shall  ever  esteem  to 
be  the  most  honourable  circumstance  of  my  life  ;  that  it  has 
been  deemed  worthy  of  the  honour  of  your  Royal  Highnesses 
patronage,  I  cannot  but  feel  as  the  greatest  mark  of  the 
condescension  of  your  Royal  Highness. 

I  have  the  honour  to  subscribe  myself. 
With  the  most  profound  respect, 
Your  Royal  Highnesses 

Most  obedient,  most  devoted. 

And  most  grateful  Servant, 

THOMAS  KERIGAN. 


PcrUmmUhf 
JDcctmbetg  1827. 


Digitized  by 


Google 


Digitized  by 


Google 


PREFACE. 


Although  the  importance  and  general  utility  of:  the  subjects  treated  of  in 
this  work  are  sufficient  to  recommend  it  to  public  attention,  without  the 
aid  of  prefatory  matter^  yet,  since  there  is  an  extensive  variety  of  nautical 
publications  now  extant,  I  think  it  right  to  say  nomethmg  relative  to  what 
I  have  done,  were  it  for  no  other  purpose  than  that  of  satisfying  the  reader 
that  the  present  work  is  widely  different  from  any  former  treatise  on  nau- 
tical and  mathematical  subjects.  The  jToUowing  observations  will  develope 
my  motives  for  commencing  so  laborious  an  undertaking. 

In  perusing  the  various,  nautical  publications  which  have  appeared  for 
many  years  past,  I  observed  that  they*  all  fell  considerably  short  of  the 
objects  at  which  they  professed  to  sum ;— some,  by  being  too  much  con- 
tracted, and  others  by  not  including  all  the  necessary  tables,  or  by 
being  generally  defective :  and  that,  therefore,  a  great  deal  remained  to 
be  done,  particularly  in  the  tabular  parts^  beyond  what  had  yet  been 
brought  before  the  public. 

Of  the  nautical  works  that  came  under  my  notice,  some  have  proved,  on 
examination,  to  be  so  inaccurately  executed,  as  to  be  entirely  unfit  for 
the  consultation  of  any  person  not  sufficiently  skilled  in  the  mathematics 
to  detect  their  numerous  errors..  Many  of  the  works  in  question  are  ex- 
tremely incomplete,  through  their  want  of  particular  tables,  and  their  logar- 
ithms not  being  extended  to  a  sufficient  number  of  decimal  places:  such 
as  those  by  Mendoza  Rios,  where  the  decimals  are  only  continued  to  Jive 
places  of  figures,  and  where  the  logarithmic  tangents  are  entirely  wanting; 
for,  although  the  addition  of  a  logarithmic  sine  and  a  logarithmic  secant 
will  always  produce  a  logarithmic  tangent,  yet  there  are  few  mariners  so 
far  acquainted  with  the  peculiar  properties  of  the  trigonometrical  canon^ 
as  to  be  aUe  to  find  by  Rios'  tables  the  arch  corresponding  to  a  given 

b2 


Digitized  by 


Google 


viii  PREFACE. 

logarithmic  tangent.*  Hence,  when  the  course  and  the  distance  between 
two  places  are  to  be  deduced  from  their  respectiye  •  latitudes  and  longi- 
tudes, by  logarithmical  computation,  the  mariner  is  invariably  obliged 
to  have  recourse  to  some  other  work  for  the  necessary  table  of  logarithmic 
tangents.  Besides,  since  none  of  the  nautical  works  now  in  use  exhibit 
the  principles  upon  Which  the  tables  contained  therein  have  been  con- 
structed, the  mariner  is  left  without  the  means  of  examining  such  tables, 
or  of  satisfying  himself  as  to  their  accuracy;  though  it  is  to  them  that  he 
is  obliged  to  make  continual  reference,  and  on  their  correctness  that  the 
safety  of  the  ship  and  stores,  with  the.liv^s  of  41  o^.  board,  so  materially 
depend.        .    .  *    • 

Notwithstanding  that  Mr.  Taylor's  Logarithmical  Tables  are  the  most 
extensive,  the  best  arranged,  and  by  far  the  most  useful  for  astronomical 
purposes,  of  any  that  have  ever  appeared  in  print^ — yet,  since  they,  do  not 
contain  the  necessary  navigation  tables,  they  are  but  of  little  use,  if  of 
any,  to  the  practical  qavigator  :  and,  since  the  same  objection  is  applicable 
to  the  very  excellent  system  of  tables  published  by  the  learned  Dr.  Hutton^ 
these  are,  also,  ill  adapted  to  nautical  purposes,  and  but  rarely  consulted 
by  mariners. 

Being  thus  convinced  that  thei'e  was  something  either  deficient  or 
very  defective  in  all  the  works  that  had  hitherto  been  published  on  this 
subject^  I  was  ultimately  led  to  the  conclusion  that  a  general  and  com- 
plete ^t  qf  Nautical  Tables  was  still  a  desideratum  to  mariners :  with 
this  conviction  on  my  mind,  I  was  at  length  induced  to  undertake  the 
laborious  task  of  drawing  up  the  following  work ;  in  the  prosecution  of 
which  I  found  it  necessary  to  exercise  the  most  determined  perseverance 
and  industry,  in  order  to  surmount  the  fatigue  and  anxiety  attendant  on 
such. a  long  series  of  difficult  calculations. 

These  points  premised,  \t  .remains  to  present  to  the  reader  a  familiar 
$s\d  comparative  view  of  the  nature  of*  this  work,  and  of  the  improve- 
ments that  have  been  made  in  the  tables  immediately  connected  with'  the 
elements  of  narigation  and  nautical  astronomy :  confining  the  attention  to 
those  that  possess  the  greatest  claims  to  originality,  or  in  which  the  most 
useful  improvements  have  bee^  made. 

Table  VI.  contains  th.e  parallaa^es  of  the  .  planets  in  altitude ;  and 
will  be  found  particularly  useful  in  deducing  the  apparent  time  from  the 
altitudes  of  the  planets,  and,  also,  in  prbblems  relating  to  the  longitude. 
The  hint  respecting  this  was  originally  taken  from  the  Copenhagen  edition 
of  "The  Distances  of  tlie  Planets  from  the  Moon's  Centre,  for  the  Year 
1823  ^"  but  this  design  has.  been  considerably  enlarged  and  improved 
upon. 

•  See  Remarki  page  98  ^  with  dia^am  apd  calculations^  page  99. 

Digitized  by  VjOOQ IC 


PRBFACB.  IX 

Table  VIH.  is  so  arranged  that  the  mean  astronomical  refraction  may 
be  taken  out  at  first  sight,  without  subjecting  the  mariner  to  the  necessity 
of  making  proportion  for  the  odd  minutes  of  altitude.  This  improvement 
wOI  have  a  tendency  to  facilitate  nautical  calculations. 

Table  X. — The  arrangement  of  this  table  is  an  improvement  of  that 
originally  given  by  the  author^  in  his  treatise  called  ^^  The  Young 
Navigator's  Guide  to  the  Sidereal  and  Planetary  Parts  of  Nautioaf  Astro-* 
nomy/'  By  this  improved  table,  the  correction  of  the  polar  star's  altitude 
may  be  readily  taken  out,  at  sight,  to  the  nearest  second  of  a  degree,  by 
means  of  five  columns  of  proportional  parts;  and^  to  render  the  table 
permanent  for  at  least  half  a  century,  the  aimual  variation  of  that  star's 
correction  has  been  carefiilly  determined  lo  the  hundredth  part  of  a  second. 
By  means  of  this  table,  and  that  which  immediately  folfows  (Table  XL), 
the  latitude  may  be  very  correctly  inferred  at  any  hour  of  the  night,  in  the 
northern  hemisphere,  to  every  degree  of  accuracy  desirable  for  nautical 
purposes. 

Tables  XHI.  and  XIV.  contain  the  equations  to  equal  altitudes  of  the 
sun  :  these  have  been  computed  on  a  new  prinpiple,  so  as  to  adapt  them  to 
proportional  logarithms,  by  means  of  which  they  are  rendered  infinitely 
more  simple  than  those  given  under  the  same  denomination  in  other 
treatises  on  nautical  subjects ;  they  will  be  found  strictly  correct,  and,  from 
their  simplicity,  a  hope  may  be  entertained  that  the  truly  correct  and  ex- 
cellent method  of  finding  the  error  of  a  watch  or  chronometer  by  equal 
altitudes  of  the  sun,  will  be  brought  into  more  general  use« 

Tables  XV.  and  XVI.,  whfch  are  entirely  new,  contain  correct  equations. 
for  readily  reducing  the  longitudes,  right  ascensions,  declinations,  &c.  &c., 
of  the  sun  and  moon,  as  given  in  the  Nafttical  Almanac,  to  any  given 
meridian,  and  to  any  given  time  under  that  meridian, 

TmUe  XVII.  contains  the  equation  corresponding  to  the  mean  second 
difference  of  the  moon's  place  in  longitude,  latitude,,  right  ascension,  or 
decUnation  ;  this  table,  besides  being  newly*arranged,  will  be  found  more 
exteasive  than  those  under  a  similar  denomination,  usually  met  with  in 
bocdca  on  navigation. 

.  Table  XVI IL  is  so  arranged  as  to  exhibit  the  true  correction  of  the 
moon's  apparent  altitude  corresponding  to  every  second  of  horizontal 
parallax,  and  to  every  miiiute  of  altitude  from  the  horizon  to  the  zenith  : 
and  will  prove  very  serviceable  in  dl  problems  where  the  moon^s  altitude 
forms  one  of  the  u^ments  either  given  or  required. 

Table  XIX.  is  fijUy  adapted  to  the  reduction  of  the  true  Utitudef  of  the 
hesranly  bodies,  obtained  by  calculation,  to  their  apparent  central  alti- 
tudes :  the  lednctions  of  altitude  may  be  very  readily  taken  out  to  the  de- 
cimal part  of  a  second.    This  table  will  be  found  of  considerable  utility  in 


Digitized  by 


Google 


X  PRBFACB. 

deducing  the  longitude  from  the  lunar  observations^  when  the  distance 
only  has  been  observed. 

Table  XX.  is  new;  and  by  its  means  the  operation  of  reducing 
the  apparent  central  distance  between  the  moon  and  sun,  a  fixed  star,  or 
planet,  to  the  true  central  distance,  is  very  much  abridged,  as  will  appear 
evident  by  referring  to  Method  I.,  vol.  i.,  page  433,  where  the  true  central 
distance  is  found  by  the  simple  addition  of  five  natural  versed  sines. 

T^ble  XXL,  which  is  also  new,  contains  the  correction  of  the  auxiliary 
angle  when  the  moon'si  distance  from  a  planet  Ls  observed :  this  will  be  of 
great  use  in  finding  the  longitude  by  the  moon's  central  .distance  from  a 
planet.    '     . 

Table  XXIV. — ^The  form  of  this  table  is  entirely  origin^  5  and  though 
It  is  comprised  in  nine  pages,  yet  it  is  so  arranged  that  the  logarithmic 
difference  may  be  obtained,  strictly  correct,  to  the  nearest  minute  of  the 
moon's  apparent  altitude,  and  to  every  second  of  her  horizontal  parallax* 
This  table  will  be  found  of  almost  general  use  in  the  problem  for  finding 
the  longitude  by  the  lunar  observations.  -  • 

Table  XXVL,  which  is  original,  contains  the  correction  of  the  logarithmic 
difference  when  the  moon^s  distance  from  a  planet  is  observed :  this  table 
will  be  found  of  great  use  in  computing  the  lunar  observations  whenever 
the  moon's  distance  from  the  planets  appears  in  the  Nautical  Almanac;  an 
improvement  which,  from  the  advertisement*  prefixed  to  the  late  Alma- 
nacs, may  be  shortly  expected  to  take  place. 

Table  XXVIL,  Natural  Versed  Sines,  &c.— Tlie  numbers  corresponding 
to  the  first  90  degrees  of  this  table  are  expressed  by  the  arithmetical  com- 
plements of  those  contained  in  the  Table  of  Natural  Co-sines  published 
by  the  author  in  ^^  The  Young  Navigator's  Guide,".  &c. ;  the  arithmetical 
complement  of  the  natural  co-«ine  of  an  arch  being  the  natural  versed  sine 
of  the  same  arch.  The  numbers  contained  ii)  the  remaining  90  degrees  of 
this  table  are  expressed  by  the  natural  sines,  frotn  the  abovementipned 
work,  augmented  by  the  radius. 

This  table  is  so  arranged  as:  to  render  it  general  for  every  arch  contained 
in  the  whole  semi-oircle,  and  conversely,  whether  that  arch  or  its  corre- 
lative be  expressed  as  tf.  natural  versed  sine,  natural  versed  sine  supplement, 
natural  co-versed  sine,  natural  sine,  or  natural  co-sine. 

Table  XXVIII.  is-  an  extension  of  that  published  by.  the  author  in 
^^  The  Young  Navigator's  Guide/'  &c. :  it  is  arra,nged  in  a  familiar 
maimer,  and,  thoi^gh  concise,  contains,  all  the  numbers  that  can  be  use- 
fully employed  in  the  elements  of  navigation ; '  for^  by  means  of  nine  co- 
lumns of  proportional  parts,  the  logarithmic  value  of  any  natural  number 
under  1839999  may  be  obtained  nearly  at  sight,  and  conversely. 

Tables  XXX.,  XXXI.,  and  XXXII.,  have  been  carefully  drawn  up,  and 
proportional  parts  adapted  to  them,  by  means  of  which  the  logarithmic 


Digitized  by 


Google 


FRBFXCB.  XI 

half-elapaed  time,  middle  time,  and  logarithmic  rising  may  be  very  readily 
taken  out  at  the  iiret  sight,  and  conversely. 

Table  XXXV.,  Logarithmic  Secants.— The  arrangement  of  this  table  is 
original,  as  well  as  its  leng^ :  the  numbers  contained  therein  are  expres- 
sed, by  the  arithmetical  complements  of  those  contained  ih  the  table  of 
logarithmic  co-siries  published  by  the  author  iii  '^  The  Young  Navigator's 
Guide,"  &c. 

This  table  is  so  drawn  up  as  to  be  property  adapted  to  every  arch 
expressed  in  degrees,  minutes,  and  seconds,  in  the  whole  semi-cirole, 
whether  that  arch  or  its  correlative  be  considered  as.  a  secant  or  a  co-secant; 
and  by  means  mf  proportional  parts,  the  absolute  value  of  any  arch,  and 
conversely,  may  be  readily  obtained  at  sight. 

Table  XXXVI., -Logarithmic  Sines. — ^This  table  is  rendered  general  for 
every  degree,  minute,  and  second,  in  the  whole  semicircle.  The  Table  of 
Logarithmic  Tangents,  which  immediately  fellows,  is  ako  rendered  gene- 
ral to  the  same  extent;  and  by  means  of  proportional  parts,  the  true  yabie 
of  any  arch,  and  conversely,  may  be  instantly  obtuned,  without  the  trouble 
of  either  multiplying  or  dividing  t  this  improvement,  to  the  practieal  navi- 
gator, must  be  an  object  of  great  importance,  in*  redueing  the  labour 
attendant  on  computations  in  Nautical  Astronomy. 

Table  JCXXVIIL  has. been  newly  computed  to  the  nearest  second  of 
time,  so  that  the  mariner  may  be  readily  enabled  to  reduce  the  time  of  the 
moon'«6  passage  over  the  meridian  of  Greenwich  to  that  of  her  passage  over 
any  other  meridian.  This  table  will  be  found  very  useful  in  determining 
the  apparent  time  of  the  moon's,  rising  or  setting,  and  also  in  ascertaining 
the  time  of  high  water  at  any  given  place  by  means  of  Table  XXXIX. 

Table  XLU.— This  general'  Traverse  Table,  so  useful  in  practical  navi- 
gation, is  arranged  in  a  very  different  manner  froih  the  Traverse  Tables  given 
in  the  generality  ef  nautical  book»;  and  although  comprised  in  38  pages, 
is  more  comprehensive  than  the  two  combined  tables  of  6  T  pages  usually 
found  in  those  books,  under  the  head  ^'Difference  of  Latitude ^and  Depar- 
ture." Ill  this  table,  every  page  exhibits  all  the  angles  that  a  ship's  cour^ 
can  possibly  make  with  the  meridian,  expressed  both  in  points  and  de^ 
grees ;  which  does  away  with  the  necessity  of  consulting  two  tables  in  find- 
ing the  difference  of  latitude  and  departure  corresponding  to  any  given 
course  and  distance. 

Table  XLIV.  contains  the  mean  right  ^censions  and  declinations  of  the 
principal  fixed  stars.  The  eighth  column  of  this  table,  which  is  origi- 
nalj  and  is  intended  to  facilitate  the  method  of  finding  the  latitude  by  the 
altitudes  of  two  fixed  stars  observed  at  any  hour  of  the  night,  contains 
the  true  spherical  distance  between  the  stars  therein  contained  and 
those  preceding  or  abreast  of  them  on  the  same  horizontal  line.  The 
mnth  or  last  colunm  of  the  page  contains  tha  annual  variation  of  that 


Digitized  by 


Google 


Xll  PRBFAGB. 

dhtancBf  expressed  in  seconds  and  decimal  p&rts  of  a  second.  Gieat 
pains  have .  been  taken^  in  order  to,  find  the  absolute  value  of  the  an- 
nual variation  of  the  true  spherical  distance  between  the  fixed  stars ;  and 
the  author  ti^ists.that  he  has  so  far  succeeded  as  to  render  this  part  of  the 
table  permanent  foi;  aloHg  period  of  years  wbseqiient  to  1824*  • 

Tables  XLV.  and  XL VI  *j  which  are  adapted  to  the  reduction  of  sidereal 
time  into  mean  solar  time,  and  conversely,  have  been  newly  constructed  : 
these  will  be  found  considerably  more  extensive  and  uniform,  than  those  ge- 
nerally given  under  the  same  denomination. 

Tables. LL.aiid  LIL  are  entirely  new:  .these  will  be  found  exceedingly 
useful  in  finding  the  latitude  by  the  altitude  of  a  celestial  -object  observed 
at  certain  intervals  from  the  meridian.;  and  since  they  are  adapted  to  pro* 
portional  logarithms,  the  operation  of  finding,  the  latitude  thereby  becomes  , 
extremely  simple^*  and  yet  far  more  accurate  than  tKat  resulting  from  don* 
ble  altitudes,,  even  after  repeating  a  trouUesome  operation,  and  then 
applying  correctkmto  correctiofu 

Table  LIV«— This  table  will  be  of  service  to  Masters  iki  the  Royal  Navy, 
to. officers  employed  in  maritime  surveys,  and  to  all  others  who  may  be 
desir6us  of  constructing  charts  agreeably  to  Merpator's  principles  of 
projection.  •  . 

Table  LVL  will  be  found  essentially  useful  in  reducing  the.  Erench 
centesimal  division  of  the  circle  into  the  English  sexagesimal  division, 
and  conversely ;  and  since  most  of  the  mpdem  French  works  on  astronomy 
are  now  adapted  to  the  centesimal  principle,  this  table  will  be  found  of  aik- 
sistance  in  consulting  those  works  j^nor  will  it  be  of  less  advantage  to  the 
French  navigator,  in  enabling  him  readily  to  consult  the  works  of  Jthe  BngHsh 
astronomers,  where  the  degrees,  &c.,  are  expressed  agreeably  to  the  original 
or  sexagesimal  principle. 

Table  LVIL  is  new;  and  although  -it  m^y  not  immediately  affect  the 
interest  of  the  mariner,  yet  it  qannot  fail  to  be  usefiil  to  officers  in  charge 
of  His  Majesty's  Victualling  Stores,  in  consequence  of  the  late  Act  of  Par-* 
liament  for  the  establishment  of  a  new  generd  standard  or  imperial  gallon 
measure  throughout  the  United  Kingdoms^ — See  Practical  Gauging, 
page  596  to  606. 

Table  LVIII.  contains  the  latitudes  and  longitudes  of  all  the  principid 
sea-ports,  islands^  capes,  shoals,  rocks,  &c.  &c.)  in  the  known  worid ;  tl^se 
are  so  arranged  as  to  exhibit  to  the  navigator  the  whole  line  of  coast  along 
which  he  may  have  occasion  to  sail,  or  on  which  he  may  chaiiee  to  be 
employed,  agreeably  to  the  manner  in  which  it  unfolds  to  his  view  on  a 
Mercator's  chart ;  a  mode  of  arrangement  much  better  adapted  to  nautical 
purposes  than  the  alphabetical.  But  since  the  table  is  not  intended'  for 
general  geographical  purposes,  the  positions  of  places  inland,  which  do 
not  immediately  concern  the  mariner,  have,  with  a.  few  exceptions,  beeii 


Digitized  by 


Google 


PkfS»ACA.  xifi 

pmpoeely  omitted.  The  time  6f  high  water,  at  the  full  and  change  of 
the  moon/ is  giveu  at  all  places  where  it  is  known;  which  will  be  found 
considerably  more  convenient  than  referring  for  it  to  a  separate  table. 

The  series  of  latitudes  and  longitudes  that  have;  been  established,  astro- 
nomically and  chronometricdly,  by  Captatn  William  F1t£william  Owen, 
of  Hia  Majesty's  ship  Eden,  during  his  recent  and  extensive  survey  along 
the  coasts  of  Africa,  Arabia,  Madagascar,  Brazil,  &c.,  follow  as  an 
Appendix  to  the  last^mentioned  table.  These  series  are  published  by  the 
express  permission  of  Captain  Owen ;  And  from  his  general  kno}vIedge  as 
a  navigator^  hydrographer,  and  practical  astronomer,  there  is  every  rea« 
SOD  to  believe  that  the  geographical  positions  have  been  determined  with 
astronomical  exactness. 

A  general  Victualling  Table  forms  ah  addition  to  the  Appendix ;  And  as 
this  exhibits  the  foil  allowance  of  sea  provisions  (calculated  agreeably  to 
the  n6w  Victualling  Scale),  from  one  man  to  any  given  number  of  men, 
it  will  lie  found  useful  l;o  the  Pursers  of  the  Royal  Navy,  to  Lieutenants 
serving  as  Commanders  and  Pursers,  and  to  the  gentlemen  who  are 
officially  employed  in  the  auditing  of  the  Naval  Victualling  Accounts.  ' 

The  smt's  declination  is  not^  given  in  this  work ;  nor  is  it  necessary  that 
it  should  be,  since  it  is  contiuned,'  in  the  most  ample  manner,  in  the  Nctu^ 
tieal  Almanac;  a  work  which  is  so  truly  valuable  to  mariners  that  few 
now  go  to  sea  without  it ;  the  judicious  never  ^ilL 

Hairing  thus  taken  a  survey  of  the  principal  part  of  the  Tables,  I  must 
briefly  norice  their  cbsimpfion  and  U$e  ;->-these  will  be  found  at  the  com-* 
menocanent'  of  the  first  Volume.  The  principles  and  methods  of  their 
computation  are  here  fully  detailed ;  and  the  reader  is.  furnished  with  the 
means,  in  the  most  simpkr,foTmttlffi>  of  examining  any  part  of  the  Tables ; 
which  ia  far  more  satisfactory  than  trusting  to  the*  author^  mere  word  for 
their  entire  accuracy  \  though,  I  flatter  myself  with  the  hope  t)>at,  in  this 
extensive  mass  of  flgarea,  very  few  errors  Will  be  found  ;-^at  all  events, 
none  of  pruicipU. 

My  origbal  plan  bad  been  to  clofte  the  work  with  the  description  and  use 
of  the  Tables^  but  being  apprehensive  that  a  series  of  Tables  alone,  how- 
ever well  arranged,  or  clearly  illustrated,  would  not  be  sufficient  to  ensure 
genehd  aoceptation,  I  was  induced  to  show  their  direct  application  to  the 
different  elements  connected  with  the  sciences  of  navigation  and  nautical 
astronomy,  as  well  as  to  other  subjects- of  a  highly  interesting  nature,  such 
as  the  art  of  gunnery,  &c<  &c.  In  this  part  of  the  work,  since  my  design 
did  not  extend  beyond  an  ample  illustration  of  the  various  mathematical 
purposes  to  Which  these  tables  may  be  applied,  I  have  restricted  myself  to 
the  practical  parts  of  the  sciences  on  Which  I  have  Had  occasion  to  touch  ; 
because  those  are  the  points  which  most  concern  themariner,*and  the  com- 
inerei&l  intereets  of  this  maritime  nation.    Nevertheless^  wherever  it  has 


Digitized  by 


Google 


appeared  necessary  to  notice  the  elementary  parts  of  the  sciences,  refer* 
ence  has  been  made  to  relative  problems  iq  *^  llie  Young  Navigator's 
Guide,"  where,  it  is  hoped,  the  reader  will  find  his  inquiries  fully  satisfied. 

The  various  sciences  touched  upon  commence  with  a  concise  system  of 
decimal  arithmetic,  and  complete  courses  of  plane  and  spherical  trigono- 
metry. In  the  latter,  the  solution  of  the  quadrantal  triangles  vdll  be  found 
much  simplified.' 

Thepractical  parts  of  Navigation  begin  with .  parallel  sailing ;  but,  with 
the  view  of  preventing  the  work  from  swelling  to  an  unnecessary  size,  the 
cases  of  plane  sailing,  usually  met  with  iti  other  nautical  books,  have  been 
omitted  in  this ;  as  these  are,  in  effect,  no  more  than  a  mere  repetition  of 
the  cases  of  right  angled  plane  trigonometry  under  a  different  denoniination. 
Middle  latitude  sailing  will  be  found  exceedingly  simplified  by  means  of  a 
series  of  familiar  analogies  or  proportiions :  and  in  Mercator*s  sailing  a  se- 
ries of  rational  proportions  is  given  ;  which,  it  ishoped,  may  tend'to  in- 
duce mariners  to  substitute  the  rules  of  reason  for  the  rules  of  rote  j  and 
thus  do  away  with  the  mistaken  system  of  getting  canone  by  hearty  a 
system  which  has  too  long  prevailed  in  the  Royal  Navy. 

The  two  very  useful  sailmgs,  pblique  and  windward,  which  have  been 
hitherto  little  noticed  by  mariners,  are  also  rendered  so  simple,  parliculariy 
the  latter,  that  it  is  to  be  hoped  they  will,  ere  long,  be  brought  into  general 
use. 

In  current  sailing  (Example  3,)  the  true  principles  of  steering  a  vessd  in 
a  current,  or  tideway  are  familiarly  illustrated.  This  problem  cannot  fail  of 
behig  interesting  to  every  person  who  is  at  all  curious  in  the  art  of  !»•« 
vigation. 

The  solution  of  a  problem  in  great  ^circle  sailing  is.  given,  which  will  be 
found  essentially  useful  to  ships. bound  from  the  Cape  of  Good  Hope  to 
New  South  Wales*:  comprising  a  table  which  exhibits,  at -sight,  all  the 
scientific  particulars  attendant  on  the  true  spherical  track  between  those 
two  places ;  by  which  it  will  be  seen  that  a  saving  of  585  miles  may  be 
effected  by  sailing  near  the  arc  of  a  greatxircle  a»laid  down  in  that  table  $ 
which  saving  ought  to  be  an  object  ofvery  high  consideration  to^all  sliips 
bound  from  the  Cape  of  Good  Hope  to.  Van  Diemen*s  Land,  <v  to  Us 
Majesty's  colony  at  New  South  Wales  with  either  troops  or  convicts ;  .be* 
cause  the  length  of  the  voyage  on  the  old  track,  or  that  deduced  from  the 
common  principles  of  navigation,  generally  occasions  a  great  scarcity  of 
fresh  .water,  and  this,  eventually,  adds  distress  to  the  many  privations  vnder 
which  those  on  board  usually  labour.  In  the  same  problem,  there  is  a  table 
showing  the  true  spherical  route  from  Port  Jackson,  in  New  South  Wales, 
to  Valparaiso,  on  the  coast  of  Chili :  in  this  route  there  is  a  saving  of  745 
miles  when  compared  with  that  resulting  from  Mercator*s  sailing  ;  and  this 
must  be  of  considerable  importance  to  the  captain  of  a  ship  sailing  between 


Digitized  by 


Google 


PRBVACB.  Xy 

these  places,  who  is  desirous  of  making  his  port  in  the  shortest  space  of 
time ;  particularly  since*  few  shijps  can  carry  a  liberal  allowance  of  fresh 
water  to  serve  during  a  passage  which  measures  very  nearly  one  fourth  of 
the  earth's  circumference. 

The  introductory  problems  to  Nautical  Astronomy  will  be  found  rang^ 
in  the  most  natural  order ;  all  of  which,  except  those  relating  to  the  alti- 
tudes of  the  objects,  are  concisely  solved  by  proportional  logarithms : 
the  greater  part  of  these  will  appear  entirely .  new  to  the .  navigator. 
The  Vlth  problem  relating  to  the  latitude  exhibits  the  method  of  finding 
the  latitude  by  an  altitude  of  the  uorth  polar  star  taken  at  any  hour  of 
the  night,  which  will  be  found  very  useful  in  all  parts  of  the  northern 
hemisphese. — ^The  Vlllth  problem  shows  the  method  of  finding  the  latitude 
by  the  altitudes  of  two  stars  taken  at  any  time  of  the  night,  agreeably  to 
the  computed  spherical  distance  between  them  contained  in  Table  XLIV ; 
this  method  of  ascertaining  the. latitude  is  general;  it  will  be  found  very 
correct,  «nd  far  less  troubletome  than  that  by  double  altitudes  which  im- 
mediately precedes  it.— Problems  IX,  X,  XI,  and  XII,  contain  new  and 
accurate  methods  of  deducing  the  latitude  from  the  altitudes  of  the  celestial 
bodies  observed  at  given  intervals  from  the  meridian  :  the  operation  con- 
sists of  very  little  more  than  the  common  addition  of  three  proportional 
Icgarithms,  and  yet  the  latitude  resulting  from  it  will  always  be  as  cor- 
rect as  that  deduced  from  the  object's  •  meridional  altitudes,  provided  the 
watch  shows  apparent  time  at  the  place  of  observation,  and  the  altitudes 
be  taken  within  the  limits  prescribed.  These  problems  will  be  found 
highly  advantageous  to  the  practical  navigator ;  because,  in  the  event  of 
the  sun's,  or*  other  celestial  object's  meridional  altitude  being  neglected 
to  be  taken,  or  of  it's  being  obscured  by -clouds  at  the  time  of  transit,  he 
is^  thus,  provided  with  the  most  sale  and  jready  means  of  determining  his 
latitude  with  as  much  certainty  as  if  the  altitude  of  the  object  had  been 
observed  actually  upon  the  meridian  eitlier  above  or  below  the  poIe«  See 
remark,  page  368. 

A. most  ingenious  problem  in  this  part,  of  the  work,  for  determining  the 
latitude,  which  for  neatness  and  general  utility  stands  unrivalled,  ha»  been 
communicated  to  the  author  by  the  scientific  Captain  W.  F.  W.  Owen. 

In  the  methods  of  computing  the  altitudes  of  the  heavcf^ly  bodies,  the 
solutions  to  the  several  problems  are  rendered  exceedingly  concise  and 
explicit. 

The  Ilird,  IVth,  Vth>  and  Vlth  problems  relating  to  the  longitude  con- 
tain the  methods  of  finding  the  longitude  by  a  chronometer  and  the  res- 
pective altitudes  of  the  sun,  stars,  planets,  and  the  moon  i  the  three  last 
of  which  will  be  found  considerably  elucidated. 

The  lunar  observations  commence  with  the  Vllth  problem  on  the  lon- 
gitude.    In  this  problem  thirteen  methods  are  given  for  reducing  the  ap« 


Digitized  by 


Google 


jtfi  PREFACB* 

parent  central  distance  between  the  moon  and  enn^  a  fixed  star,  or  planet, 
to  the  true  central  distance  ;  several  of  which  are'entirely  original,  and  all 
of  them. adapted  to  solve  this  interesting  and  important  problem  in  the 
moqt  simple  and  expeditious  manner. 

•  In  the  series  of  problems  relative  to  finding  the  variation  of  the  compass 
byamplitudes,  azimuths,  transits  of  the  fixed  stars  and  planets,  and  by  ob« 
servations  of  the  circumpolar  stars,  Problem  II  exhibits  a  n«tom^fAod  for 
computing, the  true  azimuth  of  a  celestial  object :  and  Problems  V  and  VI, 
contain  the  methods  of  reducing  or  correcting  the  true  and  the  magnetic 
courses,  between 'two  places,  tigreeably  to  any  given  variation  of  the.  com- 
pass.—-An  improved  azimuth  compass  card  is<described  in  this  part  of  the 
work,  which  may  be  applied  to  the  determination  of  the  longitude  by  the 
lunar  observations  : — See  the  last  two  paragraphs  in  page  499.  '  * 

The  series  of  problems  for  finding  the  apparent  times  of  the  rising  or 
setting  of  the  celestial  bodies,  and  of  the  beginning  or  tHe  end  of  twi-' 
light  ;^and  that  for  determining  the  interval  'of  time  between  tiie  rising 
or  setting  of  the  sun's  upper  and  lower  limbs,  it  is-hoped  will  prove  ac« 
cep table  to  the  lovers  of  the  science  of  Nautical  Astronomy ;  ^likewise  the 
art  of  Dialling,  which,  although  it  may  appear  foreign  or  irrelevant  to  the 
pursuits  of  the  mariner,  cannot  fail  to  be  interesting  as  a. branch  of 
science.  *  It  is  here  treated  of  in  a  fieimiliar  manner.  *   .  '    -■ 

The  IVth  Problem  in  the  mensuration  of  heights  and  distances,  exhibits 
the  method'  whereby  the  officers  on  board  two  ships*  .of  war  can  readily 
ascertain  their  absolute  distance  firom  any  fort  or  garrison  which  tfiey  may 
be  directed  to  cannonade  ;-^afler  which  follow  several'  problems  that  will 
be  fpund  exceedingly  useifol  on  many  military  occasions.-^See  remark  at 
page  53S,  and  also  at  page  543.  Problem  XL  showing  the  method  of  re*- 
ducing  a  base  line,  measured  on  any  elevated  horizontal  plane,  .to  its  true 
level  at  the  surface  of  the  sea;,  and  Problem  XIII.  exhibiting  a  new  rule 
for  Ending  the  height  of  e  mountain,  or  other  eminence,  by  means*  of  two 
barometers  and  two  thermometers,  may  be  of  considerable  use  to  engi- 
neers, or  to  others  employed  in  conducting  surveys*  A  problem  is  also 
given  for  finding  the  direct  course  steered  by-  a  ship  seen  at  a  distance; 
and  being  a  subject  highly  interesting  to  all  nautical  persons,  it  is  reduced 
to  every  desirahle  degree  of  simplicity  both  by  geometry  and  trigonometry. 

All  the  problems  in  Practical  Gunnery  are  readily  solved  by  logarithms  x 
it  contains  three  very  concise  tables  whicji  considerably  facilitate  the  oper- 
ation for  finding  the  greatest  range  of  a  shot  or  shell,  and  the  elevation  of 
the  piece  to  produce  that  range.  A  small  table  is  also  given,  which  will 
be  found  extremely  useful  in  problems  relating  to  shells,  when  it  is  requirecl 
that  they  should  strike  an  object  at  a  given  distance. — The  rules  and  oper- 
ations for  computing  the  time  of  flight  of  a  shell  in  Problems  XXVII> 
XXXIV,  and  XXXVI,  wiU  be  found  very  simple  and  concise. 


Digitized  by 


Google 


PRSFACB.  XVn 

Although  the  art  of  gunnery  may^  in  some  measure,  be  eonsidered  as 
not  being  immediately  connected  with  that  of  navigation;  yet  it  is  a 
subject  with  which  all  naval  officers  ought  to  have  some  acquaintance ; 
since  it  very  frequently  happens,  in  time  of  war,  that  they  are  called  upon 
to  go  on  shore  with  a  party  of  men  for  this  purpose  of  working  the  great 
guns  of  the  besieging  batteries  in  co-operation  with  his  Majesty'a  Land 
Forces.: — and  since  this  truly  interesting  art  is  here,  for  the  first  time,  un- 
veiled of  its  mystic  dressy  and  reduced  to  a  state  of  simplicity,  every 
officer  may  make  himself  thoroughly  acquainted  with  it  in  a  very  little 
time,  without  any  other  assistance  than  that  afforded  in  this  treatise. 

The  problems  on  the  mensuration  of  planes  may  be  found  useful  on 
many  occasions  ;  particularly  to  persons  employed  in  carrying  on  surveys 
on  shore. 

Practical  Gauging  contains  a  few  interesting  problems ;  the  last  of  which 
will  be  found  essentially  useful  to  such  persoQS  as  may  have  occasion  to 
purchase  wine,  or  spirits  on  his  Majesty's  account  in  foreign  countries; 
because  it  enables  them  to  ascertain,  in  a  very  few  minutes,  the  absolute 
number  of  gallons '  contained  in  any  given  quantity  of  foreign  liquor,* 
agreeably  to  the  newly  established  standard  or  Imperial  gallon  measure. 

The  compendinm  of  Practicial  Navigation,  given  in  this  volume,  exhibit- 
ing the  direct  manner  of  making  out  a  day^s  work  at  sea,  is  intended  for 
the  benefit  of  such  persons,  as  may  be  unacquainted  with  the  elements  of 
geometry  and  trigonometry  :  and  includes  the  true  method  of  finding  the 
index  error  of  a  sextant  or  quadrant  so  as  to  guard  against  the  error  arising 
from  the  elasticity  or  spring  of  the  index  bar,  with  the  method  of  appl}ing 
the  corrections  to  altitudes  taken  on  shore  by  means  of  an  artificial 
horizon. 

A  new  and  correct  method  of  finding  the  longitude  of  a  place  on  shore 
by  means  of  the  moon's  altitude  (observed  in  an  artificial  horizon,)  and  the 
apparent  time  of  observation,  follows  the  above  compendium;  and  will  be 
found  of  considerable  utility  in  settling  the  geographical  positions  of  places 
inland  or  along  the  sea  coast.  An  Appendix,  which  concludes  the  first 
volume,  contains  everything  relating  to  the  doctrine  of  compound  interest; 
and  developes  the  extraordinary  powers  pf  logarithipical  numbers  in  a  more 
striking  point  of  view  than  any  other  department  of  science  to  which 
they  have  been  applied. 

I  have  thus  given  a  brief  account  of  the  more  original  parts  of  the  sub- 
jects comprised  in  this  work,'  the  completion  of  which  has  cost  me  several 
years  of  incessant  labour ;  during  which  time  I  had  to  contend  with  as 
many  infirmities,  vexations,  and  disappointments  as  generally  fall  to  the  lot 
of  persons  doomed  to  drudge  through  the  toils  of  life :  but  stimulated  by 
the  hope  of  ultimately  succeeding  in  rendering  myself  useful  to  the  Naval 
Service  of  his  Majesty,  and  to  the  nautical  world  in  general,  I  have  been 


Digitized  by 


Google 


XYIU  PRBFACB. 

SO  far  enabled*  to  bear  up  against  the  vicissitudes  of  health  and  fortune^ 
as  to  bring  my  long  and  arduous  task  to  a  close. 

How  far  I  have  succeeded  in  my  endeavour  to  supply  the  desideratum 
which  has  l)een  hitherto  felt  by  navigators,  it  is  not  for  me,  but  for  a  gene- 
rous British  public  to  determine :  to  their  decision  I  submit  my  labours, 
tinder  the  conviction  tl)at,  whatever  may  be  the  defects  in  its  execution, 
they  will  do  justice  to  my  motives,  in  this  attempt  to  lessen  the  existing 
obstructions'  in  the  way  of  attaining  a  practical  knowledge  of  the  elements 
of  Navigation  and  Nautical  Astronomy* 

,        .  TOOMAS  KERIGAN. 

PwUmmiik,  December  l«r., 
182r. 


Digitized  by 


Google 


LIST  OF  SUBSCRIBERS. 


His  Royal  Higfaneas  the  Duke  of  Clarei^pe  and  St  Andrews,  Lord  High  Admiral 

of  the  United  Kingdoms  of  Great  Britain  and  Ireland,  dec.  drc.  Sec* 
The  Right 'Honourable  the  (late)  Lords  CommisaionerB  of  the  Admiralty,  One 

Hundred  Guineas  for  10  copies. 
The  Elder  Brethren  of  the  Honourable  Trinity  Corporation^  One  Hundred  Pounds 

for  5. copies. 
The  Court  of  Directors  of  the  Honourable  East  India  Company,  One  HuAdred 

Guineas  for  10  copies. 

The  Honourable  the  Commissioners  of  His  Majesty's  Nafy,  5  copies. 

The  Honourable  the  Commissibners  for  Victualline  His  Mi^esty's  Navy^  6  copies. 

The  Right  Honourable  and  Honourable  the  Directors  of  Greenwich  Hosftital, 

The  Committee  of  Lloyd's,  Ten,  Guineas  for  2  copies. 

The  Royal  Naval  Club,  New  Bond  Street.  *  '  * 

The  British  library,  St.  Helier*s,^  Jeney. 

Capt.  R.  Anderson,  R.N. 

Capt»  F,  W.  Austen,  C.B.,  R.N.,  Gosport 

Lieutenant  J.  W.  Aldridge,  R.N.,  North  Street,  Bristol. 

Liettt.  H.  T;  Austin,  R.N:,  Chatham. 

Mr.  Herbert  Allen,  H.M.S.  Heron. 

ISenry  Adcock,  esq.,  Polygoq,  Somen'  Town. 

Vice  Admual  the  Hon.  Sir  Henry  Blackwood^  bart.  K.C.B.,  Commander  in 

Chief  at  the  Nore. 
Commodore 'C.  Bulten^C.  B.    . 
Capt.H.W.  Bayfield,  R.N. 
Cape.  A.  B.  Branch,  R.N. 
Capl.  J.  W.  Beechey,  H.M.S.  Blossom. 
Capt.  Edward  Brace,  C.B.,  It.N. 
Capt.  R.  L.  Baynes,  H.  M.  S.  Alacrity. 
Lieut.  A.  B.  Becher,  R.N.,  Hydrographical  Office,  Admirarty* 
Lieut.  Philip  Bisson,  R.N.,  St.  Heller's,  Jersey. 
The  Honourable  Frederic  Byug. 
lient.  Jacob  Bucknor,  R.N. 

Robert  Bried,  esq.,  Surgeon,  R.N.,  Spencer  Street,  ClerkenwelL 
Mr.  Wm.  H.  Brown,  Purser,  H.M.S.  Musquito. 


Digitized  by 


Google 


XX      ^  LIST  OF  SUBSCRIBERS. 

Mr.  W.  P.  Browne,  R.N.,  Plymouth. 
Mr.  John  Browning,  R.N.,  Ann's  Hill  Place,  Gosport. 
Thovias  Best,  esq.  ' 

Alexander  P.  Bond,  esq.,  Edgewortbstown,  Ireland. 
Mr.  James  Bradlej,  Hanoyer  Street,  Portsea. 


Admiral  Sir  Isaac  Coffin,  hart.,  Titley  Court,  Hereford. 

Yice  Admiral  Sir  Edward  Codnngton,  ^.C.B.,  Commander  in  Chief,  Mediter- 
ranean, 6  copies. 
Capt.  Janes  Campbell*  H.M^.  Slaney. 
Capt.  Henry  D.  Chads,  C.B.,  R.N. 
Capt  E.  Chetharo,  C.B.,  R.N.,  Gosport. 
Capt  D.  C.  Clavering,  H.M.S.  Redwing. 
Capt  Benj.  Clement,  R'.N.,  Chawton,  Hants/ 
•Capt.  Augustus  W.  J.  CliflFord,C.B.,  H.M.S.  Undaunted.    . 
Capt  Charles  Crole,  R.N. 
Capt  E.  Curzon,  H.M.S.  Asia. 
Lieut  Edward  St.  L.  Cannon,  Q.M.^.  Wolf. 
Mr.  Jfunes  Cannon,  H.M.S.  Thetis:  * 

Lieut.  W..J.  Cole,  Royal  George  Yacht* 
Lieut  P.  E.  Cqjlins,  H.N. 
Lieut  Edward  Corbet,  R.N. 
Mr.  Champronier,  H.M.S.  Eden'. 
Mr.  Thos.  Cox,  Purser,  H.M.S.  Pyramus. 
Simon  Cock,  esq.  New  Bank  Buildings,  London. 
William  Curtis,  esq.,  Portland  Place. 

The  Rev.  Colin  Campbell,  Widdington  Rectory,  Bishop's  Stortford,  Ettex. 
Mr.  Comerford,  Bookseller,  Portsmouth,  6  copies. 
Mr.  Crew,  Bookseller,  High  Street,  Portsmouth,  6  copies, 


Capt  Nevinson  IJeCourcy,-  R.N*.,  Stoketon  House,  Plymouth. 

C^pU  Manley  Hall  Dixon,  R.N.,  Stoke,  near  Deyonport. 

Capt  George  Shepherd  Dyer,  R.^.  • 

Lieut.  Henry  M.  Denham,  Linnet  Surveying  Vessel. 

The  Rev.  E.  Davies,  H.M.S.  Pyramus. 

—  Douthwaite,  esq.,  Commander  of  the  Circassian  India  Ship. 

Admiral  tlie  Right  Hon.  Lord  Viscount  Exmotttbi  G.C.B, 

Capt.  R.  Evans,  R.N. 

Lieut.  The  Hon.  Wm.  Edwardes,  H.M.S.  Asia. 

Lieut  John  Evans,  (a)  R.N. 

Lieut.  Thos.  Eyton,  R.N. 

Lieut.  W.  W.  Eyton,  H.M.S,  Wolf.  . 

The  Rev.  J.  M.  Edwards,  H.M.S.  Otdatea. 


Digitized  by 


Google 


THE 

DESCRIPTION    AND    USE 

OF  TBB 

TABLES; 

WITH   TB« 

PRINCIPLES  UPON  WHICH  THEY  HAVE  BEEN  COMPUTED. 


Table  I. 
To  convert  longitude^  or  Degrees^  into  Time,  and  conversely. 

THIS  Table  consists  of  six  compartments,  each  of  which  is  divided 
iiito  two  columns :  the  left-hand  column  of  each  compartment  contains 
the  longitude,  expressed  either  in  degrees,  minutes,  or  seconds ;  and  the 
right-hand  column  the  corresponding  time,  either  •  in  hours,  minutes, 
seconds^  or  thirds.  The  proper  signs,  for  degrees  and  time,  are  placed  at 
the  top  and  bottom  of  their  respective  columns  in  each  compartment,  with 
the  view  of  simplifying  the  use  of  the  l^ble :— hence  it  will  appear  evident 
that  if  the  longitude  T)e  expressed  in  degrees,  the  corresponding  time  will 
be  either  in  hours  or  minutes ;  if  it  be  expressed  in  minutes,  the  corre- 
sponding time  will  be  either  in  minutes  or  seconds ;  and  if  it  be  expressed 
in  seconds,  the  corresponding  time  will  be  expressed  either  in  seconds  or 
thirds.   The  converse  of  this  takes  place  in  converting  time  into  longitude. 

The  extreme  simplicity  of  the  Table  dispenses  with  the  formality  of  a 
rule  in  showing  its  use,  as  will  obviously  appear  by  attending  to  the^folbw- 
iog  examples. 

Example  1. 
Required  the  time  corresponding  to  47?47'47?  of  longitude  ? 

47  degrees,  time  answering  to  which  in  the  Table  is  3*  8?  0!  Of 

•  47  minutes,  answering  to  which  is     •    •    «    .0.  3.  8.  0 

•  .47  seconds,  answering  to  which  is    •  '  •    .  0.  0.  3.  8 

Urn.  47?47'47^  the  time  corresponding  to  which  is    .  3M1?U!8! 

B 

Digitized  by  VjOOQ IC 


.•  .  jDESCHIPTION  A^  USB  OF  THB  TABLES* 

Example  2.- 

Required  the  longitude  corresponding  to  the  given  time  8*52T28!  ? 
8  hours;  longitude  answering  to  which  in  the  Table  is  •  120?0'0^ 

•  52  minutes,  answering  to  which  is    .•..••     13.0.0 

•  .    28  seconds^  answering -to  which  is      .    *    .   ^      0.0.7 


'rime8t52?28!,  the  longitude  corresponding  to  which  is   .  133?0'7^ 

Besides  the  use  of  this  Table  in  the  redactioh  of  longitude  into  time, 
and  the  contrary,  it  will  also  be  found  very  convenient  in  problems  relating 
to  the  Moon,  where  it  becomes  necessary  to  turn  the  right  ascension  of 
that  object  into  time. 

Example. 
The  right  ascension  of  the  Moon  is  3S5?44C48r;  required  the  corre- 
sponding time  ? 

355  degrees,  time  answering  to  which 

intheTableis  .    .    .    .    •    23*40?  0!  Of 
•    44  minutes,  answering  to  which  is    *  0.   2. 56.   0 
«     .    48  secs.^  answering  to  which  ifl       0.  0.   3,12 

Right  ascen^on  355?44U8Tj  tlie  time  corresponding  to 

which  is      •    •    •    .    23*42T59!12f 

Since  the  Earth  makes  one  complete  revolution  on  its  axis  in  the  space 
<Jf  24  hours,  it  is  evident  that  every  part  of , the  equator  will  describe 
a  great  circle  of  360  degrees  in  that  time,  and,  consequently,  pass 
the  plane  of  any  given  iheridian  once  in  every  24  hours ;  whence  it  is 
manifest  that  any  given  number  of  degrees  of  the  equator  will  bear  the 
same  proportion  to  the  great  circle  of  360  degrees  that  the  corresponding 
time  does  to  24  hours;  and  that  any  given  portion  of  tdme  will  be  in  the 
same  ratio  to  24  hours  that  its  corresponding  number  of  degrees  is  to  360. 

Now  since  24  hburs  are  correspondent  or  equal  to  360  degrees,  1  hour 
must,  therefore,  be  equal  to  15  degrees;  1  minute  of  time  equal  to  15 
minutes  of  a  degree;  1  second  of  time  to  15  seconds  of  a  degree,  and  so 
on.  And  as  1  minute  of  time  is  thus  evidently  equal  to  15  minutes  or  one 
fourth  of  a  degree,  it  is  very  clear  that.4  minutes  of  time  are  exactly  equal 
to  1  degree ;  wherefore  since  d^ees  and  time  are  similarly  divided,  we 
have  the  following  general  rule  for  converting  longitude  into  time,  and 
vice  t)0f«a»  , 

Multiply  the  given  degrees  by  4,  and  the  product  will  be  the  corre* 
sponding  time  :-*<*observing  that  seconds  multiplied  by  4  produce  thirds ; 
minutes,  so  multiplied,  produce  seconds,  and  degrees  minutes  j  which, 
divided  by  60,  will  give  hours,    llie  convene  <tf  thb  is  evident  :--thw. 


Digitized  by 


Google 


BBSCAimON  AND  USB  OF  THB.TABUUU 


reduce  the  hotm  to  minutes;  then  these  minutes^  divided  by  4,  trill  give 
d^eea ;  the  seconds^  ao  divided,  will  give  minute,  and  the  thirdsi  if  any^ 
seeonds.  Hence  the  prificiples  upon  which  the  Table  has  been  copiputed. 
The  following  eumples  are  given  for  the  puipoae  of  illustrating  the  above 
rale. 


Example  !• 

Required  the  time  corresponding 
toS6?44:32f? 

Given  degrees  =  36?44'32r 
Multiplied  by  4 

Corresponding  time  2^26r58!8! 


JExampZe2. 

Required  the  degrees  correspond- 
ing to  3  *  45^48 1 20  f  ? 

Given  times3M5?48f20f 
60 


Divide  by    4)225.48.20 
Corresponding  degs.  56?27-5T 


Tablb  II. 

Depression  of  the  Horizon. 

The  depression  or  dip  of  the  horizon  is  the  angle  contuned  between  a 
horizontal  line  passing  through  the  eye  of  an  observer,  and  a  line  joining 
his  eye  and  the  visible  horizon. 

This  Table  contains  the  measure  of  that  angle,  which  is  a  correction 
expressed  in  minutes  and  seconds  answering  to  the  height  of  the  observer's 
eye  above  t^e  horizon ;  and  which  being  subtracted  from  the  observed 
central  altitude  of  a  celestial  object,  when  the  fore  observation  is  used,  or 
added  thereto  in  the  back  observation,  will  show  its  apparent  central  altitude. 
The  corrections  in  this  Table  were  deduced  from  the  following  considera- 
tions, and  agreeably  to  the  principles  established  in  the  annexed  diagram. 


Let  the  small  circle 
A  B  C  6  represent  the 
terrestrial  globe,  and  eO 
the  height  of  the  ob- 
server's eye  above  ita 
surface  ;  then  HOQ, 
drawn  parallel  to  a  tan- 
gent line  to  the  surface 
at  e,  will  be  the  true  or 
sensible  horizon  of  the 
observer  at  O;  and  O  P, 
touching  the  surface  at 
T,  the  apparent  horizon. 


Digitized  by 


Google 


4  BMCftlPTION  AN0  USB  OF  THB  TAKLBS. 

•  Let  S  be  an  object  whose  altitude  is  to  be  taken  by  a  fore  observation, 
by  bringing  its  image  in  contact  with  the  apparent  horizon  at  P ;  then  the 
angle  SOP  will  be  the  apparent  altitude,  which  is  evidently  greater  than 
the  true  altitude  S  O  H  by  the  arc  P  H,  expressed  by  the  angle  of  horizontal 
depression  PO  H.  But  if  the  altitude  of  the  object  S  is  to  be  taken  by  a 
back  observation,  then,  the  observer's  back  being  necessarily  turned  to  the 
object,  his  apparent  horizon  will  be  in  the  direction  O  F,  and  his  whole 
horizontal  plane  represented  by  the  line  D  O  F ;  in  which  case  his  back 
horizon  O  D,  td  which  he  brings  the  object  S,  will  be  as  much  elevated  above 
the  plane  of  the  true  horizon  HOQ  as  the  apparent  horizon  OF  will  be 
depressed  below  it ;  because,  when  two  strdght  lines  intersect  each  other,  the 
opposite  angles  will  be  equal.  (Euclid,  Book  I.,  Prop.  15.)  In  this  case  it  is 
evident  that  the  arc  or  apparent  altitude  S  D  is  too  little;  and  that  it  must 
be  augmented  by  the  arc  D  H  =  the  angle  of  horizontal  depression  FO  Q, 
in  order  to  obtain  the  true  altitude  S  H.  Hence  it  is  manifest  that  altitudes 
taken  l>y  the  fore  observation  must  be  diminished  by  the  angle  of  horizontal 
depression^  and  that  in  back  observations  the  altitudes  must  be  increased  by 
the  value  of  that  angle. 

The  absolute  value  of  the  horizontal  depression  may  be  established  in 
the  following  manner  :*-From  where  the  apparent  horizon  O  P  becomes  a 
tangent  to  the  earth's  surface  at  T  (the  point  of  contact  where  the  sky  and 
water  seem  to  meet)  let  a  straight  line  be  drawn  to  the  centre  E,  and  it 
will  be  perpendicular  to  OP  (Euclid,  Book  III.,  Prop.  18) :  hence  it  is 
obvious  that  the  triangle  E  TO  is  right-angled  at  T.  Now,  because  O  T  is 
a  straight  line  making  angles  from  the  point  O  upon  the  same  side  of  the 
straight  line  O  E,  the  two  angles  EOT  and  TO  H  are  together  equal  to  the 
angle  EO  H  (Euclid,  Book  I.,  Prop.  13) ;  but  the  angle  EO  H  is  a  right 
angle;  therefore  the-  angle  of  depression  TOH  is  the  complement  of  the 
angle  EOT,  or  what  the  latter  wants  of  being  a  right  angle :  but  the  angle 
T  E  O  is  also  the  complement  of  the  angle  E  OT  (Euclid,  Book  I.,  Prop.  32); 
therefore  the  angle  T  E  O  is  equal  to  the  angle  of  horizontal  depression ; 
for  magnitudes  which  coincide  with  one  another,  and  which  exactly  fill 
up  the  same  space,  are  equal  to  one  another*  Then,  in  the  right-angled 
rectilineal  triangle  ETO,  there  are  given  the  perpendicular  TE,  »=  the 
earth's  semidiameter,  and  the  hypothenuse  E  O,  =  the  sum  of  the  earth's 
semidiameter  and  the  height  of  the  observer's  eye,  to  find  the  angle  T  E  O 
s=  the  angle  of  horizontal  depression  TO  H : — hence  the  proportion  will  be, 
as  the  hypothenuse  EO  is  to  radius,  so  is  the  perpendicular  T E  to  the 
cosine  of  the  angle  T  E  O,  which  angle  has  been  demonstrated  to  be  equal 
to  the  angle  of  horizontal  depression  HOP.  But  because  very  small  arcs 
cannot  be  strictly  determined  by  cosines,  on  account  of  the  differences  being 
so  very  trivial  at  the  beginning  of  the  quadrant  as  to  run  several  seconds 
without  producing  any  sensible  alteration,  and  there  b^ing  no  rule  for  showing 


Digitized  by 


Google 


OSSCRIFTIONAND  USB  OF  THB  TABLS8.  5 

why  one  second  should  1>e  preferred  tp  another  in  a  choice  of  so  many,  the 
following  method  is  therefore  given  as  the  most  eligible  for  computing  the 
true  value  of  the  horizontal  depression,  and  which  is  deduced  from  the 
86th  Prop,  of  the  third  Book  of  Euclid. 

Because  the  apparent  horizon  OP  touches  the  earth's  surface  at  T,  the 
square  of  the  line  O  T  4s  equal  to  the  rectangle  contained  under  the  two 
lines  CO  and  6 O.  Now  as  the  earth's  diameter  is  known  to  be  41804400 
English  feet,  and  admitting  the  height  of  the  observer's  eye  eO  to  be  290 
feet  above  the  plane  of  the  horizon;  then,  by  the  proposition,  the  square 
rootof  CO,  41804690  x  eO,290=  the  line  OT,  110105.75  feet;  the 
distance  of  the  visible  horizon  from  the  eye  of  the  observer  independent 
of  terrestrial  refraction. 

Then,  in  the  right-angled  rectilineal  triangle  ET  O,  there  are  given  the 
perpendicular  ET  =  20902200  feet,  the  earth's  semidiameter,  and  the  base 
0T=  110105.75,  to  find  the  angle  TEO.    Hence, 

As  the  perpendicular  TB  ^  20902200  feet,  log.  arith.  compt.^  2. 679808 
Is  to  the  radius  .  .  .  .  90?0'0r  log.  sine  .  .  .10.000000 
SoisthebaseOTs  .    •  110105.75  feet,  logi 5:041810 


TotheangleTEO=s      .     18f7^  =  log.  tang.  .    .    .    7,721618 

But  it  has  been  shown  that  the  angle  TEO,  thus  found,  is  equal  to  the 
angle  HOP;  therefore  the  true  value  of  the  angle  of  horizontal  depression 
HOP,  is  18 '7^  Now,  according  to  Dr. Maskelyne,  the  horizontal  de- 
pression is  affected  by  terrestrial  refraction,  in  the  proportion  of  about  one- 
tenth  of  the  whole  angle ;  wherefore,  if  from  the ,  angle  of  horizontal 
depression  18^7^  we  take  away  the  one- tenth,  viz.  1C49'',  the  allowance 
for  terrestrial  refraction,  there  will  remain  16 ^8^  for  the  true  horizontal 
depression,  answering  to  290  feet  above  the  level  of  the  sea.  The  prin- 
ciples being  thus  clearly  established,  it  is  easy  to  deduce  many  simple  for- 
mufae  therefrom,  for  the  more  ready  computation  of  the  horizontal  de- 
pression ;  of  which  the  following  will  serve  as  an  example. 

Tq  the  proportional  log.  of  the  height  of  the  eye  in  feet,  (estimated  as 
seconds,)  add  the  constant  log.  .4236,  and  half  the  sum  will  be  the  propor- 
tional log.  of  an  arc ;  which  being  diminished  by  one-tenth,  for  terrestrial 
refraction,  will  leave  the  true  angle  of  horizontal  depression. 

Example. 

Let  the  height  of  the  eye  above  the  level  of  the  sea  be  290  feet,  required 
tlie.depresaion  of  the  hori^n  conesponding  thereto  ^ 


Digitized  by 


Google 


6  DEiCRIFTlOH  AKD  USB  OF  THB  TABLS8. 

Height  of  the  eye  !290  feet,  esteemed; as  Bec8.s=4:50r,  proporJog.ssl.  5710 
Constaatlog. .    •    .    .        .j4236 

Sum=5  1.9946 

Arc= 187^    Proportional  log,    .9973 

Deduct  one-tenth     =5     1.49,  for  terrestrial  refraction. 


True  horizontal  depression   16 '  18^,  the  same  as  by  the  direct 
method. 

In  using  ihe  Table,  it  may  hot  be  unnecessary  to  remark  that  it  is  to  be 
entered  with  the  height  of  the  eye  above  the  level  of  the  eea,  in  the  column 
marked  Height,  tfc. ;  opposite  to  which,  in  the  following  column,  stands 
the  corresponding  correction ;  which  is  to  be  subtracted  from  the  observed 
altitude  of  a  celestial  object  when  taken  by  the  fore  observation ;  but  to  be 
added  thereto  when  the  back  observation  is  used,  as  before  stated.  Thus 
the  dip,  answering  to  20feet  above  the  level  of  the  sea,  is  4'  17^ 


Tabu  III« 
Dip  of  the  Hariztm  at  different  Dittances  from  the  Obeerver. 

If  a  ship  be  nearer  to  the  land  than  to  the  visible  horizon  when  uncon* 
fined,  uiid  that  an  observer  on  board  brings  the  image  of  a  celestial  object 
in  contact  with  the  line  of  separation  betwixt  the  sea  and  land,  the  dip  of 
the  horizon  Mali  then  be  considerably  greater  than  that  given  in  the  preced- 
ing Table,  and  will  increase  as  the  distance  of  the  ship  from  the  land 
diminishes :  in  this  case  the  ship's  distance  from  the  land  is  to  be  estimated, 
with  which  and  the  height  of  tiie  eye  above  the  level  of  the  sea,  the  angle 
of  depression  is  to  be  taken  from  the  present  TaUe*  Thus,  let  the  distance 
of  a  ship  from  the  land  be  1  mile,  and  the  height  of  thie  eye  above  the 
sea  30  feet;  with  these  elements  entear  the  TaUe,  and  in  the  angle  of 
meeting  under  the  latter  and  opposite  to  the  former  will  be  found  17 - 
which,  therefore,  is  the  correction  to  be  applied  by  subtraction  to  (he 
observed  altitude  of  a  celestial  object  when  the  fore  observation  is  used, 
aud  vice  versa. 

The  corrections  in  this  Table  were  computed  after  the  following  manner  } 
viz.,— 

Let  the  estimated  distance  of  the  ship  from  the  land  represent  the  base 
of  a  right-angled  triangle,  and  the  height  of  the  eye  above  the  level  of 
the  sea  its  perpendicular;  then  the  dip  of  the  horizon  will  be  expeessed 


Digitized  by 


Google 


AUCftfPTIOll  AND  tf 8fi  OF  THll  TABUM.  7 

by  the  measure  of  tke  angle  opposite  to  the  perpendicular :  hence^  since  the 
base  and  perpendicular  of  that  triangle  are  known>  we  have  the  following 
general 

/Zu/e.— As  the  base  or  ship^s  distance  from  the  land,  is  to  the  radius,  so 
is  the  perpendicular,  or  height  of  the  eye  above  the  level  oiP  the  sea  to  the 
tangent  of  its  opposite  angle,  which  being  diminished  by  one-tenth,  on 
account  of  terrestrial  refiraetion,  will  leave  the  correct  horizontal  dip,  as  in 
the  subjoined  example. 

Let  the  distance  of  a  ship  from  the  land  be  1  mile,  and  the  height  of 
the  eye  above  the  level  of  the  sea  25  feet,  required  the  corresponding 
horizontal  dip 

As  distance  1  mile,  or  5280  feet,  Logarithm  Ar.  Comp.ss  6. 277^66 
Is  to  radius  •  .  .  90?,  Logarithmic  Sine  .  .  10.600000 
So  is  height  of  the  eye      25  feet.  Logarithm    ....     L  397940 


To  Angle 16'.  17^=Log. Tang.  =  7.676306 

Deduct  one-tenth  for  terrestrial 

refraction     ««••••        L37 


True  borisootal  dip »    f    •    •       14'40r,orl5:nearlyasintheTable« 

iS^arlc-^AlQiough  a  skilful  mariner  can  always  estimate  the. distance 
of  a  ship  from  the  shore  horizon  to  a  sufficient  degree  of  accuracy  for 
taking  ouMhe  horizontal  dip  from  the  Table,  yet  since  some  may  be  de- 
sirous of  oDtaining  the  value  of  that  dip  independently  of  the  ship's  dis- 
tance  from  the  land,  and  consequently  of  the  Table^  the  following  rale  is 
ghren  for  their  guidance  in  such  cases  :— 

Let  two  observers^  the  one  being  as  near  the  mast  head  as  possible,  and 
the  other  on  deck  immediately  under,  take  the  sun's  altitude  at  the  same 
instant.  Then  to  the  arithmetical  complement  of  the  logarithm  of  the 
difference  of  the  *  heights,  add  the  logarithm  of  their  sum,  and  the  loga- 
rithmic sine  of  the  difference  of  the  observed  altitudes ;  the  sum,  rejecting 
10  from  the  index,  will  be  the  log.  sine  of  an  arch ;  half  the  sum  of  which 
and  the  difference  of  the  observed  altitudes  will  be  the  horizontal  dip  cor- 
responding  to  the  greatest  altitude,  and  half  their  difference  will  be  that 
contqponding  to  the  least  altitude^ 

Example. 

Admit  the  height  cyf  an  observer's  eye  at  the  main'-topmast  head  of  a  ship 
elose  in  with  the  land,  to  be  96  feet,  that  of  another  (immediately  under) 
on  dedt  24  feet;  the  altitude  of  the  sun's  lower  limb  found  by  the  forme? 
to  be  89^37  ^  and  by  the  latter^  taken  at  the  same  instant,  39?21 ' ;  required 
the  dip  of  the  shore  horizon  corresponding  to  each  attitude  1 


Digitized  by 


Google 


8  ]>B$CEIPTION  AND  USE  OF  THE  TABLES^ 

Height  of  mast  head  observer  96  feet. 
He^ht  of  deck  observer    .    24  do.     . 

Difference  of  heights  ,  •  72  do,^  Log.Ar.Comp.sS.  142667 
Sum  of  ditto  ....  120  do.  Logarithm  .  2.079181 
Difference  of  altitudes    •       16'         Log.  sine       •    7-  667845 

Arch  s  26U0?  Log.  siae  7. 889693 

Sum    =42'40r,|=2i:20^=diptothegreatestheight 
Diff.     =  10. 40, 1=  5. 20=dip  to  the  least  height. 

^0^6.— When  the  dip  answering  to  an  obstructed  horizon  is  thus  care- 
fully determined,  the  ship's  distance  from  the  land  may  be  ascertained  to 
the  greatest  degree  of  accuracy  by  the  following  rule :  viz.  As  the  Log. 
tangent  of  the  horizontal  dip  of  the  shore  horizon  is  to  the  logarithm  of 
the  height  of  the  eye  at  which  that  dip  was  determined,  so  is  radius  to  the 
true  distance. 

Thus,  in  the  above  example  where  the  horizontal  dip  has  been  deter- 
mined to  the  corresponding  height  of  the  eye  and  difference  of  altitudes. 

As  horizontal  dip  =  5  ^26f  Log.  tang.  ar.  compt.=:2.  809275 
Is  to  the  height  of  the  eye  24  feet^  Logarithm  •  .  •  1 .  3802 1 1 
So  is  radius.    ...     90?        Logarithmic  sine    .     10.000000 

To  true  distance     •    •     15469.8  feet    .    Logarithm=4.f69486 

The  same  result  will  be  obtuned  by  using  the  greatest  dip  and  its  cor- 
responding height ;  and  since  the  operation  is  so  yery  simple^  it  cannot 
fell  of  being  extremely  useful  in  determining  a  ship's  true  distance  from 
the  shore. 


Table  IV. 

jfugfiieniation  of  the  Moon's  Semidiameter. 

Since  it  is  the  pr(q)erty  of  an  object  to  increase  its  apparent  diameter  in 
proportion  to  the  rate  in  which  its  distance  from  the  eye  of  an  observer  is 
diminished ;  and,  since  the  moon  is  nearer  to  an  observer,  on  the  earthy 
when  she  is  in  the  zenith  than  when  in  the  horizon,  by  the  earth's  semi- 
diameter;  she  must,  therefore,  increase  her  semidiameter  by  a  certain, 
quantity  as  she  increases  her  altitude  from  the  horizon  to  the  zenith.  This 
increase  is  called  the  augmentation  of  the  ^loon^s  semidiameter^  and  d^« 
p^nds  upon  the  follovidng  pnn^^jplest 


Digitized  by 


Google 


1NI5CRIPTI0N  AND  USE  OF  THB  TABLB8» 


Let  the. circle  A  BCD 
represent  the  earth ;  A  E 
its  semidiameter^  and  M 
the  moon  in  the  hori- 
zon. Let  A  represent  the 
place  of  an  observer  on  the 
earth's  surface;  BDM 
his  rational  horizon^  and 
H  A  O,  drawn  parallel 
thereto^  his  sensible  hori- 
zon extended  to  the  moon's 
orbit ;  join  A  M^  then 
A  ME  is  the  angle  under 
which  the  earth's  semi- 
diameter  A  E  is  seen  from 
the  moon.  M,  which  is 
equtl  to  the  angle  M  A  O^  the  moon's  horizontal  parallax ;  because  the 
straight  line  A  M  which  falls  upon  the  two  parallel  straight  lines  E  M  and 
AO  makes  the  alternate  angles  equal  to  one  another. (Euclid,  BookL  Prop. 
29.)  Let  the  moon's  horizontal  parallax  be  assumed  at  57 '30^,  which  is 
about  the  parallax  she  has  at  her  mean  distance  from  the  earth  j  then  in  the 
right  angled  triangle  A  E  M,  there  are  given  the  angle  A  M  E=57'30^,  the 
moon's  horizontal  parallax,  and  tjie  side  AE=3958.  75  miles,  the  earth's 
semidiameter ;  to  find  the  hypothenuse  AM=the  moon's  distance  from 
the  observer  at  A :  hence  by  trigonometry, 

As  the  angle  at  the  moon,  A  M  E=a57'30r  Log.  sine  ar.  comp.  1. 776626 
Is  to  the  earth's  semidiameter=A  E=3958.  75  miles.  Log.  .  3. 597558 
So  is  radius     .......      90?     .     .    .    Log.  sine  lO.OOOOOO 

To  moon's  horizontal  distance  A  M=236692.35  miles,  Log.  .    5. 374 184 

Now,  because  the  moon  is  nearer  to  the  observer  at  A,  by  a  complete 
semidiameter  of  the  earth  when  in  the  zenith  Z,  than  she  is  when  in  the 
horizon  M,  as  appears  very  evident  by  the  projection ;  and,  because  the 
earth's  semidiameter  A  E  thus  bears  a  sensible  ratio  to  the  moon's  distance ; 
it  hence  follows  that  the  moon's  semidiameter  will  be  apparently  increased 
when  in  the  zenith,  by  a  small  quantity  called  its  augmentation;  and 
which  may  be  very  clearly  illustrated  as  follows,  viz. 

Let  the  arc  Z  O  M  represent  a  quarter  of  the  moon's  orbit ;  Z  her  place 
in  the  zenith^  and  Z  S  her  semidiameter  :  join  E  Z,  A  S,  and  E  S ;  then  the 
angles  Z  E  S  and  Z  A  S  will  represent  the  angles  under  which  the  moon's 
semidiameter  is  seen  from  the  centre  and  surface  of  the  earth;  their  diffe* 


Digitized  by 


Google 


10  MSCEfPTtOK  AND  USB  OT  TRB  TABLB8. 

rence^  viz.,  the  angle  A  S  E  is,  therefore,  the  augmentation  of  the  moon's 
semidiameter,  which  may  be  easily  computed ;  thus---» 

In  the  oblique  angled  triangle  A  S  E,  there  are  given  the  side  A  E 
=3958. 75  miles/ the  earth's  semidiameter ;  the  side  A  S,r=A  M  — *  AEsa 
23273.3. 6  miles,  the  moon's  distance  when  in  the  zenith  from  the  observer 
at  A ;  and  the  angle  AE.S=15'  SCT,  the  moon's  mean  semidiameter;  to 
find  the  angle  ABE=:the  greatest  augmentation  corresponding  to  the 
given  horizontal  parallax  and  horizontal  semidiameter :  therefore, 

As  moon's  zenith  distance  =  AZ=232733. 6  miles.  Log.  ar.  co.  4. 633141 
Is  to  moon's  semidiameter     A  E  S  =  15 '  30r  Log.  sine     7-  654056 

So  is  earth's  semidiameter     E  A  =  3958. 75  miles.  Log.  •    .    3. 597558 

To  augment,  of  semidian^      ASE=:0'16r  Log.  sine     5.884755 

Now,  having  thus  found  the  augmentation  of  the  Moon-'s  semidia- 
meter, when  in  the  zenith,  answering  to  the  assumed  horizontal  parallax 
and  horizontal  semidiameter ;  the  increase  of  semidiameter  at  any  given 
altitude,  from  the  horizon  to  the  zenith,  may  be  computed  in  the  following 
manner. 

Let  S  A  be  produced  to  F.  and  draw  E  P  parallel  to  Z  S ;  then  will  E  P 
represent  the  greatest  augmentation  to  the  radius  E  Z.  Let  the  moon 
be  in  any  other  part  of  her  orbit,  as  at })  with  an  altitude  of  45  degrees  ; 
joinDE,  and])F,  and  makeDG=])E;  then  will  EG  (the  pleasure  of 
the  angle  E]>G  to  the  radius  E}) ,)  be  the  augmentation  corresponding  to 
the  given  altitude.  Then,  in  the  right  angled  triangle  BGP,  right 
angled  at  G,  there  are  given  the  angle  EFG=43  degrees,  the  moon*s 
apparent  altitude,  and  the  side  E  F=?16  seconds,  the  augmentation  of 
semidiameter  when  in  the  zenith ;  to  find  the  side  E  G,  which  expresses 
the  augmentation  of  semidiameter  at  the  given  altitude.  And,  since  the 
angles  expressing  the  augmentations  are  so  very  small,  the  measure  of  each 
may  be  substituted  for  its  sine,  vehich  will  simplify  the  calculation]  iho». 

As  radius •     90?0'0'' Log.  sine  ar.  comp.    0.000000 

Is  to  moon's  greatest  augment,  of  semidiam»=sE  F  16^,  Log.  as  1. 204120 
So  is  moon's  given  apparent  alt.  =5  ^  E  FG,  45?     Log;  sine  a:  9. 849485 

To  the  augmentation,  or  side         .    E  G  =  1 P p  3 1 .     Log.  s  1 .  053605 

iffaicby  therefixre^  is  the  augmentation  of  the  moon's  semidiameter  cor-* 
responding  to  the  given  apparent  altitude  of  45  degrees ;  horizontal  semi* 
diameter  15 ^SOl!  and  horizontal  parallax  57 '  30? 

Explanation  of  the  Table. 

His  Table  contains  the  augmentation  of  the  moon's  semidiameter  (de- 
termined after  the  above  manner,)  to  every  third  degree  of  altitude :  the 


Digitized  by 


Google 


DXSC&IPTION  AND  ITSS  OF  TUB  TABLBB,  11 

augmentatioti  is  expressed  in  seconds,  and  is  to  be  taken  out  by  entering 
the  TaUe  mth  the  moon's  horizontal  scmidiameter  at  the  top,  as  given  in 
the  Nautical  Almanac,  and  the  apparent  altitude  in  the  left-hand  column  ; 
in  the  angle  of  meeting  will  be  found  a  correction,  which  being  applied 
by  addition  to  the  moon's  horizontid  scmidiameter  will  give  t)}e  true  semi-« 
diameter,  corresponding  to  the  ^ven  altitude.  Thus  the  augmentation 
answering  to  moon's  apparent  altitude  30  degrees,  and  horizontal  semi- 
diauiete^  16C30?  is  0  seconds;  and  that  corresponding  to  altitude  60? 
and  aemidiameter  16*  is  14  seconds. 


Tablb  V. 

Contraction  of  the  semidiameters  oftlie  Sun  and  Moon^ 

Since  all  parts  of  the  horizontal  semidiameter  of  the  sun  or  moon  are 
equally  elevated  above  the  horizon,  all  those  parts  must  be  equally  affected 
by  refraction,  and  thereby  cause  the  horizontal  semidiameter  to  remain 
invariable.  Bu^when  the  semidiameter  is  inclined  to  the  plane  of  the 
horixon^  the  lower  extremity  will  be  sO  much  more  affected  by  refraction 
than  the  upper,  as  to  suffer  a  sensible  contraction,  and  *thus  cause  the 
aemidiameter,  so  inclined,  to  be  something  less  than' the  horizontal  semi* 
diameter  given  in  the  Nautical  Almanac,  Hence  it  is  manifest  that  the 
aemidiameter  of  a  celestial  object,  measured  in  any  other  manner  than 
that  parallel  to  the  plane  of  the  horizon  will  be  always  less  than  the  true 
aemidiameter  by  a  certain  quantity  : — ^this  quantity,  called  the  contraction 
of  semidiameter  is  contained  in  the  present  Table  i  the  arguments  of 
which  are,  the  apparent  altitude  of  the  object  in  the  left-hand  column,  and 
at  the  top  the  angle  comprehended  between  the  measured  diameter  and 
Aat  parallel  to  the  plane  of  the  horizon ;  iii  the  angle  of  meeting  will  be 
foBod  a  correction,  which  being  subtracted  from  the  horizontal  semi* 
diameter  in  the  Nauticar  Almanac,  will  leave  the  true  semi-'diameter. 
Thus,  let  the  sun's  or  moon's  apparent  altitude  be  5  degrees,  and  the 
inclini^on  of  its  semidiameter  72  degrees ;  now,  in  the  angle  of  meeting, 
of  these  arguments,  stands  23  seconds  •  which,  therefore,  is  the  contraction 
of  aemidiameter,  and  which  is  to  be  applied  by  siiblraetioti  to  the  semi- 
diaftictef  given  in  the  Nautical  Almanac. 

To  compute  the  contraction  of  Semidiameter. 

Bule. — Find  by  Table  VIII.  the  refraction  corresponding  to  the  object's 
apparent- central  altitude,  and  also  the  refraction  answering  to  that  altitude 
augmented  by  the  semidiameter}  (whichi  for  this  purpose,  may  be  estiamted 


Digitized  by 


Google 


12  DBSCRIPTION  AND  USB  OF  THB  TABLB8. 

at  16  minutes,)  and  their  difference  will  be  the  contraction  of  the  vertical 
semidiaroeter.  Now,  having  thus  found  the  contraction  corresponding  to 
the  vertical  semidiameter,  that  answering  to  a  semidiameter  which  forms 
any  given  angle  with  the  plane  of  the  horizon^  will  be  found  by  multiply- 
ing the  vertical  contraction  by  the  square  of  the  angle  of  inclination. 

Example. 

Let  the  sun's  or  moon's  apparent  central  altitude  be  3?  and  the  indi- 
nation  of  its  semidiameter  to  the  plane  of  the  horizon  72?;  reqjiired  the 
contraction  of  the  semidiameter  ? 

Apparent  central  altitude    .  3?  0'  Refractions  14.' 36f 
Do.  augmented  by  semidiam.  =  3  ?  1 6 '  Ditto  •  =  13 .  46. 

Contraction  of  the  vertical  semidiameter  .  .  .  0 '  50rLog.= 1 .  698970 
Inclination  of  semidiameter    ,  =72?  twice  the  log.  sine     •  =19.956412 

Required  contraction  of  semidiameter  •    .     .     .45'.  22  Log.=l.  655382 

And  so.  on  of  the  rest.— It  is  to  be  remarked,  however,  that  the  correc- 
tion arising  from  the  contraction  of  the  semidianieter.of  »  celestial  object 
is  very  seldom  attended  to  in  practice  at  sea. 


Table  VL 

Parallax  of  the  Planets  %7i  AUitude. 

The  arguments  of  this  Table  are  the  apparent  altitude  of  a  planet  in 
the  left  or  right-hand  margin,  and  its  horizontal  parallax  at  the  top  ; 
under  the  latter,  and  opposite  the  former,  stands  the  corresponding  parallax 
in  altitude ;  which  is  always  to  be  applied  by  addition  to  the  planets  ap- 
parent altitude.  Hence,  if  the  apparent  altitude  of  a  planet  be  30  degrees, 
and  its  horizontal  parallax  27  seconds,  the  corresponding  parallax  in 
altitude  will  be  23  seconds;  additive  to  the  apparent  altitude. 

Tlie  parallaxei  ofJUUude  m  this  Table  were  computed  by  the  follomng 

J&ifc.— To  the  proportional  logarithm  of  the  planet's  horizontal  parallax 
add  the  log.  secant  of  its  apparent  altitude,  and  the  sum,  abating  10  in 
the  index,  will  be  the  proportional  logarithm  of  the  parallax  in  altitude. 

Example. 

If  the  horizontal  parallax  of  a  planet  be  23  seconds,  and  itsapparei^t 
i^ltitude  30  de^ees }  required  the  parallax  in  altitude  ? . 


Digitized  by 


Google 


BBSCftlPTION  AND  USE  OF  TMB  TABLES.  13 

HoricoBtal  parallax  of  the  planet=23  Seconds^  proportional  log.=  2. 6717 
Apparent  altitude  of  ditto    .    .      30  Degrees^  log.  secant    .    .10. 0625 


Parallax  in  altitude    .    .    •    «      20  Seconds^  proportional  log.    2. 7342 


Table  VII. 

Parallax  of  the  Sun  in  Altitude* 

lie  difference  between  the  places  of  the  sun^  as  seen  from  the  surface 
and  centre  of  the  earth  at  the  same  instant,  is  called  his  parallax  in  al- 
ticnde^  which  may  be  computed  in  the  following  manner. 

To  the  log.  cosine  of  the  sun's  apparent  altitude,  add  the  constant  log. 
0.945124,  (the  log.  of  the  sun's  mean  horizontal  parallax  estimated  at 
8'.  813,)  and  the  sum,  rejecting  10  from  the  index,  will  be  the  log.  of 
the  parallax  in  altitude;  as  thus, 

Given  the  sun's  apparent  altitude  20  degrees ;  required  the  correspond- 
ing parallax  in  altitude  ? 

Sun's  apparent  altitude  20  degrees,  log.  cosine    ;     .    9. 9729S6 
Constant  log 0.945124 

Parall.  corresponding  to  the  given  altitude  S"".  282  Log.  0. 9181 10 

This  Table,  which  contains  the  correction  fo^  parallax,  is  to  be  entered 
with  the  sun*8  apparent  altitude  in  the  left-hand  column ;  opposite  to  which, 
in  the  adjoining  column,  stands  the  corresponding  parallax  in  altitude  ;— 
thus,  the  parallax  answering  to  10?  apparent  altitude  is  9  seconds ;  that 
answering  to  40?  apparent  altitude  is  7  seconds,  &c.  &c. — And  since  the 
parallax  of  a  celestial  object  causes  it  to  appear  something  lower  in  the 
heavens,  than  it  really  is ;  this  correction  for  parallax,  therefore,  becomes 
always  additive  to  the  sun's  apparent  altitude. 


Table  VIII. 

Mean  Astronomical  Reaction. 

Since  the  density  of  the  atmosphere  increases  in  proportion  to  its  prox- 
imity to  the  'earth's  surface,  it  therefore  causes  the  ray  of  light  issuing  from 
a  celestial  object  to  describe  a  curve,  in  its  passage  to  the  horizon ;  the 
convex  side  of  which  is  directed  to  that  part  of  the  heavens  to  which  a 
tangent  to  that  curve  at  the  extremity  of  it  which  meets  the  earth,  would 


Digitized  by 


Google 


14  PBSCEIPTIOK  AND  D«B  OF  THB  TABUS. 

be  directed.  Hence  it  is,  that  the  celesUal  objects  are  apparently  more 
elevated  in  the  heavens  than  they  are  in  reality ;  and  this  apparent  increase 
of  elevation  or  altitude  is  called  the  refraction  of  the  heavenly  bodies ;  the 
effects  of  which  are  greatest  at  the  horizon,  but  gradually  diminish  aa 
the  altitude  increases,  so  as  to  entirely  vani&h  at  the  zenith. 

In  this  Table  the  refraction  is  -computed  to  every  minute  in  the  first 
8  degrees  of  apparent  altitude ;  consequently  this  part  of  the  Table  is  to 
be  entered  with  the  degreea  of  apparent  altitude  at  the  top  or  bottom, 
and  the  minutes  in  the  left-hand  coluixm :  in  the  angle  of  meeting,  stands 
the  refraction. 

In  the  rest  of  the  Table  the  apparent  altitude  is  given  in  the  vertical 
columns,  opposite  to  which  in  the  adjoining  columns  will  be  found  the 
corresponding  refraction.  Thus,  the  refraction  answering  to3?27«  appa- 
rent altitude,  is  13'14f;  tiiat  corresponding  to  9?46'  is  5'52r;  that 
corresponding  to  17^55  'is  2'54?,  and  so  on.  The  refraction  is  always  to 
be  applied  by  subtraction  to  the  apparent  altitude  of  a  celestial  object,  on 
account  of  its  causing  such  object  to  appear  under  too  great  an  angle  of 
altitude.  The  refractions  in  this  Table  are  adapted  to  a  medium  state  of 
the  atmosphere ;  that  is,  when  the  Barometer  stands  at  29. 6  inches,  and 
the  Thermometer  at  50  degrees ;  and  were  computed  by  the  following  ge^ 
neral  rule,  the  horizontal  refraction  being  assumed  at  33  minutes  of  a 
degree. 

To  the  constant  log.  9. 999279  (the  log.  cosine  of  6  times  the  horizontal 
refraction)  add  the  log.  cosine  of  the  apparent  altitude ;  and  the  sum, 
abating  10  in  the  index,  will  be  the  log.  cosine  of  an  arch.  Now,  one- 
sixth  the  difference  between  this  arch  and  the  given  apparent  altitude  will 
be  the  mean  astronomical  refraction  answering  to  that  altitude. 

Example. 

Let  the  apparent  altitude  of  a  celestial  object  be  45?,  required  the  cor- 
responding refraction  ? 

Constant  log 9. 999279 

Given  apparent  altitude   45?0'0T  Log.  cosine   9.849485 

Arch 45?5U2r  Log.  cosine    9.848764 

Difference 0^5 U2r  ^  6  =  0^57^  ;  which, therefore, 

is  the  mean  astronomical  refraction  answering  to  the  given  apparent  alti- 
tude. 


Digitized  by 


Google 


DBSCEIPTION  AND  T78B  OP  THB  TABLB8«  15 

Tabib  IX. 
Corrictknofihe  Mean  J$tr(m(nnical  Refract 

Since  the  refraction  of  the  heavenly- bodies  depends  on  the  density  and 
temperature  of  the  atmosphere,  which  are  ever  subject  to  numberless  varia- 
tions ;  and  since  the  corrections  contained  in  the  foregoing  Table  are 
adapted  to  a  medium  state  of  the  atmosphere,  or  when  the  barometer  stands 
at  29. 6  inches,  and  the  thermometer  at  50  degrees :  it  hence  follows,  that 
when  the  density  and  temperature  of  the  atmosphere  differ  from  those 
quantities,  the  amount  of  refraction  will  also  differ,  in  some  measure,  from 
thut  <»ntained  in  the  said  foregoing  Table.  To  reduce,  therefore,  the 
corrections  in  that  Table  to  other  states  of  the  atmosphere,  the  present 
Table  has  been  computed ;  the  arguments  of  which  are,  the  apparent  ald^ 
tude  in  the  left  or  right  hand  margin,  the  height  of  the  thermometer  at  the 
top,  and  that  of  the  barometer  at  the  bottom  of  the  Table;  the  correspond- 
ing corrections  will  be  found  in  the  angle  of  meeting  of  those  arguments 
respectively,  and  are  to  be  applied,  agreeably  to  their  signs,  to  the  mean 
refraction  taken  from  Table  VIII,  in  the  following  manner  :— 

Let  the  apparent  altitude  of  a  celestial  object  be  5  degrees;  the  height 
of  the  barometer  29. 15  inches,  and  that  of  the  thermometer  48  degrees; 
required  the  true  atmospheric  refraetion  I 

Apparent  altitude  5  degrees, — mean  refraction  in  Table  VIII  =  .  .  9'54T 
Opposite  to  5  degrees,  and  over  29. 15,  in  Table  IX,  stands  •  .  —  0.   9 
Opposite  to  5  degrees,  and  under  48  degrees,  in  ditto .    .     •    •  +  0.   3 


True  atmospheric  refraction,  as  required 9 '.48? 

The  correction  of  the  meitn  astronomical  refraction,  may  be  computed  by 
the  following  rule,  viz. 

As  the  n^an  height  of  the  barometer,  29.6  inches,  is  to  its  observed 
hright,  so  is  the  mean  refraction  to  the  corrected  refraction ;  now,  the 
diflereuce  between  this  and  the  mean  refraction  will  be  the  correction  for 
barometer,  which  will  be  afEurmatlve  oc  negative,  aoe<mling  as  it  is  greater 
or  less  than  the  latter.— And, 

As  350  degrees*  increased  by  t|ie  observed  height  of  Fahrenheit's  ther- 
mometer, are  to  400  degrees  f,  so  is  the  mean  refraction  to  the  corrected 
refraction  ;  the  difference  between  which,  and  the  mean  refraction,  will  be 
the  correction  for  thermometer;  which  will  be  affirmative  or  negative,  ac- 
cording as  it  is  greater  or  less  than  the  latter. 

*  Seven  times  50  degrees,  the  mean  temperature  of  the  atmosphere. 
t  Eifht  timet  J^Odegrcss,  the  mean  temperature  of  the  atmosphere. 


Digitized  by 


Google 


16  DB8CRIPTI0K  AND  USB  OF  THS  TABLBS. 


Bsample  I. 

Let  the  apparent  altitude  be  1  degree^  the  mean  refraction  24^29^,  the 
height  of  the  barometer  28. 56  inches,  and  that  of  the  thermometer 
32  degrees;  required  the  respective  corrections  for  barometer  and  ther- 
mometer ? 

As  mean  height  of  barometer  •     •  29. 60.    Log.  an  co.  •    •    8. 528708 

Is  to  observed  height  of  ditto  •    •28.56.     Log 1.455758 

Soismean  refraction  24 ;29r=    .   )469r    Log 3.167022 

To  corrected  refraction  •     .    •    •    1417^    Log 3.151488 

Correction  for  barometer    ...     •     —  52^,  which  is  negative^  because  the 
corrected  refraction  is  the  least. 

And 
As  350?+ 32?=.    .....    382?     Log.  ar.  co.    .    .    7.417937 

Isto  ...:......    4()0?     Log 2.602060 

So  18  mean  refraction  24:29^'=  .    1469r     Log 3.167022 

To  corrected  refraction     .     .    .     1538r     Log 3.187019 

Correction  for  thermometer     .    .     +69^=1 '9?^  which  is   affirmative, 
because  the  corrected  refracUon  is  the  greatest. 

Example  2. 

Let  the  apparent  altitude  be  7  degrees,  the  mean  refraction  7-201"^  the 
height  of  the  barometer  29.  75  inches,  and  that  of  the  thermometer  72  de- 
grees ;  required  the  respective  corrections  for  barjometer  and  thermometer  ? 

As  mean  height  of  barometer  .  .  29. 60.  Log.  ar.  co.  .  .  8.628708 
Is  to  observed  height  of  ditto  ..  29.75.  Log.  .  .  .  .  1.473487 
Soismeanrefraction7'20^  =  .     .  440f    Log 2.643453 

To  corrected  refraction     .    •    .    .  442f    Log 2. 645648 

Correction  for  barometer  .    •    .    •  +'  2^,  which  is  affirmative. 

And 

As  350?  +  72?= 422?     Log.  ar.  co.  .    .    7.374688 

Is  to 400?     Log 2. 602060 

So  is  mean  refraction  7'20r  =  .    .  440'/     Log 2. 643453 

- 

To  corrected  refraction    .    .    .    .417^    Log 2.620201 

Correction  for  thermometer  .    •    •  —  231",  which  is  negative. 


Digitized  by 


Google 


DBSCRIPTION  AND  V8E  OF  THB  TABLKS*  17 

Table  X. 
To  find  the  LaHiude  by  an  JUitude  of  the  North  Polar  Star. 

^  The  correction  of  altitude,  contained  in  the  third  column  of  this  Table^ 
expresses  the  difference  of  altitude  between  the  north  polar  star,  and  the 
north  celestial  pole,  in  its  apparent  revolution  -round  its  orbit,  as  seen  from 
the  equator :  the  correction  of  altitude  is  particularly  adapted  to  the  be- 
ginning of  the  yesf  1824  *,  but  by.  meana  of  its  annual  variation,  which  is 
determined  for  the  sake  of  accuracy  to  the  hundredth  part  of  a  second, 
it  may  be  readily  reduced  to  any  subsequent  period,  (with  a  sufficient  de- 
gree of  exactness  for  all  nautical  purposes,)  for  upwards  of  half  a  century, 
as  will  be  seen  presently. 

The  Table  consists  of  five  compartments ;  the  left  and  right  hand  ones 
of  which,  are  each  divided  into  two  columns  containing  the  right  ascension 
of  the  meridian:  the  second  compartment,  which  forms  the  third  column 
in  the  Table,  contains  the  correction  of  the  polar  star's  altitude  :  the  third 
compartment  consists  of  five  small  columns,  in  which  are  contained  the 
proportional  parts  corresponding  to  the  intermediate  minutes  of  right 
ascension  of  the  meridian;  by  means  of  which  the  correction  of  aldtude,  at 
any  given  time,  may  be  accurately  taken  out  at  the  first  sight :  the  fourth 
compartment  contains  the  annual  variation  of  the  polar  star's  correction, 
which  enables  the  mariner  to  reduce  the  tabular  correction  of  altitude  to 
any  future  period  :  for,  the  product  of  the  annual  variation,  by  the  number 
of  years  and  parts  of  a  year  elapsed  between  the  beginning  of  1824,  and 
any  given  subsequent  time,  being  applied  to  the  correction  of  the  polar 
star's  altitude  by  addition  or  subtraction,  according  to  the  prefixed  sign^ 
mil  give  the  true  correction  at  such  subsequent  given  time* 

JSxample  I. 

Required  the  correction  of  the  polar  star's  altitude  in  January  1834,  th6 
right  ascension  of  the  meridian  being  6  hours  and  22  minutes  ? 

Correction  of  altitude  answering  to  6t20C,    is    •    •    •    •    ;    0?16;  9?, 
Proportional  part  to  2  minutes  of  right  ascension     «    «    •    •  0. 50 

G>rfection  of  polar  star's  altitude  in  January  1824s:  •    «    •      0. 15. 19 
Annual  variation  of  correction     i    •+  2^.90 
Number  of  years  after  1824     •    •    •    .    .     10 

Product ^    .    .     +29''.0=s    •    •    •     +      0.29 


Correction  of  the  polar  star's  altitude  in  Jan.  1834,  as  required    0?  15  M8?, 

c 

/Google 


Digitized  by ' 


18  DESCRIPTION  AND  USB  OV  THE  TABLES. 

*    Example  2. 

Required  the  eorrection  of  the  polar  star's  dtitude  in  January  1854, 
the  right  ascension  of  the  meridian  being  5  hours  and  13  minutes  i 

Correction  ofaltitude  answering  to  5 MOrifl 0944124? 

Proportional  part  to  3  minute  of  right  ascension  •    •    •    *  ^^*   ^ 

Correction  of  polar  star's  altitude  in  Jan,  1824      •    •    •    .     0. 43. 16 
Annual  variation  of  cortection  .     .     —3'^.  34 
Numberofyears  after  1824     ^    .  30 

Product -100".20=  .    .    :    -       1.40 


Correction  of  the  polar  star's  altitude  in  Jan.  1 854,  as  required,  0?4 1 1  d6f 
t^hich  diflfers  but  8  seconds  from  the  true  Result  by  spherical  trigonometry, 
as  will  be  shown  hereafter ;  and  which  evidently  demonstrates  that  the 
column  of  annual  variation  may  be  safely  employed  in  reducing  the  correc- 
tion of  altitude  to  any  future  period,  for  a  long  series  of  years,  since  the 
error  in  the  space  of  thirty  years  only  amounts  to  8  seconds  of  a  degree, 
which  becomes  insensible  in  determining  the  latitude  at  sea. 

The  corrections  of  altitude  contained  in  the  present  Table  were  com* 
ptited  iri  conformity,  with  the  following  principles  :— 

Since  to  an  observer  placed  at  the  equator,  the  poles  of  the  world 
will  appear  to  be  posited  in  the  horizon,  the  polar  star  will,  to  such 
observer,  apparentiy  revolve  round  the  north  celestial  pole  in  its  diurnal 
motion  round  its  orbit.  In  this' apparent  revolution  round  the  celestial 
pole,  the  star's  meridional  or  greatest  altitude  above  the  horizon  will  be 
always  equal  to  its  distance  from  that  pole ;  which  will  ever  take  place, 
when  the  right  ascension  of  the  meridian  is  equal  to  the  right  ascension  of 
the  star.  In  six  hours  ctfter  this,  the  star  will  be  seen  in  the  horizon,  west 
of  the  pole ;  in  six  hours  more  it  will  be  depressed  beneath  the  horizon  (on 
the  meridian  below  the  pole),  the  angle  of  depression  being  equal  to  its 
p<rfar  distance ;  in  six  hours  after,  it  will  be  seen  in  the  horison  east  of  the 
pole ;  and  in  $ix  hours  jnore,  it  will  be  seen  again  on  the  meridian  above  the 
pole  t  allowance  being  made,  in  each  case,  for  its  daily  acceleration. 

Now,  since  the  north  celestial  pole  represents  a  fixed  point  in  the  hea<* 
Tens,  and  that  the  star  apparently  moves  round  |t  tn  an  uniform  manner, 
making  determinable  angles  with  this  meridian  j  it  is,  therefore,  easy  to 
compute  what  altitude  the  star  will  have,  as  seen  from  the  equator,  in 
every  part  of  its  orbit ;  for,  in  this  computation^  we  have  a  spherical  trian- 
gle  to  work  in,  whose  three  sides  are  expressed  by  the  complement  of  the 
latitude,  the  complement  of  the  polar  starts  altitude,  and  the  complement 


Digitized  by 


Google 


DSSCHIPTIOIf  AND  tTSB  OF  THB  TABLES.  19 

of  its  declination  $  ip  which  there  are  given  two  sides  and  the  included 
angle  to  find  the  third  side;  via,,  the  star's  co-declination  or  polar  distance 
and  the  complement  of  the  latitude,  with  the  comprehended  angle,  eqpal 
to  the  star's  distance  from  the  meridian,  to  find  the  star's  co-altitude ;  the 
difference  between  which  and  90  degrees  will  be  the  correction  of  altitude, 
or  the  difference  of  altitude  between  the  polar  star  and  the  north  celestial 
pole,  as  seen  from  the  equator. 


In. January  1854,  the  mean  right  ascension  of  the  north  polar  star 
will  be  1!5T23!,  and  its  polar  distance  l?28^5''j  now,  admitting  the 
right  ascension  of  the  meridian  to  be  5*13T,  the  correction  of  thp  polar 
star's  altitude,  as  seen  frpm  the  equator,  is  required  ? 


Right  asc.  of  the  merid.  5*13"  Oi 
Right  asc.  of  the  pol.  star  1 .    5 .  23 


P.  star's  disu  from  mend.  4 1  7*37  ?  =  61  ?54  i  15? 

Half  dd.  do,  in  degrees    .    .    .    .  30.67  •  7i  Twice  (he 

log.sinesl9.4S2469 
Star's  polar  distance  ^  •  ,  .  .  1 .  28 .  5  Log.  sine  8*  408572 
Complement  of  the  latitude    ,    .    ,    90.   0.  0   Log*  sine  10.000000 

I    ■  Sum    37.881041 


DMT.  between  polar  dist.  and  eo-lat.      88?3l  ^55?    Half  sum  18. 915520} 

Half  do ,    .    .    .    44?  16'.  574^    Log.  sine 0.843849 

Arph«  6?48:35KW,»ngt9t 071671* 
Log,  sine  of  this  arch    ,    ^    •  9. 068670 

Half  the  polar  star's  co-altitude    ,    ,44?39:i6r    Log.  sine  9.846850* 

Pokr  star's  co-altitude 89?16^32C' 

will       I   ■     ■■  >■    l» 

Cor.  of  polar  star's  alt.  in  Jan.  1 854  =»  0?4 1  ^  28^  Now,  by  comparing 
this  result  with  that  shown  in  Example  2  (page  18),  it  will  be  seen 
that  the  correction  of  altitude,  deduced  directly  from  the  Table,  may  be 
reduced  to  any  period  subsequent  to  1624,  without  its  being  affected  by 
any  error  of  sufficient  magnitude  tp  e^cjaoger  the  interest  of  the  mariner 
in  any  respect  whatever. 

c2 

Google 


Digitized  by 


20  DSSCUIPnON   AND   USE  OF  THB  TABIDS. 

ffotej^Vor  further  information  on  this  subject,  the  reader  is  referred  to 
the  author's  Treatise  on  the  Sidereal  and  Planetary  Parts  of  Nautical 
Astronomy,  page  144  to  156. 


Tablb  XL 
Correction  of  (he  Latitude  deduced  froth  tlie  preceding  Table. 

Although  the  latitude  deduced  from  Table  X.  will  be  always  sufficiently 
correct  for  most  naufacal  purposeSi,  yet,  «ince  observation  has  shown  that  it 
will  be  something  less  than  the  truth  in  places  distant  from  the  equa- 
tor, the  present  Table  has  been  computed ;  which  contains  the  number 
of  minutes  and  seconds  that  the  latitude,  so  deduced,  will  be  less  than  what 
would  result  from  actual  observation  at  fevery  tenth  or  fifth  degree  from 
the  equatoY,  to  within  five  degrees  of  the  north  pole  of  the  world. 

The  elements  of  this  Table  are,  the  approximate  latitude,  deduced  from 
Table  X.,  at  top,  and  the  right  ascension  of  the  meridian  in  the  left  or 
right-hand  column ;  in  the  angle  of  meeting  will  be  found  the  correspond- 
ing correction,  which  is  always  to  be  applied  by  addition  to  the  approx- 
imate latitude.  Hence,  if  the  approximate  latitude  be  50  degrees,  and 
the  right  ascension  of  the  meridian  GMQ?,  th^  corresponding  correction 
willbel^SSr  additive. 

Hernark. — Since  the  corrections  of  altitude  in  Table  X.  have  been  com- 
puted on  the  assumption  that  the  motions  oi  the  polar  star  were  witnessed 
from  the  equator,  they  ought,  therefore,  to  show  what  altitude  that  star 
ivill  have  at  any  given  time,  in  north  latitude,  when  applied  to  such  latitude 
with  a  contrary  sign  to  that  expressed  in  the  Table  ;  this,  however,  is  not 
the  case;  because  when  the  altitude  of  the  polar  star  is  computed  by  sphe- 
riciJ  trigonometry,  or  otherwise,  it  will  alw&ys  prove  to  be  something  less 
than  that  immediately  deduced  from  Table  X. :  it  is  this  difference,  then, 
thftt  becomes  the  correction  of  latitude  in  Table  XI.,  and  which  is  very 
•easily  determined,  as  may  be  seen  in  the  following 

Example. 

Let  the  right  ascension  of  the  meridian  in.  January  1824  be  6 MO?,  and 
the  latitude  60  degrees  north ;  required  the  true  altitude  of  the  polar  star, 
and  thence  the  correction  of  latitude  ?  . 

Latitude  or  elevation  of  the  pole    •    •    .     •  '  60?0^  OC  north. 
Correction  in  Table  X.,  answ.  to  6M0r,  is  +     0. 7. 41 

Altitude  of  polar  star,  per  Table  X.  =  .    ;    60?7'4K 

Digitized  by  VjOOQ IC 


BSSCJtJPTfON  ANJD  USB  OF  THB  TABLB|k  31 

Now,  to  compute  the  true  altitude  of  the  polar  star,  on  spherical  prin- 
ciples, at  the  given  time  and  place,  we  may  either  proceed  as  in  last 
example,  or,  more  readily,  as  follows : — 

Right  ascension  of  the  mend.  6t  40?  0! 
Star's  right  ascension    •    •    0.58.  1 


Star's  dist.  from  the  meridian  6 1 4 1  ?59 !  .  .  •  -Log.  .rising  5  •  96448 1 
Star's  polar  distance  .  .  l?37'48r  .  .  .  Log.  sine  8.454006 
Complement  of  the  latitude  30.   0.   0       •    .    •    Lfog.'sine    9.698970 


Difference  28 .  22  . 1 2  Nat  cos.  879897  ■ 

Natural  number      ....    013106  Log.=4. 117457 


Sur's  true  altitude    .     .    60?5n6r   Nat.  sine  866791 
SUr's  alt.  per  Tab.  as  above  60 . 7. 41 


Difference 0?2' 25'^;  which,  therefore,  is  the  correction 

of  latitude. 

JVote.— The  correction  of  latitude,  thus  found,  differs  4  seconds  from  that 
given  m  Table  XL :  this  difference  is  owing  to  the  star's  apparent  polar 
distance  having  been  -taken,  inadvertently,  from  .the  Nautical  Almanac  of 
1824,  instead  of  its  mean  polar  distance ;  but  since  this  can  only  lead  to  a 
trifling  difference,  onii  not  to  any  erroTy  it  wa^  not,  therefore,  deemed 
necessary  to  alter  or  recompute  the  Table. 


Tablb  XII. 

Mean  Right  Ascension  qfthe  Sun. 

This  Table  may  be  used  for  the  purpose  of  finding  the  approximate  time 
of  transit  of  a  fixed  star,  when  a  Nautical  Almanac  is  not  at  hand ;  it  may 
also  be  employed  in  finding  the  right  ascension  of  the  meridian,  or  mid- 
heaven,  when  the  latitude  is  to  be  determined  by  an  altitude  of  the  north 
polar  star :  for,  if  to  the  sun's  right  ascension,  as  given  in  this  Table,  the 
apparent  time  be  added,  the  sum.  (rejecting  24  hours  if  necessary)  will  be 
the  right  ascension  of  the  meridian,  sufficiently  near  the  truth  for  deter- 
lipning  the  latitud^^ 


Digitized  by 


Google 


88  9S8CltIPnO|f  ANB  USB  OF  TUB  TABLW* 

Table  XIII. 

Equations  id  equal  Altitudes. — first  part. 

The  arguments  of  this  Table  are^  the  interval  between  the  observations 
at  top  or  bottom,  and  the  latitude  in  either  of  the  side  columns ;  in  the 
angle  of  meeting  stands  the  corresponding  equatipn,  expressed  in  seconds 
and  thirds.:  hence  the  equation  to  interval  6  hours  40  minutes  and  latitude 
60  degrees,  is  15  seconds  and  33  thirds. 

The  equations  in  this  Table  were  computed  by  the  following  rule,  viz.  :— 
To  the  log.  cb-tangent  of  the  latitude,  add  the '  log.  sine  of  half  the 
interval  in  degrees ;  the  proportional  log.  of  the  whole  intierval  in '  time 
(esteemed  as  minutes  and  seconds),  and  the  constant  log.  8.  8239;*  the 
sum  of  these  four  logs.,  rejecting  29  from  the  index,  will  be  the  propor- 
tional log.  of  the  corresponding  equation  ii\  minutes  and  seconds,  which 
are  to  be  considered  as  seconds  and  thirds. 

Example. 

Let  the  latitude  be  50  degrees,  and  the  interval  between  the  observed 
equal  altitudes  of  the  sun  4  hours ;  required  the  corresponding  equation  ? 

•     Latitude 50?0C0^   Log.  co-tang.  9. 9238 

Half  int.  =  2  hours,  in  deg;8.=:  30. 0. 0  Log.  sine  .  9. 6990 
Whole  interval  4  hours,  esteemed  as  4  min.^  propor*  log.  1 .  6532 
Constant  log 8.8239 

Required  equation  *    .    .    .     U'^IS'^     Plropor.log.     1.0999 
The  equations  in  the  abovementioned  Table  were  computed  by  Mrs.  T. 

Kerigan. 

Table  XIV. 
Equations  to  equal  Jltitudes.^^VART  sbcond. 

In  this  Table,  the  interval  between  the  observations  is  m^irked  at  top  or 
bottom,  and  the  sun*s  declination  in  the  left  or  right-hand  margin ;  under 
or  over  the  former,  and  opposite  to  the  latter,  stands,  the  corresponding 
equation,  expressed  in  seconds  and  thirds :  thus,  the  equation  answering  to 
6  hours  40  minutes,  and  declination  18?30',  is  2  seconds  and  48  thirds. 
The  equations  contained  in  this  Table  were  computed  as  follows,  viz. :— • 
To  the  log.  co-tangent  of  the  declination,  add  the  log.  tang,  of  half  the 
interval  in  degrees ;  the  proportional  log.  of  the  whole  interval  in  time 
(esteemed  as  minutes  and  seconds),  and  the  constant  log.  8. 8239}t  the 


*  t  The  viUunetical  compleitLent  of  12  hours  considered  u  i 

/Google 


Digitized  by ' 


DB8C&IPTi<^  AND  USB  OF  THfi  TABU&  28 

sam  of  these  four  logs.,  rejecting  29  from  the  index^  will  be  the  propor- 
tional log.  of  the  corre^onding  equation  in  minutes  and  secondfly  which 
are  to  be  considered  as  seconds  and  thirds.    . 

Example. . 

Let  the  sun*s  declination  be  18?30',  and  the  interval  between  the 
obserred  equal  altitudes  of  the  sun  4  hours ;  required  the  corresponding 
equation  ? 

Sun's  declination  ....  1893G'  Log.  co-tang.  10. 4755 
Half  interval  =  2  ho.  in  degs.=30 .  0 .  Log.  tang.  .  9.  7614 
Whole  interval  4  ho.  esteemed  as  4  min.  Prop.  log.  1 .  6532 

Constant  log 8.8239 


Required  equation  =5  •    .    .    3729r      Pirop.  log.    .     1.7140 
The  equations  in  the  abovementioned  Table  were^  also>  computed  by 
Mrs.  T.  Kerigan. 


To  find  the  Equation  of  Equal  Altitudes  by  Tables  XIU.  and  XIV. 

Rule. 

Enter  Table  XIIL,  with  the  latitude  in  the  side  column  and  the  interval 
between  the  observations  at  top  $  and  find  the  corresponding  equation^  to 
which  j^refix  the  sign  +  if  the  sun  be  receding  from  the  elevated  poIe>  bul 
the  sign  —  if  it  be  advancing  towards  that  pde. 

Enter  Table  XIV.,  with  the  declination  in  the  side  eolumn,  and  the 
inlerral  between  tiie  observations  at  top,  and  take  out  tiie  corresponding 
equation,  to  which  prefix  the  sign  +  when  the  sun's  declination  is  increoi^ 
big,  but  the  sign  —  iwheh  it  is  decreasbig.  • 

liofw,  if  those  two  equations  are  of  the  same  signs ;  that  is,  both  afSniH 
ative  or  both  n^ative,  let  their  sum  be  taken)  but  if  contrary  signs,  namely^ 
one  affirmative  and  the  otiier  negative,  their  difierence  is  to  be  taken :  then. 

To  tlie  proportional  log.  of  this  sum  or  difference,  considered  as  minutes 
and  seconds,  add  the  proportional  log.  of  the  (bdly  variation  of  the  sun's 
declination ;  and  the  sum,  rejecting  1  from  the  index,  will  be  the  propor* 
tional  log.  of  the  true  equation  of  equal  altitudes  in  minutes  and  seconds, 
which  are  to  be  esteemed  as  seconds  wd  thirds,  and  which  will  be  always 
of  the  same  name  with  tiie  greater  equation. 

Example  1. 

In  latitude  49?  south,  the  interval  between  equal  altitudes  of  the  sun 
was  7*20?  j  the  suns  declination  18?  north,  increasing,  and  the  variation 
of  decUnation  15  '12? ;  required  the  true  equation  of  equal  altitudes  ? 


Digitized  by 


Google 


24  DBSCRIPTIOK  AND  USB  O^  THB  TABLB8* 

Opposite  lat.  49?  under  7^20?  Tab. XIII.  8tand8+ 15^27^ 
Opposite  dec.  18?  under  7 '  20r  Tab.  XIV.  stands  +  2. 30 

Sum 17^57^^  Pro. log.l. 0012 

Variation  of  declination     .    .     15 '.  I2r  Pto.  log.l .  0734 


True  equation,  as  required     +  1 5 ri Or  Pro.  log. 1 . 0746 
Example  2. 

In  ladtude  50?north,  the  interval  between  equal  altitudes  of  the  sun  was 
5^207;  the  sun*s  declination  IS^SO^north^  increasing,  and  the  daily  vari- 
ation 6(  declination  14^34?;  required  the  true  equation  of  equal  altitudes  ? 

Op.lat.  50?      under5*20?Tab.XIII.8tands-l4r5br 
Op.dec.l8?30'.under5.20  Tab. XIV. stands  +  3.11 

Diflference   ....     -Iir39r  Pro. log.  =  1.1889 
Variation  of  declination      1 4 '  34 ?  Pro.  log.  =  1 .  09 1 9 


True  equation,  as  required^  9^26^  Pro.  log.  =  1 .  2808 

Memarktr^ln  north  latitude  the  sun  recedes  from  the  elevated  pole  from 
the  summer  to  the  winter  solstice ;  that  m^  from  the  21st  June  to  the  21st 
December  $  but  advances  towards  that  pok  from  the  winter  to  the  summer 
solstice;  viz.,  from  the 21st  December  to  the  2l8t  June.  The  converse  of 
this  takes  place  in  soutli  latitude:  thus,  from  the  2l8t  June  to  the  21st 
December,  the  sun  advances  towards  the  south  elevated  pole ;  but  recedes 
from  that  pole  the  rest  of  the  year,  viz.,  from  the  2Ut  December  to  the 
21st  June. 

Here  it  may  be  necessary  to  observe,  that  in  taking  out  the  equations 
from  Tables  XIII.  and  XIV.,  allowance  is  to  be  made  for  the  excess  of  the 
givaui  above  the  next  less  tabuhir  arguments,  as  in  the  following  examples; 

Example  U 

Required  the  equation  from  Table  XIIL,  answering  to  latitude  50?48^9 
and  interval  between  the  ob^iervations  5  hours  10  minutes  ? 


Equation  to  latitude  50?,  and  interval  4 UOr  =    ^    .     14r33r 
Tabulardiff.tol?oflat,==  +  3irjnow,^^^'  =      +  0,24^ 

Tab.  diff.to40f  of  inter.  =  +  I7'J'jnow,l^J??-  ^      +  0. 12| 
Equation^  as  required'  ••«•,*,•••    .15^10? 

Digitized  by  VjOOQ IC 


DJUCRIPnoN  AND  USB.  OF  THE  TABtM.  25 

jtxample  2, 

Required  the  equation  from  Table  XIV.,  answering  to  sun's  decUnatloa 
20?47 ' ,  and  interval  between  the  observations  5  hours  10  minutes  ? 

Equation  to  declination  20?30;  and  intenral    4?40Tz3        3r44r    ' 

TabdardiflF.  to  30^  declination  ==+ 6^5. now,  5liiJ2- =5 +0.   3i 

3u. 

Tabular  diff.  to  401  interval  =  -  lOT;  now,  ^9L^f^^^  0.   7J 

Equation,  as  required    .    •    •    • 3^40^^ 

JVo/tf.— Should  the  latitude  exceed  the  limits  of  Table  XIII.,  which  is 
only  extended  so  far  as  to  comprehend  the  ordinary  bounds  of  navigation, 
viz.,  to  60  degrees,  the  first  part  of  the  equation,  in  this  case,  must  be 
determined  by  the  rule  under  which  that  Table  was  computed,  as  in 
page  22. 


Table  XV. 

To  reduce  the  Sim*8  Longitadej  Right  Jscemion,  and  Declination  ;  and 
also  the  Equation  of  Time,  as  given  in  the  NaiUical  Almanac j  to  any 
givefi  Meridian,  and  to  any  given  Time  wider  that  Meridian. 

This  Table  b  so  arranged,  that  the  proportional  part  corresponding  to  arty 
given  time,  or  longitude,  and  to  any  variation  of  the  sun's  right  ascension, 
declination,  &c.  &e.,  may  be  taken  out  to  the  greatest  degree  b(  accuracy, 
— even  to  the  two  hundred  and  sixteen  tliousandth  part  of  a  second,  \f 
necessary. 

.    Precepts. 

In  the  general  use  of  this  Table  it  will  be  advisable  to  abide  by  tiie  solar 
day ;  and  hence,  to  estimate  the  time  from  noon  to  noon,  or  from  0  to  24 
hours,  after  the  manner  of  astronomers,  without  paying  any  attention  to 
either  the  nautical  or  the  civil  division  of  time  at  midnight.  And  to  guard 
against  falling  into  error,  in  applying  the  tabular  proportional  part  to  the 
sun's  right  ascension,  declination,  &c.  &c.,  it  will  be  best  to  reduce  the 
apparent  time  at  ship  or  place,  to  Greenwich  time ;  as  thus : 

Turn  die  longitude  into  time  (by  Table  I.),  and  add  it^to  the  given  time 
at  ship  or  place,  if  it  be  ti;e»/;  but  subtract  it  {{east;  and  the  sum,^  or 
^foeocei  w31  be.  the  c9rr^pQnd}ng  time  at  Greenwich^ 


Digitized  by 


Google 


9§  DBSCRIpTION  AND  USB  09  Tfll^  TA^LM 

From  page  II.  df  the  month  in  the  Nautical  Almanac,  take  out  the 
sun's  right  ascension,  declination,  &c.  &c.,  for  the  noons  immediately 
preceding  and  folhmng  the  Greemoich  time,  and  .find  their  difference^ 
which  will. express  the  variation  of  those  elements  in  24  hours ;  then, 

Enter  the  Table. with  the  variation,  thus  found,  at  top,  and  the  Ghreen- 
wich  time  in  the  left-hand  column ;  under  the  former  and  .opposite  the 
latter  will  be  found  the  corresponding  equation,  or  proportional  part*  And^ 
since  the  Greenwich  time  may  be  estimated  in  hours,  minutes,  or  seconds, 
and  the  variation  of  right'  ascension,  &c.  &c.  &c.,  either  in  minutes  or 
seconds  s  the  sum  of  the  several  proportional  parts  making  up  the  whole 
of  such  time  and  variation  will,  therefore,  express  the  required  proportional 
part*  The  proportional  part,  so  obtained,  is  always  to  be  applied  by 
addition  to  the  sun's  longitude  and  right  ascension  at  the  preceding  noon  ; 
but  it  is  to  be  applied  by  additioUy  or  subtraction,  to  the  sun's  declination 
and  the  equation  of  time  at  that  noon,  according  as  they  are  increasing  or 
decreasing. — See  the  following  examples : — 

Example  I. 

Required  the  sun's  right  ascension  and  declination,  and  also  the  equation 
of  time  May  6th,l 824,  at  5  *  1 OT,  in  longitude  64 ?45  ^  west  of  the  meridian 
of  Greenwich? 

Apparent  time  at  ship  or  place 5^  10? 

Longitude  64?45^  west,  in  time  =:  .    .    •     +  4. 19 

Greenwich  time     •••••...•        9^29? 


To  find  tlie  &ia'8  Right  Ascension  :~ 

Sun's  right  ascension  at  noon,  May  6th,  1824,  per  Nautical 

Almanac, .    « 2^53:3l!42f 

Variation  iq  24*  =3^52'/ 

Pro.  part        to  9*  0?and  31  Or=  V,  7^30r  OV 
Do,  to  0. 29  and  3.   0  =  0.   3. 37. 30 

Do.  to  9.   0  and  0.50  =  0.18.45.   0 

Do.  to  0.29  and  0.50  =  0.    1.   0.25 

bo.  to  9.   0  and  0.   2  =  0.   0.45.   0 

Do.  to  0. 29  and  0.   2  =  0.   0.  2. 25 


Pro.  part        to  9*29?and  3^52?  is  1.81.40.20±:     +r31*40r 


Sun's  right  ascenrion,  as  required    •    •    •  ,  #  . «    •    •  2^55?  3-22> 

Digitized  by  VjOOQ IC 


Df  iCRIPTI^N  M«9  USB  QV  Ttt«  TA1IJUB9*  27 


To  find  the  Sun's  Declination  :-— 

San*8  declination  at  noon^  May  6th^  1824,  "pet  Nautical 

Almanac^      .*......•,.•....•     16?36'5? 

north,  increasing,  and  var.  in  24  ho.=  16^38T 

Pro.  part        to  9*  OTand  16^  0^=  6^  01  OT  0"." 
Do.  to  0.29  and  16.   0  =  0.19.20.   0 

Do.  to  9,   0  and    0.30  =  O.ll.lS.H) 

Do,  to  0.29  and    0.30  =  0.   0.36.15 

Do.  to  0.   0  and    0.   8  =  0.   3.   0.   0 

Do.  to  0. 29  and    0.   8  =  0.   0.   9. 40 

Pro.  part      'to  9*29rand  IG^SSris  6.34.20.55  «  +  6^34r 


Sim's  declination,  as  required     »    •    •    .    •    •    •    •    •     16?42'39C 


To  find  the  Equation  of  Time : — 

Equation  of  time  at  noon,  May  6thy  1624^  p«r  NtuUeal 

Almanac, •    •    .    3T36!  6f^ 

increasing,  and  variatioti  in  24  hours  aa  4?d6T 

Pro.  part        to  0*  0?and  4r  OT  «  l*30r  OV 

Do.  to  0.20  and  4.  0    »  (K   4.50 

Do.  to  9.  0  and  0.30    »  O.lhlS 

Do.  to  0«29  and  0.30    s  0.  0.36 


Pro.  part        to  9t29?  is    4r30r  =  K46.41  =     +  1^47^ 

* 'tm ' 

Equation  of  time,  as  required i    •   3T37'53f 

Example  2.  ' 

Required  th^  sun's  right  ascension  and  declination,  and  alio  the  eqtiation 
of  time,  August  2d,  1824,  at  19^22?,  in  longitude  98?45t  east  of  the 
meridian  of  Greenwich  ? 

Apparent  time  at  ship  or  place        .    >.    .    .     19^22? 
Longitude  98?45:  east,  in  time  a      •    .    .-6.35 

Greenwich  time       •..«•••••    12M7* 

Digitized  by  VjOOQ IC 


^9  BBSCRIPTIOK  AND  USB  OP  THB  TABIJ&I. 

To  find  the  Sun's  Right  Ascension  :— 

Sun's  right  ascension  at  noon^  August  2d^  1824^  per  Nautical 

Almanac, •    8*50rO!48! 

Variation  in  24  hours  =3^521^ 

Pro.  part        to  12t  Or  and  3^  Or  =  l^SOr  OT  0''/ 
•  Do.  to  .0.47     and  3.   0    =  0.   5.52.30 

Do.^  to  12.   0    and  0.50     =  0.25.   0.   0 

Do.       ^     to    0.47     and  0.50  ,=  0.    1.37.55 
Do.  to  12.   0    and  0.   2    =0.    I.   0.   0 

Do.  to    0. 47    and  0.   2    z=  0.   0.  3. 55 

Pro.  part        to  12*47^  and  3^52^  is   2.   8.34.20=  +2^3r34r 

Sun's  right  ascension,  as  required •      8^52^4!  22  f 

To*  find  the  Sun's  Declination  :— 

Sun's  declination  at  noon,  August  2d,  1824,  per  Nautical 

Almanac, w    .     .     17?44Mir 

north,  decreasing,  and  var.  in  24t  =:  15^  36? 
Pro.  part        to  12*  OTand  15C  07  =  7'30r  OT  OV 
Do.  to    0.47  «nd  15.  0    =  0.29.22.30 

Do.  to  12,   0  and    0.30    =:  0.15.   0.   0 

Do.  to    0. 47  and    0. 30    =0.   0. 58. 45 

Do.  to  12.  0  and    0.   6    =0.   3.  0.   0 

Do.  to    0.47  and    0.   6     =:  0.   0.11.45 

Pro.  part        to  12*47rand  15:36r  is    a.  18.33.   0=    -     8^9? 

gun's  declination,  as  required 17?36^22r 

To  find  the  Equation  of  Time :— - 

Equation  of  time  at  noon,  August  2d,  1824,  per  Nautical 

Almaniie, Sr54!24f 

decreasing,  and  variation  in  24  hours  =:  4^30^ 

Pro.  part        to  12*  OTand  4?  Or  =  2r  Or  OV 
Do.  to    0.47  and  4.   0     ==  0.   7.50 

Do.  to  12.  0  and  0.30     s  0.15.   0 

Do.  to    0.47  and  0.30    =  0,   0.58 

Pro.  part        to  12t47^and  4r30r  is    2.23.48  =  -    2r24r 

Equation  of  titpc;  as  required     .'••••,•,•    5752!  0! 

Digitized  by  VjOOQ IC 


BBSCRIFTION   AND  USB  OP  THE  TABLES.  29 

Hemark.^^hould  the  proportional  part  corresponding  to  the  daily 
variation  of  the  sun's  longitude  and  any  given  time  be  required,  it  may  be 
taken  from  the  first  page  of  the  Table,  by  esteeming  the  seconds  of  varia- 
tion, in  that  page,  as  minutes,  and  then  raising  the  signs  of  the  correspond- 
ing proportional  parts  one  grade  higher  ihan  what  are  marked  at  the  top 
of  the  said  page :  the  seconds  of  variation  will,  of  course,  be  taken  out  after 
the  usual  manner.    Thus, 

Suppose  that  the  daily  variation  of  the  sun's  longitude  be  57-  40T,  and 
the  Greenwich  time  9  hours  50  minutes,  to  find  the  corresponding  equationy 
or  proportional  part. 


Vto.  part  to  9*  OTand  50C  Or  =  l8M5r  Or  OV 

Do.  to  9.   0  and    7.   0     =  2.37.30.   0 

Do.  to  0.50  and  50.   0     s  1.44.10.   0 

Do.  to  0.50  and    7.   0    s  0.14.35.  0 

Do.  to  9.   0  and    0.40     =  0.15.   0.   0 

Do.  to  0.50  and     0.40  '=  0.    1.23.20 


Pro.  part        to.9t50rand  57'40r  is   23.37.38.20=  23J38f  + 


Kote. — It  is  easy  to  perceive  that  the  foregoing  operations  might  have 
been  much  contracted,  by  taking  out  two  or  more  of  the  proportional  parts 
at  once ;  but,  lest  doing  so  should  appear  anywise  ambiguous  to  such  as 
are  not  well  acquainted  with  the  method  of  taking  out  tabular  numbers,  it 
was  deemed  prudent  to  arrange  the  said  operations  according  to  their 
present  extended  form,  so  as  to  render  them  perfectly  intelligible  to  every 
capacity. 

The  present  Table  was  computed  agreeably  to  the  established  principles 
of  the  rule  of  proportion ;  viz..  As  one  day,  or  24  hours,  is  to  the  variation 
of  the  sun's  right  ascension,  declination,  &c.  &c.,  in  that  time,  so  is  any 
other  portion  of  time  to  the  corresponding  proportional  part  of  such 
variAtion. 


Digitized  by 


Google 


30  BESCniPTlON  AND  TJSB  OF  THK  TABUSS. 

Tablb  XVI, 

To  reduce  the  Moon's  Longitude,  Latitude,  Right  Jscension,  Declination, 
Semidiameter,  and  Horizontql  Parallax,  as  given  in  the  Tiautical 
Almanac,  to  any  given  Meridian,  and  to  any  given  Time  under  that 
Meridian. 

This  Table  is  arranged  in  a  manner  so  nearly  similar  to  the  preceding, 
that  any  explanation  of  its  use  may  he  considered  almost  unnecessary ;  the 
only  difference  being,  that  the  proportional  parts  are  computed  to  wiatioa 
in  12  hours,  instead  of  24.  By  means  of  the  present  Table,  the  proper* 
tional  part  corresponding  to  any  variation  of  the  moon's  longitude,  latitude, 
right  ascension,  &c.  &c.  &c.,  may  be  easily  obtained,  to  the  greatest  degree 
of  accuracy,  as  follows ;  viz. 

Turn  the  longitude  of  the  ship  or  place  into  time  (by  Table  I.),  and  add 
it  to  the  apparent  time  at  such  ship  or  plaise,  if  it  be  west;  but  subtract  it 
if  east :  and  the  sum^  or  difference,  will  be  the  corresponding  time  at 
Qreenwich. 

Take  from  pages  V.,  VI.,  and  VII.  of  the  month,  in  the  Nautical 
Almanac,  the  moon's .  longitude,  latitude,  right  ascension,  declination, 
semidiameter,  and  horizontal  parallax,  (or  any  one  of  tliese  elements, 
a<icording  to  circumstances,)  for  the  noon  and  midnight  imme^Kfttely 
preceding  and  following  the  Greenwich  time,  and  find  their  difference ; 
which  difference  will  express  the  variation  of  those  elements  in  12  hours. 

Enter  the  Table  with  the  variation,  thus  found,  at  top,  and  the  Green- 
wich time  in  the  left-hand  column ;  in  the  angle  of  meeting  will  be  found 
the  corresponding  equation,  or  proportional  part,  which  is  always  to  be 
added  to  the  moon's  longitude  and  right  adcension  et  the  preceding  noon 
or  midnight,  but  to  be  applied  by  addition,  or  subtraction^  to  the  moon's 
latitude,  declination^  semidlanieter,  and  horizontal  parallax,  according  as 
they  are  increasing  or  decreasing.  And,  since  the  Greenwich  time  and  the 
variation  in  12  hours  will  be  very  seldom  found  to  correspond  exactly;  it  is 
the  sum,  therefinre,  of  the  several  equations  making  up  those  terms^  that 
will,  in  general,  express  the  lEequired  proportional  part* 

*  JExample. 

Required  the  moon's  longitude,  latitude,  right  ascension,  decliiMtaon, 
semidiameter,  and  horizontal  parallax,  August  2d,  1824,  at  3  MO",  in 
longitude  60?30',  west  of  the  meridian  of  Greenwich  ? 

Apparent  time  at  ship  or  place     .     ,     .    .    .     3*10? 
Longitude  60?  30^  west,  in  time  =     .    ,-  .    4.   2 


Greenwich  time       ••••••,••    7n-2? 

/Google 


Digitized  by ' 


DBSC&fFTtOK  AND  USE  OF  THB  TABIS9.  81 

To  find  the  Moon's  JLiongitude :— 


oon's  longitude  at  noon,  August  2d,  1824,  per  Nautical 

Almanac, 

.......; 7M7?16(27? 

Variation  i 

in  12*  =6°  snsgr 

PropoT.  -part 

to  7*  or  and  6?  0'.  07  =  3»30^  0?  Or 

Do. 

to  0. 12    and  6.  0.  0    =z  0.   6.   0.  0 

Do. 

to  7.  0    and  0.30.  0    =  0.17'.30.  0 

Do. 

to  0.12    and  0.30.  0    =0.  0.30.  0 

• 

Do. 

to  7.   0    and  0.   1.  0    =0.   0.35.  0 

Do. 

to  0.12    and  0.   1.  0    =s  0.   0.    1.   0 

Do. 

to  7.  0    and  0.  0.50    =  0.  0.29.10 

Do. 

to  0. 12    and  0.   0.50    =  0.   0.  0.50 

Do. 

to  7.  6    and  0.  0.  9    =  0.  0.  5. 15 

Do. 

to  0. 12    and  0.  0.   9    =  0.  0.  0.  $ 

Proper,  part 

to  7^2?  and  6?3i:597  U   3.55.11.24= 
tude,  as  required 7 

;+S?55MI* 

Moon's  lengi 

!21?li:38' 

To  find  the  Moon's  Latitude  ^-««- 

Moon's  latitude  at  noon^  August  2d,  1824,  per  Nautical 

Almanac, 4?6'59: 

south,  decreasing,  and  var*  in  12  hours  s  23' 35  Y 
Proportional  part  to  7?  0?  and  20^  O:  =  U'AOl  OT 
Do.  to  0.12    and  20.   0     =    0.20.   0 

Do.  to  7.   0    and    3.   0     =     1.45.   0 

Do.  to  0. 12    and    3.   0     =     0.  3.   0 

Do.  to  7.   0    and    0.30    =     0.17.30 

Do.  to  0. 12    ^d    0.30    =z     0.   0.30 

Do.  to  7.   P    and    6.   5     df    0.    2.55 

Do.  to  0. 12    and    0.   5     =     0.   0.   5 


Proportional  part  to  7*12?  and  23:35r   is  14.   9.   0  =s-  W.  9t 


Moon's  latitude,  as  required     .    «    .    • 3?52^50r 

Note. — In  consequence  of  the  unequal  motion  of  the  moon  in  12  hours, 
(when  her  place  is  to  be  determined  with  astronomical  precision,)  the 
proportional  part  of  the  Tariation  of  her  longitude  and  latitude,  found  as 
above,  mtist  be  corrected  by  the  equatioa  of  second  difference  contained  in 
Table  XVIL ;  and  the  same  may  be  obsenred  of  her  right  ascension  and 
declination. 


Digitized  by 


Google 


82  BJBSCRIPTION  AND   USE  OF  THB  TABLB8# 

Tq  find  the  Moon's  Right  Ascension  :— 

Moon's  right  ascension  at  noon,  August  2d,  1824,  per 

Nautical  Almanac, 223?33^36^ 

Var.inl2*  ='6?51M9r 

Propor.  part  to  7i  or  aAd  6?  Of  Or  =  3?30f  01  OT 
Do.  to  0. 12  and  6.  0.  0  =  0.  6.  0.  0 
Do.  to  7.  0  and  0.50.  0  =:  0.29.10.  0 
Do..  to  0.12  and  0.50.  0  =  0.  0.50.  0 
Do.  to  7.  0  and  0.  1.  0-  =  0.  0.35.  0 
Do.  to  0.12     and  0.    1.   0     =  0.   0.  .1.   0 

Do.  to  7.  0  and  0.  0.40  rs  0.  0.23.20 
Do.  to  0.12  and  0.  0.40  =0.  0.  0.40 
Do.  to  7.  0  and  0.  0.  9  =s  0.  0.  5. 15 
Do.'         to  0.12    and  0.   0.   9     =  0.   0.   0.   9 


Propor.  part  to  7*12?  and  6?5i:49ris    4.   7.   5. 24= +4?  7^  51 
Moon's  right  ascension,  as  required  ••••.«.      227?40'4lr* 

To  find  the  Moon's  Declination  :— 

Moon's  declination  at  noon,  August  2d,  1824,  per  Nautical 

Almanac,-.   '. 20?57f   H 

8outh,  increasing,  and  var.  in  12  ho.=  I  ^23' 431 


•or.  part  to  7-  0' 

and  1?  0'.  or 

:= 

35^  or  or 

Do. 

to  0.12 

and  1.   0.   0 

= 

1.  0,  0 

Do. 

to  7.  0 

and  0. 20.   0 

^z 

11.40.   0 

Do. 

to  0.12 

and  0.20.   0 

zz 

0.20.  0 

Do. 

to  7.  0 

and  0.  3.   0 

ZZ 

1.45.  0 

Do. 

to  0. 12 

and  0.   3.  0 

= 

0.  3.  0 

Do. 

to  7.  0 

and  0.  0.40 

"" 

0.23.20 

Do. 

to  Oi.12 

andO.   0.40 

™* 

0.  0.40 

Do. 

to  7.  0 

and  0.   0.   3 

^ 

0.    1.45 

Do. 

to  0.12 

and  0.   0.  3 

= 

0.   0.  3 

Propon  pirt  to  ■7M2?  and  'l  ?23'43r   is  50. 13. 48  =    +  50;  14r 


Moon's  declination^  as  required 21?47;2lr* 

*  When  ftecuracy  is  required,  the  moon's  rif^ht  aicension  and  declination  must  be  cor- 
rected by  the  equation  of  second  dlfferencCi  on  account  of  the  Irref^arities  of  her  motioa 
in  12  hours. 


Digitized  by 


Google 


» 


DBSCRIPTiON  AND   USB  OF  THB  TAJBLB6.  S3 


To  find  the  Moon's  Semidtameter :— « 

Moon's  semidiameter  at  noon,  August  2d,  1824,  per  NouUcal 

Almanac, #    •    .    .    •      15'33^ 

decreasing,  and  var.  in  12  hours  =  %". 
Proportional  part  to  7*  0?  and  6r  =  SrSOT 
Po.  to  0.12    and  6     s=  0.   6 

Proportional  part  to  7*12?  and  6r   is  3.36      =s  -    4 

Moon's  semidiameter,  as  required      ••«;••••     15^291 

To  find  the  Moon'9  Horizontal  Parallax  :— 

Mood's  horizontal  parallax  at  noon,  August  2d,  1824,  per 

Nautical  Almanac,       •    •    • •     57*6! 

decreasing,  and  var.  in  12  hours  =  231" 
Proportional  part  to  7*  OT  and  20r  =  lir40r 
Do.  to  0.12    and  20    =     0.20 

Do.  to  7.   0    and    3     =     1.45 

Do.  to  0. 12     and    3     =    0.   3 


» 


Proportional  part  to  7*12?  and  23r   is  13.48      =        -     14? 

Moon's  horizontal  parallax,  as  required      ^    t    «     •    •    «    56'52T 

Remarks. — 1.  It  is  evident  that,  in  the  above  operations,  the  greater 
part  of  the  figures  might  have  been  dispensed  with,  by  taking  out  two  or 
more  of  tlie  proportional  parts  at  once  ;  but  since  they  were  merely  intended 
to  simplify  and  render  familiar  the  use  of  the  Table,  the  whole  of  the  pro- 
portional parts  have  been  put  down  at  length. 

2.  This  Table  was  computed  according  to  the  rule  of  proportion;  viz.:-^  , 
As  12  hours  are  to- the  variation  of  the.  moon's  longitude,  latitude,  right 
ascension,  &c.  &c.  &c.,  in  that  interval,  so  is  any  other"  given  portion  of 
time  to  the  corresponding  proportional  part  of  such  variation. 

Tabls  XVII. 

Squation  of  Secmd  Difference. 

Since  the  moon's  longitude  and  latitude,  and  also  her  right  ascension 
and  declination,  require  to  be  strictly  determined  on  various  astronomical 
pceasioi^;  porUcularl^  the  twQ  latter  when  the  apparetU  time  is  to  be 

Digitized  by  VjOOQ IC 


S4  DESCRIPTION  AND  tJSfi   6F  THE  TABLES. 

inferred  from  the  true  altiiude  of  that  object;  and  since  the  reduction 
of  these  elements^  to  a  given  instant, 'cannot  be  performed  by  even  propor- 
tion, on  account  of  the  great  inequalities  to  which  the  lunar  motions  «« 
subject }— a  correctio/r,  therefore,  resulting  from  these  .inequalities,  must  be 
applied  to  the  proportional  paYt  of  the  moon's  longitude  or  latitude,  right 
ascension  or  declination,  answering  to  a  given  period  after  noon  ^f  mid- 
night, as  deduced  from  the  preceding  Table  or  otherwise,  in  4»rder  to  have 
it.truly  accurate.  This  correction  is  contained  in  the  present  Table,  the 
arguments  of  which  are, — ^the  mean  second  difference  of  the  moon's  place 
at  top ;  and  the  apparent  or  Greenwich  time  past  noon,  or  midnight,  in 
the  left  or  right-hand  column ;  in  the  angle  of  meeting  stands  the  corre- 
sponding equation  or  correction. 

The  Table  is  divided  into  two  parts !  the  upper '  part  ia  adapted  to  the 
mean  second  difference  of  the  moon's  place  in  seconds  of  a  degree,  and  in 
which  the  equations  are  expressed  in .  seconds  and  decimal .  parts*  of  a 
second ;  the  lower  part  is  adapted  to  minutes  of  mean  second  difference  ; 
the  equations  being  expressed  in  minutes,  and  seconds,  and  decimal  parts 
of  a  second. 

In  using  this  Table,  should  the  mean  second  difference  of  the  moon's 
place  exceed  its  limits,  the  sum  of  the  equations  corresponding  to  the 
several  terms  which  make  up  the  mean  second  difference,-  in  both  parts  of 
the  Table,  is  in  such  case  to  be  taken.  The  manner  of  applying  the 
equation  of  second  difference  to  the  proportional  part  of  the  moon's 
'motion  sn  latitude,  longitude,  right  ascension,  or'  declination,  as  deduced 
from  the  preceding  Table,  or  obtained  by  even  proportion,  will  be  seen  in 
the  solution  to  the  following 


Problem*    * 

7b  reduce  the  JU9m*s  LatUude,  Longiiude,  Eight  McentUm^  <»td  J>e^&a- 
tion,  a  given  w  the  NauHcal  jUmame,  to  any  given  Time  under  a 
hwm  Meridian. 

Rule. 

Turn  the  longitude  into  time,  (by  Table  I.)  and  apply  it  to  the  apparent 
time  at  ship  or  place  by  addition  in  west,  or  subtraction  in  east  longitude  j 
and  the  sum,  or  difference,  will  be  the  corresponding  time  at  Greenwich. 

Take  from  the  Nautical  Almanac  the  two  longitudes,  latitudes,  right 
ascensions,  and  declinations  immediately  preceding  and  following  the 
Greenwich  time,  and  find  the  difference  between  each  pair  successively  j 
find  also  the  second  difference,  and  let  its  mean  be  taken. 

Find  the  proportiood  part  of  the  middle  Jbrst  difliereiice,  (the  tariation 


Digitized  by 


Google 


BiiscRipnoN  akd  ttsb  of  fnA  fABLSs.  65 

of  the  moon's  motion  in  1^  hours^)  by  Table  XVL^  answering  to  the 
Greenwich  lime^ 

With  the  mean  second  difference,  found  as  above,  and  the  Greenwich 
time,  enter  Table  XVIL,  and  take  out  the  corresponding  equation.  Now, 
thia  equation  being  •  addtd  to  the  proportional  part  of  the  moon's  motion 
if  the  first  first  difference  is  greater  than  the  third  first  difference^  but 
fubtracted  if  it  be  less,  the  sum  or  difference  will  be  the  correct  propor- 
tional part  of  the  mood's  motion  in  1 2  hours. 

The  correct  proportional  p&rt,  thus  found.  Is  always  to  be  added  to  th4 
moon's  longitude  and  right  ascensioa  at  the  noon  or  midnight  preceding 
the  Greenwich  time  ^  but  to  te  applied  by  addition  or  subtraction  to  her 
latitude  and  declination,  accoitling.as  they  may  be  increasing  or  de- 
creasing. 

^  Example. 

Required  the  moon's  correct  longitude,  latitude,  right  ascension,  and 
declination,  August  2(1, 1^24,  at  3^  10?  apparent  time,  in  longitude  60^30., 
west  of  the  meridian  of  Greenwich } 

Appajrent  time  at  ship  or  place  •    •    •    •    .    SMO? 
Longitude  60?30^  west,  in  time  a     «    •    •    4.  2 

Greenwich  time  •    «    •   V 7M2? 

To  find  the  Moon's  comet  Longitude  :-^ 

First      Second   Mean 
Diff.         Diff.    2dDiff. 
Moon's  long.  Aug.  l8t,atmidnt.  7*  10^38 M9r  1^^007/00//! 

Do.  2     atnoon    7.17.16.27  [Ai  JLi^'^^'^U'oji^ 

Do.  i     atmidnt.7.23.48.26  j«-31.59  jg  j^  j^*?*- 


Do.  3     atnoon    8.   0.15/ 9  l6.26.43 


Pfopor.  part  from  Table  XVL,ans.to7M2?and6?3l^59ris3?65nK24r 
Bq.  fromTab.  XVIL,  corres.  to  7*  12?  and  5 ^  Or  =:  36^  0 

and0.20    =2.4 
andO.   7J=    0  .9 


Eq.ofmeaiisecoriddiff.an6.to7*12Tand5^27|ris39  .3=  +    39n8r 

Correct  proportional  part  of  the  moon's  motion  in  longitude  3?55  '60^42? 
Idoon's  longitude  at  noon,  August  2d,  1824 .     .     .    .    7  •  17. 16. 27.   0 

Moon's  correct  longitude,  at  the  given  time  •    •    .    .    7 '  2 1  ?  1 2 : 1 7  742r 

j)2 

Digitized  by  VjOOQ IC  ' 


SQ,  2>BSCR1FTI0N  AND   USB  OF  THB  TABLBS. 

To  find  the  Moon's  correct  Latitude :— • 

First    Second  Mean 
Diff.      Diflf.    2dDiff. 
Moon's  lat.  Aug.  Ist,  at  midnt.  4?27 '37^  S*  lon^^fi'^i 

Do.  2      atnoon    4.   6.59        I  f2^57^lo/^^. 

Do,  2     at  midnt.  3. 43. 24        }23.35   \^  gj  J2.44. 

Do.  3     atnodn   3.17.18        }26.   6  ^ 

Pro.partfromTableXVI.,an8.to7*12rand23^35r  is0?14^  9r 
Bq.fromTab.XVIL,cor.to7*12r  and2^  Or=  14^4 

and  0.40  =4.8 

andO.   4=0.5 


Eq.ofmean8ec.diff.,ans.to7*l2rand2U4fis  19  .7=  —  19^^.7 

Correct  proportional  part  of  the  moon's  motion  in  lat.  0?13C49^.3 
Moon's  latitude  at  noon^  August  2d,  1824   •    •    •    .   '4.   6.59  .0  S. 

Moon's  correct  latitude  at  the  given  time  •    •    .    «    3?53'  9^".  7south. 

To  find  the  Moon's  correct  Right  Ascension:— 

First       Second  Mean 
'    •  Diff.         Diff.   2d  Diff. 

Moon's  R.  A.  Aug.  1st,  at  midnt  216?44  ;43r « ^\,  .<. ,  .„«. 

Do.  2     atnoon    223.33.36     °  [2;56ri„,„q, 

Do..  2     atmidnt.230.25.25.  6-51.49  I2    gJ 

Do.  3     atnoon    237' 1^.22  r6.53.57  '   ' 

Propor.partfromTableXVI.,an8.to7M2?and6?51M9ri8  4?  7'  5r24r 
Eq.  from  Table  XVII.,  ans.  to  7  M  2?  and  2  C  Or  =  14".4 

andO.30   =   3  .6 
andO.   2   =   0.2 


^.ofmeansec.  diff.,ans.to7M2?and  2!32ris  18  .2  =     -     18M2r 


Correct  prbpor.  part  of  the  moon's  motion  in  right  ascension  4?  6^47^12^ 
Moon's  right  ascension  at  noon,  August  2d,  1824     .    .    223.33.36.   0 

Moon's  correct  right  ascension  at  the  given  time   .     .    •     227?40'23ri2*r 

To  find  the  Moon's  correct  Declination : 

First  .     Second    Mean 
DiflF.         Diflf.     2dDiflF. 
Moon's  dec,  Aug.  1st,  at  midnt*  19?  15  C49r  8.1,0^1/10.^1 

Do.  2      atnoon    20.57.    7       L  !  h7-35r),-..ft^ 

Do.  2     at  midnt.  22. 20. 50       U.23.43  {^g  gj   jl/'^^- 

Do.  3     atnoon    23.26.12    '  }l.   5.22   * 


Digitized  by 


Google 


JttSCRlPnON  AND  ITSB  OF  THB  TABLES.  87 

fto,  part  fir.  Tab.  XVI.,  ans.  to  7  *  1 2?  and  1 923  M3r  is  0?50r  13r48r 
Eq.fr.Tal>.XVII.cor.to7M2?andl5^  0^=1  M8^0 

and   2.  0  =0.14  .4 

and  0.50  =0.   6  .0 

and  0.   8  =0.    1  .0 


Eq.ofmn8ec.diff.,an8.to:r*12?andl7'Mrw2.  9  .4=  +  2^  9r24r 


Correct  prop,  part  of  the  moon's  motion  in  declination    0?52 '  23T 12T 
Moon's  declination  at  noon^  August  2d^  1824     .    •    20.57.   7.   OS. 


Moon^s  correct  declination  at  the  given  time     .     •     21  ?49 '  30?  1 2T  south. 

Note. — It  frequently  happens  that  the  three  ^r«t  differences  first  increase 
and  then  decrease^  or  tnce  versa,  first  decrease  and  then  increase ;  in  this 
case  half  the  difference  of  the  two  second  differences  is  to  be  esteemed  as 
the  mean  second  difference  of  the  moon's  place  :  as  thus^ 

Rrst    Second    Mean 
Diff.      Diff.     2dDiff. 
Mn'sdec.Aug.l8th,1824,atmidt.24?23C26rN.>,Q,rt,..> 

Do.  19  atnoon  24.41.47       T  [l4'26fi-,o«^ 

Do.  19  atmidt.24.37.52       /  3-55  {33  jo  r*^^* 

Do.  20  atnoon  24.10.39       }27. 13  ^ 

Here  the  two  second  differences  are  14 '26?,  and  23'  18f  respectively; 
therefore  half  their  difference/viz.,  8f52r  h-  2  =  4C26r  is  the  mean 
Second  difference.  Now,  if  the  Greenwich  time  be  5  MO?  past  noon  of 
the  19th,  the  corresponding  equation  in  Table  XVII.  will  be  33f  sub-' 
traciive^  because  the  first^W  difference  is  less  than  the  third  Jirst  differ^ 
ence;  had  it  been  greater,  the  equation  would  be  addiiive* 

Remark. — ^When  the  i^parent  time  is  to  be  inferred  from  the  true  ' 
altitude  of  the  moon's  ccfntre,  the  right  ascension  and  declination  of  that 
object  ought,  in^neral,  to  be  corrected  by  the  equation  of  second  differ- 
ence ;  because  an  inattention  to  that  correction  may  produce  an  error  of 
about  2|  minutes  in  the  right  ascension,  and  about  4  minutes  in  the  de- 
elination ;  which,  of  course,  will  affect  the  accuracy  of  the  apparent  time.— 
See  the  author's  Treatise  on  the  Sidereal  and  Planetary  Parts  of  Nautical 
Astronomy,  pages  171  and  172, 

Tlie  equation  of  second  difference,  contained  in  the  present  Table,  was 
computed  by  the  following  . 

Rule. 

To  tha  constant  log.  7«  540607  add  the  log.  of  the  mean  second  differ- 
mce  reduced  to  seconds ;  the  log.  of  the  time  from  noon,  and  the  log.  of 


Digitized  by 


Google 


98  IffiSCRIfTfOlif  4ND  U8H  09  THl  TABU9, 

the  diiferenc^  of  ttial  time  to  12  hours  (both  expressed  !n  houro  and 
decimal  parts  of  an  hour) :  the  sum,  rejecting  IQ  from  the  index,  will 
be  the  log.  of  the  equation  of  second  difference  in  seconds  of  a  degree* 

Example. 

Let  the  mean  second  difference  of  the  m<>on'8  plac6  be  8  mii^utesi  and 
the  apparent  time  past  noon  or  midnight  3  t20T;  required  the  correspond- 
ing equation }  ' 

Mean  second  difference,  8  minutes  =  480  seconds.  Log.  =z  2. 681241 
Apparent  time  past  noon  or  midnight  ==  3* .  333  Log.  ==  Q.  $3SS835 
Difference  of  do.  to  1 2  hours  8  * .  666        Log.  =  0. 9378 1 9 

Constant  log.  (ar.  co.  of  log.  of  288  =  24  x  12)     .    .      =7. 640607 

Required  equation    • 48''.  14  Log.  ==  1 .  68250S 


Table  XVIII. 
Correction  qf  the  Mgon^s  Jpparent  Jltitude. 

By  the  correction  of  the  mqon's  apparent  altitude  is  ^ant^  the  diBfer- 
ence  between, the  parallax  of  that  object,  at  any  given  altitude,  and  the 
refraction  corresponding  to  that  altitude. 

This  correction  was  computed  by  the  following  rule ;  viz. 

To  the  log.  secant  of  the  moon's  apparent  altitude,  add  the  proportional 
log.  of  her  horizontal  parallax;  and  the  sum,  abating  10  in  the  index,  wilt 
be  the  proportional  log.  of  the  parallax  in  altitude ;  which,  being  diminished 
by  the  refraction,  will  leave  the  correction  of  the  moon's  apparent  altitude. 

Let  the  moon's  apparent  altitude  be  25?40^,  and  her  horizontal  parallax 
59  minutes ;  required  the  correction  of  the  apparent  altitude  ? 

Moon's  iq>parent  altitude    .    ,    ,    .    25?40^     Log,  secant  =:  10.0451 
Moon's  horizontal  parallax      •    •    •    *  0.  $9      Propor,  log.  =:    0*  4844 

Moon^  parallax  in  altitude     ;    .    .    53'IH  scProporJog.s:    0.5295 
Refraction  ans.  to  app.  ait.  in  Tab.  VIIL    1.58 

Correction  of  the  moon's  appar.  altitude  51^131^ 

The  correction^  thus  compnt^d,  is  arranged*  in  the  present  Tahle,  where 
it  is  given  to  every  tenth  minute  of  apparent  altitude^  and  to  each  minute 


Digitized  by 


Google 


]»4CKIFnON  AND  US^  OF  THS  TABLES*  89 

of  horizontal  parallax.  The  proportional  part  for  the  excess  of  the  g^ven 
above  ttie  next  less  tabular  altitude^  is  contained  in  the  right-hand  column 
of  each  page ;  and  that  answering  to  the  seconds  of  parallax  is  given  in 
the  intermediate  part  of  the  Table^ 

This  correction  is  to  be  taken  out  of  the  Table  in  the  following  manner  ; 
viz. 

BntCK  the  Table  with  the  moon's  apparent  altitude  in  the  left-hand 
eoluDiii,  or  the  altitude  iiext  iess  if  there  be  any  odd  minutes ;  opposite  to 
whichy  and  wider  the  minutes  of  the  moon's  horizontal  parallax^  will  be 
found  the  approximate  correction.  Enter  the  compartment  of  the  ^'  Pro-« 
poitiona]  parta  tP  seconds  of  parallax/'  abreast  of  the  approximate  correct 
tioDy  with  the  tenths  of  seconds,  of  the  moon'«  horizontal  parallax  in  the 
vertical  column,  and  the  units  at  t(ie  top ;  in  the  angle  of  meeting  will  ba 
feuttd  the  proportioiial  pari  for  second^  which  add  to  the  approximate, 
correction.    Then, 

Enter  the  last  or  right-band  column  of  the  page,  abreast  of  the  approxi* 
mate  coivection  or  nearly  so,  and  find  the  proportional  part  corresponding^ 
to  the  odd  minutes  of  altitude.  Now,  this  being  added  to  or  subtracted 
from  tlfte  approximate  correction,  according  to  its  signy  will  leave  the  trua 
eocraelfam  of  the  moonV  apparent  altitude, .  And  since  the  apparent  alti- 
tude of  a  eelestial  oltfect  is  depressed  by  parallax  and  raised  by  refraction, 
and  the  lunar  parallax  being  always  greater  than  the  refraction  to  the 
aaiqe  allitude,  it  hence  ioUaws  that  the  correction,  thus  deduced^  i»  always 
to  liQ  iiPpUdi  by  {^Mlieu,  (o  th^  moon's  apparent  altitude, 

Example  U 

Let  Iha  bbooii*s  apparent  aiti^de  be  &?38C,  aad  her  bori«(mtal  parallax 
57^46?  I  requiied  dM»  corresponding  c<weetion } 

Correction  to  alL  8?3a^  and  horiz.  parallax  57 'Or  is    50n4r 
P^opor.  part  to  46  seconds  of  horiz.  parallax    •    •     +     0. 46 
Do.       to  8  min.  of  alt,  (8*  x  Of.  5=4 -'.  0)  =  +     0.   4 

Correction  of  the  moon's  apparent  altitude^  as  required  5 1 C  it 

Let  the  moon's  apparent  altkude  be  S3?I0C,  and  her  horizontal  parallax 
59^34T ;  required  the  corresponding  correction  ? 

Correction  to  alt  33?  10^  and  horiz.  parallax  59^ Or  is    47'56r 
Propor.  part  to  34  seconds  of  horiz.  parallax    •    •    •     +     28 
Do.      to  6  minutes  of  altitude —      3 

Correction  of  the  moon's  apparent  altitude,  as  req^irfd    48 '  2  K. 

Digitized  by  VjOOQ IC 


40  bESCRlPf  ION  AND   USS  OF  THE  TABLES. 

•Table  XIX. 

To  reduce  the  Tme  Jllitudes  qf  the  Sun,  Moon,  Stars,  and  PUmefs,  * 
to  their  apparent  Jltitudes. 

This  Table  is  particularly  useful  in  that  method  of  finding  the  longitude 
by  lun{ir  observations,  where  the  distance  only  is  given,  and  where,  of 
bourse,  the  altitudes  of  the  objects  must  be  obtained  by  computation. 
'  The  Table  consists  of  two  pages,  each  page,  b^ing  divided  into  two* 
parts  :  the  left-hand  part  contains  four  columns ;  the  first  of  which  com- 
prehends the  true  altitude  of  the  sun  or  star;  the  second  the  reduction  of 
the  sun's  true  altitude ;  the  third  the  reduction  of  a  stair's  true  altitude  f 
ind  the  fourth  the  common  difference  of  those  reductions  to  1  minute  of 
altitude  for  sun  or  star. 

The  other  part  of  the  Table  is  appropriated  to  the  moon ;  in  which  the 
true  altitude  of  that  object  is  given  in  the  column  marked  "  Moon's  true 
altitude,"  and  her  horizontal  paralla^f  at  top  or  bottom ;  the  two  last  or 
right  hand  columns  of  each  page  contain  the  difference  to  1  minute  of 
idtitude,  and  1  second  of  parallax  respectively ;  by  means  of  Which  the 
reduction  may  be  easily  taken  oiit  to  minutes  of  altitude  and  seconds  of 
horizontal  parallax. 

The  first  part  of  the  Table  is  to  be  entered  with  the  sun's  or  star's  true 
altitude  (or  the  altitude  next  less  when  there  are  any  odd  minutes,  as  there 
generally  will  be,)  in  the  left-hand  column ;  abreast  of  which,  in  the  proper 
column,  will  be  found  the  approximate  reduction ;  from  which  let  the  pro- 
duct of  the  difference  to  1  minute  by  the  excess  of  the  odd  minutes  above 
the  tabular  altitude,  be  subtracted,  and  the  remainder  will  be  the  true 
*  reduction  of  altitude  for  sun  or  stftr. 

Example  1.  . 

Let  the  true  altitude  of  the  sun's  centre  be  8?  15  f  j  required  the  reduc- 
tion to  appar^it  altitude  ?' 

Correction  corresponding  to  altitude  8  de^ees     .    ,    .     .     .    .    6^5^ 
Cor.  for  min.  of  alt. ;  viz.  diff.  tol  min.  of  alt.=0''.  70  x  15  C  =t  lO''.  5  =— M) 

Required  reduction  =     ••••••«•••••,•    6'5r 

Example  2. 

Given  the  true  altitude  of  a  star  19?45; ;  the  reduction  to  apfmrent 
altitude  is  required  ? 


Digitized  by 


Google 


MMCKIPTIOV  AKD  tTSB  OV  THE  TABLES.  41 

CSorrection  corresponding  to  altitude  19  degrees      •    •    ^    /    #    2M4f 
Cor.formin.of  alt;viz.  diff«to  1  inin.ofalt«=:0'^.  15x4S'z:6''.75=--7 


Required  reduction  =r 


The  reduction  of  the  moon's  true  altitude  is  to  be  taken  from  the  second 
part  of  the  Table,  by  entering  that  part  with  the  true  altitude  in  the  proper 
column  (or  the  altitude  next  les9  when  there  are  any  odd  minutes)  and  the 
horizontal  parallax  at  top  or  bottom ;  in  the  angle  of  meeting  will  be  found 
a  correction ;  to  which  apply  the  product  of  the'  difference  to  1  minute  by 
the  excess  of  the  odd  minutes  above  the  tabular  altitude  by  Mubtradian,  and 
the  product  of  the  difference  to .  1  second  by  the  odd  seconds  of  parallax  by 
addition :  and  the  true  reduction  will  be  obtained^  as  may  be  seen  in  the 
following 

Sstample,        -     - 

Let  the  true  altitude  of  the  moon's  centre  be  29?  13 ' ,  and  her  horizontal 
paralUx  58(37?  ;  required  the  corresponding  reduction  to  apparent  alti- 
tude ? 

Correc.  corres.  to  alt  29  degs. ^  and  horiz.  parallax  58;=       .     •    49 '  22  ? 
Cor.  for  min.of  alt.;  viz.,  diff.to  1  min.of  alt.=:  0''.41  x  13^=5*'. 3=^5 
Gor.for  sees,  of  par.;  viz.^  diff.  tolsec.  of  par.=0''.  90  x  37''=:33'',  3= +33 

Required  reduction //.'.'.'.     .     .     .    49' 50? 

2ZefifarJlr.->-The  reduction  of  the  sun's  true  attitude*  is  obtained  by  increas- 
ing that  altitude  by  the  difference  between  the  refraction  and  parallax  cor- 
responding thereto :  then,  the  difference  between  the  refraction  and  paral- 
tax  answering  to  that  a;ugmented  altftuJe,  witf  be*the  reduction  of  the  true 
altitude. 

MxampU. 
« 
Let  the  true  altitude  of  thd  sun's  centre  be  5  degrees ;  required  the 
reduction  to  apparent  altitude  } 

Sun's  true  altitude  .  .  .  .  5?  OC  OT 
ftefract.Tab. VlIL=9^54q  ..„  .  .,  .,, 
Faral. table VII.      0.   9  J^*- +  »-^*- 

Augmented  altitu<le  •    •    5?  9' 451,  refrac.  ans.  to  which  is  9^38? 

and  parallax    •    0.  ^ 

Bequir^  reduction  =:    •••••*••••••••    tt'291 

Digitized  by  VjOOQ IC 


Ti|e  cprr^ctiQn  for  reduciing  a  itar'a  tni^  alUMe  to  iU  upparenl^  la 
obtained  ip  the  same  manner,  omitting  what  relates  to  p^n^llax.  Thv^s  if 
the  true  altitude  of  a  star  be  8  degrees^  and  the  corresponding  refraction 
9"29%  (heir  $uni/  viz.^  8^6;29r  will,  be  the  augmented  altitude;  diQ 
refraction 'answering  to  this  is  6^24^,  which,  therefore,  is  the  reduction  of 
the  true  to  the  apparent  altitude  of  the  star* 

The  correction  for  reducing  the  true  altitude  of  the  moon  to  the  appa- 
rent, is  found  by  diminishing  the  true  altitude  by  the  difference  between 
the  parallax  and  refraction  answering  thereto ;  then  the  difference  between 
the  parallax  and  refraction  corresponding  to  the  altitude  so  diminished, 
will  be  the  reduction  of  the  true  to  the  apparent  altitude.    As  thus  s —    • 

Let  the  true  altitude  of  the  moon^s  centre  be  10  degrees,  and  her  hori* 
Bontal  parallax  57  minutes  )  required  the  reduction  to  apparent  altitude  ? 

Moon's  true  altitude     •     •    .    •    ,     10?  0^  0^     Log.  secant  10.0066 
Do.  horizontal  parallax     ,     .    .    •  57-0^     Propor.log.    0.4994 


l^iraHaK  in  altitude! 56^  8r    Propor.log.    0.506Q 

Refrac.  to  altitude  16?,  Table  VIIL  =:         5. 15 

jOiffcrence  between  parallax  and  refrac.  s  50'.  53^ 


Dfanhiished  altitude 9?  9f  7^    Log.  secant  10.0056 

Horizontal  parallax 57-0      Proper,  lo^.    0*4994 

Parallax  ip  altitude    *.,..«  56 U6?    Propor^log*    0,5050 

Refrac,  to  dimini8he4  alt.  Table  VIII.  5.42 

Difference    «    •    •    •    •    ^    •    ,    ^  50^34^  j  whiph^  therefore^  U 
the  required  reduction. 


AuxiKary  Angles. 

Since  the  solution  of  the  Problem  for  finding  the  longitude  i^t  sea,  hy 
celestial  observation,  is  very  conaideraMy  abridged  by  the  ii^trodu^pn  <^ 
an  auxiliary  angle  into  the  openition;|  the  true  central  dbtance  being  hence 
f  ^adily  determine  to  the  nearest  second  of  a  degree  by  the  simple  additi(H;i 
of  fiv^  natur^  yer9ed  ^iiies ;  this  Table  has,  therefore,  been  computed  ',  and 
to  render  it  as  convenient  as  possible,  it  is  extended  to  every  tenth  minute 
ff  ih$  m.oon's  apparent  altitude|  and  to  each  minute  of  ly^  I^wmqiIM 


Digitized  by 


Google 


p«fall»}  ivilh  proportion^}  parts  adapted  to  the  iatenufdif^U  minvtei  <if 
altitude,  and  to  the  seconds  of  horizontal  parallax* 
This  Table  was  calculated  in  the  following  manner  :-r^ 
To  the  moon's  apparent  altilude  apply  the  correction  from  Table  XVIIL, 
^^d  the  sum  will  be  hH^r  true  al|i|«de.  j  fron\  thf  Ipg.  tsosiM  of  whioh  (^ 
indax  being  augmented  by  10)  subtract  the  log.  cosine  of  her  apparent 
altitude,  and  Ibe  remainder  will  be  a  log.^  which,  being  diminished  by  the 
canstaiH  Iqg.  .900910/  will  give  the  logarithmip  cosine  of  the  auxiliary 

Iiet  tha  mopn.'s  apparent  altitude  be  4  degrees,  and  her  horizontal  paral- 
lax  55  minutes ;  required  the  corresponding'  auxiliary  angle  ? 

Moon's  apparent  altitude  .     4?  0^  Of    Log.  cosine     •    9.998941 
Correction  fromTable  XVIII.  +  43..  2 

Moon-s  true  altitude    ;    .    4?43'  2r     Log.  cosine    •    9.99atS27 


tl    W     MlllliM 


Log.     .    .    ,    9.999)586 
CotMtantli^,      0.S0091Q 


ij  I    nwinyn 


AiixiHar7angk,a8requbed60M:2ir»L<i(.eMiM    .'^.Omii 

The  correction  of  the  auxiliary  angle  for  the  sun's  or  star's  appartot 
altitude,  given  at  the  bottom  of  each  page  of  tbe  Talile,  was  oompated  by 
the  following  rule — ^viz.  •     . 

FVom  the  log.  cosine  of  the  sun's  or  star's  true  altitude  subtract  dia  logk 
coane.  of  the  apparent  altitude^  and  And  the  diffsfeuc^  between  the 
remainder  and  the  constant  log.  .  OOOlSO.f  Now  thia  ^iftience,  heiag 
subtracted  from  the  lo^.  cosine  of  60' degrees,  will  leave  the  log.  cosine  of 
an  arch]}  the  diiference  between  which  and  60  degrees  Drill  be  the  eovrec- 
tion  of  the  auxiliary  angle  depending  on  the  apparent  altiUide  of  the  sun 
or^tar. 

I^t  thf^  sun's  or  star^  apparent  altitude  be  3  degrees;  required  the 
^rrection  of  the  auxiliary  angle  ? 

*  TUs  u  the  lof.  se^uit,  1ms  ndiutt  of  (^  decrees  dinuaithed  by  •  000 120t  the  differf nee 
^etweea  tbelfif .  oMines  of  a  star's  true  and  apparent  altitude  betwixt  30  and  90  degrees. 

t  This  is  the  difference  betweeo  the  lo^.  cosines  of  a  star's  true  and  apparent  altiluda, 
hetvcco  3#  end  9(^degrecs« 


Digitized  by 


Google 


44  JhtitAtVnoV  ASB  tSSJt  OP  <rHB-  TABI JM. 

Sun's Apptirent  altitude      .    .    .  '.    3?  0'  Or    Log.  cosine  9. 999404 
Refract. Table VHI.  WMI)  ,.-,  i^'o-r* 

Pardlax.TableVH.         9  (  ^ffe^'x^^-l^^/r 

<       II  ■    li 

8un'^  true  altitude    .    .    •    ^    .    «    2?45;33f     Log.  cosine  9. 999497 


Remainder  0.0Q0093 
Const,  log.  0.000120 


Difference   0.000027 
60?  0'.  01    Log.  cosine  9. 69S970 


Arch    •    .    .    .    .  \.    •    .    *    .    60.   0.   8      Log.  cosine  9. 698943 


Difference  .•••••...    0?  0^  8T ;  which^  thereforCi  is  the 
required  correction  of  the  auxiliary  angle. 

In  this  Table  the  auxiliary  angle  is  giveu  to  every  tenth  minute  of  the 
moon's  apparent  altitude  (as  has  been  before  observed)  froift  the  horizon 
to  the  zenith^  and  to  each  minute  of  horizontal  parallax.  The  proportional 
part  for  the  excess  of  the  given,  above  the  next  less  tabular  altitude  is  con- 
tained in  the  right-hand  column  of  each  page ;  and  that  answering  to  the 
Seconds  of  )>arallax  is  given  in  the  intermediate  part  of  the  Table.  The 
correction  depending  on  the. sun's  or  star's  apparent  altitude  is  placed  at 
the  bottom'  of  the  Table  in  each  page. 

As  the  size  of  the  paper  would  not  admit  of  the  complete  insertion  of 
the  auxiliary  angle,  except  in  the  first  vertical  column  of  each  page  under 
or  over  53  C  ;  therefore,  in  the  eight  following  columns,  it  is  only  the  excess 
of  the  auxiliary  angle  above  60  degrees  that  is  given :  hence,  in  taking  out 
the  auxiliary  angle  from  those  columns^  it  is  always  to  be  prefixed  with  60 
degrees. 

The  auxiliary  ang^e  is  to  he  taken  out  of  the  Table,  as  thus  :— 
..  Enter  the  Table  with  the  moon's  apparent  altitude  in  the  left-hand 
colump  of  the  page,  or  the  altitude  next  less  if  there  be  any  odd  minutes^ 
opposite  to  which  and  under  the  minutes  of  the  moon's  horizontal  parallax 
at  top,  will  be  found  the  approximate  auxiliary  angle. 
,  Enter  the  cpmpartment  of  the  ^'Proportional  parts  to  seconds  of  paral- 
lax," abreast  of  the  approximate  aiixiltary  angle,  with  the  tenths  of  seconds 
of  the  moon's  horizontal  parallax  in  the  vertical  column,  and  the  units  a£ 
the  top ;  in  the  angle  of  meeting  will  be  found  a  correction,  which  place 
under  the  approximate  auxiliary  angle ;  then  enter  the  last  or  right-hand 
column  of  the  page  abreast  of  where  the  approximate  auxiliary  angle  was 
found,  or  nearly  so,  and  find  the  proportional  part  corresponding  to  th« 


Digitized  by 


Google 


PESCRIPTIOM  AND  U8B  OF  THIt  TJkBUm^  4$ 

odd  minutes  of  altitude,  which  place  under  the  formen  To  these  three 
let  the  correction,  at  the  bottom  of  the  Tfible,  answering  to  the  sun's  or 
star's  apparent  altitude,  be  applied,  and  the  eum  will  be  the  correct 
auxiliary  angle. 

JBxilmple. 

Let  the  moon's  apparent  altitude  be  2o?37^9  the  sun's  apparent  altitude 
58?20'9  and  the  moon's  horizontal  parallax  59U7^;  required  Uie  cor* 
responding  auxiliary  angle  ? 

Aux.  angle  aiis.  to  moon's  wp.  alt,  25^30%  and hor. par.  59'  is 60?13'47  v 
Proportional  parts  to  47  seconds  of  horizontaf  parallax  is     •  12  ^ 

Proportional  part  to  7  minutes  of  altitude  is 4 

ConrectioQ  corresponding  to  sun's  ajpp.  alt.  (5  8?  20^)  is    |    ^      .  4 

Auxiliary  angle^  as  required  •    •    •    • 60fl4'  7^ 


Table  XXI. 

^rr^ciioii  of  the  Auxiliary  Angle  vohen  the  Maon^s  DUtance  from  a 
Planet  if  observed* 

The  argi*ments  of  this  Table  are,  a  planet's  apparent  altitude  in  the  left 
or  right-hand  column^  and  its  horizontal  parallax  at  top ;  in  th^  angle  of 
meeting  stands  the  correction,  which  is  always  to  be  applied  by  addition  to. 
the  auxiliary  angle  deduced  from  the  preceding  Table  :  hence,  if  the  appa-< 
rent  altitude  of  a  planet  be  26  degrees,  and  its  horizontal  parallax  23 
seeoncb^  the  correction  of  the  auxiliary  angle  will  be  6  seconds,  additive. 

This  Table  was  calculated  by  a  modification  of  the  rule  (page  43) 
for  computing  the  correction  of  the  auxiliary  angle,  answering  to  the  sun's 
or  star's  apparent  altitude ;  lis  thus  :— *- 

To  the  logarithmic  secant  of  the  planet's  apparent  altittde,  add  the 
logarithmic  cosine  of  its  true  altitude,  and  the  constant  logarithm. 
9. 698850;*  and  the  sum  (abating  20  in  the  index)  will  be  tlie  logarithmic 
cosine  of  On  arch ;  the  difference  between  which  and  60  degrees  will  be  the 
required  correction. 
I  ■■  .        —  *  ■  III.  1       -  ' 

*  Tbif  is  tlve  \og»  cosine  of  60  dcffrecs  diminished  by  .  000120,  the  diflference  between  the 
las*  cosines  of  the  true  and  SpfKUrent  altitude  of  #.fi;^e4  9^  between  30  and  M  deems^ 


Digitized  by 


Google 


A6  ]>ft6clitM6»  kM  mic  6t  tHS  tA)StJI^« 


ReampU* 

Let  the  apparent  altitude  of  a  -planet  be  30  degrees^  and  Its  horisOQtel 
parallax*  23  seconds^  required  the  correction  of  the  auxiliary  angle  ? 

Planet's  apparent  altitude    .     .    .    30?  0^  Or     Log.  secant  10. 062469 

ParalUx,TabkVL0.20  } 

■  Const  log.    9.698850 

True  altitude  of  the  planet    .    .    •    29?58'.42f     Log.  cosine  9. 937626 

ktchtA       •••..•..    60?  0'  7^= W  cosine  9.698945 

60.  0.  0 


Difference      i    i  \    .    •    •    •    .    0?  0'  7^  J  which  1§  the  requited 
correction. 


Table  XXIL 

Error  ariring  from  a  Deviatimi  o/  one  Minute  in  the  ParaUelism  of 
the  Surfaces  of  the  Central  Mirror  qf  the  Grcular  Jnstrument  of 
B^fiecHofi. 

This  Tahle  contiuns  the  error  of  observation  arising  from  a  deviation  of 
ona  nutate  in  the  parallelistn  of  Ihe  surfaces  of  the  central  mirror  of  the 
reflecting  eircley  the  axi^  of  the  telescope  being  supposed  to  make  an  ahgle 
•f  80  degrees  iftdth  the  hori^eon  mirfor }  it  i^  very  useful  in  finding  the 
verifieatioti  of  the  parallelism  of  the  surfaces  of  ihe  central  tnirror  in  the 
^fleeting  circle,  or  of  the  index  glass  in  ttie  sextant  $  as  thus  f-^ 

Let  the  instrument  be  carefully  adjusted,  and  then  take  four  or  live 
observations  of  the  angular  distance  between  two  well^d^ed  objects, 
whose  distance  is  not  less  Ihan  100  degrees }  the  sum  of  these,  divided  by 
their  number;^  will  be  the  mean  observation.    Then, 

Take  out  ik  central  mirror,  and  turn  it  b6  that  the  (fdge  which  Was 
before  uppefmoM  may  ndw  be  downwards,  or  next  the  plane  6f  the  instru- 
ment ;  rectify  its  position,  and  take  an  equal  ndmber  of  observation!!  of  th^ 
angular  distance  between  the  same  two  objects,  and  find  their  mean,  as 
before :  now,  half  the  difference  between  the  mean  of  these  and  that  of  the 
former,  will  be  the  error  of  the  mirror  answering  t6  the  observed  Migles 
If  the  first  mean  exceeds  the  second,  the  error  is  subtractive  |  otherwise 
additive  i  the  mirror  being  i&  its  first  or  natutal  position.    Hemse^  if  the 


Digitized  by 


Google 


i>Js8CRirridi^  AM  t^BB  ol^  tHft  tabus.  47 

mean  of  the  first  set  of  obsetvations  be  I15?9^40?,  atid  that  of  the  second 
114?59'20r,  half  their  difference,  viz.,  ICSOf  4-  2  =2  40r,  will  be  the 
error  of  the  obsenred  angle,  and  is  subtractive;  because*  the  first  mean 
angular  distance,  or  that  taken  with  the  mirror  in  its  natural  position,  \i 
greater  than  the  second,  ot  that  taken  With  the  mirror  inverted. 

Having  thus  determined  the  error  of  the  observed  angle,  that  aiisW^ring 
to  any  giveii  angle  may  be  readily  computed  by  means  of  the  present  TVtble, 
as  follows  :^— - 

Enter  the  left-hand  column  of  the  Table  with  the  angular  distance,  by 
which  the  error  of  the  central  mirror  was  determined;  and  take  out  the 
corresponding  number  from  the  adjoining  column,  or  that  marked  ^Ob- 
servation to  the  right  ;^^  in  the  same  manner  take  out  the  number  answer- 
ing to  the  given  angle ;  then, 

To  the  arithmedcal  complement  of  the  proportional  log.  of  the  Jirsi 
number,  add  the  proportional  log.  of  the  second,  and  the  proportional  log. 
of  the  observed  error;  the  sum  of  these  three  logs.,  rejecting  10  from  the 
tndex^  urill  be  the  proportional  log.  <rf  the  error  answeriDg  to  such  given 
angle. 

jfirample. 

Hanng  found  the  error  snAng  from  h  Ahhtt  of  parallelism  in  the  centra! 
mirror,  at  an  angle  of  115  degrees,  to  be  40  seconds  subtractive;  required 
the  error  corresponding  to  an  angle  of  S5  degrees } 

Obs.  ang.1 15  deg.  opp.  to  which  is  3 '23?  Arith.  cdMp.  prop,  log.^8. 2741 
Given  ang.  85  deg.  opp*.  to  which  is  1 H5  ?  Propor.  log.    .    •    =:     2. 1584 

Observed  error  of  central  mirror    0.40    Propor.log.    .    •    =:    2.4313 

■  •  — 

Required  error  =  •    •    .    •     -  O'lSr  =  Propor.  log.   •    =:    2.8638 


Tabui  XXIII. 

JError  qfObiervatim  ariringfrom  an  IneUnaHon  o/tke  Line  qfColUmoh 
Hon  to  the  Plane  qfthe  Sextant,  or  to  tliat  qfthe  circular  Imtrument 
of  Refleetion. 

If  the  line  of  sight  is  not  parallel  to  the  plane  of  the  instrument,  the 
tngle  measured'  by  such .  instrument  will  always  be  greater  than  the  true 
angle.  This  Table  contains  the  error  arising  from  that  cause,  adapted  to 
the  most  probable  limits  of.  the  inclination  of  the  line  of  coUimation,  and 
to  any  angle  under  120  degrees :  hence  the  arguments  of  the  Table  are^ 
tbe  oibserved  angle  in  the  left*hand  column^  and  the  inclination  of  the  liiie 


Digitized  by 


Google 


48  ;dbscription  and  usb  of  thk  tablbs. 

of  collimation  at  top;  oppoSte  the  former,  and  under  the  latter,  will  be 
fbund  the  corresponding  correction. 

Hius,  if  the  observed  angle  be  60  degrees,  and  the  inclination  of  the 
)ine  of  collimation  30  minutes,  the  corresponding  error  will  be  13  seconds. 
The  error  or  correction  taken  from  this  Table  is  always  to  be  applied  by 
m^btracHon  to  the  observed  angle. 

Tlie  corrections  in  this  Table  were  computed  by  the  following 

nule. 

To '  the  log.  sine-  of  half  the  observed  angle,  add  the  log.  cosine  of  the 
inclination  of  the  line  of  collimation ;  and  the  sum,  rejecting  10  in  the 
index,  will  be  the  log.'sine  of  an  arch.  Now,  the  difference  between  twice 
(his  arch  and  the.  observed  angle,  ivill  be  the  error  of  the  line  of  collimation. 

Example^ 

Let  the  observed  angle  be  €0  degrees,  and  the  inclination  -ot  the  line  of 
collimation  1?30^ ;  required  the  corresponding  correction  ? 

Obs.  angle  80  degs.  and  80?  -^  2  =  40?  Log.  sine  9. 808068 
Inclinatofline  ofcoUim,    ,    .    \    .    .     1?30'  Log.  cosine  9.999851 


Arch  ~  39?59:   11  -  Log.  sine     9.807919 


Twice  the  arch  =  79?58'  21 


bitference    0?  1^58?,  which,  therefore,  is  the   require4 
error. 


Table  XXIV. 

LogdrUhmic  Difference. 

This  Table  contains  the  logarithmic  difference,  adapted  to  every  tenth 
minute  of  the  moon's  apparent  altitude  from  the  horizon  to  the  zenith,  and 
to  each  minute  of  horizontal  parallax.  The  proportional  part  for  the 
excess  of  the  given  above  the  next  less  tabular  altitude,  is  contained  in  the 
right-hand  compartment  of  each  page,  and  that  answering  to  the  second^ 
of  parallax  is  given  in  the  intermediate  part  of  the  Table. 

As  the  size  of  the  paper  would  not  admit  of  the  complete  insertion  of 
the  logarithmic  difference,  except  in  the  first  vertical  column  of  each  page^ 
under  or  over  53^,  therefore  m  the  eight  following  columns  it  is  only  the 


Digitized  by 


Google 


BBaCRIFTION  AND  ttSB  OF  TRB  TA9I3S«  49 

foHr  last  figures  of  the  logarithmic  difference  that  Bie  given :  hence,  in 
taking  out  the  numbers  from  these  columns,  they  are  always  to  be  prefixed 
by  the  characteristic,  and  the  two  leading  figures  in  the  first  column.  The 
logarithmic  difference  is  to  6e  taken  out  in  the  rollpwing  manner. 

Bnter  the  Table  with  the  moon's  apparent  altitude  in  the  If ft-hand 
column  of  the  page,  or  the. altitude  next  lets  if  there  be  any  odd  minutes^ 
opposite  to  which^  and  under  the  minutes  of  the  moon's  horizontal  paral-* 
lax,  at  top,  will  be  found  a  number,  which  call  the  appraximate  logarUhmic 
^Sfference. 

Enter  the  compattment  of  tha  ^  Proportional  parts  to  seconds  of  paral^ 
lax,"  abreast  of  the  approximate  logarithmic  difference,  with  the  tenths  of 
seconds  of  the  moon's  horizontal  parallax  in  the  vertical  column,  and 
the  units  at  the  top,  and  take  out  the  corresponding  correction.  Enter  the 
right-hand  compartment  of  the  page,*  abreast  of  where  the.  approximate 
logarithmic  di&renoe  was  found,  or  nearly  so,  with  the  odd  minutes  of 
altitude,  and  take  out  the  corresponding  correction,  wliich  place  under  the 
former.  £nter  Table  XXVV  or  XXVI.,  with  the  sun's,  star's,  or  planet's 
apparent  altitude,  apd  take  out  the  corresponding  correction,  which  also 
place  under  the  former.  Now,  the  suin  of  these  three  corrections -being  , 
taken  from  the. approximate  logarithmic  difference,  will  leave.tho  correct 
I<^;arithmic  difference,  / 


ExampU  1. 

Let  the  moon's  apparent  altitude  be  19?25C^  her  horizontal  paralW 
60' 38^,  and  the.  siin's  apparent  altitude  33  degrees;  required  the  loga- 
rithmic difference  7 

Log.  difference  to  app.  alt.  19?20',  and  hor.  par.  60'  is    9. 997669 
Propor.  part  to  38  seconds  of  parallax  is      .     28*1 
Propon  part  to  5  minutes  ci  akiti|de  is,      «   .«     1 1  >sum  =  —  49 
Cor.  from  Tab.  XXV.  ans.  to  sun's  apparent  alt.  is     1 0  J 


Logarithmic  differedce,  as  required     .»,;«;    4    9.997620 


*  lo  taking  out  the  correctioa  correspondiDg  to  the  odd  minatei  of  altitude  in  this  com- 
Tartaient,  attention  is  to  be  paid  to  the  moon's  horizontal  parallax  :  thus,  if  the  paratlax 
he  between  &3'  and  h^^  tlie  correction  is  to  be  taken  out  of  the  first  column,  or  that  adjoin- 
ing the  minutes  of  altitude ;  if  it  be  between  56'  and  59',  the  correction  is  to  be  taken  out  of 
the  second,  or  middle  column ;  and  if  it  be  between  59^  and  62'^  the  correction  is  to  be 
taken  out  of  the  tUfd^  or  last' column. 


Digitized  by 


Google 


80  MsomiPTioK  Am>  utB  or  ran  mbum* 


Example  2. 

Let  the  moon's  apparent  altitude  W  63^87',  fcerhorirontal  parallax 
58U3r,  the  apparent  altitude  of  a  planet  85?  10',  and  its  horicontal 
parallax  237 }  required  the  logarithmic  diflerenee  ?  ' 

Log.  difference  to  i^par.  alt.  63?30C,  and  hor.  par.  58t,  is  9.993682 
Propor.  part  to  4a?  of  parallax  is       •    •    •    »    88^ 
Ph>por.  part  to  7  •  of  altitude  is      •    «    •    •    »      7  ?9ttm  =s-»l  18 
Cor.  from  Tab.  XXVI.  ans.  to  planet's  q>par.  alt;       28  j 

Iiogarithmic  difference,  as  required     •««••••    9.993504 
JBemarfc.-*The  logarithmic  difference  waa  oonpnled  by  the  f 

2bcb. 

To  the. logarithmic. secant  of  the  moon^s  apparent  altitude,  add  the 
^  logarithmic  cosine  of  her  true  altitude,  and  the  constant  log.  .000120;* 
the  sum  of  these  three  logs.,  abating  10  in  the  index,  will  be  the  loga- 
rithmic difference. 

Exafnple. 

Let  the  moon's  apparent  altitude  be  19?20t,  and  her  borisontal  parattaa^ 
60  minutes ;  required  the  logarithmic  difference  ? 

Moon's  apparent  altitude    .    .     19?20'  Or    Log.  secant      10.025208 
Correction  from  Table  XVIIL  5S«  56    .  Conataat  log.    0. 000120 

Moon's  true  altitiide  •    •    «    •    90.18.5S      Lof.eosbe       9.972341 


Logarithmic  difference,  as  re<}uired      •    ,    •    •    ,        •    «    9.997669 

*  The  <^lference  between  the  Io|*.  coiines  of  the  true  and  sppsrent  tlUtode  of  s  star 
betwixt  3Q  sad  90  degrees. 


Digitized  by 


Google 


SmCfUFTION  AND  ITM  OF  fHS  TABUS*  51 


Table  XXV. 

CcrrecAon  of  th^  Logarithmic  Difference. 

lliis  Table  is  divided  into  two  parts :  the  first,  or  left-hand  part,  con- 
tains the  correction  of  the  logarithmic  difference  when  the  moon's  distance 
from  the  sun  is  observed  j^and  the  second,  or  right-hand  part,  the  correc- 
tion of  that  log.  when  the  moon's  distance  from  a  star  is  observed.  Thus^ 
if  the  son's  apparent  altitude  be  35  degrees,  the  corresponding  correction 
will  be  11 ;  if  a  star's  apparent  altitude  be  20  degrees,  the  corresponding 
correction  will  be  1;  and  so  on.  These  corrections  are  always  to  be 
applied  by  mdiiriaction  to  the  logarithmic  difference  deduced  from  the 
preceding  Table. 

The  corrections  contained  in  this  Table  were  obtained  in  the  following 
naiiner,  viz. 

To  the  log.  secant  of-  the  apparent  altitude,  add  the  log.  cosine  of  the 
true  altitude;  aiidthesum,  rejecting  10.  from  the  index,  will  be  a  log. ; 
whieh  being  mibtracted  from  the  comtaat  log.  .000120^*  will  leave  the 
tabttlar  correctioR, 

Easamfle  1. 

Let  the  sun's  apparent  altitude  be  35  degrees  j  required  the  tabular  cor<« 
rection? 

Given  apparent  altitude      ss        85?  0'  Or  Log.  secant  =:  10.086635 
Rcfrac.  TaWe  VIII.    V.2V.\..^        \  aa 
Parallax  Table  Vll.        7   /^^^•=-"*-*^       • 

Siitiytrae  akituda      •    •    ..  .    84?58:46?  Log.  cosine    •    9.913474 

Sum     •    .    •    6.000109 
Constant  log.      0.000120 

Tabular  oonection^  as  required    .«••••••«•    0.000011 

BxampU2. 

Let  the  apparent  ^titude  of  a  star  be  10  degrees ;  required  the  tabular 
correction? 


•  aBtNi9te»iisce50. 
k2 


Digitized  by 


Google 


52  BBSCRIPTION  AND  USB  OF  THE  TABUS* 

Star's  apparent  altitude     10?  0 '  0^  Log-  secant  =10. 006649 
Refraction  Table  VIIL      -.5.15 

Star's  true  altitude    •    •    9.54.45     Log.  cosine  =  9.093467 

Sum=     .    .    0.000116 
Constant  log.     0.000120 

Tabular  oorrcctien,  IB  required 0.000004 


Tablb  XXVI. 

Correction  dfthe  Logarithmic  Difference  iohen  the  MomttB  jlKstance 
from  a  Planet  is  observed. 

The  arguments  of  this  Table  are^  the  apparent  altitude  of  a  planet  in 
the  left  or  right-hand  marginal  column,  and  it9  horizontal  parallax  at  top; 
in  the  angle  of  meeting  stands  the  corresponding  correction,  which  is  to  be 
applied  by  subtractum  to  the  logarithmic  difference  deduced  from  Table 
XXIV.,  when  the  moon's  distance  from  .a  planet  is  observed.  Hence,  if 
the  apparent  altitude  of  a  planet  be  20  degrees,  and  its  horizontal  parallax 
21  seconds,  the  corresponding  correction  will  be  16  subtractive,  and  so  on. 

This  Table  was .  computed  by  the  rule  in  page  51,  under  which  the 
correction  corresponding  to  the  sun's  apparent  altitude  in  Table  XXV. 
was  obtained,  as  thiis  :^- 

Let  the  apparent  altitude  of  a  planet  be  23  degrees,  and  its  horizontal 
parallax  21  seconds;  required  the  correction  of  the  logarkhmie  difference  ? 


Planet's  apparent  altitude    ...    23?  0'  0^  Log.  secant  10.035974 
Refrac  Table  VIIL  =  2^  14^  \  ,.^  _        •   -  . 
Parallax  Table  VL   ==     20r/      '""^     •   "* 


Planet's  true  altitude    •    .    .    ..  22?58:  6?  Log.  cosine    9.964128 


Sum     .    •    0.000102 
Constant  log.  0. 000120 


Correction  of  the  logarithmic  difference,  as-required     .    .    .    0.000018 

/Google 


Digitized  by ' 


PEfCAIFTION  AND   VSB  OF  THB  TAfiLBSt 


53 


Tablb  XXVII. 


Natural  Verged  Sines,  and  Natural  Sines^ 

Since  the  methods  of  computing  the  true  altitudes  of  the.  heavenly 
bodies^  the  apparent  time  at  ship  or  place^  and  the  true  central  distance 
between  the  moon  and  sun,  or  a  fixed  star,  are  considerably  facilitated  by 
the  ^plication-of  natural  versed  sines,  or  natural  sines,  this  Table  is  giv^; 
which,  with  the  view  of  rendering  it  generally  useful  and  convenient,  is 
extended  to  every  tenth  second  of  the  semicircle,  with  proportional  parts 
corresponding  to  the  intermediate  seconds;  so  that  either  the  natural 
versed  sine,  natural  versed  sine  supplement,  natural  co-versed  sine,  natural 
sine  or  natural  cosine  of  any  arch,,  may  be  readily  taken  out  at  sight. 

The  numbers  expressed  in  this  Table  may.be  obtained  in  the  following 
manner : — 

Let  ABC  represent  a  qua* 
drant,  or  the  fourth  part  of  a 
circle;  and  let  the  radius  CB 
=  unity  or  1,  be  divided  into 
an  indefinite  number  of  decimal 
parts:  as  thus,  1.0000000000, 
&c.  Make  B  D  =  the  radius 
CB;  and  since  the  radius  of  a 
circle  is  equal  to  the  chord  of 
60  d^rees,  the  arc  BD  is  equal 
to  60  degrfies  i  draw  DM,  the 
sine  of  the  arc  BD,  and,  at 
right-angles  thereto,  the.  cosine 
]>B:  biaect  the  arcBD  in  F, 
and  draw  FN  and  FG  at  rights 
angles  to  each  other;  then  will 
the  former  represent  th^  sine, 
and  the  latter  the  cosine  pf  the 
arc  BF  =  30  degrees:  bisect 
BF  in  H;  then  HOwill express 
the  sine,  and  H I  the  cosine  pf 
the  arc   BH    ss  ,15  dctgrees* 

Proceeding  in  thi^  manner,  after  12  bisections,  we  come  to  an  arc  of 
0?0' 52^44^3  V45^r,  the  cosine  of  which  approxin^ates  so  very  clcsely  to 
the  radius  C  B,  that  they  may  be  considered  as  being  of  equal  value.  Now, 
the  absolute  measure  of  this  arc  may  be  obtained  by  numerical  calculation, 
us  foUdws^  vias. 


Digitized  by 


Google 


54  DtSCklPTION  AKD  tfSB  OV  THX  TABLBS* 

Because  the  chord  line  B  D  is  the  side  pf  a  hei^^n,  inscribed  In  a  circle^ 
it  is  the  subtense  of  60  degrees,  and,  consequently,  equal  to  the  radius  C  B 
(corollary  to  Prop,  15,  Book  IV.,  of  EucUd) ;  wherefore  half  the  radius  B  S 
=  BM,  will  bp  the  sine  of  30  degrees  =  FN,,  which,  therefore,  is 
.  5000000000.  Now,  having  found  the  sine  of  80  degrees,  its  cosine  may 
be  obtained  by  Euclid,  Book  L,  Prop.  47 :  for  in  the  right-angled  triangle 
FNC,  thehypothenuse.FC  is  given  =  the  radius,  or  !•  0000000000, 
and  the  perpendicular  F  N  =  half  the  raditis,  or  •  6000000000> 
to  find  the  base  C  N  =  the  cosine  G  F ;  therefore 

V  FC  X  FC-FN  X  FN  =  CN  .8660254037,  or  its  equal  OF; 
hence  the  sine  of  30  degrees  is  .  5000000000,  bnd  its  cosine  .  8660254037- 
Again, 

In  the  triangle  FNB,  the  perpendicular  FN  is  given  =  •  5000000000> 
and  the  base  CB-CN=:NB=.  1339745963,  to  find  the  hypothenuse 
B  F :  but  half  the  side  of  a  polygon,  in9cribed  in  a  circle,  is  equal  to  the 
sine  of  half  the  circumscribing  arc ;  therefijrc  its  half,  BT  =  H  O,  will  be 
the  sine  of  the  arc  of  15  degrees :  hence  '/FN  x  FN  +  NB  x  NB 
=  BF  .51763809025  the  half  of  which,  viz.,  .  2588190451,  is  therefore 
equal  to  BT,  or  to  its  equal  HO,  the  sine  of  15  degrees,  and  from  which 
its  cosine  HI  maybe  easily  obtained;  for,  in  the  triangle  COH|  the 
hypothenuse  C  H  is  given  = '  1^  0000000000,  and  the  perpendicular  H  O 
=  .  2568190451,  to  find  the  base  C  O  z=  the  cosine  H  L    Now^ 

^/CH  X  CH- HO  x  HO  =  CO  .9659258263  =  the  cMitine  HI; 
hence  the  sine  of  15  degrees  is  .  2588190451,  and  its  oosiiie  » 965925826S. 

Thus  proceeding,  the  sine  of  the  12th  bisection,  viz^  52r44rS^MS'?^9 
will  be  found  =  .  0002556684;  And  becaoae  smidl  Arc^  are  very  neariy 
as  their  corresponding  sines,  the  measure  of  1  minute  may  be  easily  deduced 
from  the  sine  of  the  small  arc,  or  12th  bisection  determined  as  above;  for^ 

As  die  arc  df  52f44r3''f45^'^  is  to.an  are  of  1  minute,  so  is  the  sine  of 
the  former  to  the  sine  of  the  latter  :  that  is,  as  52r44r8V45^^  ;  K  :t 
.0002556634  :  i  0002908882;  which,  therefore,  is  the  sine  of  1  minute^ 
the  cosine  of  whicb  is .  9999999577 ;  but  this  approximates  so  very  dosely 
to  the  radius,  that  it  may  be  esteemed  as  being  aetually  equal  to  H  m  all 
calculations ;  and  hence,  that  the  cosine  of  1  ^  is  1 .  0000000000* 

'Now,  having  thus  found  the  sine  and  cod&e  of  one  tnifiute,  the  sihes  of 
every  minute  in  the  guadrant  may  be  obtained  by  the  following  rule ;  viz. 

As  radius  is  to  twice  the  cosine  of  1  minute,  so  is  the  sine  of  a  mean  are 
to  the  sum  of  the  sines  of  the  two  equidistant  extremes ;  from  which  let 
either  extreme  be  subtracted,  and  the  remainder  vrill  be  the  sine  of  the 
other  extreme :  as  thus^ 


Digitized  by 


Google 


lUMeitapnoH  amo  va  <a  nm  tabu»«  5S 

Tb^Snd'  tlu  Siae  ^the  Arc  afTMxmiet, 

As  radius  =1:2:!  .0002908882  to  .0005817764,  and  .0005817764 

-  .0000000000  =  .0005817764;  which,  thiurtfore,  is  the  sine  of  the 
arc  of  2  nunutes. 

lb  fi»A  th«  <9iM  ({f  3  Mtmfev. . 

Aamdin  =  1:2::  .0005817764  to  .001 1635528,  and  .0011635528 
* .  0002908882  =  .  0008726646 }  which,  theiefore,  is  the  sine  of  an  are 
of  S  minwUa. 

As  nuiiuB  =  1:3::.  0008726646  to  .  0017453292,  and .  0017453292 

-  .0005817764  =  ^0011635528;  which,  therefore,  is  the  sine  of  the 
arc  of  4  minutes. 

To  JmA  the  Sine  ofi  Minutes. 

As  radins  =;  1  : 2::  .0011635528  to  .0023271056,  and  .0023271056 

-  •  008726646  s  .  0014544407  i  which,  therefore,  is  the  sine  of  the  arc 
of  5  minutes. 

In  this  manner,  the  sines  may.  be  found  to  60  degrees;  from  which,  to. 
the  end  of  the  quaidrant,  they  may  he  obtained  by  addition  only;  for  the 
dne  of  an  arc  greater  than  60  degrees;  is  equal  to  the  sine  of  an  arc  as 
inuch  less  than  60,  augmented  by  the  sine  of  the  excess  of  the  grven  are 
above  60  dej^ees :  thus. 

An  the  sines  being  fomid  to  60  degrees;  required  the  sine  of  61  degrees)   ' 


Aifalioiiw^Sine  of  699s .  8571673,  and  sim  of  1?  r=  .  0174524 ;  their 
rnns  •8746197s«lueh,  timsfore,  iatbesineof  61  d^ees^  as  required* 
Agdn, 

All  the  sines  being  found  to  60  degrees ;  required  the  sine  of  62 
degrees? 

iSbfefioii.— Siaeof  58?  =  . 8480461,  and  sine  3?  sr  . 0348995 ;  their 
=  .  8829476 ;  wUeb,  therefore,  is  the  sine  of  62  dqirees,  |»  required. 


Now,  liie  nstBBal  sines  beiiig  thus  found,  the  natural  versed  rines^  natu^ 
nd  tangents,  and  natnral  aecants,  may  be  readily  deduced  therefrom^ 
agreeahiy  to  die  tviacifiks  of  aimifair  triangles,  as  demonatrated  in  Euclid^ 
Book  VI^  Prop.  4.    I^ 

Digitized  by  VjOOQK 


56  2>B8CAIPTION  AND  USK  OF  TU£  TABUS. 


To  find  the  NaUtral  Versed  Sine  of  30  Degrees  =  N  B,  in  tJie  Diagram. 

Since  the  versed  aine  of  aif  arc  is  represented  by  that  part  of  the  diame- 
ter which  is  contained  between  the  sine  and  the  arc  ;  therefore  N  B  is  the 
versed  sine  of  the  arc  B  F^  which  is  the  arc  of  30  degrees ;  and  since  the 
versed  sines  are  measiired  upon  the  diameter^  from -the  extremity  B  to  C 
continued  to  the  other  extremity,  the  natural  versed  sines  under  90  degrees  * 
are  expressed  by  the  difference  betweeil  the  radius  and  the  cosine,  and 
those  above  90  degrees  by  the  sum  of  the  radius  and  the  sine :  henc^,  the 
radius  C  B  1, 0000000  -  the  cosiqe  FG,  or  its  equal  N  C  .  8660254  = 
^B  p  13S9746;  whicb^  therefore,  U  the  natural  versed  sine  of  30  degrees. 


To  find  the  Natural  Tangent  qfQO  degrees  zzBQ,in  the  Diagram. 

As  the  cosine  C  M  is  to  the  sine  D  M,  so  is  the  radius  CB  to  the  tangent 
BQ:  that  is, 

As  CM. 5000000  :  DM  .8660254  ::  CB  UOOOOOOO  :  BQ  = 
1  •  7320508  ]  which  is  the  natural  tangent  of  60  degrees. 


To  find  the  Natural,  Secant  of  60  Degrees  ==  C  Q,  in  the  Diagram. 

As  the  cosine  CM  is  to  the  radius  C  D^  so  is  the  radius  C  B  to  the  secant 
C  Q :  that  is. 

As  CM  .6000000  :  CD  KOOOOOOO  ::  CB  1.0000000  :  CQ  =: 
2. 0000000 ;  which  is  the  natural  secant  of  60  degrees;  Hence,  the  man- 
ner of  computing  the  natural  co^^tAngent  A  P,  the  natural  co-secant  C  P, 
and  the  natural  co-versed  sine  E A,  will  be  obvious.*  The  versed  sine  sup- 
plement of  an  arc  is  represented  by  the  difference  between  the  versed  sine 
of  that  arc  and  the  diameter  or  twice  the  r^ius:  thus,  the  versed. nne 
supplement  of  the  arc  B  F  is  expressed  by  the  difference  between  twice  the 
radius  C  B,  and  the  versed  sine  N  B ;  viz.,  twice  C  B  =  2. 0000000  -  N  B 
•  1339746  =  1. 8660254  ;  which,  therefore,  is  the  natural  versed  sine  sup- 
plement of  the  arc  B  F  or  the  arc  of  30  degrees,  and  so  of  any  other. 

Now,  the  natural  sines,  versed  sines,  tangents,  and  secants,  found  as 
above,  being  principally  decimal  numbers,  on  account  of  the  radius  being 
assumed  at  unity  or  1 3  therefore,  in  order  to  render  these  numbers  all 
affirmative,  they  are  to  be  multiplied  by  ten  thousand  millions  respectively; 
and  then  the  conmnon  logs,  corresponding  thereto  will  be  the  logarithmic 
sines,  versed  sines,  tangents,  and  secants^  which  are  generally  given  in  tho 
different  mathematical  Tables  under  these  denQQuiliations, 


Digitized  by 


Google 


DBSCJIIPnON  AND  0SS  OF  THJt  .TABLES*  57 


Of  m  Table. 

In  this  Tablei  the  natural  verted  sines  are  given  to  every  tenth  second  of 
the  semicircle ;  the  corresponding  arcs  being  arranged  at  the  top,  in  nume- 
rical order,  from  0  to  1 80  degrees.  The  natural  versed  sines  supplement 
are  given  to  the  same  extent;  but  their  corresponding  arcs  are  placed  at 
the  bottom  of  the  Table^  and  numbered, from  the  right  hand  towards  the 
left,  or  contrary  to  the  order  of  the  versed  sines.  Hie  natural  co-versed 
sines  begin  at  the  end  of  the  first  quadrant,  or  of  the  90tb  degree  of  the 
verted  sines ;  the  arcs,  corresponding  to  which  are  given  at  the  bottom  of 
the  page  and  numbered,  like  the  versed  sines  supplement,  towards  the  left 
hand  from  0  to  90  degrees,  and  then  continued  at  top  of  the  page  from  90 
to  180  degrees,  towards  the  right  hand,  until  they  terminate  at  the  90th 
degree  of  the  versed  sines,  where  they  first  began.  The  natural  sines  begin 
where  the  co*versed  sines  end ;-  viz.,  at  the  end  of  the  first  quadrant^  or  90th 
degree  of  the  versed  sines,  with  which  they  increase  by  equal  increments  ; 
the  arcs  corresponding  to  those  are  placed  at  the  top  of  the  page  to  every 
tenth  second  o(  the  quadrant,  the  90th  degree  of  which  terminates  with 
the  180th  of  the  versed  sines.  The  natural  cosines  b^gin  with  the  versed 
sines  supplement  5  the  ares  corresponding  to  which  are  given  at  the  bottom 
of  the  page,  being  numbered,  like  the  latter,  contrary  to  the  order  of  the 
versed  sines  and  natural  sineSf  to  every  tenth  second  from  0  to  90  degrees, 
or  to  the  end  of  the  first  quadrant  of  the  versed  shies,  thus  ending  whera 
the  co-versed  sines  begin. 

Note. — ^In  the  general  use  of  this  Table,  it  is  to  be  remarked,  that  the 
naturaVversed  sine  supplement,  natural  conversed  sine  Under  90  degrees,  or 
natural  cosine,  of  a  given  degree,  is  found  in  the  same  page  witK  the  next 
less  degree  in  the  column  marked  0?  at  top,  it  being  the  first  number  in 
that  column ;  that  answering  to  a  given  degree  and  minute  is  found  on  the 
same  line  with  the  next  less  minute  in  the  column  marked  60^^  at  the  bottom 
of  the  page ;  and  that  corresponding  to  an  arch  expressed  in  degrees, 
minutes,  and  seconds,  is  obtained  by  deducting  the  proportional  part,  at 
bottom  of  the  page,  from  the  natural  versed  sine  supplement,  natural  co- 
versed  sine  under  90  degrees,  or  natural  cosine  of  the  given  4icgree,  minute, 
and  less  tenth  second* 


Digitized  by 


Google 


58  PBSCRIFTIOM  AND  08B  OW  THB  TABLM* 

*  ♦      : 
PROBLEMS-  TO  ILLUSTRATE  THE  USE  OP  THE  TABLE. 

Probibic. 

To  find  the  Natural  Vened  Sine^  Naiwral  Versed  Sine  Si^^pkmeni, 
Naiumd  CSo'Vened  Sme^  Naharal  Sine^  and  Noiwrul  Coeine,  e/  an^ 
gwen  Jrch,  evpreeeed  in  D^eeSy  MmUes,  and  Seeondim 

AuLB. 

Enter  the  Table^  and  ftnd  the  natural  Yetaed  aine,  versed  sine  aopplement^ 
co-^rersed  aine,  naitaral  ame,  or  natural  cosine,  anawermg  to  the  ghren 
d^^ee^  minute,  and  next  leaa  tenth  aecond ;  to  which  add  the  proportional 
part  anawering  to  the  odd  aeeonds,  found  at  the  bottom  of  the  page,  if  a 
natural  versed  sine,  eo-versed  sine  ^bove  90?^  or  nolural  sine  be  wanted; 
but  subtract  the  proportional  parl^  if  a  versed  sine  supplement,  co-versed 
dne  under  90?,  or  nolifrol  come,  be  required  t  and  the  sum,  or  remainder^ 
will  be  the  natural  versed  sine,  natural  sine,  natural  versed  sine  supplenieat, 
oo^versed  sine,  or  naiwral  come,  of  the  given  arch. 

MxampU  1« 

Required  the.  natural  versed  sine,  versed  sine  supplement,  co-versed 
sine,  natural  me,  and  natural  cosuie,  answering  to  42?  12'36T? 

To  find  the  Natural  Versed  Sine  :-p 

Natural  versed  sine  to    42?12^90r  ^    .....    .    S5929S 

Proportional  part  to    •  6?  »    •    •    •    •    Add  20 


/KfcnArch   42?12:s<{r    Natural  versed  sine  B.85931S 

To  find  the  Versed  Sine  Supplement :— - 

Versed  sine  supplement  to    42?12'30r    « 1.740707 

Proportional  part  to    •    «  6r        ...  Subtract  20 


Given  arch  .    •    42?  12^36?      Versed  sine  sup.  =;  1.740687 

/Google 


Digitized  by ' 


DBsoBirriow  and  uib  of  thb  tablbs«  59 

To  find  the  Co-veraed  Sine  t«— 

Convened  sine  to  .    »    42?12'30r    ......    S28173 

Proportional  part  to^         ^  6T    «    •     Subtract  21 


Giveq.arch      42?12:36r       Co^vers.  sine  =  328151 

To  find  the  Natural  Sine  :—f  . 

Natural  sine  to      »    ,    42n2:30r    •    •    •    .    .    .    671828 
pR>porUonal  part  to  ^  6T    •    •    •    Add  21 

Given  arch.    42?12:3Qr        Nat.  sine   ::?    671849 

1V>  find  the  Natural  GMine  i'^ 

Natural  cosine  to      •    42?12:80r    .\ 740707 

Proportional  part  to  6^    •    .     Subtract  20 

Given  arch      42?  12:36r        Nat.  cosine  s  74068/ 

Example  2. 

Required  the  natural  versed  sine^  versed  sine  supplement,  co-versed 
one,  Mliiral  iiniy  and  natural  came,  answering  to  109?53!45?? 

To  find  the.NaUual  Vetted  Sine  :— 

Natural  versed  sine  to     109?53:40r  =    /  .    ...      1.340288 
nopoiiioDal  pan  to  &<r    is    #    •    •    Add  23 

■  ■  '  '■  ^ 

Given  arch.    .    109?53'45?    Nat.  versed  sine  =  1.340311    '■ 

To  find  the  Versed  Bine  Supplement :-« 

Ycisdlsfaiesup.to    109t53!40? 659712 

Phiportiond  pan  to  5'        i    .     Subtract  23 

GIvenarch  109^53:45?        Vers,  sine  sup,  s  659689 
To  find  Ae  Co-versed  Sine  :•« 

Co-vened  sine  to    lOQfSSUO?         ;    059679 

Fn^rtionalpartto  -    S?         •    «    .    .Add  8 

Oina  nvh     .MOtSSta?       Gs-verftd  idne  ss  059687 

Digitized  by  VjOOQ IC 


,00  DfiSCMVTlOS  AND  VSB  OF  THB  TA«UI9» 


To  find  the  Natural  Sine :— 

Natural  8i«c  to   •    70?6nOr    Sup.  to  l09?63:50-r       940305 
Proportional  part  to  5^  ,    •    •    •    Add  8 


SuppHement     70^6 1 15r  to  given  arch^  nat.  sine  =:  940313 

To  find  the  Natural  Coeine  :— 

Natural  cosine  to    70?6ClOr  Sup.  to  109?53:50r  .    340334 
Proportional  part  to        '    Sr        ;      •    .    Subtract  23 


Supplement      70?6 '.  1$  ^to  given  arcb^  nat.  co8ine=34031 1 

'JSfmariir.— -Since  the  natural  rines  and  natural  comes  are  hot  extended 
beyond  90  degrees^  therefore^  when  the  given  arch  exceeds  that  quantity, 
its  supplement,  or  what  it  wants  of  180  degrees,  is  to  be  taken,  as  in  the 
above  examyte.  And  when  the  given  arch  is  expressed  in  degrees  and 
minutes,  the  corresponding  versed  sine  supplement,  co-versed  sine  under 
90  degrees,  and  natural  cosine,  are  to  be  taken  out  ogreeably  to  the  not« 
in  page  57>  which  see. 


Probuoc  IL 

To  find  the  Arch  corresponding  to  a  gwen  Natural  Verged  &nef  Versed 
Sine  Supplemeiay  Conversed  Sine,  Natural  Sine,  and  Natural  Cosine*  . 

R0LB. 

Enter  the  Table,  and  find  the  arch  answering  to  the  next  less  natural 
versed  sine,  or  natural  sine,  hnt  to  the  next  greater  versed  sine  supfrie- 
ment,  co-versed  sine,  or  natural  cosine;  the  difference  between  which  and 
that  given,  being  found  in  the  bottom  of  the  page,  will  give  a  number  of 
seconds,  which,  being  added  to  the  arch  found  as  above,  will  give  the 
required  arch. 

Example  1. 
Re<)uired  the  arch  answering  to  the  patUral  versed  9me  363985  ? 

Digitized  by  VjOOff  IC 


DSSC&IPTION  AKD  tSB  OF  THB  TABLES.  6l' 

&6*/ion.— The  next  less  natural  versed  sine  is  363959,  corresponding  to 
which  is  50?30'  lO^j  the  difference  between  363959  and  the  given  natural 
versed  sine,  is  26;  corresponding  to  which,  at  the  bottom  of  the  Table,  is 
7^  which,  being  added  to  the  above-found  arch,  gives  50?30' 17%  the 
required  arch. 

Note. — ^The  arch  corresponding  to  a  given  natural  sine  is  obtained 
preciiely  in  the  same  manner. 


Example  2. 

Required  the  arch  corresponding  to  the  natural  versed  sine  supplement 
1,464138? 

SbZttJion^— The  next  greater  natural  versed  sine  supplement  Is  1 .  464 1 55 ; 
corresponding  to  which  is  62?20'40r;  the  difference  between  1.464155 
and  the  given  natural  versed  sine  supplement,  is  17 ;  answering  to  which, 
at  the  bottom  of  the  Table,  is  4f ,  which,  being  added  to-  the  above-found 
arch,  gives  62t20U4^,  the  required  arch. 

No^e.— The  arch  corresponding  to  a  given  cci-versed  sine,  t>r  natural 
cosine,  is  obtained  in  a  similar  manner. 

Bemarkh 

The  logarithmic  versed  sine  of  an  arch  may  be  found  by  taking  out  th^ 
eommon  logarithm  of  the  product  of  the  natural  versed  sine  of  such  arch 
by  10000000000;  as  thus: 

Required  the  logarithmic  versed  sine  of  78?30U5f  ? 

The  natural  versed  sine  of  78?30U5r  is  .  800846,  which,  being  multir 
plied  by  10000000000,  gives  8008460000 ;  the  common  log.  of  this  is 
9. 903549 ;  which,  therefore,  is  the  logarithmic  versed  sine  of  the  given 
arch,  as  required. 

Remark  2. 

The  Table  of  Logarithmic  Rising  may  be  readily  deduced  from  the 
natural  versed  sines ;  as  thus  : 

Reduce  the  meridian  distance  to  degrees,  by  Table  I.,  and  find  the 
natural  versed  sine  corresponding  thereto ;  now,  let  this  be  esteemed  tfi 
an  integral  number,  and  its  corresponding  common  log.  will  be  the  loga« 
rithmic  rising. 


Digitized  by 


Google 


62  mtscRiPTioN  Aim  VOL  OF  m  TABUBS. 

Required  the  logarithmic  rising  answering  to  4^50*45  ?  ? 

4!50r45!  s  72?4r.l5r,  the  natural  versed  sine  of  which  is  702417  ; 
the  common  log.  of  this  is  5. 846595,  whicb^  therefore,  is  the  logarithmie 
rising  required. 


Tabu  XXVIII. 
LogarUhnu  of  Iiumber$k 

Logarithms  are  a  series  of  numbers  invented,  and  'first  published  in 
1614,  by  Lord  Napier,  Baron  of  Merchiston  in  Scotland,  tot  (he  purpose 
of  facilitating  troublesome  calculations  in  plane  and  spherical  trigonometry. 
These  numbers  are  so  contrived,  and  adapted  to  other  numbers,  that  the 
sums  and  di£ferences  of  the  former  shall  correspond  to,  and  show,  the  pro* 
ducts  and  quotients  of  the.  latter. 

Logarithms  may  be  defined  to  be  the  numerical  exponents  of  ration,  or 
a  series  of  numbers  in  arithmetical  progression,  answering  to  another 
series  of  numbers  in  geometrical  progmston }  as^ 

Thusi 


0,  1. 

2. 

9. 

4. 

5. 

6. 

7. 

8.  tud-ortog. 

1.2. 

.4. 

8. 

16. 

32. 

64. 
Or, 

128. 

S56.ge<Kprag. 

0.  1. 

2. 

3. 

4. 

5. 

6. 

7. 

8.  ind.orlog. 

1.  s. 

9. 

27. 

81. 

243. 

729. 
Or, 

2187. 

6561.  geo.pro. 

0.  1. 

2. 

3. 

4. 

5. 

6. 

7. 

8.  ind.orl<^. 

1. 10. 100. 1000. 10000. 100000. 1000000. 10000000. 100000000  ge.  pro. 

Whence  it  is  evident,  that  the  same  indices  serve  equally  for  any 
geometrical  series ;  and,  consequently,  there  may  be  an  endless  variety  of 
systems  of  logarithms  to  the  same  common  number,  by  only  changing  the 
second  term  2.  3,  or  10.  &c.  of  the  geometrical  series  of  whole  mmi« 
hers* 

In  these  series  it  is  obvious,  that  if  any  two  indices  be  added  together. 


Digitized  by 


Google 


]>BtGBIPTIOM  Aim  VtM  OV  TU  TABLW.  M 

thdr  turn  iriU  be  the  index  of  that  number  whidi  !■  eqilal  to  the  product 
of  the  two  terms^  in  the  geometrical  progression  to  whieh  those  indicee 
belong :  thus^  the  indices  2.  and  $•  being  added  together^  make  8;  and 
the  corresponding  tenns  4.  and  64.  to  those  indices  <in  the  first  series), 
being  multiplied  together^  produce  256,  which  is  the  number  correspohding 
totheindexS.  * 

It  is  also  obvious^  that  if  anyone  index  be  subtracted  from  another,  the 
differenee  will  be  the  index  of  that  number  which  is  equal  to  the  quotient 
of  the  two  corresponding  terms :  thus,  the  index  8.  mimto  the  index  3  s= 
5  ;  and  the  terms  corresponding  to  these  indices  are  256  and  8^  the  quo- 
tient of  which,  viz.,  32,  is  the  number  corresponding  to  the  index  5,  in  the 
first  series. 

And,  if  the  logarithm  of  any  number  be  multiplied  by  the  index  of  its 
power,  the  product  will  be  equal  to  the  logarithm  of  that  power ;  thus,  the 
mdex,  or  l<«arithm  of  16,  in  the  first  series,  is  4 ;  now,  if  this  be  multiplied 
by  2,  the  product  will  be  8,  which  is  the  logarithm  of  256,  or  the  square 
of  16. 

Again,— if  the  logarithm  of  any  number  be  divided  by  the  Index  of  its 
foot,  the  quotient  will  be  equal  to  the  logarithm  of  that  foott  thus,  the 
index  or  logarithm  of  256  is  8;  now,  8  divided  by  2  gives  4 ;  which  is 
the  logarithm  of  16^  or  the  square  root,  of  256,  according  to  the  first 
series. 

The  logarithms  most  convenient  for  practice  are  such  as  are  adapted  to  a 
geometrical  series  increasing  in  a  tenfold  ratio,  as  in.  the  last  of  the  fore- 
going series ;  being  those  which  are  generally  found  in  most  mathematical 
woika,  and  which  are  usually  tailed  oommon  kgarUhn^  in  Older  to  distin^ 
goish  them  from  ether  species  of  logarithms. 

In  this  system  of  log^thms,  the  index  or  logarithm  of  1,  is  0 ;  that  of 
10,  is  1 ;  that  of  100,  is  2;  that  of  1000,  is  3}  that  of  10000,  is  4,  &c. 
&e.;  whence  it  is  manifest,  that  the  logarithms  of  the  intermediate  num- 
bers between  1  and  10,  must  be  0,  and  some  fractional  parts ;  that  of  a 
number  between  10  and  100,  must  be  1,  and  some  fractional  parts ;  and  so 
on  for  any  other  number :  those  fractional  parts  may  be  computed  by  the 
followii^ 

Attle.— To  the  geometrical  series  h  10. 100. 1000. 10000.  &c.,  apply  the 
arithmetical  series  0.  1.  2.  3.  4.  &c.,  as  logarithms.  Find  a  geometrical 
mean  between  1  and  10,  or  between  10  and  100,  or  any  other  two  adja- 
cent terms  of  the  series  between  which  the  proposed  numbet  lies.  Between 
the  mean  thus  found  and  the  nearest  extreme,  find  another  geometrical  mean 
in  the  same  manner,  and  so  on  till  you  arrive  at  the  number  whose  loga- 
rithm is  sought,  fmd  as  many  aritiimetieal  means,  according  to  the  order 
in  which  the  geomelrical  ones,  were  founds  and  they  will  be  the  logarithms 


Digitized  by 


Google 


64  DBSCHTPnON  AK]>  t78«  OF  TUB  TABLEd. 

of  the  said  geometrical  means ;  the  last  pf  which  will  be  the  logarithm  of 
the  proposed  number. 

Esample. 

To  compute  the  Log«  of  2  to  eight  Places  of  Dechnals  :-— 

Here  the  proposed  numbl|r  lies  between  1  and  10. 
Firsts        The  log.  of  1  is  0,  and  the  log.  of  10  is  1 ; 

therefore  0+l-ft-2=:.5isthe  arithmetical  mean, 
and  V  1  X  10  =  3. 1622777  is  the  geometrical  mean : 
hetice  the  log.  of  3. 1622777  is  .  5. 

Second,    The  log.  of  1  is  0,  and  the  log.  of  3. 1 622777  is  .  5 ; 
therefore  0  +  5  -^  2  s  .  25  is  the  arithmetical  m^an, 
and  V  1  X  3. 1622777  =  1. 7782794  the  geometrical  mean : 
hence  the  log.  of  1. 7782794  is  .  25. 

Third,      The  log.  of  1 .  7782794  is  .  25,  and  the  log.  of  3. 1622777  is  .  5  ; 
therefore  .25  +  .5^2  =  .  375  is  the  arithmetical  mean^ 
and  n/ 1.7782794  x  3.1622777=  2.3713741  the geo. mean: 
hence  the  log.  of  2. 37 13741  is  .  375. 

Fourth,    The  log.  of  1 .  7782794  is  .  25,  and  the  log.  of  2. 371374 1  is .  375  ; 
therefore  .  25  +  .  375  -f-  2  =  .  3125  is  the  arithmetical  mean, 
and  V  1.7782794  x  2.3713741  =  2.0535252  the  geo.  mean : 
hence  the  log.  of  2. 0535252^  is  .  3 125. 

Fifth,       Thelog.  of  1. 7782794 is .  25,  and  the  log. of  2. 053925218 .3125; 
:     therefore  .  25  +  .  3125  -f-  2  =  .  28125  is  the  arith.  mean, 
and  V  1.7782794  x  2.0535252  =  1 .  9109530  the  geo.  mean : 
hence  the  log.  of  1 . 9 109530  is  .  281 25. 

Sixth,      Thelog.ofL9109530is.28125,&thelog.of2. 053525218. 3125| 
therefore  .  28125  +  .  3125  -ft-  2  =  .  296875  is  the  arith.  mfean^ 
and  V  1.9109530  x  2.0535252  =  1.0809568  the  geo.  mean: 
hence  the  log.  of  1.  9809568  is .  296875. 

Seventh,  Thelog.ofl.  9809568  is.  296875,  &  the  log;  of  2. 0535  252  is  .3125; 
therefore .  296875  +  .  3125  -f-  2  t=.  3046875  is  the  arith.  mean^ 
and  V  1.9809568  x  2.0535252  =  2. 0169146  the  geo. mean: 
hence  the  log.  of  2. 0169146  is .  3046875. 

Eighth,  Thelog.of2. 0169146 is.  3046875, &log.of  1 . 98095 68 is 296875 ; 
therefore .  3046875  +'.  296875-4-  2=.  30078125  is  the  ar.  mean, 
and  V  2.0169146  x  1.9809568  =  1. 9988548  the  geo.  mean  s 
hence  the  log.  of  1. 9988548  is .  30078125/      . 


Digitized  by 


Google 


DESCRIPTION   AND  USB  OP  THB  TAfiLBS. 


65 


Proceeding  in  this  manner,  it  will  be  found,  after  25  eKtractions,  that 
the  log.  of  L  9999999  is  .  30103000;  and  since  1. 9999999  may  be  con- 
sidered as  being  essentialjy  equal  to  2  in  all  the  practical  purposes  to 
^ich  it  can  be  applied,  therefore  the  log.  of  2  is  .  30103000. 

If  the  log.  of  3  be  determined,  in  the  same  manner,  it  will  be  found  that 
the  twenty-fifth  arithmetical  mean  will  be  .  47712125,  and  the  geometrical 
mean  2. 9999999 ;  and  since  this  may  be  considered  as  being  in  every  re* 
spect  equal  to  3,  therefore  the  log.  of  3  is  .  47712125. 

Now,  from  the  logs,  of  2  and  3,  thus  found,  and  the  log.  of  10,  which 
18  giTen^l,  a  great  many  oth^r  logarithms  may  be  readily  raised;  because 
the  sum  of  the  logs,  of  any  two  numbers  gives  the  log.  of  their  product ;  and 
the  difference  of  their  logs,  the  log.  of  the  quotient ;  the  lo^.  of  any  num- 
ber, being  multiplied  by  2,  will  give  the  log.  of  the  square  of  that  number; 
or,  multiplied  by  3,  will  give  the  log,  of  its  cube;  as  in  the  following 
examples  :— 


Example  1. 

To  find  the  Log.  of  4  :— 

To  the  log.  of  2  =       .  30103000 
Add  the  log.  of  2  =x       .  30103000 

Sum  is  the  log.  of  4  ».  60206000 

Example  2. 

To  find  the  Log.  of  5  :— 

From  the  log.  of  10=1. 00000000 
Take  the  log.  of  2  =    .  30103000 

Rem.  is  the  log.  of  5  s .  69897000 

Example  3. 

To  find  the  Log.  of  6  :— 

Totbelog.  of  3=       .47712125 

Add  the  log.  of  2  =      .  30103000 

Sum  is  the  log.  of  6  =  .  77815 125 

Example  4. 

To  find  the  Log.  of  8  :— 

To  the  log.  of  4  =3       .  60206000 

Add  the  log.  of  2  =      .  30103000 

Sum  is  the  log.  of  8  =  .  90309000 


Example  5. 

To  find  the  Log.  of  9  :— 
To  the  log.  of  3  =        .  47712125 
Add  the  log.  of  3  =s      .  477 1 21 25 

Sum  is  the  log.  of  9  =  •  95424250 

Example  6. 

To  find  the  Log.  of  1 5  : — 

To  the  log.  of  5  =        .  69897000 

Add  the  log.  of  3  a      .  47712125 

Sumisthelog.of  15r=L  17609125 

Example  7. 

To  find  the  Log.  of  81  s  th^ 

square  of  9  :— 

Log.of9=   •    .    .    .95424250 

Multiply  by        •    «    •  2 

Pro.  is  the  log.  of  81 = 1 .  90848500 
Example  8. 

To  find  the  Log.  of  729  :;=:  the 

cube  of  9: 

Log.  of  9 «    ,    .    .    .95424250 

Multiply  by  .    .      .  3 

Pro.is  thelog.of  729*52. 86272750 


Digitized  by 


Google 


66  BBSCRIPTtOK  AND  USB  OF  THB  TABLBS. 

Siuce  the  oddnumbere  7.  11.  13.  17.  19.  23.  29.  &c.  cannot  be  exactly 
deduced  from  the  multiplication  or  division  of  any  two  numbers,  the  logs, 
of  those  must  be  computed  agreeably  to  the  rule  by  which  the  logs,  of  2 
and  3  were  obtained ;  after  which,  the  labour  attending  the  construction  of 
a  table  of  logarithms  will  be  greatly  diminished,  because  the  principal  part 
of  the  numbers  may  then  be  very  readily  found  by  addition,  subtraction, 
-and  composition* 

Of  the  Table. 

This  Table,  which  is  particularly  adapted  to  the  reduction  of  the  appa- 
rent to  the  troe  central  distance,. by  certain  concise  methods  of  computa- 
tion, to  be  treated  of  in  the  Lunar  Observations,  is  divided  into  two  parts : 
ibBjirst  of  which  contains  the  decimal  parts  of  the  logs.,  to  six  places  of 
figures,  of  all  the  natural  numbers  from  unity,  or  1^  to  999999 ;  and  the 
iecondj  the  logs,  to  the  same  extent,  of  all  the  natural  numbers  from 
1000000  to  1839999 ; — and  although  the  logs,  apparently  commence  at 
the  natural  number  100,  yet  the  logs,  of  all  the  natural  numbers  under  that 
are  also  given  :  thus,  the  log.  of  1,  or  10,  is  the  same  as  that  of  100  5  the 
k^,  of  2,  or  20,  is  the  same  as  that  of  200;  this  log.  of  3,  or  30,  is  equal 
to  that  of  300;  that  of  11,  to  110;  that  of  17>  to  170;  that  of  99,  to  990; 
and  BO  on :  using,  however,  a  different  index.  And  as  the  indices  are  not 
affixed  to  the  logs.,  they  must  therefore  be  supplied  by  the  computer :  these 
indices  are  always  to  be  considered  as  being  one  less  than  the  number  of 
integer  figures  in  the  corresponding  natural  number.  Hence  the  index  to 
the  log.  of  any  natural  number,  from  1  to  9  inclusive,  is  0 ;  the  index  to 
the  log.  of  any  number  firom  10  to  99  inclusive,  is  1 ;  that  to  the  log.  of  any 
number  from  100  to  999,  is  2 ;  that  to  the  log.  of  any  number  from  1000 
to  9999,  is  3;  &c.  &c.  &c.  The  second  part  of  the  Table  will  be  found 
very  useful  in  computing  the  lunar  observations,  by  certain  methods  to  be 
given  hereafter,  when  the  apparetit  distance  exceeds  90  degrees,  or  when 
it  becomes  necessary  to  take  out  the  log.  of  tt  natural  number  connsting 
of  seven  placer  of  •figures,  and  conversely. 

In  the  left-hand  column  of  the  Table,  and  in  the  uppw  or  lower  hori* 
zontal  row,  are  given  the  natural  numbers,  proceeding  in  regular  succes- 
sion ;  and,  in  the  ten  adjacent  vertical  columns,  their  corresponding  loga- 
rithms. 

As  the  size  of  the  paper  would  not  admit  of  the  ample  insertion  of  the 
logs.,  except  in  the  first  column,  therefore  only  the  four  last  figures  of  each 
log.  are  given  in  the  nine  following  columns ;  the  two  preceding  %ures 
belonging  to  which  will  be  found  in  the  first  column  under  0  at  top,  or 
over  0  at  bottom ;  and  where  these  two  preceding  figures  change,  in  the 
body  of  the  Table,  large  dots  are  introduced  instead  of  O's,  to  catch  the 


Digitized  by 


Google 


'  PBSCRIPnOM  ANB  USB  OF  THB  TABLBSf  07 

eye  and  to  incUcate  that  from  thence,  through  the  rest  of  the  line,  the 
said  two  preceding  figures  are  to  be  taken  from  the  next  lower  line  in  the 
column  under  or  over  0  :  those  dots  are  to  be  accounted  as  ciphers  in  tak^ 
ing  out  the  logarithms. 

The  log.  of  any  natural  number  consisting  of  four  figures,  or  under,  and 
conversely,  is  found  directly  by  the  Table ;  but  because  the  log.  of  a  natural 
number  consisting  of  five  or  more  places  of  figures,  and  the  converse,  is 
frequently  required  in  th^  reduction  of  the  apparent  to  the  true  central 
distance,  and  also  in  many  other  astrc^nomical  calculations  j  proportional 
parts  are,  therefore,  adapted  to  the  Table,  and  arranged  in  the  nine  small 
columns  on  the  right-hand  side  of  each  page;  by  means  of  which  the  loga- 
rithms of  all  the  natural  liumbers,  not  consisting  of  more  than  seven  places 
of  figures,  and  vice  versGy  may  be  found  to  a  sufficient  degree  of  accuracy 
for  all  nautical  purposes,  as  may  be  seen  in  the  following  problems. 


Probum  L 

Gioen  a  Natural  Number  consisimg^  of  Jive^  siXf  or  seven  Placei  qf 
Figures,  to  find  the  corresponding  Logarithm, 

RULB. 

Look  for  the  three  first  figures  of  the  giren  natural  number  in  the  left-hand 
column  3  opposite  to  which,  and  under  the  fourth  figure,  in  the  horizontal 
column  at  top,  will  be  found  the  log.  to  the  four  first  figures  of  the  given 
natural  number :  on  the  same  line  witfi  this,  and  under  the  fifth  figure  of 
the  natural  number  at  top,  in  the  proportional  parts,  will  be  found  a  numberi 
which,  being  added  to  the  above,  yn\l  give  the  log.  to  five  places  of  figures 
of  the  given  natural  number;  on  the  same  line  of  proportional  parts,  and 
under  the  sixth  figure  of  the  natural  number  at  top,  will  be  found  a  number, 
which,  being  divided  1^  10,  aqd  the  quotient  added  to  the  last  found  log., 
will  pye  the  log.  to  six  places  of  figures  of  the  given  natural  number.  In 
the  same  manner,  the  log,  may  be  taken  out  to  seven  places  of  figures ; 
observing,  that  the  number  in  the  proportional  parts,  corresponding  to  the 
seventh  figure  of  the  natural  number,  is  to  be  divided  by  100. 

JVb^e.— In  dividing  by  10  or  100,  we  have  only  to  strike  off  the  right- 
hand,  or  two  right-hand  figures. 

Example. 

Rw|aii«d  ihdlog.  oonesponding  to  the  gifen  natural  number  1378078? 

y2 


Digitized  by 


Google 


68  DESCRIPTION  AND  ITSB  OF  THB  TABLBS. 

Log.  corresponding  to  1378         (four  first  figures)  is     ...    .  139249 
5th  fig.  of  the  nat.  num.       .9      ans.  to  which  in  the  pro.  parts  is  284 

6th  fig.  of   .     do.  . .  7     ans.  to  which  in  the  pro.  parts  is 

221,  which,  divided  by  10, 

gives  22.1 22 

7th  fig,  of        do,  ...  8  ans.  to  which  in  the  pro.  parts  is 

252,  which,  divided  by  100, 
gives  2. 52    .     •    •    •    •  2 


Given  liatural  number  1378978  Corresponding  log.   =s     .    •  6. 139557* 


'     Problem  II. 

To  find  the  Natural  Number  to  five,  six,  or  ieoen  Places  of  Figures, 
corresponding  to  a  given  Logarithm. 

Rule. 

Find  the  next  less  log.  answering  to  the  given  one  in  the  column  under 
0 ;  continue  the  sight  along  the  horizontal  line,  and  a  log,,  either  the  same 
as  that  given,  or  somewhat  near  it,  wiQ  be  found ;  then,  the  three  first 
figures  of  the  corresponding  natural  number  will  be  found  in  the  left-^hand 
column,  and  the  fourth  figure,  above  the  log.,  at  the  top  of  the  Table.  Should 
the  given  log.  be  found  exactly,  let  one,  two,  or  three  ciphers  be  annexed 
to  the  natural  number  found  as  above^  according  to  the  number  of  figures 
wanted,  and  it  will  be  the  natural  number  required.  But,  if  the  log.  cannot 
be  exactly  found  (which  in  general  will  be  the  case),  find  the  difference 
between  the  given  log.  and  the  next  less  log.  in  the  Table :  with  this 
difference,  enter  the  proportional  pai'ts,  on  the  same  horizontal  line  in 
which  the  next  less  log.  was  found,  and  find  the  next  less  proportional 
part;  answering  to. which,  at  the  top  or  bottom,  will  be  found  the  fifth 
figure  of  the  required  natural  number :  find  the  difference  between  the 
above-found  difference  and  the  aforesaid  next  less  proportional  part; 
which  being  multiplied  by  10,  and  the  product  foimd  in  the  same  line  of 
proportional  parts,  the  number  corresponding  thereto,  at  top  or  bottom, 
will  be  the  sixth  figure  of  the  required  natural  number,  Now,  the  differ- 
ence between  the  above  product  and  its  next  less  proportional  part,  being 
multiplied  by  10,  also,  and  the  product  found  in  the  same  line  of  propor- 
tional parts,  the  number  answering  thereto  at  top  or  bottom  will  be  the 
seventh  figure  of  the  required  natural  number. 


*  The  iadtx  6  is  prefixed,  because  the  given  natural  number  consists  of  seven  places  of 
fi^ires. 


'Digitized  by 


Google 


DBSCRIPITON  AND  USE  OP  THB  TABLBS.  69 

Example. 

Required  the  natural  number  correspondingto  the  given  log,  6, 1 19558? 

Given  log 6,119558 

1316  s=  fourfirst  figs,  ofthe  required  nat.  num. 

answering  to  next  less  log.        •    .      .1 19256 

Difference  ; 302 

.  9  =  fifth  fig.  ofthe  required  nat.  num.  ans. 

to  the  pro.  part  next  less    ...  297 

Difference .    5  x  lQ=50product, 

. .  l=sixth  fig.  of  the  required'  nat.  num. 

ans.  to  pro.  part  next  less        ••••••    33 

Difference 17x10=170 

• . .  5=:seventh  fig.  of  the  required  nat.  num.  ans.  to  the 

nearest  pro.  part ••#••165 

1316915  which  is  the  natural  number  corresponding  to  the  given  log. 
6. 119558^  as  required. 

Note. — ^From  the  above  Problems^  the  manner  of  using  the  second  part 
of  the  Table  will  appear  obvious. 

Remarks. 

1.  The  whole  of  the  operation  is  inserted  at  length,  for  the  purpose  of 
illustrating,  more  clearly,  the  use  ofthe  Table;  but  in  practice,  the  logs, 
may,  in  most  cases,  be  taken  out  at  sight,  and  cbnversely ;  particularly 
from  the  second  part,  where  the  natural  numbers  are  given  to  five  places  of 
figures,  froni  lOQOOOO  to  1839999. 

2.  In  taking  out  the  log.  of  a  decimal,  fraction,  or  any  number  less  than 
unity,  if  the  first  decimal  place  be  a  significant  figure,  the  index  of  its  log. 
is  to  be  accounted  as  9  ;  but  if  the  first  significant  figure  of  the  decimal 
stands  in  the  second,  third,  or  fourth  place,  &c«,  the  index  of  the  corre- 
sponding log.  is  to  be  taken  as  8,  7,  or  6,  &c.  The  converse  of  this, — that 
is,  finding  the  significant  decimals  corresponding  to  a  given  log.,  will 
appear  obvious. 

3.  Hie  arithmetical  complement  of  a  log,  is  what  that  log.  wants  of  the 

Digitized  by  VjOOQ IC 


70 


0BtCllIFTION  AN0  VMM  OF  TMB  tAJUJU. 


radius  of  the  Table ;  viz.^  of  10. 000000 :  this  is  most  easily  found,  by  begin- 
ning at  the  left  hand,  and  subtracting  each  figure  from  9,  except  the  last 
significant  one,  which  is  to  be  taken  from '10;  as  thus: 

The  arithoiedeal  complement  of  tiie  log.  4.97^8Sd  is  5.627147  j  and 
80  on. 


Problbm  III. 
To  petform  MuUvficaii(m  ijf  Lagariihm* 

tlCLB. 

To  the  log.  of  the  multiplicand,  add  the  log.  of  the  multiplier,  or  add 
the  logs,  of  the  foctors  together,  and  the  sum  will  be  the  log.  of  the' pro- 
duct ;  the  naturaT  number  corresponding  to  which  will  be  the  product 
required. 


Example  h 
Multiply  436  by  19.7. 

436    Log.  =    .    .    2.639486 
19.7    Log.  =:     .    .     1.294466 


Prod.=:8589. 18  Log.=:3. 933952 

Example  2. 

Multiply  437. 8  by  14.07,  and 
ako  by  0. 239. 

437.8        Log.r: 

14.07      Log.=: 

0.239    Log.  = 


•  2.641276 
.     1.148294 

•  9.378398 


Example  3. 

What  is  the  product  of  0.049^ 
9. 875,  and  0.753? 

0.049  Log.=  .  .  8.690196 
9.875  Log.=  .  .  0.994537 
0.753    Log.=    .    .    9.876795 


Prod.=0.3642  Log.  =  9. 561528 

Example  4. 

What  is  the  product  of  0. 0567 
and  0.00339? 


Pro.=1472.204Log.  =  3. 167968 


0. 0S67    Logf.  >6 
0. 00BS9  Logi  a 


«  6.768588 
.  7.580200 


Pro.«0. 0001628Log.«6. 883783 

Abte.-«R6speeting  the  index  of  a  decimal  fraction^  akid  eonmtiely^  see 
Remaik  2,page69, 


Digitized  by 


Google 


PBSCftlPnON  AND  USB  OV  TBI  TABUUU 


71 


Paoblbm  IV. 
To  perfima  I)ms%on  by  Logarithms. 

RULB* 

From  the  log.  of  the  diyidend^  subtract  the  log.  of  the  divisor^  and  the 
remainder  will  be  the. log.  of  the  quotient;  the  natund 'number  corre-' 
sponding  to  which  will  be  the  quotient  required. 


JEsQxnplti  1*. 
Divide  1497  by  98.       .    ,    „ 

1497    Log.  =     .    .    3.175222 
93    Log.  =     .    •'  1:988483' 


Quo.=  16.0968    Log.=  1.2067S9 

JEjr(impfe2. 
Divide  469. 76  by  0.937. 

469.76    Log.  s      ,    2.671876 
0.937  Log.  =.     .    9.971740 


Qao.sS01.84S    Log.s2. 700136 


EsampteS. 

,     Divide49.73by0.06S2. 

49.73  Log.   =     1.696618 

0.0632        Log.   =    8.800717 


Qtto.=:786.869=Log.=2. 895901 

Example  4. 

Divide  0. 00815  by  0. 000275. 

0.00815    Log.  =1    •    7.911158 
0.000275  Log.  =:    •    6.439333 


Quo.=29. 6368  Log.  =  1. 471825 


Problbm  V. 

To  petform  Proportkm^or  ike  RdeofThree^ or  Golden  Hub^ by 
.    '         LogftnihiM.* 

RULB. 

To  the  aridlmetical  coippleDient  of  the  log.  of  th^.  first  term,  add  the 
logs,  of  the  gjscond  and  third  terms  {  and  the  sum  will  be  the  log,  of  the, 
fourth  term,  or  answer.  * 

Example  U 

If  a  ship  sails  19$  miles  in  2i  hours^  how  many  miles  will  she  run,  at 
the  same  rate,  in  24  hours  ? 

As  2.25  hours,  arith.  comp.  log.    =:    .    .    •    9.647817 

Is  to  19. 5  miles,  log.      . 1.290035 

So  is  24  hours,  bg.       « ;     1.380211 


To  208  mfles,  log.    =      .......    2.318063 


Digitized  by 


Google 


72  DBSCRIPTION  AMD  USB  OF  TH«  TABLBS. 

Example  2, 

If  the  interest  of  1001.  for  365  days  be  41  lOs.,  what  will  be  the  interest 
ofl78I.15«.  for213daya? 


.   5  100    Arith.  conp.  of  log. 
■^1365  Do.        dp. 

T    ♦  J  178. 
I»  »°l213. 


8.000000 
.    .    7.437707 


•        • 


178.75    Log 2.252246 

Log 2.328380 

So  18    4.5  Log.'     ......    0.653213 

To      4.69403  Log 0.671546 

Example  3. 

A  man  of  war^  sailing  at  the  rate  of  9  knots  an  hour,  descried  a  ship, 
distant  26  miles,  sailing  at  the  rate  of  6^  knots,  to  which  she  gave  chase  : 
after  two  hours'  chase,  the  breeze  freshened,  and  increased  the  man  of  war's 
rate  of  sailing  to  1 1^  knots,  and  that  of  the  chase  to  8^.  In  what  time  did 
the  man  of  war  come  up  with  the  chase  ? 

Sb&iHon.— Since  the  man  of  war  gained,  atthe  commencement,  2^  miles 
an  hour  on  the  chase,  therefore,  at  the  end  of  the  first  two  hours,  the  dis- 
tance between  them  was  reduced  to  21  miles ;  during  the  rest  of  the  chase, 
the  hourly  gain  of  the  man  of  war  was  2}  miles. 

Hence,  As  the  hourly  gun  2. 75  Ar.  comp.  log.  9. 560667 
Is  to  ....  1  hour,.  Log.  •  •  0. 000000 
So  is  distance     .  21  miles,  Log.  .    .     1.322219 

To     ....    7.6363     Log.    =     0.882886, 
or  7  hours  and  38  minutes  from  the  time  the  breeze  freshened. 


Problem  VI. 
To  perform  InvobMon  by  Logarithms. 

Rule. 

Multiply  the  log.  of  the  given  number  by  the  index  of  the  power  to 
which  it  is  to  b^  raised,  and  the  procjuct  will  be  the  log.  of  the  required 
power.  • •        .   -  , 


Digitized  by 


Google 


DBSCRIPTION  AND  USB  OF  THB  TABLB8* 


73 


Mxample  I. 

Required  the  square  of  346  ? 

346    Log.       .    .    .    2.539076 
Ind.  of  the  power=  2 


Answer  119716   Log.=5. 078152 


Example  2. 
Required  the  cube  of  754  ? 

754    JLog 2.877371 

Ind.  of  the  .power=  3 


Ans.  428661064  Log.=8.632113 


Pboblbm  VIL 

To  perform  Evolution  by  Lc>garUhms, 

Rulb. 

Divide  the  log.  of  the  given  number  by  the  index  of  the  power^  and  the 
quotient  will  be  the  log.  of  the  root. 


Example  1. 

Required   the    square    root    of 
76176? 

76176    Log.     .    .     y4. 881818 


Ans.  276        Log.  =    2.440909 


Example  2. 

Required    the     cube    root    of 
21952000  ? 

21952000    Log.  .     f 7. 341475 


Ans.  280    Log.    =:     2.447158^ 


PaoBLBM  Vin. 
To  find  ike  Tonnage  of  a  Ship  by  Logarithms. 

RuiB. 

To  the  log.  of  the  length  of  the  keel^  reduced  to  tonnagCy  add  the  log.  of 
the  breadth  of  the  beam,  the  log.  of  half  the  breadth  of  the  beam,  and  the 
constant  log.  8.026872*;  the  sum,  rejecting  10  from  the  index,  will  be 
the  log.  of  the  required  tonnage. . 

Example. 

Let  the  length  of  a  ship's  keel,  reduced  to  tonnage^  be  120. 5  feet,  and 
the  breadth  of  the  beam  35.  75  feet;  required  the  ship's  tonnage  ? 

*  This  is  the  uithmetical  complemeat  of  the  logt  of  94 ;  the  common  divisor  for  fiiiding 
the  fopno^e  of  sl)ipt» 


Digitized  by 


Google 


74  J>K8CRIPTtON  AVD  USB  OF  TBB  TABUS. 

Length  of  the  ked  for  tonnage  .     1 20. 5      feet  Log.  2. 080987 

Breadth  of  the  beam    ....      35.75     feet  Log.  1.553276 

Half  ditto 17.875  feet  Log.  1.252246 

Constantlog.     •  .« 1    .    .   8.026872 


Required  tonnage 819.18  .    .    .    Log.  2.913381 


Problem  IX. 

Given  the  Measured  Length  of  a  Knot,  the  Number  of  Seconds  run  by 
the  Glass,  and  ihe  Distance  sailed  per  Log,  to  find  the  true  Distance 
by  Logarithms.  * 

RULB. 

Tq  the*  arithmetical  complement  of  the  log.  of  the  oomber  of  aeoonda 
run  by  the  glass,  add  the  log.  of  the  measured  length  of  a  knot,  the  log.  of 
the  distance  sailed,  and  the  constant  log.  9. 795880* ;  the  sum  of  these 
four  logs.,  rejecting  20  from  the  index,  will  be  the  log.  of  the  true  distance. 

Example  1. 

The  distaoce  sailed  by  the  log  is  ISO  mil&ij  the  measured  length  of  a  knot 
is  43  feet,  and  the  time  by  the  glass  32  seconds ;  required  the  true  distance  ? 

32  seconds,  arith.  comp.  log.        .    .    •  8. 494850 

43  feet,  log.     s       1.633469 

180  mUes,  log.  =       2.255273 

Constant  log.    as  9.795880 


True  distance  =  151. 2  miles.       Log.  s  2. 179472 

Example  2. 

The  distance  sailed  by  the  log  is  210  miles ;  the  measured  length  of  a  knot 
19  5 1  feety  and  the  time  by  the  glass  27  seconds ;  required  the  true  distance  ? 

27  seconds,  arith.  comp.  log.    th     .    .    6. 568636 

51  feet,  log 1.707570 

210  miles,  log ....    2.322219 

Constant  log.  9. 795880 

True  distance  =  247. 9  miles.     Log.  =     2.394305 

•  This  It  the  ram  of  Om  arlUimtClcsl  compkoMnt  of  Hie  hog.  ol  46  (the  gtnenl  teagtii 
of  a  knot)  and  the  log.of  30  secondi,  the  true  measure  of  the  half-miDttls  flaas; 

Digitized  by  VjOOQ IC 


DUCaiPTIoy  AND  USB  OF  THB  TABLBO.  79 

Tabu  XXIX. 
Prapcrtional  Logarithmic 

This  Table  cpntuns  the  proportional  log.  conesponding  to  all  portions 
of  time  under  three  hours^  and  to  every  second  under  three  degrees.  It 
^vas  originally  eomputed  by  Or.  Maskelyne^  and  particularly  adapted  to  the 
operation  for  finding  the  apparent  time  at  Greenwich  answering  to  a  given 
distance  between  the  ftioon  and  sutr^  or  a  fix^  star;  but  it  is  now  applied 
to  many  other  important  purposes^  as  will  be  seen  hereafter. 

Proportional  Logarithms  may  be  computed  by  the  following 

RULB.' 

From  the  common  log.  of  3  hours,  reduced  to  seconds^  subtract  the 
common  log.  of  the  given  time  in  seconds ;  and  the  remainder  will  be  the 
proportional  log.  corresponding  thereto. 

Example. 

R^red  the  proportional  log.  corresponding  to  0^40^26!  ? 

3  home  reduced  to  seconds  s     lOSOOr  Log.    :£:    <.  033424 

40r26!  given  tilne^  in  sees,  s      2426?  Log.    a    3.384891 


Proportional  log.  corresponding  to  the  given  time  s  0*  6485*  33 

As  hours  and  degrees  are  similarly  divided,  therefore,  in  the  general  use 
ot  this  Table,  the  hours  and  parts  of  an  hour,  may  be  considered  as 
degrees  and  parts  of  a  degree,  and  conversely.  And  to  render  the  use  of 
it  more  extensive,  one  minute  ihay  be  Esteemed  as  being  either  one  degree, 
•r  one  second^  and  vice  versa. 

Since  proportion  is  performed  by  adding  together  the  arithmetical  com- 
plement of  the  proportional  logarithm  of  the  first  term,  and  the  pr<^or- 
tional  logarithms  of  the  second  and  third  terms,  rejecting  10  from  the 
index,  the  present  Table  is  of  great  use  in  reducing  the  altitudes  of 
the  moon  and  sun,  or  a  fixed  star,  to  the  mean  time  apd  distance,  when 
fhe  obeerradons  are  made  by  one  person,  as  will  appear  evident  by  the 
following 

•  Example. 

Let  the  lirsl  aWlude  of  the  moon*s  lower  limb  be  27?25t20'',  and  the 
corresponding  tim6  per  watch  21M2r8!^  and  the  last  altitude  25?24;20ry 


Digitized  by 


Google 


76  DBSCRIPTION  AND  USE  OF  TAB  TABLBS. 

and  its  corresponding  time  21^55T57- ;  }t  is  required  to  reduce  the  first 
altitude  to  what  it  should  be  at  21M9T33!,  the  time  at  which  the  mean 
lunar  distance  was  taken  ? 

l8ttime2lM2r  8!  Isttime21?42r  8!  l8tait,27?25C20r  27?25:20r 
Last  do.  21. 55*57  Mean  do.  21. 49. 33  Last  do. 25. 24. 20 


Ditt    •    0.18.49    Diff.    ,   0.  7.25    Diff  .    2.   1.  0 

As  13?49!,  arithmetical  comp.  frop.  log.  =  8. 8851 
Is  to  7*^25 !  proportional  log.  •  •  •  •  =1.3851 
So  is    2?  1' proportional  log =0.1725 

To  prop.  log.  of  reduction  of  Moon's  alt. .    :±  0. 4427  =  —    1?  4 '57* 


Moon's  alt.  reduced  to  mean  time  of  observation    •    •   =      26?20^23T 

And  in  the  same  manner'  may  the  altitude  of  the  sun^  or  a  fixed  star^  be 
reduced  to  the  time  of  taking  the  mean  lunar  distance. 

Remark. — ^Although  this  Table  is  only  extended  to  3  hours  or  3  degrees^ 
yet  by  taking  such  terms  as  exceed  those  quantities  one  grade  lower^  that 
is,  the  hours^  or  degrees,  to  be  esteemed  as  mintites,  and  the  minutes  as 
seconds,  the  proportion  nlay  be  worked  as  above :  hence  it  is  evident  that 
the  Table  may  be  very  conveniently  applied,  to  the  reduction  of  the  sun's^ 
moon's,  or  a  planet's  right  ascension  and  declination  to  any  given  time  afiter 
noon  or  midnight ;  and,  also,  to  the  equation  of  time  3-— for  the  illustration 
of  which  the  following  Problems  are  given. 


Problbm  L 

To  reduce  the  Sun's  Longitude^  Itight  Ascension  and  Decimation;  and, 
also,  the  Equation  of  Tlrne,  as  given  in  the  Nautical  Ahnanac,  to  anjf 
.   given  Meridian,  o^td  to  any  given  time  under  that  Meridian. 

RULB. 

To  the  apparent  time  at  ship,  or  place,  (to  be  always  reckoned  from  the 
preceding  noon  *,)  add  the  longitude,  in  time,  if  it  be  west,  but  subtract  it 
if  east ;  and  the  sum,  or  difference,  will  be  the  Greenwich  time. 

From  page  II.  of  the  month  in  the  Nautical  Almanac,  take  out  the  sun's 


•  See  precepte  to  Tftblc  XV.— page  25, 

/Google 


Digitized  by ' 


DBSCttlPTlON   AND  VSB  OF  THE  TABLES.  77 

longitude^  right  ascension,  declination,  or  equation  of  time  for  the  noons 
immediately  preceding  and  following  the  Greenwich  time,  and  find  their 
difference  5  Uien, 

To  the  proportional  log.  of  this  difference,  add  the  proportional  log.  of 
the  Greenwich  time  (reckoning  the  hours  as  minutes,  and  the  minutes  as 
seconds,)  and  the  constant  log.  9.  1249* ;  the  sum  of  these  three  logs.,  re- 
jecting 10  from  the  index,  will  be  the  proportional  log.  of  a  correction 
which  is  always  to  be  added  .to  the  sun's  longitude  and  right  ascension  at 
the  noon  precedkig  the  Greenwich  time ;  but  to  be  applied  by  addition  or 
subtraction  to  the  sun's  declination  and  the  equation  of  time^  at  that  noon, 
according  as  they  may  be  increasing  or  decreasing, 

Esample  1. 

Required  the  sun's  longitude,  right  ascension  and  declination,  and  also 
the  equation  of  time.  May  6th,  1824,  at  5 MO?,  in  longitude  64?4S^  west 
qf  the  4aiieridian  of  Greenwich  ? 

Apparent  time  at  ship  or  place^  = 5  MO? 

Longitude  64?45C  west,  in  time,  =     .    .  *  •    .    .+4. 19 

Greenwich  time,  = 9*29? 

To  find  the  Sun's  Longitude. 

Diff.  in  24  hours  =  57 '59?  prop,  log =    .4920 

Greenwich  time  =    9*29?  prop.  log.    ...;...     =1.2783 
Constant  log.    . =9. 1249 

Correction  of  sun's  long =     .  +  22:55?  p.  log.  =  0.8952 

Sun's  long,  at  noon.  May  6,  1824   .  =  1 !  15?51  ^  13? 

Sun's  long,  as  required      .     .    .    .  =   1M6?14'.   8? 

To  find  the  Sun's  Right  Ascension. 

Diff.  in  24  hours  =    3^52?  prop.  log.    ; =1.6679 

Greenwich  time   =    9*29?  prop,  log *   =1.2783 

Constant  log =  9. 1249 

Correction  of  sun's  right  asc.  .    .    .     =  +     1  ^  32?  p.  log.  =  2. 07 1 1 
Sun's  right  asc.  at  noon,  May  6, 1824,  =    2*53?31 ' .  7 

Sun's  right  asc.  as  reqtured  •    .    v    •     =^    2*55?  3*.  7 
t  The  aridunetical  €oniplemeiit  of  the  proporUonsl  lo^*  of  24  hours  esteemed  as  minutes. 

Digitized  by  VjOOQ IC 


78  PSBCRIPTION  AND  USB  OF  TOB  TABUfl^ 

To  find  the'  Sun'^  Declination* 

Diff.  in  24  hours  =  16^38r  prop,  log ,    .     =  1,0343 

Greenwich  time    =    9*29T  prop.  log.  *.• =1.2783 

Constant  log.   •    ,    .    .    .    .    .    ,     ==  9. 1249 

Correetion  of  sim'a  dee ss+     6^34?  p.  log. -a  1.4375 

Sun's  dec.  at  noon^  May  6^  1824,  .    .    sl6?S6f  51  north. 

rfi  ■  ■■■■III     I     111^ 

Sun's  dec.  as  required     •    ,    .    •    •    bb16?41I^39?  north. 

To  find  the  Equation  of  Time. 

Diff.  in  24  hours  ss    4^.5    prop,  log,        •    ,    •    •    »    «    7=3.3829 

Greenwich  time  cs    9i297prop.log.        »,..•«    ;=  1,2783 

Constant  log.    »«,•«»»;=  9. 1249 

Correction  of. the  equation  of  time     «    :s'+     l'«  8   p,  logf  s:  3*  7861 
Equation  of  time^  May  6,1824     .    .    s    STSd'.l 


Equation  of  time  as  required     .    .    •  -  z    8?37 '  •  9 

JRanorfc.— Since  the  daily  difference  of  the  equation  of  time  is  ex- 
pressed, in  the  Nautical  Almaiiao,  in  secoAds  and  tenths  of  a  second ;  i^ 
therefore,  these  tenths  be  multiplied  by  6,  the  daily  differenoe  will  be  re- 
duced to  seconds  and  thirds  :-^NoW)  if  those  seconds  and  thirds  be  esteem- 
ed as  minutes  and  seconds,  the  operation  of  reducing  the  equation  of  time 
will  become  infinitely  more  simple  ;  because  the  necessity  of  making  pro- 
portion for  the  tenths,  as  above,  will  then  be  done  away  with  2— remember- 
ing, however,  that  the  minutes  and  seconds  corresponding  to  the  sum  of  the 
three  logs,  are  to  be  considered  as  seconds  and  thirds. 

Example  2. 

Required  the  sun's  longitude,  right  ascension,  and  declination,  and  also 
the  equation  of  time,  August  2d,  1824,  at  19^22?,  in  longitude  98 ?45^  east 
of  the  meridian  of  Greenwich  ? 


Apparent  time  at  ship,  or  place,  19^22' 

Longitude  98 ?45'.  east,  in  time,  3=     •    •    «    »    —  6.35 


'm 


Greenwich  time^    •    «    •    •    »        •    •    •    •    «  12t47? 

/Google 


Digitized  by ' 


J>BiCEIFnOM  AKO  VBM  OF  THB  TABLES.  79 

To' find  the  Sun's  Longitude. 

Dlff.  in24hours=:  57'28rprop.log =    .4959 

Greenwich  time  =:  12M7*  prop,  log.    .......    =1.1486 

Constant  log .    =  9. 1249 

Correction  of  sun's  longitude,     .    •    =     +  30^37^  p.  log.  =5  0. 7694 
Sh&'s  long,  at  noon,  Aug.  2,  1824,     =4!  10?  Si  8r 

Son's  long,  as  required     .    •    •    «.    ^4;iO?3SM5; 

To  find  the  Sun's  Right  Ascension. 

Diff.  in  24  hours  =    3!52r  prop.  log ;    s  U6679 

Graenwich  time   s=  12^7?  prop.  log. •  «    s=  1.1486 

Constant  log =9. 1249 

CorreGtionof  sun's  right  asc.     •    •  =:+  2C  4?  p.  log.  •    =:  1.9414 
Sim's  right  ase.  at  noon,  Aug.  2,  1 824,:^  8 1 507  0.  8 

Son's  right  ase*  as  required  «    •    •    =:8i52?  4'.8 

To  find  the  Sun*s  Declination. 

Diff.  in  24  hours  =  15'36r  prop.  log.    .......     =1.0621 

Greentdch  time   =  12^47"  prop.  log.    .    .    .    .    •    •    .    =1. 1486 

Constant  log.   ........    =9.1249 

Correction  of  sun's  dec.         .    .    .    =     -     8^9^  p.  log.  =  1. 8356 
Su'sdec.  at  noon,  Aug.  «,  1824^;  .    =  17?  44Mlf  north. 

Sun's  dec.  as  required,      ....    =  17?  36'22r  north. 

To  find  the  Equation  of  Time. 

Diff.  in  24  hours  =    4r30r  prop,  log =1.6021 

Greenwich  time   =  12^47*  prop.  log. =1.1486 

Constant  log •    .    =  9. 1249 

>j ■  ■  ■ .  ■   — 

Correction  of  the  equation  of  time  .    =    —     2^24^  p.  log.  =  1. 8756 
Bqn.of  time,  at  noon,  Aug.  2,  1824,   =     5?  54!24f 

Eqiitof  tiflM  as  required^      •    •    •    s    57  52!  0! 

Digitized  by  VjOOQ IC 


80  BBSCRIPTIOK  AND   USE  OF  THB  TABLES. 

Problem  II. 

To  reduce  the  Moon's  Longitude,  Latitude,  Right  Ascension,  Declination, 
•  Semi-diameter  and  Horizontal  Parallax,  as  given  in  the  Nautical  Alma- 
nac,  to  any  given    Meridian,   and  to.  any  given    time  under  that 

Meridian, 

Rule. 

To  the  apparent  time  at  ship,  or  place,  (reckoned  from  the  preceding 
noon  or  midnight,)  add  the  longitude,  in  time,  if  it  be  west,  but  subtract  it 
if  east,  i^d  the  sum,  or  difference,  will  be  the  Greenwich  time  past  that 
noon  or  midnight,  according  as  it  may  be. 

Take  from  page  V.  VI.  ox  VII.  of  the  month  in  the  Nautical  Alma- 
nac, the  moon's  longitude,  latitude,  right  ascension,  declination,  semidi- 
ameter,  or  horizontal  parallax  for  the  noon  and  midnight  immediately 
preceding  Bud  following  the  Greenwich  time,  and  find  their  difference; 
then. 

To  the  proportional  log.  of  this  difference,  add  the  proportional  log.  of 
the  Greenwich  time  past  the  preceding  noon  or  midnight,  (reckoning  the 
hours  as  minutes,  and  the  minutes  as  seconds,)  and  the  constant  logarithm 
8. 823.9  *  ;  the  sum  of  these  three  logsl,  abating  10  in  the  index,  will  be 
the  proportional  log.  of  a  correction  which  is  always  to  be  added  to  the 
moon's  longitude  or  right  ascension  at  the  noon  or  midnight  preceding  the 
Greenwich  time ;  but  to  be  applied  by  addition  or  subtraction  to  her  lati- 
tude, declination,  semidiameter  or  horizontal  parallax,  at  that  noon  or.  mid-, 
night,  according  as  it  may  be  increasing  or  decreasing. 

Nof«.— Since  the  difference  of  the  moon's  longitude  and  right  ascension, 
in  12  hours,  will  always  exceed  the  limits  of  the  Table,  and  also  the  differ- 
ence of  her  declination  in  that  interval,  at  times ;  if,  therefore,  the  one  half 
or  one  third  of , such  difference  be  taken,  and  the  correction,  resulting  there- 
from, multiplied  by  2  or  3,  the  required  correction  will  be  obtained. 

Example. 

Required  the  moon's  longitude,  latitude,  right  ascension,  declination^ 
semidiameter  and  horizontal  parallax,  Aug.  2d,  1824,  at  3!  10?  past  ttoon> 
in  longitude  60?30'  west  of  the  meridian  of  Greenwich  ? 

Apparent  time  at  ship,  or  place,    •    .     .     ,     r:        3M0?- 
Longitude  60?30^  west,  in  time,.      •    .     ,     =:   +  4.   2. 


Greenwich  time,  past  noon,  Aug.  2,  1824,      =         7*12T 
f  The  arithmeticitl  complement  of  the  proportional  log.  of  12  hou^s  esteemed  as  mintU^, 

Digitized  by  VjOOQ IC 


DBSORIPTION  AND   USB  OF  THB  TABJLBS.  81 


To  find  the  Moon's  Longitude. 

Diff.  in  12  hours  =  6?31  ^59?  -*-3=29l0^39|-r,  prop,  log,  =  ,  1391 
Greenwich  time    = 7M2?  =     prop.  log.     =1.3979 

Constant  log. =8. 8239 

^  '" 

One  third  the  corr.  of  the  moon's  long.  =3     1  ?  1 8  ( 25  f  p,  tog,  as  0. 3609 

Multiply  by  ,    ,    ,    ,    .      3. 

Corr.  of  moon's  long.     .    ,    .    .    .  +      3?55'15r 
Moon's  long,  at  noon,  Aug.  2, 1824  .  =7: 17?16^  27r 

Moon's  lon^,  as  required     •    •    .    ,      '7!21?ir.42r 


To  find  the  Moon's  Latitude. 

DiflF.inl2l]iOurs=:23'.35rprop.  log.     .....,:     a    .8827 

Greenwich  time  =   7H2+ prop.  log.     .........      =;  1.3979 

tlonnant  tog^      .....«••     ss  8. 8239 

Correction  of  moon's  lat - 14^  9^  p.  log.  a  1. 104S 

Moon's  lat.  at  noon,  Aiig.  2, 1824,  .     =      4?  6/.59r  south. 

Moon'3  lat.  as  required  .    ...    •    •    •  8?52'50?  south, 


To  find  the  Moon'i  Kigfat  Ascension  :-^ 

Dirf.  in  12  hours  =s  ^?5r.48r  ^3«2?  17^  16ir,  prop.  log.      ss    .  1 177 

Greenwich  time,  :s  .    .    .    i    .    .7*12^     prop.  log.      =1.3979 

Constant  log. .    .=8.8239 

One  third  the  corr.  of  the  moon'«  rtasc.  cs  1?22'22?  p.  tog,  =0.3395 
Multiply  by  .....     3. 

Corr.  of  moon's  right  asc -f-'4?  7-  6^ 

NIoon's  rt.  asc.  at  noon,  Aug.  2,  1824,  s=  223. 33. 36. 


Moon's  right  asci  as  wquirccl    .    .    .      227?40:421' 

o 

Digitized  by 


Google 


82  HBSeaiPTlOK  and  USB  of  tub  TABtlS. 


To  find' the  Moon's  Declination  :— 

biff,  in  12hoifr8=:l?23M3r,  prop.  log.    ......  =    -3325 

Greenwich  time  =  7*12?        prop,  log, =1.8979 

•    •     •  Constant  log. \    .    •  =8.82^ 


(Donrectlbn  of  moon's  dec,    •    .    .    .    •     +  50M4r  p.  Jog.  =  0,6J43 
Moon's  dec.  Aug.  2,  1824,  at  noon, .    :=  20967^  7"-  south. 


Moon's  declination  as  re<pired    •    .    .    2t?47'%l'  south. 

^ofe.— The  cpnectiofti  or  proportional  part  of  the  moon's  jnptioq,  found 
as  above,  must  be  corrected  by  the  e*quatioh  of  second  difference  contained 
in  Table  XVII.,  as  expluned  in  pages  33  and  34. 


To  find  the  Moon's  Semidiameter  :— 

0iff,  in*  12  hours  r:  '    6'/  prop.  log.  •    •    •    ,    • 

Greenwich  time  =:7*  12?  prop.  log.  •    »    •    •    • 

G>n8taiit  log.   • .   •  .  •    •    • 


Corf,  of  the  moon's  semidiametec  •    •    •    •    —    4f 
Moon's  semidiameter  at  noon,  Aug.  2, 1824,2 13-^38 


•  •  !!!?  o«  2553 
.  .  =  1.3979 
.    ..  =  8.8239 


p.  log,  3  3.4771 


Moon's  ^mkBameter  as  required  •  •    •    •    •    15C 29^ 


T«  fiBd  tlie  Moon's  Horisontal  ?wMmX  :— 

Diff.  in  14  houra  r=    28f      prop,  log/  •  .•    .    •.  •    •  •5:2.6717 

Greenwich  time  zsl      7Marprop.log •    «  •^1«3979 

Constantlog.      .«««'..»  .  =  ^8.8239 


Conr.  of  moon's  horiz.  para} «    ;     -^  14?  p.  k>g«  s  2. 8935 

Moon's  horiz.  paraL  at  noon,  Aug.  2^  1824,  s57t  6T 


Moon's  horiz, paraL  as  required    •    •    •    •    56'52^    /• 

iVbfo.— ^The  moon's  semidiameter,  thus  found,  must  be  augmented  by 
the  correction  contained  in  Table  IV.,.  as  jexflained  ia  pagc»  10« 


Digitized  by 


Google 


JDBSOSIPTIOV   AVD   USB  OF  TBB  T1BLB8«  BS 

PROBLBIf   III. 

To  reduce  the  Eight  Ascension  and  Declination  qf  a  Planet,  as  given  in 
the  Nautical  Jlmanacj  to  any  ^ven  time  under  a  knoum  Meridian^ 

RULB, 

Turn  the  longitude  iiitd  tlmey^  and  add  it  lo  the  apparent  time  at  ship  e? 
place  if  it  be  west^'but  subtract  it  if  east ;  and  the  sum^  or  difference^  will  be 
the  corresponding  time  at  Greenwich. 

From  page  IV,  of  the  month  in  the  Nautical  Almanac^  Cake  oot  the  pla» 
net's  right  ascension  and  declination  for  the  nearest  days  preceding  and 
following  the  Chreenwich  time^  and  find  the  difference ;  find,  also,  the 
difference  between  the  Greenwich  time  and  die  nearest  preceding  day ; 
then, — 

To  the  proportional  log.  of  this  difference,  add  the  proportional  log.  of 
the  difference  ef  right  ascension,  or  declination,  and  the  constant  leg. 
9.d031  *;  the  sum^of  these  three  legs.,  rejecting  10  from  the  index,  will  be 
the  proportional  log.  of  a  correction,  which  being  applied  by  addition,  oi 
subtraction,  to  the  right  ascension,  or  declination,  (on  the  nearest  day  pre- 
ceding' the  Greenwich  time,)  according  as  it*  may  be  increasing  or  decreas- 
ing, the  sum,  or  difference,  will  be  the  correct  right  ascension  or  declin-* 
ation  at  the  given  time  and  place.  - 

*  Example. 

Required  the  right  ascension  of  tiie  planet  Venus,  July  3,  1S24,  at 
10^20?  apparent  time^  at  a  place  75?SO^  west  of  the  meridian  of  Green- 
wich ?  ^  . 

Apparent  time  at  given  place,  -=:•••«•     10^20? 
Longitude  75 ?80C  west,  in  time  =     •    •    •    •      +  5.   2. 

Oieenwich  tfane     •••.    =    ••      8  days,  15!  22? 

To  find  tiie  Right  Ascension  :— 

R.  A.  ofVenus,  July  1  =  6*  8?    .    .    .    '.    .  1*.  0*  0? 
Ditto     •    •    •    •    7  =  6. 40.      Gr.  time.=  .  3-.  15. 22 


I 


Diflference.      =  0*a2r     Diff.   ,   =.2-15*22?    s    aS?22?j 
which  are  to  be  Esteemed  as  nunutes  and  seconds  t — Whence,    * 


*  The  arithmetical  complement  of  the  proportional  log.  of  144  hour}  (6  day^}  citeemed 
ummates;  and,  hence  taken  as  2  houfs  and  24  miautes. 

02 


Digitized  by 


Google 


64  DESCRIPTION  AND   USB  OF  THB  TABLES. 

Diff.  bet  G.  time  and  nearest  preceding  day  63*22?  prop,  log,  =  .  4534 

DiflF.  of  right  ascension  in  6  days  .     •     .    •0*32*  prop.  log.  =  .  7501 

Constant  log.      • •-    •    9»903l 


Correction  of  right  ascension       *    .    .    .     +  14C  5^  p.  log,:i:l.  1066 
Planet's  R.  A.  on  July  1,1824  =  ..    •    .  .6*  8?  0! 

R.  A.  ofVenus,  as  required      •    •    #-  •    .    6i22T  5! 

To  find  the  Declinatioiv:— 

Dec.ofVenus,July  1  =:23?36C  N.  .    .    ...  H  0*  Or    . 

Ditto.    .    .    .    .  7  =23.32.  N.Gr.  time  =  3.15.22 


Difference     .    .    0?  4^        Diff.      =2^115*22.?=    63*22?} 
which  are  to  be  esteemed  as  minutes  and  seconds ;  hence,     . 
Diff;  bet.  G.  time  and  nearest' day  preceding  63*22?  prop.  log.  =  .4534 
Diff.  of  declination  in  6  days  =     .    .    .    .  0?  4^  prop.  log.     1. 6532 
Constant  log 9.9031 

Correction  of  declination  =     •    .^      —     n46^p.  log,    =2.0097 
Ranet'sdec.on  July  1,  1824, .    .     =  23.36.  0.  north.  • 

Dec.  of  Venus,  as  required     .     .     •    •  23?34i:  14?  north. 


Tablb  XXX. 

LogQinihmic  Half -elapsed  Time.  • 

This  Table  is  useful  in  finding  the  latitude  by  two  altitudes  of  the  sun ; 
aiid  also  in  otlier  astronomical  calculations,  as  will  be  shown  hereafter. 
The  Table  is  extended  to  every  fifth  second  of  time  under  6  hours,  with 
proportional  parts,  adapted  to  the  intermediate  seconds,  in  the  right  hand 
margin  of  each  page ;  by  means  of  which,  the  logarithmic  half-elapsed  time 
answering  to  any  given  period,  and  converisely,  m.ay  be  readily  obtained  at 
Bight.  . 

As  the  size  of  the  page  would  not  admit  of  the  indices  being  prefixed  to 
the  logs,  except  in  the  first  column,  under  Of,  therefore  where  the  indices 
change  in  the  other  columns,  a  bar  is  placed  over  the  9,  or  left  hand  figure 
of  the  log.,  as  th\is,  9,  to  catch  the  ey^,  and  to  indicate  that  from  thence, 
through  the  rest  of  the  line,  the  index  is  tp  be  taken  from  the  next  lower 
line  in  the  first  column,,  or  that  marked  0!  at' top  and  bottom.  It  is  to 
be  observed,  however,  that  the  indices  are  only  susceptible  of  change  when 
the  half-elapsed  time  is  under  23  minutes. 


Digitized  by 


Google 


DESCRIPTION  AND  USB  OF  THE  TABLES.  B5 

■ 

The  logarithmic  half-elapsed  time  corresponding  to  any  given  period,  is 
to  be  taken  out  by  entering  the  Table  with  the  hours  and  fifths  of  seconds 
at  the  top,  or  next  less  fifth  if  there  be. any  odd  seconds,  and  the  minutes 
in  the  left-hand  column ;  in  the  angle  of  meeting  will  be  found  a  number, 
which  being  diminished  by  the  proportional  part  answering  to  the  odd  se*- 
conds^  in  the  right  hand  margin,  will  give  the  required  logarithm, 

Exatnple* 

Required  tlie  logarithmic  half-elapsed  time  answering  to  2t47*28!  ?  ' 
2*47?25!  answering  to  which  is    .    .    .    ,    ,  0. 17572 
Odd  seconds      «    • .  3.  pro.  part  answering  to  which  is    •     •  -*       II 

Given  time  =  2M7^28?  corresponding  log.  hf.-elapsed  time .    0.  I756I 

In  the  converse  of  this,  that  is,  in  finding  the  time  corresponding  to  a 
given  fog.  j-^if  the  given  log.  can  be  exactly  found,  the  corresponding 
hours,  minutes,  and  seconds,  will  be  the  time  required :— but  if  it  cannot 
be  exactly  found  (which  in  general  will  be  the  case),  take  out  the  hours, 
minutes,  and  seconds  answering  to  the  next  greater  log.; 'the  difference 
between  which,  and  .that  given,  being  found  in  the  column  of 'proportional 
parts,  abreast  of  where  the  next  greater  log.  was  found,  or  nearly  so,  will 
give  a  certain  number  of  seconds,  which  being  added  to  the  hours,  minutes 
and  seconds,  found  as  above,  will  give  the  required  time. 

Example. 

Required  th^  time  corresponding  to  the  logarithmic  half-elapsed  time 
0.14964? 

Sohaion. — ^Th^  next  greater  log.  is  0. 14973,  corresponding  to  which  is 
3^0*25!  ;  the  difference  between  this  log.  and  that  given  ifl  9  ;  ans\vering 
to  which  in  the  column  of  proportional  parts  is.  3  seconds,  which  being 
added  to  the  above  found  time  gives  3*0T28t  for  that  required. 

Amarfc.— The  numbers  in  this.  Table  are  expressed  by  the  logarithmic 
^o-secants* adapted  to  given  intervals-of  time,  the  index  being  diminished 
by  radius,  as  thus  : 

Let^the  hal^elapsed  time  be  3t20T45'.  ;  to  compute  tlje  corresponding 
logarithm. 

Given  time  =:.3^20T45!  in  degrees  =  50?11U51' ;  log.  co-secant  less 
radius  =  0. 1 14557 ;  which,  therefoie,  is  the  required  log. ;  and  since  it  is 
not  necessary  that  this  number  should  be  extended  beyond  five  decimal 
places,  the  sixth,  or  right  hand  figure,  may  be  struck  off;  observing, 
however,  io  increase  the  fifth  figure  by  unity  or  1,  when  the  right  hand 
figure,  so  struck  off,  amounts  to  5  or  upwards : — hence,  the  tabular  number 
corresponding  to  3!20T45  ^  is  0. 1 1456 }  mid  so  of  others. 

Digitized  by  VjOOQ IC 


86  MMCRlPTIdK  AN0  U8B  OF  tBB  tA&lM. 

JLojrari^Amic  Middle  Time. 

Thid  Table  is^  also^  useful  in  finding  the  latitude  by  two  'altitudes  of  the 
sun ;  for  which  purpose  It  is  extended  to  every  fifth  second  under  6  hours^ 
with  proportional  parts  for  the  intermediate  seconds^  in  the  right-hand 
margin  of  each  page ;  by  means  erf  which  the  logarithmic  middle  time  an- 
swering to  a|iy  given  period^  and  conversely^  may 'be  readily  taken  out  at 
sight.  '  . 

As  the  indices  are' only  prefixed  to  the  logs,  in  the  first  column^  there- 
fore where  those  change  in  the  other  columns  a  bar  is  placed  over  the  cy« 
pher^  as  thus,  0,  to  catch  the  eye,  and  to  indicate  that  from  thence  through 
the  rest  of  the  line,  the  index  is  to  be  taken  from  the  next  lower  line,  in 
the  first  column. 

The  Idgarithmie  middle  time  answering  to  any  ghreii  period  is  to  be 
taiken  out  by  entering  the  Table  with  the  hours  and  fifths  bf  seeoiids  at  die 
top,  or  the  nextUssJifih  second  (when  there  are  atiy  odd  seconds,  as  there 
generally  will  be),  ahd  the  mihutes  in  the  left-hand  eolutnn  yvx  the  angle  tff 
meeting  will  be  found  a. number,  which  being  aikgmMbd  by  the  propor- 
tional part  answering  to  the  odd  ^econds^  in  the  compartment  alireast  of  the 
angle  ef  meeting,  will  give  the  log.  required.  ' 

Example. 

Required  the  logarithmic  middle  time  answering  to  •  3  M  7*23?  ? 

3M7':^20!  ahswering  to  which  is  ..    .    ^    •    .  6. 18099 
Odd  seconds      .    •    3.  pro.  part  answering  to  which  is      •    •  +         8 


Given  tune  s  3M7"23!  corresponding  log.  middle  time    •    ..  ^.  18107 

The  time  corresponding  to  a  given  logarithmical  number,  is  founcl  by 
taking  out  the  hours,  minutes,  and  seconds,  dtiswering  to  the  iiext  Use  ta- 
bular number ;  the  differenee  between  which  and  that  given,  being  fostld  in 
the.  compartment  of  proportional  paits,  abreast  of  the  saidnejrt  1^  tabular 
number,  will  give  a  certain  number  of  seconds,  which  being  added  to  the 
hours,  minutes^  and  seeonds  found  as  aboire,  will  be  the  time  required* 

.  Example. 

Required  the  time  eorrespondltig  to  the  bg«  middle  time^      6»  01787  } 

Solution.— The  next  Ic^«  tabular  log.  is  6;  01757,  answering  to  which  is 
2?5?30?  J  the  difference  between  this  log.  and  that  given  is  10,  answering 
to  which  in  the  colunm  of  propQrtiond  parts  is  2  seconds,  which  being 
added  to  the  time  founds  aa  above,  gives  2!5?32!,  for  that  required. 


Digitized  by 


Google 


mscamioN  and  vn  of  thb  tabu8#  $7 

• 

Hemarlc.^^The  logarithmic  middle  time  may  be  readily  eomputed  by  the 
following  rule ;   via  :— ^ 

To  the  logarithmic  sine  of  the  given  time  expressed  in  degrees^  add  the 
constant  log.  Q.  30lQ30|  and  the  sum,  abating  10  in  the  index,  will  be  the 
required  logarithm.  '  .     •• 

Let  the  middle  time  be.  4^  10T25 ' ,  Tequircd  the  corresponding  log* } 
.Given  tim^  =  4^  10^25'.,  in  degs.  =  62?36^  l^r  log.  sine  »  0. 948339 
Constatttlqg«    » •    »  6.301030 


Logarithmic  middle  lime,  as  required  ..;••••=  6. 249369  ; 
and  since  it  is  not  necessary  that  this  log.  should  be  extended  t^yond  five 
plaoei;  of  decimals^  the  sixths  or  rjght-hand  figure  may,  therefore,  be  struck 
off;  observing,  however,  to  increase  the  fifth  figure  by  unity  or  I,  when  the 
right-hand  figure,  so  struck  off,  amounts  io  5  or  upwards ;  hence  the  tabu- 
lar number  corresponding  to  4*  10T25 !,,  is  6. 24937^  and  so  on. 


Table  XXKIU 
Logarithmic  Rising. 

This  Table,  vrith  the  two  preceding,  is  particularly  useful  in  finding  the 
latitude  by  two  altitades  of  tfie  sUn )  it  is  also  of  oon^derable  ute  in  mtfny 
other  asti'onomical  calculations,  such  as  'h  computiBg  the  apparent  time 
from  the  altitude  of  a  celestial  object;  determining  the  altitude  of  a  celes- 
tial obyeel  from  the  i^parent  time,  &c.  &ew— -The  arrangement  of  the  pre-* 
sent  Table  is  so  yery  uniform  with  the. preceding,  that  it  is  not  deemed 
necessary  to  enter  into*  its  description  any  farther  than  by  observing  that 
the  indices  are  only  prefixed  to  the  logs,  in  the  first  column  :— thfit  where 
those  change  in  the  other  columns,  large  dots  are  introduced  instead  of  O's 
to  catch  the  eye,  for  the  purpose  of  indicating  that  '^rom  thence  through 
the  rest  of  the  line,  the  index  is  to  be  taken  from  the  next  lower  line  in  the 
first  column ;  and  that,  in  the  general  use  of.  the  Table^  these  dots  are  to 
be  accounted  as  cyphers.  •  • 


Digitized  by 


Google 


88  BKSCRIPTIOK  AND  -USE  OP  THB  TABLSS*. 

Required  the  logarithmic  rising  pmsweringto  1M3?27'  ? 

1*43725!,  answering  to  which  is    •     .    .    •    •  5.00040 
Odd  seconds    •    •  2,  pro.  paiit  answering  to  which  \s    •  *  •  28 


Given  time  =  1M3T27 ',  conesponding  logarithmic  rising        5. 00068  . 

The  converse  of  this,  that  is,  finding  the  time  corresponding  to  a  given 
log.  will  appear  obvious ;  thus,  . 

Let  the  given  logarithmic  rising  be  5.^9088,  to  find  the  corresponding 
thne.  *.  • 

The  next  less  tabular  log.  is  5. 66071,  answering  to  which  i»3*57"30' ; 
the  difference  b^tV^een  this  log.  and  that  given  is^  17>  answering  to  which 
in  tKe  column  of  proportional  parts,abreastof  the  tabular  log.,  is  3  se- 
conds ;  now,  this  being  added  to  the  tifnefouiid,  as  above,  gives  3^57*33' ; 
which,  therefore,  is  the  time  corresponding,  to  the  given  logarithmic 
rising.  # 

Nbfe.— The  numbers  in  this  Table  were  computed,  by  the  following 
rule,  vt« : —  *       *      ^ 

To  twice  the  logarithmic  sine  of  half  the  meridian  distance,  in  degrees, 
add  the  constant  log.  6. 301080,  and  the  sum,  rejecting  20  from  the  index^ 
will  be  the  logarithmic  riaiifig.    * 

Example. 

Required  the  logarithmic-rising  answering  to  4^  10^45'.  ? 
(Kven  meridian  distance  =:  4^  10T45  ?,  in  degrees  =  62?41  i  15f 


Halfthe  meridian  distance,  in  degrees  •    •    •    •  =:  3 1 .  20. 37^9  twice  the 

logarithmic  sine     ••••••*•••••.•    •    19.432293 

Constant  log, .    ••...*...      6.301030 


Logarithmic  rising  answering  to  the  given  meridian  distance  r:  5, 733323 

Hie  numbers  in  the  present  Table  may  be  also  computed  by  means  of  tlie 
natural  versed  sines  contained  in  Table  XXVIL,  as  thus  ; 

Reduce  the  meridian  distance  to  degrees,  and  find  the  natural  versed 
sine  corresponding  thereto;  the  common  log,  of  Which  will  be  the  loga- 
rithmic rising. 


Digitized  by 


Google 


BifiCRIPTION  AND  TTSB  OF  THB  TABUTS.  89 

Example. 

Required  the  logarithmic  rising  answering  to  4^22r30^  or  65?37^30^  ? 
Meridian  distance  in  degrees  =  eS^S/'SO"^,  natural-versed  sine  =  587293^ 
log.  :s=  5. 768855-5  which^  therefore,  is  the  logarithmic  rising  answering 
to  the  given  meridian  distance* 

In  this  method  of  computing  the  logarithmic  rising,  the  natural  versed. 
9ine  is  to  be  conceived  as  being  multiplied  by  1000000,  the  radius  of  the 
Table^  and  thus  reduced  to  a  whole  number. 


Table  XXXIIL 
To  reduce  Pointf  qfthe  Compass  to  Degrees,  and  conversely. 

This  Table  is  divided  into  six  columns ;  the  two  first  and  two  last  of 
which  contain  the  names  of*  the  several  points  and  quarter  points  of  the 
compass ;  the  third  column  contains  the  corresponding  number  of  points 
and  quarter  points' reckoned  from  the  meridian;  and  the  fourth  column  the 
degrees  and  parts  of  a  degree  answering  thereto.-7-The  manner  of  using 
this  Table  is  obvious  )  and  so  is  the  method  by  which  it  was  computed  :— - 
for  since  the  whole  compass  card  is  divided  into  32  points,  and  the  whole 
circle  into  360  degrees ;  it  is  evident. that  any  given  number  of  points  will 
be  to  their  corresponding  degrees  in  the  ratio  of  32  to- 360 ;  and  vice  versa, 
that,  any  given  nXimber  of  degrees  will  be  to  their  corresponding  points  as 
360  is  to  32  : — H^nce,  to  find  the  degrees  corresponding  to  one  point.— 
As82f  :  360?  ::  If:  11?15:;  so  that  one  point  contains  11  degrees  and 
15  minutes  ) — two  points,  22  degrees  30  minutes,  &c.  &c. 


Tabus  XXXIV. 

Ijogarithmic  Sines,  Tangents,  and  Secants,  to  every  Point  and  Quarter 
Point  of  the  Compass* 

In  this  Table  the  points  and  quarter  points  ate  contuned  in  the  left  and 
right  hand  maq;inal  columns,  apd  the  log.  sines,  tangents,  and  secants, 
correisponding  thereto^  in  the  intermediate  columns. 

If  the  course  be  given  in  points,  it  will  be  found  more  convenient  to  take 
the  log.  sine,  tangent,  or  secant  of  it  from  this  Table,  than  to  reduce  those 
points  to  degrees,  and  then  find  the  corresponding  log.  sine,  &c.  &c.  in 
either  of  the  following  Tables.— The  mariner  of  using  this  Table  must 
appear  obvious  at  first  sight. 


Digitized  by 


Google 


80  usomipnoif  avo  vsb  or  thb  TAVMi 

TAButJOCXV. 

Logarithmic  Secants. 

In  the  first  10  degrees  of  this  Table,  the  logarithmic  secants  are  giren  to 
arery  tet^h  second,  with  proportional  parts,  answering  to  the  intermediate 
seconds,  iti  the  right  hand  marginal  colniiin. — ^Thence  to  88  degrees,  the 
log.  secants  are  given  to  every ^ft  second,  with  proportional  parts,  adapted 
to  the  intermediate  seconds,  in  the  right  hand  column  of  each  page  :— ^tmd 
because  the  numbers  in<;rease  rapidly  between  60  and  88  degrees,  produc- 
ing very  considerable  diflferences  between  any  t^o  adjacent  logs. ;  there- 
fore betwixt  those'  limits,  there  are  two  pages. allotted  to  a  degree ;  every 
page  being  divided  into  two  parts  of  15  minutes  eacb^  so  that  ub  portion 
whatever  of  the  proportional  parts  might  be  lost,  and  that  the  whole  might 
have  room  to  be  fully  inserted. — In  the  twq  last  degrees,  viz.  from  88  to 
90,  the  log.  secants  are  gTven  to  every  second.— The  Table  is  so  arranged 
as  to  be  extended  to  every  second  in  the  -semicircle,  or  from  0  to'  180  de- 
grees ;  as  thus  t  the  arcs  corresponding  to  the  log.'  secants  are  giv^n  in 
regular  succession  at  top  from  0  t&  90  degrees,  and  then  continued  at  bot- 
tom, reckoning  towards  the  left  hand,  from  90  to  180  degrees  2-*the  arcs 
correspdnding  to  the  eo-secants  are  placed  at  t^e  bottom  of  the  Tabled  in 
numerical  order,  from  the  right  hand-  towards  the  left  (li^e  the  secants  in 
the  second  quadrant),  from  0  to  90  degrees,  and  then  continued  at  top^ 
agreeably  to  the  order  of  the  secants  in  the  first  quadrant,  from  90  degrees 
to  the  end  of  the  semicircle.— Tins  mode  of  arrangement,  besides  doing 
away  with  the  necessity  of  finding  the  supplement  of  an  arch  when  it* 
exceeds  90  degrees,  possesses  the  peculiar  advantage  of  ^a^ling  the 
reader  to  take  out  the  log.  secant,  or  co- secant  of  any  arch  whatever,  and 
conversely,  at  sight,  as  will  appear  evident  by  the  followhig  problems. 

No/e.— The  log.  co-secant  of  a  giyen  degree,  or  secant  of  a  degree  above 
90,  will  be  found  in  the  same  page  with  the  next  less  degree  in  the  first 
column  under  0^  at  top,  it  being  the  first  number  in  that  column  i  and  the 
log.  co-secant  of  a  given  degree  and  minute,  or  secant  of  a  degree  and 
minute  above  90,  will  be  found  on  the  aame  line  vrith  the  neati  less  minnto 
in  the  eohunn  macked  60jf  at  bottom  of  th^  Tia)le. 


Digitized  by 


Google 


'»88CfttPtlO!f  ANI>  VM  OF  THS  tABUEI.  01 


Problbm  i.  • 

To  find  ihe  LogarUhndc  Secant^  and  Ca-secatU  of  any  given  Jrchy 
expresied  in  IhgreUy  ffSnuleSf  and  Seconds* 

• 
If  the  gir^n  ftrch  bfe  cotxiprieed  iVlthiti  the  limits  of  the  two  last  degrees 
qf  the  first  quadrtirit^  that  is,  between  88  and  90  degrees,  the  Table  will 
direotljr  exhibit  hs  corresponding  log.  sceant  or  co*secant ; — but  when  it 
falk  without  those,  limits,  then  find  the  log.  secant,  or  co-secant,  in  the 
^iigfe  of  meethig  nrade  bjr  the  giren  degree  and  il^t  less  fifth  or  tenth 
second  at  top,  and  iht  iiiinutef  in  the  lefthand  coliitnn ;  to  which,  Aid 
the  propoftiotial  ptirt  eorrespcaiding  to  th^  odd  seconds  from  the  right  hand 
column  abreast  of  the  angle  of  meeting,  if  a  secant  be  wanted,  ot«a  co-ie^ 
eoKt  above  90  iegtet9  ;  but  subthict  that  part  when  a  co-secant  i^  required, 
or  a  secant  fibove  90  degrees  ;  and  the  sum,  or  difiierence,  will  be  the  \og. 
secant  or  co-secant  answering  to  the  given  arch. 

Example  U 

Reqmred  the  logarithmic  secant^   and    co-secant»  correspoEding   to 
23?14:23r? 

To.find  the  Log«  Secant  :— 

S3?14;3(K^iin8«lo.whiehis  «    i    .    10.086747 
Odd  sccoiMb  3    pnqpor.  part  to  which  is  +  8 


Gitenarch  =r  88?I4!2af  Corres.log.  secant  =:  10.036750 

To  iind  the  Log.  Co-secant  :-^ 

23?l4^20r,ans.towhichis      .    .  10.403881 

Odd  seconds                 3    propor.  part  to  which  is  —             15 


lii  n. 


Girenarcbs  23?14C23r  Corres.  log:  co-secant  =  10.403866 

Be^dred  th*  leg.  smmt,  and  co^eaot,  eoirespoiKliog  to  Ud?83M7?  } 

y  Google 


Digitized  by ' 


92  .  PBSCRIPTION  AND  USB  OF  THE  TABUBS. 

To  find  the  Log.  Secant  :— 

113?23M5r,ans.  towhichis     .    .     10.401121 
Odd  seconds  2    propor.  part  to  which  is   -r    '     10. . 

Given  arch  =;  1 13?23U7?  Corres.  log.  secant  r:      10. 401 11 1 

To  find  the  Log.  Co-secant :— * 

113?23M5r,au8.*to  which  IS    •    ,    10.037260 
Odd  seconds  2    propor.  part  to  which  is  +  2 

%       • 

Given  arch  =  113?23<477  Corres.log.  cb-8ccant=:l 0.087262 

Note. — In  that  part  of  the  Table  which  lies  between  10  and  80  degrees, 
the  size  of  the  page  would  not  admit  of  the  indices  -  being  prefixed  to  ahy 
other  logs,  than  those  contained  in  the  first  jrolumn  of  each  page ;  nor,  in- 
deed, is  it  necessary  that  they  should  be,  tince  they  are  uniformly  the  same 
as  those  contained  in  the  said  first  coliimn;  viz.,  10  for  each  log.  secant  or 
co-secant* 


PbpBLEM  n. 

To  find  the  Arch  corresponding  to  a  gwen  Logarithmic  Secant  or 

Co-secant: 

If  ihe  given  log.  secant,  or  co-sectuit,  exceeds  the  secant  of  88  degrees, 
viz.,  1 1. 457181,  its  corresponding  arch  will  be  found  at  8rst  sight  in  the 
Table ;  but  if  it  be  under  tliat  number,  find  the  arch  answering  to  the  next 
less  secant,  or  next  greater  co-secant;  tfie  difierence  b^twe'en  which  and 
that  given,  being  found  in  the  column  of  proportional  parts,  abreast  of  the 
tabular  log.,  will  give  a  certain  number  of  seconds,  which,  being  added  to 
the  above-found  arch,  will  give  that  required. 

Example  I.  ^     . 

Required  the  arch  corresponding  to  the  given  log.  secant  10. 235421  ? 

Sblufion.— The  next  less  ^cmt,  in  the  Table,  is  10.235412,  com- 
sponding  to  which  is  54?  26^25  ^ ;  the  difierence  between  this  log.  secan^ 
and  that  given,  is  9 ;  answering  to  which,  in  the  column  of  proportional 
jparts  abreast  of  the  tabular  log.,  is  3? ;  which,  beii\g  added  to  the  al^ve- 
found  arch,  gives  54?26U8r  for  that  required. 


Digitized  by 


Google 


DSSCRIFTION   AND   USB   01^  THB  TAfiJLBS.         ,  93 

Example  2. 
Required  the  arch  corresponding  to  the  given  I<^.  co-secant  10«  5621 14  ? 

So&iiion.— The  next  greater  co-secant,  in  the  Table,  is  10.562129, 
corresponding  to  which  is  15945 '25r;  the  difference  between  this  log. 
co-secant  and  that  given,  is  15 ;  answering  to  which,  in  the  column  of  pro- 
portional parts  abreast^  of  the  tabular  log.,  is  21  *,  which,  being  added  to 
the  above-found  arch,  gives  15?45'.27?  for  that  required. 

Remark. — ^llie  log.  secant  of  any  arch  is  expressed  by  the  difference 
1)etween  twice  the  radius  and  the  log.  co-sine  of  that  arch ;  and  the  co- 
secant of  an  arch,  by  the  difference  between  twice  th^  radius  and  the  log. 
sine  of  such  arch.  Hence,  to  find  the  log.  secant  of  50M0'. — ^The  log. 
co-sine  of  50?40^  is  9,  8019^4,  which,  being  taken  from  twice  the  radius, 
viz., 20. 000000,  leavea  10. 198026  for  the  log.  secant:  from  this,  the 
manner  of  computing  the  co-secant  will  be  obvious. 


Tablb  XXXVI. 

ZfOgarithmic  Sines. 

Of  all  the  Logarithmic  Tables  in  this  work,  this  is,  by  far,  the  most 
generally  useful,  particularly  in  the  sciences  of  Navigation  and  Nautical 
Astronomy ;  and^  therefore,  much  pains  have  been  taken  in  reducing  it  to 
that  state  of  simplicity  which  appears  to  be  best  adapted  to  its  direct 
application  to  the  many  other  purposes  for  which  it  is  intended,  besides 
those  above-mentioned. 

In  this  Table,  the  log.,  sines  of  the  two  first  degrees  of  the  quadrant  are 
given  to  every  second.  The  next  eight  degrees,  viz^  from  2  to  10,  have 
their  corresponding  log.  sinesf  to  every  fifth  second,  with  proportional  parts 
answering  to  the  intermediate  seconds  in  the  adjacent  right-hand  column ; 
and  because  the  log.sines  increase  rapidly  in  those  degrees,  two  pages  are 
allotted  to  a  degree ;  every  page  being  divided  into  two  parts,  and  each 
part  containing  15  minutes  of  a  degree :  so  that  no  portion  whatever  of 
the  proportional  parts  might  be  lost,  and  that  the  whole  might  have  room 
to  be  fully  inserted.  In  the  following  seventy  degrees,  that  is,  from  10  to 
80,  the  log.  sines  ar^  alsp  given  to  every  fifth  second,  wjth  proportional 
parts  corresponding  to  the  intermediate  seconds  in  the  right-hand  column 
of  each  page.  In  this  part  pf  the  Table,  each  page  contains  a  degree;  and, 
for  want  of  sufficient  room,  the  indices  ar^  only  prefixed  to  the  logs,  ex- 
pressed in  the  first  column. 


Digitized  by_ 


Gqogle 


94  ^         2>pscEiFnofir  and  usb  of  thb  tabu|8. 

From  80  to  90  degrees^  the .  log.  sines  are  only  given  to  every  tenth 
second,  because  of  the  small  increments  by  which  the  sines  increase 
towards  the  end  of  the  first  quadrant ;  the  pr opordooal  parts  for  thf{  inter- 
mediate seconds  are  giveain  the  right-hand  column  of  each  page^  as  in  the 
preceding  part  of  the  Table. 

The  Table  is  so  arranged,  as  to  be  extended  to  every  second  in  the 
Beraicirde,  t)r  from  0  to  180  degrees ;  as  thus  :  the  arcs  corresponding  to 
the  log.  sines  are  given  in  regular  succession  at  top,  from  0  to  90  d^prees^ 
and  then  continued,  at  bottom,  reckoning  towards  the  left  hand,  from  90 
to  180  degrees.  The  arcs  corresponding  to  the  co-siines  are  given  at  At- 
torn of  the  Table,  and  ranged  in  numerical  order  towards  the  left  hand^ 
from  0  to  90  degrees,  (according  to  the  order  of  the  sines  between  90  and 
180  degrees,)  and  then  continued  at  top,  from  90  degrees  to  the  end  of  the 
semicircle,  agreeably  to  the  order  of  the  sines  in  the  first  quadrant.  This 
mode*  of  arrangement  does  away^th  the  necessity  of  finding  the  siipple- 
ment  of  an  arch*  when  it  Exceeds  90  degrees,  and  possesses  the  peculiar 
advantageof  enabling  the  navigator  to  take  out  the  log.  sine  or  co-sine  of 
any- arch,  and  conversely,  at  sight,  as  will  appear  obvious  bjr  the  following 
Problems. 

JNbto.— -The  log.  co-sine  of  a  given  degree  is  found  in  the  same  page  with 
the  next  less  degree  in  the  column  marked  O''  at  top,  it  being  the  firtt 
number  in  that  column ;  and  the  co-sine  of  a  given  degree  uid  minute  is 
found  on  the  same  line  with  the  nest  le$$  minute  in  tba  ooluiyiii  mAAed 
60' at  bottom  of  the  page.  •-       '      ^ 


Problem  I. 

To  find  the  Logarithmic  Sine,  and  Co-^ine  ofemy  pven  Arch,  espreseed 
in  Degrees,  MmUeSy  and  Sewnds. 

RULB. 

If  the  given  arch  be  comprised  within  the  limits  of  the  two  first  degrees 
of  the  quadrant,  the  Table  will  directly  exhibit  its  corresponding  log.  sine 
or  co-sine;  but  when  it  exceeds  those  limits,  then  find  the  log.  sine,  or  co- 
sine, in  the  an^le  of  meeting  made  by  the  given  degree  and  next  less  fifth 
or  tenth  second  at  top,  and  the  minutes  in  the  left-hand  column ;  to  which 
add  the  proportional  part  corresponding  to  the  odd  seconds  in  the  right- 
hand  column  abreast  of  the  angle  of  meeting,  if  a  sine  be  wanted,  at  a 
co-sine  above  90  degrees;  But  subtract  that  part  when*  a  co-sine  is  required^ 
or  a  sine  above  90  degrees:  and  the  sum,  or  difference,  will  be  (he  log.  sine, 
or  co-sine,  answering  to  the  given  arch. 


Digitized  by 


Google 


jmSCtlPTIOM  AMD  USB  OF  TUB  TlBLBt* 

Example  I. 

Required  the  log.  sine,  and  co-sine,  corresponding  to  23?I4'23f  ? 

To  find  the  Log.  Sine  :-^ 

23?14^20r,an8.  towhichis  .    .    .    9.596119. 
Odd  seconds  3     propor.  part  to  which  is  +  \5 


Given  arch  =  23?14C23r  Corresponding*  log.  sine  9.596134 

To  find  the.  Log.  G)-8iqe : — 

23?14^20r,an8.towhiehi8   •    •    .  9.96S253 

Oddseeonds                 S    propor.  pwt  to  whieh  !•  «-            8 


-•m 


Given^ch  ==  23?14C23r  Corresponding  log; op-siiie  9, 968250 

Example  2. 

Required  the  log.  sine^  and  co-sine^  conespondipg  U>  113?23'47?  ? 

To  find  the  Log,  Sine  s— 

I13?SSU57,en8.towhiehi8      .    .    9.962740 
Odd  iseeonda  2    propor.  part  to  which  is  ^  3      « 

limn ■       '■       ^» 

Given  arch  s  113?23M7r  Conmpondiog  log.  sine  9.062798 

To  find  the  Log.  Cocaine  :«*- 

ll$?23'45r,ans.towhichi6     •    ,    9,598879 
Odd  seconds  2    propor.  part  to  which  is  +         10 

Given  arch  =     1 13?23 :47?  Corresponding  log.  co-s.  9. 598889 

Problem  U. 
Tojind  the  Arch  corfeipMding  to  a  given  LogaHthmic  Smef,  or  Co^fmf^ 

RULB. 

If  the  given  log.  sine,  or  co-sine^  be  less  than  the  sine  of  2  degrees,  viz., 
8. 54281 9»  its  corresponding  arch  will  be  found  at  first  sight  in  the  Table  3 
but  if  it  exceeds  that  number^.find  the  arch  an9wering  to  the  next  less  sine, 
or  next  greater  co^s|ne ;  the  difference  between  which  and  that  given,  being 
found  ia  the  column  of  proportional  parts  abreast  of  the  tabular  log.,  will 
give  e  certain  number  of  seooodaj  wUcb,  being  added  to  the  above-found 
arcbj  will  give  that  required. 


Digitized  by 


Google 


96  DBSCaiFTION   AND   USE  OF  THB  TABLES. 

Abf^.— Since  the  arcs  corresponding  to  the  sines  between  90  and  180 
degrees  are  found  at  the  bottom  of  the  Table^  and  those  corresponding  to 
the  c6-sines  between  the  same  limits  at  its  top ;  if,  therefore,  it  be  required 
to  find  the  arch  above  90  degrees  answering  to  a  given  log.  sine,  or  co-sine, 
the  first  term  is  to  be  taken  out  as  if  it  w^e  a  cosine  tinder  90  degrees^  and 
the  other  term  as  if  it  were  a  sine  und[er  90  degrees. 

Example  I. 

Required  the  arch  corresponding  to  the  given  log.  sine  9. 437886  ? 

Sohdionj^^The  next  less  log.  sine  in.the  Table  is  9. 437871,  correspond- 
ing to  which  is  15?54^25^ ;  the  difference  between  this  and  that  given,  is 
15 ;  answering  to*  which,  in  the  column  of  proportional  parts,  abreast  of 
the  tabular  log.,  is  2';  which,  being  added  to  the  above-found  arch,  gives 
15?54'. 27T  for  that  required. 

Example  2. 

Required  the  arch  corresponding  to  the  given  log.  co-fsine  9. 764570  i 

Sbltifion.— The  next  greater  co-sine  in  the  Table  is  9. 764588,  corre- 
sponding to  which  is  54?  26 '25?;  the  diffel^nce  between  this  and  that 
given,  is  9  J  answering  to  which,  in  the  column  of  proportional  parts, 
abreast  of  the  tabular  log.,  is  8^3  which,  being  added  to  the  above-found 
arch^  gives  54?26C28r  for  Aat required. 

JSem'arfc.— The  log.  ^ines  are  deduced  directly  from  the  natural  sines ;  as 
thus: — 

Multiply  the  ftatural  sine  by  10000000000;  find  the' common  log.  of 
the  product,  and  it  will  be  the  log.  sine. 

Example  1. 

Require  the  log.  sine  of  S9?30'  ? 

Sblutioii.— The  natural  sine  of  39?30^  is  .  636078,  which,  being  multi- 
plied by  10000000000,  gives  6360780000.000000,  the  common  log.  of 
which  is  9. 8035 1 1 5  which,  therefore,  is  the  log.  sine  of  39  degrees  and  30 
minutes,  as  required. 

Example  2. 

Required  the  log.  co-sine  of  68  degrees  ?  \ 

Solutim.rr-The  natural  co-sine  of  68  degrees  is  .  374607,  which,  being 
multiplied  by  10000000000,  gives  3746070000. 000000,  the  common  log. 
of  which  is  9, 573575  j  which,  therefore,  ia  the  log.  co-sine  of  68  degrees^ 
as  required.       • 


Digitized  by 


Google 


BBSCRIfTION  AND  USB  OP  THB  TABLES.  97 

Table  XXXVII. 

LogarUhmic  Tangents. 

Tliis  Table  is  arranged  in  a  manner  so  very  nearly  similar  to  that  of  the 
I<^.  sines,  that  it  is  not  deemed  necessary  to  enter  into  its  description  any 
fiEUther  than  by  observing,  that  it  is  computed  to  every  second  in  the  two 
first  and  two  last  degrees  of  the  quadrant,  or  semicircle,  and  to  every  fifth 
second  in  the  intermediate  degrees.  The  log.  tangent,  or  co- tangent,  of  a 
given  arch,  and  conversely,  is  to  be  found  by  the  rules  for  the  log.  sines  in 
pages  94  and  95. 

Example  1. 

Required  the  log.  tangent,  and  co-tangent,  correspondbg  to31?10M7?? 

Tf  find  the  Log.  Tangent  :— 

31?10M5r,an8.towhichis    .    •    •    9.781846 
Odd  seconds  2    propor.  part  to  i^hich  is  +  10 


Given  arch  =  31?10^47^  Corre8pondinglogitang.=:9.  781855 

To  find  the  Log.  Co-tangent  :-— 

31M0:45r,ans.towhichis      .    .     10.218155 
Odd  seconds  2    propor.  part  to  which  is   —  10 


Givenarchzi  31?  10^47^  Corrcs.  log.  co-tang.  =    10.218145 

Example  Z 

Requiied  the  log.  tangent,  and  co*tangent,  corresponding  tol39?  1 1  '53?? 

To  find  the  Log.  Tangent  :— 

i39?lll50r,ans.towhichis      .    .    9.936142 
Odd  seconds  3     propor.  part  to  which  is   —  13 

CHvenarch=:  139911C53?  Corres.log.tang.  =    .    9.936129 

To  find  the  Log.  Co-tangent  :-?* 

139?in50r,  ans.  to  which  is    •    .     10. 063858 
Odd  seconds  8    propor.  part  to  which  is  +  13 


Givenai€b=:  139?lH53r  Corres. log. co-tang.  =:  10.063871 

/Google 


H 

Digitized  by ' 


98  DBSCRIPnOV  AffP  USB  09  m  TABUS. 

Eicampk  3. 

Required  the  arch  corresjtonding  to  the  given  log.  tang,  10. 155436  ? 

Sohition.'^Tht  next  leu  log.  tangent  in  the  Table  U 10. 165428,  cone- 
sponding  to  which  is  55?2'25f ;  the  difference  between  this  log.  tlWgent 
^d  that  given^  is  13;  answering  to  which,  in  the  column  of  proportional 
parts  abreast  of  the  tabular  log.,  is  3r ;  which,  being  added  to  the  above- 
found  archi  gives  55  ?  i2  ^  28 T  for  that  required. 

Example  4. 
Required  the  arch  corresponding  to  the  given  log.  co-tang.  9. 792048  ? 

Solutim.-^The  next  greater  Ipg.  co-tangent  in  the  Table  is  9. 792057, 
corresponding  to  which  is  58?  13'  ISI' ;  the  difference  between  this  log.  co- 
tangent and  that  given,  is  9 ;  answering  to  which,  in  the  column  of  propor- 
tional parts  abreast  of  the  tabular  log.,  is  2^ ;  which,  being  added  to  the 
above-found  arch,  gives  58?  13U7  •  for  that  required. 

Remark. 

The  arch  corresponding  to  a  given  log.  tangent  may  be  found  hj  means 
of  a  Table  of  log.  sines,  in  the  following  manner }  vb^. 

Find  the  natural  number  corresponding  to  twice  the  given  log.  tangent, 
rejecting  the  index,  to  which  add  the  radius,  and  find  the  common  log.  of 
the  sum;,  now,  half  this  log.  will  be  the  log.  secant,  less  radius,  of  the 
required  arch ;  and  which^  being  subtracted  from  the  given  log.  tangent, 
will  leave  the  log.  sine  corresponding  to  that  arch. 

EsfOimple. 

Let  the  given  log.  tangent  be  10. 064 158 ;  nquivad  theiuidi  Aomapond- 

ing  thereto  by  a  table  of  log.  sines  ? 

Given  log;  tang. .  084153  x  2  =  .  168306,  Nat  num.  =:  1. 473349 

to  which  add  the  radius  =  1. 000000 


Sum= 2.473349,      the 

common  log.  of  which  is  0. 393286 ;  the  half  of  this  is  0. 196643,      the 

secant,  less  radius  of  the  required  arch. 

Given  log.  tangent  =  . •    •    .  10.084153 


Corresponding  log.  sine  =    ...,.»••.    9.887510« 

answering  to  which  is  50?31' ;  and  which,  therefore,  is  the 
required  arc)i  eomespooding  to  the  given  log*  tmgipu 


Digitized  by 


Google 


DMCAIFnON  AM]>  USB  OF  THB  TABLB8. 


9Q 


Tlie  arch  corresponding  to  a  given  log.  tangent  may  also  be  found  in  the 
following  manner^  which,  it  is  presumed,  will  prove  both  interesting 
and  instructive  to  the  student  in  this  department  of  science. 

Find  the  natural  tangent,  that  is^  the  natural  number  corresponding  to' 
the  given  log.  (angent,  to  the  square  of  which  add  the  square  of  the  radius ; 
extract  the  square  root  of  the  sum,  and  it  will  be  the  natural  secanf  corre- 
sponding to  the  required  arch ;  then,  say,  as  the  natural  secant,  thus  found, 
is  to  the  natural  tangent,  so  is  the  radius  to  the  natural  sine :  now,  the 
degrees,  &c,  an9Wering  to  this  in  the  Table  of  Natural  SineSy  will  be  the 
arch  reqii|^ed^  or  that  corresponding  to  the  given  log,  tangent* 


Esample^ 

Let  the  givei^  log.  tangent  be  10;  084153  $  it  is  required  to  (n4  the  arch 
corresponding  thereto  by  a  Table  of  Natural  Sines  ? 

Soluium. — Given  log.  tangent  =  .084153;  the  natural  number  corre- 
sponding to  this  is  1.213816;  which,  therefore,  is  the  natural  tangent 
answering  to  the  given  log.  tangent. 


F- 


In  the  anne3^e4  diagnmn,  let  B  C  represent  the 
natural  tangebt  =:  1. 213816,  and  AB  the  radius 
==  1. 000000.    Now,  since  the  base  and  perpendi- 
cular  of  the    right«angled    triangle    ABC   are 
known,  the  hypothenuse  or  secant  AC  may  be 
determined  by  Buclid,  Book  L,  Prqs.  47.     Hence 
^/BC«=  1.213816*  +  AB>=    1.000000*  = 
A  C  =  1. 572689,  the  natural  secant  corresponding 
to  the  given  log.  tangent.     Having  thus  found  the  natural  secant  A  C,  the 
natural  sine  DE  may  be  found  agreeably  to  the  principles  of  similar  tri-' 
angles,  as  demonstrated  in  Euclid,  Book  VI.,  Prop.  4 ;  for,  as  the  natural 
secant  A  C  is  to  the  natural  tangent  B  C,  so  is  the  radius  AD  =:  AB  to 
the  natural  sine  D  E :  hence, 

AsAC  1.572689  :BC  1.213816  ::  AP  1.000000  :  DE- 7718JO, 
the  corresponding  natural  sine ;  now,  the  arch  answering  to  this,  in  the 
Table  of  Natural  Sines,  is  50?31C ;  which,  therefore,  is  the  arch  corre- 
sponding to  the  given  log.  tangent,  as  required. 


JWo/e^^^The  Table  of  log.  Qingents  may  be  very  readily  deduced  from 
Tables  XXXV.  and  XXXVI.,  as  thus  :— To  the  log.  secant  of  any  given 
aich,  add  its  log.  sine;  and  the  sum,  abating  10  in  the  index,  will  be  the 
Ii^.  tangent  of  that  arch;  the  difference  between  which  and  twice  the 

radiiia,  will  be  its  co*tangent. 

h2 


Digitized  by 


Google 


100  BBSC&IFTION  ANJ>  USB  OF  THB  TABLB8. 

Example. 

Required  the  log.  taDgent,  and  co-tangent,  of  25?27 -35^  ? 

Log.  secant  of  the  given  arch  25?27'35r    =     10.044366 
Log.  sine  of  ditto 9.633344 

Log.  tangent  corres.  to  the  given  arch  =:     •      9.677710 

Log«  co-tangent  corres.  to  ditto      •    ;    •    •    10.322290 

The  Table  of  log.  tangents  may  also  be  computed  in  th€  following  man- 
ner ;  viz.. 

From  the  log.  sine  of  the  given  arch,  the  index  being  increased  by  10, 
subtract  its  log.  co-sine,  and  the  remainder  will  be  the  log.  tangent  of  that 
arch;  the  difference  between  which  and  twice  the  radius,  will  be  its  log. 
co-tangent. 

Example. 

Required  the  log.  tangent,  and  co-tangent,  of  32?39U0r  ? 

Log.  sine  of  the  given  arch  32?39 U07  =  .    .    9. 732 1 28 
Log.  co-sine  of  ditto 9. 925249 


Log.  tangent  corres.  to  the  given  arch  r=     .    «    9.806879 


Log.  co*tangent  corresponding  to  ditto       •    •  10. 193121 


Table  XXXVIII. 

7b  reduce   the   Time  of  the  Moon^s  Passage  over  the  Meridian  qf 
Greenwich,  to  the  Time  of  her  Passage  over  any  other  Meridian. 

The  daily  retardation  of  the  moon's  passage  over  the  meridian,  given  at 
the  top  of  the  Table,  signifies  the  difference  between  two  successive  trans- 
its of  that  object  over  the  same  meridian,  diminished  by  24  hours ;  as  thus: 
the  moon's  passage  over  the  meridian  of  Greenwich,  July  22d,  1824,  is 
21  *7'^^  and  that  on  the  following  day  22*9?;  the  interval  of  time  between 
these  two  traxisits  is  25^2?,  in  which  interval  it  is  evident  that  the  moon  is 
1^2?  later  in  coming  to  the  meridian;  and  which,  therefore,  is  the  daily 
retardation  of  her  passage  over  the  meridian. 


Digitized  by 


Google 


DBSCRIPTION  AND   USB  OP  THB  TABLES.  101 

This  Table  contains  the  proportional  part  corresponding  to  that  retard- 
ation and  any  given  interval  of  time  or  longitude ;  in  computing  which,  it  is 
easy  to  perceive  that  the  proportion  was. 

As  24  hours,  augmented  by  the  daily  retardation  of  the  moon's  transit 
over  the  meridian,  are  to  the  said  daily  retardation  of  transit,  so  is  any 
given  interval  of  time,  or  longitude,  to  the  corresponding  proportional  part 
of  such  retardation.  The  operation  was  performed  by  proportional  logs., 
as  in  the  following 

Example. 

Let  the  daily  retardation  of  the  moon's  transit  over  the  meridian  be  1  ?2?; 
required  the  proportional  part  corresponding  thereto,  and  9?  40?  of  time,  or 
145  degrees  of  longitude  ? 

As  24  hours  +  1*2?  (daily  retard.)=: 25^  2?  Ar.  comp.  pro.  log.  9. 1432 
Is  to  daily  retardation  of  transit  z:  .  1.  2  Propor.log.  •  •  0.4629 
So  is  given  interval  of  time  =:  •    •    •    9.40   Propor.log.  •    •     1.2700 


To  corresponding  proportional  part  =:        23T57  •  =  Pro.  log.  =:   0. 876 1; 
and  in  this  manner  were  all  the  mimbers  in  the  Table  obtained. 

The  corrections  or  proportional  parts  contained  in  this  Table  are  ex- 
pressed in  minuted  and  seconds,  and  are  extended  to  every  twentieth 
minute  of  time,  or  fifth  degree  of  longitude  :  these  are  to  be  taken  out  and 
applied  to  the  time  of  the  moon  s  transit,  as  given  in  the  Nautical  Almanac, 
in  the  following  manner  :— 

Pind,  in  page  VL  of  the  month  in  the  Nautical  Almanac,  the  difference 
between  the  moon's  transit  on  the  given  day  (reckoned  astronomically) 
and  that  on  the  day  followmg,  if  the  longitude  be  west ;  but  on  the  day 
preceding,  if  it  be  east.  With  this  difference  enter  the  Table  at  the  top, 
and  the  given  time  in  the  left-hand,  or  the  longitude  in  the  right-hand 
column ;  in  the  angle  of  meeting  will  be  found  a  correction,  which,  being 
applied  by  addUion  to  the  time  of  transit  on  the  given  day,  if  the  longitude 
be  west,  but  by  subtraciumy  i(  east,  the  sum,  or  difference,  will  be  the 
reduced  time  of  transit. 

Exodtvple  1. 

Required  the  apparent  time  of  the  moon's  passage  over  a  qimdian  80 
d^ees  west  of  Greenwich^  July  22^1824  ? 


Digitized  by 


Google 


102  '      PBSCfi.lH'ld^r  AND  U8B  OF  THE  TABLB8* 

Mn'0pafi.t>veriner«ofGreenw.ongtv.dayis21t7"      •    •    •    2H  7*  0! 
Ditto  on  the  dsy  following  =  22. 9 

Retardation  of  moon's  transit  =     .     •    .       l*2!';an8.towhich 

and80deg8.i8+I3. 13 


Apparent  time  of  the  moon's  transit  over  the  given  meridian  s:  21  t20T13! 

Example  2. 

Required  the  apparent  time  of  the  moon's  passage  over  a  meridian  120 
degrees  east  of  Greenwich,  August  SOth,  1824  ? 

Mn'8pa6.overmer^ofGreenw.ongiv.dayi820^54r    •    «    •    20^14?  Ot 
Dittd  on  the  day  precedit^zz  1 9. 54 


Retardation  of  the  moon's  transit  =     •    •    1  *  0?;  ans.  to  which 

andl20deg8,is-19.12 


Apparent  time  of  the  moon's  tratlsit  over  the  given  meridian  r:  SOt 34*^48! 


Table  XXXlX. 

Correcthn  to  be  appUed  eo  the  Time  of  the  Mom'e  I\xmit  mjm^g 
the  Time  of  High  Water. 

Since  the  moon  is  the  principal  agent  in  riusing  llie  tides,  it  might  be 
expected  that  the  time  of  high  water  would  take  place  at  the  moment  of 
her  passage  over  the  meridian;  but  observation  has  shown  that  this  is 
not  the  case,  and  that  the  tide  does  tlot  cease  flowing  for  some  tii^e  allir : 
for,  since  the  attractive  influence  of  the  moon  is  only  diminished,  imd  dot 
entirely  destroyed,  in  passing  the  meridian  of  any  place,  die  ascepding  im* 
pulse  previously  communicated  to  the  waters  at  that  place  tnust|  therefore^ 
Continue  to  act  for  some  time  after  the  moon's  meridional'  paBStige.  The 
ascending  impulse,  thus  imparted  to  the  waters,  ought  to  cause  the  time  of 
the  highest  tide  to  be  about  80  minutes  after  the  moon's  passage  over  the 
meridian ;  but  owing  to  the  disturbing  force  of  the  sun,  the  actual  time  ef 
high  water  difiers,  at  times,  very  considerably  from  that  period. 

The  effect  of  the  moon  in  raising  the  tides  exceeds  that  of  the  sun  in 
the  ratio  of  about  2^  to  1  ];  but  this  effect  is  far  from  being  uniform :  for, 
since  the  moon's  distance  from  the  earth  bears  a  very  sensible  proportion 
to  the  diameter  of  this  planet,  and  since  fthe  is  constantly  chatiging  Aat 
d}9timce,  (being  sometimes  nectreri  aad  At  otiw  tiom  mere  remete  m  everjf 


Digitized  by 


Google 


AS^CRIPTION  AND  USB  OP  THB  TABLBS.  103 

limation,)  it  fa  evidfiht  that  die  must  attract  the  waters  of  the  oceati  with 
^ry  imeqtial  forces:  but  the  sun's  distance  ft'om  the  earth  being  so  very 
immense^  that,  compared  with  It^  the  diameter  of  this  planet  is  rendered 
nearly  inseaeible^  his  attraction  is  consequently  more  uniform^  and  there- 
fore is  affeets  the  different  parts  of  the  ocean  with  nearly  an  equal  force. 

By  the  conibined  eff^t  of  these  two  forces,  the  tides  eome  on  sooner 
when  the  tnoon  is  in  her  Jirst  and  third  quarters,  and  later  when  in  the 
second  and  jbiHA  qualrtera,  than  they  would  do  if  raised  by  the  sole  lunar 
agieuey :  it  is,  therefore,  the  mean  quantity  of  this  acceleration  and  retard- 
adon  tiiat  is  contained  in  the  present  Table,  the  arguments  of  which  are, 
the  apparent  times  of  the  moon's  reduced  transit }  answering  to  which,  iil 
the  dijoitiing  colttiiin,  stands  a  correction^  which,  being  applied  to  the  ap- 
parent time  of  the  moon's  passage  over  the  meridian  of  any  given  place  by 
addition  or  sobtraetioti^  according  to  its  title,  the  sum,  or  dilierence,  will  be 
the  corrected  time  of  transiti  Now,  to  the  corrected  time  of  transit,  thus 
found,  let  the  time  of  high  water  on  full  and  change  days,  at  any  given 
place  in  Table  LVL,  be  applied  by  addition,  and  the  sum  will  be  the  time 
of  high  water  at  that  place,  reckoning  from  the  nooti  of  the  given  day: 
should  the  mm  exceed  12^24?,  or  24?48?,  subtract  one  of  those  quantities 
from  it,  and  the  remainder  will  be  the  time  of  high  water  very  near  the 
truth* 

Example  h 

Required  the  time  of  high  water  at  Cape  Florida,  America,  March  7th, 
1824 ;  the  longitude  being^  80?5 '  west,  and  the  time  of  high  water  on  full 
and  change  days  7t30?? 

Moon's  transit  over  the  meridian  of  Greenwich,  per  Nautical 
Almanac,  March  7th,  1824,  is 5*  2*  Of 

Correction  from  Table  XXXVIII,,  answering  to  retardation  of 
tranttt58r,andlongitude80?5^  west  =     .....     -f     12.23 

Moon's  transit  reduced  to  the  meridian  of  Cape  Florida    •    *    5  ^  14?23 ! 
Cotrectaoii  aoswerii^  to  reduced  transit  (5*  14T23')  in  Table 
XXXDC^fa     ,...'•.•*•..•.•-  1.  5.   0 

Corrected  time  of  transit 4^9^23! 

Time  of  high  water  at  Ci^  Florida  on  full  and  change  days    ?•  30.   0 

I^&eofhigfa  water  at  Cq>«  Florida  on  the  given  days    «    •  U?39r23! 

Example  2. 

Reqnhred  the  time  of  high  water  in  Queen  Charlotte's  Sound,  Nev^ 
Zeakmd,  April  IStfa,  1824;  tiie  longitude  being  174?56:  easti  and  t)l« 
time  of  high  wi^t^r  on  Mmi  change  da^  9^0?  ? 

Digitized  by  VjOOQ IC 


104  DBSCKIFTION   AND  USB  OF  THE  TABtBS. 

Moon's  transit  over  the  meridian  of  Greenwich^  per  Nautical 

Almanac,  April  13th,  1824,  is      ........    .10*27^0* 

Correction  from  Table  XXXVIIL,  answering  to  retardation  of 

transit  50T,  and  longitude  174?56'  west  =     ....     ^     23.29 

Moon's  transit  reduced  to  the  meridian  of  Queen  Charlotte's 

Sound 10!  3T31I 

Correction  answering  to  reduced  time  of  transit  (lOtSTSl!) 
in  Table  XXXIX.,  is +     23.  0 

Corrected  time  of  transit 10i26?31! 

Time  of  high  water  at  given  place  on  full  and  change  days    •    9.  0.   0 

Trnie  of  high  water  at  Queen  Charlotte's  Sound,  past  noon  of 

the  given  day    .    •    .    . 19?26T8l! 

Subtract  12.24.  0 


Hme  of  high  water  at  given  place,  as  required 7  •  2T31 ! 


Table  XL. 

Beduction  qf  tlie  Moon's  Horizontal  Parallax  on  account  of  the 
Spheroidal  Figure  qfthe  Earth. 

Since  the  moon's  equatorial  horizontal  parallax,  given  in  the  Naiftical 
Almanac,  is  determined  on  spherical  principles,  a  correction  becomes 
necessary  to  be  applied  thereto,  in  places  distant  from  the  equator,  in 
order  to  reduce  it  to  the  spheroidal  principles,  on  the  assumption  that  the 
polar  axis  of  the  earth  is  to  its  equatorial  in  the  ratio  of  299  to  300 ;  and, 
when  very  great  accuracy  is  required,  this  correction  ought  to  be  attended 
to,  since  it  may  produce  an  error  of  seven  or  eight  seconds  in  the  computed 
lunar  distance.  The  correction,  thus  depending  on  the  spheroidal  figure 
of  the  earth,  is  contained  in  this  Table}  the  arguments  of  which  are,  the 
moon's  horizontal  parallax  at  the  top,  and  the  latitude  in  the  left-band 
column  }  in  the  angle  of  meeting  will  be  found  a  correction,  expressed  in 
seconds,  which  being  subtracted  from  the  horizontal  parallax  given  in  th^ 
Nautical  Almanac,  will  leave  the  horizontal  parallax  agreeably  to  the  sphe- 
roidal hypothesis. 

Thus,  if  the  moon's  horizontal  parallax,  in  the  Nautical  Almanac,  be 
57-58^,  and  the  latitude  51?48^ ;  the  corresponding  correction  will  be  7 
seconds  subtractive.  Hence  the  moon's  horizontal  parallax  ou  the  sphe*- 
roidal  hypothesis,  in  the  given  latitude,  is  57  *5K, 


Digitized  by 


Google 


DSSCUPTtON  AMD  VSB  OF  THE  TABLBS«  105 

JtewuurJc^^The  correctioiii  contained  in  this  Table  may  be  computed  by 
the  following 

Rule. 

To  the  logarithm  of  the  moon's  equatorial  horizontal  parallax,  reduced  to 
seconds,  add  twice  the  log.  sine  of  the  latitude,  and  the  constant  log* 
7. 522879  ;*  the  sum,  rejecting  the  tens  from  the  index,  will  be  the  loga*. 
rithm  of  the  corresponding  reduction  of  parallax. 

Example. 

Let  the  moon's  horizontal  parallax  be  57 '58?,  and  the  latitude  51?48^ ; 
required  the  reduction  of  parallax  agreeably  to  the  spheroidal  hypothesis  ? 

=  3478r  .  Log.  =  3.541330 
Twice  the  log.  sine  =19. 790688 
Constant  log.     .    .    7.522879 


Moon's  equatorial  horiz.  par.  57^58r  =  3478?      .  'Log.  =     3.541330 
Latitude; 51?48'     Twice  the  log.  sine  =19. 790688 


Reduction  of  horizontal  parallax  =      .    .    7^  159        Log.=0. 854897 

Table  XLL 
Jteduetion  of  LaiUude  on  account  of  the  Spheroidal  Figure  of  the  Earth. 

Since  the  figure  of  the  earth  is  that  of  an  oblate  spheroid,  the  latitude  of 
a  place,  as  deduced  directly  from  celestial  observation,  agreeably  to  the 
q>herical  hypothesis,  must  be  greater  than  the  true  latitude  expressed  by 
the  angle,  at  the  earth's  centre,  contained  between  the  equatorial  radius 
and  a  line  joining  the  centre  of  the  earth  and  the  place  of  observation. 
This  excess,  which  is  extended  to  every  second  degree  of  latitude  from  the 
equator  to  the  poles,  is  contained  in  the  present  Table ;  and  which,  being 
iubtracted  from  the  latitude  of  any  given  place,  will  reduce  that  latitude 
to  what  it  would  be  on  the  spheroidal  hypothesis  :  thus,  if  the  latitude  be 
50  degrees,  the  corrjespoiiding  reduction  will  be  1 1 M2?,  subtractive ;  which, 
therefore,  gives  49?48^  18?  for  the  reduced  or  spheroidal  latitude. 

Remark. — ^The  corrections  contained  in  this  Table  may  be  computed  by 
the  following  rule ;  viz.. 
To  the  constant  log. .  003003,t  add  the  log.  co-'tangent  of  the  latitude. 


*  The  arithmetical  complemeDt  of  the  log.  of  the  earth'i  ellipticity  assumed  at  ^^ 
t  The  excess  of  the  spherical  above  the  elliptic  arch  in  the  paraUel  of  45  dei^rees  from  the 
equator,  is  1K887»  or  1^53^'  (Robertson's  Navi^tion,  Book  VIII.,  Article  134)  i  hence 
45«  -  ir  53^  a  44*  48^  7",  tiis  lo(.  co-t«iigeat  of  which,  lejeclins  the  index,  U .  003003. 


Digitized  by 


Google 


106  DBtCRimOK  AND  VtB.  OF  THB  TABUIti 

and  tte  turn  will  be  the  log.  e o-tangent  of  ui  arch ;  the  difference  between 
which  and  the  given  latitude  will  be  the  required  reduction. 

Example. 

Let  it  be  required  to  reduce  the  spherical  latitude  50?48f  to  what  it 
would  be  if  determined  on  the  spheroidal  principles ;  and^  hence^  to  find 
the  reduction  of  that  latitude. 

Latitude 50?48'  Or  Log.  co-tang.=:9.911467 

Constant  log.=  .003003 

Reduced  or  spheroidal  latitude  =     50^36^21?  Log.  co-tang.f:9. 914470 

Reduction  of  latitude^  as  required     0?llC39r 


Tablb  XLIL 

A  General  Traverse  Tabk  ;  or  Difference  of  Latitude  and  Depaartwre. 

This  Table,  so  exceedingly  useful  in  the  art  of  navigation,  is  drawn  up  in 
a  manner  quite  different  from  those  that  are  given,  under  the  same  deno- 
mination, in  the  generality  of  nautical  books  i  and,  although  it  occupies 
but  88  pages,  yet  it  is  more  extensive  dian  the  two  combined  TaUea  tff 
61  pages,  which  are  contained  in  those  books.  In  this  Traverse  Table, 
every  page  exhibits  all  the  angles  that  a  ship^s  course  can  make  with  the 
meridian,  expressed  both  in  points  and  degrees;  which  does  away  with  tfatf 
necessity  of  consulting  two  Tables  in  finding  the  diflerence  of  latitude  and 
the  departure  corresponding  to  any  given  course  and  distance.  If  the 
course  be  under  4  points,  or  45  degrees,  it  will  be  found  In  the  left-hand 
compartment  of  each  page ;  but  that  a&ote  4  points,  or  45  degrees,  in  the 
right-hand  compartment  of  the  page.  The  distance  is  given,  in  numerical 
order,  at  the  top  and  bottom  of  the  page,  from  unity,  or  1,  to  304  miles  $ 
which  Qlomprehends  all  the  probable  limits  of  a  ship's  run  in  24  hours  j  and, 
by  this  arrangement,  the  mariner  is  spared  the  trouble  of  turning  over 
and  consulting  twenty-three  additional  pages.  Although  the  manner  of 
using  this  Table  m\ut  appear  obvious  at  first  sight,  yet  since  its  mode  of 
arrangement  differs  so  very  considerably  from  the  Tables  with  which  the 
reader  may  have  been  hitherto  acquainted^  the  following  Problems  are 
given  for  its  illustration. 


Digitized  by 


Google 


MioRimoif  AND  vnm  op  thi  tabus.  107 

Problbm  L 

Ghen  fftv  C(mr$e  and  Distance  fotkil,  or  betwem  two  Places,  to  find  the 
Difference  qf  Latitude  and  the  Depdarturei 

RULB. 

Enter  the  Table  with  the  course  in  the  left  or  right-hand  column^  and 
the  distance  at  the  top  or  bottom  |  opposi^  to  the  former^  and  under  or 
over  the  latter,  will  be  found  the  corresponding  difference  of  latitude  and 
departure  i  these  are  to  be  taken  out  as  marked  at  the  top  of  the  respective 
columns  if  the  course  be  tmcfer  4  points  or  45  degrees,  but  as  marked  at 
the  bottom  if  the  course  be  more  than  either  of  those  quantities. 

Note* — If  the  distance  exceed  the  limits  of  the  Table,  an  aliquot  part 
thereof  may  be  taken,  as  a  half,  third,  fourth,  &c. ;  then  the  difference  of 
latitude  and  departure  corresponding  to  this  and  the  given  course,  being 
multiplied  by  2,  3,  4,  &c.,  (that  is,  the  figure  by  which  such  aliquot  part 
was  found,)  the  product  will  be  the  difference  of  latitude  and  departure 
answering  tb  the  given  course  and  distance. 

Example  1. 

A  ship  sails  S.S.W.  |  W.  176  miles;  required  the  difference  of  latitude 
and  the  departure  i 

Opposite  ^  points  and  under  176  milesi  stand  155. 2  and  88. 0 1  heiie« 
the  dlffefenee  of  latitude  is  155^  2^  and  the  departure  SS.  0  miles. 

Example  2. 

A  ship  sails  N«  57?  B.  236  miles ;  required  the  difference  of  hititude  and 
the  departure  ? 

OffOMe  to  57?5  and  under  236  miles^  stand  128. 5  and  197^  9 :  hence 
Use  diilbi«nce  of  hititude  is  126. 5^  and  the  departure  197*  9  miles. 

Example  3. 

'the  course  between  two  places  is  E.  b.  S.  ^  S^  and  the  distance  540 
miles ;  required  the  difference  of  latitude  and  the  departure  ? 

Distance  divided  by  2,  gites  270  miles }  under  or  over  which,  and  oppo« 
Hie  to  8i  points,  stand    .    .    .    91.0  and  254.2 
Multiply  by  2  2 

IVoducU  =:    182. 0  and  508. 4 1  hence  (he  differeuce 
pf  Mtode  is  182. 0|  and  the  departure  508*  4  milest 


Digitized  by 


Google 


lOS  DBSCRIPtlON  AND   USB  OP  THE  TABLES. 

Example  4. 

TTie  course  between  two  places  is  N.  61  W.  anil  the  distancel  176  miles; 
required  the  difference  of  latitude  and  the  departure  ? 

Distance  1 176  divided  by  4,  gives  294  miles ;  under  or  over  wluch^  and 
opposite  to  61?^  stand    •    •    •     142.5  and    257*1 
Multiply  by  4  4 


Product  =      570. 0  and  1028. 4  :  hence  the   <tifier- 
ence  of  latitude  is  570. 0^  and  the  departure  1028. 4  miles. 


Problem  IL 

Given  the  Difference  ofLat^ude  and  the  Departure,  to  find  the  Omree 

and  Distance. 

Rule. 

With  the  given  difference  of  latitude  and  departure^  enter  the  Table  and 
find,  in  the  proper  columns  abreast  of  each  other,  the  tabular  difference  of 
latitude  and  departure  either  corresponding  or  nearest  to  those  g^ven }  then 
the  course  will  be  found  on  the  same  horizontal  line  therewith  in  the  left  or 
right-hand  column,  and  the  distance  at  the  top  or  bottom  of  the  compart- 
ment where  the  tabular  numbers  were  so  found. 

Note.-^If  the  difference  of  latitude  be  greater  than  the  departure,  the 
course  will  be  less  than  4  points^  or  45  degrees ;  and,  therefore,  it  b  to  be 
taken  from  the  left-hand  column :  but  when  the  difference  of  latitude  is 
less  than  the  departure,  the  course  will  be  more  than  4  points  or  45  degrees^ 
and,  consequently,  it  must  be  'taken  from  the  right-hand  column. 

Note,  also,  that  when  the  difference  of  latitude  and  the  departure,  or 
either  of  them,  exceed  the  limits  of  the  Table,  aliquot  parts  are  to  betaken, 
as  a  half,  third,  fourth,  &c.,  with  which  find  the  course  and  distance  as 
before  $  then  the^distance,  thus  found,  being  multiplied  by  2,  3,  4,  &c.,  the 
product  will  be  the  tu/ioie  distance  corresponding  to  the  given  difierence 
of  latitude  and  departure.  The  course  is  never  to  be  multiplied,  because 
the  angle  will  be  the  same  whether  determined  agreeably  to  the  whole  dif- 
ference of  latitude  and  the  departure,  or  according  to  their  corresponding 
aliquot  parts* 


Digitized  by 


Google 


BBSCRIFTION  AND   USB  OP  THB  TABLES*  109 

Exatnple  I. 

If  the  difference  of  latitude  made  by  a  ship  in  24  hours  be  177*  4  miles 
north,  and  the  departure  102. 6  miles  east,  required  tlie  course  and  distance 
made  good? 

SohUim. — The  tabular  difference  of  latitude  and  departure,  nearest 
corresponding  to  those  g^Yen,  are  177. 5  and  102. 5  respectiYcly :  these  are 
found  in  the  compartment  under  or  over  205,  and  opposite  to  30  degrees ; 
hence  the  course  made  good  is  N.  30  E.,  and  the  distance  205  miles. 

trample  2. 

The  difference  of  latitude  made  by  a  ship  in  24  hours,  is  98.5  miles 
south,  and  the  departure  140.  iS  miles  west;  requfared  the  course  and  dis- 
tance made  good  ? 

SohUmi. — The  tabular  difference  of  latitude  and  departure,  nearest  to 
those  giTcn,  are  98. 7  and  140. 9  respectively :  these  are  found  in  the  com- 
partment under  or  over  1 72,  and  opposite  to  55  degrees ;  hence  the  course 
made  good  is  S.  55?  W.,  and  the  distance  172  miles. 

Example  3. 

The  difference  of  latitude  is  700  miles  south,  and  the  departure  928  miles 
west;  required  the  course  and  distance  ? 

Solution, — Since  the  difference  of  latitude  and  the  departure  exceed  the 
limits  of  thcTable,  take  therefore  aify  aliquot  part  of  them,  as  one  fourth,  and 
they  will  be  175  and  232  respectively :  now,  the  tabular  numbers,  answering 
nearest  to  those,  are  175. 1  and  232. 4 ;  these  are  found  in  the  compartment 
under  or  over  29.1,  and  opposite  to  53  degrees :  hence  the  course  is  S.  S3? 
W«,  and  tiie  distance^91  X  4  =  1  lfi4  miles,  as  required. 

Remarks — ^Whenever  it  becomes  necessary  to  take  aliquot  parts  of  the 
difference  of  latitude,  the  same  must  be  taken  of  th^  departure,  whether  it 
falls  without  the. limits  of  the  Table  or  not;  and,  vice  versGy  whenever  it 
becomes  necessary  to  take  aliquot  parts  of  the  departure,  the  same  must 
be  taken  of  the  difference  of  latitude. 

And,  in  all  cases  where  the  tabular  numbers  differ  considerably  from 
those  given,  proportion  must  be  made  for  that  differeuce. 


Digitized  by 


Google 


IIQ  DiacRifnov  Aim  va  ot  tbb  vabui. 


Problem  .III. 

Given  the  proper  Difference  of  Latitude  between  two  Places,  the  Meridimal 
Difference  of  Latitude,  and  the  Departure,  to  find  the  Course,  Distance, 
and  Difference  of  Longitude. 

With  tht  proper  diftrenci  pf  latitude  and  the  cbparture^  find  t]i«  couna 
and  di^tanoct  by  Problem  IL;  tben^  with  the  course  thus  found  and  the 
meridional  difference  of  latitude,  (in  a  latitude  column,)  take  out  the  cor- 
responding departure,  and  it  will  be  the  differenpe  of  longitude  required ; 
as  thus :  run  the  eye  along  the  horizontal  line  answering  to  the  course, 
from  where  the  ptopex  difference  of  latitude  was  found,  {ahjoays  to  the 
right  hand,)  and  find,  in*  a  latitude  column,  the  tabular  difference  of  lati- 
tude answering  nearest  to  the  given  meridionid  difference  of  latitude  | 
abreast  of  which,  in  the  departure  column,  will  be  found  tl^fs  difference  of 
longitude. 

The  proper  difference  of  latitude  between  two  places,  is  142  miles 
north,  the  departure  107  miles  west,  and  the  meridional  difference  of 
latitude  169  miles ;  required  the  course,  distance,  and  difference  of  Ion- 
^tudei 

S6Itt(ia»K— The  tabular  difference  of  latitude  and  departure  answering 
xu^est  to  those  giyei),  are  142. 2  and  107. 3  respectively  i  these  are  (bond 
in  the  compartment  under  or  over  17S,  and  opposite  to  37  degr^ies:  heppfi 
the  ppur^e  is  N.  37?  W.,  a^d  tb^  distance  178  miles.  Now,  with  the 
cpurse  37  degreos,  and  the  meridional  difference  of  latitude  169  mleu, 
the  difference  of  longitude  is  foundt  as  thus :  from  where  the  proper  dif- 
ference of  latitude  was  fpundj  run  the  eye  along  the  horizpntat  liu^  answer- 
ing to  37  degrees,  {ahoays  towards  the  right  hand,)  and  the  tabular  differ- 
ence of  latit|ide  answering  nearest  to  the  given  meridional  diffisrence  of 
latitude  will  be  found  in  the  compartment  under  or  over  212,  viz.  169.  S;  ^ 
correspanding  to  which,  in  the  departure  column,  is  127«63  and  which^ 
therefore,  is  the  difference  of  bnptude,  as  required. 


Digitized  by 


Google 


uacftimoii  ▲>»  088  w  thb  yablsi.  Ill 

Paqbdsm  IV, 

Qioen  the  proper  Differefice  qf  LalUwUy  the  Meridkmal  Diffisrence  qf 
Latitude,  and  the  Difference  of  Longitude,  to  find  the  Course  and 
Distance. 

Rule. 

^th  the  meridional  difference  of  Ulitttde  and  the  difference  of  longi- 
tude, esteenied  as  difference  of  latitude  and  departure,  find  the  course  by 
Problem  II. }  then  with  the  course^  thus  found,  and  the  proper  difference 
of  latitude^  the  dbtance  is  to  be  obtained,  as  thus  :  run  the  eye  {akoays  to 
the  ^fi  hand)  f^png  the  horizontal  line  answering  to  the  comae^  from  wl^re 
the  meridional  differei|ce  of  latitude  was  found|  mi  ieekj  in  th^  proper 
cohimn,  the  differeiic^  of  latitude  answering  nearest  to  that  given ;  over  or 
under  which,  at  the  top  or  bottom  of  the  column,  will  be  foup4  (he 
required  distance. 

Note. — ^When  the  meridional  difference  of  latitude  exceeds  the  differ- 
ence of  longitude,  the  course  is  ^  be  taken  from  the  lefk-hand  column  i 
but  otherwise  from  the  right. 

ExampJe. 

Hie  proper  differoice  of  latitude  between  two  places  is  78  miles  south, 
the  meridional  difference  of  latitude  107  miles  south,  and  the  difference  of 
longitude  119  miles  east;  required  the  course  and  distance  ? 

Mmtioeu-^Th^  tirinilar  diflbrence  of  latitude  md  departmie^  answering 
nearest  (o  th^  meridional  difference  of  latitiidn  md  the  dtiEerenee  of  longi- 
tudiK,  if«  107. 1  and  U8»0  respeittmiy  i  these  are  foi|nd  in  the  comparts 
ment  under  or  pver  )60,  and  opposite  to  48  degrees  s  heiice  the  eoune  i« 
S.  48?  EL  Now,  the  eye  being  run  along  tlie  horizontal  line  fmsweriog  to 
48,  [towards  the  ^ft  hand,)  the  nearest  tabular  difference  of  latitude, 
answering  to  the  proper  difference  of  latitude,  will  be  found  in  the  com- 
partmant  imder  or  over  U7  s  hence  the  distance  is  117  miles. 


Probum  V. 

Gioen  the  middle  Latitude^  and  the  Meridian  DietwmcfDepoflme^ 
to  find  the  Difference  €f  J^n^ptude^ 

Rule. 

Bntat  tha  Tablt  with  the  middle  latitude,  taken  as  «  eeimi^  and  the 
departure  in  a  latitude  eolumnj  run  the  eye  along  the  horizontal  Una 


Digitized  by 


Google 


112  DSSCRIPTION  AND  USB  OF  THB  TABLES. 

answering  to  that  course  (towards  the  right  hand  or  the  left,  acoording  as 
the  first  tabular  difference  of  ItOitude  which  meets  the  eye  therein  is  greater 
or  less  than  the  given  departure),  and  find  a  difference  of  latitude  that 
either  agrees  with,  or  comes  nearest  to,  the  given  departure ;  then  .the 
distance  Corresponding  to  this,  at  the  top  or  bottom  of  Uie  column,  will  be 
the  difference  of  longitude. 

Example. 

The  middle  latitude  between  two  places  is  20?  north,  and  the  meridian 
distance  or  departure  140  miles;  required  the  difference  of  longitude? 

Solution. — The  middle  latitude,  20  degrees,  taken  as  a  course,  and  the 
departure  140,  as  difference  of  latitude,  will  be  found  to  correspond  in  the 
compartment  under  or  over  149 :  hence  the  difference  of  longitude  is  149 
miles,  as  required. 


Problbm  VI, 

Given  the  middle  Latitude,  the  Difference  of  Latitude,  and  the  Difference 
of  Longitude  between  two  Places,  to  find  the  Coureeand  Distance, 

RULB. 

Enter  the  Table  with  the  difference  of  longitude,  esteemed  as  distance, 
at  the  top  or  bottom  of  the  page,  and  the  middle  latitude,  taken  as  a  course, 
in  the  left  or  right-hand  column ;  answering  to  which,  in  the  difference  of 
latitude  column,  will  be  found  the  departure.  Now,  with  this  departure 
and  the  given  difference  of  latitude,  the  course  and  distance  are  to  be  found 
by  Problem  II. 

Example. 

The  middle  latitu4.e  is  26  degrees  north,  the  difference  of  latitude  200 
miles  north,  and  the  difference  of  longitude  208  miles  east;  required  the 
course  and  distance  ? 

Solution. — In  the  compartment  under  or  over  208  miles  (the  given  longi* 
tude),  and  opposite  to  26  degrees  (the  middle  latitude  taken  as  a  course), 
stands  186.9  in  the  difference  of  latitude  column,  which,  therefore,  is  the 
departure.  Now,  the  tabular  numbers  answering  nearest  to  the  given 
difference  of  latitude  and  the  departure,  thus  found,  are  200. 4  and  186. 9 
respectively ;  these  are  found  in  the  compartment  under  or  over  274,  and 
opposite  to  43  degrees:  hence  the  course  is  N.  43?  E.,  and  the  distance  274 
miles. 


Digitized  by 


Google 


DfiSCRIFTION  AND  USB  OF  THfi  TABLES. 


113 


-Bemorfc.— The  numbers  in  the  general  Traverse  Table  were  computed 
agreeably  to  the  following  rule;  via.. 

As  radius  is  to  the  distance^  so  is  the  co-sine  of  the  coufse  to  the  differ- 
ence of  latitude ;  and  so  is  the  sine  of  the  course  to  the  departure. 

Bsample. 

Given  the  course  35  degrees,  and  the  distance  147  miles  j  to  compute 
the  difference  of  latitude  and  the  departure. 


To  find  the  Difference  of  latitude. 


As  radius  •  .  • 
Is  to  distance  .  . 
So  is  the  course   • 


s     90?  log.  sine      .    .     s  10.000000 

.       147  miles      .     .    log.  =    2.167317 

=     35?  log.  co-sine     .     =    9.913365 


To  difference  of  lat.       =     120.4,  miles     .    •    log.  =    2.080682 


As  radius  .  . 
Is  to  distance  . 
So  is  the  course 

To  departure 


To  find  the  Departure, 

.     .  s  90?  log.  sine  .    .     s  10.000000 

.     .       147  miles    .  .    log.  =:  2. 167317 

,     .  s=  35?,  log.  sine  .    .     =  9-758591 

=         84. 3  miles      .  •    log.  =  1.925908 


Table  XLIII. 

Meridimal  Parts* 

This  Table  contains  the  meridional  parts  answering  to  each  degree  and 
minute  of  latitude  from  the  equator  to  the  poles ;  the  arguments  of  which 
are,  the  degrees  at  the  top,  and  the  minutes  in  the  left  or  right  hand  mar- 
ginal columns ;  under  the  former,  and  opposite  to  the  latter,  in  any  given 
latitode,  will  be  found  the  meridional  parts  corresponding  the^to,  and 
conversely.  Thus,  if  the  latitude  be  50?48C ,  the  corresponding  meridional 
parts  will  be  3549. 8  miles. 

Remark. — The  Table  of  meridional  parts  may  be  computed  by  the  fol- 
lowing rule ;  viz.. 

Find  the  logarithmic  co-tang^t  less  radius  of  half  the  complement  of 
any  latitude,  and  let  it  be  esteemed  as  an  integral  number  ;  now,  from  the 


Digitized  by 


Google 


114  DB8CRIPTION  AND  USB  OF  THB  TABUW. 

common  logarithm  of  this,  subtract  the  constant  log.  2.101510*,  and 
the  remainder  will  be  the  log.  of  the  meridional  Iparts  answering  to  that 

latitude.  .„         ,    , 

Example  i. 

Required  the  meridional  parts  corresponding  to  latitude  50?48^  ? 
Given  lat.  =  50M8-  complement  =  39?  12^^.2  =      19936.,   the  half 

complement;  hence,  aaojao   ^\.    %        f 

Halfcomp.«19^36:  log.  co.tangent7««radit«  =«  ^ *^^**\^t,2f''' 

whichis «-65  71^^ 

Constant  log ^'^Q^^^" 

Meridional  parts  correspoudinj?  to  given  lat,  3549. 78«log.=:3. 550202 

Example  2. 

Iteqtured  the  mcridiotial  parts  corresponding  to  latitude  89^30^  ? 
Given  lat.     =  89?80^ }  comp.  =  0?30^  -^2  ^  0?15^  the  half  comple- 

meut;  hence,  k^aida     \.   t 

Half  comp.  =    0*ll5^  log.  co-tangent  less  radius  =  2.360180,  the  log. 

ofv^hichis       . •    6.372945 

Constantlog 2.101510 

Meridional  parts  corresponding  to  given  lat  18682. 49=log.=:4. 271435 


Table  XLIV. 

T%e  Mean  Right  Jscensions  and  Declinations  of  the  principal  ficed 

Stars. 

This  Table  contains  the  viean  right  ascensions  and  declinations  of  the 
principalfixed  stars  adapted  to  the  beginning  of  the  year  1824.— The  stars 
are  arranged  in  the  Table  according  to  the  order  of  right  ascension  in 
ivhich  they  respectively  come  to  the  meridian;  the  annual  variation,  in 
right  ascension  and  declination,  is  given  in  seconds  and  dedmal  parts  of  a 
second}  that  of  the  former  being  expressed  in  time,  and  that  of  the  latter 
motion. 

The  stars  marked  ft  h«ve  been  taken  horn  the  Nautical  Almaiiae  for 
the  year  1824.— The  stars  that  have  asterisks  prefixed  to  tiiem  are  tfaoee 
from  which  the  moon's  distance  is  computed  in  the  Nautical  Almanac  for 
the  purpose  of  finding  the  longitude  at  sea. 

*  TliemttSQTe  affile  arc  of  1  minute  (pa^  54,)  is  .00029088821;  which  beinf  multiplied 
by  10000000000,  (the  radius  of  the  Tables)  produces  290.8882000000 ;  and,  this  being: 
multiplied  by  the  modulus  of  the  common  ^iguflthms,  tia.,  Adl29448190y  ghres 
126.331140109823580 1«  the  coanitt  kg.  of  which  is  2.l01dl0»«s  abov«. 


Digitized  by 


Google 


ABSCRIPTIOK  Airs  nSS  OF  THB  TABLB9.  115 

The  places,  of  the  etars,  as  given  in  this  Table,  may  be  reduced  to  any 
future  period  by  multiplying  (he  annual  variation  by  the  number  of  yean 
and  parts  of  a  year  elapsed  between  the  beginning  of  1824,  and  such  fiiture 
period  :  the  product  of  right  ascension  is  to  be  added  to  the  right  ascen* 
sions  of  alt  the  stars,  except  fi  and  Z,  in  Ursa  Minor,  from  whose  right  as- 
censions it  is  to  be  subtracted  :.  but  the  product  of  declination  is  to  be  ap- 
plied, according  to  the  sign  prefixed  to  the  annual  variation  in  the  Table, 
to  the  declinations  of  all  the  stars  without  any  exception ;— thus, 
To  find  the  right  ascension  and  the  declination  of  a  Arietis^  Jan.  1st,  1884. 

R.  A.  of «  Arietis,  perTab.  1*57^16!,  and  its  dec.    .    .      22?37'83r  N. 
Annual  var.      .     +3*'.  35  Ann.  var.+  17'«40. 

Number  of  years  Num.  of  yrs. 

after  1824=       10  after  1824  =     10 


Product.     +33-.5      +0^38^  5      Prod.+  174''.Os=  +  8(54f 


Rt.  asc.  of  a  Arietis,  as  req.  1^57*49'.  5, and  its  declination  22?40^27^  N. 

Should  the  places  of  the  stars  be  required  for  any  period  antecedent  to 
1824,  it  is  evident  that  the  product^  of  right  ascension  ^nd  deplination  iwt^ 
be  applied  in  a  contrary  manner. 

The  eighth  column  of  this  Table  contains  the  true  spherical  distance  and 
the  approximate  bearing  between  the  stars  therein  contained  and  those  prer 
ceding,  or  abreast  of  them  op  the  same  horizontal  line ;  and  the  pinth^ 
or  last  column  of  the  page,  the  annual  variatiop  of  that  distance  expressed 
in  seconds  and  decimal  parts  of  a  second.— By  means  of  the  last  column, 
the  tabular  distance  may  be  reduced  very  readily  to  any  future  period,  by 
multiplying  the  years  and  parts  of  a  year  between  any  such  period  and  the 
epoch  of  the  Table,  by  the  annual  variation  of  distance ;  the  product  being 
applied  by  addition  or  subtraction  to  the  tabular  distance,  according  as  the 
sign  may  be  affirmative  or  negative,  the  sum  or  difference  will  be  the  dis- 
tance reduced  to  that  period. 

flsample. 

Required  the  distance  between  a  Arietis  and  Aldebaraii,  Jan.  1st,  1844  ? 
TdHilar  dist.  between  the  two  given  stars  ==  .     .     .    35?32'7^ 
Annual  var.  of  distance  .     —  0-.  02 
Number  of  years  after  1824  «       20 


Product.    .    f-.0<',40= -0^.40 


True  spherical  distance  between  the  two  given  stars,  as 

required • 85?32;6^60. 

x2 


Digitized  by 


Google 


116  DESCRIPTION  AND  USB  OF  THB  TABtSS. 

Ifemari,— The  trae  spherical  distance  between  any  two  stars,  whose 
right  ascensions  and  declinations  are  known,  may  be  computed  by  the  fol- 
lowing rule }  viz.. 

To  twice  the  log.  sine  of  half  the  difference  of  right  ascension,  in  jdegr  ees 
add  the  log.  sines  of  the  polar  distances  of  the  objects ;  from  half  the  sum 
of  these  three  logs,  subtract  the  log.  sine  of  half  the  difference  of  the  polar 
distances,  and  the  remainder  will  be  the  log.  tangent  of  an  arch ;  the  log. 
sine  of  which  being  subtracted  from  the  half  sum  of  the  three  logs.,  will 
leave  the  log.  sine  of  half  the  true  distance  between  the  two  given  stars. 


Example. 

Let  it  be  required  to  compute  the  true  spherical  distance  between  a 
Arietis  and  Aldebaran,  January  1,  1844. 

R.  A.  of  a  Arietis  red.  to  1844  =:  U5Sr23!,  and  its  dec.  =22?43^2ir  N. 
R.  A.  of  Aldebaran  red.  to  1844  =  4. 26. 58. 6,  and  its  dec.  =  16. 1 1. 28    N. 


Difference  of  light  ascension    =  2 !  28'?35 ' .  6 = 

37?8^54r^2=18?34:27.r 


":Sil!^r!'"^*-'r'*:}i8?34c27ri;«.n?i:}i9. 0063060 

«.poi«ai...»f.Arua.=:  {67.16.39    {^ }      9.9649129 
N.p«i«d»it.«fAia,ur«=  {73. 48. 32     {|ff}      9. 9824236 


Sum  .  .  38.9536425 


Diff.  of  Polar  dists.6?3i:53r    Half=19.4768212|  .  .  .  19.4768212.5 


Half  diff.  of  ditto    3?15r56jr  Log  S.  8.7556177i 


Arch  79?14:27^  5826  log.  tang.     .  10. 7212035  Log.  S.  9. 9922976. 3 
Half  the  req.  dist.   ....     17?46'.3'^.  4424  .    Log.  S.  9. 4845236. 2 

True  spher.  dist.  between 
the  two  given  stars      ,    .    35?32^6''.  8848  on  Jan.  1,  1844. 


Digitized  by 


Google 


BESCRIPTION  AND  USB  OF  THE  TABLB8.  117 

Now,  by  comparing  this  computed  distimce  with*that  directly  deduced 
from  the  Table,  as  in  the  preceding  example,  it  will  be  seen  that  the  differ- 
ence amounts  to  very  little  more  than  the  fifth  part  of  a  second  in  twenty, 
years ;  which  evidently  demonstrates  that  the  tabular  distances  may  be  re- 
duced to  any  subsequent  period,,  for  a  considerable  series  of  years,  with  all 
the  accuracy  that  may  be  necessary  for  the  common  purpose9  of  navi- 
gation. 

Note* — ^The  tabular  distances  will  be  found  particularly  useful  in  deter- 
mining the  latitude,  at  sea,  by  the  altitudes  of  two  stars,  as  will  be  shown 
hereafter. 


Tablb  XLV, 

Acceleration  of  the  Fixed  Stare ;  or  to  reduce  Sidereal  to  Mean  Solar 

Time. 

Observation  has  shown  that  the  interval  between  any  two  consecutive 
transits  of  a  fixed  star  over  the  same  meridian  is  only  23t56T4' .  09,  whilst 
that  of  the  sun  is  24  hours : — the  former  is  called  a  sidereal  day,  and  the 
latter  a  solar  day ;  the  difference  between  those  intervals  is  3*55 '.  91,  and 
which  difference  is  called  the  acceleration  of  the  fixed  stars. 

This  acceleration  is  occasioned  by  the  earth's  annual  motion  round  its 
orbit :  and  since  that  motion  is  from  west  to  east  at  the  mean  rate  of  5  9  ^  8  '^ .  3 
of  a  degree  each  day ;  if,  tlierefore,  the  sun  and  a  fixed  star  be  observed  on 
any  day  to  pass  the  meridian  of  a  given  place  at  the  same  instant,  it  will  be 
found  the  next  day  when  the  star  returns  to  the  same  meridian,  that  the 
sun  will  be  nearly  a  degree  short  of  it ;  that  is,  the  star  will  have  gained 
3"  56 ! .  55  sidereal  time,  on  the  sun,  or  3t  55'  •  9 1  in  mean  solar  time ;  and 
which  amounts  .to  one  sidereal  day  in  the  course  of  a  year: — for 
3r55'.91  X  365'. 5 U8^ 48*.=  23t56"r4!:— hence  in  365  days  as  mea- 
sured  by  the  transits  of  the  sun  over  the  same  meridian,  there  are  366  days 
as  measured  by  those  of  a  fixed  star. 

Now,  because  of  the  earth's  equable  or  uniform  motion  on  its  axis,  any 
given  meridian  will  revolve  from  any  particular  star  to  the  same  star  again 
in  every  diurnal  revolution  of  the  earth,  without  the  least  perceptible  differ- 
ence of  time  shewn  by  a  watch,  or  clock,  that  goes  well : — and  this  pre- 
sents us  with  an  easy  and  infallible  method  of  ascertaining  the  error  and 
the  rate  of  a  watch  or  clock  :*«to  do  which  we  have  only  to  observe  the 
instant  of  the. disappearance  of  any  bright  star,  during  several  success  ive 
nights^  behind  some  fixed,  object,  as  a  chimney  or  pomer  of  a  bouse  at  a 


Digitized  by 


Google 


116  BJIftCIlIPTlbN  AWB  USB  OF  THB  TABLSd. 

litdii  dlsUtice^  lh6  position  o(  thi^  ejnei  bleing  fix^d  dt  8otil6  pftrtittulBr  ftpot, 
auch  as  at  ft  smftll  hole  in  a  window-dhutter  nearly  in  the  plane  of  the  meri- 
.  dian  5  then  if  the  observed  timed  of  disappearance  correspond  with  the  ac- 
celeration contftihed  iti  the  second  coliimn  of  the  first  compartment  of  the 
jpresent  Table,  it  will  be  an  undoubted  proof  that  the  wateh  is  well  regik- 
lated  :-^hence,  if  the  wiitch  be  exactly  true,  the  disappeiirance  of  the  samfc 
star  will  be  3*? 5 6'. earlier  every  night;  that  is,  it  will  disappear  3^56' 
sooner  the  first  night ;  7*52!  sooner  the  second  night;  1 1*48'.  sooner  the 
third  night,  and  so  on,  as  in  the  Table. — Should  the  watch,  or  clock  de- 
viate from  those  times,  it  must  b^  corrected  acbordingly ;  and  sinc6  the  dis^ 
appearance  of  a  star  is  instantaneous,  we  may  thus  determine  the  rate  df  a 
watch  to  at  least  half  a  second. 

The  first  compartment  of  this  Table  consists  of  two  columns ;  the  first  of 
which  contains  the  sidereal  days,  or  the  interval  between  two  successive 
transits  of  a  fixed  star  over  the  same  meridian,  and  the  second  the  accele- 
ration of  the  stats  expressed  in  mean  solar  time  $  which  is  extended  to  80 
days,  so  as  to  afford  ample  opportunities  for  the  due  regulation  of  clocks  or 
watches.— The  five  following  compartments  consist  of  two  columns  each, 
and  are  particularly  adapted  to  the  reduction  of  sidereal  dmfe  into  mean 
solar  time :— the  correction  expressed  in  the  column  marked  dcceleralfen, 
&c.  being  subtracted  from^  its  corresponding  sidereal  time,  will  reduce  it  to 
mean  solar  time ;  as  thus. 

Required  the  meah  solar  time  corresponding  to    14^40^55*.  iid^eal 

time } 

Given  sidereal  time  :=        14^40^55'. 

Corresponding  to  14  hours  is  •  ,  2*  17 '  •  61 ") 

Do.  40 minutes,  .0.   6  .55  > Sura  as  —     2*^24  .81 

Do.  55  seconds  •  .  0.   0.15  J  -:— ^ ^.^....^ 

Mean  solar  time  as  required     ...,.•••  14t38r30'.69 

J{6mdr)lr.-^Thi6TaMe  was  computed  in  the  fddlowing  manner;  vit.^ 

Since  the  earth  performs  its  revolution  round  its  orbit^  that  is,  roftnd 
the  sun,  in  a  solar  year;  therefore  as  366f5*48'?48'.  ;  360?::  1*.  :  69;8''.a| 
which,  therefore,  is  the  earth's  daily  advance  in  its  orbit :  but  winle  the 
earth  is  going  through  this  daily  portion  of  its  orbit,  it  turns  once  rowid 
on  its  axis,  from  West  to  east,  and  thereby  describes  an  arc  of  360?59'  8""^  8 
in  a  mean  solar  day,  and  an  arc  of  360?  in  a  sidereal  day. 

Httice,  as  360?59:8''.3  :  24*::3609  :  23*56?4'.09,.the  length  of  a 
sidereal  day  in  mean  solar  time;  and  which,  therefore^  evidently  anticipaites 
3"  55 ' .  9 1  upon  the  solar  day  as  before-mentioned,    Now, 


Digitized  by 


Google 


nSORIPnON  AND  U8B  OV  THB  TABUI8«  119 

Ab  one  aidtteal  day,  is  to  3*55'  *  91,  so  is  any  given  portion  of  sidereal 
time  to  its  corresponding  portion  of  mean  solar  time  i-^-^and  hence,  the  me* 
thod  hy  which  the  Table  was  computed. 


Tabus  XLVJ. 
Ta  reduce  Mean  Solar  Time  into  Sidereal  Time. 

Since  this  Table  is  merely  the  converge  of  the  preoeding,  it  is  presumed 
that  it  does  not  require  any  explanation  farther  than  by  observing,  that  the 
correction  is  to  be  applied  by  addition  to  the  corresponding  mean  solar 
time,  in  order  to  reduce  it  into  sidereal  tifife  ;  as  thus. 

Required  the  sidereal  time  corresponding  to  20M 5*33!  mean  solar 
time? 

Given  mean  solar  time  = •    20M5"3dl 

Corresponding  to  20  hours  is  3t  17' .  13^ 

Do.  15  minutes  0.   2  .46V Sum  =  .     +     3rl9;.68 

Do.  33  seconds  0,   0  .  09)  — 

Sidereal  time  as  required        .•..••..    20?18T52%68 


Tabijs  XLVIL 

Time  from  Noon  tohen  the  Sun*s  Centre  is  in  the  Prime  Vertical;  being 
the  instant  at  which  the  Altitude  of  that  Object  should  be  observed  in 

crier  ioaseertain  the  apparent  Time  with  the  greatest  Accuracy. 

• 

Since  the  change  of  altitude  of  a  celestial  object  is  quickest  when  that 
object  is  in  the  prime  vertical,  the  most  proper  time  for  obser^ng  an  alti- 
tude from  wiuch  the  apparent  time  is  to  be  faiferred,  h  therefiMPe  when  the 
object  is  due  east  or  west;  because  then  the  apparent  tine  is  not  likely  to 
be  affected  by  the  unavoidable  errors  of  observation,  nor  by  the  inaccuracy 
of  the  assmned  latitude. — This  Table  contains  the  lyparent  time  when  a 
celestial  object  is  iu  the  above  position.«-The  declination  is  marked  at  top 
and  bottom,  and  the  latitude  in  the  left  and  right  hand  marginal  columns  z 
hence,  if  the  latitude  be  5{)  degrees,  and  the  declination  10  degrees,  both 
being  of  the  same  name,  the  object  will  be  due  east  or  west  at  5 !  26*?  from 
its  time  of  transit  or  meridional  jMssage. 


£emarfc,<— TUs  Tabic  vsfl  computed  by  the  felk^wing  rule  j  vii^ 

/Google 


Digitized  by ' 


120  DBSCRIFTION   AND  USB  OF  THB  TABLES. 

To  the  log.  co-tangent  of  the  latitude,  add  the  log.  tangent  of  the  decli- 
nation ;  and  the  sum,  abating  10  in  the  index,  will  be  the  log.  co-sine  of  the 
hour  angle^  or  the  object's  distance  from  the  meridian  when  its  true  bearing 
is  either  east  or  west. 


Esamfle. 

Let  the  latitude  be  50  degrees,  north  or  south,  and  the  sun's  declination 
10  degrees,  north  or  south ;  requiredthe  apparent  time  when  that  object 
will  bear  due  east  or  west  ? 

Given  latitude  = 50?    log.  co-tangent  =  9.923814 

Declination  of  the  sun  =  .    .     .     10?    log.  tangent      ^  9. 246319 


Hour  angle  =      •    .    .    81 929'30'r =lpg.  co-sine      =  9. 170133 

In  time        ss       .    .    .      5^25?58!  j  which,  therefore,  is  the  ap- 
parent time  when  the  sun  bears  due  east  or  west. 

Note. — ^During  one  half  of  the  year,  or  while  the  sun  is  on  the  other  side 
of  the  equator,  with  respect  to  the  observer,  that  object  is  not  due  east  or 
west  while  above  the  horizon ;  in  this  case,  therefore,  the  observations  for 
determining  the  apparent  time  must  be  made  while  the  sun  is  near  to  the 
horizon ;  the  altitude,  however,  should  not  be  under  3  or  4  degrees,  on  ac- 
count of  the  uncertiunty  of  the  effects  of  the  atmospheric  refraction  on  low 
altitudes. 


Table  XLVIIL 

Altitude  of  a  Celestial  Object  {when  its  centre  is  in  the  Prime  Vertical,) 
most  proper  for  determining  the  apparent  T^me  with  the  greatest 
Accuracy. 

This  Table  is  nearly  similar  to  the  preceding;  the  only  difference  being 
that  that  Table  shows  the  apparent  time  when  a  celestial  object  bears  due 
east  or  west,  and  this  Table  the  true  altitude  of  the  object  when  in  that  po- 
sition ;  being  the  altitude  most  proper  to  be  observed  in  order  to  ascertain 
the  apparent  time  with  the  greatest  accuracy  : — thus,  if  the  latitude  be  50 
degrees,  and  the  declination  10  degrees,  both  being  of  the  same  name,  the 
altitude  of  the  object  will  be  13?61 ,  when  it  bears  due  east  or  west  from 
the  observer;  which,  therefore,  is  the  altitude  most  proper  to  be  observed^ 
for  the  reasons  assigned  in  the  explanation  to  Table  XL VII. 

7s^o<e.f-This  Table  was  computed  by  the  following  rule ;  viz.^ 

Digitized  by  VjOOQ IC 


]>BSCRIPTION  AND  U8IS  OF  THE  TABLBS.  .  121 

If  the  declination  be  less  than  the  latitude  ;  from  the  log.  sine  of  the 
former  (the  index  being  increased  by  10).  subtract  the  log.  sine  of  the  lat- 
ter, and  the  remainder  will  be  the  log.  sme  of  the  altitude  of  the  object 
when  its  centre  is  in  the  prime  vertical :-— But,  if  the  latitude  be  less  than 
the  declination,  a  contrary  operation  is  to  be  used ;  viz.,  from  the  log.  sine  of 
the  latitude,  the  index  being  increased  by  10,  subtract  the  log.  sine  of  the 
declination,  and  the  remainder  will  be  the  log.  sine  of  the  altitude  of  the 
object  when  its  centre  is  in  the  prime  vertical,  or  when  it  bears  due  east  or 
ivcsti 


Example  h 

Let  the  latitude  be  50?,  and  the  declination  of  a  celestial  object  10"?, 
both  being  of  the  same  name ;  required  the  altitude  of  that  object  when  iu 
centre  is  in  the  prime  vertical. 

Declination  of  the  object  s=  10?  log.  sine  s  9. 239670 

Latitude 50.  log.  sine  =  9. 884254 

Altitude  required  .    .    •  I3?6'6r      log.  sine  =s  9.355416 


Example  2. 

Let  the  latitude  be  3?,  and  the  declination  of  a  celestial  object  14?,  both 
being  of  the  same  name ;  required  the  altitude  of  that  object  when  its 
in  the  prime  vertical. 

Latitude 3?  log.  sine  =  8.718800 

Declination  of  the  object  =  14  log.  sine  =  9.383675 

Altituderequired  .       12?29'S8r       log.  sine  =  9.335125 

^ole.— Altitudes  under  3  or  4  degrees  should  not  be  made  use  of  in 
computing  the  apparent  time,  on  account  of  the  uncertainty  of  the  atmos- 
pheric refraction  near  the  horizon. 

And  since  the  Table  only  shows  the  altitude  of  a  celestial  object  most 
favourable  for  observation  when  the  latitude  and  declination  are  of  the  same 
name;  therefore  during  that  half  of  the  year  in  which  the  sun  is  on  the 
other  side  of  the  equator,  with  respect  to  the  observer, '  and  in  wliich  he 
does  not  come  to  the  prime  vertical  while  above  the  horizon,  the  altitude 
is  to  be  taken  whenever  it  appears  to  have  exceeded  the  limits  ascribed  to 
the  Oncertainty  of  the  atmospheric  refraction  in  page  120. 


Digitized  by 


Google 


182  raSCEIPTION  AKB  VME  OP  TUB  TAAUS* 


Tablb  XLDC. 

AmptUudes  of  a  Celestial  (^ect,  reckoned  from  the  true  East,  or  West 
Point  of  the  Horizcn. 

^1^e  arguments  of  this  Table  are,  the  declination  of  a  celestial  object  at 
top  or  bottom,  and  the  latitude  in  the  left,  or  right  hand  column ;  in  the 
angle  of  meeting  will  be  found  the  amplitude :  proportion,  however,  is  to 
be  made  for  the  excess  of  the  minutes  above  the  next  less  tabular  argu- 
ments. 

Example  1, 

Let  the  latitude  be  50?48' north,  and  the  sun's  declination  10?25C 
north ;  required  the  sun's  true  amplitude  at  its  setting  ? 

True  amplitude  corresp.  to  lat.  50?,  and  dec.  10?,  =W.  15?  40C  N. 

Tab.  diff.  to  1^  of  lat.  =21 '. ;  now    ^^^      =        +17,  nearly; 

T.difr.tol?ofdec.=l?36:,or96^inow^^^^'   =        +40 
Sun's  tme  amplttude  at  required      •    •    «    •    •    as  W.  16.  S7«  N. 

Example  2. 

Let  the  latitude  be  S4?24'  north,  and  the  sun's  declination  16?  48' 
south ;  required  the  sun*s  true  amplitude  at  the  time  of  its  rising  ? 
True  amplitnde  corresponding  to  latitude  S4?  N.  and 

declination  16?30r  S.  = E.  20?  2'  S. 

Tab.di£tDl?of]aU  ^  15?}  Mwl^^^iH^a.    •         +    6 

Tab.  diff.  to  30i  of  dec.  =  37^}  now  ^^iili'  m.    .         +22,  aetriy. 

Sun's  tone  amplitude  as  required  s       ...««&  20?30'  S, 

Remark. 

This  Table  was  computed  agreeably  to  the  following  rule ;  viz.. 
To  the  log.  sec^t  of  the  latitude,  add  the  log.  sine  of  the  declination, 
and  the  sum,  abating  10  in  the  index,  will  be  the  h^.  sine  of  the  true  am- 
plitude. 


Digitized  by 


Google 


J^SICftlPnON  AKD  Va  Of  TBS  TAftUIS*  12^ 

Example* 

hbt  the  latitude  be  50?48'.^  and  the  dedinatioa  of  a  celeetial  olject 
10^25' ;  required  the  true  amplitude  of  that  object  ? 

Latitude .    .    .    •    •    4  .  ^    50?48C    log.  secant    10. 190263 
Deelination 10. 2S     log.  sine  9.2S72U 

True  amplitude  asrequired  16?37'22r  log.  sine  .    •  9. 456474 


Table  L. 

Tx>JM  the  Timet  ttfUm  Ritbig  ond  Setting  tfa  Cde&tM  Olgeet. 

HiU  Table  contains  the  semidiurnal  arch,  or  the  time  of  half  the  continu- 
ance of  a  celestial  object  above  the  horizon  when  its  declination  is  of  the 
same  name  with  the  latitude  of  the  place  of  observation;  or  the.  time  of 
Kalf  its  continuance  below  the  horizon  when  its  declination  and  the  lati- 
tude are  of  different  denominations. — The  semudiurnal  arch  espreesee  the 
time  thai  a  celestial  object  takes  in  ascending  from  the  eastern  horizon  to 
the  fNeridian;  or  of  its  descending  from  tke  meridian  to  the  western 
hcriton. 

As  the  Table  is  only  extended  to  2d|  degrees  of  declination^  being  the 
greatest  declination  of  the  sun,  and  to  no  more  than  60  d^ees  of  latitude ; 
therefore,  when  the  declination  of  any  other  celestial  object  and  the  lati* 
tude  of  the  place  of  observation  exceed  those  limits,  the  semi-diurnal  arch 
is  16  be  computed  by  the  following  rule ;  viz.. 

To  the  log.  tangent  of  the  latitude,  add  the  log.  tangent  of  the  decHna- 
tion^  and  the  sum,  rejecting  lO  in  the  index,  will  be  the  log.  sine  of  an 
vdi;  whieh  being  converted  into  time,  and  added  to  6  hoyrs  when  the 
latitude  and  declination  are  of  Ihe  same  name ;  or  subtracted  from  6  hours 
when  these  elements  are  of  coiitrary  names  3  the  sum,  or  difference,  will  be 
the  eemi-diumal  arch. 

JSTmrnpIe  1. 

Let  the  latitude  be  61  degrees,  north,  and  the  declination  of  a  celestial 
olgectN|^5?  10^,  north ;  required  the  corresponding  semi-cUumal  arch  ? 
Latitude  %    «    .     .    .  61?  0^  north,  log.  tangent      10. 256248 
Declination.    •    .    .25.10    north,  log.  tangent        9.671963 

Afch=      .    .    .        57?57f2ir  =  log.  sine.    .    .9.928211 

Arch  eeov.  mtotime     3^51^49!  +  6i  s  9tSlU9'.,  the  semrdittr* 
Hal  arch,  as  r^^yArefk 


Digitized  by 


Google 


I 
124  PBSCRIPTION  AKD  USB  OF  THE  TABLBS. 

Example  2. 

Let  the  latitude  be  20?40',  south,  and  the  declination  of  a  celestial  ob- 
ject 80?29',  north  5  required  the  corresponding  semi-diurnal  arch  ? 
Latitude.     .    •     .    •  20?40:   south,  log.  tangent  .  .9.576576 
Declination      ...  30. 29    north,  log.  tangent  .  .  9. 769860 

Arch  = I2?49:45r  =  log.  sine      ...  9. 346486 

Arch  conv.  into  time     0*51-?19f  j  and  6t -0?51'?19?  =  5*8?4l!, 
the  semi-diurnal  arch. 

The  present  Table  has  been  computed  agreeably  to  the  first  example ; 
but  as  in  most  nautical  computations,  it  is  not  absolutely  necessary  that 
the  semi-diurnal  arch  should  be  determined  to  a  greater  degree  of  accu- 
racy than  the  nearest  minute;  the  seconds  have,  therefore,  been  reject^^ 
and  the  nearest  minute  retained  accordingly. 

Since  the  Table  for  finding  the  time  of  the  rising  or  setting  of  a  celes- 
tial object  (commonly  called  a  Table  of  semi-diurnal  and  semi^nocturnal 
arcs,)  is  scarcely  applied  to  any  other  purpose,  by  the  generality  of  nau- 
tical persons,  than  that  of  merely  finding  the  approximate  time  of  the  rising 
or  setting  of  the  sun  ;  the  following  problems  are,  therefore,  given  for  the 
purpose  of  illustrating  and  simplifying  the  Aise  of  this  Table ;  and  of  show- 
ing how  it  may  be  employed  in  determining  the  apparent  times  of  the  rising 
and  setting  of  all  the  celestial  objects  whose  declinations  come  within  its 
limits. 


Problbm  L 

Given  the  Latitude  and  the  Sun\8  DecJination,  to  find  the  Thne  of  Us 

RiHng  or  Setting. 

RULB. 

Let  the  sun's  declination,  as  given  in  the  Nautical  Almanac,  be  reduced 
to  the  meridian  of  the  given  place  by  Table  XV.,  or  by  Problem  L,  page 
76  5  then, 

Enter  the  Table  with  this  reduced  declination  at  top,  or  bottotti,  and 
the  latitude  in  either  of  the  side  columns ;  under  or  over  the  former,  and 
opposite  to  the  latter,  will  be  found  the  approximate  time  of  the  sun's  set- 
ting when  the  latitude  and  declinatioti  are  of  the  same  name ;  or  that  of 
its  rising  when  they  are  of  contrary  names.— The  time  of  setting  being 
taken  from  12  hours  will  leave  the  time  of  rising,  and  vice  versa,  the  time 
of  rismg  being  taken  from  12  hours  will  leav^  that  of  setting* 


Digitized  by 


Google 


BBSCRIPTION  AND   USB  OF  THB  TABLES.  125 

Note. — ^Proportion  must  be  made,  as  usual,  for  the  excess  of  the  minutes 
of  latitude  and  declination  above  the  next  less  tabular  arguments. 

Example  1. 

Required  the  approximate  times  of  the  sun's  rising  and  setting  July  13, 
1824,  in  latitude  50?48',  north,  and  longitude  120  degrees  west  ? 
Sun's  declination  July  13th.  per  Nautical 

Almanac,  is    ,..•.....  21?49C5ir  north. 
Correction  from  Table  XV.,  answering  to 
'   var.  of  dec.  StSS^,  and  long.  120?  W.  -     2^59^       • 

Sun's  dec.  reduced  to  given  meridian  •     •  21?46'52r;  or  21?47',  N.  * 

Time,  in  Table  L.,  ans.  to  lat.  50?,  north,  and 

dec.  21  ?30^,  north    =...., 7*52? 

4'  v4fi' 
Tabular  diflference  to  1?  of  lat.  =  4C]  now      ^ ^^,       =+3 

3'  X  17' 
Tab.  diflference  to  30'    of  dec.  =  3';  now     '   a^    •  =:+     2,  nearly. 

Approximate  time  of  the  sun's  setting 7^57" 

Approximate  time  of  the  sun's  rising        .    ..     ..       4*3? 

Nofe.— Twice  the  time  of  the  sun's  setting  will  give  the  length  of  the 
day  ;  and  twice  the  time  of  its  rising  will  give  the  length  of  the  night. 

Example  2. 

Required  the  approximate  times  of  the  sun's  rising  and  setting  October 
1st,  1824,  in  latitude  40?30'  nortli,  and  longitude  105  degrees  east  ? 
Sun's  declination  October  1st.  per  Nautical 

Almanac,  is 3?16C   6T  south. 

Correction  from  Table  XV.,  answering  to 

var.  of  dec.  23:20r,  and  long.  105?  E.    -  6:48r 

Sun's  dec.  reduced  to  the  given  meridian    3?  9' 18^,  or  3?9'  south. 
Time  in  Table  L.,  ans.  to  lat.  40?  north,  and 

dec.  3?  south,  is 6*  10? 

Tab.diff.  to  iroflat.  =  Oi  ;  now      qqT     =  0 

Tab.diff.  to  1?  of  dec.=  3:  5  now  ^^   =  0 

Approximate  time  of  the  sun's  rising €M0? 

Approximate  time  of  the  sun's  setting 5*50? 

*  Tiie  nearcjit  minute  of  declination  is  sufBcieiitly  exact  for  the  purpose  of  finding  the 
approuinate  thnet  of  the  risUlg  and  setting  of  a  celestial  object. 


Digitized  by 


Google 


126  DBSCaiPTION  AVD  VHB  Of  THB  TABLIU. 

Remark. 

Since  the  times  of  the  sun's  rising  and  settings  found  as  above,  will  differ 
a  few  minutes  from  the  observed^  or  apparent  times  in  consequence  of  no 
notice  having  been  taken  of  the  combined  effects  of  the  horizontal  refracr 
tion  and  the  height  of  the  observer's  eye  above  the  level  of  the  sea,  by 
which  the  time  of  rising  of  i^  celestial  object  is  accelerated,  and  that  of  its 
setting  retarded ;  nor  of  the  horizontal  par^lax  which  iiffects  these  tivnes 
in  a  contrary  manner;  a  correction,  (I)^refore,  must  be  applied  to  tl^e  ap- 
proximate times  of  rising  find  setting,  in  order  to  reduce  them  to  the  appa- 
rent tijDes.-^Tbis  correction  may  be  ooioputed  by  tha  Mlowing  role;  by 
which  the  apparent  times  of  the  son's  rising  and  setting  will  be  always 
found  to  within  a  few  seconds  of  the  truth* 

Rule.^^To  the  approximate  times  of  rising  and  setting,  let  the  longitude, 
in  time,  be  applied  by  addition  or  subtraction,  ac^prding  as  it  is  west  or 
east,  and  the  corresponding  times  at  Greenwich  will  be  obtained :  to  these 
times,  respectively,  let  the  sun's  declination  be  reduced  by  Table  XV*,  or  by 
Problem  I.,  page  76  5  then. 

Find  the  sum  and  the  difference  of  the  natural  sine  of  the  latitude,  and 
the  noturtd  co-pine  of  the  declination  (rejecting  the  two  right  band  figpres 
from  each  term),  and  take  out  the  common  log.  answering  thereto,  reject-, 
ing  also  the  two  right  hand  figures  from  each  : — now,  to  half  the  sum  of 
these  two  logs,  add  the  proportional  log.  of  the  sum  of  the  horizontal  re- 
fraction and  the  dip  of  the  horizon  diminished  by  the  sun's  horizontal 
parallax,  and  tha  constant  log.  1 . 1 76 1  * ;  the  sum  of  these  three  logs.,  abat- 
ing 4  in  the  index,  will  be  the  proportional  log.  of  a  correction ;  which 
being  subtracted  from  the  approximate  time  of  rising,  and  added  to  that 
of  setting,  the  apparent  times  of  die  sun's  rising  and  setting  will  be 
obtiiined. 

Tht»,«^Let  it  be  required  to-  reduce  the  approximate  tipies  of  the 
sun's  rising  and  setting,  as  found  in  the  last  Example,  to  the  respective  ap- 
parent times ;  the  horizontal  refraction  being  33^ ;  the  dip  of  the  horizon 
5^  15?,  and  the  sun's  horizontal  parallax  9  seconds* 

The  sun's  declination  reduced  to  the  .approximate  time  of  ridupg,  ia 
3?3'37^  and  to  that  of  setting  3514:^58?  south. 


*  This  is  the  proportional  log.  of  12  hours  esteemed  as  mfaiQtcs. 


Digitized  by 


Google 


SMCKIFTION  ANS  VSM  OF  THX  TABLBS.  127 

LaUtudfe  .    ;  40?30:  Or  nat.  tine  .    ,  :?  6494 
DecUuation   ,    3?  S:37'  nat.  co-tine  .  =  9986 

Sum 1648a  log.    =4.2170 

Difference 3492  hg.    =  3.5431 

Sum 7. 7601 


■^^ 


Half-rams  .    .    .      3.8800| 
33^+5n5r-9r=38^6r,prop.log.      .    .    .      0.6743 
Constant  log. 1.1761 

Correction ~  8^2K|nopJof.Bl.7a04| 

Approximate  time  of  rinng  =  .    .    .  6M0?  0! 

Appwent  time  of  aiQi'a  riaing  s:     .    .6^6739: 


Latitade  .    .  40?30:  0?  nat.  aine  .    .  =  6494 
Declination  .    3?14'58f  nat.  co-aine   .   ==  9984 

Sum     ..,..,.     16478  loy.  =  4.2169 
Diffnence 3490  log.  s  3.^428 

Sum,    .    •    .    p    .    7.7597 

Half  sum  =  .    •    ,     3.8798^ 
33^  +5M5r-9r=38^6r,  prop,  log.  =     .     0. 6743 
Constant  log.  .    .    .    .    ^    ,    •    •    •    •     1. 17<Sl 

Correction         .......  -f  St21f  prop.  log.  =:  1.7S02| 

Approximate  time  of  setting  =: .    .  5*50?  Of 

Apparent  time  of  son's  setting  =:    .5^53721? 

Note. — In  this  method  of  reducing  the  approximate  to  the  appaiwH 
time  of  rising  or  setting,  it  is  inunatcriai  whether  the  latitude  and  declina- 
tion be  of  the  same,  or  of  contrary  names :— nor  b  it  of  any  consequence 
whether  t)te  dedtnation  ht  reduced  to  the  approximate  tfmea  of  rising  and 
setting  or  not,  since  the  declination  at  noon  will  be  fdways  sufteientiy 
uact  to  determJM  the  correction  within  two  iv  three  seconds  of  Um»  trutliu 
on  aoooimt  of  its  natural  co-sine  h^xtg  only  required  to  four  places  of 
wiUan^air  evkkot  by  Arfefriag  to  tfae  ftbovt  Man 


Digitized  by 


Google 


128  DESCRIPTION  AND   USS  OP  THB  TABLES. 

although  there  is  a  difference  of  11 '2K  between  the  reduced  declinations 
at  the  approximate  times  of  rising  and  setting ;  yet  this  difference  has  no 
sensible  effect  on  the  correction  corresponding  to  those  times. 


Problem  II. 

Given  the  LatUude  of  a  Place  and  the  Decimation  of  a  fixed  Star,  to  find 
the  Times  of  its  Rising  and  Setting. 

Rule. 

Let  the  right  ascension  and  declination  of  the  star,  as  given  in  Table 
XLiV^  be  reduced  to  the  given  day ;  then^  from  the  right  ascension  of  the 
star,  increased  by  24  hours  if  necessary,  subtract  that  of  the  sun,  at  noon 
of  the  given  day ;  and  the  remainder  will  be  the  approximate  time  of  the 
star's  transit,  or  passage  over  the  meridian  3  from  which,  let  the  correction 
answering  thereto  and  the  daily  variation  of  the  sun's  declination  (Table 
XV.,)  be  subtracted,  and  the  apparent  time  of  the  star's' transit  will  be  ob- 
tained. 

If  much  accuracy  be  required,  and  the  place  of  observation  be  mider  a 
meridian  different  from  that  of  Green\^nch,  a  correction  depending  on  the 
longitude  and  variation  of  the  sun's  right  ascension  (Table  XV.,)  must  be 
applied  to  the  time  of  transit : — this  correction  is  subtractive  in  west,  and 
additive  in  east  longitude ;  the  time  being  always  reckoned  from  the  pre^ 
chding  noon :  now. 

Enter  Table  L.,  with  the  declination  at  top  or  bottom,  and  the  latitude 
in  the  side  column ;  and  in  the  angle  of  meeting  will  be  found  the  sesri- 
diurnal  arch,  or  the  time  of  half  the  star's  continuance  above  the  horizon, 
when  the  latitude  and  declination  are  of  the  same  name;  but  if  these  ele* 
ments  are  of  different  names,  the  time,  so  found,  is  to  be  subtracted  from 
12  hours,  in  order  to  obtain  the  half  continuance  above  the  horizon: 
then  this  half  continuance  *  being  applied  by  subtraction  and  addition  to 
the  apparent  time  of  transit,  will  give  the  approximate  times  of  the  star's 
rising  and  setting.  .    < 

Example  1. 

At  what  times  will  the  star  a  Arietis  rise  and  set  January  1st,  1824^  in 
latitude  50';48^  north? 


*  In  strictness  the  semi-diurnal  arch,  or  half  continuance  above  the  horizon  ought  to  be 
corrected  by  subtracting  therefrom  the  proportional  part  (Table  XV;,)  corjrespondiiif  to  it 
and  the  rariation  of  the  sun's  right  ascension  for  the  given  day. 


Digitized  by 


Google 


DBSCftlPTION   AND   US]^  OF  THB  TA^LBS.  129 

Star's  dec.  on  given ^ay  is  22?37'33r,  or  22?38C  north, 

and  its  right  ascension 1?57"16! 

Sun's  right  ascension  at  noon  of  the  given  day  is  •    •     •       18. 43. 58 

Approximate  time  of  star's  transit 7*  13. 18 

Correction  from  Tab.  XV.,  ans.*  to  7 *  Wrl 8 !,  and 4 ^  24^, 
the  v&r.  of  the  sun's  right  ascension  . —  1.20 

Apparent  time  of  star's  transit,  or  passage  over  the  meridian     7  *  1 1 T58 ! 
Time,  in  Tab.  L.  ans.  to  lat.  50?  N.,  and  dec. 

22?30'.  N.=      ..;.....        7^58? 

Tabular  diff.  to  V.  of  lat.  =  5:^  now  ^^— =+     4 
Tab.  diflf.  to  30^  of  dec.  =  4'.-,  now  ^'3^.^'=+     ^ 


Semi-diurnal  Arch,  or  time  of  half  the  star's 
continuance  above  the  horizon      •     •     •     =s     8^  3?.     •8?3?0'. 


Approx.  time  of  star's  tis^ng,  past  noon  of  Dec.  31st,  1823    23^  8T58! 

Approx.  time  of  star's  setting,  past  tioon  of  the  given  day    •  15M4T58! 

Example  2. 

At  what  times  will  the  star  Sirius  rise  and  set  January  1st,  1824,jn  lat. 
40?30C  north,  and  long.  120  degrees,  west  of  the  meridian  of  Greenwich  ? 

Star's  dec.  on  given  day  is  16^28^53:  or  16?29^  south, 
and  its  right  ascension 6*37T23! 

Sun's  right  ascension  at  noon  of  the  given  day  is   .    •     .     18. 43. 58 

Approximate  time  of  the  star's  transit  •  •  •  •  .  •  11.53.25 
Corr.  from.Table  XV.,  ans.  to  ll*53r25  f,  and  4^24% 

the  var.  of  the  sun's  right  ascension <^     2.11 

Corr.  from  ditto,  ans.  to  long.  120?  west,  and  4'24T  the 

var.  of  the  sun's  right  ascension    .    ' —     1.28 


Appar.  time  of  star's  transit  over  the  given  meridian  .     .     •     1 1 1 49T46' 
Time,  in  Table  L.,  an^.  to  lat.  40?  north,  and 

declination  16?  S.  =    .......     .    6*56? 

Tab.  diff.  to  1?  of  lat.  =  2^  ;  now  ^^^^  =     +  1 

Tab.  diff.  to  30i  of  dec.  =  2^ ;  now      y*    s=     +  2,  nearly. 

Semi-nocturnal  arch 6*59?, 

which  being  subtracted  from  12*  leaves 5*   1?  0'. 


Approximate  time  of  the  star.'s  rising  • 6*48?46'. 


Approximate  time  of  the  star's  setting    •••....    16*50746! 

/Google 


Digitized  by ' 


130  DBSCElipTION   AND  X78B  OF  THB  TABLB8. 

iZemarft.— 'The  approximate  times  of  the  rising  and  setting  of  a  fixed 
star  may  be  readily  reduced  to  the  respective  apparent  times  by  the  rule 
given  for  those  of  the  siin^  in  page  126  3  omittingi  however,  the  first  part, 
or  that  which  relates  to  the  reduction  of  declination :  and,  since  the  fixed 
stars  have  no  sensible  parallax,  the  words  '^horizontal  parallax''  are,  also, 
to  be  omitted }  thus : — 

To  reduce  the  approximate  times  of  rising  and  settings  as  found  in  the 
last  example,  to  the  respective  apparent  times,  the  dip  of  the  horizoh  being 
assumed  at  6'30r 

Latofplace  of observ.  40930'  Nat.  sine  =  6494 
Declin.  of  the  star  =:    16. 29    Natco-sine=:9589 

Sum  r:     •    «    •    16083    Log.  =     .    4. 2064 
Difference  =:     .      3095    Log.=:     »    3:4907 


Sum=    •    7.6971 


Half  sum  =3.84851 
Horiz.  iPefrac.=:33^  +dip  of  horiz.=6^30^=39:30r  Prop.  log.=  0. 6587 
Constant  log.  = ....'•     1.1761 


Corrections.    .........      SM4r  Prop.  log.  =:  1.6833  J 

Now,  this  correction  being  subtracted  from  the  approximate  time  of 
rising,  and  added  to  that  of  setting,  shows  the  former  to  be  6!45*2!,  and 
the  latter  16*54?S0: 


Pboblbm  III. 

Given  the  Latitude  qf  a  Place,  and  the  DecUnatim  qf  a  Planet,  to  find 
the  Times  of  its  Rising  and  Setting. 

Rule.  . 

Take,  from  page  IV«  of  the  month  in  the  Nautical  AlmatiaC,  the  times 
of  the  i^anet's  transits  for  the  days  nearest  preceding  and  following  the 
given  day,  and  find  their  difference;  then  say,  as  6  days  arc  to  this  differ- 
ence, so  is  the  interval  between  the  given  day  and  the  nearest  preceding 


Digitized  by 


Google 


DBSCJtIPTfON  AVD  USB  OF  THB  TABUS.  131 

davj  to  a  correction ;  which,  being  applied  by  addition  or  subtraction  to 
the  time  of  transit  on  the  nearest  preceding  day,  according  as  it  i^  increa^ 
ing  or  decreasing,  the  sum  or  difference  will  be  the  ajJproxiipate  time  of 
transit.  Find  the  interval  between  the  tiiAes  of  transit  on  the  days  nearest 
preceding  and  following  the  given  day ;  and  then  say,  as  the  interval 
between  the  times  of  transit  is  to  the  difference  of  transit  in  that  interval, 
so  is  the  longitude,  in  time,  to  a  correction ;  which,  being  added  to  the  ap- 
proximate time  of  transit  if  the  longitude  be  west  and  the  transit  increasing, 
or  subtracted  if  decreasing,  the  sum  or  difference  will  be  the  apparent  time  of 
the  planet's  transit  over  the  meridian  of  the  given  place ;  but  if  the  longi- 
tude be  east,  a  contrary  process  is  to  be  observed :  that  is^  the  correction  b 
to  be  subtracted  from  the  approximate  time  of  transit  if  the  transit  be 
increasing,  but  to  be  added  thereto  if  it  be  decreasing. 

.To  the  apparent  time  of  transit,  thus  found,  apply  the  longitude,  in  time, 
by  addition  or.  subtraction,  according  as  it  is  west  or  east ;  and  the  sum  or 
difference  will  be  the  corresponding  time  at  Greenwich.  To  this  time,  let 
the  planet's  declination  be  reduced  by  Problem  IIL,  page  83;  or -as 
thus:— 

Take,  from  the  Nautical  Almanac,  the  planet's  declination  for  the  days 
nearest  preceding  and  following  the  Greenwich  time,  and  find  the  differ- 
ence ;  find,  also,  the  difference  between  the  Greenwich  time  and  the  nearest 
preceding  day  :.then  say,  as  6  days  are  to  the  difference  of  declination,  so 
is  the  difference  between  the  Greenwich  time  and  the  nearest  preceding 
day,  to  a  correction ;  which,  being  applied  to  the  declination  on  the  nearest 
preceding  day,  by  addition  or  subtraction,  according  as  it  may  be  increas- 
ing or  decreasing,  the  sum  or  difference  will  be  the  planet's  correct  decli«* 
nation  at  the  time  of  its  transit  over  the  given  meridian.     Now, 

With  the  planet's  declination  and  the  latitude  of  the  given  place,-  enter 
Table  L.,  and  find  the  corresponding  semidiurnal  arch^  by  Problem  II.,  page 
128;  and,  thence,  the  approximate  times  of  rising  and  setting,  in  the  same 
manner  as  if  it  were  a  fixed  star  that  was  under  consideration. 


Example  1. 

At  what  times  will  the  planet  Jupiter  rise  and  set,  January  4th,  1824,  in 
latitude  36?  north,  and  longitude  135?  west  of  the  meridian  of  Greenwich  7 


*  In  ftrictaesa  the  semidiurnal  arch  ought  to  be  corrected  by  adding  thereto^  or  sub* 
tracting  therefrom,  the  proportional  part  corresponding  to  it  and  the  daily  yariation  of 
transit,  according  as  the  transit  may  be  increasing  or  decreasing. 

k2 


-^ 


Digitized  by 


Google 


132  DKSCRTWION   AND   USE  OF  THB  TABLES, 

Timcofpreced.trans. Jan.l,is ll*38?nearestprec.dayl8t,traa8.ll*38T  0! 

Time  of  follow,  trans.  Jan.7>  is  1 1 .    8  given  day  4th 

•  ,  ,  , 

As6fi8toO!30T,8ois  3f  to      -    ISr  0? 


Approximate  time  of  transit  on  the  given  day  =     ••    •     •      11*23?0! 
Time  of  preceding  transit  =     •     •     I'll  ^SS? 
Time  of  following  transit  =.    •     •    7*11«   8 


Interval  between  the  times  of  trans.=5  f  23 1 30? 

As  interval  between  times  of  trans.=5 f  23*30*  !  diff.  of 

transits  30"  ::  longitude  in  time  =:  9*  to  .     .     .      —   1.53 


Apparent  time  |)f  transit  over  given  merid.  Jan.  4th,  1824  =     11*21?  7' 
Longitude  135  degrees  west,  in  time  zz 9.   0.   0 

Corresponding  time  at  Greenwich  =:     .     . 20*21?  7* 

Plahet'sdec.Jan.lis=23?17'N.;near.prec.lf  0*  0?0!  dec.23«17.'  O^N. 
Ditto  7  is=23. 20  N.^Gr.  tim.=4.  20-  21 . 7 


As    efistoO?  3:  BO  is    .    .  3f20?2ir7'    to     +  1.55 


Jupiter's  dec.  reduced  to  his  app.  time  of  transit  over  the 

given  meridian  = 23?18'55rN, 

Time,  in  Table  L.,  ans.  to  lat.  36?  north,  and  dec.  23?'N.  =     7*  12?  0! 
Tabular  diflference  to  30:  of  dec.  =  2',;  now^^'^/^'  =     .     +1.16 

Semidiur.  arch,  or  time  of  half  planet's  contin.  above  the  hor.  =  7^  13?  16! 
Apparent  time  of  Jupiter's  transit  over  the  given  meridian  =   11. 21.    7 

Approximate  time  of  Jupiter's  rising  at  the  given  meridian  =:     4t  7?51 ! 

Approximate  time  of  Jupiter's  setting  at  ditto  =  «...     18*34?23f 

Example  2. 

At  what  times  will  the  planet  Mars  rise  and  set,  January  16th,  1824, 
in  latitude  40?  north,  and  longitude  140?  east  of  the  meridian  of  Green- 
wich ? 


Digitized  by 


Google 


BESCRIPTION  AND  USE  OF  THE  TABLES*  133 

Timeof prcccd.trans.lSth, i8l6*54?3  near.prec.dayl3th,tran8.=  16?54?  0! 
Tiineoffollow,trans.l9th,j8l6.34;   given  day      16th 

As  6f  is  to 0? 20?,  80  is      .     .    3f   to-    .  -     10?  0! 


Approximate  time  of  transit  on  the  given  day  =:••••     16M4?  0! 

Interval  between  the  times  of  transit  =  5"?  23*40? 
Aa  interval  between  times  of  transit  ==  5*23?40!  ;  diff,  of 

trans.  =  20?  ::  long,  in  time  =  9*20?,  to     .     .     .     •         +     1. 18 


Apparent  time  of  trans,  over  given  merid.  Jan.  16th,  1824  =:  16*45?18! 
Longitude  of  the  given  merid.  =  HO?  east,  in  time  =:    •    •      9. 20.   0 

Corresponding  time  at  Greenwich  = 7* 25?  18' 

Dcc.ofMars,Jan.l3,i8  0?37'S.;near.prec.l3f  0*  0?  0:dec.0'?37'  O^S. 
Ditto  19,isl.ll  S.;Gr.time=16.    7.25.18 


As   6f  i8to0?34^8ois     .      3f  7*25?18'.  to  +  18.45 


Dec.  of  Mars  reduced  to  his  apparent  time  of  transit  over 

the  given  meridian  = «•«..•    0?55U5^S. 

Semidiurnal  arch  in  Table  L.,*  answering  to  lat.  40?  N.  and 

dcc.0?55M5rS.,ia6*2?48!;sub.  from  12*  leaves  •  .  .  5?S7?12.' 
Apparent  time  of  the  planet's  transit  over  the  given  meridian:=16. 45. 18 

Approximate  time  of  rising  of  the  planet  Mars  =    .    •    •     .  10^48?  6 .' 

Approximate  time  of  setting  of        ditto  =    .    •    .    •  22M2?30! 

IZemarfc.— The  approximate  times  of  a  planet's  rising  and  setting  may 
be  reduced  to  the  respective  apparent  times,  by  the  rule  in  page  126,  for 
reducing  those  of  the  sun ;  omitting,  however,  the  first  part,  or  that  which 
relates  to  the  reduction  of  declination,  and  reading  planet's  instead  of  sun's 
horizontal  parallax :  this,  it  is  presumed,  does  not  require  to  be  illustrated 
by  an  example.      . 


Digitized  by 


Google 


134  DBSCRIITION  AND   USB  OF  THB  TABLES. 

Paoblem  IV. 

Given  the  Latitude  of  a  Place,  and  the  Moon's  DediMiion^  to  find  Hie 
Times  of  her  Rising  and  Setting. 

Rule. 

Take;  from  page  VI.  of  the  month  in  the  Nautical  Almanac^  the  moon's 
transit,  or  passage  over  the  meridian  of  Greenwich,  on  the  given  day,  and 
also  her  declination.  Let  the  time  of  transit  be  reduced  to  the  meridian  of 
the  given  place  by  Table  XXXVIII.j  to  which  apply  the  longitude,  in  time^ 
by  addition  or  subtraction,  according  as  it  is  west  or  east;  and  the  sum,  or 
difference,  will  be  the  corresponding  time  at  Greenwich  :  to  this  time,  let 
the  declination  be  reduced  by  Table  XVL,  or  by  Problem  II.j  page  80;«— 
then. 

With  this  reduced  declination,  and  the  latitude  of  the  given  place,  find 
the  moon's  semidiurnal  arch,  or  the  time  of  half  her  continuance  above  the 
horizon,  by  IVoblem  IL,  page  128,  and,  thence,  the  approximate  times  of 
rising  and  setting,  in  the  same  manner  precisely  as  if  it  were  a  fixed  star 
that  was  under  consideration :  call  these  the  estimated  times  of  rising  and 
setting. 

To  the  estimated  limes  of  rising  and  setting,  thus  found,  let  the  longitucfe^ 
in ,  time,  be  applied  by  addition  or  subtraction,  according  as  it  is  west  or 
east ;  and  the  sum,  or  difference,  will  be  the  corresponding  times  at  Green- 
wich. 

To  these  times  respectively,  let  the  moon's  declinati<Hi  be  reduced  by 
Table  XVL,  or  by  Problem  11^  page  80;  with  which,  and  the  Utitttde, 
find  the  moon's  semidiurnal  arch  at  each  of  the  estifnated  times. 
-  To  the  respective  semidiurnal  arches,  thus  found,  apply  the  corrections 
corresponding  thereto,  and  the  retardation  of  the  moon's  transit  (Table 
XXXVI JL)  by  addition,  and  the  correct  semidiurnal  arches  will  be  ob* 
tained. 

Now,  the  semidiurnal  arch  answering  to  the  estimated  time  of  rising, 
being  subtracted  from  the  moon's  reduced  transit,  will  leave  die  approx- 
imate time  of  her  rising  at  the  given  place ;  and  that  corresponding  to  the 
estimated  time  of  setting,  being  added  to  the  moon's  reduced  transit,  will 
give  the  approximate  time  of  her  setting  at  the  sud  place. 

Example  1. 

Required  the  times  of  the  moon's  rising  and  setting,  Jan.  17th,  1824,  in 
latitude  51?29'  north,  and  longitude  78?45C  west  of  the  meridian  pf 
Greenwich  ? 


Digitized  by 


Google 


MSCftlFTtON.AND  VSft  OF  THB  TABLM.  13S 

Moon's  transit  over  mend,  of  Greenwich  on  the  given  day 'is     13*34?  0' 
Corr.  fr.  Tab.  XXXVIIL^ans.  to  retard.  53  %  and  long.  75?  west  +   10.39 

App.  time  of  moon's  transit  reduced  to  the  given  meridian  •     13M4T39! 
Longitude  78?45^  west,  in  thne  zz     .......    .      5. 15.   0 

Corresponding  time  at  Greenwich 18t59?39! 

Moon's  dec.  red.  to  Gr.  time,  by  Table  XVI.,  is  10?25f  SO^N. 

Semidiurnal  arch,  in  Table  L.,  answering  to  lat.  51?29'N., 

and  declination  1 0?  25  ^N.,  is      ......;,.      6t54?  Of 

Moon's  reduced  transit     ••••••• 13.44.39 

JBrtJmaled  time  of  the  moon's  rising    .    •    «        •    .-  •    •      6t50r39'. 

J&tima<€d.  time  of  the  moon's  setting      • 20^38739! 

To  find  the  ap(Mroximale  Time  of  Rising  :•«- 

Estimated  time  of  rising        6^50739! 

Loi^tude  78?45^  west,  in  time  =    ,    « 5. 15.   0 

Greenwich  time  past  noon  of  the  given  day 12t  5?39! 

Moon's  dec.  reduced  to  Greenwich  time,  is  12?  10^  53^N. 

Time,  inTableL,ans.tolat.51?29'N.anddec.  12?ll'N.,is    7*  3r  0! 
ConrectSon,Table.XXXVIU.,An^tQ53:aiid7^3r  =      »    .  -h  15.  0 

Moon's  correct  semidiurnal  arch  at  rising  .     • 7^8?  0! 

Moon's  reduced  transit       .>•    \    •«•••.••  13. 44. 39 

Approximate  time  of  moon's  rising '•    6t26T39! 

•To  find  Utie  approximate  Time  of  Setting  :*-- 

EfflMMKed  time  of  setting 20^38r39! 

Longitude  78M5<  west^  in  time  s: 5. 15.  0 

Greenwichtimepastnoonof  the  18th      • 1^53^39! 

Moon's  dec.  reduced *to  Green^vich  time,  is  8?41 '  1 KN.  . 

Time,  in  Table  L.,  answ.  to  lat.  51?29:N.  and  dec.  8?41  <N.,  is  6M4r  0! 

Correction,TableXXXVlII.,ans.  to53C  and6*44r  =   ;     .     +14.0 

Moon's  correct  sepiidiurpal.arch  at  setting 6^58T  0' 

Mood's  reduced  transit 13.44.39 

Approximate  time  of  moon's  setUng    ••«•...«    20M2T39: 

Digitized  by  VjOOQ IC 


136  PB6CR1PTI0N  AND   USB  OF  THB  TABLBS. 

•  '  Example  2. 

Required  the  approximate  times  of  the  moon's  rising  and  setting,  Janu- 
ary 20th,  18^24^  in  latitude  40^30'  north,  and  longitude  80  degrees  east  of 
the  meridian  of  Greenwich  ? 

Moon's  transit  over  the  merid.  of  Greenwich  on  the  given  day  is  16S  6T  0! 
Cor.  fr.  Tab.  XXXVIIL,  ans.  to  retard.  49:  and  long.  80^  east   —  10. 32 

Moon's  transit  reduced  to  the  given  meridian     •    •    •'  .    .     1S^55?28! 
Longitude  80  degrees  east,  in  time  ;= 5.20.   0 

Greenwich  time 10^35T281 

Moon's  dec.  red.  to  Green,  time,  by  Table  XVI.,  is  5?55t40rS. 

Seminoctumal  arch,  in  Table  L.,  angering  to  lat.  40?30^N. 
and  dec.  5?56^S.  =  6*20r,   subtracted  from  12^,  leaves   5M0r  0! 
^Moon's  reduced  transit •     •    •    •    .    .     15.55.28 

£9timafed  time  of  the  moon's  rising 10M5T28! 

£^ma<ed  time  of  the  moon's  setting 21t35?28' 

To  find  the  approximate  Time  of  Rising  :— 

JStffimaf^  time  of  rising 10M5r28! 

Longitude  80  degrees,  east,  in  time  == 5. 20.   0 

Greenwich  time  = •    .    .    •      4t55?28! 

Moon's  dee.  reduced  to  this  time,  is  4?30'49TS. 

Time,  in  Table  L.,  answering  to  lat.  40?30'N.,  and  dec. 

4?31  CS.  is  6t  I5r,  which,  subtracted  fwm  12\  leaves  .  5U5?  0! 
Corr.  Table  XXXVIIL,  answering  to  49^  and  5M5r      ..+11.0 

Moon's  correct  semidiurnal  arch  at  rising      ....    .    •    •      5t56T  0! 
Moon's  reduced  transit 15.55.28 

Approximate  time  of  moon's  rising 9^59T28! 

To  find  the  approximate  Time  of  Setting : — 

JSsHmafcd  time  of  setting 21*35?28! 

Longitude  80  degs.  east,  in  time  = 5.20.  0 

Greenwich  time  = 16*15T28! 

Moon's  dec.  reduced  to  this  time,  is  7°  17^52^8, 


Digitized  by 


Google 


]>B8CRtFriON   AND  VSi  OF  THS  TABLES.  137 

Time,  in  Table  L.,  answering  to  lat.  40?30^N.  and  dec. 

7?18'S,,  is  6i25T,  which,  subtracled  from  12t,  leaves      .      5*35?  0! 
Corr.  Table  XXXVIIL,  ans.  to  49C  and  5*35?       ...*.+   11.   0 

Moon's  correct  semidiurnal  arch  at  setting 5*46T  0! 

Moon's  reduced  transit    •     •    •    •* •     •     •     15.55.28 


Approximate  time  of  moon's  setting 21*41?28! 

Remark. — ^The  approximate  times  of  the  moon's  rising  and  setting  may 
be  reduced  to  the  respective  apparent  times  by  the  following  rule  ;  viz., 

Find  the  sum  and  the  difference  of  the  natural  sine  of  the  latitude  and 
the  natural  co-sine  of  the  declination  at  the  estimated  times  of  rising  and 
setting  (rejecting  the  two  right-hand  figurcjs  from  each  term),  and  find  the 
common  log.  answering  thereto^  rejecting  also  the  two  right-hand  figures 
from  each.  Now,  to  half  the  sum  of  these  two  logs,  add  the  constant  log. 
1. 1761,*  and  the  proportional  log.  of  the  difference  between  the  horizontal 
parallax  and  tlie  sum  of  the  horizontal  refraction  and  dip  of  the  horizon : 
the  sum  of  these  three  logs.,  abating  4  in  the  index,  will  be  the  propor- 
tional log.  oC  a  correction,  which,  being  added  to  the  approximate  time  of 
rising  and  gubtracled  from  that  of  setting,  the  respective  apparent  times  of 
rising  and  setting  will  be  obtained  :  thus, 

Let  it  be  required  to  jeduce  the  approximate  times  of  rising  and  settings 
as  found  in  the  last  example,  to  the  respective  apparent  times,  the  dip  of 
the  horizon  being  4  ^  50^ 

Note* — The  moon's  horizontal  parallax  computed  to  the  reduced  eiti- 
fnJated  time  of  rising,  is  59 '6?,  and  that  at  the  reduced  time  of  setting 
58U0? 

Latitude        40?30C     Nat.  sine    6494 
Declination     4.31       Nat.co-sine9969 

Sum     ......     1B463  Log.  .....    4.2165 

Difference      ....      8475  Log.  ...    .    3.5410 

Sum  •    ...    •    7.7575 


Half  sum     •    .    3. 8787i 
.59^6f-37'50^  (33C  +  4^50r)  =:  2ia6^     Prop.  log.    .    .    0.9276 
Constant  log. 1.1761 

Correction +    1^52'/     Prop. log.    .     .     1.9824^ 

Approximate  time  of  rising   =     9* 59^28 f 

Apparent  time  of  moon's  rising  =  10 1   1  r20'. 

^  lliis  is  the  proportional  to^*  of  12  hours  esteemed  as  minutes. 

Digitized  by  VjOOQ IC 


ISA  OBSCftlPTIOK  AND  tT6B  O^  THB  tA&LBft« 

Latitude       40?30f  Nat.  sine      6494 
Declination     7^18    Nat.  co-sine  9919 

Sum     •...••    .     16413  Log 4.2152 

Difference      .    •    .    ;      3425  Log 3.5347 


.Sum  .    •    .    •    7.7499 


Half  sum      •    .    3.8749^ 
58!40r-37'50r(33:  +  4r50r)=:20C50r  Prop.  log.    .    .    0.9365 
Constant  log 1.1761 

Correction ~  1^5K  Prop.  log.  •    .     1.98751 

Approximale  tiqae  of  setting  =:   .    2M41'r28! 

>    ■  • 

Apparent  time  of  moon's  setting  =  21  t39T37  '• 

Nofe.— The  direct  method  of  solving  thb  and  the  three  preceding  Pro* 
blems,  by  spherical  trigonometry,  is  given  in  some  of  the  subsequent  pages 
of  this  work. 


Tables  LI.  akd  LU. 

For  computing  the  Meridianal  JltUude  of  a  Celestial  ObjecL 

Since  it  frequently  happens,  at  sea,  that  the  meridional  altitude  of  thft 
sun,  or  other  celestial  object,  cannot  be  taken,  in  consequence  of  the  iater* 
position  of  clouds  at  the  time  of  its  coming  to  the  meridian  ;  and  since  it 
is  of  the  utmost  importance  to  the  mariner  to  be  provided  at  all  tines,  with 
the  means  of  determining  the  meridional  altitude  of  the  heavenly  bodies, 
for  the  purpose  of  ascertaining  the  exact  position  of  his  ship  .with  respect  to 
latitude,  these  Tables  have  therefore  been  carefully  computed  j  by  means  of 
which  the  meridional  altitude  of  the  suq,  or  any  other  celestial  object  whose 
declination  does  n6t  exceed  28  degrees,  may  be  very  readily  obtained  to  a 
sufficient  degree  of  accuracy  for  all  nautical  purposes,  provided  the  altitude 
be  observed  within  certain  intervals  of  noon,  or  time  of  transit,  to  be 
governed  by  the  meridional  zenith  distance  of  the  object:  thus, /or  thesun^ 
the  number  of  minutes  ai)d  parts  of  a  minute  contained  in  the  interval 
between  the  time  of  observation  and  nopn,  must  not  exceed  the  number  of 
degrees  and  parts  of  a  degree  contained  in  the  object's  meridional  zenith 
distance  at  the  place  of  observation.  And  since  the'meridional  zenith  dis- 
tance of  a  celestial  object  is  expressed  by  the  difference  between  the  lati- 
tude and  the  declination  when  they  are  of  the  same  name,  or  by  their  sum 


Digitized  by 


Google 


DiSCBIPTION  AND  USB  OF  THB  TABtBS.  139 

when  of  contrary  names ;  therefore  the  extent  of  the  interval  from  noon 
(within  which  the  altitude  should  he  observed)  maybe  determined  by 
means  of  the  diff(ftrence  between  the  latitude  and  the  declinadon  when  they 
are  both  north  or  both  south,  or  by  their  sum  when  one  is  north  and  the 
other  south.  Thus,  if  the  latitude  be  40  degrees,  and  the  declination  8 
degrees,  both  of  the  same  name,  the  interval  between  the  time  of  taking 
the  altitude  and  noon  mast  not  exceed  32  minutes ;  but  if  they  be  of  differ- 
ent names,  the  altitude  may  be  taken  at  any  time  within  48  minutes  before 
or  after  noon :  if  the  latitude  be  60  degrees,  and  the  declination  10  degrees, 
both  of  tbe  same  name,  the  interval  between  the  time  of  observation  and 
noon  ought  not  to  exceed  50  minutes ;  but  if  one  be  north  and  the  other 
south,  the  interval  may  be  extended,- if  necessary,  to  70  minutes  before  or 
after  noon,  and  so  on. 

The  limits  within  which  the  altitudes  of  the  other  celestial  objects  should 
be  observed,  may  be  determined  in  tbe  same  manner;  taking  care,  how- 
ever, to  estimate  the  interval  from  the  time  of  transit  or  passage  over  thi 
meridian,  instead  of  from  noon. 

Now,  if  the'  altitude  of  the  sun  or  other  celestial  object  be  observed  at 
am/  time  within  the  limits  thus  prescribed,  and  the  time  of  observation 
be  caieAilIy  noted  by  a  well-regulated  watch,  the  meridional  altitude  of 
such  object  may  then  be  readily  determined,  to  every  desirable  degree  of 
accuracy,  by  the  following  rule;  viz., 

Enter  Table  LI.  or  Lll.,  according  as  the  latitude  and  the  declination 
are  of  the  same  or  of  contrary  names,  and  with  the  latitude  in  the  side 
cohunn,  and  tfie  declination  (reduced  to  the  meridian  of  the  place  of  6b- 
serration)  at  the  top  or  bottom ;  take  out  the  corresponding  correction  in 
seconds  and  thirds,  which  are  to  be  esteemed  as  mimUei  aitd  seconds  ; — 
then. 

To  the  proportional  log.  of  this  correction,*  add  twice  the  proportional 
log.  of  the  interval  between  the  time  of  observation  and  noon,  or  time  of 
transit,  and  the  constant  log.  7*  3730 ;  and  the  sum  urill  be  the  proportional 
log.  of  a  correction,  which,  being  added  to  the  true  altitude  deduced  from 
observation,  will  give  the  correct  meridional  altitude  of  the  object. 

Note  ]  • — In  taking  out  the  numbers  from  Tables  LI.  and  LIL,  proportion 
must  be  made  for  the  excess  of  the  given  latitude  and  declination  above 
the  next  less  tabular  arguments. 


*  When  the  object  either  ooracf  to,  or  wlthio,  one  decree  of  the  zenith,  the  anple  of 
meetiiic  nade  by  the  latitude  and  declination  will  fall  ivithin  the  aip^ag  double  lines 
which  mn  through  the  body  of  Table  LI.,  and  (hroogfi  the  upper  left-hand  corner  of  Table 
UI :  ia  IliU  ease,  since  the  intenral  between  the  time  of  obserfation  and  noout  or  meri* 
dional  paisa^,  most  not  exceed  one  minute,  tbe  corresponding  number  wiU  be  tbe  correc- 
tion of  tltitude  direct,  independently  of  any  calculation  whatever. 


Digitized  by 


Google 


140  DESCRIPTION  AND  VSB  OF  TUB  TABLEB. 

2.-^The  interval  between  the  time  of  observation  and  noon  may  be 
always  known  by  means  of  a  chronometer,  or  any  well-regulated  watch ; 
making  proper  allowance,  however,  for  the  time  comprehended  under  the 
change  of  longitude  since  the  last  observation  for  determining  the  error  of 
such  watch  or  chronometer. 


Example  I. 

In  latitude  45?  north,  at  34740!  before  noon,  the  sun's  true  altitude  was 
found  to  be  54?12'49'/,  when  his  declination  was  10?  north;  required  the 
meridional  altitude  ? 

Corr.  in  Table  LL,  ans.  to  lat.  45?  and  dec.  10?,  is  2^23*^.  1  j 

the  propor.  log.  of  which  is »     K8778 

Interval  between  time  of  obs.  and  noon,  34740!,  twice  prop.  log.=  1.4308 
Constant  log. «    .    «     7*2730 

Correction  of  altitude     .    .     0?47'  lOr        Prop.  log.  =:     .    ,    0. 5816 
True  alt.  at  time  of  observ.    54. 12. 49 


Sun's  meridional  altitude       54?59^59!';  which  is  but  one  second  less 
than  the  truth. 

Example  2» 

In  latitude  48?  north,  at  1^5748!  past  noon,  the  sun's  true  altitude  was 
found  to  be  20?25'5^',  w;hen  his  declination  was  20  degs*  south}  required 
the  meridional  altitude  ? 

Corr.  in  Table  LIL,  answering  to  lat.  48?N.  and  dec.  20?S.,  is 

1^19*^.9,  the  propor  log.  of  which  is  V.  ...  ,  .  .  .  2.1308 
Intend  between  time  of  obs.  and  noon  1*5748!,  twice  prop.  log.=0. 8740 
Constant  log.      ....«««.,••».««*    7-  2730 

Correction  of  altitude    •     •       1?34!57^         Prop.  log.  =       .    0.2778 
True  alt.  at  time  of  obseiy*     20. 25.   5 


Sun's meridionalaltitude     .     22?  OC  2''}  which  is  but  two  seconds  more 
than  the  truth. 

Example  3. 

At  sea,  March  22d,  1824,  in  latitude  51?  16!  north,  at  50732'.  past 
noon,  the  sun's  true  altitude  was  found  to  be  38?20!567  ;  required  the 
meridional  altitude,  the  declination  being 0?43!5 17  north? 


Digitized  by 


Google 


DESCRIPTION   AND   USB  OF  THE  TABLES.  141 

Corr.  in  Table  LI.,  answering  to  lat.  5 1  ?  16Und  dec.  0?43  '51? 

is  1  ^35*'.  6,*  the  propor.  log.  of  which  is 2, 0530 

Interval  between  time  of  obs.  and  noon  50T32',  twice  prop.  log.=s  1. 1034 

Constant  log :....• 7-2730 

Correction  of  altitude  •     .       1?  6^58r         Prop.  log.  s     .    .  0. 4294 
True  alt.  at  time  of  observ.     38. 20. 56 


Sun's  meridional  altitude   .    39?27'541';  which  differs  but  three  seconds 
from  the  truth. 

Example  4. 

At  sea,  December  2l8t,  1824,  in  latitude  60? 22^  north,  at  10*36f  10! 
A.M.,  or  1^23T5p'  before  noon,  the  sun's  true  altitude  was  found  to  be 
4?26C38?;  required  his  meridional  altitude,  the  declination  being 
23?27'45r  south?     • 

Corr.  in  Table  LII.,  ans,  to  lat.  60922C  and  dec.  23?27U5?, 

is  OrSS*'.  8,t  the  propor.  log.  of  which  is 2.3026 

Interval  between  time  of  obs.  and  noon  1*23?50!,  twice  prop.log.ssO.  6638 
Constant  log. 7. 2730 

Correction  of  altitude     .     .     1  ?43'43r         Prop.  log.  =  ,  •     .    0. 2394 
True  alt.  at  time  of  observ.      4 .  26. 38 


Sun's  meridional  altitude     .     6?10'2K;  which  differs   but  six  seconds 
from  the  truth. 

After  this  manner  may  the  meridional  altitude  of  the  moon,  a  planet,  or 
a  fixed  star  be  obtained,  when  the  declination  does  not  exceed  the  limits  of 
the  Table. 

Remarks,  ifc^ 

From  the  above  examples  it  is  manifest,  that  by  means  of  the  present 
Tables  the  meridional  altitude  of  a  celestial  object  may  be  readily  inferred 


♦  Corr.tolat.50Oanddec.0o- l"38'^^8 

Diff.  to  2"*  lat.  =  6"'.8;now.6«/.8x76'+120'    »     -    4    .3 
Diff.  to  l**  dec.  «  .1'".  5  ;  now,  1'".  5  x  44'+60'    =     +     1.1 


Corr.toUt.50o32'anddec,0<'43'51''»   .....  r35'''.6 

t  Corr.  to  lat;  60O  and  dec.  23**   « 0*54'".  6 

Diff.  to  2o  lat.  »  3'".  5  ;  now  3'".5  x  22'+120'  «   .    .  -0.6 

Diff.tol«dcc.-0"^5JnowO'".5x28'-^  60^=  .    .  -   0   .2 


Corr.  to  lat  60<»22' and  dec.  23*^7'45''  a      ....    0"53'".8 

/Google 


Digitized  by ' 


142  rasCAIFTtON  AND  USB  OF  THB  TABLB9. 

from  its  true  altitude  oI)8enred  at  a  knowa  interval  from  noon  (within  the 
limits  before  prescribed),  with  all  the  accuracy  to  be  desired  in  nautical 
operations  ;  and  that  it  is  immaterial  whether  the  observation  is  made  before 
or  after  noon,  of  time  of  tjrai\sit,  provided  the  tiine  be  but  correctly  known; 
and,  since  most  sea-going  ships  are  furnished  with  chronometers,  there  can 
be  but  very  little  diiRculty  in  ascertaining  the  apparent  time  to  within  a 
few  seconds  of  the  truth. 

It  is  to  be  observed,  however,  that  the  nearer  to  noon  or  time  of  transit 
the  observation  is  made,  the  less  susceptible  will  it  be  of  being  affeeted  by 
any  error  in  the  time  indicated  by  the  watch  :  thus,  in  example  4,  where  the 
interval  or  time  from  noon  is  1?  23?50?,  an  error  of  one  minute  in  that 
interval  would  produce  an  error  of  2^  minutes  in  the  sun^s  meridional 
altitude;  but  if  the  observation  had  been  made  within  a  quarter  of  an  hoiir 
of  noon,  an  error  offive  minuiee  in  the  time  would  scarcely  affect  the  meri- 
dional altitude  to  the  value  of  2  minutes :  hence  it  is  evident,  that  although 
the  observation  may  be  safely  made  at  any  time  from  noon  to  the  fiili  extent 
of  the  interval,  when  dependance  can  be  placed  on  the  time  shown  by  tho 
watch,  yet  when  there  is  any  reason  to  doubt  the  truth  of  that  time,  it  wiH 
be  advisable  to  take  the  altitude  as  near  to  noon,  or  the  time  of  transit,  as 
circumstances  may  render  convenient. 

In  all  narrow  seas  trending  in  an  easterly  or  westerly  direction,  where 
the  meridional  altitude  of  a  celestial  object  is  of  the  greatest  consideration, 
such  as  in  the  British  Channel,  the  mariner  vrill  do  well  to  avail  himself  of 
this  certain  method  for  its  actual  determination ;  particularly  during  th» 
winter  months,  when  the  sun  is  so  very  frequently  obscured  by  clouds  at  the 
time  of  its  coming  to  the  meridian. 

These  Tables  were  computed  by  the  following  rule;  viz., 

To  the  constant  log.  0. 978604,*  add  the  log.  co-sines  of  the  latitude  and 

the  declination ;  the  sum,  rejecting  20  from  the  index,  will  be  the  log.  of  a 

natural  number,  which,  being  subtracted  from  the  natural  co«sine  of  the 

difference  between  the  latitude  and  the  declination,  when  they  are  of  the 

same  name,  or  from  that  of  their  sum  if  of  contrary  names,  will  leave  the 

natural  co-sine  of  an  arch ;  now,  the  difference  between  this  arch,  and  the 

•difference  or  sum  of  the  latitude  and  the  declination,  according  as  they  are 

of  the  same  or  of  contrary  names,  will  be  the  change  ofaltitude  in  one 

minute  from  noon. 

Example  1. 

Let  the  latitude  be  18  degrees,  and  the  declination  of  a  celestial  object  2 
degrees,  both  of  the  same  name;  required  the  variation  or  change  ofaltitude 
in  one  minute  from  noon  ? 

*  This  it  the  log.  versed  sine,  or  loff.  rblDf,  of  one  miaute  of  tine. 

Digitized  by  VjOOQ IC 


BXSGBIPTION  AND  USB  OF  TUB  TABLBI.  148 

Constant  log.  = , .    0.978604 

Latitude  =:     •     13  degrees.  Log.  co-sine  •    9. 988724 

Dfcltnatioii  =s      2  degrees.  Log.  co-«s{ne  •    9. 999735 

Difference  zs  .     11  degrees.  Nat.  co-sine=981627 

Nat^umberc  9. 269:=:Log«  0. 967063 


Arch     =     .    .  11?  0:i0r  =:Nat,co.s.=:981617.731 


Difference  =!    •    0?  0^  \0f ;  which^  therefore^  is  the  change  of  altitude  in 
one  minute  from  noon. 


Let  the  latitude  be  40  degrees^  and  the  declination  of  a  celestial  object 
8  degrees/  of  a  contrary  name  to  that  of  the  latitude  j  required  the  Tariation 
or  change  of  altitude  in  one  minute  from  noon  ? 

Ckmstant  log.  =: 0.978604 

Latitude    =     40  degrees.  Log.  co<^ne   .    9.884254 

Declination  =:     8  degrees.  Log.  co-sine  .    9. 995753 

Sum  =:     •    •    48  degrees.  Nat  c6-sine=:6691Sl 

Nat.  num.  =  7. 221  Log.=0. 85861 1 

Arch  =3    •    .    48?  0^  2r=rNat.co-sine=669123.779 


Di&renoe  =:      0?  0'  2? ;  which^  therefore^  is  the  change  of  altitude  in 
one  minute  from  noout 

It  is  to  be  observed^  however^  that,  with  the  view  of  introducing  every 
possible  degree  of  accuracy  into  the  present  Tables,  the  natural  and  log. 
co-sinesy  &c.,  employed  in  their  construction,  have  had  their  orcspective 
numbers  extended  to  seven  places  of  decimals. 

Nofe.«^The  difference  between  the  meridional  altitude  of  a  celestial 
object  and  its  altitude  at  a  given  interval. from  noon,  is  found,  by  actual 
observation,  to  be  very  nearly  proportional  to  the  square  of  that  interval, 
under  certain  limitations,  as  pointed  out  in  page  138 ;  and  hence  the  rule, 
in  page  139^  for  computing  the  meridional  altitude  of  a  celestial  object. 


Digitized  by 


Google 


144  description  akd  use  of  thb  tables. 

Table  LIIF, 

7%6  Miles  and  Parts  of  a  Mile  in  a  Degree  of  Longitude  at  every 
Degree  of  Latitude. 

This  Table  consists  of  seven  compartments :  the  first  column  in  each 
compartment  contains  the  degrees  of  latitude,  and  the  second  column  the 
miles  and  parts  of  a  mile  in  a  degree  of  longitude  corresponding  thereto. 
In  taking  out  the  numbers  from  this  Table,  proportion  is  to  be  made,  as 
usual,  for  the  minutes  of  latitude ;  this  proportion  is  subtractive  from  the 
miles,  &c.,  answering  to  the  given  degree  of  latitude. 

Example* 

Required  the  number  of  miles  contained  in  a  degree  of  longitude  in 
latitude  37 ?48'.  ? 

Miles  in  a  degree  of  longitude,  in  latitude  37  degrees  s     .    •    •    47*  92 
Difference  to  1  degree  of  latitude  =  .  64 ;  now  ^^.  r^  =       —       -51 


Miles  in  a  degree  of  long,  in  latitude  37  degs.  48  min.,  as  required^s  47. 41 

Remarks* — Since  the  difference  of  longitude  between  two  places  on  the 
earth  is  measured  by  an  arch  of  the  equator  intercepted  between  the  meri- 
dians of  those  places ;  and  since  the  meridians  gradually  approach  each 
other  from  the  equator  to  the  poles,  where  they  meet,  it  hence  follows  that 
the  number  of  miles  contained  in  a  degree  of  longitude  will  decrease  in 
proportion  to  the  increase  of  the  latitude ;  the  ratio  of  decrease  being  as 
radius  to  the  co-sine  of  the  latitude.  Now,  since  a  degree  of  longitude  at 
the  equator  contains  60  miles,  we  have  the  following  rule  for  computing 
the  present  Table ;  viz.. 

As  radius  is  to  the  co-sine  of  the  latitude  of  any  given  parallel,  so  is  the 
measure  of  a  degree  of  longitude  at  the  equator  to  the  measure  of  a  degree 
in  the  given  parallel  of  latitude. 

Example. 

Required  the  number  of  miles  contained  in  a  degree  of  longitude  in  the 
parallel  of  latitude  37  degrees  ? 

As  radius    .     .    90  degrees        Log.  sine  =  .    .     10.000000 

.        Is  to  latitude  =    37  degrees         Log.  co*sine  .     .       9. 902349 

Sow       .    .    .60  miles  Log.  =     .     .    .       1.778151 


To      ....    47. 92  miles     Log.  =     .    .    .       1.680500; 

Hence  the  measure  of  a  degree  of  longitude  in  the  given  parallel  of  lati- 
tude, is  47. 92  miles. 


Digitized  by 


Google 


DSSCRIPTION  AND  USB  OF  THB  TABLES*  l45 


Table  LIV- 

Proportional  Miles  for  constructing  Marine  or  Sea  Charts. 

In  this  Table  the  parallels  of  latitude  are  ranged  in  the  upper  horizontal 
column,  beginning  at  0?,  and  numbered  10?,  20?,  30?,  &c.,  to  89? ;  the 
horizontal  column  immedi^ltely  under  the  parallels  of  latitude  contains  the 
number  of  miles  of  longitude  corresponding  to  each  parallel's  distance 
from  the  equator ;  under  which,  in  the  horizontal  column  marked  "  Differ- 
ence of  the  Parallels,  &c.,"  stands  the  number  of  miles  of  longitude  con- 
tained between  the  parallel  under -which  it  is  placed  and  that  immediately 
preceding  it. 

The  left-hand  vertical  column  contains  the  intermediate  or  odd  degrees 
of  latitude,  from  0?  to  10?  ;  opposite  to  which,  and  under  the  respective 
parallels  of  latitude,  will  be  found  the  number  of  miles  of  longitude  corre- 
sponding to  each  degree  of  latitude  in  those  parallels :  these  are  intended 
to  facilitate,  and  render  more  accurate,  the  subdivision  of  the  different 
parallels  of  latitude  into  degrees  and  minutes. 

To  rnake  a  Chart  of  the.  World,  in  which  the  Parallels  of  Latitude  and 
Longitude  are  to  consist  of  10  Degrees  each. 

Draw  a  straight,  or  meridian,  line  along  the  right  hand,  or  east  margin  x>f 
the  paper  intended  to  receive  the  projection;  bisect  that  line,  and  from  the 
point  of  bisection  draw  a  straight  line  perpendicular  to  the  former,  which 
continue  to  the  left-hand  or  west  margin  of  the  paper,,  and  it  will  represent 
the  equator. 

Fi'om  any  diagonal  scale  of  convenient  size  take  600  miles  in  the  com^ 
passes  (the  number  of  miles  of  the  equator  contained  in  10  degrees  of 
longiitude),  and  lay  it  off  from  the  point  of  bisection  along  the  equator^ 
and  it  will  graduate  it  into  36  equal  parts  of  10  degrees  each;  through 
which  let  straight  lines  be  drawn  at  right  angles  to  the  equator,  and  parallel 
to  that  drawn  along  the  right-ht^d  margin,  and  they  will  represent  the 
meridians  or  parallels  of  longitude.  Take,  from  the  same  scale^  60  miles 
in  the  compasses,  and  it  will  subdivide  each  of  those  36  divisions,  or  paral- 
lels of  longitude,  into  ten  equal  parts  consisting  of  one  degree  each ;  and 
then  will  the  equator  be  divided  into  360  degrees  of  60  miles  each. 

On  the  meridian  lines  drawn  along  the  right  and  left-hand  margins  of 
the  paper,  let  the  parallels  of  latitude  be  laid  down,  as  thus : — ^For  the  first 
parallel,  or  10  dc^ees  from  the  equator,  take  603. 1  miles  in  the  compasses 
(found  in  the  horizontal  column  immediately  under  the  parallels  of  latitude^ 
and  marked^*  Ditto  in  miles  of  the  Equator^  &c/')j  place  one  foot  on  the 

L 


Digitized  by 


Google 


146  BBSCRIPTION  AND   USB  OF  THB  TABLBS. 

equator,  and  where  the  other  falla  upon  the  right  and  left-hand  marginal 
lines,  when  turned  northward  and  southward,  there  make  points ;  through 
which  let  straight  lines  be  drawn  parallel  to  the  equator,  and  they  will 
represent  the  parallels  of  latitude  at  10  degrees  north  and  south  of  the 
equator:  in  the  same  manner,  for  20  degrees,  lay  oflF  1225. 1  miles;  for 
30  degrees^  1888. 4  miles ;  for  40  degrees,  2622. 6  miles,  and  so  on. 

But  since  the  common  compasses  are  generally  too  small  for  taking  off 
such  high  numbers^  it  will  be  found  more  convenient  to  lay  down  the  paral- 
lels of  latitude  by  the  numbers  contained  in  the  third  horizontal  column, 
or  that  marked  *^  Difference  of  the  Parallels,  &c.''  Thus,  for  10  degrees, 
take  603. 1  miles  in  the  compasses ;  place  one  foot  on  the  equator^  and 
with  the  otlier  make  points  north  and  south  thereof  on  the  east  and  west 
marginal  lines,  through  which  let  straight  lines  be  drawn,  and  they  will 
represent  the  parallels  of  latitude  at  10  degrees  north  and  south  of  the 
equator.  From  these  parallels  respectively,  lay  off  622, 0  miles,  by  pUcing 
one  foot  of  the  compasses  on  the  respective  parallels  and  the  other  on  the 
east  and  west  marginal  lines ;  through  the  points  thus  made  by  the  com* 
passes  draw  straight  lines,  and  they  will  represent  the  parallek  of  latitude- 
at  20  degrees  north  and  south  of  the  equator.  From  the  parallels,  thus 
obtidned,  lay  off  663. 3  miles,  and  the  parallel  of  30  degrees  will  be  deter- 
mined: thence  lay  off  734. 2  miles,  and  it  will  show  the  parallel  of  40 
degrees ;  and  so  on  for  the  succeeding  parallels. 

The  numbers  for  subdividing  those  parallek  will  be  found  in  the  vertical 
columns  under  each  respectively,  and  are  to  be  i4)plied  as  follows;  thus,  to 
graduate  the  parallel  betwcfen  50  and  60  degrees  ;  take  94. 3  miles  in  the 
compasses,  and  lay  it  off  from  50  degrees  towards  60  degrees,  and  it  will 
give  the  parallel  of  5 1  degrees ;  from  which  lay  off  96. 4  miles,  and  it  will 
show  the  parallel  of  52  degrees;  from  this  lay  off  98.6  miles,  and  the 
parallel  of  53  degrees  will  be  obtained ;  and  so  on  of  the  rest.  In  the 
same  manner  let  tiie  other  parallels  of  latitude  be  subdivided ;  then  let  the 
parallels  of  latitude  be  numbered  along  the  east  and  west  marginal  columns^ 
from  the  equator  towards  the  poles,  according  to  the  number  of  degrees 
contained  in  that  arc  of  the  meridian  which  is  intercepted  between  th.em 
and  the  equator^  as  10?,  20?,  30?,  40?,  &c.  &c.;  and  let  the  parallels 
of  longitude  be  numbered  at  the  top  and  bottom,  and  also  along  the 
equator ;  these  are  to  be  reckoned  east  and  west  of  the  first  meridian^  as 
10?,  20?,  30?,  40?,  &c.,  to  180?,  both  ways ;  and  since  the  first  ineridian  is 
entirely  arbitrary,  it  may  be  assumed  as  passing  through  any.  particular 
place  on  the  earth,  such  as  Greenwich  Observatory :  then  will  the  chart  be 
ready  for  receiving  the  latitudes  and  longitudes  oiF  all  the4>rincipat  places 
on  the  earthy  and  which  are  to  be  placed  thereon  by  the  following  rule  i 
via.. 
Lay  a  ruler  Qver  the  giv^a  loogitud^  found  at  the  top  wd  botloia  of  the 


Digitized  by 


Google 


BSSCRIPTION   ANB  USB  OF  TBB  TABLBS.  147 

chart,  and  with  a  pair  of  compasses  teke  the  latitude  from  the  east  or  west 
marginal  columns ;  which  being  applied  to  the  edge  of  the  ruler,  placing 
one  foot  on  tlie  equator  or  on  the  parallel  that  the  latitude  was  counted 
fiom,  the  other  foot  turned  north  or  south  according  to  the  name  of  the 
latitude,  will  point  out  or  fall  upon  the  true  position  of  the  given  latitude 
and  longitude. 

From  what  has  been  thus  laid  down,  the  maimer  of  constructing  a  chart 
for  any  particular  place  or  coast  must  appear  obvious, 

iVp/e.— Since  this  Table  is  merely  an  extract  from  the  Table  of  Meridi- 
onal parts,  the  reader  is  referred  to  page  1 13  for  the  method  of  computing 
the  different  numbers  contained  therein. 


Tablb.lv. 

Tb  Jind  the  Distance  of  Terrestrial  Ohfects  at  Sea. 

If  an  observer  be  elevated  to  any  height  above  the  level  of  the  earth  or 
sea,  he  can  not  only  discern  the  distant  surrounding  objects  much  plainer 
than  he  could  when  standing  on  its  surface,  but  also  discover  objects 
which  are  still  more  remote  by  increasing  his  elevation.  Now,  although 
the  great  irregularity  of  the  surface  of  the  land  cannot  be  subjected  to  any 
definite  rule  for  determining  the  distance  at  which  objects  may  be  seen 
from  different  elevations;  yet,  at  sea,  where  there  is  generally  an  uniform 
curvature  of  the  water,  on  account  of  the  spherical  figure  of  the  earth,  the 
distance  at  which  o|pjects  may  be  seen  on  its  sur&ce  may  be  readily 
obtained  by  means  of  die  present  Table ;  in  which  the  distance  answering 
to  the  height  of  the  eye,  or  to  that  of  a  given  remote  object,  is  expressed 
in  nautical  miles  and  hundredth  parts  of  a  mile ;  allowance  having  been 
niade  for  terrestrial  refraction,  in  the  ratio  of  the  one-twelfth  of  the  inter- 
cepted arch. 

No^e.— The  distance  between  two  .objects  whose  heights  are  given,  is 
found  by  pdding  together  the  tabular  distances  corresponding  to  those 
heights.  And,  when  the  given  height  exceeds  the  limits  of  the  Table,  an 
aliquot  part  thereof  is  to  be  taken  ;  as  one  fourth,  one  ninth,  or  one  six- 
teenth, &c.;  then,  the  distance  corresponding  thereto  in  the  Table,  being 
multiplied  by  the  square  root  of  such  aliquot  part,  viz«,  by  2,  3,  or  4,  &c., 
according  as  it  may  be,  will  give  the  required  distance. 

l2 

Digitized  by  VjOOQ IC 


148  DESCRIPTION  AND  ^SE  OF  THE  TABLES. 

Example  I. 

The  look-out  man  at  the  mast-head  of  a  man-of-war,  at  an  elevation  of 
160  feet  above  the  level  of  the  sea,  saw  the  top  of  a  light-house  in  the 
horizon  whose  height  was  known  to  be  290  feet;  required  the  ship's  dis- 
tance therefrom  ? 

The  distance  answering  to  160  feet  is     .     .     14., 57  miles. 
Ditto     .  to'290  feet  is      .    •     19. 62  do. 


Required  distance  = 34. 19 miles; 

which,  therefore,  is  the  ship's  distance  from  the  light-house. 

Example  2. 

The  Peak  of  Teneriffe  is  about  15300  feet  above  the  level  of  the  sea;  at 
what  distance  can  it  be  seen  by  an  observer  at  the  mast-head  of  a  ship, 
supposing  his  eye  to  be  170  feet  above  the  level  of  the  water  ? 

One  ninth  of  15300  is  1700,  answering  to.which  is  47*50  miles;  this 
being  multiplied  by  3  (the  square  root  of  one  ninth)  gives  142. 50  miles. 
Distance  ans.  to  170  feet  (height  of  the  eye)  is     .       15.03  do. 

Required  distance  = 157*  53  miles. 

JRetnark  1. — Since  the  distances  given  in  this  Table  are  expressed  in 
nautical  miles,  whereof  60  are  contained  in  one  degree,  and  there  being 
69. 1  English  miles  in  the  same  portion  of  the  sphere  ;  if,  therefore,  the 
distance  be  required  in  English  miles,  it  is  to  be  found  as  follows ;  viz.. 

As  60,  is  to  69.1;  so  is  the  tabular  distance  to  the  corresponding  distance 
in  English  miles ;  which  may  be  reduced  to  a  logarithmic  expression^  aa 
thus ; — 

To  the  log.  of  the  given  tabular  distance,  add  the  constant  logarithm 
0. 061327,*  and  the  sum  will  be  the  log.  of  the  given  distance  in  English 
miles. 

Example. 

Let  it  be  required  to  reduce  157. 53  nautical  miles  into  English  miles  ? 

Given  distance  in  nautical  miles  =  157*53,  log.  =    2. 197364 
Constant  log ,...,...     0.061327 

Distance  reduced  to  English  miles  181.42  =  Log.  s  2.258691 

•  Thelog.of69.1  «  1.839478,  less  the  log. of  60  »  1. 778 IM  is  0.061327;  which,  there- 
fore, is  the  constant  logarithm. 


Digitized  by 


Google 


DBSCRIPTION  AND   V8E  OF  THE  TABLES.  149 

The  converse  of  this  (that  is^  to  reduce  English  miles  into  nautical  miles^) 
must  appear  obvious. 

Remark  2. — ^This  Table  was  computed  by  the  following  rule  3  viz., 

To  the  earth's  diameter  in  feet,  add  the  height  of  the  eye  above  the 
level  of  the  sea,  and  multiply  the  sum  by  that  height;  then,  the  square  root 
of  the  product  being  divided  by  6080  (the  number  of  feet  in  a  nautical 
mile),  will  give  the  distance  at  which  an  object  may  be  seen  in  the  visible 
horizon,  independent  of  terrestrial  refraction.  This  rule  may  be  adapted 
to  logarithms,  as  Uius : — 

Let  the  earth's  diameter  in  feet  be  augmented  by  the  height  of  the  eye  ; 
then,  to  the  log.  thereof  add  the  log.  of  the  height  of  the  eye ;  from  half 
the  sum  of  these  two  logs,  subtract  the  constant  log.  3.  783904,*  and  the 
remainder  will  be  the  log.  of  the  distance  in  nautical  miles,  which  is  to  be 
increased  by  a  twelfth  part,  of  itself,  on  account  of  the  terrestrial  refrac- 
tion. 

Example. 

At  what  distance  can  an  object  be  seen,  in  the  visible  horizon,  by  an 
observer  whose  eye  is  elevated  290  feet  above  the  level  of  the  sea  ? 

Diameter  of  the  earth  in  feet  =  41804400 

Height  of  the  eye    .....  290    Log.=     2.462898 


Sum= 41804690    Log.  =     7.621225 

Sum    .  10.083623 


Halfsum5.0418ni 
Constant  log.  =      .    . 3.783904 

Distance  uncorrected  by  refraction    1 8. 1 1  =Log.  =:     1 .  257907J 
Add  one*  1 2th  part  on  ace.  of  refr^c.  1.51 

Distance  as  required  ss     •    .    •     19. 62  nautical  miles. 

^o^^.— rFor  the  principles  of  this  rule,  see  how  the  distance  of  the  visible 
horizon,  expressed  by  the  line  O  T,  i&..determined  in  page  5. 


*  This  IS  the  lop  of  6080,  the  number  of  feetiu  a  nautical  mile. 


Digitized  by 


Google 


ISO  2IB9CRlFT|eN  AND  V9E  OF  THB  TABUI8« 

Table  LVI. 

To  reduce  the  French  Centesimal  Division  of  the  Circle  info  the  EtigUsh 
Sexagesimal  Division;  or,  to  reduce  French  Degrees,  Ifc,  into  English 
Degrees,  jfc,  and  conversely. 

This  Table  is  intended  to  facilitate  the  reduction  of  French  degrees  of 
the  circle  into  English  degrees,  and  conversely.  The  TVble  is  divided  into 
two  parts :  the  first  or  upper  part  exhibits  the  number  of  English  degrees 
and  parts  of  a  degree  contained  in  any  given  number  of  French  degrees 
and  parts  of  a  degree  ;  and  the  second  or  lower  part  exhibits  the  number 
of  French  degrees,  &c.,  contained  in  any  given  number  of  English 
degrees,  &c. 

Note, — In  the  general  use  of  this  Table,  when  any  given  number  of 
French  degrees  exceeds  the  limits  of  the  Arst  part^  take  out  for  100  degrees 
first,  and  then  for  as  many  more  as  will  make  up  the  given  number;. and, 
when  any  given  number  of  English  degrees  exceeds  tlie  limits  of  the  second 
part,  take  out  for  90  degrees  first,  and  then  for  as  many  more  as  will  make  ' 
up  the  given  number. 

Ejtample  1. 

If  the  distance  between  the  moon  and  a  fixed  star^  aeoording  to  the 
French  division  of  the  circle,  be  128?93'9.6'/,  required  the  distance  agree- 
ably to  the  English  division  of  the  circle  ? 

100  French  degrees  are  equal  to  .     .  90?  0'  01    English. 

28          Ditto           are  equal  to  ..  25.  12.   0          do. 

93  French  miniates  are  equal  to  .    .      0. 50. 13  .  20  do. 

96  French  seconds  are  equal  to  .    •      0.   0.31  .10  do. 

Distance  reduced  to  English  degs.,  as  required  116*?  9^4',  30 

Example  2. 

If  the  distance  between  the  moon  und  sun,  according  to  the  English 
division  of  the  circle,  be  116?53'47'',  required  the  distance  agreeably  to 
the  French  division  of  the  circle  ? 

90  English  degrees  are  equal  to  .  .  100?  0'  Or  French. 

26         Ditto             are  equal  to  .  .  28. 88. 88   .  89  do. 

53  English  minutes  are  equal  to  .  •  0. 98. 14   .  81  do. 

47  English  seconds  are  equal  to  .  .  0.    1.45   .06  do. 

Distance  reduced  to  French  degs,  as  required  =  129?88C48''.  76 

Digitized  by  VjOOQ IC 


DBSCRlpnON  AND  USB  OP  TUB  TABLBS,  151 

Remark  1. — ^ITiis  Table  was  computed  in  conformity  with  the  following 
considerations  and  principles ;  viz.^ 

The  French  writers  on  trigonometry  have  recently  adopted  the  cente- 
simal division  of  the  circle^  as  originally  proposed  by  our  excellent  coun- 
tryman Mr.  Henry  Briggs,  about  the  year  1600.  In  this  division,  the 
circle  is  divided  into  400  equal  parts  or  degrees,  and  the  quadrant  into  100 
equal  parts  or  degrees;  each  degree  being  divided  into  100  equal  parts  or 
minutes,  and  each  minute  into  100  equal  parts  or  seconds :  these  degrees, 
&c.  &c.,  are  written  in  the  usual  maimer  and  with  the  customary  signs,  as 
thu8  5l28?93:96^ 

Hence,  the  French  degree  is  evidently  less  than  the  English,  in  the  ratio 
of  lUO  to  90 ;  a  French  minute  is  less  than  an  English  minute,  in  the  ratio 
of  100?  X  lOOUo  90?  X  60  f  5  and  a  French  second  is  less  than  an  Eng- 
lish second,  in  the  ratio  of  100?  X  100'.  x  lOOr  to  90?  x  60^  x  60'/ : 
now,  the  converse  of  this  being  obvious,  we  have  the  following  general  rule 
for  converting  French  degrees  into  English,  and  the  contrary. 

As  100,  the  number  of  degrees  in  the  French  quadrant,  is  to  90,  the 
number  of  degrees  in  the  English  quadrant ;  so  is  any  given  number  of 
French  degrees  to  the  corresponding  number  of  English  degrees. 

As  10000,  the  number  of  minutes  in  the  French  quadrant,  is  to  5400, 
the  number  of  minutes  in  the  English  quadrant;  so  is  any  given  number  of 
French  minutes  to  the  corresponding  number  of  English  minutes.    And, 

As  1000000,  the  number  of  seconds  in  the  French  quadrant,  is  to  324000, 
the  number  of  seconds  in  the  English  quadrant;  so  is  any  given  number  of 
French  seconds  to  the  corresponding  number  of  English  seconds. 

English  degrees,  minutes,  and  seconds^  are  reduced  into  French  by  a 
cronverse  proportion;  viz.^ 

As  90,  is  to  100 ;  so  is  any  ^ven  number  of  English  degrees  to  the  corre- 
sponding number  of  French  degrees. 

As  5400,  is  to  10000 ;  so  is  any  given  number  of  English  minutes  to  the 
corresponding  number  of  French  minutes.    And^ 

As  324000,  is  to  1000000  >  so  is  any  given  number  of  English  seconds  to 
the  corresponding  number  of  French  seconds. 

Remark  2. — ^French  degrees  and  parts  of  a  degree  may  be  turned  into 
English,  independently  of  the  Table^  by  the  following  rule ;  viz.. 

Let  the  French  degrees  be  esteemed  as  a  whole  number,  to  which  annex 
the  minutes  and  seconds  as  decimals ;  then  one-tenth  of  this  mixed  num- 
ber^ deducted  from  itself,  will  give  the  corresponding  English  degrees^  &€. 


Digitized  by 


Google 


152  BB8CBIPTI0N  AND  USB  OF  THE  TABUS, 

•  Example. 

The  latitude  of  Paris^  acc6rding  to  the  French  division  of  the  quadrant, 
is  54?26'36^  north;  required  the  latitude  agreeably  to  the  English  divi- 
sion of  the  quadrant  ? 

Given  latitude  =  54?26:36r=54^  2636 


Deduct  one-tenth 

.    .    5  .42636 

English  degrees,  &c.    » 

^  .48».  83724 
60 

•50'.  23440 
60 

14'.  06400 

Hence,  die  latitude  of  Paris,  reduced  to  the  English  division  of  the  qua- 
drant, is  48?50: 14r  north. 

Remark  3. — English  degrees  and  parts  of  a  degree  may  be  turned  into 
French,  independently  of  the  Table,  as  thus  : — 

Reduce  the  English  minutes  and  seconds  to  the  decimal  of  a  degree,  and 
annex  it  to  the  given  degrees ;  then  one-ninth  of  this  mixed  number,  being 
added  to  itself,  will  give  the  corresponding  French  degrees,  &c. 

Example. 

The  latitude  of  the  Royal  Observatory  at  Greenwich  is  51°28M0r 
north,  agreeably  to  the  English  division  of  the  quadrant ;  required  the 
latitude  according  to  the  French  division  of  the  quadrant  ? 

Given  latitude=  51?28U0r  =  51°.  4777777,  &c. 
Add  one -ninth 5  .7197530,  &c. 

French  degrees,  &c 57^  1975307  =  57?  19'.75''.  307 

Hence,  the  latitude  of  Greenwich  Observatory,  according  to  the  French 
division  of  the  quadrant,  is  57?  19'75''.  307  N.  ' 


Tabus  LVII. 

A  general  Table  for  Gatigwg,  or  finding  the  Content  of  all  Orcular- 

headed  Casks. 

Although  this  Table  may  not  directly  affect  the  interest  of  the  mariner; 
yet,  since  it  cannot  fail  of  being  exceedingly  useful  to  officers  in  charge  of 


Digitized  by 


Google 


DBSCAIFTION  AKD   USB  OF  THB  TABLES.  153 

His  Majesty's  victualling  stores  (such  as  Pursers  of  the  Royal  Navy^  Lieu- 
tenants commanding  gun-brigs^  &c.  &c.),  it  has  therefore  been  deemed 
advisable  to  give  it  a  place  in  this  work^  particularly  since  it  may  be  found 
interesting  to  those  whom  it  immediately  concerns. 

This  Table  is  divided  into  two  parts :  the  first  part  consists  of  five  com- 
partments^ and  each 'compartment  of  three  columns;  the  first  of  which 
contains  the  quotient  of  the  head  diameter  of  a  cask  divided  by  the  bung 
diameter  ;  the  second  the  corresponding  log.  adapted  to  ale  gallons ;  and 
the  third  the  log.  for  wine  gallons.  The  second  part  of  the  Table  contains 
the  bung  diameter  and  its  corresponding  Ic^rithm.  - 

The  use  of  this  Table  will  be  exemplified  in  the  following 

Problem. 

Given  the  Ditnensums  of  a  Cask,  to  find  its  Contents  in  Ale  and  Wine 

GaUons. 

Rule. 

Divide  the  head  diameter  by  the  bung  diameter  to  two  places  of  deci- 
mals in  the  quotient ;  then  add  together  the  log.  for  ale  or  wine  gallons, 
corresponding  to  this  quotient,  in  the  first  part  of  the  Table ;  the  log.  cor- 
responding to  the  bung  diameter,  in  the  second  part  of  the  Table,  and  the 
common  log.  of  the  length  of  the  cask ;  the  sum  of  these  three  logs.,  reject- 
ing 10  in  the  index,  will  be  the  log.  of  the  true  content  of  the  cask,  in  ale 
or  wine  gallons,  according  as  the  content  may  be  required. 

Example. 

Let  the  bung  diameter  of  a  cask  be  25  inches,  the  head  diameter  19. 5 
inches,  audits  length  31  inches;  required  the  contents  in  ale  and  wine 
gallons  ? 

25)  19. 50(.  78,  quotient  of  the  head  diameter  divided  by  the  bung 
175  diameter. 

200 
200 


.  78  ■=  quotient,  log.  for  ale  gallons  =  •  .  .  7. 862671 
25  inches,  bung  diameter,  corresponding  log.  =  2.  795880 
31  inches,  length  of  the  cask,  common  log.  =       1. 491362 

Content  in  ale  gallons  ^  44. 66  common  log,  ss  1. 649913 

Digitized  by  VjOOQ IC 


154  mUCRIPTION  AND  USB  OP  THS  TA8LB8. 

•  78  ts  quotient)  log.  for  wine  gallons  as  •  •  7*  449340 
25  inches,  bung  diameter,  corresponding  log.  ss  2. 795880 
Si  inches,  length  of  the  caak,  common  log.  ss      1 .  49136% 

Content  in  wine  gallons  s=  54. 52  common  log.s  1. 736582 

jRtfmofJk.^— Should  the  bung  diameter  liot  come  within  the  limits  of  the 
second  part  of  the  Table ;  that  is,  should  it  be  under  10  or  above  50 
inches,  the^  twice  the  common  log.  corresponding  thereto  will  express  the 
Ictg,  of  the  said  bung  diameter,  with  which  proceed  as  before :  hefice,  the 
rule  becomes  universal  for  all  circular-headed  casks,  be  the  me  ever  so 
great  or  ever  so  trivial. 

This  subject  will  be  revived  in  a  subsequent  page  of  the  present  work. 


Tablb  LVIII. 

Latitudes  and  Longitudes  of  the  prindpal  Sea-Ports,  Islands,  Capes, 
^c.  ^.c,  with  the  Time  of  High  fVaier  at  the  Full  and  Change  of  the 
Moon  at  all  Places  where  it  is  known. 

In  drawing  up  this  Table,  the  greatest  pains  have  been  taken  to  render 
it  not  only  the  most  accurate,  but  also  the  most,  extensive  of  any  now 
extant*  Perfect  accuracy,  however,  is  not  to  be  expected  in  a  Table 
which  principally  depends  on  the  observations  made,  at  different  periods^ 
by  the  navigators  of  most  civilized  nations  j  because^  in  those  periods,  or 
at  the  time  when  a  very  considerable  portion  of  the  latitudes  and  lon^tudea 
were  established,  the  nautical  instruments  and  tables  employed  in  their 
determination  were  far  from  being  in  that  highly-improved  state  in  which 
they  are  found  at  present :  besides,  it  is  a  fact  well  known  to  the  generality 
of  nautical  persons,  that  if  two  or  more  navigators  be  directed  to  ascertain 
the  position  of  any  particular  place,  they  will,  in  most  ease^  difier  four  or 
five  miles  in  the  latitude,  and  perhaps  thrice  as  many  in  the  longitude. 

In  constructing  all  the  other  Tables  in  this  work,  there  were  fixed  data 
to  work  uponj  with  certain  means  of  detecting  and  exterminating  errors  ; 
but,  in  this,  tiiere  were  no  determinate  means  of  ensuring  the  desired 
degree  of  accuracy,  except  in  those  positions  where  chancn  or  profes- 
sional duties  happened,  from  time  to  time,  to  conduct  the  author. 
Hence,  although  every  possible  degree  of  attention  has  been  paid  in  con- 
sulting the  most  approved  works  of  the  present  day,  and  in  colUting  this 
with  the- best  modem  Tables;  yet  the  mariner  must  not  expect  to  find 
it  perfectly  free  from  blemishes ;  though,  doubtless,  he  will  find  it  con« 
siderably  less  sq  than  any  with  which  he  may  have  been  hitherto 
acquainted. 

Since  this- Table  is  aol  inteaded  for  general  geog^phkal  purposes,  the 


Digitized  by 


Google 


DESCRIPTION   AND  USE  OF  THE  TABLES.  155 

positions  of  places  inland,  which  do  not  concern  the  navigator,  have,  with 
one  or  two  exceptions,  been  ptirposely  omitted :  hence,  the  latitudes  and 
longitudes  are  limited  to  maritime  places.  These  are  so  arranged  as  to 
exhibit  to  the  mariner  the  whole  line  of  coast  along  which  he  may  chance 
to  sail,  or  on  which  he  may  be  employed,  agreeably  to  the  manner  in 
which  it  unfolds  to  his  view  on  a  Mercator's  chart.  This  mode  of  arrange- 
ment is  evidently  much  better  adapted  to  nautical  purposes  than  the 
alphabetical  mode. 

With  the  view  of  keeping  up  the-  identity  of  the  Table  with  the  line 
of  coast  laid  down  on  partiQular  charts,  a^/eip  positions  have  been  inserted 
a  second  time.  This,  it  Is  presumed,  if  not  conducive  to  good,  will  not, 
at  least,  be  productive  of  any  evil,  since  the  repetition  is  so  very  trivial  as 
not  to  embrace,  in  the  whole,  more  than  ten  or  twelve  positions. 

The  time  of  high  water,  at  the  full  and  change  of  the  moon,  is  given ' 
at  all  places  where  it  is  known.     This,  it  is  hoped,  will  be  found  not  a 
little  coDvenienti  since  it  does  away  with  the  necessity  of  consulting  a 
separate  Table  far  that  particular  purpose* 

In  order  to  render  this  Table. still  more  complete,  an  alphabetical 
reference  has  been  annexed,  which  will  very  essentially  contribute  towarcU 
assisting  the  mariner  in  readily  finding  outmost  of  th«  principal  coasts 
an4  islands  contiuned  in  that  Tablet 


The  page  which  immediately  follows  the  alphabetical  reference  to  Table 
LVIII.  contains' the  form  of  a  Transit  Table,  and  the  next  page  a  variety 
of  numbers  with  their  corresponding  logarithms,  &c.,  which  may,  perhaps, 
be  found  useful  on  many  occasions.  At  the  foot  of  these  numbers  there 
is  a  small  Table,  showing  the  absolute  time  at  which  the  hour  and  minute 
hands  of  a  well-regulated  watch  or  clock  should  exactly  be  in  conjunction, 
and  also  in  opposition,  in  every  revolution. 

Having  thus  completed  the  Description  and  Use  of  the  Tables  eontained 
in  this  work,  it  now  remains  to  show  their  application  to  the  different 
elements  connected  with  the  sciences  of  navigation  and  nautical  astronomy. 
In  doing  this,  since  the  author's  design  carries  him  no  farther  than  that 
of  giving  an  ample  iUustratkm  of  the  various  purposes  to  which  they 
may  be  apfdied}  the  reader  must  nbt,  therefore,  expect  to  find  the 
dementary  part  of  the  seiences  treated  of.  Hence,  in  this  part  of  the  work, 
tha  anthor  will  endeavour  to  confine  himself  to  such  Problems  and  subject 
matters  as  may  appear  to  he  most  interesting  and  useful  to  nautical  per* 
sons,  without  entering  into  particulars  or  the  minutiae  of  the  sciences, 
and  duis  sw€Uing  the  work  to  an  unnecessary  size^; — a  thing  which  he 
most  anxiously  wishes  to  avoid* 


Digitized  by 


Google 


156 


A  CONCISE  SYSTEM 


OF 


DECIMAL   ARITHMETIC. 


Although^  from  what  has  been  6aid  in  the  last  paragraph^  it  may  appear 
somewhat  irregular,  and  even  contrary  to  the  general  tenor  of  this  work, 
to  introduce  any  subject  therein  that  does  not  come  immediately  under  the 
cognizance  of  logarithms  ;  yet,  since  the  reader  may  be  desirous  of  having 
some  little  acquaintance  with  the  nature  of  decimal  fractions  previously  to 
his  entering  on  the  logarithmical  computations,  the  following  concise 
system  is  given  for  that  purpose.-*-It  haa  been  deemed  advisable  to 
touch  upon  this  subject  for  two  cogent  reasons  ;**first,  because  a  short  ac- 
count of  decimals  may  be  acceptable  to  the  mariner  whose  early  entrance 
on  a  sea  life  prevents  him  from  going  through  a  regidar  course  of  scholas- 
tic education  on  shore ;  and,  second,  that  he  may  have  directly  under  his 
view  all  tliat  is  essentially  necessary  to  be  known  in  the  practically  useful 
branches  of  science,  without  being  under  the  necessity  of  consulting  any 
other  author  for  the  purpose  of  assisting  him  in  the  comprehension  of  the 
different  subjects  contained  in  this  work. 


DECIMAL  FRACTIONS. 

A  decimal  fraction  signifies  the  artificial  manner  of  setting  down  and  ex- 
pressing natural  vulgar  fractions  as  if  they  were  whole  number8.--'A  decimal 
fraction  has  always  for  its  denominator  an  uait  (1,)  with  as  many  ciphers 
annexed  to  it  as  there  are  places  in  the  numerator ;  and  it  is  generally  ex- 
pressed by  setting  down  the  numerator  only,  with  a  point  before  it,  on  the 
left  hand  ;— thus,  ^%  is  .  5  ;  ^-Vir  is  .  75  j  -rgg^  is  •  025  ;  ^uVolny  w  .00114, 
&c.  &c.  :--hence  the  numerator  must  always  consist  of  as  many  figures  as 
there  are  ciphers  in  the  denominator. 


Digitized  by 


Google 


BBCIMAL  FRACTIONS. 


157 


A  mixed  number  is  made  up  of  a  whole  number  and  a  decimal  fraction, 
the  one  being  separated  from  the  other  by  a  point  i  thus  5. 75  is  the  same 

Ciphers  on  the  right  hand  of  decimals  do  not  increase  their  value;  for 
•  5 .  50 .  500  •  5000, 8ic,,  are  decimal  fractions  of  the  same  value,  each  being 
equal  to  Z^,  or  i; — But  when  ciphers  are  placed  on  the  left  hand  of  a  deci- 
mal they  decrease  its  value  in  a  tenfold  proportion ; — thus,  .  5  is  -^  or  5 
tenths;  but  .05  is  only  -j^  or  5  hundredths;  .005  is  only  -j-^jf  or  5 
.thousandths,  and  so  on  : — ^hence  it  is  evident  that  in  decimals  as  well  as 
in  whole  numbers,  the  value  of  the  place  of  the  figure  increases  towards 
the  left  hand,  and  decreases  towards  the  right,  each  being  in  the  same 
tenfold  proportion. 


ADDITION  OP  DECIMALS.    . 

Addition  of  decimals  is  performed  in  the  same  way  as  addition  of  whole 
numbers,  obsemng  to  place  the  numbers  right ;  that  is,  all  the  decimal 
points  under  each  other,  units  under  units,  tenths  under  tenths,  hundredths 
under  hundredths.  Sic, ;  taking  care  to  point  off  from  the  total  or  sum  as 
many  places  for  decimals  as  there  are  in .  the  line  containing  the  greatest 
number  of  decimal  places. 


Example  1. 

Add  together    41.37;  3.7625 
137.03;  409,  and  .3976. 

41.37 
3.762 
137-03 
409; 
.3976 


591.5596,  the  sum. 


Example  2. 


Add  together  3. 

268; 

208 

276  J 

4.7845,  and  1.07. 

3.268 

208.1 

276. 

•     4.7845 

1.07 

493. 2225,  the  sum. 


SUBTRACTION  OP  DECIMALS. 

# 

Subtraction  of  decimals  is  likewise  performed  the  same  way  as  in  whole 
numbers;  observing  to  place  the  numbers  right;  that  is,  the  decimal 
points  under  each  other,  units  under  units,  tenths  under  tenths,  hundredths 
under  hundredths,  &c.  &c. 


Digitized  by 


Google 


158 


DBCIMAL  FRACTIONf. 


Example  1. 

Pfom    •    •    .    .    489.7265 
Take    .     .    .    •      98.283 


Remains  • 


34 K 4435 


Example  2. 

Prom    -    .    .    .  179.087 
Take    ....     54.932468 


Remains   .    .    .  124.104532 


MULTIPLICATION  OF  DECIMALS. 

Multiplication  of  decimals  is  also  performed  the  same  way  as  in  whole 
numbers ;  observing  to  cut  off  as  many  decimal  places  in  the  product  as  there 
are  decimal  places  in  both  factors ;  that  is^  in  the  multiplicand  and  muU 
tiplier. 

Example  \,  Exampk2. 


MulUply  .    .    .    .    2.4362 
By.    ....    .      .275 


Product  ss 


121810 
170534 
48724 

0.6699550 


Multiply 
By     .    . 


Product  ss 


.    376.09 
.      13.48 

112827 
150436 
112827 
37609 

5050. 8887 


Note. — If  a  decimal  fraction  be  multiplied  by  a  decimal  fraction  the  pro- 
duct will  be  less  than  either  the  multiplicand  or  the  multiplier. — ^And  if  any 
number  eiAer  whole  or  mixed^  be  multiplied  by  a  decimal  fraction^  the  pro- 
duct will  be  always  less  than  the  multiplicand,  as  in  example  1 } — ^hence  if 
a  decimal  fraction  be  multiplied  by  itself,  its  value  will  decreaee  in  the  pro- 
portion of  its  multiple  :-^thus. 


Multiply ...         .25 
By      ....         .25 


Product 


125 
50 

.0625 


Multiply ...        .75 
By 75 


Product 


375 
525 

.5625 


DIVISION  OF  DECIMALS. 

Division  of  decimals  is  performed  in  the  same  manner  as  in  whole 
numbers }  observing  to  point  off  as  many  decimal  places  in  the  quo- 


Digitized  by 


Google 


DECIMAL  FRACTIONS. 


159 


tient  as  the  decimal  places  in  the  dividend  exceed  those  in  the  divisor  :-<- 
But  if  there  be  not  as  many  figures  in  the  quotient  as  there  are  in  that 
excess^  the  deficiency  must  be  supplied  by  prefixing  ciphers,  with  a 
point  before  them; — for  the  decimal  places  in  the  divisor  and  quotient 
taken  together^  must  be  always  equal  to  those  in  the  dividend.— When 
there  happens  to  be  a  remainder  after*  the  division  ;  or  when  the  decimal 
places  in  the  divisor  are  more  than  those  in  the  dividend^  then  ciphers 
may  be  annexed  to  the  latter^  and  the  quotient  carried  on  as  far  as  may  h€ 
necessary. 


Example  1. 
Divide  .6699550  by  .375 

Dividend.     Quotieni^ 

IKotfor. 275 \. 6699550  /2.4362 
;  550  V 


EjttunplB  2. 
Divide  5050.8887  by  13.43 

Div.  13. 43  \  5050. 8887  /376. 09 


;4029 

10218 
9401 


\ 


.8178 
8058 


.12087 
120S7 


Nofe.— -If  a  decimal  fraction  be  divided  by  a  decimal  firaction^  the  quo- 
tient will  be  greater  than  either  the  divisor  or  dividend^  as  in  Example  1. 
Andy  if  any  whole,  or  mixed  number  be  divided  by  a  decimal  fraction,  the 
quotient  will  be  greater  than  the  dividend ;  but  if  a  decimal  fraction  be 
divided  by  a  whole,  or  mixed  number,  the  quotient  will  be  less  than  the 
dividend, — ^If  a  decimal  fraction  be  divided  by  itself,  its  value  will  increase 
in  the  proportion  of  its  division,  or  of  the  decrease  of  the  parts  into  which 
the  decimal  is  divided ;  because,  in  this  case,,  the  quotient  will  be  a  natural 
number: — thus, . 25  divided  by  .25,  quotes  1.— -And,  .5625,  divided  by 
.  5625,  quotes  1  also.  Hence  it  is  manifest  that  the  dividing  of  a  decimal 
fraction  by  itsdf  increases  its  value. 


Digitized  by 


Google 


160 


DECIMAL   FRACTIONS. 


REDUCTION  OF  DECIMALS. 

Case  L 

To  reduce  a  Vulgar  Fraction  to  a  Decimal  Fraction  of  equal  value. 

RULB. 

Annex  a  cipher  or  ciphers  to  the  numerator ;  then  divide  by  the  denomi* 
nator,  as  in  whole  numbers,  and  the  quotient  will  be  the  required  decimal. 


Examples, 
Reduce  |  to  a  decimal  fraction. 

4)100 
Required  dec.  s  •  25 


Reduce  f  to  a  decimal  fraction. 

4)300 
Required  dec.=  .  75 


Examples, 
Reduce  i  to  a  decimal  (ra^tion. 

2)10 
Required  dec.  =  « 5 


Reduce  { to  a  decimal  ifiraction. 

8)5000 
Req.dec.  ==  ,  .625 


Casb  II. 

To  reduce  Numbers  of  different  Denominations^  such  as  Degrees^  Time, 
Cdny  Measure,  ^c.  into  Decimals. 
Rule. 
Reduce  the  given  degrees,  time,  coin,  measure^  &c.  into  the  lowest  de- 
nomination mentioned,  for  a  dividend,  aimex  ciphers  thereto,  and  then 
divide  by  the  integer,  reduced  also  into  the  lowest  denomination  mentioned  ; 
the  quotient  will  be  the  required  decimal  fraction. 


Examples. 

Reduce  30  minutes  to  the  deci- 
mal of  a  degree. 

The  given  number  being  in  the 
lowest  denomination  required,  an- 
nex a  cipher  and  divide  by  60,  the 
number  of  minutes  in  a  degree; 
the  quotient  will  be  the  required 
decimal ; — thus, 

60\  800  / .  5,  the  Answer. 
>/300V 


Examples. 

Reduce  49^30?  to  the  decimal  of 
a  degree. 

The  given  number  being  reduced 
to  the  lowest  denomination  men* 
tioned,  gives  2970'' ;  to  this  annex . 
ciphers,  and  divide  by  3600,  .the  se- 
conds in  a  degree ;  the  quotient  will 
be  the  required  decimal : — thus> 

3600\  2970. 000  / .  825,  Answer « 
; 28800       \ 
..9000 
7200 

18000 
18000    ' 


Digitized  by 


Google 


I>SC1MAL  FRACTIONS. 


161 


Reduce  15?50f  to  the  decimal 
of  an  hour. 

The  given  terms  being  reduced 
to  the  lowest  denomination  give 
950  seconds ;  annex  ciphers  and 
divide  by  3600^  the  seconds  in  an 
hour ;  as  thus, 

3600\ 950. 0000/.  2639  nearly 
Moo         V  Ans. 

23000 
21600 
.14000 
10800 
.32000 
32400 


Reduce  4?  10T50!  to  the  decimal 
of  a  day. 

The  given  time  being  reduced  to 
the  lowest  denomination  mentioned 
is  15050  seconds;  annex  ciphers 
and  divide  by  86400,  the  seconds 
in  a  day,  or  24  hours ; — thus, 
86400\  15050. 00000  /.  17419 

)  86400          A  nearly  Ans. 
641000 
604800 
.382000 
345600 
. 164000 
86400 
776000 

Reduce  3 '.  4  f  to  the  decimal  of  a 
pound  sterling. 

The  given  sum  being  reduced  to 
the  ioiyest  denomination  mentioned 
gives  40  pence,  annex  ciphers  and 
divide  by  240,  the  pence  in  a 
pound  sterling ;  as  thus, 
240\40.0000  /.  1666  Answer. 
;24W  V 

1600 
1440 

.1600 
1440 
.1600 

1440 

.160 


Reduce  45  minutes  to  the  deci- 
mal of  an  hour. 

The  given  number  being  in  the 
lowest  denomination  mentioned, 
annex  ciphers  and  divide  by  60, 
the  minutes  in  an  hour ;  as  thus, 

60\  45. 00  \ .  75  which  is  the 
)420     J         '  Ans. 

•  300 
300 


Reduce  100  fathoms  and  2  feet 
to  the  decimal  of  a  nautical  mile. 

The  given  measure  being  reduced 
to  the  lowest  denomination  men- 
tioned is  602  feet ;  annex  ciphers 
and  divide  by  6080,  the  number  of 
feet  in  a  sea  mile  ;  as  thus, 

6080\  602. 00000/.  09901  Ans. 
;  54720        V 


.54800 
54720 

...8000 
6080 

1920 


'     Reduce  3  qrs.  21  lb.  to  the  deci- 
mal of  a  hundred  weight. 

The  given  weight  being  reduced 
to  the  lowest  denomination  men- 
tioned is  105  lbs.  annex  ciphers,  and 
divide  by  112,  the  number  of 
pounds  in  a  himdred  weight  y  as 
thus, 

112\  105. 0000/. 9375  Ans. 
;  1008         V 


.420 
336 


.840 
784 
.560 
560 


M 


Digitized  by 


Google 


Casb  III. 


lb  find  the  vahke  of  any  Decimal  FYaction  in  the  known  parts  of  an 
Integer  ;  such  as  Degrees,  Time,  Coin,  Weighty  Measure^  ic^ 

Rule. 

Multiply  the  given  deeinial  by  the  number  of  parts  Contained  in  the  next 
inferior  denomination  ;  and,  from  the  right  hand  of  the  product,  point  oiF 
so  many  figures  as  the  given  decimal  consists  of. — Multiply  those  figures 
so  pointed  off  by  the' number  of  parts  contained  in  the  next  inferior  deno* 
mination,  and  from  tRe  'result  cut  off  the  decimal  places  as.  before:-— 
proceed  in  this  manner  till  the  ieavt  known,  or  required  parts  of  the  integer 
are  brought  out  ;-^hen,  the  several  denominations  on  the  left  hand  of  the 
decimal  points,  will  express  the  value  of  the  given  decimal  fraction. 


Example  1. 

Requbed  the  vahte  of  •  825  of  a 
degree. 

Given  decimal  •  825 
Multiply  by         60  minutes. 


49'. 500 
Multiply  by  60  seconds. 


30-.  000 
Hence,  the  reqmred  value  is  49  ^  30'/ 

Example  3. 

Required  the  vahie  of  .  166666 
of  a  pound  sterling. 

Given  decimal  s  •  166666 
Multiply  by  20  shffl. 


S-. 333320 
Multiply  by  12pence 


3*.  999840 

Henee^  the  reqiured  vahie  is  3;4f 
very  nearly. 


Example  2. 

Reqttred  the  value  of  •  2639  of 
an  hour. 

Given  deeimal  ss  •  2639 
Multiply  by  60  min. 

15\8840 
Multiply  by  60  seconds. 


50*. 040 


Hence,  the  required  value  is  15? 
50*.  040. 


Example  4. 

Beqiiirad  Um  nlue  of .  09901  of 
a  naatieal  or  m»  mik. 

Given  deeimal  =  .09901 
Multiply  by  60eO,tlMft. 

ia  a  aea  mjU,  ••  ■•  - 

792080 
594060 


601.98080 

Hence,  the  required  value,  is  602 
feet  veiy  nearly. 


DBCIMAL  VRACnONg.  IflB 


THE  RULE  OF  PROPORTION  IN  DECIMALS. 

Prepare  the  terms  by  reducing  the  fractional  parts  to  the  highest  deno- 
mination mentioned ;  .then  state  the  question  and  proceed  as  in  the  com- 
mon Rule  of  Three  Direct  5 — thus,  place  the  numbers  in  such  order  that  the 
first  and  third  may  be  of  the  same  kind,  and  the  second  the  same  as  the 
number  required  :— bring  the  first  and  third  terms  into  the  same  name^ 
and  the  second  into  the  highest  denomination  mentioned. — Then, 

Multiply  the  second  and  third  terms  together ;  divide  the  product  by  the 
first  term,  and  the  quotient  will  be  the  answer  in  the  same  denomination 
as  the  second  number  ;^— observing,  however,  to  point  off  the  decimal 
places ;  the  value  of  which  is  to  be  found  by  the  Rule  to  Case  III.^ 
page  162. 

iVbte.— In  the  rule  of  proportion  there  are  always  three  numbers  given 
to  find  a  fourth  proportional ;  two  of  these  are  of  supposition  and  one  of 
demand;  the  latter  must  ever -be  the  third  term  in  the  statement  of  the 
question ;  and,  as  this  is  interrogatory,  it  may,  therefore,  be  known  by  the 
words — ^What  will  ?  What  cost  ?  How  many  ?  How  far  ?  How  much  ?, 
&c, — ^The  jirst  term  must  always  be  of  the  same  name  as  jthe  third  j  the 
fourth,  or  term  sought,  will  be  of  the  same  kind  and  denomination  as  the 
second  term  in  the  proportion. 

Example  U 

If  a  degree  of  longitude,  measured  on  the  surface  of  the  earth  under  the 

equator,  be  69. 092  English  miles ;  how  many  miles  are  contained  in  die 

earth's  circumference  under  the  same  parallel,  it  being  divided  into  360 

degrees  ? 

As    ...     1?    :    69-. 092  ::  360? 

360 


4145520 
207276 


Answer      ...    24873. 120  English  miles. 

Example  2. 

The  earth  turns  round  upon  its  axis  in  23t56T ;  at  what  rate  per  hour 
are  the  inhabitants  carried  from  west  to  east  by  this  rotation  under  the 

M  2 


Digitized  by 


Google 


.164  ABCIMAL  FRACTIONS, 

equator  where  the  earth's  circumference  measures  24873. 12  English  miles ; 
and  at  what  rate  per  hour  are  the  inhabitants  of  London  carried  in  the 
same  direction,  where  a  degree  of  longitude  measures  42. 99  miles. 

First. — For  the  InhaUtantt  at  the  Etputtor, 

23  hours  56  minutes  are  equal  to  23. 9333  hours. — ^Now, 

Aa  23*. 9333    :    24873-.  12::  V.    :    1039  miles. 

24873. 1200 
239333 

..939820 
717999 


2218210 
2153997 

..64213 


Second.— i'br  the  Inhabitants  of  London. 

360  degrees  multiplied  by  42. 99  miles,  give  15476. 4  miles ; — ^And, 
As23\9333    :    15476"'. 4  ::  1*   :    646  miles. 


15476.4000 
1435998 


> 1116420 
957332 

. 1590880 
1435998 

.  154882 


Hence,  the  inhabitants  under  the  equator  are  carried  at  the  rate  of  1039 
miles  every  hour,  and  those  of  London  646  miles  per  hour,  by  the  earth's 
motion  round  its  axis. 


Example  3. 

If  a  ship  sails  at  the  rate  of  11^  knots  per  hour ;  in  what  time  would 
she  circumnavigate  the  globe,  the  circumference  of  which  b  24873. 12 
miles? 


Digitized  by 


Google 


PROPORTION^  AND  PROPRRTtBS  OF  NUMBERS.  165 

11 J  knots  are  equal  to  1 1. 25  mile8.^Now, 

As  11-'.  25   :    1!  ::  24873-M2    :    2210.9  hours. 
2250 


.2373 
2250 

.1231 
1125 

.10620 
10125 


..495 


Hence^  the  required  time  is  2210. 9  hours ;  or  92  days^  2  hours^  and  54 
minutes. 


PROPORTION,  AND  PROPERTIES  OF  NUMBERS. 

If  three  quantities  be  proportional,  the  product  or  rectangle  of  the  two 
extremes  will  be  equal  to  the  square  of  the  mean. 

If  four  quantities  be  proportiotial,  the  product  of  the  two  extremes  will 
be  equal  to  the  rectangle  or  product  of  the  two  means. — ^Thus, 

Let  2.4. 8. 16  be  the  four  quantities  ;  then,  the  rectangle  of  the  ex- 
tremes, vi2.  16  X  2,  is  equal  to  the  rectangle  of  the  means,  viz.  4x8, 
or  32. 

If  the  product  of  any  two  quantities  be  equal  to  the  product  of  two 
others,  the  four  quantities  may  be  turned  into  a  proportion  by  making  the 
terms  of  one  product  the  meanSf  and  the  terms  of  the  other  product  the 
extremes.— Thus, 

Let  the  terms  of  two  products  be  10  and  6,  and  15  and  4,  each  of  which 
is  equal  to  60 ;  then,  As  10:  4::  15  :  6.  ^4:  6::  10:  15.  As  6:  15:: 
4:10,  &c.  &c. 

If  four  quantities  be  proportional,  they  "shall  also  be  proportional  when 
taken  inversely  and  alternately. 

If  four  quantities  be  proportional,  the  sum,  or  difference,  of  the  first  and 
second  will  be  to  the  second,  as  the  sum,  or  difference  of  the  third  and 
fourth  is  to  the  fourth. — ^Thus,  let  2.4.8.16  be  the  four  proportioufil 
quantities;  then 

A«2  +  4:4::S+l6:l6}  or,a84-2:4::i6-8: 16. 


Digitized  by 


Google 


16C  PROPBRTIBS  OF  NUHBBR8. 

If  from  the  sum  of  any  two  quantities  either  quantity  be  taken,  the  re^ 
mainder  will  be  the  other  quantity. 

If  the  difference  of  any  two  quantities  be  added  to  the  less,  the  sum  will 
be  the  greater  quantity ;  or  if  subtracted  from  the  greater,  the  remainder 
will  be  the  less  quantity. 

If  half  the  difference  of  any  two  quantities  be  added  to  half  their  sum, 
the  total  will  give  the  greater  quantity  3  or  if  subtracted,  the  remainder  will 
be  the  less  quantity. 

If  the  product  of  any  two  quantities  be  divided  by  either  quantity,  the 
quotient  will  be  the  other  quantity. 

If  the  quotient  of  any  two  quantities  be  multiplied  by  the  less,  the  pro- 
duct will  be  the  greater  quantity. 

The  rectangle  or  product  of  the  sum  and  difference  of  any  two  quan- 
tities, is  equal  to  the  difference  of  their  Squares. — ^Thus, 

Let  4  and  10  be  the  two  quantities  j  then  4+ 10  =  14 ;  10-4  =  6,  and 
14x6=84.— Now,  lOxlOelOO;  4x4  =  16,  and  100-16  =  84. 

The  difference  of  the  squares  of  the  sum  and  difference  of  any  two  quan- 
tities, is  equal  to  four  times  the  rectangle  of  those  quantities. — ^Thus, 

Let  10  and  6  be  the  two  quantities;  then  10+6  =  16x16  =  256;— 
10-6  =  4x4=  16.— Now, 256-16  =  240;  and  10x6x4  =  240. 

The  sum  of  the  squares  of  the  sum  and  difference  of  any  two  quantities, 
is  equal  to  twice  the  sum  of  their  squares. — ^Thus, 

10+6=  16x16  =256;  and  10-6=4x4=16}  then 256+16*272. 
Again,  10x10=100;  6x6  =  36,  and  100  +  36  =  136x2^272. 

If  the  sum  and  difference  of  any  two  numbers  be  added  together,  the 
total  will  be  twice  the  greater  number. — ^Thus, 

10+6  =  16;  and  10.-6  «  4;  then  16+4  «  20;  and  10x2  =  20, 
If  the  difference  of  any  two  numbers  be  subtracted  from  their  sum,  the 
remainder  will  be  twice  the  less  number. — ^Thus, 

10-6  =  4;  and  10+6  =16;  then  16-4s  12;-^and6x2  =  12. 
The  square  of  the  sum  of  any  two  numbers  is  equal  to  the  sum  of  their 
squares,  together  with  twice  their  rectangle. — Thus, 

10+6=16;  and  16x16  =  256.  Again,  10x10=  100;  6x6  =  36, 
and  100+36=  136;  then,  10x6x2  =  120;  and  120+136  =  256. 

The  sum,  or  difference,  of  any  two  number9  will  measure  the  sum,  or 
difference,  of  the  cubes  of  the  same  numbers ;  that  is,  the  sum  will  mea- 
sure the  sum,  and  the  difference  the  difference. 

The  difference  of  any  two  numbers  will  measure  the  difference  of  the 
squares  of  those  numbers. 

The  sum  of  any  two  numbers  differing  by  an  unit  (1,)  is  equal  to  the  dif- 
ference of  the  squares  of  those  numbers. — ^Thus, 
9+8=17;  and9x9  =  81;  8x8=64;  now,  81-64  =  17. 
If  the  sum  of  any  two  numbers  be  multiplied  by  each  aunber  respect- 


Digitized  by 


Google 


.fLANB  TftlGONOMXTBT*  167 

ively,  the  sum  of  the  two  rectangles  will  be  equal  to  the  square  of  the  sum 
of  those  numbers. 

Thus,  10+6=16;  now,  16x10=160;  16x6  =  96;  and  160+96 
=  256. 

Again,  10+6=  16;  and  16x  16  =  256. 

The  square  of  the  sum  of  any  two  numbers  is  equal  to  four  times  the 
square  of  half  their  sum. — ^Thus, 

10+6  s  16 ;  and  16  X 16  =  256 ;  then  10+6  =  16-4*2  =  8,  and  8  k  8 
x4=2S6. 

The  sum  of  the  squares  of  any  two  numbers  is  equal  to  the  square  of 
their  difference,  together  with  twice  the  rectangle  of  those  numbers.— 
Thus, 

10x10=  100;  6x6x3  36;  and  100+36=  136.— Again^ 

10^6  =  4;  and  4x4=  16;  10x6x2  =  120;  and  120+16=136. 

The  numbers  3,  4  and  5,  or  their  multiples  6,  8  and  10,  &c.  &c.,  will 
express  the  three  sides  of  a  right  angled  plane  triangle. 

The  sum  of  any  two  square  numbers  whatever,  their  difference,  and 
twice  the  product  of  their  roots,  will  also  express  the  three  sides  of  a  right 
angled  plane  triangle. — ^Thus, 

Let  9  and  49  be  the  two  square  numbers : — then  9+49  =  58 ;.  49—9  =i 
40.— Now,  the  root  of  9  is  S,  and  that  of  49  is  7 ;— then  7  x  3  x  2  =  42  i 
henee  the  three  sides  of  the  right  angled  plane  triangle  will  be  58,  40, 
and  42. 

The  sum  of  the  squares  of  the  base  and  perpendicular  of  a  right  angled 
plane  triangle,  is  equal  to  the  square  of  the  hypothenuse. 

The  diference  of  the  squares  of  the  hypothenuse  and  one  leg  of  a  right 
angled  plane  triangle,  i»  equal  .to  the  square  of  the  other  leg. 

The  rectangle  or  product  of  the  sum  and  difference  of  the  hypothenuse 
and  one  leg  of  a  right  angled  plane  triangle,  is  equal  to  the  square  of  the 
other  leg. 

The  cube  of  any  number  divided  by  6  will  leave  the  same  remainder  as 
the  number  itself  when  divided  by  6.—- The  difference  between  any  number 
and  its  cube  will  divide  by  6,  and  leave  no  remainder. 

Any  even  square  number  will  divide  by  4,  and  leave  no  remainder;  but 
an  uneven  square  number  divided  by  4  will  leave  1  for  a  remainder. 


Digitized  by 


Google 


168  FI.ANB  TRIGONOMETRY. 


PLANE  TRIGONOMETRY. 

The  Resolution  of  the  different  Problems,  or  Cases^  in  Plane  Trigonometry, 

by  Logarithms^ 

Although  it  is  not  the  author's  intention  (as  has  been  already  observed^) 
to  enter  into  the  elementary  parts  of  tlie  sciences  on  which  he  may  have 
occasion  to  touch  in  elucidating  a  few  of  the  many  important  purposres  to 
which  these  Tables  may  be  applied  3  yet,  since  this  work  may,  probably, 
fall  into  the  hands  of  persons  not  very  conversant  with  trigonometrical 
subjects,  he  therefore  thinks  it  right  briefly  to  set  forth  such  definitions,  &c. 
as  appear  to  be  indispensably  necessary  towards  giving  such  persons  some 
little  insight  into  this  particular  department  of  science* 

Plane  Trigonometry  is  that  branch  of  the  mathematics  which  teaches 
how  to  find  the  measures  of  the  unknown  sides  and  angles  of  plane  trian- 
gles from  some  that  are  already  known. — It  is  divided  into  two  parts ;  right 
angled  and  oblique  angled  : — in  the  former  case  one  of  the  angles  is  aright 
ungle,  or  90?  ;  in  the  latter  they  are  all  oblique. 

Every  plane  triangle  consists  of  six  parts ;  viz.,  three  sides  and  three  an* 
gles ;  any  three  of  which  being  given  (except  the  three  angles),  the  other 
three  may  be  readily  found  by  logarithmical  calculation. 

In  every  triangle  the  greatest  sideis  opposite  to  the  greatest  angle ;  and, 
vice  versa,  the  greatest  angle  opposite  to  the  greatest  side. — But,  equal 
sides  are  subtended  by  equal  angles,  and  conversely. 

The  three  angles  6f  every  plane  triangle  are,  together,  equal  to  two  right 
angles,  or  180  degrees. 

If  one  angle  of  a  plane  triangle  be  obtuse,  or  more  than  90?,  the  other  two 
are  acute,  or  each  less  than  that  quantity :  and  if  one  angle  be  right,  or  90?, 
the  other  two  taken  together,  make  90?  : — ^hence,  if  one  of  the  angles  of  a 
right  angled  triangle  be  known,  the  other  is  found  by  subtracting  the  known 
one  from  90? . — If  one  angle  of  any  plane  triangle  be  known,  the  sum  of  the 
other  two  is  found  by  subtracting  that  which  is  given  from  180? ;  and  if  two 
of  the  angles  be  known,  the  third  is  found  by  subtracting  their  sum 
from  180? 

The  complement  of  an  angle  is  what  it  wants  of  90?  5  and  the  supple-^ 
ment  of  an  angle  is  what  it  wants  of  ISO? 

In  every  right  angled  triangle,  the  side  subtending  the  right  angle  is 
called  the  hypotlieniise ;  the  lower  or  horizontal  side  i&  called  the  6(we,  and 
that  which  stands  upright,  the  peTpendkuIar* 


Digitized  by 


Google 


PtANB  TRIGOKOMBTRY. 


169 


If  the  hypothenuse  be  assumed  equal  to  the  radius^  the  sides^  that  is,  the 
base  and  the  perpendicular,  will  be  the  sines  of  their  opposite  angles.  And, 
if  either  of  the  sides  be  considered  as  the  radius,  the  other  side  will  be  the 
tangent  of  its  opposite  angle,  and  the  hypothenuse  the  secant  of  the  smne 
angle» 

Thus. — Let  A  B  C  be  a  right  angled  plane  triangle ;  if  the  hypothenuse 
A  C  be  made  radius,  the  side  B  C  will  be  the  sine  of  the  angle  A,  and 
AB  the  sine  of  the  angle  C— If  the  side  AB  be  made  radius,  BC  will 
be  the  tangent,  and  A  C  the  secant,  of  the  angle  A :— -And,  if  B  C  be  the 
radius,  A  B  will  be  the  tangent,  and  A  C  the  secant  of  the  angle  C. 

For,  if  we  make  the  hypothenuse  A  C  radius 
(Fig.  l.)>  and  upon  A,  as  a  centre,  describe  the 
arch  C  D  to  meet  A  B  produced  to  D ;  then 
it  is  evident  that  B  C  is  the  sine  of  the  arch  D  .C, 
which  is  the  measure  of  the  angle  B  A  C ;  and 
that  A  B  is  the  co-sine  of  the  same  arch  : — and 
if  the  arch  A  E  be  described  about  the  centre 
C,  to  meet  C  B  produced  to  E,  then  will  A  B  be 
the  sine  of  the  arch  A  E,  or  the  sine  of  the  angle 
A  C  B,  and  B  C  its  co-sine. 

Again,  with  the  extent  A  B  as  a  radius  (Fig.- 
2.),  describe  the  circle  B  D ;  then  B  C  is  the 
tangent  of  the  arch  B  D,  which  is  evidently  the 
measure  of  the  angle  BAC;  and  AC  is  the 
secant  of  the  same  arch,  or  angle. 

Lastly,  with  CB  a(s  a  radius  (Fig.  3.),  describe 
the  arch  B  D ;  then  A  B  is  the  tangent  of  the 
arch  B  D,  the  measure  of  the  angle  A  C  B,  and 
A  C  the  secant  of  the  same  arch  or  angle. 

In  the  computation  of  right  angled  triangles,  any  side,  whether  given  or 
Tcquired,  may  be  made  radius  to  find  a  side  ;  but  a  given  side  must  be 
made  radius  to  find  an  angle  :  thus, 

To  find  a  Side;— 

Call  any  one  of  the  sides  of  the  triangle  radius,  and  write  upon  it  the 
word  rodtitf  .•^^-observe  whether  the  other  sides  become  sines^  tangents,  or 
secants,  and  write  these  words  on  them  accordingly,  as  in  the  three  pre* 
ceding  figures  :  then  say,  as  the  name  of  the  given  side,  is  to  the  given 
side}  80  is  thename  of  the  side  required,  to  the  side  required* 


Digitized  by 


Google 


170  PLAVB  miQovoumr. 


Andj  to  find  an  Angle  :-r- 

Call  one  of  the  gweti  rides  the  radius^  and  write  upon  it  theirord  radius  t 
observe  whether  the  other  sides  become  sines^  tangents,  or  secants,  and 
write  these  words  on  then)  accordingly,  as  in  the  three  foregoing  figures  ; 
then  say,  as  the  side  made  radius,  is  to  radius ;  so  is  the  other  gicen  ride 
to  its  name :  that  is^  to  the  sine,  tangent,  or  secant  by  it  represented. 

Now,  since  in  plane  trigonometry  the  sides  of  a  triangle  may  be  eonsi- 
dered,  i^thout  much  impropriety,  as  being  in  a  direct  ratio  to  the  sines  of 
their  opposite  angles,  and  conversely ;  the  proportion  may,  therefore,  be 
stated  agreeably  to  the  established  principles  of  the  Rule  of  Three  Direct, 
by  saying 

As  the  name  of  a  given  angle,  is  to  its  opposite  given  side  $  so  is  the 
name  of  any  other  given  angle  to  its  opposite  side.-— And,  as  a  given  side, 
is  to  the  name  of  its  opposite  given  angle ;  so  is  any  other  given  side  to  the 
name  of  its  opposite  angle. 

The  proportion,  thus  stated,  is  to  be  worked  by  logarithms^  m  the  fol* 
lowing  manner ;  viz., 

To  the  arithmetical  complement  of  the  first  term,  add  the  logs*  of  the  se- 
cond and  third  terms,  and  the  sum  (rejecting  20,  or  10  from  the  index, 
according  as  the  required  term  may  be  a  side  or  an  angle,)  will  be  the 
logarithm  of  the  required,  or  fourth 'term. 

Rem(n'ks,r^l.  The  arithmetical  complement  of  a  logarithm  is  what 
that  logarithm  wants  of  the  radius  of  the  Table  ^  viz.^  what  it  is  short  of 
10. 000000 ;  and  the  arithmetical  complement  of  a  log.  sine,  tangent,  or 
secant,  is  what  such  logarithmic  sine,  &c.  &c.  wants  of  twice  the  radius 
of  the  Tables,  viz.,  20. 000000. 

2.  The  arithmetical  complement  of  a  log.  is  most  readily  found  by  be- 
ginning at  the  left  hand  and  subtracting  each  figure  from  9  except  the  last 
significant  one,  which  is  to  be  taken  from  10,  as  thus ; — ^If  the  given  log; 
be  2. 376843,  its  arithmetical  complement  will  be  ?•  623157  : — ^if  a  given 
log.  sine  be  9. 476284^  its  arithmetical  complement  will  be  10. 523716^  and 
soon, 

3.  The  arithmetical  complement  of  the  log.  sine  of  an  arch,  is  the  log. 
co-secant  of  that  arch  ;-^the  arithmetical  complement  of  the  log.  tangent  of 
an  arch,  is  the  log.  co- tangent  of  that  arch;  and  conversely^  in  both 
cases*  •  ,       : 


Digitized  by 


Google 


PLANS  TRIOONOMITBT.  t71 

Solution  of  Right-angled  Plane  Triangles,  by  Logarithms. 

PROBLBItf   h 

Given  the  Jnglei  and  the  Hypothenuse,  to  find  the  Base  and  the 
Perpendicular. 

'    '    Example. 

Let  the  hypothcniise  A  C,  of  the  annexed  trian- 
gle ABC,  be  246.5,  and  theangle  A  58?7M8r  | 
required  the  base  A  fi,  and  the  perpendicular  B  C  ? 

Note. — Since  there  is  no  more  intendedf  in  this 
placcy  than  merely  to  show  the  use  efthe  Tables; 
the  geometrical  construction  of  the  diagrams  is, 
therefore,  purposely  omitted. 

By  making  the  hypothennse  A  C  radius ;  B  C  'becomes  the  sine  of  the 
angle  A9  and  A  B  the  co-sine  of  the  same  angle, — Hence, 

To  find  the  Perpendicular  B  C  :— 

As  radius  =:  •  .  •  .  90?  =:  Log.slne  =  •  •  10. 000000 
btokypotbenu0eAC=:246.5  Log,  =s ,  .  .  «  2.391817 
So ia  the  angle  A=  53?7M8r  Log.  sine  zz    .    .    9. 903090 


To  the  perpendicular  B  C  =  1 97-  2  =  Log.  =      ...    2. 294907 
To  find  the  Base  A  B  :— 

As  radius  =  .    .    .    ^  90?  =  Log.  sine  =     .    .     10. 000000 

Is  to  hypotbenuse  A  C  =  246. 5  Log.  =  ...  2. 391817 
So  is  the  angle  A  =  53?7 '  481  Log.  co-sine  =:     .      9. 778153 

Tfc the  base  AB  =  147.9  =■  .  Log.  =       ...      9. 169970  - 

Making  the  base  A  B  radius ;  B  C  becomes  the  tangent  of  the  angle  A^ 
and  A  C  the  secant  c^the  same  angle. — Henee^ 

To  find  tfie  Perpendicular  B  C  :— 

As  the  angle  A  =  •  53? 7  •  48?  Log.  secant  Ar.  comp.a  9. 778 153 
Is  to  hypothennse  A  C  =  246. 5  Log.  =z  .  •  .  .  .  2.391817 
So  is  the  angle  A  =  53?7'48r       Log.  tangent  =:     .    .  10. 124937 


TotbepcriKsiMtteuUrBC^  179.2  =Wp.;5     »   .- •    ?.294907    . 

/Google 


Digitized  by ' 


172  PtANB  TRIGONOBfBTRT. 


To  find  the  Base  A  B:— 


As  the  angle  A  =  53?7'48r  Log.  secant  Ar.  compt.  =  9. 778153 
Is  to  hypothenuse  A  C  =  246. 5  Log.  =:«..•»  2. 391817 
So  is  radius  =  90"?  Log.  sine  =:  .    •    •    •  10. 000000 

To  the  base  AB      147.9=:  Log.  =  •    ,    4    •    «    2.169970 

The  perpendicular  B  C  being  made  radius ;  the  base  AB  becomes  the  tan- 
gent of  the  angle  C,  or  co-tangent  of  the  angle  A,  and  the  hypothenuse  A  C 
the  secant  of  the  angle  C^  or  co-secant  of  the  angle  A.*— -Hence, 

To  find  the  Perpendicular  B  C : 

As  the  angle  A  ==  53?7'48?  Log.  co-secant  .Ar.  compt.  =:  9. 903090 

Is  to  hypothenuse  A  C  ==  246. 5  Log •    «    2. 391817 

So  is  radius  =  90"?      «    •    •    Log.  sine    •    •    •    •    •  10.000000 

To  the  perpendicular  B  C  =  197. 2  =  Log.  =  .    .    •    •    2.  294907 

To  find  the  Base  A  B:— 

As  the  angle  A  it  53?7'48?  Log.  co-secant  Ar.  compt.  =  9. 903090 

Is  to  hypothenuse  AC  zz  246. 5  Log.  = 2. 891817 

So  is  the  angle  A  =:  53?7  •  48r     Log.  co-Ungent      .    •    9. 875063 

Tothe  base  AB=:  147.9=        Log.  = 2.169970 


Problem  II. 

Given  the  Afigles  wdOne  Side,  to  find  the  Hypothetme  and  the  other 

&de.' 

Example. 

Let  the  base  A  B  of  the  annexed  triangle  ABC, 
be  300.5,  and  the  angle  A  40? 54  MO'/  5  required 
the  hypothenuse  A  C,  and  the  perpendicular  B  C  ? 

J  CCS 

Tlie  hypothenuse-  A  C  being  made  radius;  the  perpendicular  B  C  mil  be 
the  «ine  of  the  angle  A^  and  the  base  A  B  the  co-sine  of  the  same  a&gle« 


Digitized  by 


Google 


PLANB  TRIGONOMETRY*  l?? 

To  find  the  HypotheDUse  A  C : — 

Aa  the  angle  A  =  4O?54U0r  Log.co-sine  Ar.  compt.  =  10. 121635 

Is  to  the  base  AB=:  300.5      Log.  = 2.477845 

So  is  radius  =  90?  Log.  sine  =     .    •    .    .     10. 000000 

To  the  hypothcnuse  A  C  =  397- 6  =  Log.  =     .    .    •      2. 599480 

To  find  the  Perpendicular  B  C  :— 

As  the  angle  A  =  40?54M0r  Log.  co-sine  Ar.  compt.  =  10. 121635 

Is  to  the  base  AB  =  300.6  Log.  = 2.477845 

So  is  the  angle  A  =:  40?54M0r  Log.  sine  =  *    ,    .    •    9. 816167 


To  the  perpendicular  B  C  =  260. 4  =  Log.  =  .    .    .    .    2.41 5647 

The  base  A  B  being  made  radius ;  the  perpendicular  B  C  will  be  the  tan- 
gent of  the  angle  A,  and  the  hypothenuse  A  C  the  secant  thereof.— Hence, 

To  find  the  Hypothenuse  AC: — 

As  radius  =  90?  Log.  sine  = •    .    •    .     10.000000 

Is  tothe base  AB  =  300.5  Log.  = 2.477845 

So  is  the  angle  A  =:  40?54M0^'  Log.  secant    ....  10. 121635 


To  the  hypothenuse  A  C  =  397. 6  =  Log.  =  ....    2. 599480 

To  find  the  Perpendicular  B  C  :— 

As  radius  =  90*?  =  Log.  sine  = 10.000000 

Is  to  the  base  A  B  =  300. 5  Log •    .      2.477845 

So  is  the  angle  A  =r  40?54M0r  Log.  tangent    ....      9. 937802 

To  the  perpendicular  B  C  =  260. 4  r:  Log.  =      ...    2. 415647 

Th^  perpendicular  B  C  being  made  radius ;  the  base  A  B  will  be  the  tan- 
gent of  the  angle  C,  or  co-tangent  of  the  angle  A,  and  the  hypothenuse  the 
secant  of  the  angle  C,  or  co-secant  of  A.— Hence,. 

To  find  the  Hypothenuse  AC : — 

As  the  angle  A  dt  40?  54 :40r  Log.  co-tang.  Ar.  compt.  =  9. 937802 

Is  to  the  base  A  B  =  300. 5  Log.  = 2. 477845 

So  is  the  angle  A  =  40?54M0r  Log.  co-secant  =      ..      10. 183833 


To  the  hypothenuse  A  C  =  397. 6  =  Log.  =    •    .    .    •  2. 599480 

/Google 


Digitized  by ' 


174 


PLANB  TRIGONOMSTET* 


To  find  the  Perpendicular  B  C  :— 

As  the  angle  A  =  40?54U0r  Log.  co-tang.  Ar.  compt.  =:  9. 937802 

Is  to  the  basi  A  B  ^  300. 5  Log.  = 2. 477845 

So  is  radius  =:  90?  Log.  sine  = 10.000000 


To  the  perpcildicular  BC  =  260. 4  =:  Log.  =; 


2.415647 


pROfiLBM  IIL 

Oicen  tlie  Hypothenuse  and  One  Side,  to  find  the  Angles  and  the 

Other  Side. 


Example. 

Let  the  hypothenuse  A  C,  of  the  annexed  tri- 
angle ABC,  be  330. 4,  and  the  base  A B  280.3 ; 
required  the  angles  A  and  C^  and  the  perpendi- 
cular B  C  ? 


2Sa.j 


By  making  the  hypothenuse  A  C  radios ;  the  perpendicular  B  C  becomes 
the  sine  of  the  angle  A,  and  the  base  A  B  the  co-sine  of  the  same  angle, — 
Hence^ 

To  find  the  Angle  A:— 

As  the  hypothenuse  A  C  ==  330. 4  Log.  Ar.  compt.  =        7. 480960 

Is  to  radius  =:  90?  Log.  sine  = 10.  OOOOOO 

So  is  the  base  AB  =  280.3  Log.  = 2.447623 

To  the  angle  A  =31?57'56r  Log.  co-sine  =1   .    •    .      9.92858» 

To  find  the  Perpendicular  B  C. 

As  radius  =  90?  Log.  sine  = ,    *    •     10.  OOOOOO 

Is  to  hypothenuse  A  C  =:  330. 4  Log.  s= 2. 519040 

SoistheangleA=:31?57^56rLog.aines      .    .    .      9.723791 

To  the  perpendicular  B  C  =  174. 9  =  Log.  =    •    .    .      2. 24283 1 

The  base  A  B  being  made  radius  ; .  the  perpendicular  B  C  becomes  the 
tangent  of  the  angle  A>  and  the  hypothenuse  A  C  the  secant  of  that  angle. 
— Hence^       


Digitized  by 


Google 


VUaa  TBIQOMOlIBTaT. 


17S 


To  find  the  Angle  A  : — 

As  the  base  A  B  =  280. 3  Log.  Ar.  compt.  =    .    .    •  7. 552377 

Is  to  the  radius  =  90?  Log.  sine  =        10. 000000 

So  is  the  hypothenuse  A  C  >=s  330. 4=  Log.  :&:...  2.5 19040 


TotlieangleA  =  31^57?56rLog.  secanta      .    . 
•    -  TofindthePerpeiKiicidarBC:^ 

As  nMlius  s=  90?  .Lg^.  sine  »••••.••* 

Is  to  the  base  AB  =  280. 3  Log.  = 

So  is  the  angle  A  ^  31?57 -56^  Log.  tangent  a     . 

1*0  the  perpendicwlaF  B  C  a  174»  9  a  Log.  »    » 


10.071417 


10.000000 
2.447623 
9. 795208 

2.242831 


jBpmari.^The  perpendicular  B  C  may  be  found  independently  of  the 
angles  by  the  following  rule  (deduced  from  Euclid,  Book  I.  Prop.  47,  and 
Book  U.  Prop.  5^9  viz.. 

To  the  log.  of 'the  sum  of  the  hypothenuse  and  giren  side,  add  the  log. 
0f  Atk  diffevenee ;  then,  half  the  sum  of  these  two  li>gs.  will  be  the  log.  of 
the  required  side : — as  thus  ; 


Hypothenuse 
Base .    .    , 

AC  = 
AB  = 

rBC  = 

330.4 
280.3 

Difference  . , 

610.7  Log.    . 
50.  i  Log.   . 

Sum    . 

174.9  =  Log. 

2.785828 
1.699S38 

4.485666 

Perpendicula 

2.242833 

PUOBLBM  IV. 

Giioen  the  Base  and  the  Perpendicular^  to  find  the  Jngles  and  the 

Hypothenuse, 

Example. 

Let  the  base  AB,  of  the  annexed  triangle  ABC, 
be  262. 5,  and  the  perpendicular  B  C  210. 4 ;  re* 
quired  the  aogleSi  and  the  hypothenuse  A  C  2 


Digitized  by 


Google 


176 


FLANB  TRIGONOMSTRT.. 


By  making  the  base  A  B  radius ;  the  perpendicular  B  C  becomes  the 
tangent  of  the  angle  A,  and  the  hypothenuse  A  C  the  secant  thereof. 
—Hence, 

To  find  the  Angle  A : — 

As  the  base  A  B  =  262. 5  Log,  Ar.  compt.  =  ....    7. 58087 1 

Is  to  radius  =:  90?  Log.  sine  ae 10. 000000 

So  is  the  perpendicular  B  C  =  210. 4  Log.  s  •    .    .    •    2. 323046 

To  the  angle  A  s38?42M7i:  Log.  tangents     «    .    «    9.903917 

To  find  the  Hypothenuse  A  C  :— 

As  radius  s  90^.  Log.  sine  s      .    .    .    .    ^    .    .    .    10.000000    ' 

Is  to  the  base  AB  =  262.5  Log.  = 2.419129 

Sobth«angleA=x.38?42M7^  Log.  secant  =  .    .    .     10.107745 

To  theliypothenuse  A  C  =  336. 4  =  Log.  =     .    •    .      2. 526874 

The  perpendicular  B  C  being  made  radius ;  the  base  A  B  will  be  the  tan- 
gent of  the  angle  C,  or  co-tangent  of  the  angle  A,  and  the  hypothenuse  A  C 
will  be  the  secant  of  C,  or  the  co-secant  of  the  angle  A. — ^Hence, 

To  find  the  Angle  A  :— 

As  the  perpendicular  B  C  =  210. 4  Log.  Ar.  compt.  s=     •  7*  676954 
Is  to  radius  =  90?  Log.  sinfe  =  .    .    .    .    .    .    .     .•    10.000000 

Soistheba8eABs=262.5Log.  = 2.419129 


To  the  angle  A  =  38?42:47?  =  Log.  co-tangent  =   .     10. 096083 

To  find  the  Hypothenuse  A  C  :— 

As  radius  =  90?  Log.  sine  = lO.OOOOOO 

Is  to  the  perpendicular  B  C  =:  210. 4  Log.  =     .     .     .      2. 323046 
So  is  the  angle  A  =  38?42U7^  Log.  co-secant  =  .     .     10. 203828 

To  the  hypothenuse  A  C  =;  336. 4  =  Log.  =  .      .    .      2. 526874 

The  angle  A  subtracted  from  90?  leaves  the  angle  C ;  thus  90?  —  38? 
42C47^  =  51?17'.  W^  the  measure  of  the  angle  C. 

Bcmarfc.— The  hypothenuse  A  C  may  be  found  independently  of  the  an- 
gles by  the  following  rule,  deduced  principally  from  Euclid,  Book  I* 
Prop.  47 }  Book  U.  Prop.  5  -,  and  Book  VL  Prop.  8^  viz.. 


Digitized  by 


Google 


TLAKB  TRtGONOMBTRY. 


177 


From  twice  the  log.  of  the  base  subtract  the  log.  of  the  perpendicular^ 
and  add  the  corresponding  natural  number  to  the  perpendicular ;  then^  to 
the  log.  of  this  sum  add  the  log.  of  the  perpendicular^  and  half  the  dum  of 
these  two  logs,  will  be  the  log.  of  the  hypothenuse.    As  thus : — 

Base  A  B  =  .    .    •  262. 5  twice  the  log.  ==  4. 838258 
PerpendicutarBC=  210.4  Log.     .     .     =  2.323046  •    .  2.323046 


Natural  number   zz  327*  5  Log.     • 
Sum  •    •    •    .    •    537. 9  Log.  s  • 

Hypothenuse  A  C  =:  336. 4  Log.  =  . 


=  2.515212 


.  .  .  2.730702 
Sum  =  5.053748 


.  ..  2.526874 


Solution  of  Oblique-angled  Plane  Triangles  by  Logarithms. 

Problem  L 

Given  ike  Angles  and  One  Side  qfan  Oblique^angkd  Plane  TYiangle,  to 
find  the  other  Sides. 

RULB. 

'  As  the  Log,  sine  of  any  given  angle^  is  to  its  opposite  given  side ;  so  is 
the  log.  sine  of  aiiy  other  given  abgle  to  its  opposite  side* 

Example. 

Let  the  side  A  B,  of  the  triangle  ABC, 
be  300.2,  the  angle  A  39?39^^0r  the 
angle  C  90?33C26r  and,  hence,  the  angle 


B  49?47'14- 
BC. 


to  find  the  sides  A  C  and 


Ci^fi.  z 


TofindtheSide  AC:— . 

As  the  angle  C  =:  90?33'.26:  Log.  sine  ar.  compt.       =  10. 000021 

IstothesideBC  =  300.2Log 2.477411 

So  is  the  angle  B  =.49?47 '  Ur  Log.  sine       ....     9. 882895 

TothesideAC  =;  229,3=;  Log.  r=     ......    2.360327 


Digitized  by 


Google 


178  PLAMS  TRlGOMOMBTRt. 


To  find  the  Side  BCr- 


As  the  angle  C  =  90*33 '.  26f  Log.  wn*  ar.  compt.  =      U).  000021 

Is  tothe8ideBC  =  300.2  Log.  = 2.477411 

So  is  the  angle  A  =  39?39;  20r  tog.  dne  =  .    ...    .    9. 804937 

To  the8ideBC=  191.6  =  Log.  = 2.282369 

Note.— When  a  log.  sine,  or  log.  co-sme,  is  the  first  term  in  the  propor- 
tion, the  arithmetical  complement  thereof  may  be  taken  directly  from  the 
Table  of  secants  Iqr  using  a  log.  co-secant  in  the  foimer  case,  and  a  log. 
secant  in  the  latter. 


Probi^bm  II* 

Givea  two  Sidet  and  an  Angle  apposUe  tooneqf  ikem,  tojind  the  other 
Angles  and  the  third  Side. 

Ruui. 

As  any  given  side  of  a  triangle  is  to  the  log.  sine  of  its  opposite  given 
angle,  so  is  any  other  given  side  to  the  log.  sine  of  the  angle  opposite 
thereto. 

The  angles  being  thos  foond,  the  tlurd  side-is  to  be  computed  by  the 

preceding  Problem. 

Example.     * 

Let  the  side  A  B,  of  the  triangle  A  B  C,  be  4^.  7$ 
the  side  AC  684. 5,  and  the  angle  B  100?7C36?  ;  re* 
quired  the  angles  A  and  C,  and  the  side  B  C  ? 


^^f7 


TofiDdtfaeane^C:— » 


As  the  side  AC  s  .  684. 5  Log.  ar.  comp.  7*  164626 
IstotheangleA=  100?7^35?  Log.sine=:  9.993181 
SoisthesideABs  436.7  Log.  =    .    .    2.640183 

TQtheaDgleCs38?54:22;      Log«tt   .    .    9.797990 

Digitized  by  VjOOQ IC 


PIAMV  TmiOONOMXTEY.  179 

To  find  the  side  B  C  :— 

As  the  angle  B  =    .     100?7'35'/     Log.  sine  ar.  comp,  =  10. 006819 
Is  to  the  side  A  C  =  684. 5  Log.  =      .    .     .    •      2. 835374 

So  is  the  angle  A  ==  40'?58:3^  Iiog.  sine  =    .    .    .      9. 816659 

To  the  side  B  C  =  455. 9  ;=  Log.  =      ....      2. 658852 

JVbte.-.The  angle  A  =  100?7J35r  +  the  angle  C  =  38^54^22^  = 
1399l^57r;andl80?  -  139?1^57^  =  the  angle  A  =  40?58C3r 

Remark. — ^An  angle  found  by  this  rule  is  ambiguous  when  the  given 
side  opposite  to  the  given  angle  is  less  than  the  other  given  side ;  that  is^ 
the  angle  opposite  to  the  greater  side  may  be  either  acute  or  obtuse :  for 
trigonometry  only  g^ves  the  sine  of  an  angle,  which  sine  may  either  repre- 
sent the  measure  of  the  angle  itself,  or  of  its  supplement  to  180  degrees. 
But  when  the  given  side  opposite  to  the  given  angle  is  greater  than  the 
other  given  side,  then  the  angle  opposite  to  that  (other  given)  side  is  always 
acute,  as  in  the  above  example. 


Problsm  IIL 

Given  two  Sides  and  the  inchded  Angle,  to  find  the  other  Angles  amd  the 

Hwrd  Side, 

Rule. 

Find  the  sum  and  difference  of  the  two  given  sides  j  subtract  the  given 
angle  from  180?  ;  take  half  the  remainder,  and  it  will  be  half  the  sum  of 
the  unknown  angles  i  then  say, ' 

As  the  sum  of  the  sides  is  to  their  difference ;  so  is  the  log.  tangent  of 
half  the  sum' of  the  unknown  angles,  to  the  log.  tangent  of  half  their 
difierence. 

Now,  half  the  difference  of  the  angles,  thus  found,  added  to  half  their 
sum^ gives  the  greater  angle,  or  that  which- is  opposite  to  the  greater  side; 
and  being  subtracted,  leaves  the  angle  opposite  to  the  less  side. 

llie  angles  being  thus  determined^  the  third  side  is  to  be  computed  by 
Problem  I.,  page  177* 

C 
Example. 

Let  the  side  A  B,  of  the  triangle  AB  C,  be 
210. 3,  the  side  B  C  160.  2,  and  the  angle  B 
110?!'  20^;  required  the  angles  A  and  C,  and  a 
the  side  AC?  2^0.  J 

N  2 


Digitized  by 


Google 


180 


PLAKB  TRIGONOMSTRT. 


180?  -  the  angle  B  110?i:20r  =  69?58M0r  h-  2  =  34?59^20r  = 
half  the  sum  of  the  angles  A  and  C« 


Side  AB  =: 
Side  BC  = 


210.3 
160.2 


As  sum  = 370. 5     LiOg  ar.  comp.  =:  7*  431212 

Is  to  difference  =  .    .     .       50.1     Log.  =     •    .     1.699838 
So  is  i  sum  of  angles  =  34?59C  20i:  Log.  tang.  =:     9. 845048 


Tojdiflfer.  of  angles  =    5?24^24r  Log.  tang.  =     8.976098 

Angle  C  =      .     . 
Angle  A  =      •    . 


.  40?28M4^ 
.  29?34:56- 


As  the  angle  A  =: 
Is  to  the  side  B  C  = 
So  is  the  angle  B  = 


To  find  the  side  AC: 

.  29^34^56^  Log.  sine  lar.  comp.  =:  10.306561 
.  160.2  Log.=  ....  2.204663 
.  110?lC20r    Log.  sine  =     i    .    •    9.972925 


To  the  side  A  C  =:     .      304. 9  =   Log.  = 


2.484149 


Problbm  IV. 


Gken  tlie  three  Sides  of  a  Plane  TViangle,  to  find  the  Angles. 

IluLB. 

Add  the  three  sides'  togiether,  and  take  half  their  sum )  the  difference, 
between  which  and  the  side  opposite  to  the  required  angle  call  the  remain^ 
der;  then, 

To  the  arithmetical  complements  of  the  logs,  of  the  other  two  sides,  add 
the  logs,  of  the  half  sum  and  of  the  remainder:  half  the  sum  of  these  four 
logs,  will  be  the  log.  co-sine  of' an  archj  which,  being  doubled,  will  give 
the  required  angle. 

Now,  one  angle  being  thus  found,  either  of  the  other  two  angles  miiy'be 
computed  by  ftoblem  II.,  page  178. 

Example.  • 

Let  the  side  A B,  of  the  triangle  ABC, 
be  260. 1,  the  side  AC  190. 5,  and  the  side 
B  C  140. 4  J  required  the  angles  A,  B,  and  C?    ^  . 


Digitized  by 


Google 


8PHBRICAL  TRlGONOIiETRY.  181 

The  side  A  B  =     260. 1 ' 

V  BC  =     140.4    Log.  ar.  comp.      .    •    7.852633. 
AC  =     190.5     Log.  ar.  comp,      .    .    7.720105 
•    «  ■  ■■■  « 

.  Sum  z=       .    •    .    591.0 

Half  sum  =  .    .    295.5    Log.  =       .    •    .    .    2.470558 
Remainder  =:      •      35.4     Log.  ==       ....     1.549003 


Sum  =     19.592299 


Arch  =    .    .    5H17'22r  Log.co^sine  =    .    .    9.796149§ 


Angle  C  =    .  l02?34U4r 

To  find  the  angle  B  :-^ 

As  the  side  AB   =     260.1     Log.  ar.  comp.  =    7.584860 

Is  to  the  angle  C  =102^34 '.44?  (jog.  sine  =     .    .    9.989448 

.  'SoisthesidcrAC  =2     190.5     Log.   =:       ...    2.279895 

To  the  angle  B=    45?37'45r  Log.   =      .    .    .    9.854203 

Now,  angle  C  102?34M4?  +  angle  B  45?37M5r  =   146n2^29r; 
and  180?  -  148?12^29r  =  3I?47-'3ir  =  the  angle  A. 


THE   RESOLUTION    OP   THE    DIFFERENT    PROBLEMS,    OR 
CASES,  IN  SPHERICAL  TRIGONOMETRY,  BY  LOGARITHMS. 

Spherical  Trigonometry  is  that  branch  of  the  mathematics  which  shows 
bow  to  find  the  measures  of  the  unknown  sides  and  angles  of  spherical 
triangles  from  some  that  are  already  known.  It  is  divided  into  three 
parts ;  viz.,  right-angled,  quadrantal,  and  oblique«> angled. 

A  right-angled  spherical  triangle  has  one  right  angle;  the  sides  in- 
cluding the  right  atigle  are  called  legs,  and  tt\at  opposite  thereto  the  hypo- 
thenose. 

A  quadrantal  spherical  triangle  has  one  side  equal  to  90?,  or  the  fourth 
part  of  a  circle. 

An  oblique-angled  spherical  triangle  has  neither  a  side  nor  an  angle  equal 
to  90? 

.   A  spherical  triangle  is  formed  by  the  intersection  of  three  great  circles 
on  the  surface  of  the  sphere. 


Digitized  by 


Google 


182  8PUBRICAI.  TRIGONOMBTRY. 

The  three  angles  of  a  spherical  triangle  are  always  more  than  two,  but 
less  than  six,  right  angles. 

The  three  sides  of  a  spherical  triangle  are  always  less  than  two  semi- 
eircles,  or  360? 

Any  two  sides  of  a  spherical  triangle,  taken  together,  are  greater  than  the 
third. 

The  greater  side  subtends  the  greater  angle ;  the  lesser  side  the  lesser 
angle,  and  conversely. 

Equal  sides  subtend  equal  angles,  and,  vice  versay  equal  angles  are  sub- 
tended by  equal  sides. 

The  two  sides  or  two  angles  of  a  spherical  triangle,  when  compared 
together,  are  said  to  be  alike,  or  of  the  same  affection,  when  both  are  less 
or  both  greater  than  90?  ;  but  wh^n  one  is  greater  and  the  other  less  than 
90?,  they  are  said  to  be  unlike,  or  of  different  affections. 

Every  side  of  a  right-angled  spherical  triangle  exceeding  90?,  is  greater 
than  the  hypothenus^ ;  but  every  side  less  than  that  quantity,  is  less  than 
the  hypothenuse. 

The  hypothenuse  is  less  than  a  quadrant,  if  the  legs  be  of  the  same 
affection ;  but  greater  than  a  quadrant,  if  they  be  of  different  affections: 

The  hypothenuse  is,  also,  less  or  greater  than  a  quadraQt,  according  as 
the  adjacent  angles  are  of  the  same  or  of  different  affections. 

When  the  hypothenuse  and  one  leg,  or  its  opposite  angle,  aie  of  the 
same  or  of  different  affections,  the  other  side,  or  its  opp6site  a^gle,  will 
be,  accordingly,  less  or  greater  than  a  quadrant. 

The  legs  and  their  opposite  angles  are  always  of  the  same  affection. 

The  sides  of  a  spherical  triangle  may  be  changed  into  angles,  and  con- 
versely. 

Every  spherical  triangle  consists  of  six  parts :  viz»,  three  rides  and  three 
angles  i  of  which,  if  any  three  be  given,  the  remaining  three  may  be  readily 
computed  3  but  in  right-angled  spherical  triangles,  it  is  sufficient  that  two 
only  be  given^  because  the  right  angle  is  always  known* 


SOLUTION  OF  RIGHT-ANGLED  SPHERICAL  TRUNGLES,  BY 
LOGARITHMS,  AGREfiABLY  TO  LORD  NAPIER'S  RULES* 

In  every  right-angled  spherical  triangle  there  are  five .  circular  partly 
exclusive  of  the  right  angle,  which  is  not  taken  into  consideration,  lliese 
five  parts  consist  of  the  two  legs,  or  sides  ;  the  complement  of  the  hypfh* 
thewuae;  and  the  cimplemenU  ^f  the  two  angles.  T^iey  are  caUed  circular 
parts^  because  each  of  them  is  measured  by  the  arc  of  a  great  circlet 


Digitized  by 


Google 


IPHB&JOAL  TRIOONOMBTRY*  188 

Hiree  of  these  circular  parts,  besides  the  radius,  enter  into  every  propor- 
tion ;  two  of  which  are  given,  and  the  third  required.  One  is  called  the 
Tttiddleparty  and  the  other  two  the  extremes  conjunct  or  disjunct. 

The  middle  part,  and  also  the  extremes  conjunct  or  disjuncty  may  be 
determined  by  the  foUowIng  rules. 

Mule  I.-— When  the  three  circular  parts  under  consideration  are  joined 
together^  or  follow  each  other  in  successive  order,  the  middle  one  is  termed 
Uie  middk  part,  and  the  other  two  the  extremes  conjunct,  because  they 
are  directly  conjoined  thereto. 

IhJe  2.-^When  the  three  circular  parts  do  not  join,  or  follow  each 
other  in  successive  order,  that  which  stands  alone,  or  disjoined  from  the  other 
two,  is  termed  the  middle  part,  and  the  other  two  the  extremes  disjunct, 
because  they  are  separated  or  disjoined  therefrom  by  the  intervention  of  a 
tfide^  or  an  angle  not  concerned  in  the  proportion* 

Ab/«,— In  determining  the  middte  part,  it  is  to  be  observed,^  that  the 
right  angle  does  not  separate  yr  disjoin  the  legs  :  therefore,  when  these 
^e  under  consideration,  they  are  always' to  follow  each  other  in  succession. 

These  things  befng  premised,  the  required  parts  are  to  be  competed  by 
the  two  following  equations }  viz., 

1st. — The  product  of  radius  and  the  sine  of  the  middle  part,  is  equal  to 
the  product  of  the  tangents  of  the  extremes  conjunct > 

2d/-»TA^  product  if  radius  and  the  sine  qfthe  middle  part,  is  equal  to 
the  product  of  the  co^sHmes  of  the  extremes  disjunct. 

Since  these  equations  are  adapted  to  the  complements  of  the  hypothec 
nvse  and  angles,  and  since  the  sine  or  thf  tangent  of  the  complement  of 
an  arch  is  represented  directly  by  the  co-sine  or  co-tangent  of  that  arch,-— 
therefore,  to  save  the  trouble  of  finding  the  complements,,  let  a  co-sine  or 
co-tangent  be  used  instead  of  a  sine  or  tangent,  and  a  sine  instead  o^  a 
co-sine,  &c.  &c.,  wheft  the  angles  or  the  hypothenuse  are  in  question. 

Now,  the  middle  part  being  determined  by  the  rules  1  or  2,  as  above, 
according  as  the  extremes  are  conjunct  or  disjunct,  the  terms  under  con- 
sideration are  then  to  be  reduced  to  a  proportion,  as  thus :— Put  the 
unknown  or  required  term  last,  that  with  which  it  is  connected^r^/,  and 
the  remaining  two  in  the  middle,  in  any  order;  this  being  done,  the  equa* 
tion  will  then  be  ready  for  a  direct  solution  by  logarithmical  numbers* 


/Google 


1 84  sraBRlCAL  TRICONOMBTIIT* 


Problem  L 

Given  the  Hypothevme  and  one  Leg,  to  find  the  Angles  and  the 

other  Leg. 

Example. 

Let  the  hypothenuse  A  B,  of  the  spherical  triangle  ABC, 
be  64^20M5r,  and  the  leg  AC  51  ?  10^57  ;  required  the' 
migles  A  and  B,  and  the  leg  B  C  ? 

To  find  the  angle  A :—    • 

Here  the  hypothenuse  A  B,  the  given  leg  A  C,  and  the  required  angle  A^ 
are  the  three  circular  parts  which  enter  the  proportion ;  and  since  the  angle 
A  evidently  connects  the  hypothenuse  and  the  given  leg^  it  is  ther^ore 
the  middle  parf,  and  the  other  two  the  eSDtremes  conjuncty  according  to 
rule  1,  page  183 ;  therefore,  by  equation  1,  page  183, 

Radius  x  co-sine  of  angle  A  =  tangent  of  A  C  x  co-tangent  of  A'B. 

Now,  since  radius  is  connected  with  the  required  term,  it  is  ta  be  the 
first  term  in  the  proportion.    Henc^, 

As  radius  =  ,  .  •  .  .  90?  0^  0?  Liog.  sine  ar.comp.=:l 0.000000 
Is  to  the  leg  A  C  =  «  «  5U  10. 13  Log.  tangent  =  ^  10. 094280 
So  is  the  hypothenuse  AB  ^  64. 20. 45     Log.  co-tangent  =    9. 681497 

To  the  angle  A  =:   ^    .    *    63*2 li 50:  Log.  co-sine  =    ^      9.775777 

2Vb^e.-*-The  angle  A  Is  acute,  because  the  hypothenuse  and  the  given 
leg  are  both  of  the  same  affection. 

To  find  the  angle  B :—     . 

The  three  circular  parts  which  enter  the  proportion,  in  this  case,  are  the 
hypothenuse  AB,  the  given  leg  AC,  and'  the  required  angle  B;  and  since 
the  leg  A  C  is  disjoined  from  the  other  two  parts  by  the  angle  A,  it  is 
therefore  the  tniddfe  party  and  the'  other  two  the  extremes  disjunct,  ac- 
cording to  rule  2,  page  183 ;  therefore,  by  equation  2,  page  183, 

Radios  X  sine  leg  AC  =^  sine  hyp.  AB  x  sine  of  angle  B. 

Now,  since  the  hypothenuse  is  connected  with  the  required  term,  it  is  to 
stand  first  in  the  proportion.    Hence, 


Digitized  by 


Google 


SPHBBI€4L  TRIGONOMBTRY.  185 

As  the  hypothenuse  A  B  =  64?20'  45 r  Log.  sine  ar.  comp.  =  1 0. 04507 1 

Is  to  radius  ==       ...     90.   0.   0     Log.  sine  =       .    .  10.000000 

SoistheIegAC=  .    .    51.10.15     Log.  sine  =r       .    .  9.891548 


To  the  angle  B=      .    .    59? 47. 34 r  Log. sine  =      .    .      9.93661^. 

^o<€.— The  angle  B  is  acute,  because  the  hypothenuse  and  the  giren  leg 
are  of  the  same  affection. 

To  find  the  leg  B  C  :— 

In  this  case  the  three  circular  parts  which  enter  the  proportion,  are  the 
hypothenuse  and  the  two  legs ;  and  since  the  hypothenuse  is  disjoined  from 
the  legs  by  the  angles  A  and  B,  it  is  the  middle  part,  and  the  other  two  are 
the  extremes  digunct ;  therefore, 

Radius  x  co-sine  hyp.  A  B  =  co*sine  leg  A  C  x  co-sine  leg  B  C. 

Now,*  the  leg  A  C,  being  connected  with  the  required  term,  is,  therefore, 
to  stand  first  in  the  proportion.  .  Hence, 

As  the  leg  AC  =:  .  .  51?10'15r  Log.  co-sine  ar.comp.=  10. 202732 
Is  ta  radius  =  .  .  .  90.  0.  0 '  Log.  sine  =  .  .  .  10.000000 
So  is  hypothenuse  A  B  =  64. 20. 45     Log.  co-sine  =r    ..    .      9. 836426 

To  the  leg  B  C  =     .    .    46?  19^ 52^/  ^og.  co-sine  =     .    .      9. 839158 

Note.-^The  leg  B  C  is  acute,  because  the  hypothenuse  and  the  given  leg 
are  of  the  same  affection. 


Problem  II. 

Given  the  Hypothenuse  and  one  Angle,  to  find  the  other  Angle  and 

the  tfvo  Legs. 

Example* 

Let  the  hypothenuse   AB,  of  the  spherical  triangle 
ABC,  be  jS6?44^35r,    and   the  angle  A  61*59^55^5      ^; 
required  the  angle  B  and  the  legs  A  C  and  B  C  ?  B 

To  find  the  angle  B  ;— 

Here  the  three  circular  parts  are  connected  or  joined  together;  there- 
fore the  hypothenuse  A  B  is  the  middle  part^  and  the  angles  A  and  B 
extremes  co^Junct  (rule  1,  page  183}  j  therefore,  by  equation  1,  page  183, 


Digitized  by 


Google 


186  mnuLiCAL  tiigoiioiibtey. 

Radius  x  co-nne  byp<  A  B  ±:  co-tangeht  angle  A  x  co-tangeikt  angle  B« 
Now^  the  angle  A,  being  connected  with  the  required  part,  ie  therefore 
to  stand  first  in  the  proportion.    Hence^ 

As  the  angle  A  =:  .  61?59C55r  Log.  co-tang.  ar.  comp.  =:  10. 27430O 
Is  to  radius  =  .  .  90.  0.  0  Log.  sine  =  ....  lO.OOOOOO 
Sols  the  hyp.  ABs:  66.44.35    Log.  co-sine    .    s    .    .     9.596438 

To  the  angle  B  =    .    53^24^  12r  Log.  co-tangent  =:    .    .      9. 870738 

Ao^e.— The  angle  B  is  acute^  because  the  hypothcnuse  and  the  given 
angle  are  of  the  same  affection. 

To  find  the  leg  A  Cl- 
in this  case^  the  three  circular  parts  are  joined  together ;  therefore  the 

angle  A  Is  the  nmddle  part^  and  the  bypothenuse  A  B  and  required  leg  A  C 

are  the  esr(remef  cofytmd;  therefore, 

Radius  x  co-sine  of  angle  A  :=  co-tangent  AB  x  tangent  AC*. 

And  since  the  bypothenuse  is  connected  with  the  required  part^  it  is 

therefore  to  be  the  first  term  in  the  proportion.    Hence, 

As  the  hyp.  AB  s  66?44:35r  Log.  eo-tang.  ar. comp.  =  10.366756 
Is  to  radius  =  .  .  90.  0.  0  Log.  sines  ....  lO.OOOOOO 
So  is  the  angle.  A  s    61.59.55.  Log.  cp-sitte    .    .    «    .      9.671629 


To  the  leg  AC  8   .    47?3U42r  Log.  tangent  s  .    .    »    la  038885 

Note,, — ^The  leg  A  C  is  acute,  because  the  bypothenuse  and  the  given 
angle  are  of  the  same  affection. , 

To  find  the  leg  BC:— 

In  this  case  the  leg  B  C  is  the  middle  part,  because  it  stands  alone,  or 
is  disjoined  firoift  the  other  two  circular  parts  concerned,  by  the  angle  B  : 
hence  the  hypothenuse' AB  and  the  given  angle  A  are  extremes  disjunct, 
according  to  rule  2,  page  183 ;  therefore,  by  equation  2,  page  183, 

Radius  x  sine  of  leg  B  C  ==  sine  of  hyp.  A  B  x  sine  of  angle  A. 

And  since  radius  is  connected  with  the  required  part,  it  is  to  be  the  first 
term  in  the  proportion.    Hence, ' 

As  radius  =s  ...  90?  Of  Or  Log.  sinear.  ebmp.  s=  lO.OOOOOO 
IstohypothenuseABs  66.44.35  Log.  sine  s  .  '.  •  9.963194 
So  is  the  angle  A  <=    .    61 .  59. 55    Log.  sine  =     ...      9. 945929 


To  the  leg  BC  a      .    54n2:45r  Log.  sine  a     .    .    .      9.909123 

No(«.— -The  leg  BC  is  acute,  because  the  bypothenuse  and  the  given 
angle  are  of  the  same  affection. 


Digitized  by 


Google 


SPHJiRtCAL.  TAtGOKOMBTKY*  187 

Problem  IIL 

Given  a  Leg  and  its  opposite  Angle,  to  find  the  other  Angle,  the  other 
Legj  mid  the  Hypothenuse. 

Example. 

Let  the  leg  AC,  of  the  spherical  triangle  ABC,  be 
56?30M0^  and  the  angle  B  70?23:35r;  required  the  angle 
A,  the  leg  B  C,  and  the  hypothenuse  A  B  ? 

.  To  find  the  angle  A  :*-• 

Here  the  three  circular  parts  which  enter  the  proportion,  are  the  given 
angle  B,  the  given  leg  A  C,  and  the  required  angle  A ;  and  since  the  angle 
B  is  disjoined  from  the  other  two  parts  by  the  intervention  of  the 
hypothenuse  '  AB,  itT  is  the  middle  part,  and  tlie  other  two  are  the 
extremes  disjunct,  according  to  rule  2,  page  183  ;  therefore,  by  equation  2, 
page  183, 

Radius  X  co-stne  of  the  angle  B  s  sine  of  the*  angle  A  x  co^sine  of 
the  leg  AC. 

And  since  A  C  is  connected  with  the  required  part,  it  is  to  be  the  first 
tern  in  tfie  proportion.    Hence, 

As  the  leg  A  C  s  56?30(.40r  Log.  co-sine  ar.  comp.  a  10. 258238 
la  to  radiuses  .  SO.  0.  0  Log.  sine  =  ....  10.000000 
8obtlieamteBaB70«2d.85        Log«co«sine    •    .    .    •      9.525778 


To  the  angle  A=  {  i32!32!37 '  }   ^**  ^^^^  =    .    .    .    .      9. 784016 

^oto.— The  angle  A  is  ambiguous,  since  it  cannot  be  determined,  from 
the  parts  given,  whether  it  is  acute  or  obtuse. 

TofindtheiegBC:— 

Hie  three  circular  parts  concerned  in  this  case,  are  the  legs  A  C  and  B  C, 
and  the  given  angle  A ;  and  since  the  right  angle  never  separates  the  legs, 
B  C  is  the  middle  part,  and  A  C  and  the  angle  B  are  the  extremes  conjunct, 
by  rule  1,  page  183)  therefore,  by  equation  !»  page  183, 

Radius  X  sine,  of  the  leg  B  s  tangent  leg  AC  x  co-tangent  angle  B* 
Now^  since  radius  is  connected  with  the  required  term^  it  is  to  stand  first 
io  the  pmportaen.    Hence, 


Digitized  by 


Google 


As  radius  ==  •  .  90?  0^  0?  Log.  sine  ar.  coup.  =  10.000000 
Is  to  the  leg  A  C  =  56. 30. 40  Log.  tangent  =  .  .  10. 179400 
So  is  the  angle  B  s     70. 23. 35         Log.  co-tangent  ^    .      9. 55 17 19 


To  the  leg  B  C  =  {  iJy.' 25,' 27 '  }   ^8-  "^«  =      •    •     •      9-  731 1 19 

No^e.— «The  leg  B  C  is  ambiguous^  since  it  cannot  be  determined^  from 
the  parts  given,  whether  it  is  acute  or  obtuse. 

To  find  the  hypothenuse  A  B  :— 

Here  the  given  leg  A  C  is  the  middle  part,  because  it  is  disjoined  from 
the  other  two  circular  parts  concerned^  by  the  intervention  of  the  angle  A: 
hence  the  angle  B  and  the  hypothenuse  AB  are  e^emes  dujjunct ;  diere- 
fore, 

"Radius  x  sine  of  leg  A  C  =  sine  of  hyp.  AB  x  sine  of  imgle  B. 

And  since  the  angle  B  is  connected  with  the  required  term>  it  is  to  stand 
first  in  the  proportion.     Hence, 

As  the  angle  B  =  •  .  70?23 '33 r  Log.  sine  ar.  comp,  =  10. 02594 1 
Is  to  the  leg  A  C  =  ^  .  56. 30. 40  Log.  sipe  s  *  •  •  9. 92 1 162 
So  is  radius  =     .    •    .  *90.  0.   0    Log.  sine  a    •    •    .    10.000000 

To  the  hyp.AB  =  Ij^^'J^'^*  jLog.si^^        •    ,    .      9.947103 


Noto.— The  hypothenuse  AB  is  amlnguous;  that  is,  it  may  be  either 
acute  or  obtuse,  from  Jtl^e  parts  given. 


Paoblbm  IV. 

Given  a  Leg  and  its  adjacent  Angle,  to  find  the  other  ybigle,  the  other 
Leg,  a^id  the  Hypothenuse. 

Example.  Pv^^ 

Let  the  leg  AC,  of  the' spherical  triangle  ABC,  be         /    ^.  >^ 
68?29M5r,  and  the  angfe  A  74?45a5r  5  required  the    .  *  '      '^  ^'^ 
angle  B,  the  leg  B  C,  and  the  hypothenuse  AB  ? 


SPHERICAL  TRIGONOMBTRT.    '  189 

To  find  the  Angle  B  :— 

Here  the  circular  parts  concerned  are,  the  leg  A  C,  the  given  angle  A, 
and  the  required  angle  B;  and  since  the  angle  B  is  disjoined  from  the 
other  two  parts  by  the  hypothenuse  A  B,  it  is  the  middle  part,  and  the 
other  two  are  the  estremei  disjunct,  by  rule  2,  page  183;  therefore/  by 
equation  2,  page  183, 

Radius  X  co-sine  angle  B  =  sine  of  angle  A  x  co-sine  leg  A  C. 

Now,  since  radius  is  connected  with  the  cfquired  term,  it  is  to  stand 
first  in  the  proportion.     Hence, 

Asradms=    .    .    90?  0'  Or     Log,  sine  ar.  comp.  =  lO.OOOOO* 

.  Is  to  the  angle  A  =  74. 45. 15       Log.  sine  =    .     .    .      9. 984440 

So  is  the.leg  A  C  »  68. 29. 45       Log.  co-sine  =    •     •      9. 564156 

Tothe-angleB=     69?17'.17^     Log.  co-sine  =    .    .      9.548596 

JNb^.— The  angle  B  is  «cute,  or  of  the  same  affection  with  its  opposite 
given  leg  A  C. 

'     To  fin4  the  Leg  B  C  :— 

III  .this  case,  since  the  right  af^gle  never  separates  the  legs,  the  three 
^circular  parts  are  joined  together :  hence  the  leg  A  C  is  the  middle  part, 
and  the  leg  B  C  and  .the  jangle  A  arQ  the  extremes  conjunct,  according  to 
rule  1,  page  183 ;  therefore,  by  equation  1,  page  183, 

Radius  x  sine  of  leg  A  C  =  co-tangent,  angle  A  X  tangent  of  leg  B  C. 

And  fiince  the  angle  A  is  connected  with  the  required  part,  it  is  to  be  the 
first  teem  in  the  proportion.    Hence, 

As  the  angle  A  =s  74?45'  15?  Log.  co-tang.  bx.  comp.  =.  10. 564549 
Id  to  radius  =  .  .  90.  0.  0  Log.  sine  =  ....  10.000000 
So  is  the  leg  A  C  =    68. 29. 45      Log.  sin^  =     .     .    .    .      9. 968666 

TothelegBC«     73 ?40^20ir  Log.  tangent  =s    .     .    .     10.533215 

Note* — ^Thei^leg  B  C  is  acute,  or  of  the  same  affection  with  its  opposite 
given  angle  A.  . 

To  find  the  Hypothenuse  A  B  :— 

In  this  case,  since  the  three  circular  parts  which  enter  the  proportion 
^e  joined  together^  the  given  angle  A  is  the  middle  part,  and  the  leg  A  C 
and  the  hypothenuse  A  B  are  the  extremes  conjunct :  therefore, 

Radius  X  co-sine  of  angle  A  =  Ungent  of  leg  A€  x  co-tangent.hypo- 
then'useAB, 


Digitized  by 


Google 


190  $?BBaiCAL  TaiGQVOMnftT* 

Now,  the  leg  AC,  being  connected  with  the  required  part,  Is  therefore 
to  be  the  first  term  in  the  proportion.    Hence, 

As  the  leg  A  C  =  .  .  68?29'45r  Log,  tang,  ar,  comp.  =  9. 595490 
Is  to  radius  =  ...  90.  0,  0  Log.  sine  =  .  .  •  10.000000 
So  is  the  angle  A  =     .    74. 45. 15       Log.  co-sine  =  .    •      9. 419891 

To  the  hypothenuse  A  Cs  84?  5i  6r    Log.  co-tangent  a      9. 015381 

Note.'^^The  hypothentise  v  acute,  because  the  given  leg  and  angle  are 
of  the  same  affection. 


.    Peoblbm  V. 
Gwen  the  two  Legiy  to  find  the  Aagle$  and  the  H^fothemue. 

.  A- 

Example. 

Let  the  1^  A  C,  of  the  spherical  triangle  A  B  C,  be 
70?10:20r,  and  the  legBC  76?38M0r;  required  the 
angles  A  and  Bi  and  the  faypoth^nuse  AB  ? 

To  find  the  Angle  A  :— 

tiere,  since  the  right  angle  woer  9eparat€9  the  legs,  the  l^g  A  C  is  the 
middle  part,  and  the  leg  B  C  and  the  jequired  angle  A  are  the  e&itemeg 
conjunct,  agreeably  to  rule  1,  page  183 ;  therefore,  by  equation  1,  page  183, 

Radius  X  sine  leg  A  C  s  tangent  leg  B  C  x  co-tangent  angle  A. 

Now,  since  the  leg  B  C  is  connected  with  die  required  part,  it  is  to  be 
the  .first  term  in  the  proportion.    Hence, 

AsthelegBC  »  76?88140r  Log.  tangent  ar.  comp.  «  9.375506 
Is  to  radius  =s  •  90.  0.  0^  Log.  sine  s=  •  •  .  .  .  10. 000000 
So  is  the  leg  A  C  «70. 10. 20      Log.  sine  =....!      9. 973459 

To  the  angle  A  ==    77?24'37^    Log.  co-tangent  =    .    .      9.348965 

JVb/e.— The  angle  A  is  acute^  or  of  the  same  affection  with  its  oppoaite 
given  legBC, 

To  find  the  Angle  B  :— 

In  this  case  the  leg  BC  is  the  middle  part,  and  th^  leg  AC  and  the 


Digitized  by 


Google 


8PMRICAL  TRI430NOMBfF&T.  191 

required  angle  B  are  the  extremes  conjunct,  according  to  rule  1,  page  183 ; 
therefore,  by  equation  1,  page  183, 

Radius  x  sine  of  the  leg  B  C  s=  tangent  of  leg  A  C  x  co-tangent 
angle  B. 

And  since  the  leg  A  C  is  connected  with  the  required  part,  it  is  to  be  the 
first  term  in  the  proportion*    Hence^ 

As  the  leg  A  C  =  70?  10^  20r  Log.  tangent  an  comp.  =  9. 556990 
Is  to  radius  =  •  .  90.  0.  0  Log.  sine  ==  •  •  •  •  10.000000 
So  is  the  leg  B  C  =:  76. 38^40      Log.  sine  =      .    «    .    .      9. 988093 

To  the  angle  B  =:    70?40!  Si':  Log.  co-tangent  =:    .    •      9. 545083 

No/e.— The  angle  B  Is  aeute^  or  of  the  same  a&ction  with  its  opposite 
pvnlegAC. 

To  find  the  Hypothenus^  A  B  :-^ 

Here  the  hypotheniise  AB  is  the  middle  part,  because  it  Is  disjoined 
from  the  legs  by  the  angles  A  and  B:  hence  AC  and  BC  are  exiremes 
digjuncty  agreeably  to  rule  2,  pagq  183 ;  therefore,  by  equation  2,  page  183> 

Radius  X  co-sine  hypotbanuae  A9  =:  co-sine  kg  AC  x  co-sitie  leg  BC. 

4Qd  radius^  being  connected  with  the  middle  par^  b  therefore  to  be  th» 
fijst  term  in  the  proportion.    Hra^e^ 

As  radius  =  .  .  90?  Ot  0?  Log.  sine  ar.  comp.  i=:  •  .  10.000000 
Is  to  the  leg  AC  =:  70. 10. 20  Log.  co-sine  =  .  .  •  9. 530448 
So  b  the  leg  B  C  =  76. 38. 40      Log.  co-sine  ==      ...      9. 363599 

Tothehyp.AB  =  85?30^22?     Log.  co-sine  ;=      •    .    .      8.894047 

Hoiee-^Tht  hypothenuse  AB  b  acute^  because  the  given  legs  AC  and 
B  C  are  of  the  same  affection. 


Peoblsm  VI. 

Gwenthe  tu>o  Jngles,  to  find  the  Hypothenuse  and  ilie  two  Legs. 

ExamplSm 

Let  the  angle  A,  of  the  qpherical  triangle  ABC,  be 
50?  10r20%  and  the  angle  B  64?20^25f ;  r«|mred  Ibe  lege 
ACandBC,  and  the  hypothenuse  A B ?  H 


Digitized  by 


Google 


192  SPHERICAL  TRIGONOMBTRY. 


To  find  the  Hypothenuse  AB  :—    • 

Here,  because  the  three  circular  parts  are  joined  together^  the  hypothe- 
nuse AB  is  the  middle  part,  and  the  angles  A  and  B  are  the  extremes 
coigunct,  agreeably  to  rule  1,  page  183  5  therefore,  by  equation  1,  page  183^ 

Radius  x  co-sine  hypothenuse  A  B  1=  co-tangent  angle  A  x  co-tangent 
angle  B. 

Now,  since  radius  is  connected  with  the  required  part,  it  is  to  be  the 
^  first  term  in  the  proportion,    Hence^  . 

As  radius  =  .  .  •  90?  O:  Or  Log,  sine  ar.'  comp.  *=  10.000000 
Is  to  the  angle  A  s=  50. 10.  20  Log.  co-tangent  =  .  9. 921 161 
So  is  the  angle  B  =     64.20.25       Log.  co-tangent    =     .      9.681605 

To  the  hyp.  AB  =     66*?22'.52r     Log.  co-sine  =       .    .      9.602766 

Note,, — The  hypothenuse  A  B  is  acute,  because  the  given  angles  A  and 
C  are  of  the  same  aiSfection. 

To  find  the  leg  AC:— 
f 
Here,  since  the  angle  B  is  disjoined  by  the  hypothenuse  A  B  from  the 
dther  two  circular  parts  concerned,  it  is  the  middle  part ^  and  the  angle  A 
and  the  required  leg  A  C  are  the  extremes  disjtincty  agreeably  to  rule'  2, 
page  183  5  therefore,  by  equation  2,  page  183, 

Radius  x  corsine  angle  B  =  sine  of  angle  A  X  po-sine  of  leg  AC. 
And  because  the  angle  A  is  connected  with  the  required  part,  i<  Is  to 
stand  first  in  the  proportion.  .  Hence, .     • 

As  the  angle  A  =  Stt?  10' 20^  Log.  sine  ar.  comp.  =  .  10. 1 14654 
Is  to  radius  =  .  90.  0.  0  Log.  sine  ==  ....  lO.OOOOOO 
So  is  the  angle  B  =  64. 20. 25       Log.  co-sine     ....      9. 6365 14 


TothelegAC   =     S5M0'38r     Log.  co-sine  =:     .    .    .      9.751168 

^ofe.-^The  leg  A  C  is  acute,  or  .of  (he  same  affection  with  its  opposite 
given  angle  B. 

To  find  the  Leg  B  C  :— 

In  this  case  the  angle  A  is -the  middle  part,  because  it  is  disjoined  from 
the  other  two  circular  parts  by  the  hypothenuse  A  B  i  hence  .the  angle  B 
and  the  required  leg  B  C  are  extremes  disjwictj  therefore,. 

Radius  x  co-sine  of  angle  A  =  sine  of  angle  B  x  co-sine  of  leg  B  C. 

And  as  the  angle  B  is  connected  with  the  required  part,  it  is  to  be  the 
first  term  in  the  proportion.    Hence,  *  .     * 


Digitized  by 


Google 


SPHSmCAL  TRt  GONOIIBTRY.  1 93 

As  the  angle  B  =     64?20^45r     Log.  sine  ar.  comp*  =     .  10. 045091 

Is  to  radius  =      .    90.   0.   0      Log,  sine  =     ....  10.000000 

So  is  the  angle  A  =  50. 10.  20      Log.  co-sine 9. 806507 

TothelegBCzz    44?43niir    Log.  co-sine  = 9.851598 

Note. — ^The  leg  B  C  is  acute^  or  of  the  same  affection  with  its  opposite 
given  angle  A.  - 


SOLUTION  OP  QUADRANTAL  SPHERICAL  TRIANGLES, 
BY  LOGARITHMS. 


Problem  I. 

Given  a  Quadrantal  Side,  its  opposite  jingle^  and  an  adjacent  Angle,  to 
find  the  renuwiing  Angle  and  the  other  two  Sides, 

'  IZemarlr.— Since  the  sides  of  a  spherical '  triangle  may  be  turned  into 
angles,  and,  vice  versa,  the  angles  into  sides,  all  the  cases  of  quadrantal 
spherical  triangles  may^  be  re<K>lved  agreeably  to  the  principles  of  right- 
angled  spherical  triangles ;  as  thus  :  let  the  quadrantal  side  be  esteemed 
the  radius ;  the  supplement  of  the  angle  subtending  that  side,  the  hypo- 
thenuse ;  and  the  other  angles  legs,  or  the  legs  angles,  as  the  case  may  be. 
Then  the  middle  part,  and  the  extremes  conjunct  or  disjunct,  being  esta- 
blished, the  required  parts  are  to  be  computed,  and  the  affections  of  the 
angles  and  sides  determined,  in  the  same  manner  precisely  as  if  it  were  a 
right-angled  spherical  trijangle  that  was  under  consideration. 

Example. 

Let  AB,  in  the  spherical  triangle  ABC,  be  the  qua- 
drantal side  =  90?,  the  angle  C  120?19^30^,  and  the 
angle  A  47?d0^  20r ;  required  the  sides  A  C  and  B  C^  and 
the  angle  B? 

Solution. — Let  the  supplement  of  the  angle  C  (59M0'30^),  subtending 
the  quadrantal  side  A  B,  represent  the  hypothenuse  a  6  of  the  dotted 
spherical  triangle  abc.  Let  the  given  angle  A  47^30' 20T  represent  the 
leg  i6  c  of  the  said  dotted  triangle,  and  the  required  angle  B  the  leg  a  c. 


Digitized  by 


Google 


ThePi  in  the  rigfat-uigled  spherical  triangle  a  be,  given  the  ^ 

hypothennac  ab  59°40C30r,  and  the  leg  i5c  47?30120^,  to  |/\ 

find  the  1^  a  c  =  the  angle  B  in  the  quadrantal  triangle ;  the  ^/     \ 

angle  «  ■=  the  leg  B  C,  and  the  angle  i  =  the  leg  A  C,  of  the  ^  <^         \  ^ 

said  quadranul  triangle.  '^'Si'^^- 

To  find  the  Leg  ac  ==  the  Angle  B  in  the  Quadrantal  Triangle :~ 

Here  the  hypothenuse  ab  is  the  middle  party  and  the  legs  &c  and  ac 
are  the  extremes  diejunct  -,  therefore^ 

Radius  x  co-sine hyp.  abzz  co-sine  leg  & c  x  co-sine  legac. 

Now»  lince  the  leg  6  c  is  connected  with  the  required  part,  it  is  to  be  the 
first  term  in  the  proportion.    Hence^ 

As  the  leg  6 c  =  47?30' 20r  Log.  co-sine  ar.  comp.  =  .  10. 170363 
Is  to  radius  =:  •  90.  0.  0  Log.  sine  =:  •  •  .  .  10. 000000 
So  u  the  hyp.  a  6=  59. 40. 30      Log.  co-sine  =  ,    ,    .    .      9. 703209 

Tothelegac  ;;;    41?97'54?    Log.  co-sine  r:  .    .    •    .      9.873572 

Noie.'^^Tht  leg  a  c  is  acute,  because  thfc  hypothenuse  and  the  given  leg 
are  of  the  same  dfcction :  hence  the  angle  B  (in  the  quadrantal  triangle), 
represented  by  the  leg  ac,  is  also  acute  »  4l?37'54' 

To  find  the  Angle  a  =  th^  Leg  B  C  -in  tb^  Quadrantal  Triangle  ^^ 

Here  the  leg  b  c  is  the  middle  party  and  the  hypothenuse  a  b  and  angle  a 
are  the  extremes  di^unct ;  therefore, 

Radius  x  sine  of  leg  be^  sine  of  hypothenuse  ab  x  sine  of  the 
angle  a. 

And  since  the  hypothenuse  is  connected  with  the  required  part,  it  is  to 
be  the  first  term  in  the  proportion.    Hence, 

As  the  hyp.  a  c  =  .  59?40f  SOr  Log.  slne.ar.  comp.  =  10. 068901 
Is  to  radius  =  .  .  90.  0.  0  Log.  sine  =  ...  10.000000 
So  is  the  leg  J  c  =    „   47. 80. 20      Log.  sine  =     .    .    .      9. 867670 

To  the  angle  a  =     •    58940C26r     Log.  sine  =     ...      9.931571 

JNb^e.— The  angle  a  is  acute,  becduse  the  hypothenuse  and  the  gi?en 
leg  are  of  the  same  afiection :  hence  the  leg  BC  (of  the  qu^dn^ntal 
Uiangle)^  represented  by  th«  an^le  a,  is  also  acut«  ^  53?40  C  26r 


SPHBEICAt  TRIOONOMBTEY*  195 

To  find  the  angle  b  =  the  Leg  A  C  in  the  Quadrantal  Triangle :~ 

In  this  case  the  angle  b  is  the  middle  part,  and  the  hypothenuse  ab  and 
the  leg  b  c  are  the  exir^es  conjunct ;  therefore, 

Radius  x  co-sine  of  the  angle  b  =  co-tangent  hypothenuse  ab  x  tan- 
gent of  leg  b  c. 

And  radius^  being  connected  with  the  required  part|  is,  therefore,  to 
stand  first  in  the  proportion.    Hence, 

As  radius  =  ,  .  .  .  90?  0'  OT  Log«  sine  ar.  comp.  =  10. 000000 
Is  to  the  hyp.  a  i  =  ,59. 40. 30  Log.  co-tangent  =:  9. 7671 10 
So  is  the  leg  ic=     .    .    47.30,20    Log.  tangent  ;;:    •    10.038032 

To  the  angle  A  -     ,    .    50?19'.19f  Log«co*Mne  =:  .    t      9.805142 

iVbto.-— llie  angle  b  is  acute,  because  the  hypothenuse  and  the  given  leg 
are  of  the  same  affliction.  Hence,  tke  leg  A  C  (of  the  quadrantal  triangle), 
repreMnted  by  the  angle  £,  is  also  acute  s  50?  19'  19?. 


Problem  II. 

Qxom  the  QuadraaiUal  &de  and  the  other  two  Sides,  to  find  the  three 

Angles. 

Example. 

Let  AB,  in  the  spherical  triangle  ABC,  be 
the  quadrantal  side  =  90? ;  the  side  A  C, 
U5?19M$f;  and  the  side  BC,  I17?39:35r: 
required  the  angles  A,  B,  and  C  ? 

jSb2ti/ton.— Let  the  angle  c,  iii  the  dotted 
spherical  triangle  a  i  c,  be  radius,  and  represent 
the  side  AB  =  90?  of  the  quadrantal  triangle 
ABC.  Let  the  angle  a,  of  the  dotted  .triangle, 
represent  the  side  B  C  of  the  quadrantal  triangle 
r=  I17?39'45r,  and  let  the  angle  £  represent 
the  side  A  C  of  the  said  quadrantal  triangle  == 
115?19M5r«  Then,  in  the  right-angled  sphe- 
rical triangle  a  be,  right-angled  at  c,  given  the  angle  a  =:  117?39^35^, 
and  the  angle  £  =  115?19'45^,  to  find  the  hypothenuse  a  i,  the  leg  ac, 
and  the  leg  i  c;  the  first  of  which  represents  tht  supplement  of  the  angle 

o2 


Digitized  by 


Google 


196  sraBRlCAL  TRIGOKOBfBTRT. 

C  opposite  to  the  quadrantal  side  AB^  in  the  triangle  ABC;  the  second 
represents  the  an^le  B ;  and  the  third  the  angle  A,  in  the  said  quadrantal 
triangk» 

To  find  the  Hypothenuse  a  i  =  the  Supplement  of  the  Angle  C^ 
subtending  the  Quadrantal  Side  A  B :— > 

Here  the  hypothenuse  a£  is  the  middle  part,  and  the  given  angles  a  and 
h  are  the  extremes  conjunct ;  therefore, 

Radius  X  co-sine  hypothenuse  ab  =  co-tangent  of  angle  a  x  co-tan- 
gent of  angle  6.-^Now,  since  radius  is  connected  with  the  required  part,  it 
is  to  be  the  first  term  in  the  proportion. — Hence, 

As  radius  =  .    .    .    90?  OC  Or  Log.  sine  ar.  coropt.  =  10.000000 
Is  to  the  angle  a  =*    117.39.35     Log.  co-tangent  =  9.719427 

So  is  the  angle  6  =  115. 19. 45     Log.  co-tangent  =      ;    9. 675 156 

To  the  hypo.  ai=     75?38nir  Log.  co-sine  =  .    .    .    9.394583 

JNb<e.— The  hypothenuse  ai  is  acute  because  the  given  angles  are  of  the 
same  affection :— but  since  it  only  represents  the  supplement  of  the  angle  C ; 
therefore  the  angle  C  is  obtuse,  or  104'?21'49r. 

To  find  th^e  Leg  ac  =  the  Angle  B  in  the  Quadrantal  Triangle. 

The  angle  b,  in  this  case,  is  the  middle  part, .  and  the  angle  a  and  leg  a  c 
extremes  disjunctj^Thevefore,  radius  x  co-sine  of  angle  b  =  sin^s  of  angle 
a  X  co-sine  of  leg  ac. 

And  the  angle  a  being  connected  with  the  required  part,  is,  therefore,  to 
be  the  first  term  in  the  proportion. — Hence, 

As  the.angle  a  =  117''^l9^35r  Log.  sme  ar.  compt.  =  10.052703 
Is  to  radius  =.  .  90.  0.  0  Log.  sine  =  .  .  .  10.000000 
So  is  the  angle  is  115.19.45     Log.  eo-sine  =     .    .      9. 63 1 259 


To  the  side  a c  S3    118952^57^  Log.  co-sine  =    .    .        9.683962 

Note. — The  side  ac  is  obtuse,  or  of  the  same  affection  with  its  opposite 
angle  b : — ^and  since  a  c  represents  the  angle  B  ;  therefore  the  angle  B,  in 
the  quadrantal  triangle,  is  obtuse,  or  1 18?52'57r. 

To  find  the  Leg  £  c  =  the  Angle  A  in  the  Quadrantal  Triangle. 

In  this  case  the  angle  a  is  the  middle  part^  and  the  angle  b  and  leg  i  c 
extremes  dw/tmct.— ^Therefore,  radius  x  co-sine  of  the  angle  a  =  sine  of 
the  angle  b  X'  co-sine  of  the  leg  b  c« 


Digitized  by 


Google 


SPHERICAL  TRIGONOMETEY.  197 

And  since  the  angle  b  is  connected  with  the  required  part,  it  is  to  be  the 
first  term  in  the  proportion.— Hence, 

As  theangle  b  =  115?19M5r  Log.  sine  ar.  compt.  =  10.043896 
Is  to  radius  =  .  .  90.  0.  0  Log.  sine  s  .  .  •  .  10. 000000 
So  is  the  angle  a  =  11 7. 39. 35    Log.  co-sine  =  .    .    .    9. 666723 


To  the  leg  i  c  :=     120^54^  \2t  Log.  co-sine  =  .    .    .    9. 710619 

Vote. — The  leg  i  c  is  obtuse,  or  of  the  same  affection  with  its  opposite 
angle  a: — and' since  the  leg  b  c  represents  the  angle  A,  in  the  quadrantal 
triangle;  therefore  the  angle  A  is  obtuse,  or  120?54^  12?. 

jRtfmorJIr.— From  the  ample  solutions  of  the  two  preceding  Problems,  it 
must  appear  obvious,  that  all  the  cases  of  quadrantal  spherical  triangles 
may  be  easily  resolved  by  the  equations  for  right-angled  spherical  triangles. 
And  if  the  analogies  of  those  two  Problems  be  well  understood,  all  the  ap- 
parent  difficulty  attending  the  trigonometrical  solution  of  quadrantal  trian- 
gles will  entirely  vanish. 


SOLUTION  OF  OBLIQUE-ANGLED  SPHERICAL  TRUNGLES 
BY  LOGARITHMS. 

The  most  natural,  and,  perhaps,  th^  easiest  method  of  solving  the  four 
first  ProblemSy  or  cases  of  oblique-angled  spherical  triangles,  is  by  means 
of  a  perpendicular  let  fall  from  an  angle  to  its  opposite  side,  continued  if 
necessary ;  and  thus  reducing  the  oblique  into  two  right-angled  spherical 
triangles.— The  perpendicular,  however,  should  be  let  fall  in  such  a  manner 
that  two  of  the  given  parts  in  the  oblique  triangle  may  remain  known  in 
one  of  the  right-angled  triangles  : — ^Then,  the  other  parts  may  be  readily 
computed  by  means  of  Lord  Napier's  analogies,  as  given  in  the  equations  1 
and  2,  page  183. — ^But,  since  the  solution  of  oblique-angled  spherical  trian- 
gles without  a  perpendicular  is  possessed  of  many  advantages  in  astrono- 
mical calculations ;  and,  besides,  since  the  author's  object  is  to  establish 
the  use  of  the  Tables  contained  in  this  work  by  a  variety  of  rules  and  for- 
mulae which,  it  is  hoped,  may  not  be  found  quite  uninstructive  to  persons 
but  slightly  informed  on  trigonometrical  subjects ;  the  different  cases  of  ob- 
lique triangles  will,  therefore^  be  resolved  independently  of  a  perpendicular, 
agreeably  to  the  propositions  generally  used  in  such  cases. 


Digitized  by 


Google 


108  SPHXRICAL  TRIGONOMirrET. 


Problem  I. 


ISiven  Ttjoo  Sides  of  an  Oblique-angled  Spherical  Triangle,  and  an  jingle 
opposite  to  one  of  them;  to  find  the  remaining  Angles  and  the  Third 
S&te. 

RUUE. 

l.^Tbfind  on  angle  opposite  to  one  of  the  gioen  sides. 

As  the  log.  sine  of  the  side  opposite  to  the  given  angle,  is  to  the  log.  sine 
of  the  given  angle ;  so  is  the  log.  sine  of  the  other  given  side^  to  the 
log.  sine  of  its  opposite  angle. 

Now,  to  know  whether  the  angle  thus  found  is  determinate ;  that  ie,' 
whether  it  is  ambiguous,  acute,  or  obtuse,  proceed  in  the  following  man- 
ner, viz.<^To  the  angle  so  found,  and  its  supplement,  add  the  given  angle^ 
or  tl)at  used  in  the  prop6rtion. — ^Then,  if  each  of  these  sums  be  of  the  same 
affeciim  with  respect  to  180?  as  the  stun  of  the  two  given  sides^  or  those 
used  in  the  proportion,  the  angle  is  amAigtiottf  ;  that  is,  it  may  be  either 
acute  or  obtuse ;  and,  therefore,  indeterminate.*^But,  if  those  sums  are  of 
different  affections  with  respect  to  the  sum  of  the  sides,  the  angle  is  deter- 
minate, and,  therefore,  not  ambiguous : — ^In  this  case  that  value  of  the 
angle  is  to  be  taken,  whether  acute  or  obtuse,  which,  when  added  to  the 
given  angle,  produces  a  quantity  of  the  same  affection  with  the  sum  of  the 
two  sides. 

2.-^Tofind  the  angle  contained  between  the  two  given  sides. 

Find  half  the  difference,  and  half  the  sum  of  the  two  given  sides  :--find^ 
also,  half  the  difference  of  their  opposite  angles.    Then  say, 

As  the  log*  sine  of  half  the  difference  of  the  sides,  is  to  the  log.  sine  of 
half  their  sum ;  so  is  the  log,  tangent  of  half  the  difference  of  their  oppo« 
site  angles,  to  the  log.  co-tangent  of  half  the  angle  contained  between  the 
two  given  sides ;  the  double  of  which  will  be  the  angle  sought. 

3.— To  find  the  third  side. 

Since  the  sides  are  proportional  to  the  sines  of  their  opposite  angles  | 
therefore  the  third  side  may  be  found  by  the  converse  of  the  first  part  of 
the  rule ;  as  thus  : 

As  the  log.  sine  of  a  given  angle  opposite  to  a  given  side,  is  to  the  log, 
sine  of  that  side ;  so  is  the  log.  sine  of  the  given  angle  opposite  to  the  re- 
quired side,  to  the  log.  sine  of  the  required  side« 

Note. — ^When  the  angle  comes  out  ambiguous,  or  indeterminate,  in  the 
first  proportion ;  the  contained  angle  and  the  third  side,  found  by  the  other 
proportions,  ^^11  also  be  ambiguous. 


Digitized  by 


Google 


8P»fia(CAt  tniQoKoittrHY.  199 


Example. 


In  the  oblique-angled  spherical  triangle  A  B  C^  let 
the  side  A  B  be  74?59^50r,  the  side  B  C  68?10:30r 
and  the  angle  A  63?58'.32f ;  required  the  angles  B 
and  C^  and  the  side  AC? 

To  find  the  Angle  C  :— 

As  the  side  BC  =  68?10f30r  Log.  sine  ar.  compt.  2=  10.032301 
Is  to  the  angle  A  a  63.58.32  Log.  sine  .  •  .  =  9.953570 
SoisthesideABs  74.59.50    Log.  sine     .    .     ,     s    9.98493^ 


To  the  angle  C«     69?  13^37^  Log.  sine     .    .    ,    =    9.970809 


To  determine  whether  the  Angle  C  is  Ambiguous,  Acute,  or  Obtuse : — 

Angle Ctt69?  13^37:  Sap.  »  1 10^46^ 23?  SideBC»  68?]0C30r 
AngleA S3 63.58.32  AngUA   »  68.58.33    SideAB»    74.59.60 

Sum  =    133?12'.  9r  Sum  =       174?44;55r  Sums        I43?l0r20r 

Here,  since  .the  three  sums  are  of  the  same  affection  with  respect  to  180? 
the  angle  C  is  ambiguous ;  therefore  it  may  be  either  69?  13 '37?  or  the 
supplement  thereof;  viz.,  110?  461 23r. 

To  find  the  Angle  B '.-^ 

Astheside  AB^thesideBC-H2=:3?24U0?Log.S.  ar.compt.  11.325483 
IsittheS.AB+tbeS.BC-H2=='71.35.10  Log.siness  .  .  9.977174 
SoistheaDg.C-theang.A-t-2a=2.37.32^    Log.  tangent  =     8.661426 

To  half  the  angle  B  m  •    *    •   5^?49C  22?  Log.  co-tangent     9. 864089 

Angle  B  =5  .  •  «  ,  •  •  107.38.44}  Which  is  amb^ous  because 
the  angle  C  came  out  indeterminate. 

TofindtheSide  AC:— 

As  the  angle  A  s  •  «  63?58'.32?  Log.  sine  ar«  compt.  10.046430 
Is  to  the  side  BC  s:  .68. 10.30.  Log.  sine  »  •  •  .  »  9.967699 
So  is  the  angle  B  s    .  107. 38. 44.   Log.  sine  s    «...    9. 979070 

To  the  side  A  C=:  ,  100?  6'47T  Log.  sine  =  .  .  ,  ,  9.993199 
The  side  A  C  is  also  ambiguom  because  the  angle  C  came  out  indeter- 
minatet 


■  Digitized  by 


Google 


200  SPHB^ICAL  TRIGONOMBTRY,   . 

Peoblem  II. 

Gioen  Ttoo  Angles  of  an  Oblique  Angled  Spherical  Triangle,  and  a  Sde 
opposite  to  one  of  them  ;  tojind  tlw  remaining  Angle  and  the  other  Two 

Sides. 

Rule. 

\.— Tojind  a  side  opposite  to  one  of  the  given  angles. 

A«  the  log.  sine  of  the  angle  opposite  to  the  given  side,  is  to  the  log.  sine 
of  the  given  side :  so  is  the  log.  sine  of  the  other  given  angle,  to  the  log; 
sine  of  its  opposite  side. 

Now,  to  know  whether  the  side  thus  found  is  ambiguous,  aeute,  or  ob- 
tuse, proceed  as  follows  3  viz.. 

To  the  side  so  found,  and  its  supplement,  add  the  given  side,  or  that 
used  in  the  proportion.— Then,  if  each  of  these  sums  be  of  the  same  affec- 
ttofi  with  respect  to  180?  as  the  sum  of  the  two  given  angles,  or  those  used 
in  the  proportion,  the  side  is  ambiguous  ;  that  is,  it  may  be  either  aCute^ 
or  obtuse ;  and,  therefore,  indeterminate. 

But,  if  those  sums  are  of  different  affectiom  with  respect  to  the  sum  of 
the  angles,  the  side  is  norambiguous :  in  this  case  that  value  of  the  side  is 
to  be  taken,  whether  acute  or  obtuse,  which,  when  added  to  the  given 
side,  produces  a  quantity  of  the  same  affection  with  the  sum  of  the  angles. 

2.— Tojind  the  side  contained  between  the  two  given  angles. 

Find  half  the  difference,  and  half  thel  sum  of  the  two  given  angles  :'- 
find,  also,  half  the  difference  of  their  opposite  sides. — ^Then  say. 

As  the  log.  sine  of  half  the  difference  of  the  angles,  is  to  the  log.  sine  of 
half  their  sum  j  so  is  the  log.  tangent  of  half  the  difference  of  their  oppo^ 
site  sides,  to  the  log.  tangent  of  half  the  side  contained  between  the  two 
given  angles ;  the  double  of  which  will  be  the  side  sought 

3. — Tojind  the  third,  or  remaining  angle. 

As  the  log.  sine  of  a  given  side  opposite  to  a  given  angle,  is  to  the  |og« 
sme  of  that  angle  ;•  so  is  the  log.  sine  of  the  side  opporfte  to  the  required ' 
angle,  to  the  log.  sine  of  the  required  angle. 

Note.^-^When  the  side  comes  out  ambiguous,  or  indeterminate,  in  the 
first  proportion ;  the  contained  side  and  the  third  angle,  found  by  the  other . 
poportions,  will  also  be  ambiguous. 


Digitized  by 


Google 


8PHBE1CAX.  TRIGONOMSTRY.  201 

*  Example. 

Let  the  angle  A,  of  the  spherical  triangle  ABC,  /  ^^o^X. 

be  130?40M3r,  the  angle  C  41?39U0r,  and  the         /       %  *i\ 

side B  C  1 15°  10' 25^  ;  required  the  angle  B,  and  B  ^ — Z5??""^\^  v\ 
the  sides  A  B  and  AC?  ""^-'^^ 

To  find  the  Side  A  B  :— 

As  the  angle  A  s  130?40U3^  L<«.  sine  ar.  coropt.  =  10. 120114 
Is  to  the  side  B  C  =  1 15. 10. 25  Log.  sine  ='«,..  9. 956660 
So  is  the  angle  C  =:    41.39. 40   Log.  sine  =      .     .     »    .    9. 822641 

To  the  side  A  B  =     52?29^ 28r  Log.  sine  =      •    .    •    .    9.  899415 

To  determine  whether  the  Side  A  B  is  Ambiguous,  Acute,  or  Obtuse : — 

Side  AB  52?29:28r  Supplement  =:  127^30^32^  Angle  A  130?40U3? 
SideBC115.10.25    Side  B  C  =       115.10.25     Angle  C    41.39.40 

Sum  =  167?39C53r  Sum  =     .     .    242?50:57?  Sum  =     I72?20:23r 

Here,  since.the  two  first  sums,  viz.  A  Band  B  C,  and. the  supplement  of 
A  B  and  B  C,  are  of  different  affections  with  respect  to  180?,  the  side  A  B  is 
not  ambiguous; — and  since  the  sum  of  the  acute  value  of  AB  added  to 
B  C  is  of  the  same  affection  .with  the  sum  of  the  angles }  therefore  the  side 
ABisacuter=52?29C28r. 

TofindtheSide  ACi- 
Astheang.  A— theang.  C-^2r=44?30'31i^  Log.S.  ar.compt.  10. 154271 
Is  to  angle  A  +  angle  C  -h  2=S6.  10. 1 1|    Log.  sine  =:   .    .     9. 999029 
Sois  the  S.BC-S.  A  B-h2^31.  10.28^   Log.  tangents:    .    9.784614 

TohalftheBideAC=   •    .    40^55^  6 ^  Log.  tangent       •    9.937914 

Side  AC  = 81?50n2r;  which  is  acute,  because  the 

side  A  B  came  out  determinate,  and  that  its  acute  value  applied  to  B  C  is 
of  the  same  affection  with  the  sum  of  the  angles. 

To  find  the  Angle  B  :— 

Afl  the  side  BCs  .  .  115^10' 25  V  Log.  S.  ar.compt.  .  .  10.043340 
Is  to  the  angle  As  .  130. 40. 43  Log.  sine  =  .  .  .  .'  9. 879886 
So  is  the  side  A  C  ss  •    81.50.12     Log.  sine  ==    .    .    /.    9.995577 

TotbeaiigleB»  .    .    56?  2Ml^rLog.8iae  s  .    .    .    .    9.918803 

Digitized  by  VjOOQ  IC 


202  SPttfiAICAl  tHlOONOMiTRt. 

Note.'^The  aiigle  B  is  acute  like  its  opposite  side  A  C,  because  the  side 
A  B  is  not  ambiguous ;  and  that  its  acute  value  applied  to  the  side  B  C  is  of 
the  same  affection  with  the  sum  of  thd  angles. 


PaofiLBM  IIL 

Owen  T\oo  Sides  of  an  ObUque-aiigled  Spherical  Ti-iangle,  and  the  Angle 
contained  between  them  ;  to  find  the  other  T\oo  Jnglee  and  the  Third 
Side. 

RUJLB. 

1. — To  find  the  other  two  atiglea. 

As  the  log.  co^sine  of  half  the  sum  of  the  ^two  given  sides^  is  to  the  log. 
co-sine  of  half  their  difference  $  so  is  the  log.  co-tangent  of  half  the  con- 
tained anglc^  to  the  log.  tangent  of  half  the  sum  of  the  other  two  angles* 

Half  the  sum  of  the  angles  thus  found,  will  be  of  the  same  affeetbn  with 
half  the  sum  of  the  sides^— -Again :  As  the  Icfg.  sine  of  half  the  sum  of  tli« 
two  given  sidets^  is  to  the  log.  eine  of  half  their  diffemiee }  so  is  the  log.  co- 
tangent of  half  the  contained  angle,  to  the  1<^.  tangent  t>f  half  the  differ* 
ence  of  the  other  two  angles.— Half  the  difference  Of  the  angles,  thua  foiiiid^ 
will  always  be  acute. 

Now,  hair  the  sum  of  the  two  angles,  added  to  half  their  difference,  will 
give  the  greater  angle ;  and  half  .the  difference  of  the  angles  subtracted  from 
half  their  sum  will  leave  the  lesser  angle. 

2.-'Tofind  the  third  side. 

.The  angles  being  known,  the  third  or  remuningside  is  to  be  computed 
by  Rule  3,  Problem  L,  page.  198. 

'Example. 

Let  the  side  A  C,  of  the  spherical  triangle 
ABC,  be  78?45n6:,  the  side  A B  69^55! 55 r, 
and  the  contained  angle  I26?30l20r;  required 
the  angles  B  and  C,  and  the  side  B  C  ? 


Digitized  by 


Google 


8l>fflBftTCAL  TIttGOKOBflSTRlr.  ^3 

To  find  the  Angle  B  s-. 

As  the  side  AC+AB-H2a74?20^55r  Log.  co-sine  ar.comp.=  IO.  568984 
IstothesideAC-AB^2=  4.24.40  Log.  co-sine  =  .  .  .  9.998712 
So  is  the  angle  A  -»-  2  ==  63. 15, 10  Log.  co-tang.  =  •    .    .    9. 702414 


To  i  the  sum  of  the  an.  =  61"? 46^  8^  Log.  tangent  =  •  •  .10. 2701 10 
Half  diff.  of  the  angles  =    2. 18. 19,  as  below 

Sum=    ...  64?  4127^  =  Angle  B. 

To  find  the  Angle  C  :— 

As  theside  AC  +  AB  -^  2  =  74?20'.55';  Log.  sine  ar.  compt.=  10. 016409 
Istotb6  8ideAC-^AB-^-2s  4.24.40  Log.  sine  «  •  «  .  8.885996 
So  isthe  angle  A  -H  2  =      63. 15. 10    Log.  co-<tangent  ss    .    9.  702414 

To  half  the  diff.  of  the  ang.s  2<^  18'  19?  Log.  Ungent  =  .  .  8i  604819 
Half  sum  of  the  angles  s  ^61. 46.   8>  as  abov^ 

Difference  =      .     .  59^27 '49?  =  Angle  C. 

Note. — The  half  sum  of  the  angles  came  out  acute,  because  the  half  sum 
of  the  sides  is  acute  ^  the  half  difference  of  the  angles  ii  alway$  acute. 

To  find  the  Side  BC:— 

As  the  angle  B  s=  64?  4^27?  Log.  sine  ar.  compt.  =  10. 046066 
Is  to  the  side  A  C  s  78.45. 15  Log.  sine  ss  .  .  »  .  9. 991580 
So  is  the  angle  A  =  1 26. 30.  20    Log.  sine  =    .    .  \    .    9.  905 1 48 

TothesideBCs  118?46:   ir  Log.  sines   .    .    .    .    9.942794 

Remark  l.-^The  side  B  C  may  be  found  directly,  independently  of  the 
ati^ea  B  and  C,  by  the  following-  general  Rule. 

To  twice  the  log.  sine  of  half  tip  contained  angle,  add  the  log.  sines  of 
the  two  containing  sides;  from  half  the  sum  of  these  three  logs,  subtract 
the  log.  nne  of  half  the  difference  of  the  sides,  and  the  remainder  will  be 
the  log.  tangent  of  an  arch :  the  log.  sine  of  which  being  subtracted  from 
the  half  sum  of  the  three  logs,  will  leave  the  log.  sine  of  half  the  required 

aide* 

Example. 

Let  the  side  A  C,  of  a  spherical  triangle^  be  62?  10^25?,  the  sido  A  B 
50?14C45^  and  the  included  angle  A  123?li:40r;  required  the  aide 
BC? 


Digitized  by 


Google 


204  SPHERICAL  TRIGONOMETRY. 

Half  ang.  A=  61^35^50?  r:f.7*!^'»:}= 19. 888596 
Side  A  C  =  62. 10,  25  Log.  sine  =  9.  946632 
SideAB=      50.14.45    Log.  sine    =9.885811 


Sura  =  39.  721039 


DiflF.ofSides    ll?55M0r         Half  =  19. 86051 9i,    ...19.8605194 


Half  ditto    =    5?57^50r  Log.sine=  9.016622 


Arch  =  .     .    81  ?50;52r  Log.  tang.  =  10.  8438971  Log.  S.=9. 9955881 


iSideBC  =  47?^  6^50r  =Log.8ine   =.......    9.864931 


Side  B  C  s    94?  13 ^  40r,  as  required. 

Remark  2.«— The  side  B  C  may  be  also  computed  by  the  following  ge« 
neral  rule,  viz. 

To  twice  the  log.  si^e  of  half  the  contuned  angle,  add  the  log.  sines  of 
the  two  containing  sides,  and  the  sum  (rejecting  30  from  the  index,)  will  be 
the  log.  of  a  natural  number.— -Now,  the  sum  of  twiot  this  natural  number 
and  the  natural  versed  sine  of  the  difference  of  the  containing  sides,  will  be 
the  natural  versed  sine  of  the  third  side. 

Thus,  to  find  the  side  B  C  in  the  above  example. 

Half  included  ang.  A=6-l.?35 '.  50?  twice  the  log.  sine=     .     .19. 888596 
Side  AC  ==  .    .    .      62.10.25  Log.  sine  ==   ..    ...    .    9.946632 

SideAB=  .    .    '.      50.14.45   Log.  sine  = -9.885811 


Natural  number  =     . 526065  »:  Log.   9. 721039 

Twice  the  nat.  numb.=:   .......  1052130 

Diff.  of  the  given  sides  ll?55UO:  N.V.S.i=  021591 


SideBC=     .    .    .  94?  13^  40r  N.V.S.  1073721;  the  same  as  above. 

Note. — In  taking  out  the  natural  number  corresponding  to  the  sum  of 
the  three  logs. :  if  the  index  be  9,  the  natural  number  is  to  be  takien  out  to 
six  places  of  figures ;  if  8,  to  five  places  of  figures ;  if  7j  to  four  places  of 
figures,  &c. 


Digitized  by 


Google 


SPHERICAL  TRIQONOMBTRT.  205 


Problem  IV, 

Given  Two  jingles  of  a  Spherical  Triangle,  and  the  Side  comprehended 
between  them  ;  to  find  the  remaining  jingle  and  the  other  T\vo  Sides. 

Rule. 
l.'^Tofihd  the  other  two  sides. 

As  the  log.  co-sine  of  half  the  dum  of  the  two  given  angles,  is  to  the  log. 
co-Bine  of  half  their  difference ;  so  is  the  log.  tangent  of  half  the  compre- 
hended side,  to  the  log.  tangent  of  half  the  sum  of  the  other  two  sides. 

Half  the  sum  of  the  sides,  thus  found,  will  be  of  the  same  affection  with 
the  half  sum  of  the  angles. 

Again. — ^As  the  log.  sine  of  half  the  sum  of  the  two  given  angles,  is  to  the 
log.  sine  of  half  their  difference ;  so  is  the  log.  tangent  of  half  the  compre- 
hended side,  to  the  log.  tangent  of  half  the  difference  of  the  other  two 
sides. 

Half  thetlifference  of  the  angles,  thus  found,  will  always  be  acute. 

Now,  half  the  sum  of  the  two  sides,  added  to  half  their  difference,  will 
give  the  greater  side;  and  half  the  difference  of  the  two  sides,  subtracted 
from  half  their  sum^  will  leave' the  lesser  side. 

2.^Tofind  the  remaining  angle.  . 

The  sides  and  two  angles  being  known,  the  remaining  or  third  angle  is 
to  be  computed  by  Rule  3,  Problem  II.,  page  200. 

Example. 

x. 

Let  the  angle  A,  of  the  spherical  triangle  ABC,, 
be  63?50'.25r  ;  the  angle  C  58?40'.  15r,  and  the 
comprehended  side  A  C  87?30MO'r ;  required  the 
sides  A  B  and  B  C,  and  the  remaining  angle  B  ? 

To  find  the  Side  B  C  :— 

As  the  angle  A+angleC^2=61  <"  15 '.  20^  L.  co-sine  ar.com.  =  10. 317942 
Is  to  the  ang.  A— ang.CH-2=  2. 35.  5  Log  co-sine  =  .  .9. 999557 
So  is  the  side  A  C  -h  2  s::       43. 45. 20     Log.  tangent  :=   •    .  9. 981 129 

To  half  the  sum  of  (he  sides=63?18^28^  Log.  tangent  =  .      10. 298628 

Halfdifferenceof  the  sides  =  2.49. 10,  as  in  the  next  operation. 

.    ■  —— .»  . 

Sum  =       66?  7 '  38?  s=  the  side  B  C. 

Digitized  by  VjOOQ  IC 


206  SPHBRICAl.  TRIGONOMBTRT* 

To  find  the  Side  A  Bi- 
as the  angle  A4-angleC^2=61  ?  15 ^ 20r  L.  sine  ar.  compt.  =  10. 0571 13 
Is  to  angle  A  —  angle  C-h2=:  2.35.   5    Log,  sine  =       .    .    8. 654144 
SoisthesideAC -t- 2=     43.45.20    Log  tangent       ,    .    9.981129 

Tohalfthediff.oftheside8=2949n0?  Log  tangent  =      .    8..  692386 
Half  sum  of  the  sides  =    .    63. 18.28,  as  in  the  last  operation. 
Difference  ^  60V29: 18r  »  the  tide  A  B« 

No/e.~The  half  sum  of  the  sides  joaxae  out  acute  because  the  half  sum  of 
the  angles  is  acute  ;  the  half  difference  of  the  sides  must  be  akoayi  acute. 

To  find  the  Angle  B  :— 

As  the  Side  B  C  s  66?  J'SSI    Log.  sine  ar.  compt.  s     10. 038842 

Is  to  the  angle  A  a  63. 50. 25     Log.  sine  s       ....    9. 953068 

SoisthesideAC  =  87.30.40    Log.  sine  ==:       .    .    .    .    9.999590 

To  the  angle  B  ss     .  78^42^  3r  Log.  sine  »     .    «    .    .9.991500 

Bemark  1.— The  angle  B  may  be  found  directly  by  the  following  gene- 
ral rule. 

To  twice  the  log,  co-sine  of  half  the  given  side^  comprehended  between 
the  two  given  angles^  add  the  log.  sities  of  tho^e  angles  :  from  half  the  sum 
of  these  three  logs,  subtract  the  log.  sine  of  half  the  difference  of  the  ui- 
gles,  and  the  remainder  will  be  the  log.  tangent  of  an  arch.— -Now,  the  log. 
sine  of  this  arch  being  subtracted  from  the  half  sum  of  the  three  logs,  will 
leave  the  log.  sine  of  half  the  required  angle.  ^ 

Thus^  to  find  the  angle  B  in  the  above  example. 

Half  side  A  C  =5  43?45 '.  20^  C1iS&!^'  }=«  19. 717432 
Angle  A  =  63. 50. 25  Log  sine  =  9. 953068 
Angle  C  s         58. 40. 15  Log.  sine  ^^     9. 931557 


Sum=  39.602057 


Diff.oftheang.=5?10U6r         Half  =  19. 8ai028i  .    .     19.801028J 

Halfdiff.ofdo.=:2.  35.   5    Log.  sine  =     8.654144 

Arch  »  85?55 :  17^     .     U>g.  tangents:  1 1 .  146884^1  Lg.S.s9. 998899 


Halfthe  required  angle  =  39?^inrLdg.sine    =     .    .    .    9.802129^ 

Hence,  the  angle  B  is  =    78?42:2r}  which  differs  K  from  the  angle 
found  as  above. 


Digitized  by 


Google 


8PBBAICAL  TRIGOVOIiBTilY.  807 

Remark  2.— The  angle  B  may  be  also  very  readily  computed  by  the  fol- 
lowing general  Rule  5  viz.^ 

To  twice  the  log.  co-nne  of  half  the  given  side^  comprehended  between 
the  two  given  aisles,  add  the  log.  sines  of  those  angles^  and  the  sum  (reject-* 
ing  30  from  the  inde^c)^  will  be  the  log.  of  a  natural  number.-— Now,  the  sum 
of  twice  this  natural  number  and  the  natural  versed  sine  of  the  difference  of 
the  angles^  will  be  the  natural  versed  sine  of  the  required  angle. 

Thus^  to  find  the  angle  B  in  the  last  example. 

Half  the  given  side  A  Cs  43?45 ',  20r  twice  the  log.  co-sine  s  19. 717432 
Angle  A «  .  .  .  •  68. 50. 25  Log.  sine  s  ....  9. 953068 
Angle  Cs     k    .    .    •    58.40. 15  Log.  sine  =    .    .    .    .    9.931557 

Natural  number  =:,...    S99998=:Log.  9. 602057 

Twice  the  natural  number  =:    .     799996 
Diff.oftheang.=  5?10nOrnat  versed  sine  =  004067 


Angle  B  =  78?42'  2",  nat.  versed  sine  =  804063  j  the  same  as  by 
the  former  Rule. 


Pboblim  V. 
Gftoen  the  Three  Sides  of  a  Spherical  JHangle,  tojind  the  Angles. 

RULB, 

Add  the  three  sides  together  and  take  half  their  sum  ;  find  the  difference 
between  this  half  sum  and  the  side  opppsite  to  the  required  anglcj  which 
eall  the  remainder ;  then. 

To  the  log.  co-secahts,  less  radius^  of  the  other  two  sides,  add  the  log. 
aihes  of  the  half  sum  and  the  remuqder  :-<i>-half  the'sum  of  these  four  logs, 
will  be  the  log.  co^sii)^  of  an  arch,  which  being  doubled  will  be  the  required 
angle. 

.  Chie  angle  being  thus  found,  the  remaining  amgies  may  be  computed  by 
Role  3^  Problem  IL,  page  200. 

Example. 

In  the  spherical  triangle  ABC,  let  the  side  A  B        ^7 
be70?llM5r,  the  side  AC  81?59'.55r,  and  the     ^  "^ ' 
aideBC  120?  IOC  SOT;  required  ^the  angles  A,  B,  ^ 
and  C? 


Digitized  by 


Google 


20S 


SPHBRICAL  TRIGONOMETRT. 


SideBCs 

Side  AC  = 
SideABs: 


Sum 


To  find  the  Angle  A  :— 

120nOC50^ 

81 .  59, 55  Log.  co-secant|  less  radias=0. 004248 
70. 11.45  Log.  co-secant,  less  radiusssO.  026477 

272.22.30 


Halfsums  136.11. 15' Log.  sines.    ....  9.840295 

Remainders  16.   0.25  Log.  sine  =  .....  9.440522 

Sum  =  .    .    .  19.311542 

.Arch=.    ;  63?  5'  8r   =  Log.  cp-sine  =      .    .  9.655771 
AngIeAs=  126?10n6r 


As  the  side  B  C  =  . 
Is  to  the  angle  A  = 
So  is  the  side  A  C  = 

To  the  angle  B  =  . 


To  find  the  Angle  B:—' 

.     120?10:50r  Log. co-secants  10.063262 

.     126.  le.  16  Log.  sine  s     .    .  9.907012 

.      81 .  59, 55   Log.  sine  =     .    .  9.  995752 


67?37^52r  Log,  sines 


9. 966026 


As  the  side  B  C  s  . 
Is  to  the  angle  A  = 
So  is  the  side  A  B  s 

To  the  angle  C  s  .  . 


To  find  the  Angle  C  .— 

.  120?10:5ar  Log.  co-secant  s  10.063262 
.  126.10.16  Log.«ine=  .  .  9.907O12 
.       70.11.45  Log.  sine  =        .    .     9.973523 

.    6l928:31.r  Log.sine    =.     .    "^.943797 


-JR^mr^.^-The  required  angle  of  a  spherical  triangle  (when  the  three- 
sides  are  given),  may  be  also  found  by  the  following  general  Rule ;  viz,. 

Add  the  three  sides  together  and  take  half  their  sum  :  find  the  difierence 
between  this  half  sum  an.d  each  of  the  sides  containing  the  required  an^le, 
and  note  the  remainders. — ^Then, 

To  the  log.  co-secants,  lesis  radius,  of  those  sides,  add  the  log.  sines  of  the 
two  remainders  z^-half  the  sum  of  these  four  logs,  will  be  the  log.  sine  of 
half  the  required  angle. 

'  Thus,  to  find  the  angle  A  in  the  last  example. 


Digitized  by 


Google 


SPHERICAL  TRlGONbMETRT.'  209 

SideBC  =  .    .    .     120?10f50r 

Side  A  C  =       .    .      8 1 .  59. 5a  Log.  co-secant,  less  radiu8=0. 004248 . 

Side  A  B  =      .     .      70. 11. 45  Log.  co-secant,  less  radiu8=0. 026477 

Sumss  .    .    ,    .     272.22.30    * 


Half  sum     ,     .  136°lin5r 

Remainder,  by  AC  =   54. 11.20  Log.  sine  -    .    .    .    ;    .9.908994 

Remainder,byAB  =  65.59.30  Log. sine  =     •*    .    •    .    •9.960702 


Sum  =  .    .  19.900421 


Half  thie  angle  A  =     63?  5^  8?  Log.  sine  ^      .    .    •    .    9.950210J 

Which  beirtg  doubled,  shows  the  angle  A  to  be  128?  10n6r  j  the  same 
as  by  the  former  rule. 


Probum  VI. 
Owen  the  Three  Angles  of  a  Spherical  THangUy  to  find  the  Sides. 

Add  die  three  angles  together  and  take  half  their  sum ;  find  the  differ- 
ence between  the  half  sum  and  the  angle  opposite  to  the  required  side, 
which  call  the  remainder. — ^Tlien,  % 

To  the  log«  do-secants,  less  radius,  of  the  other  two  angles,  add  the  log. 
CO- sines  of  the  hsilf  sum',  dnd  the  remainder;  half  the  sum  of  these  four 
logs,  will  be  the  log.  sine  of  half  the  required  side; 

One  side  being  thus  found,  the  remaining  sides  may  be  computed  by 
Ruled.  Problem  L,  page  198. 

E:tample0 

In  the  spherical  triangle  A  B  C,  let  the  angle  A 
be  I25?16^25?;  the  angle  £84?  20  ^50r,  and  the 
angle  C  72?40: 15r ;. required  the  sides  B  C,  A  B,      /jv 
«ndAC? 


Digitized  by 


Google 


210  SPHERICAL  TOIOONOMKTRY. 


To  find  the  side  B  C  :— 

Angle  A  =      .    .  125?16r25r 

Angle  B  =■     .    .    84. 20. 50  Log.  co-secant,  less'  fadius  s=     0. 002117 

Angle  C=     ,     .     72.40. 15  Log.  co-secant,  less  radius  =     0.020174 

Sum=  ..    .    .    282.17.30 


Half  sum  =  .    .    141?  8M5r  Log.  co-sine  =       '    '    '    '    9-891395 
Remainder  3=    .      15. 52. 20  Log.  co-sine  =        .    .    .    .    9.983118 

Sam  s  .    .19. 896804 


HalfthesideBC*.    .    62?37a3?  Log.  sine  =»    ...  .9.948402 
The  double  of  which  gives  125?14'.26'r,  for  thewhole  Vide  B  C. 


To  find  the  Side  A  B :— 

As  the  angle  A  =  .  .  125?  i6C25f  Lpg»  coisecant  =  .  .10. 088095 
l8tothesideBC=  •  125. 14. 26  Log.  sine  =  .  .  .  .  9.912083 
So  is  the  angle  C  a  .      72.40,15  Log.  sine  »  .    •    •    .    9.979826 

TothesideAB  =  .    .    72  944  U6r  Log.  sine  =      .    ,.,9.980004 


To  find  the  Side  A  C  :— 

As  the  angle  A  =  126?  16^  25r  Log.  co-sectot  s  ...  10. 088093 
Is  to  the  side  B  C  =  125. 14. 126  Log.  sine  =.....  9. 912083 
So  is  the  angle  B  =    84.20.50  Log.  sine  =  •    .     ,    .    ,    9.997883 


To  the  side  AC  =      84?35f  25r  Log.  sine  »       ....    9. 998061 

IZ^arA;.—- The  required  side  of  a  spherical  triangle  (when  the  three  an- 
gles are  given,)  may  he  also  found  by  the  (pUowing  general  rule ;  viz.,  • 

Add  the  three  angles  together  and  take  half  their  siim ;  find  the  differ- 
ence between  the  half  sum  and  each  of  the  angles  comprehending  the  re- 
quired side,  and  note  the  remainders. — ^Then  to  the  log.  co-secants  less  ra- 
dius, of  those  angles,  add  the  log.  co-sines  of  the  two  remainders  :  half  the 
sum  of  these  four  bgs.  will  be  the  log.  co-sine  of  half  tlm  required  side. 

Thus,  to  find  |)&e  side  B  C  in  the  last  example. 

Digitized  by  VjOOQ IC 


MAVIOATION.  211 

Angle  Ass.    ;    125?16*25r 

Angle  B  a      .      84. 20. 50  Log.  co-secant^  less  radius  s     0. 002 11 7 

Anjie  C  =s      .      72. 40. 15  Log.  co-secant,  less  radius  =s      0.020174 

Sum  =  282. 17.30 


Half  sum  ss  .    •  141?  8'.45r 

Remainder  byB  =  56. 47. 55  Log.  co-sine  =     ....     .9.738450 

Remainder  by  Cs  68.  28.  SO  Log.  co-sine  a 9.564556 


Sums  19.325297 


Half  Side  B  C  a    .    .    .    .   62?S7<  I3r  Log.  co*sine  a  .    9. 662648| 

Which  being  doubled  gives  S3  U5?14C26f^  for  the  tide  BCj  the  same 
as  by  the  foriper  rule. 


THE  HESOLUTION  OP  PROBLEMS  IN  NAVIGATION  BY  LOG- 
ARITHMS) AND,  ALSO,  BY  THE  GENERAL  TRAVERSE 
TABLE. 

Lest  the  mariner  should  feel  some  degree  of  disappointment  in  not  find- 
ing a  regular  course  of  navigation  in  this  work :  the  author  thinks  it 
right  to  remind  Kmj  that  his  present  intention  carrieta  him  no  farther  than 
merely  to  show  the  proper  application  of  the  Tables  to  some  of  the  inost 
useAil  paris  of  the  sciences  on  which  he  may  touch : — it  being  completely  at 
variance  with  the  plan  of  this  work,  to  enter  into  such  parts  of  the  sciences 
as' could  reasonably  be  dispen^d  with,  without  ^itirely  losing  sight  of  their 
principles. — Hence  it  is,  that  the  cases  of  plane  sailing,  usually  met  with  in 
books  on  narigation,  will  not  be  noticed  in  this.— However,  since  it  is  not 
improbable  that  this  voluine  may  fall  into  the  hands  of  persons  not  very 
deeply  versed  in  nautical  matteis ;  it  therefore  may  not  be  deemed  unne* 
eeseary  to  give  a  few  introductory  definitions,  &c.  for  their  immediate  guid- 
ance, previously  to  entering  upon  the  essentially  useful  parts  of  the  sailings. 


Navigation  is  the  art  of  conducting  a  ship,  through  the  wide  and  path- 
leas  ocean,  from  one  part  of  the  world  to  another. — Or,  it  is  the  method  of 
finding  the  latitude  and  longitude  of  a  ship's  place  at  sea ;  and  of  thence 
determining  her  course  and  distance  from  that  place,  to  any  other  given 
place. 

p  2 


Digitized  by 


Google 


212  .NAVIGATION. 

The  Equator  is  a  great  circle  circumscribing  the  eartl?,  every  point  of 
which  is  equally  distant  from  the  poles ;  thus  dividing  th^  globe  into  two 
equal  parts,  called  heniispheres  :  that  towards  the  North  Pole  is  called  the 
northern  hemisphere,  and  the  other,  tlie  southern  hemisphere.— Tlie  equa- 
tor, like  all  other  great  rircles,  is  divided  into  360  equal  parts,  called  de- 
grees ;  each  degree  into  60  equal  parts,  called  minutes  j  each  minute  into 
60  equal  parts,  called  seconds,  and  so  on. 

The  Meridian  o{  fiXiy  pUce  on  the  earth  is  a  great  circle  passing  through 
.that  place  and  the  poles,  and  cutting  the  equator  at  right  angles.— Every 
point  on  the  surface  of  the  sphere  may  be  conceived  to  haye  a  meridian 
line  passing  through  it  5  —hence  there  may  be  as  many  pieridiahs  -as  there 
are  points  in  the  equator.— Since  the  First  Meridian  is  merely  an  imagin- 
ary circle  passing  through  any  remarkable  place  and  the  poles  of  the 
world ;  therefore  it  is  entirely  arbitrary.— Hence  .it  is  that  the.  British 
reckon  their  Jvrst  meridian  to  be  that  which  passes  through  the  Royal  Ob- 
servatory at  Greenwich  :'  the  French  esteem  their  first  meridian  to  be.  that 
which  passes  through  the  Royal  Observatory  at  Paris ;  the  Spaniards  that 
which  passes  througK  Cadiz,  &c.  &c.  &c. 

Every  meridian  line  may  be  said,  with  respect  ta  the  place  through 
which  it  passes,  to  divide  the  surface  of  the  sphere  into  two  equal  parts^ 
called  the  pastern  and  westei^n  hemispheres. 

The  Latitude  of  any  place  oh  the  earth  is  that  portiofi  of  its  meridian 
which  is  intercepted  between  the  equator  and  the  given  place ;  and  is  named 
north  or  south,  according  as  the  giveii  place  is  in  the  northern  or  southern 
hemisphere. — As  the  latitude  begins  at  the  equator,  where  it  is  nothing, 
and  is  reckoned  thence  to  the  poles,  where  it  terminates ;  therefore  the 
greatest  latitude  any  place  can  have,  is  90  degrees. 

The  Difference  of  Latitude  between  two  places  on  the  earth  is  an  arc  of 
the  meridian  intercepted  between  their  corresponding  parallels  of  latitude  ; 
showing  how  far  one  of  them  is  to  the  northward  or  southward  of  the 
other :— The  dlflference  of  latitude  between  two  places  can  never  exceed 
180  degrees. 

The  Longitude  of  any  place  on  the  earth  is  that  arc  or  portioir  of  the 
equator  which  is  contained  between  the  first  meridian  and  the  meridian  of 
the  given  place  ;  and  is  denominated  east,  or  west,  according  as  it  may  be 
situated  with  respect  to  the  first  meridian. — ^As  the  longitude  is  reckoned 
both  ways  from  the  first  meridian  (east  and  west)  till  it  meets  at  the  same 
meridian  on  the  opposite  part  of  the  equator ',  therefore  .the  longitude  of 
any  place  can  never  exceed  180  degrees* 


Digitized  by 


Google 


^AVIGATIO^.  218 

The  difference  of  Longitude  between  two  places  on  the  earth  is  an  arc  of 
the  equator  intercepted  between  the  meridians  of  those  places ;  shovHng 
how  far  one  of  them  is  to  the  eastward  or  westward  of  the  other  : — ^The 
difference  of  longitude  between  two  places  can  never  exceed  180  degrees. 

When  the  latitudes  of  two  places  on*  the  earth  are  both  north  or  both 
south ;  or  their  longitudes  both  east  or  both  West,  they  are  said  to  be  of  the 
same  name. — But,  when,  one  latitude  is  north  and  the  other  south ;  or  one 
longitude  east  and  the  othef  west ;  then  they  are  said  to  be  of  different 
names. 

The  Horizon  is  that  great  circle  which  is  equally  distant  from  the  zenith 
and  n^dir,  and  divides  the*  visible  from  the  invisible  hemisphere ;  this  is 
called  the  rational  horizon.«-The  sensible  horizon  is  that  which  terminates 
the  view  of  a  spectator  in  any  part  of  the  world. 

Tie  Manner's  Compass  is  an  artificial  representation  of  the  horizon : — it 
is  divided  into  32  equal  parts,  callett  points ;  each  point  consisting  of 
U?15C. — Hence  the  whole  'Compass  card  contains  360  degrees;  for 
ll?15f  multiplied  by  32  points  =  360 degrees. 

A  Rhumb  line  is  a  right  line,  or  rather  curve,  drawn  from  the  centre  of 
the  compass  to  the  horizon,  and  obtains  its  name  from  the  point  of  the  ho- 
rizon it  falU  in  with. — Hence  there  may  be  as  many  rhimib-lines  as  there 
are  joints  in  the  horizon. 

The  Course  steered  by  a  ship  is  tlie  angle  contained  between  the  meri- 
dian of  the  place  sailed  from,  and  the  rhumb-line  on  which  she  sails ;  and 
is  either  estimated  in  points  or  degrees.  . 

The  Distance  is  the  number  of  miles  intercepted  between  any  two 
places,  reckoned  on  the  rhumb  line  of  the  course  $  or  it  is  the  absolute 
length  that  a  ship  has  sailed  in  a  given  time. 

The  D^parfKre.is  the  distance  of  the  ship  from  the  meridian, of  the  place 
s^led  from,  reckoned  on  the  paraltel  of  latitude  at  which  she  arrives ;  and 
is  named  ea«t  ot  west)  according  as  the  course  is  in  the  eastern  or  western 
hemisphere.  .        ^ 

If  a  ship's  course  be  due  north  or  south,  she  sails  on  a  meridian,  and 
therefore  giakes  no  departure  : — hence  the  distance  Bailed  will  be  equal  to 
the  difference  of  latitude. 

If  a  ship's  coufse  be  due  east  or  west,  she  sails  either  on  the  equator,  or 
on  some  parallel  of  latitude ;  in  this  case  since  she  makes  no  difference  of 
latitude^  the  xlistance  sailed  willj  therefore,  be  equal  to  th^  departure. 


Digitized  by 


Google 


914  .  NAVIQATIOir. 

« 

Whea  the  course  is  4  points^  or  45  degrees,  the  difference  of  latitude 
and  departure  are  equal. 

When,  the  course  is  less  than  4  points^  or  45  degrees,  the  difference  of 
latitude  exceeds  the  departure ;  but  when  it  is  iiiore  than  4  points,  or  45 
degrees,  the  departure  exceeds  the  difference  of  latitude. 

Note. — Since  the  distance  sailed,  the  difference  of  latitude,  and  the  de- 
pai^ture  form  the  sides  of  a  right  angled  plane  Jriangle  |  in  which  the  hypo- 
thenuse  is  represented  by  the  distance ;  the  perpendicular,  by  the  differ- 
ence of  latitude ;  the  base,  by  the-  departure ;  the  angle  opposite  to  the 
base,  by  the  course;  and  the  angle  opposite  to  the  perpendicular,  by  the. 
complement  of  the  course }  therefore  any  two  of  these  five  parts  being 
given,  the  remaining  three  may  be  readily  found  by  the  analogies  for  right 
angled  plane  trigonometry. 

%. 
These  being  premised,  we  will  now  proceed  to  the  following  Introduciory 

Problems. 


Problbm  L 
Gioen  the  LatUwies  qf  Jloo  Places  on  the  Eairth,  to  find  the  difference  of 

RuuB. 

When  the  latitudes  are  of  the  same  name ;  that  is,  both  north,  or  both 
south,  their  difference  will  be  the  difference-  of  latitude ;  but  when  one  is 
north  and  tbo  other  south,  th^ir  sum  will  express  the  difference  of  la- 
titude, 

Note.— The  same  Rule  is  to  be  observed  in  finding  the  meridicmal  differ- 
ence of  latitude  between  two  places. 


"Exomple  L, 
Required  the  difference  of  la-, 
titude  between    Portsmouth   and 
Cape  Trafalgar? 

Lat.  of  Portsmouth  =50?47fN. 
Lat.  of  C.  Trafalgar  =  86. 10   N. 

Diff.  of  Lat.  =    •    .     14?37t 


Pitto  in  Miles  =♦    ,    877 


Exarnple  2. 
Required  the  difference, of  la« 
titude   between    Portsmouth  and 
James  Town,  Stw  Helena  ? 
Lat.  of  Portsmouth  =  «0?47'  N. 
Lat.ofJariiesTown=s  15.55   S« 


Diff.ofLat.se     .\    461*42; 
PittpiDMil^ss    ,    ,2809 


Digitized  by 


Google 


DIFF£RBNCB  OP  LATlttTDB  AND  LONGITUDE. 


215 


Note.^^lu  finding  the  diiFerence  of  latitude,  or  the  difierence  of  longi* 
tude  between  tviro  i^aceii  (when  any  of  the  sailings  are  under  consideration), 
it  will  be  sufficiently  exact  to  take  out  the  latitudes  and  longitudes  from 
Table  LVIIL  to  the  nearest  minute  of  a  degree^  as  above. 


Problbm  IL 


Oven  the  Latitude  left  and  the  difference  of  Latitude,  to  find  the 

Latitude  tn. 

RUtB. 

When  the  latitude  left  and  the  difference  of  latitude  are  of  the  same  name 
their  sunr  will  be  the  latitude ;  but  when  they  are  of  contrary  denomin- 
ations^  their  difference  will  be  the  latitude  required  : — ^This  latitude  will 
always  be.  of  the  same  name  with  the  greater  quantity. 


Example  1. 

A  ship^  from  a  place  in  latitude 
30?45:  north  sajled  497  miles  in 
a  northerly  direction ;  required  the 
latitude  at  which  she  arrived  ? 

Latitude  left  »     .    .  30N5:  N« 
Diff.ofLiit3iB497m«.or8.17    N. 


Lat.  arrived  at  = 


sa?  2:  N. 


Example  2. 

A  ship  from  a  place  in  latitude 
2?50'  norths  sails  530  miles  in  a 
southerly  direction;  required  the 
latitude  come  to  ) 

Latitude  left  m  ^    .    .  2?50'  N. 
'Diff.oflata530ms/or  8.50    8. 


Lat.  come  to  = 


6?  0^  S. 


Peoblbm  III. 

Gioen  the  Longitudes  of  Tioo  Places  on  the  Earth,  to  find  the  difference 
^  qf  Longitude. 

Ru£b. 

When  the  longitudes  are  of  the  same  name  :  that  is^  both  east,  of  both 
west,  their  difference  will  express  the' difference  of  longitude  >  but  when 
one  is  east  and  the  other  Wesf,  their  sum  wHl  be  the  difference  of  longitude. 
If  the  sum  of  the  longitudes  exceed  ISO?,  subtract  it  from  360%  and  the 
remaioder  will  be  the  difference  of  longitude* 


Digitized  by 


Google 


316 


NAVIGATION* 


Example  I. 

Required  the  difference  of  lon-^ 
gitude  between  Portsmouth  and 
Fayal,  one- of  the  western  islands  ? 

Long,  of  Port8mouth=  19  6^  W. 
Long,  of  Fayal,  HorU,28. 43    W. 

Diff.  of  long,  =.    .    27^37^ 

Ditto  in  miles  =  •    •    1657 


JSxample  2. 

Required  the  difference  of  lon^ 
gitude  between  Canton  and  Point 
Venus^  in  the  island  of  Otaheite  ? 

Long,  of  Canton  =      1 13?  3  C  E, 
Long,  of  PointVenus=  149. 36  W. 


Sum  =       «    •    • 

Diff.  of  Long.  =  • 
Ditto  in  miles  = 


262939C 


97921 C 
.  5841 


Paobleat  IV. 

Given  the  Jjmg%tude\left  and  the  difference  of  Longitude^  to  find  the 

Longitude  in. 

RULE; 

When  the  longitude  left  and  the  difference  of  longitude  are  of  thie  same 
name^  their  sum  will  be  the  longitude  in;  should  that  sum  exceed  180?, 
subtract  it  from  360?  ;  and  the  remainder  ytill.be  the  longitude  in,  of  a 
contrary  name  to  the  longitude  left^-^Buiy  when  the. longitude  left  and  the 
difference  of  longitude  are  of  contrary  names,  their  difference  will  be  the 
longitude  in,  of  the  same  name  with  the  greater  quantity. 


Example  1. 

A  ship  from  a  pla<;e  in  longitude 
50?40'  west,  sails  westward  till 
her  difference  of  longitude  iir  4H) 
miles ;  required  the  longitude  in  ? 

Long,  left  =  .     •    .  50940^  W; 
Diff.oflong.=41pm8.or6..50    W.  , 


Longitude  in  si  •    •  57?30C  W. 


Example  2. 

Let  the  long,  left  be  174945.' 
west,  and*  the  difference  of  longi* 
tude   13?  17 f  west;  riequired  the 
'longitude  in? 

Longitude  left  =  .     Hr4?45^  W. 
Diff.  of  Long.  =  .      13.17    W. 


Sum: 


Longitude  in  i 


188.  2 


171958CE. 


Digitized  by 


Google 


PARAtLBt  SAILIKC. 


21? 


Example  3. 

Let  the  longitude  left  be  4 1  ^  37  • 
east,  and  the  difference  of  longi* 
tude  ll?20'  west;  required  the 
longitude  come  to  ? 


Longitude  left  =  • 
Diff.  oflong.  =    . 

Longitude  in  =    • 


41?37r  E. 
11.20W. 

30^.17f  E. 


Example  4. 

Let  the  longitude  left  be  5?40f 
east,'  and  the  difference  of  longi* 
tude  10?  17'  west;  required  the 
longitude  in  ? 


Longitude  left  = 
Diff.  oflong.  =    4 

Long,  in  =      *    . 


5?40^  E. 
10. 17  W. 


4?37IW. 


Remarks. — ^If  a  ship  be  in  north  latitude  sailing  northerly,  or  in  south 
latitude  sailing  southerly,  she  hicreases  her  latitude,  and  therefore  the  dif«- 
ference  of  latitude  must  be  added  to  the  latitude  left,  in  order  to  find  the 
latitude  in  r—rbut,  in  north  latitude  sailing  southerly,  or  in  south  latitude^ 
northerly,  she  decreases  htr  latitude ;  therefore  the  difference  of  latitude 
subtracted  from  the  latitude  left  will  give  the  latitude  in  :— should  the  dif- 
ference of  latitude  be  the  greatest,  the  latitude  left  is  to  be  taken  from  it ; 
in  this  case  the  ship  will  be  on  the  opposite  side  of  the  equator  with  res- 
pect to  the  latitude  sailed  from. — Again, 

If  a  ship  be  in  east  longitude  sailing  easterly,  or  in  west  longitude  sailing 
westerly,  she  increases  her  longitude ;  therefore  the  difference  of  longitude 
added  to  the  longitude  left  will  give  the  longitude  in ;  should  the  sum  ex- 
ceed 180?,  the  ship  will  be  on  tlie  opposite  side  of  the^^t  meridian  with 
respect  to  the  longitude  sfliiled  from. — But,  in  east  longitude  sailing  west* 
erly,  or  in  west  longitude  sailing  easterly,  she  decreases  her  longitude,  and 
therefore  the  difference  of  longitude  is  to  be  sttbtracted  from  the  longitude 
left;,  in  order  to  find  the  longitude  in;— should  the  difference  of  longitude 
be  the  greatest,  the  longitude  left  is  to  be  taken  from  it ;  in  this  case  the 
ship  will,  also,  b^  on  the  opposite  side  of  the  first  meridian  with  respect  tq 
the  longitude  sailed  from.  *  These  remarks  will  appear  evident  on  a  com* 
parison  with  the  above  Examples; 


SOLUTION  OF  PROBLEMS  IN  PARALLEL  SAILING. 

Parallel  iSailing  is  the  method  of  finding  the  distance  between  two  places 
situate  under  the  same  parallel  of  latitude ;  or  pf  finding  the  difference  .of 
longitude  corresponding  to  the  meridional  distance,  when  a  ship  sails  due 
east  or  west. 


Digitized  by 


Google 


^8  NAVIGATlOlf. 


Problem  I. 


Given  the  Difference  ofLongiiude  between  two  Places,  both  tii  the  same 
PardUel  qf  Latiixidey  to  find  their  Distance^ 

Ruus. 

As  radius,  is  to  the  co-sine  of  the  latitude;  so  is  the  difference  of  long!-* 
fude,  to  the  distance. 

Example* 

Required  the  distance  between  Portsmouth,  in  longitude  1?6'  W,^  and 
Qreen  Island,  Newfoundland,  in  longitude  55?35'  W.j  their  oomtnon  lati- 
tude being  50?47  C  N.  ? 


Long,  of  Portsmouth=i  1?  6^W. 
Long.of Green  l8land=55 .  35   W, 


Diff.  of  long.  =      .    54?29I  =3269  miles. 


Sohi&on* 

In  the  right-angled  triangle  A  B  C,  where  the  hypothenuse  A  C  represenU 
the  difference  of  longitude  between  the  two  giv^  places,  the  angle  A  the 
latitude  of  the  parallel  of  those  places,  and  the  base  AB  their  meridional 
distance:  given  the  side  AC  =  3269  miles,  and  the  angle  A  =  50?47C, 
to  find  the  side  A  B.'   Hence,  by  right-angled  plane  trigonometry,  problem 
1.,  page  171, 

« 
As  radius  ......    90?  0'  Or  Log,  co-secant  s     .    10.000000 

I»to  the  diff.  of  long.  A  C  '=  3269  miles         Log.  =     .    .      3. 514415 
So  is  the  lat.=the  angle  A=50?47 '  Or  Log.  «o-sine  =r   .    ,     .9.  800892 

Tothemerid.  dist.  AB  =     2066. 8  miles     Log.  =     .    ,      S.315S07 

To  find  the  Meridional  Distance  by  Jn^pecHon  in  the  general  Traverse 

Table:— 

Note.'^ThiB  case  mny  be  solved  by  Problem  L,  page  107,  as  thus  r 
.To  latitude  50?  as  a  bourse,  and  one-eleventh  of  the  difference  of  longi- 
tude  (via.  297. 2)  aa  a  distance,  the  corresponding  difference  of  latitude  is 
190. 9;  and  to  latitude  5 1  ?^  and  distance  297. 2,  the  difference  of  l^titud^ 


Digitized  by 


Google 


PAHALLU  SAILING.  219 

is  186. 9 :  hence  the  change  of  meridional  distance  (represented  by  diiFer- 
ence  of  latitude,)  to  1?  or  60'  of  latitude,  is  4^  Now,  4!  x  47^  -i-  60^ 
=  3 ' .  1 ;  this  being  subtracted  from  the  first  difiTerence  of  latitude,  because 
it  is  decreasing,  gives  187. 8;  and  187-8  multiplied  by  U,  the  aliquot 
part,  gives  2065. 8  for  the  meridional  distance ;  which  comes  within  one 
pile  of  the  result  by  calculation. 


PaofiLSM  II. 

Given  the  Distance  between  two  Places,  both  in  the  same  Parallel  of 
JLij^iude,  tojind  the  Difference  of  Ixngitude  between  those  Places. 

RULB. 

As  the  co-sine  of  the  latitude,  is  to  radius ;  so  is  the  distance,  to  the 
difiTerence  qf  longitude. 

Example* 

A  ship  from  Cape  Clear,  in  latitude  5 1  ?25 '  N.  and  longitude  9?29^  W., 
sailed  due  west  1040  miles  5  required  the  longitude  at  which  she  then 
arrived  ? 

JSobaim j--ln  the  right  angled  triangle  ABC,  let  the  bypothenuse  A  O 
represent  the  difference  of  longitude ;  the  angle  A,  the  latitude  of  the  parallel 
on  which  the  ship  sidled ;  and  the  base  A  B,  the  q 

neridiooal  distance:  then,  in  this  jangle,  there 
are  '^vmn^  the.  angle  A  as  51?25;,  and  the  base 
A  B  =  1040  miles^  to  find  the  9ide  'A  C.    Hence,  ^ 

by  right  angled  plane  trigonometry.  Problem  II  .^ 
page  172, 

As  radius  »  .  .  .  .  ^  90?  0^  0?  Log.  co-secant  a.  10.000000 
Is  to  the  merid.  dist  AB  =  1040  miles.  Log.  «  .  3.017033 
So  is  the  lat  s  die  angle  A  =  5 1  ?25 '  Or  Log.  secant  =  .     10. 205057 


To  the  difference  of  long.  A  C  =  1667. 6  miles.      Log.  =  •      3. 222090 

LongitndeofCape  Clear  ss 9?29Cwese. 

IMfftrence  of  longitude  ;i  667. 6  miles,  or  .    .^ .  48  west^ 

I^ongitade  4^  which  (be  shig  arrived  ^    • 


Digitized  by 


"SiZO  KAVI6ATIOK* 

To  find  th^  Difference  of  Longitude  by  Inspection  in  the  general 
Traverse  Table  :— 

Note,— Tliis  case  falls  under  Problem  V.,  page  111:  bence^ 

To  latitude  51?  as  a  course^  and  one^eighth  of  the  meridional  distance 
=:  ]  30,  in  a  difference  of  latitude  column,  the  corresponding  distance  is 
207 ;  and  to  latitude  52?,  and  difference  of  latitude  130,  the  diatanee  is 
211:  hence,  the  difference  of  distance  to  1?  of  latitude,  is  4  niiles.  Now, 
4'.  X  25^  -H  60^  =  1'. 6,^  which  being  added  to  the  first  distance, 
because  it  is  increasing,  gives  208. 6 ;  this  being  multiplied  by  8  (the 
aliquot  part),  gives  1668. 8  fojr  the  difference  of  longitudf. 


.     PROBf^M  III, 

Given  the  Difference  of  Longitude,  and  the  Distance  between  two  Places, 
in  the  same  Parallel  of  Z^ititudey  to  find  the  Z,atitude  of  that 
Parallel 

Rule, 

As  the  difference  of  longitude,  is  to  the  distance;  so  is  radius,'  to  the 
po-sine  of  the  latitude* 

Example, 

A  ship,  from  a  place  in  longitude  16^30'  W.,  sailed  due  east  456  miles^ 
and  then  by  observation  was  found  to  be  in  the  longitude  of  4?15t  W,; 
required  the  latitudie  of  tlie  parallel  on  which  she  sailed  ?  • 

liong.  sailed  from  =     16?30^  W. 
Long,  come  to  33     .      4. 15   W. 

Diff.oflong,  =s      «    12?15C  ^  735  rnilea* 


Mr.M4i/.  4S6 


/  &)IttHon.~In  the  right  angled  triangle  ABC,  let  the  hypothenuse  A C 
represent  the  difference  of  longitude  \  the  angle  A,  the  latitude  of  the 
parallel ;  and  the  base  A  B,  the  meridional  distance :  then,  there  are  giv^n, 
the  side  AC  =  735  miles,  and  the  leg  AB  =  456  miles,  to  find  the  angle 
A.    Hence,  by  right  angled  jrtane  trigonometry,  Problem  III.,  page  174, 


Digitized  by 


Google 


MIBDLB  XATITth>fi  SAlLtNG*  221 

As  the  diC  of  longitude  A  C  =:  735  miles.    Log.  ar.  comp.  =  7. 1337 13 
Is  to  radius  =     .    •    .    .    90?  01  Or         Log.  sine  =     .     10.000000 

50  is  the  merid.  distance  AB  =  456  miles.    Log.  =  •'    .    •      2. 658965 

Tolat.ofparall.=  ang,A=51?39a4:        Log.  co- sine  =      9-792678 

To  find  the  Latitude  of  the  Parallel  by  Inspectim  in  the  general 
Traverse  Table  :•»- 

Enter  the  Table  with  one-third  the  difference  of  longitude  =:  245  as  a 
distance,  and  one-third  tlie  meridional  distance  ==  152,  in  a  difference  of 
latitude  column ;  and  the  latitude  corresponding  to  them  will  be  found  to 
lie  between  51?  and  52?.  Now,  to  latitude  5 1  ?,  and  distance  245,  the* 
corresponding  difference  of  latitude  is  154-.2,  which  exceeds  half  the 
meridional  distance  by  2' .  2 ;  and,  to  latitude  52?,  and  distance  245,  the 
difference  of  latitude  is  150.  8, 'which  is  1 ' .  8  less  than  half  the  meridional 
distance.  .  Hence,  1 ' .  8  +  2' .  2*=:  4'  is  the  change  of  meridional  distance 
to  1?  of  latitude;    And,  as  4^  :  2\  2  ::  60'.  :  38: 3  this,  being  added  to 

5 1  ?9  gives  5 1  ?38 '  for  the  required  latitude* 


-     SOLUTION  OF  PROBLEMS  IN  MIDDLE  LATITUDE. 

SAILING. 

Middle  Latitude  Sailing  is  the  method  of  solving  the  several  cases,  or 
problems,  in  Mercator's-saiiing,  by  principles  compounded  of  plane  and 
parallel  sailing.  This  method  is  founded  on  the  supposition  \that  the 
meridional  distance,  at  that  point  which  is  a  middle  parallel  between  the 
latitude  left  and  tlie  latitude  bound  to,  is  equal  to  the  departure  which  the 
ship  makes  in  sailing  from  one  parallel  of  latitude  to  the  other.    * 

.  This  method  of  sailing,  though  not  quite  accurate,  is,  nevertheless,  suffi- 
ciently so  for  a  migle  day's  ruriy  particularly  in  low  latitudes,  or  when  the 
ship's  course  is  not  more  than  two  or  three  points  from  a  parallel.  But, 
in  high  latitudes,  or  places  considerably  distant  from  the  equator,  it  fails 
of  the.  desired:  aecuracy :  in  such  places,  therefore,  the  mariner  should 
never  employ  it  in  the  determination  of  a  ship's  place,  when  he  wishes  to 
draw  correct  nautical  conclusions  from  his  operations. 

With  the  intention  of  avoiding  prolixity  and  unnecessary  repetition,  in 
resolving  the  different  problems  in  this  method  of  sailing,  we  will  here 
briefly  give  a  general  view  of  the  -principles  en  which  the  solutions  of  those 
problems  are  founded }  as  thus  :— *  *        . 


Digitized  by 


Google 


222  NAVIGATION, 

In  the  annexed  diagram,  let  the  triangle  ABC 
be  a  figure  in  plane  sailing,  in  which  A  C  repre- 
sents the  distance,  A  B  the  difference  of  latitude, 
B  C  the  departure,  and  the  angle  A  the  course. 
Again,  let  D  B  C  be  a  figure  in  parallel  sailing, 
in  which  D  C  represents  the  difference  of  longi. 
tude,  BC  the' meridional  distance,  and  the  angle 
C  the  middle  latitude.  Hence,  the  parts  con- 
cerned form  two  connected  right  angled  triangles, 
tn  which  the  departure  or  meridional  distance  B  C 
is  a  side  common  to  both. 

Now,  in  one  of  these  triangles,  there  will  be  always  two  terms  given,  aad 
in  the  other  one  term,  at  least,  to  find  the  required  terms.  The  required 
parts  in  that  triangle  which  has  two  terms  given,  may  be  readily  found  hf 
the  analogies  for  right  angled  plane  trigonometry,  page  171  to  177;  tad^ 
hence,  the  unknown  terms  in  the  other  triangle. 

When  the  departure  B  C  is  not  under  consideration,  the  two  connected 
triangles  may  be  considered  as  one  oblique  angled  triangle,  and  resolved  as 
such.  In  this  case,  if  the  course,  distance,  middle  latitude,  and  difference 
of  longitude,  are  the  terms  in  question,  any  three  of  them  being  given^^  the 
fourth  may  be  found  by  one  direct  proportion.  Thus,  in  the  oblique  angled 
triangle  A  CD,  the  side  AC  is  the  distance  3  the  angle  A,  the  course ;  the 
angle  B  C  D,  the  middle  latitude ;  and,  consequently,  the  angle  D  its  com- 
plement, and  the  side  D  C  the  difference  of  longitude.  Now,  if  any  three 
of  these  be  known,  the  fourth  may  be  found  by  one  of  the  following  analo- 
gies; via., 

1.,  As  co-sine  middle  latitude  =  C  :  sine  of  course  s  A  ::  distance  = 
AC  :  difference  of  longitude  =:  PC 

2.  As  sine  of  course  =  A  !  co-sine  middle  latitude  s  C  ::  difference  of 
longitude  =  D  C  :  distance  =  'A  C. 

3.  As  distance  =  A  C  :  difference  of  longitude  =  D  C  : :  co-sine  of 
middle  latitude  =  C  :  sine  of  course  =  A. 

4.  As  difference  of  longitude  =  D  C  *.  distance  =  A  C  : :  sine  of  course 
=  A  :  co-sine  of  middle  latitude  =  C. 

Again,  if  the  course,  middle  latitude,  difference  of  latitude^  and  diffeitnoe 
of  loBgitude^  be  the  terms  under  consideration,  the  resulting  analogies 
will  be, 

5.  As  difference  of  latitude  =  AB  !  difference  of  longitude  =  DC  :: 
co-sine  of  middle  latitude  =  C  :  tangent  of  course  =3  A* 

6.  As  difference^of  longitude  =  DC  :  difference  of  latitude  =  AB  :; 
tangent  of  course  =  A  *•  co-sine  of  middle  latitude  =  C» 


Digitized  by 


Google 


MIDDLB  LATrrUBH  SAILING. 


228 


7f  As  co-sine  of  middle  latitude  =  C :  tangent  of  course  =8  A  :: 
difference  of  latitude  =  A  B  :  difference  of  longitude  ss  A  C. 

8.  As  tangent  of  course  =  A  :  co-sine  of  middle  latitude  =s  C  : :  differ- 
ence of  longitude  =  D  C  '.difference  of  latitude  =  A  B. 

In  these  four  analogies,  it  is  evident  that  the  course  must  be  a  tangent, 
because  the  difference  of  latitude  AB  is  concerned. 

Note.— nSince  the  sine  complement  pf  the  middle  latitudes  the  angle  D,  is 
expressed  directly  by  the  co-sine  of  the  angle  BC  D,  therefore,  with  the  view 
of  abridging  the  preceding  analogies,  the  co-sine  of  the  middle  latitude  has 
beeu  used  instead  of  its  sine  complement;  and,  in  the  operations  which 
follow,  the  same  term  will  be  invariably  employed. 

IZemarfc.— *The  middle  latitude  between  two  places  is  found  by  taking 
half  the  sum  of  the  two  latitudes,  when  they  are  both  of  the  same  namci  or 
half  their  difference  if  of  contrary  names. 


ProIbjlbm  I.  ' 

Gwen  the  LcOUfides  and  LmgUudes  o/two  Places,  tofiid  the  Cwree 
and  Distance  between  them.    . 


Exajnpk. 

Required  the  course  sjid  distance  from  Oporto, 
in  latitude.  4 1?9'  N.  and  longitude  &?37'  W. 
to  Porto  Santo,  in  latitude  33  ?3:  N.  and  longi- 
tude 16?  17^  W.?  .  , 


Latitude  of  Oporto  41?  9'  N.  Longitude  s= 
Lat.  of  Porto  Santo  33.   3   N.  Longitude  = 


.    8?37^W. 
.  16. 17   W. 


Diff.  of  latitude  =      8?  6ts486  miles.  Diff.  of  long.  =  7?40f  =460ms. 
Sum  of  latitudes  =  74?12C  h-2  =  37?6:  =  the  middle  latitude. 

To  find  the  Course  =  Angle  A  :— 

Here,  since  the  departure  is  not  in  question,  the  parts  concerned  come 
imder  the  5th  analogy  in  page  ^22  :  hence, 


Digitized  by 


Google 


224 


NAVlCATIOlf. 


As  the  diff.  of  latitude  =  486  miles,  Log.  ar.  coinp.  =  7. 313364 
Istothediff.  ofIong.=:  460  miles.  Log.  =  .  •  .  2.662758 
So  is  the  mid.  latitude  =  37'?6:  Log.  co-sine  =    .    9, 901776 

To  the  course  =*     .    •    87?3C  Log.  tangent    =    9.877898 

To  find  the  Distance  =  A  C  :— 

The  course  being  thus  found,  the  distance  may  be  determined  by  trigo- 
nometry. Problem  II.,  page  172 :  hence, 

As  radius  =    .    .    .    90^QC  Log.  co-secant  =     10.000000 

Istothediff.oflat.  =s  486  miles.  Log.  =  .  .  •  2.686636 
So  is  the  course  =     .    37°3'  Log.  secant  —     •     10. 097937 


To  the  distance  =     •    608. 9  miles.  Log.  =£ 


2. 784573 


Hence,  the  true  course  from  Oporto  to  Porto  Santo  is  S.  3/^3'  W.,  or 
S.W.  i  S.  nearly,  and  the  distance  609  miles* 

To  find  the  Course  and  Distance  by  Inspection  in  the  general 
Traverse  Table  :— 

To  middle  latitude  =  37?  as  a  course^  and  one-fourth  the  difference  of 
longitude  ^  115,  as  a  distance,  the  corresponding  difference  of  latitude  is 
91. 8  =  the  meridional  distance. .  Now,  one-fourth  the  difference  of  lati- 
tude =  121.5,  and  the  meridional  distance  91.  8  in  a  departure  column, 
are  found  to  agree  nearest  at  37?,  under  distance  152.  Hence,  the  course 
is  S.  37?  W.,  and  the  distance  152  X  4  =  608  miles. 


Problem  IL 

Gvoen  the  Latitude  and  Longitude  of  the  Place  sailed  from,  the  Course, 
and  Distance;  to  find,  the  Latitude  and  Longitude  of  the  Place 
come  to. 


Example. 

A  ship  from  Corvo,  in  latitude  39?41 '  N.,  and 
longitude  31?3:  W.,swledN.,E.  ^E.,  590 miles; 
required  the  latitude  and  longitude  come  to  ? 


Digitized  by 


Google 


MIDDLB  LATITUDE  SAILING.  225 

To  find  the  Difference  of  Latitude  =  AB  ;— 

Here  the  course  s=  A,  and  the  distance  =  A  C,  being  given,  the  differ- 
ence of  latitude^  AB  may  be  found  by  trigonometry.  Problem  I.,  page 
171 ;  as  thus : 

As  radius  =     .    .    .    90?0!  Log,  co-secant  =     10.000000 

Is  to  the  distance  =s       590  miles.     Log.   =      .     .    •     2. 770852     . 
So  IS  the  course  =     .      4|  points,  Log.  co-sine  =     •    9. 802359 

Tothediff.oflat.  =    374.3mile8,   Log.  =      .    .    •    2.573211 

Utitudelcfk= 39941:N 39?41^N. 

Diff.  of  lat.  =  374. 3  miles,  or  =    6. 14   N.        Half  =     3.   7   N. 


Latitude  come  to  s  .    •    .    .    45?55:N.    Mid.lat.s  42948^ 


To  find  the  Difference  of  Longitude  s  C  D  :— 

Here,  since  the^departure  is  not  concerned,  the  parts  in  question  come 
under  the  1st  analogy  in  page  222  :  hence. 

As  the  mid.  lat^  s  •  .  42?48f  .  Log.  secant  »  10.134464 
Is  to  the  course  =s  •  •  4^  points.  Log.  sine  =  •  9. 888185 
So  is  the  distance  =     .      590  miles.      Log.  =  •    .    .    2. 770852 

To  the  diff.  of  longitude  s  621. 6  miles.  Log.  =  .    .  .  .    2. 793501 

Longitude  left  s 31?8fW. 

Diff.  of  longitude  s  621 . 6  miles,  or  «       10. 22   E. 

Longitude  come  to  =     ......    20?41^W. 

To  find  the  Difference  of  Latitude  and  Difference  of  Longitude  by 

Inspection  :— 

Under  or  over  one-fifth  of  the  given  distance  =:  1 18,  and  opposite  to  the 
course  ^  4|  points,  is  difference  of  latitude  74. 9,  and  departure  91. 2. 
Tabular  difference  of  latitude  74. 9  x  5  =  374. 5,  the  whole  difference  of 
latitude;  whence  the  latitude  in,  is  45? 55'  N.,  and  the  middle  latitude 
42?48C.  Now«  to  middle  latitude  42?,  and  departure  91.  2,  in  a  latitude 
column,  the  corresponding  distance  is  123  miles;  and  to  middle  latitude 
43?,  and  departure  91.  2,  the  distance  is  125  miles  :  hence,  the  difference 
of  distance  to  1?  of  latitude,  is  2  miles;  and  2^  x  48'  -i- 60^  =  r.6, 
which,  added  to  123,  gives  124. 6 ;  this,  being  multiplied  by  5  (the  aliquot 
part),  gives  623  miles  ^  the  diffierence  of  longitude,  or  10?23'  E. 

Q 

Digitized  by  VjOOQ IC 


226 


KAVIOATIOK. 


Problem  III. 

« 

Gioen  both  LaAtudi^  and  the  Coune;  to  find  the  JXttanee  and  the 

L(mgitude  tit. 

JEyompfe. 

A  ship  from  Brava,  in  latitude  14t46'  N*,  and 
longitude  24?46^  W.^  sailed  S.E.  b.  S.,  until,  by 
obserration,  she  was  found  to  be  in  latitude 
10?30C  N.j  required  the  distance  sailed  and  her 
present  longitude  ? 

Lat.ofBrava=:14?46^N.       .    .    .     14?46^N. 
Lat.byob8.  =  10.30  N.       .    .    .     10.30  N. 

Diflf.oflat-=    4n6i=256m.Sum=25?16r 

Middle  latitudes  12?381 

To  find  thfc  Distance  =  A  C  r— 

With  the  course  s  A,  and  the  difference  of  latitude  ss  AB,  the  distance 
is  found  by  trigonometry,  Problem  II.,  page  172 }  as  thus : 

A&rddius  es     ....    90?0^  Log.  co*s6cant  =  .  10. 000000 

Is  to  the  diif.  of  latitude  =  256  miles  .    Log.  =      •    •    •      2, 408240 
So  is  the  course  =v  •    •    •    3  points,        Log.  secant  »    »    10. 080154 


To  the  distance  ==  • 


307. 9  miles.  Log,  = 


2.488394 


To  find  the  Difference  of  Longitude  =  C  D :— • 

Here,  since  the  departure  b  not  in  question,  the  parts  concerned  fall 
under  the  7th  analogy,  page  222  :  hence. 


As  the  middle  latitude  s  12?38: 
Is  to  the  course  :s  •    .    8  points. 
So  is  the  diff.  of  lat  ^     256  miles, 


Log.  secant  9  10.010644 
Log.  tangent  a  9.824898 
Log.  «    .    4      2.408240 

To  the  diff.  of  long.  =  175. 2  miles.         Log.  =    .    .      2.243777 


Longitude  of  Brava,  the  place  staled  from  =  24?46^  W. 
Difference  of  longitude  «  175. 2  miles,  or    =     2. 55   E. 


Longitude  of  the  ship  < 


21?51^W* 


Digitized  by 


Google 


MIBBLB  LATnum  SAILING.  227 

To  find  the  Distance  sailed,  and  the  Difference  of  Longitude^  by 
Inspection  :— 

To  the  course  3  points,  and  half  the  difference  of  latitude  sss  128^ -the 
distance  is  154,  and  the  departure  85. 5.  Now,  154  X  2  =s  308  miles,  is 
the  required  distance.  Again,  to  middle  latitude  12?,  and  departure  85. 5, 
in  a  latitude  column,  the  corresponding  distance  is  87;  and  to  latitude  13% 
and  departure  85. 5,  the  distance  is  88 :  hence,  to  middle  latitude  12'?38C, 
and  departure  85. 5,  the  distance  is  S7i ;  the  double  of  which  es  175  miles^ 
is  the  difference  of  longitude,  as  required. 


Probuem  IV. 

Given  the  Latitude  and  Longitude  qftbe  Place  sailed  fiom,  the  Couree, 
and  the  Departure;  to  find  the  Distance  sailed^  and  the  Latitude  and 
Longitude  of  the  Place  cahne  to. 


D 


Example. 

A  ship  from  Cape  Finisterre,  in  latitude 
42?54^  N.^  and  longitude  9?16(W.,  sailed 
N.W.  b.  W.,  till  her  departure  was  468  miles; 
required  the  distance  sailed,  and  the  latitude  and 
longitude  come  to  ? 


To  find  the  Distance  »  AC:~ 

With  the  course  =  A.  and  tiie  departure  B  C,  the  distance  may  be  found 
by  trigonometry.  Problem  IL,  page  172 ;  Jis  thus : 

As  radius  = 90?0i  Log.  co-secant  =  10.000000 

Is  to  the  departure  =     •    .    468  miles.    Log.  =      ...    2. 670246 
So  is  the  course  =      .    .    •      5  points.    Log.  co-secant  »  10.080154 

To  the  distance  s     .    .    .  562. 9  miles.  Log.  =      ...    2. 750400 

To  find  the  Difference  of  Latitude  =  AB  :-^ 

With  the  course  =  A,  and  the  departure  B  C,  the  distance  is  found  by 
trigonometry.  Problem  IL,  page  172 ;  as  thus : 

a  2 

Digitized  by  VjOOQ IC 


As  the  course  =:     .    .    .    »  5  points.  Log.  co-tangent  =  9. 824893 

Is  to  the  departure  =       •     •  468  miles.  Log.  =3^     .     •    •    2. 670246 

So  is  radius  = 90?0:  Log.  sine  =  .    •10.000000 

To  the  diff.  of  latitudes      .  312.7  Log.  =      •    .    .    2.495139 

Latitude  of  Cape  Finisterre  =r  42? 54:  N 42?54^  N. 

DiflF.  of  lat.  s=  313  miles,  or  =  5. 13   N,        Halfdiflf.of  lat.=  2.36   N. 

Latitude  of  the  ship  =i    .    •  48?  7^  N.        Middle  lat.  s=     45?30: 


To  find  the  DiiFerenee  of  Longitude  s  C  D  :-^ 

With  the  middle  latitude  =  B  CD,  and  the  departure  B  C,  the  differ- 
ence of  longitude  is  found  by  trigonometry.  Problem  II.,  page  172. — 

As  radius  =       .     •    .     •    90?  0'  Log.  co-secant  =  10.000000 

Is  to  the  departure  =  .     .      468  miles.      Log.  =       .     .      2. 670246 
So  is  the  mid.  lat  =    .    .    45?30:  Log.  secant  =       10. 154338 

Tothe  diff.oflong.  =     .    667. 7  miles,    Log.  =  .    .    .      2.824584 

Longitude  of  Cape  Finisterre  =     •    •    •    9?16^  W. 
Difference  of  long.  =  667. 7  miles,  or  =    1 1.   8   W. 

Longitude  of  the  ship  = 20. 24    W. 

To  find  the  Distance,  Difference  of  Latitude,  and  Difference  of  Longitude, 

by  Inspection : — 

To  course  S  points,  and  one-fourth  of  the  departure  =117,  the  distance 
is  141,  and  the  difference  of  latitude  78. 3.  Now,  141  x  4  =  564  miles, 
thedistance,  and  78.3  x  4  =  313.2,  or  5?  13',  the  difference  of  lati- 
tude ;  whence  the  latitude  in,  is  48?7*N.,  and  the  middle  latitude  45.?30'. 
Again,  to  middle  latitude  45?,  and  one-fourth  the  departure  =  117,  in  a 
latitude  column,  the  distance  is  166;  and  to  middle  latitude  46?,  and 
departure  117,  the  distance  is  168  :  hence,  to  middle  latitude  45?30^,  and 
departure  117,  the  difference  of  longitude  is  167  x  4  =  668  miles;  nearly 
the  same  as  by  calculation. 


UIDDLB  LATITt7J>B  8AILIN0. 


229 


Problsm  v. 

Given  both  LatUudes  and  the  Distance  ;  to  find  the  Course  and  Differ- 
ence of  Longitude. 


Example. 

A  ship  from  St  Agne«,  Scilly,  in  latitude  49e54C 
N.,  and  longitude  6?  19^  W.,  sailed  320  miles  between  the 
south  and  west,  and  then,  by  observation,  was  found  to 
be  in  latitude  45?8C  N.;  required  the  course,  and  the 
longitude  come  to  ? 


Latitude  of  St  Agnes  =     49?  54 '  N. 
Latitude  of  tlie  ship   =     45*    8    N. 


49?54C  N. 
45.   8    N. 


Difference  of  latitude   =       4? 46 f  =  286  miles.   Sum  =  95?  2". 

Middle  latitude  =     47?3i: 
To  find  the  Course  =  A  :— 

With  the  distance  A  C,  and  the  difference  of  latitude  s  AB,  the  course 
may  be  found  by  trigonometry,  Problem  IIL,  page  174 ;  as  thus  : 


As  the  distance  =  .  • 
Is  to  radius  =  .  .  . 
So  is  the  diff.  of  lat.  =  • 


320  miles,     Log.  ar^  comp.  ==    7«  494850 

90?  0^  or     Log.  sine  =    .    .  10.000000 

286  miles.     Log.  =      .    .     .     2.456366 


To  the  course  s  .    .    •    .    26?39'  6?    Log.  co-sine  =   •    9.951216 
To  find  the  Difference  of  Longitude  =  C  D  :— 

With  the  course,  middle  latitude,  and  distance,  the  difference  of  longi*- 
tude  is  found  by  the  Ut  analogy,  page  222  ;  as  thus : 


As  middle  latitude  =s 

Is  to  the  course  =     . 

'  So  is  the  distance  = 

To  the  diff,  of  long.  =s 


47?31'  01  Log.  secant  =  10.170455 
26.39.  6  Log.  sine  =  .  9.651825 
320  miles.    Log.  =       .    .       2.505150 


212,5        Log. 


Digitized  by 


2.327430 

Google 


230 


KAVlGATtOV. 


Longitude  of  St.  Agnes  =     .    .    .     6?19^  W. 
Diff.  of  long.  =  212. 6  miles,  or  =      3. 33    W. 

Lons^tttde  of  the  ship  =s    ....    9?S2:  W. 

The  course  is  S.  26?39C  W.,  or  S.S.W.  i  W.,  nearly. 

To  find  the  Course  and  Difference  of  Longitude  by  Inspection : — 

To  half  the  distance  =  160^  and  half  the  difference  of  latitude  =  143, 
the  course  nearest  agreeing  is  27»  and  the  departure  72. 6.  Now,  to 
middle  latitude  47?  as  a  course,  and  departure  72. 6,  in  a  latitude  column, 
the  distance  is  106 ;  and  to  middle  latitude  48?,  and  departure  72. 6,  the 
distance  is  108  c  hence,  the  differetice  of  distance  to  I?  of  latitude,  is  3 
miles;  therefore,  3^  x31'-i-60  =  r.5,  which,  added  to  105,  makes 
106.5  :  this,  being  multiplied  by  2,  gives  213  miles  =  the  difference  of 
longitude. 


PaOBJLBBC  VI. 

Given  one  Latitude,  Distance,  and  Departure  ;  to  find  the  other  LatUnde, 
the  Course,  and  the  Difference  qf  Longitude. 

Example. 

A  ship  from  Cape  B^joli,  Minorca,  in  latitude 
40?3:  N.,  and  longitude  3?52'  B.,  sailed  280 
miles  between  the  north  and  east,  upon  a  direct 
course,  and  made  186  miles  of  departure;  re-  . 
quired  the  course,  and  die  latitude  and  longitude 
come  to  ? 

To  find  the  Course  =a  A  :— 

The  distance  ss  A  C,  and  the  departure  B  C,  being  given,  the  course 
may  be  found  by  trigonometry,  Problem  III.,  page  174;  as  thus : 


As  the  distance  s  . 
Is  to  radius  s=  •  • 
So  is  the  departure  = 

To  the  course  =?    •    • 


280  miles.    Log.  ar«  comp.  ss  7. 552842 

90?  OC  or     Log.  sines    .    lO.OOOOOO 

186  miles,    Log.  =      .    .      2.269513 


41?87^39r    Log.^ne 


Digitized  by 


9.823855. 

Google 


MIDDLB  LATITITDB  SAILING.  33) 

To  find  the  DiflFerence  of  Latitude  =  AB  :— 

The  course  =  A^  and  the  distance,  being  thus  known,  the  difference  of 
latitude  may  be  computed  by  trigonometry,  Problem  III.,  page  174««^ 

As  radius  =      ...    90?  0^  01  Log.  co-secant  =  10. 000000 

Is  to  the  distance  ac    .     280  miles,  Log.  >»     .     .    .     2..447158 

So  is  the  courses.     .    41?37'39?  Log. co-sine  =  •    9.873599 

To  the  diff.  of  lat.  =  .      209. 3  miles,        Log.  =     ...    2. 320757 

Latitudeof  Cape  Bajoli  =  .    .    40?  3^  N 40?  3'    N. 

Diff.  of  lat.  =  209. 3  miles,  or  s  3.  29  N.  Half  =:     .  1. 44i   N. 

Latitude  come  to  =  ....    43?32^  N.  Middle  latitude  ss  41  ?47i«N. 

To  find  the  Difference  of  Longitude  =  *C  D : — 

The  middle  latitude  =  angle  BCD,  and  the  departure  B  C,  being  given, 
the  difference  of  lon^tude  may  be  found  by  trigonometry,  Prblblem  II., 
page  172  i  as  thus: 

Aa  radius  s    ....    90?  0^  Log.  co-secant  s     10.000000 

la  to  the  departure  s     .     186  miles,        Log.  s       •    »    .       1.269513 
So  is  the  mid.  lat.  s:     .    4 1  ?47i '  Log.  secant  =      .    10. 1 275 10 


To  the  diff.  of  long.  =  .    249. 5  miles.    Log.  =      ...      1. 397023 

Longitude  of  Cape  Bajoli  == 3?52^  E. 

Diff.  of  long.  =  249.  5  miles,  or  s  •    •    .    4.  9  E. 

Longitude  come  to  ........    8.    1  E. 

The  course  is  N.  41  ?38^  E.,  or  NJB.  i  N.,  nearly. 

To  find  the  Course,  Difference  of  Latitude,  and  Difference  of  Longitude, 

by^  Inspection  !^ 

The  cBstanee  280,  and  depatture  186,  are  found  to  agree  between  41  ? 
and  42?,  ^d  the  corresponding  difference  of  latitude  208. 1 :  whence  the 
middle  latitude  is  41?46'.  Now,  to  middle  latitude  41?,  and  departure 
186,  m  a  latitude  column,  the  corresponding  distance  is  247  j  and  to  lati- 
tude 42?,  and  departure  186,  the  distance  is  250 :  hence,  the  difference  of 
distance  to  I?  of  latitude,  is  3  miles;  and  3'  x  46  h-  60'  =  2'.3,  which, 
added  to  247»  gives  249. 3= the  difference  of  longitude,  as  required ;  which 
pearly,  igr^  Witi\  th^  result,  by  calculatioo. 


Digitized  by 


Google 


232  NAVIGATION. 


-  Paoblbm  VII. 

Gwen  both  Latitudes  and  Departures  to  find  the  Course,  Distance,  and 
Difference  of  Jj)ngitud£. 

Example. 

A  ship  from  Cape  Agulhas,  in  latitude  34?55  C 
S.,  and  longitude  20?  18'  £.^  sailed  upon  a  direct 
course  between  the  south  and  east,  till  she  was 
found,  by  observation,  to  be  in  latitude  40?47* 
S.,  and. to  have  made  436  miks  of  easting; 
required  the  course,  distance,  and  longitude  at 
which  the  ship  arrived  ? 

Latitude  of  Cape  Agulhas  =  34?55'S.     .......    34?55^S. 

Latitude  of  the  ship  .=      •    40.47   S .    40.47  S. 


Diflf.  of  latitude    =       5?52' =352  miles.      Sum  =     75.42 

Middle  latitude  =37?5 1 : 

To  find  the  Course  =  Angle  A : — 

Here,  the  difference  of  latitude  =  A  B,  and  the  departure  B  C,  being 
given,  the  course  is  jfound  by  trigonometry.  Problem  IV.,  page  175 ;  as 
thus : 

As  the  diiF.  of  lat.  =?  352  miles.  Log.  ar.  comp.  =  *  7*453457 
Is  to  radius  =  .  .  .  90?0^0r  Log. sines  .  .  10.000000 
So  is  the  departure  =  •  436  miles.     Log.  =    .     •    .      2. 639486 

To  the  courses       .    .    5l?5:5r    Log.  tangent  =      10.092943 

To  find  the  Distance  =.  A  C  :— 

With  the  course,  thus  found,  and  the  difference  of  latitude  A  B,  the 
distance  may  be  computed  by  trigonometry.  Problem  IV.,  page  175  : 
hence, 

As  radius  =  .  .  .  90?0<0r  Log.  co-secant  =  10. 000000 
Is  to  the  diff.  of  lat.  s  352  miles,  Log.  =  i  .  .  2. 546543 
So  is  the  course  =    .    5 1  ?5 '  5  r    Log.  secant  =    ,10. 201922 


To  the  distance  =    560. 4  miles,  Log«  s     »    •    .    2. 748465 

/Google 


Digitized  by ' 


MIDDI.B  LATITUDB  SAILING.  233 

Hence^  the  course  is  S.  51?5'  E.,  or  S.E.  i  B.^  nearly,  and  the  distance 
560. 4  miles. 

To  find  the  Difference  of  Longitude  =  C  D  :— 

With  the  middle  latitude  =  B CD,  and  the  departure  B  C,  the  differ- 
ence of  longitude  is  found  by  trigonometry,  Problem  IV.,  p^  175]  as 
thus : 

As  radius  s  •  ,.  .  90?  0^  Log.  co- secant  =  10.000000 
Is  to  the  departure  =  436  miles.  Log.  =  •  .  .  2. 639486 
So  is  the  middle  lat.  =:  37?5  H     Log.  secant  =s     .  10. 102582 

To  the  diff.  of  long.=s552. 2  miles.  Log.  =     .    .    ,    2. 742068 

Longitude  of  Cape  Agulhas  =   «     .     «     .     20?  1 8 '  E. 
Diff.  of  longitude  =  552. 2  miles,  or  »    .      9. 12  £. 


Longitude  at  which  the  ship  airived  =:     .    29.30  E. 

To  find  the  Course,  Distance,  and  Difference  of  Longitude,  by 
Inspection : — 
Half  the  difference  of  latitude  =  176,  and  half  the  departure  =  218, 
are  found  to  agree  nearest  at  51?  under  or  over  distance  280 :  hence,  280 
X  2  =  560  miles,  is  the  distance.    Again,  to  middle  latitude  37?  as  a 
course,  and  departure  218,  in  a  latitude  column,  the  corresponding  distance 
is  273 ;  and  to  latitude  38?  and  departure  218,  the  distance  is  277 :  hence, 
.the  change  of  .distance  to  1?  of  latitude,  is  4  miles.    Now,  4^  x  51.^  -h 
60  =  3' .  4,  which,  added  to  273,  gives  276. 4  ;  and  this,  being  multiplied 
by  2,  gives  552.  8  miles ;  which  very  nearly  corresponds  with  the  result  by 
calculation. 

-      .  -    .    ■        ■  ■■■..,  ■■ .- — — ^  ■  ■ 

Problem  VIII. 

6w€U  one  LatUude,  Departure,  and  Difference  of  Longitude  ;  to  find 
the  other  Latitude,  Cour$e,  and  Distance. 

Example. 

A  ship  from  the  Snares,  New  Zealand,  in  lati- 
tude 48?3C  S.,  and  longitude  166?20:  E.,  sailed 
upon  a  direct  course  between  the  south  and  west, 
till  she  was  found  byjobservation  to  be  in  longitude 
151?27-  E.,  and  to  have  made  546  miles  of 
departure;  required  the  latitude  come  to,  the 
course  steered,  and  the  distance  sailed  ? 


Digitized  by 


Google 


284  NAVIQATfON. 

Longitude  of  the  Snares  =     .    .    166?20fE. 
Long,  of  the  ship  by  observation  =  151. 27   E. 

Difference  of  longitude  s  •    •    •       1 4  ?  53 '  :s  893  miles. 

To  find  the  Middle  Latitude  s  the  Angle  BCD:— 

With  the  departure  =  B  C^  and  the  difference  of  longitude  s=  C  D^  the 
angle  of  the  middle  latitude  may  be  found  by  trigonometry,  Problem  IIL, 
page  174;  as  thus: 

As  the  diff.  of  long.  =  893  miles.  Log.  ar.  comp.  =  7*  049148 
Is  to  radius  s:  .  90?  O:  Or  Log.  sines  .  10.000000 
So  is  the  departure  =  546  miles,    Log.  =       .    «      2. 737193 


To  the  mid.  lat.  =  52?  1 8 '.  28r        Log.  co-sine  sa     9. 78634 1 


Twice  mid.  lat  =  104?37'  Of  nearly. 
Lat.ofthe  Snares=48.   3.   OS. 


Latitude  come  to  =56?34f  O^S. 


Diff.  of  latitude  »     8?3l!  0?s  511  miles. 

To  find  the  Course  =c  the  Angle  A  :— 

With  the  difference  of  latitude  A  B,  and  the  departure  B  C,  the  eourat 
may  be  found  by  trigonometry,  Problem  IV.,  page  175  ;  as  thus : 

As  the  diff.  of  lat.  =  511  miles.  Log.  ar.  comp.  =  7*  291579 
Is  to  radius  s  •  .  90?  0'  Or  Log.sine^  .  10.000000 
So  is  the  departure  =    546  miles,    Log.  =     .    •      2. 73/193 

To  the  course  as  .    .    46?5aM8r  Log.  tangent  at  10.026772  . 

To  find  the  Distance  ss  A  C  :~ 

Wiih  the  angle  of  the  course,  thus  found,  and  the  difference  of  lati^de 
A  B,  the  distance  may  be  computed  by  trigonometry.  Problem  IV.,  page 
175 :  hence. 

As  radius  «  .  .  90?  0^  Or  Log.  co-secant  s  10. 000000 
Is  to  diff.  of  lat  =  511  miles.  Log.  &r  .  .  .  2.708421 
So  is  the  course  =      46?53M8r  Log.secant  =    .  10. 165378 

To  the  distance  s  •    747.  Smiles,  Log.  =:   •    .    •    2.873799 

Digitized  by  VjOOQ IC 


MIDDLB  LATITODJI  SAILING.  23$ 

Hence,  the  course  is  S,  46?  54^  W.,  or  S.W,  J  W,  nearly^  and  the  dis- 
tance 747*  8  miles. 

To  find  the  Latitude  come  to,  Course,  and  Distance,  by  Inspection  in 
the  general  Traverse  Table  :— 

One^fourth  of  the  difference  of  longitude  =  223^,  taken  as  distance,  and 
one-fourth  of  the  departure  =  136. 5,  in  a  latitude  column,  will  be  found  to 
agree  between  52?  and  53?.  Now,  to  latitude  52?,  and  distance  223,  the 
difference  of  latitude  is  137.3,  which  is  0\  8  more  than  136.5;  and  to 
latitude  53?,  and  distance  223,  the  difference  of  latitude  is  134.  2,  being 
2'.  3  less  than  136.  5  :  hence,  the  difference  of  meridional  distance  to  1? 
oflatitudeisO'.S  +  2'.3  =^3M  :  therefore,  as  3M  :  0'. 8  ::  60^: 
16 C,  which,  added  to  52?  (proportion  being  made  for  the  quarter  of  a  mile 
in  the  distance),  gives  the  middle  latitude  ss  52?  18§^  :  hence,  the  latitude 
come  to  is  56?34^  S.,  and  the  difference  of  latitude  511  miles.  Again,  to 
one-fourth  of  the  difference  of  latitude  =  127*  75,  and  one-fourth  of  the 
departure  «  136. 5,  the  course  is  47 ?>  and  the  distance  187*;  which,  mufti- 
plied  by  4,  gives  748  miles  s  the  whole  distance. 


Problem  IX« 

Gken  the  Distance,  Differenae  of  Longitude,  and  Middle  ZatUude; 
to  find  the  Course  and  both  Latitudes. 

Eaample. 

A  ship,  in  north  latitude,  toiled  500  miles  upon 
a  direct  course  between  the  south  and  west,  until 
her  difference  of  longitude  was  440  miles;  required 
the  course  steered,  the  latitude  sailed  from,  and 
the  latitude  come  to;  allowing  the  middle  latitude 
to  be 43 ?45' north? 


To  find  the  Angle  of  the  Course  =s  A  :.^ 

The  course  may  be  found  by  the  3d  analogy,  page  222,  as  thus : 

As  the  distance  =  ...  500  miles,  Log.  ar.  comp.  =  7. 301030 
Is  to  the  diff.  of  longitude  5=  440  miles.  Log.  =  ^  .  .  2. 643453 
So  is  the  middle  latitude  =5  43?45'  OT  Log.  co«sine  b     9. 858756 

*  the  courses      .    .    S.39?28:i4r  W.    Log, sine  =  9. 803239 

Digitized  by  VjOOQ IC 


To  find  the  Difference  of  LaUtude  r:  A  B  :— 
The  difference  of  latitude  may  be  found  by  the  8th  analogy,  page  222j  ai 
thus: 

As  the  course  =  .  •  .  39  ?  28 '14^  Log.  co-tangent  =  10.084350 
Is  to  the  middle  latitude  =  43.45.  0  Log.  co-sine  =:  •  •  9.858756 
Soisthediff.oflong.  =:  •    440  miles,    Log.  =     .    .    .    .    2.643453 

To  the  diff.  of  latitude  =     386  miles,    Log.  =:      ....    2. 586559 

Middle  latitude  =:      .     .         ....    43?45^  N. 

Half  the  diff.  of  lat.  =:  193  miles,  or  =:  .    .  3. 13    S. 


Latitude  of  the  place  sailed  from  = 
Latitude  of  the  place  come  to  =  . 


46?58:  N. 
40.32    N. 


SOLUTION  OF  PROBLEMS  IN  MERCATOR'S  SAILING. 

Mercator's  Sailing  is  the  method  of  finding,  on  a  plane  surfiice,  the 
motion  of  a  ship  upon  any  assigned  point  of  the  compass,  which  shall  be 
true  in  latitude,  longitude,  and  distance  sailed. 

Mariners,  generally  speaking,  solve  all  the  practical  cases  in  Mercator's 
Sailing  by  stated  rules,  called  canons,  which  they  early  commit  to  memory, 
and,  ever  after,  employ  in  the  determination  of  a  ship's  place  at  sea. 
Those  mnonSf  certainly,-  hold  good  in  most  cases ;  but .  since  they  are 
destructive  of  the  best  principles  of  science,  inasmuch  as  that  they  have 
a  direct  tendency  to  remove  from  the  mind  every  trace  of  the  elements  of 
trigonometry,  the  very  doctrine  from  which  they  were  originally  deduced, 
and  on  which  the  whole  art  of  navigation  is  founded,  the  following  observa- 
tions and  consequent  analogies  are,  therefore,  submitted  to  the  attention  of 
naval  people,  under  the  hope  that  they  will  serve  as  an  inducement  to  the 
substitution  of  the  rules  of  reason  for  the  rules  of  rote »  and  thus  do  away 
with  the  necessity  of  getting-  canons  by  heart. 

In  the  annexed  diagram,  let  the  triangle  ABC 
be  a  figure  in  plane  sailing,  in  which  the  angle  A 
represents  the  course,  A  C  the  distance,  A  B  the 
difference  of  latitude,  and  B  C  the  departure.  If 
A  B  be  produced  to  D,  until  it  is  made  equal  to 
the  meridional  difference  of  latitude,  and  D  E  be 
drawn  at  right  angles  thereto,,  and  parallel  to  B  C ; 
then  the  triangle  A  D  E  will  be  a  figure  in  Merca- 
tor's sailing,  in  which  the  angle  A  represents  the 
course,  the  side  A  D  the  meridional  difference  of  _ 
latitude,  and  the  side  D  E  the  difference  of  longi-  ^     ^ 

tud^.    Now,  since  the  two  triangles  ABC  and  A  DE  are  right  angled. 


c/ 

/    ^ 

1 

B 

/ 

D 

mhrcator's  sailing.  287 

and  that  the  angle  A  is  common  to  both;  therefore  they  are  equi- 
angular :  and  because  they  are  equi-angular^  they  are  also  similar.;  there-* 
fore  the  sides  containing  the  equal  angles  of  the  one  are  proportional  to 
the  sides  containing  the  equal  angles  of  the  other, — Euclid,  Book  VI.^ 
Prop.  4. 

Now,  from  the  relative  properties  of  those  two  triangles,  all  the  analogies 
for  the  solution  of  the  different  cases  in  Mercator's  sailing  may  be  readily 
deduced  agreeably  to  the  established  principles  of  right  angled  trigono* 
metry,  as  given  in  page  I7I9  and  thence  to  177 ;  as  thus  : — 

First,  in  the  triangle  ABC,  if  the  distance  AC  be  made  radius,  the 
analogies  will  be, 

1.  As  radius  I  distance  A  C  1 1  sine  of  the  course  A  '.  departure  EC ; 
and  I  *  co^sine  of  the  course  A  I  difference  of  latitude  A  B. 

2.  As  sine  of  the  course  A  *.  departure  B  C  !  I   radius  *.  distance  A  C  ; 
and  1 !  co-sine  of  the  course  A  I  difference,  of  latitude  AB. 

3.  As  co-sine  of  the  course  A  I  difference  of  latitude  A  B  * ;  radius  I 
distance  A  C ;  and  : :  sine  of  the  course  A  '.  departure  B  C. 

4.  As  the  distance  A  C  :  radius  : :  departure  B  C  ".  sine  of  the  course  A ; 
and  1 1  difference  of  latitude  A  B  '•  co-sine  of  the  course  A. 

Agaun,  by  making  the  difference  of  latitude  A  B  radius,  the  analogies 
will  be,  . 

5.  As  the  difference  of  latitude  AB  :  radius  : !  departure  BC  :  tangent 
of  the  course  A;  and  !  *.  distance  A  C  I  secant  of  the  course  A. 

6.  As  radius  :  difference  of  latitude  A  B  : :  tangent  of  the  course  A  I 
departure.B  C ;  and  1 1  secant  of  the  course  A  '.  distance  A  C. 

And  by  making  the  departure  B  C  radius,  it  will  be, 

7.  As  the  departure  B  C  '.  radius  , ;  difference  of  latitude  AB  t  co- tan- 
gent of  the  course  A  ;  and  1 1  distance  A  C  *.  co-secant  of  the  course  A. 

8.  As  radius  I  departure  B  C  1 1  co-tangent  of  the  course  A  :  difference 
of  latitude  A  B ;  and  ',  I  co-secant  of  the  course  A  '.  distance  A  C. 

Now,  in  the  triangle  ADE,  if  the  meridional  difference  of  latitude  A  D 
be  mrde  radius,  the  analogies  will  be, 

9.  As  the  meridional  difference  of  latitude  AD  '.  radius  ! ;  difference  of 
longitude  D  E  '.  tangent  of  the  course  A. 

10.  As  radius  I  meridional  difference  of  latitude  AD.*;  tangent  of  the' 
course  A  •  difference  of  longitude  D  E. 

And  by  making  the  difference  of  longitude  D  £  radius,  it  will  be^ 

Digitized  by  VjOOQ IC 


238  MAVIGATIOlf. 

1 1.  As  the  difference  of  longitude  D  E  :  radius  : ;  meridional  difference 
of  latitude  D  £  I  co-tangent  of  the  course  A. 

12.  As  radius  :  difference  of  longitude  D  E  !  I  co«tangent  of  the  course 
A  I  meridional  difference  of  latitude  A  D. 

Finally,  since  the  triangles  ABC  and  A  D  E  are  equi-angular  and  simi-* 
lar^  we  have, 

13.  As  the  difference  of  latitude  A  B 'departure  BC  ::  meridional 
difference  of  latitude  AD  I  difference  of  longitude  D  E. 

Hie  meridional  difference  of  latitude  is  found  by  means  of  Table  XLIII., 
by  the  same  rules  as  those  for  the  difference  of  latitude  given  at  page  214; 
as  thus : — If  the  two  given  latitudes  be  of  the  same  name,  the  difference  of 
their  corresponding  meridional  parts  will  be  the  meridional  differ^ce  of 
latitude ;  but  if  the  latitudes  be  of  contrary  names,  the  sum  of  these'  parts 
will  be  the  meridional  difference  of  latitude. 

PaoBtEM  L 

Given  the  LaHtudee  and  Longitudes  of  two  Places ;  to  find  the  Course 
and  Distance  between  them. 


2Hff.lmv.  nS,  p 


.£rampfe. 

Required  the  course  and  distance  between 
Cape  Bajoli,  in  latitude  40? 3'  N.,  and  longi- 
tude 3?52;  -E.,  and  Cape  Side,  in  latitude 
43?2'  N.,  and  longitude  5?58:  E,? 


Lat.  of  C.  Bajoli  40?  3'N.  Merid.pts.  2626.6.   Longitude  3°52^E. 
Lat.  of  C.  Sicie    43.   2  N.  Merid.pts.  2865.8.  Longitude 5. 58  E. 

Diff.  of  latitude      2?59t  Merid.diff.lat.  239.2.  Diff.  long.2i  61 


s  179  miles.  s  1 26  miles. 

To  find  the  Course  =  Angle  A  :— 

This  comes  under  the  9th  analogy,. in  page  237 :  hence. 

As  the  merid.  diff.  of  lat.  =:  239. 2  miles.  Log.  ar.  comp.  =:  7*  621239 
Istotadjusz:  .  .  .  .  90?  0'.  OIT  Log, sines  .  lO.OOOOOO 
So  is  the  diff.  of  long.  =  .      126  miles,        Log.  =:      •    •      2.100371 

To  tlie  course  =:    •    .    .    27?46:42r       Log.  Ungent  =    9.721610 


Digitized  by 


Google 


MSRCATOBfs  SAILING.  289 

To  find  the  Distance  =  A  C :— 

This  comes  under  the  6th  analogy^  in  page  237  :  hence. 

As  radius  =     ...    90?  0'  Or     Log.  co-secant  =  .    .     10.000000 
Is  to  the  diff.  of  lat  s     179  miles.    Log.  ......      2. 25285S 

So  is  the  course  s    .    27?46M2r     Log.  secant  b  .     •    .     10.0^3176 

To  the  distance  =.  202. 3  miles,    Log.  = 2.306029 

Hence,  the  true  course  from  Cape  Bajoli  to  Cape  Sicie  is  N.  27?47^  E., 
or  N.N £.  4  B.  nearly,  and  the  distance  202. 3  miles. 

To  find  the  Course  and  Distance,  by  Inspection  in  the  general 
Traverse  Table :— . 

The  meridional  difference  of  latitude  239. 2,  and  the  difference  of  longi- 
tude 126,  as  departure,  are  found  to  agree  nearest  at  28?,  which,  there- 
fore, is  the  course.  Now,  to  course  28?,  and  difference  of  latitude  179, 
the  corresponding  distance  is  203  miles ;  which  nearly  agrees  with  the 
result  by  calculation. 

Paoblbm  IL 

Giioen  the  Latitude  and  Longitude  of  the  Place  eaiied  from,  the  Couth 
and  Dittance;  to  find  the  jLatUude  and  Lmgitude  of  the  Place  come 
to. 


JHff.  limni 


Example. 

A  ship  from  Cape  Ortegal,  in  latitude  43?47' 
N.,  and  longitude  7?49:  W.,  sailed  N.W.  |  N. 
560  miles ;  required  the  latitude  and  longitude 
come  to  ? 


A 

To  find  the  Difference  of  Latitude  =  A  B  :— 

This  comes  under  the  1st  analogy,  page  237  :  hence. 

As  radius  =  .  .  .  .  90?  0'  .  Log.  co-secant  :c  .  10.000000 
Is  to  the  distance  =  .  560  miles,  Log.  =  ....  2.748188 
Sp  is  the  courses     .     .    3i  points.   Log.  co-sine  =     .     .      9.888185 

To  the  diff.  of  latitude  =  432. 9  miles.  Log.  =  •    .    .    .    .      2. 636373 

Digitized  by  VjOOQ IC- 


240  NAVIGATION. 

To  find  the  Difference  of  Longitude  =  D  E : — 
This  comes  under  the  10th  analogy^  in  page  237 :  hence^ 

As  radius  =  .  •  •  90?0:  Log.  co- secant  s  .  .  lO.OOOOOO 
l8toiherid.diff.oflat.=641  miles,Log.  s:  .....  2.806858 
So  is  the  course  =  •      3^  points^  Log.  tangent  =     .     .     •      9.914173 

Tothediff*oflbng.=526.1  miles,  Log.  = 2.721029 

Lat.of  C.  Ortegal  43^47 'N.  Mer.  pts  2927. 8.  Long.of  C.Ortegal  7^9^ W. 
Diff.lat.=433m.or7. 13  N.  Diff.long.=526m.or8. 46  W. 

Latitudecometo=51?  O'N.  Mer. pts  3568. 8  Long,  come  to  =  16^35 ^W. 

Merid.  diff.  of  lat.  =     641 . 0 

To  find  the  Difference  of  Latitude  and  Difference  of  Longitude,  by 

Inspection  :— 

To  course  3|  points,,  and  half  the  distance  =  280,  the  difference  of 
latitude  is  216.4;  the  double  of  which,  or  432.  8,  is  the  difference  of 
latitude:  hence,  the  latitude  come  to  is  51?0'  N.,  and  the  meridional 
difference  of  latitude  641. 

Now,  to  course  Sf  points,  and  one-third  of  the  meridional  difference  of 
latitude  =  213.  Jy  the  corresponding  departure  is  175. 4,  proportion  being 
made  for  the  excess  of  the  given,  above  the  tabular  difference  of  latitude ; 
then  175. 4x3  =  526  miles ;  which,  therefore,  is  the  difference  of  longi- 
tude. 


Problem  III. 

Gwen  tlie  Latitude  and  Longitude  ofilie  Place  miUd  from,  ilie  Course, 
and  tlie  Departure,  to  find  the  Distance  sailed,  and  the  Latitude  and 
Longitude  of  the  Place  come  to. 

Example. 

A  ship  from  Wreck  Hill,  Bermudas,  in  latitude 
32?15'  N.,  and  longitude  64?47^  W.,  sailed 
S.W.  i  W.,  and  made  340  miles  of  departure ; 
required  the  distance  sailed,  and  the  latitude  and 
longitude  come  to  ?  Y."    ^^^  ^^ 


Digitized  by 


Google 


mbrcator's  sailing.  241 

To  find  the  Distance  =  A  C  :— 

This  comes  under  the  8th  analogy,  page  237 :  hence. 

As  radius  =  •  .  90?0'  •  Log.' co-secant  s  10.000000 
Is  to  the  departure  s  340  miles,  Log.  =  .  •  •  2. 53 1479 
So  is  the  course  =      4^  points,  Lojg.  co*secant  ==     10. 11 1815 


To  the  distance  =  439. 8  miles.  Log.  =     .    .    .      2. 643294 

To  find  the  Difference  of  Latitude  ==  A  B  :-— 

This  comes  under  the  8th  analogy,  page  237 :  hence. 

As  radius  =  .  90?  .  Log.  co-secant  =  .  lO.QOOOOO 
Is  to  the  departure  340  miles.  Log.  =:  •  .  «  •  2. 53 1 479 
So  is  the  course  =  4^  points,  Log.  co- tangent  =    •      9. 914173 

To  the  diff.  of  lat.=279  miles.  Log.  =      .    .    .    .       2. 445652 

Lat.  of  Wreck  Hill,  Bermudas,    329 15 1  N.        Merid.  parte  =     2046. 1 
Diff.  of  latitude  =  279  miles,  or    4. 39    S. 


Latitude  come  to  =     .    .    .      27?36'  N.        Merid.  parts  =s     1724.0 
Meridional  difference  of  latitudes.    •    ; 322.1 

To  find  the  Difference  of  Longitude  D  E  :— 

This  comes  under  the  10th  analogy,  page  237  :  hence, 

As  radius  =  .  .  90?0'  Log.  co-siecant  =  10.000000 
Is  to  merid.  diff.  of  ]at.=322. 1  miles.  Log.  =  .  2. 507991 
So  is  the  coarse     =     4|  points.  Log.  tangent  =    .10. 085827 

To  the  diff.  of  long.  =s  392. 5  miles.  Log.  s      .    .    2. 593818 

Longitude  of  Wreck  Hill,  Bermudas,  =    .    .    64?47'  W. 
Difference  of  longitude  =  392^  5  miles,  or  =       6.32    W* 

Longitude  come  to  =  .    ...    ^    ...    71^19^  W. 
The  distance  sailed  b  440  miles,  very  nearly. 

To  find  the  Distance  sailed,  and  the  Latitude  and  Longitude  come  to, 

by  Inspection : — 

To  the  course  4i  points,  and  half  the  departure  =  170,  the  corresponding 


Digitized  by 


Google 


Z4as 


NAVIGATION. 


difference  of  ladtude  is  139.6,  under  distance  220  j  twice  the  latter,  or 
440  miles,  is,  therefore,  the  distance  sailed ;  and  twice  139. 6  =  279. 2 
miles,  or  4?39^,  is  the  difference  of  latitude:  whence  the  latitude  in,  is 
27?36'  N.,  and  the  meridional  difference  of  latitude  322. 1.  Now,  to 
course  4|  points,  and  half  the  meridional  difference  of  latitude  a  161  miles, 
in  a  latitude  column,  the  corresponding  departure  is  196. 3;  the  double  of 
which,  or  392. 6  miles,  is  the  difference  of  longitude :  hlBnce^  the  lou^tude 
cometoi8  7ni9iC  W. 


Problem  IV. 

Oiven  both  Latitudes  and  the  Course;  to  find  the  Distance  and  the 

JLongiittde  m. 


Example. 

A  ship  from  the  east  end  of  Martha's 
Vineyard,  in  latitude  4l?2H  N.,  and  lon- 
gitude 70?24C  W.,  sailed  S.E.  i  S.,  and,  by 
observation,  was  found  to  be  in  latitude 
32?21 '  N. ;  required  the  distance  sailed^  and 
the  longitude  at  which  she  arrived  ? 


Lat.  of  the  east  end  of 

Martha's  Vineyard  =  41?21(  N. 
Lat.  in,  by  observation  =32.21    N. 


Ih/f  l^nq. 


Mend.  part^!=    2729.5 
Merid.part9=    2053.2 


Difference  of  latitude  =     9?  OC  =540  miles.     Merid.diff.oflat=676.3 

To  find  the  Distance  =  ACs~ 
This  comes  under  the  6th  analogy,  page  237 ;  therefore, 

As  radius  =  .  .  90?0C  Log.  co-secant  =  .  .  10. 000000 
Is  to  the  diff.  of  lat.  540  miles.  Log.  =  .  .  2.733394 
So  is  the  course  =  2\  pts.  Log.  secant  =   .    .    .     10. 1 1 1815 


To  the  distance  =  698. 6  miles,  Log.  =  ....    2. 844209 
To  find  the  Difference  of  Longitude  =  D  E ; — 
This  comes  under  the  10th  analogy,  pag«  237  \  therefore. 


uigitizea  oy 


Google 


850  NAVIGATION, 

To  find  the  Course  and  Distance  made  good  ^— 

To  the  whole  difference  of  latitude  and  departure,  so  found,  find  the  cor- 
responding course  and  distance  by  Problem  II,  page  108,  and  thus  the 
course  and  distance  made  good  will  be  obtaindl. 

To  find  the  Latitude  in,  by  Account,  or  Dead  Reckoning  :— 

If  the  difference  of  latitude  and  the  latitude  of  the  place  from  which  the 
ship's  departure  was  taken>  or  the  yesterday *s  latitude,  be  of  the  same  name^ 
their  sum  will  be  the  latitude  in,  by  account;  but  if  Uiey  are  of  contrary 
names,  their  difference  will  be  the  latitude  in,  of  the  same  nam*  with  the 
greater  quantity. 

To  find  the  Difference  of  Longitude; — 

With  the  course  made  good,  and  the  meridional  difference  of  latitude,  in 
a  latitude  column,  find  the  corresponding  departure,  by  Problem  III.  page 
UO^  and  it  will  be  the  difference  of  longitude. 

Or,— W^tli  the  middle  latitude  as  a  course,  and  the  departure,  in  a  lati- 
tude column,  find  the  corresponding  distance,  by  Problem  V.,  page  111, 
and  it  ^11  be  the  difference  of  longitude. 

To  find  the  Longitude  in,  by  Account,  or  Dead  Reckoning  :— 

If  the  difference  of  longitude  and  the  longitude  of  the  place  from  which 
the  ship's  departure  was  taken,  or  the  yesterday's  longitude,  be  of  the  same 
name,  their  sum  wiD  be  the  longitude  in,  by  account,  when  it  does  not  ex* 
eeed  180?  i  othenvise,  it  is  to  be  taken  from  360?,  and  the  remainder  will 
be  the  longitude  in,  of  a  contrary  name  to  thatleft :«— but,  if  the  difference 
of  longitude  and  the  longitude  left  are  of  contrary  names,  thw  difference 
will  be  the  longitude  in,  of  the  same  name  with  the  greater  quantity. 

To  find  the  Bearing  and  Distance  from  the  Ship  tothe  Port  to  which 

she  is  bound :—    . 

By  Mercator's  Sailing. 

With  the  meridional  difference  of  latitude^  in  a  latitude  colunm,  and  the 
difference  of  longitude,  as  departure,  find  the  course,  by  Problem  IV.  page 
111;  then,  with  the  course,  thus  found,  and  the  difference  of  latitude,  the 
distance  is  to  be  obtained  by  the  same  ProbIem.**Or, 

By  Middle  Latitude  Sailing, 

With  the  middle  latitude  between  the  ship  and  the  proposed  place,  as  a 
course,  and  the  difference  of  longitude,  as  distance,  find  the  corresponding 


Digitized  by 


Google 


MERCATOH'fl  SAILING. 


251 


meridional  distance^  or  departure,  by  Problem  VI.  page  112;  then^  with  this 
departure,  and  the  difference  of  latitude,  the  course  and  distance  are  to  be 
obtained  by  the  same  Problem. 

Note, — ^The  true  bearing  or  course,  thus  found,  may  be  reduced  to  the 
magnetic,  or  compass  bearing,  if  necessary,  by  allowing  the  value  of  the 
variation  to  the  right  hand  if  westerly ;  and  to  the  left  hand  if  easterly  ; 
being  the  converse  of  redtlcing  the  course  steered  by  compass,  to  the  true 
course. 

And|  this  rule  .comprises  the  substance  of  that  nautical  operation  which 
is  generally  termed  a  day's  work  at  sea. 

Example  1« 

A  ship  from  Cape  Espiehell,  in  latitude  38°  35^  norths  and"  longitude 
9?  13'  west,  bound  for  Porto  Santo,  in  latitude  33?3^  north,  and  longi* 
tude  16?17-'we8t,  by  reason  of  contrary  winds  was  obliged  to  sail  upon 
the  following  compass  courses  |  vis.^W.  by  S.  56  miles )  N.  W.  by  W, 
110  miles;  W.  N.  W.  95  miles ;  S.  by  E.  i  E.  50  miles ;  S.  by  W.  J  W. 
103  miles,  and  8. 8.  W.  1 16  miles ;  the  variation  was  2  points  westerly  on 
the  three  first  courses,  and  If-  point  on  the  three  last :  required  the  course, 
and  distance  made  good,  the  latitude  and  longitude  at  whieh  the  ship  ar- 
rived ;  with  the  direct  course,  and  diatance  from  thenge  to  her  intended 
port? 


Tkavbrsb  Tabls. 

• 

Corrected 
Course!. 

Dis- 
tances. 

Difierence  of  Latitude. 

Deparl 

ture. 

N.        1        S. 

G. 

W. 

S.W.byW. 
W.byN. 

West. 

S.E.fS. 

South. 

S.iW. 

56 

110 

95 

50 

103 

116 

n 

21.5 

99 
99 

» 

31.1 

40.2 
103.0 
115.9 

»» 

>» 

>» 

29.8 

-  » 

46.6 

107.9 

95.0 

M 

>» 

5.7 

21.5 
Diff.Latss 

290.2 
21.5 

29.8 
Departure:;^ 

255.2 
29.8 

268.7 

225.4 

Digitized  by 


Google 


252  NAVIGATION. 


To  find  the  G>nr8e  and  Distance  made  good  :«- 

Half  the  difference  of  latitude  =  134. 35,  and  half  the  departure  = 
112. 7;  are  found  to  agree  nearest  abreast  of  40?  under  distance  175; — 
now,  175  X  2  =  350  miles. — Hence,  the  course  made  good  is  S.  40?  W. 
or,  S.  W.  i  S,  nearly,  and  the  distance  350  miles. 


To  find  the  Latitude  and  Longitude  come  to,  by  Account  :— 

Lat.ofC.E8pichell=    38925 C  N,  Mer.pts.  2500. 1.    Long.=9M3nV, 
Uiff.oflat.=269m9.,  or  4.29    S.      .    .    .    •    .  Diff.  Iong.=s4.40  W. 


Latitude  come  to  =      33?56:  N.  Mer.  pts.  2166. 7*  Long.=  13?53f  W. 
Merid.diff.oflat.    .    /    s    333.4 


.To  find  the  Difference  of  Longitude  made  good  :-. 

To' the  cpurse  made  good  =  40?  and  half  the  meridional  difference  of 
latitude  =  166. 7  the  corresponding  departure  is  140. 1,  which,  multiplied 
by  2,  gives  the  difference  of  longitude  280. 2  miles  =  4?40.t  west. — Or, 

^ith  the  middle  latitude  =  36?  10^  and  half  the  departure  =  112. 7^ 
in  a  latitude  column,  the  corresponding -distance  is  139.3  (proportion 
being  made  for  the  10  minutes  of  latitude) ;  hence,  139.3  X  2  =  278.6 
miles,  the  difference  of  longitude  $  being  about  a  mile- and  a  half  less  than 
the  result  by  Mercator's  sailing. 


To  find  the  Course  and  Distance  from  the  Ship  to  her  intended  Port  :— 

Lat.  of  the  ship  33?56^  N.        M.  pts.  2166. 7.  Longitude  13?53^  W. 
Lat.  Porto  Santo  33.  3    N.        M.  pts.  2103. 1.  Longitude  16. 17    W. 

'     .  '     '   •  — — - —  [miles. 

Diff.  of  Lat.  =     0?53^s53msJd.diff.L.63.6.Diff.LoBg.  2?24'.=:144 


By  Mercator's  Sailing. 

The  meridional  difference  of  latitude  =  63.<$  in  a  latitude  column,  and 
the  difference  of  longitude  s  144,  in  a  departure  column,  are  fotmd  to 
agree  nearest  abreast  of  66?  the  course.— Now,  to  course  66?  and  differ- 
ence of  latitude  53,  the  corresponding  distance  is  190  miles.— Or, 


Digitized  by 


Google 


BfSRCATOR's  SAILING.  253 

With  the  middle  latitude  =  33^301  as  a  course,  and  the  difference  of 
longitude  a=  144  as  distance,  the  corresponding  difference  of  latitude  is 
120.  1.— Now,  witli  120, 1  in  a  departure  column,  and  the  difference  of 
latitude  =  .53,  in  its  proper  column,  the  eorresponding  course  is  66?  and 
the  distance  131  miles^ 

Hence,— The  course  made  goad  is  S.  40?  W.  or  S.  W.  |  S,  nearly. 
The  distance  made  good  is  350  miles. 
The  latitude  by  account  is  33*56 '.  north. 
The  long,  by  account  is     13. 53    west 

And, 
Porto  Santo  bears  from  the  ship  S.  66?  W.  or  W.  S.  W.  nearly. 
Distance  130  miles  as  required. 

Note. — If  the  latitude  and  longitude  bf  the  ship,  or  either  of  them,  have 
been  deduced  from  celestial  observations,  they  are  to  be  made  use  of, 
instead  of  those  by  account,  in  determining  the  course  and  distance  between 
the  ship  find  the  place  to  which  she  is  bound.— See  the  compendium  of 
Practical  Navigation  near  the  end  of  this  Volume. 

Example  2. 

A  ship  frpm  Port  Royal,  Jamaica,  in  latitude  17?58'  north,  and  longi- 
tude 7&?5S  C  west,  got  under  weigh  for  Hayti,  St.  Domingo,  in  latitude 
18?30'  north,  and  longitude  69?49^  west,  and* sailed  upon  the  following 
courses,  viz. ;  S.  40  miles ^  S. £!.  by  S.  97  miles;  N.  by  E.  72  miles; 
S.E.^S.  lOStoiles;  N/byE.^E.  114  miles;  S.  E.  126  miles;  N.N.E. 
86  miles ;  and  then  by  observation  was  found  to  be  in  latitude  16?55 '.  N., 
and  longitude  72?30'  W.;— the  lee- way  on  each  of  those  courses  was  a 
quarter  of  a  point  (the  wind  being  between  E.  S.  E.  i  S.  and  E.  by  N.'^  N.), 
and  the  variation  of  the  compass  half  a  point  easterly  : — required  the  true 
course  and  distance  made  good ;  the  latitude  and  longitude  at  which  the 
ship  arrived  by  account,  with  the  direct  course  and  distance  between  her  true 
place  by  observation  and  the  port  to  which  she  is  bound  ? 


Digitized  by 


Google 


254 


NAVIOATIOM. 


Travbiub  Tablb.                                         1 

Corrected 
•   Courses. 

Dis- 
tance's. 

•  Difference  of  Latitude. 

Departure. 

.    N.         1         S. 

E.        1      W. 

S.iW. 
S.  S.E.iE. 
N.byE.jE. 
S.S.E.|E. 
N.  by  E.  }  E. 
S.E.byS.iE. 
N.N.E.JE. 

40 

97 

72 

108 

114 

126 

86 

99 

69.8 
107^3 

99 

77.7 

39.6 

.87.7 

99 

92.6 

99 

101.2 

99 

41.5 
17.5 
65.5 

38.4 
75.1 
36.8 

5.9 
» 
»» 

» 

254.8 
Diff.  Lat.  = 

321.1 
254. 8 

264.8 
5.9 

5.9 
Departure 

66.3 

258.9  = 

To  find  the  Course  and  Distance  made  good  :^—  ' 

Half  the  difference  of  Littitude  =  33. 15,  and  half  the  departure  = 
129.45,  are  found  toagree  nearest  between  75?  and  76?,  und[er  distance 
134  ;  and  by  making  proportion  for  the  difference  between  the  ^ven'  and 
the  tabular  numbers^  the  true  course  will  be  found  :»  75?38t  ;  and  the 
distance  134,  x  2=  168  miles.^Hence  the  course  made  good  is  S.  75?88^ 
E.  or  E.  by  S,  i  S.  nearly ;  and  the  distance  268  miles. 

To  find  the  Latitude  and  Longitude  come  to,  by  Account  :-— 

Lat.of  Port Royal=17?58:  N.Mer.pts,  1096. 1  Long.  =  .76?53^  W. 
Diff.  Lat.  66.3,  or    1.  6   S.  Diff.  Long.  =4. 30    E. 

Ut.cometobyac.=:16?52f  N.  Men  pts.  1026.9  Long,  by  Ace.  72?  23  C  W. 

Merid.  diff.  of  Lat.  =      69. 2 

To  find  the  Difference  of  Longitude  made  good : — 

To  the  course  made  good  =  75f38rand  the  meridional  difference  of 
latitude  =  69.2,  the  corresponding  departure  is  270.3,  proportion  being 
made  for  the  38^  in  the  course  beyond  75? — Hence,  the  difference  of  lon- 
gitude.is  270.3,  or  4?30:  east.— Or,  with  the  middle  latitude  =  17?25^ 
as  a  course,  and  half  the  departure  made  good  ss  129.45  in  a  latitude 


Digitized  by 


Google 


OBLiaUB  SAILING.  255 

colmniiy  the  corresponding  distance,  at  top  or  bottom^  !•  1S5 }  which^ 
multiplied  by  2,  gires  the  dMhrence  of  longitude  ts  270  miles. 

To  find  the  Course  and  Distance  from  the  Ship  to  her  intended  Port : —    . 

Lat.  of  ship  by  ob.  16?55  ^  N.    Mer.  pts.  1030. 1    Long,  by  ob.  72?30^  W. 
Lat.  ofHayti=    18.30   N.    Mer.  pte.  1129.8    Lg.ofHayti69.49   W. 


Diff.  of  Lat  =r      1  fSS ',         M.D.L.  i=  99. 7    Diff.  of  Long.  2MI C 

=  95  miles*  '    a  161  miles. 

The  meridional  difference  of  latitude  as  99. 7,  and  difference  of  longi- 
tude =  161^  in  a  departure  column,  are  found  to  agree  nearest  between 
58?  and  59?  under  distances  188  aiid  194 ;  and  by  making  proportion  for 
the  difference  between  the  given  and  the  tabular  numbers,  the  true  course 
will  be  found  »  58?  14  ^— Now,  to  course  58?  14^  add  difference  of  lati- 
tude 95,  the  corresponding  distance  is  180  miles.*— Or,  with  the  middle  la- 
titude s  17?42i^  as  a  course,  and  the  difference  of  longitude  =s  161  as  a 
distance,  the  corresponding  difference  of  latitude  is  153.4:-— now,  with 
153. 4,  in  a  departure  column,  and  the  difference  of  latitude  sr  95,  fai  its 
proper  column,  the  course,  nearest  agriseing,  is  58  degrees,  and  the  dis- 
tance 181  miles.— Hence, 

The  Courte  made  good  is  S.  75?38:  E.  or  E.  by  S.  i  S.  neariy. 
Distance  made  good  a  268  miles. 
Latitude  cojne  to  by  account  »  16?  521  N. 
Latitude  by  observation  s  •    .  16?55'  N. 
Long,  come  to  by  account  »  •  72?231  W. 
Long,  by  observation   =:     .     .  72?30C  W. 
Hayti  bears  from  the  ship  N.  58?  14  C  E.  or  N.  E.  by  E.  ^  E.  nearly. 
Distance  180  miles,  as  required. 

Abte.— This  example  and  the  preceding  exhibit  all  the  particulars 
attendant  on  making  out  a  day*$  work  at  sea. — See  mofe  of  this  in  the  com- 
pendium of  Practical  Navigation  near  the  end  of  this  Volume. 


SOLUTION  OP  CASES  IN  OBLIQUE  SAILING. 

Oblique  sailing  is  the  application  of  oblique  angled  plane  trigonometry 
to  the  solution  of  certaim  cases  at  sea :  such  as  in  coasting  along  shore  ; 
approaching,  or  leaving  the  land ;  surveying  coasts  and  harbours,  &c.,  where 
it  becomes  necessary  to  determine  the  distance  of  particular  places  from 


Digitized  by 


Google 


the  ship^  and  from  each  other.— And^  alao,  when  it  is  required  to  settle  the 
position  of  any  place^  cap^  or  head-land  from  a  ship,  by  obsenrations  taken 
on  board. 


Example  1« 

A  ship  being  about  to  take  her  departure  from  Madeira,  set  the  lizard 
Point,  which  bore,  by  azimuth  compass,  N.  W.  by  N. ;  and  after  sailing 
S.  W.  20  miles,  it  was  again  set  and  found  to  bear  N.  i  E. ;  required  the 
ship's  distance  from  the  Lizard  at  both  stations. 

SoJuiion. — ^In  the  annexed  diagram 
let  the  point  C  represent  the  Lizard, 
and  the  points  A  and  B  the  stations  or 
places  of  the  ship,  whence  the  bearings 
of  the  point  C  were  taken.— Now,  the 
difference  between  the  bearing  A  C  =3 
N.  W.  by  N.  and  the  ship's  course 
A  B  =^  S.  W.  is  9  points,  which  is  the 
value  of  the  angle  BAG,  measured  by 
the  arc  a  i : — The  difference  between 
N.  W.  by  N.  and  N.iE.  is  3i  points 
ss  the  angle  A  C  B,  measured  by  the 

arc  6  d ;  and  the  difference  between  N.  |  E.  and  N.  £.  (the  opposite 
point  to  S.  W.)  is  3^  points  =  the  angle  ABC,  measured  by  the 
arc  d e.—  Then,  in  the  oblique  angled  triangle  ABC,  given  the  an- 
gles and  the  side  A  B,  to  find  the  sides  A  C  and  B  C  =  the  distance  of 
the  ship  from  the  Lizard  at  the  respective  stations.-^Hence,  by  oblique 
angled  trigonometry.  Problem  L,  page  177* 


To  find  the  Distance  A  C :--» 

As  the  angle  C  =   •    •  .  3}  pts.  Log.  co-secant  = 
Is  to  the  distance  A  B=  20  ms..  Log.  =  .     •     . 
So  is  the  angle  B  =         3i  pts.  Log.  sine  =: 

To  the  dist.  A  C=:    17-  74  ms..  Log.  n   .    .    . 
To  find  the  Distance  B  C  :— 

As  the  angle  C  =  .  .  3}  pts.  Log.  co-secant 
Is  to  the  distance  A  B  =:  20  ms,.  Log.  =  •  » •  • 
So  is  the  angle  A  =:  9  pts.  Log.  sine  r:     • 

To  the  dist,  B  C  =29. 21  miles^  Log.  =:       .    . 


10. 172916 
1.301030 
9.775027 

1.248973 


=  10.172916 

.  1.301030 

.  9.991574 

.  2.465520 


OBUQ0B  SAILING.  257 

Hence^  the  distance  of  the  ship  from  the  Lizard  ajt  the  first  station  is  I?} 
miles }  and  at  the  second  station  29i  miles  nearly. 


*  Example  2.  ' 

Two  ships  sail  from  the  same  port^  one  N.  W.  by  W.  80  miles^  and  the 
other  S;W.  68  mil^s;  required  the  bearing  and  distance* of  those  ships 
from  each  other  ? 

Solution.-^ln  the  annexed  diagram  let  the 
side  A  C  represent  the  course  steered  by  one 
of  the  ships,  and  the  side  AB  the  course 
steered  by  the  other  ship  3  and  let  the  side. 
BC  represent  the  relative  bearing  and  distance 
of  the.  ships  from  each  other.— Now,  the 
difference  between  the  bearing  A  =  N.  W. 
by  W.  and  the  bearing  A  B  =  .S.  W.  is  7 
points  =  the  angle  BAG,  measured  by  the  ^ 
arca£. — Hence,  in  the  oblique  angled  triangle  ABC,  given  the  side 
AC  80  miles;  the. side  AB  68  miles,* and  the  included  angle  A=:7 
points;  to  find  the  other  angles,  and  the  pideBC— Therefore,  by  ob- 
lique angled  trigonometry,  Problem  IfL,  page  179, 

To  find  the  Angles  B  and  C  :— 

As  the  sum  of  A  B  and  A  C  = '  148  miles,  Log.  ar.  compt.  =7. 829738 
Is  to  difference  of  A  B  and  A  C  :=  12  miles,  Lpg.  =  .  .  .  1 .  079 1 81 
So  is  i  sum  of  angles  B  and  C=;  50?37/30r  Log.  tangent  =  ,10. 085827 

To  i  diff.  of  angles  B  and  C  =         5?38:32r  Log.  tangent  =  8, 994746 


Angle B=     .  .    .  56?16C  2^ 
Angle  C±        ,    .  44^58^58r 

To  find  the  Side  B  C  =:  the  Distance  between  the  Ships  :-« 

As  the  angle  B  =  56?  16.'  21  Log.  cosecant  =     .    .     10. 080066 

Is  to  the  side  AC  =       80  miles, .   Log.  =  .    ......    .1. 903090 

So  is  the  angle  A=        7  points      Log.  sine  =    ....     .9. 991574 

To  distance  BC:=:    94.34  miles,    Log.  =:       •    •    .    .    .    1.974730 


s 


Digitized  by 


Google 


2SS  NAVIGATION. 


To  find  the  relative  Bearing*  of  the  Ship«  :— 

From  the  angle  B  =:  56?  16^  2'^  subtract  the  course  finom  A  to  B  =  45?, 
and  the  remainder  =  1 1  ?  16'  2^'  is  the  bearing  of  C  frpm  B  ==  N.  1 1?  16' 
W.  or  N.  by  W.  nearly,— And  frpm  the  course  A  C  =  56? ISC  subtract 
the  angle  C  =  44?58C58^  and  the-  remainder  =  1  l?16C2r  ia  the  course 
from  C  to  B  =  S!  1 1?  16' B.  or  S.  by  E.  nearly. 


Exampls  3. 

Coasting  along  shore  two  head-lands  were  observed ;  the  first  bore,  by 
azimuth  compass,  N.N.E.,  the  second  N.W.:— after  swlingW.  by  S. 
16  miles,  the  first  bore  N.  E.  i  E.  and  the  second  N.  by  E.  J  E.  j  required 
the  relative  bearing  and  distance  of  those  head-lands  from  each  other  ? 

SolutionJ-^ln  the  diagram  ABDC, 
let  the  side  A  B  represent  the  course 
steered  by  the  ship;  AC  the  bear- 
ing of  the  first  head-land,  and  AD  the 
bearing  of  the  second  head-land  from 
the  place  of  the  ship  at  A;  and,  let 
B  C  represent  the  bearing  of  the  first 
head-land,  and  B  D  the  bearing  of  the 
second  head-land  from  the  shipf's 
place  at  B.-^— Now,  in  the  triangle 
ABD,  the  angles  and  the  side  A  B  ^  ••...^--•-* 

are  giveit,  to  find  the  side  A*D. —  ^ 

Thuff,  the  difference  between  N.  W,  and  W.  by  S.  is  5  points  ±:  the  an- 
gle BAD|  measured  by  the  arc. a  e;— the  difference  between  N.  byE. 
i  E.  and  E.  by  N.  (the  opposite  point  to  W.  by  S.  the  ship's  course^) 
is  5^  points  =  the  angle  DBA,  measured  by  the  arc  c  d,  and  the  differ- 
ence between  N.  by  E.  ^  K  and  N.  W*  is  5^  points  =  the  angle  A  D  B, 
measured  by  the  arcce;  and  the  side  AB  =  16  miles;  to  find  the 
side  AD. — Hence,  by  oblique  angled  trigonometry.  Problem  L,  page  177, 

As  the  angle  A  D  B  =:  5|  pts.  Log;  co-secaat  =:  10. 054570 
Is  to  the  side  AB=  15  miles.  Log.  =  .  .  .  .1.176091 
So  is. the  angle  AB D  =  5^  pts.    Log,  sine  =  .    .    9. 945430 

To  the  side  AD.    t=   15  miles.      Log.  =        .    .    1.176091 

Nofe.— The  side  A  D  might  be  determined  independently  of  calculation, 
aa  thus  j  the  angles  B  and  D  are  equal,  for  each  ia  meaanred  by  aa  ctre  of 


Digitized  by 


Google 


OBL^aU^  9AIUN6.  S59 

5|  poiQta ;  and  since  equal  sKigles  are  subtended  by  eqnal  ni^es^  therefore 
the  side  A  D  is  equal  to  the  side  A  B  =:  15  miles. 

'  Ag^ih.-^Inthe  triangle  A  BC^  the  angles  and  the  aide  A  B  aia  given, 
to  find  the  side  AC;  thus^  the  difference  between  N.  N.  B.  and  W.by  S. 
ia  11  points  =:  the  angle  BAG,  measured  by  the  arc  abj  the  difference 
between  N.  N;  Q.  and  N«  G.  i  £•  is  2i  points  =:  the  angle  AC  B,  mea- 
sured by  the  arc  6  g,  and  the  difference  between  N.  B.  i  E.  and  £.  by  N. 
(the  opposite  point  to  W.  by  S.  the  ship's  course,)  is  2f  points  =  the  angle 
ABC,  measured  by  the  arc  g  d :-— t^nee,  the  side  A  C  may  be  found  by 
tl)e  abqve-pi^ntioned  Problem ;  as  thus  : 

• 

As  the  ang^e  A  C  B  =s  .3}  poinU  Log.  coHBeeiiita  10. 868008 
Is  to  the  side  AB  =:  15  miles,  Log.  =  .  •  .1. 176081 
Soiatheang^eABC  =:  SfpoiaULog.sines     .    9.7110M 

Tpdue»HeACf3l8*93mi}ef/.    Log,  c      .,    ,    }.^{f6149 

Now,  in  the.  triangle  ADC  there  are  pven,  the  side  A  D  a  15  utiles } 
the  side  A  C  =s18p  03  miles,  and  the  included  angle  D  AC,  6  points  s  the 
difference  between  N.  N.  £•  and  N.  W.  mea/Blir^  by  thelare  eb,Ui  find  the 
angles  ADC  and  A  C  D,  and  the  side  D  C.-r^Hence,  by  trigonometry^  Pror 
blem  III.,  page  178, 

As  the  sum  of  A  C  and  A  B  =:  33. 03  miles,  I^g»  air.  co)npt.=s8. 481081 
IstodiffcrenceofACandAB  =  S.03.  Log,   =     .    •    0.481443 

Soi8  4sumofangs,ADCandACD=56n5f  Ori4)g.tang.=  10,l75107 

To  I  difference  of  those  angles  :?    :    .     7?49.'  2rLog.taDg.=:  9, 137fi41 


Angle  ADC  s=     .    .    64?  4f  2f 
Angle  ACD=      .    .    49?3S'.SBr 

To  find  the  Side  DC  r— 

As  the  angle  A C  D  =      48?25f  58f  Log.  co-secant  =  .    i     10. 125995 
Is  to  the  side  AD  =15  miles.  Log.  =     .....     1. 176091 

Soi8theangl9DAC-=  6  points         Log.  sine  7  .    ,    .    .    9.965615 

To  the  side  DC  =  18.52  milesj  Log.  =     .     .    .     .    ,     1.267701 

Hence,  the  distance  betweeb  the  two  head-lands  is  15^  miles. 

s  2 


Digitized  by 


Google 


To  find  the  relative  Bearings  of  the  two  given  Head-lands  z-— 

To  the  angle  A  C  D  =  48?25 ' 58r  add  the  course  or  bearing  from  A  to 
C  =  2  points,  or  22?30'  and  the  sum  =  70?55^58r  is  the  bearing  of  D 
from  C  =  S.  70956^  W.,  or  W.  by  S.  J  S.  nearly— And,  to  the  angle 
A  D  C  =  64?4:2r  add  the  bearing  from  A  to  D  -  4  points,  or  45  de- 
grees, and  the  sum  =  109^4^2^  being  taken  from  180?  gives  70?55C58f 
s  the  bearing  of  C  from  D  s  N.  70?561  E.  or  B.  by  N.  i  N.  nearly. 

Esample  4. 

Being  desirous  of  ascertaining  the  exact  position  of  a  head-land,  with 
respect  to  latitude  and  Idngitude,  it  was  carefully  set,  by  an  azimuth  com- 
pass, and  found  to  bear  N.  Jb.  E.,  and  after  sailing  N.W.  b.  W«  12  miles, 
it  was  again  set,  and  observed  to  bear  E.  b.  N.  i  N.,  due  allowance  being 
made  for  the  variation  of  the  compass.  Now,  the  correct  latitude  of  the 
ship  at  the  last  place  of  observation  was  21?50C.2K  N.,  and  the  longitude 
85?9'6T  W. ;  required  the  latitude  and  longitude  of  the  said  head-land  ? 

Solution.— In  the  oblique  angled 
triangle  ABC,  where  the  side  AC 
represents  the  first  bearing  of  the 
head-land,  the  side  B  C  the  second 
bearing,  and  the  side  A  B  the  dis- 
tance sailed;  given  the  three  angles 
and  the  side  AB  =  12  miles,  to  find 
the  side  B  C  =  the  ship's  distance 
from  the  headland  at  the  second  sta- 
tion. Thus,  the  difference  between 
N.  b.  E.,  and  N.W.  b.  W.,  is  5  points 
=  the  angle.C  AB,  measured  by  the 
arc  ad;  the  difference  between. N. 

W.  b.  W.,  and  E.  b.  S.  i  S.,  the  opposite  point  to  E.  b.  N.  J  N.,  is  4| 
points  =  the  angle  ABC,  measured  by  the  arc  de,  and  the  difference 
between  E.  b.  N.  i  N.,  and  N.  b.  E.,  is  5 J  pointe  =  the  angle  AC  B, 
measured  by  the  arc  ab.  Hence,  by.  oblique  angled  trigonometry.  Pro- 
blem I.,  page  107,  to  find  the  side  BC  =  the  ship's  distance  from  the 
head-land  at  the  second  station. 


As  the  angle  A CB  =  SJ  points.  Log.  co-secant  =  10. 054570 
Is  to  the  side  AB  =:  12  miles,  Log.  =  .  .  .  1.079181 
Sois  tlieangleCABr:     6  points,    Log.  sine    =      .      9.965615 


To  the  side  BC  =*    12. 57 miles,  Log.  =    .    .    .      1.099366 


OBLIQUJB  SAILING,  261 

Hence,  the  distance  of  the  ship  from  the  head-land  at  the  second  station 
is  1 2^  miles,  nearly. 

To  find  the  Difference  of  Latitude  and  Difference  of  Longitude  between 
the  Ship's  Place  at  B,  and  the  Head-Land  C*:— 

In  the  right  angled  triangle  BCD,  given  the  angle  C;  6^  points  =  the 
bearing  of  B  from  C,  and  the  distance  BC  =  12..57  miles,  to  find  the 
difference' of  latitude  C  D,  and  the  difference  of  longitude  BD  3  therefore, 
by  Mercator's  Sailing,  Problem  IL,  page  239, 

As  radius  =  90<?0C  Log.  secant  =:  .  v  10.000000 
Is  to  distance  B  C  =  1 2. 57  miles^  Log.  ==  1 .  099366 
Sq  is  the  course  C=6§  points,  Log.  co-8iner=9!.  462824 

To  the  diff.  of  lat  CDszS.  65  miles,  Log.=:  &.  562190 

As  radius  =:  90?0'  Log.co-secant  =:  .  ;  10.000000 
Is  to  mer.  diff.  of  lat,  =  3.9  miles.  Log.  c:  .  0.591065 
So  is  the  course  C  =:  6}  points,  LfOg.  tangent=  10. 518061 

To  the  diff.  of  long.  =  12: 85  miles,  Log.  =       1. 109126 

Lat.of8hip=:21?50^2KN.  M.pts=1343.3    Lon.ofship=85?  9^  6rW. 
Diff.lat.3.65,or3^39rN.  Diff.lon.l2.S5,orl2.51  B. 


Lat.ofhd.W.21?54^  OrN.  M.pt8=1347.2    Lon.ofhd.ld.84?66:i6rW. 
Meridional  difference  of  latitude  =3.9  miles. 

Hence,  the  latitude  of  the  head-Und  is  2if54;0?  N.,  and  its  longitude 
84?56M5fW. 


Note. — ^The  foregoing  examples  contain  all  the  oases  in  oblique  sailing 
that  are  of  any  immediate  import  to  the  mariner.  Other  examples,  indeed, 
might  be  given ;  but  since  they  would  rather  tend  to  the  exercise  of  the 
mind  on  trigonometrical  subjects,  than  to  any  useful  nautical  purpose,  they 
have  therefore  been  intentionally  omitted. 

The  two  last  examples  will  be  found  particularly  useful  in  maritime  sur-* 
veying,  when  the  operations  are  conducted  on  board  of  a  ship  or  vessel. 


Digitized  by 


Google 


262  NAVlGAttOH. 


.    ,SOLtJTION  OP  CASBd  IN  WiNDWARD  SAILINCJ. 

Windward  Sailing  ia  the  method  of  reachihg  the  pott  or  pla^e  bbund  to 
by  the  shortest  roiit^^  when  the  wind  is  in  a  direction  Contrary  to*the  direct 
course  between  the  ship  and  the  pUc6  to  whieh  ()he  is  boundi 

When  the  wind  is  opposed  to  th6  course  which  a  ship  should  steer  from 
any  one  port  to  another^  she  is  obliged  to  sail  upon  different  tacks^  close- 
hauled  to  the  witid^  in  order  to  reach  the  port  bound  to.  The  object, 
therefore^  of  this  method  of  sailing,  is  to  find  the  prbper  course  to  be 
conned  on  feach  tadk,  sp  that  the  ship  may.ikrrite  at  the  place  to  which 
she  is  bound,  lathe  shortest  time  possible. 

EtBomfle  1% 

A  ship  tliat  can  lie  within  6  points  of  the  wind  is  bound  to  a  port  50 
miles  directly  to  windward,  which  it  id  intended  she  shall  reach  on  two 
tacks ;  the  first  being  oa  the  starboard  tack)  and  the  wind  steady  a,t  N.  b.  £•  j 
required  the  course  and  distance  to  be  run  upon  each  tack  ? 

jSblutton.-— Since  the  ship  cto  • 
lie  within  six  points  of  the  wind, 
vMnh  iA  lit  N.  b.  E.,  the  course  oh 
the  starboard  tapk  will  be  N.Wi 
b.  W.,  and  that  on  th#  larboard 
tack  E.  b.  N.  Now,  in  the  annexed 
diaigtratn,  let  the  side  A  C  represent 
the  course  and  distance  between 
the  ship  and  her  intended  port; 
A  B  the  course  and  distance  to  be 
ttiade  good  on  the  starboard  tack; 
end  B  C  the  course  and  distancis  to 
be  made  good  on  the  larboard  tack.  Then^  in  the  triangle  ABC,  Uni 
three  angles  ane  given'  to  find  the  side  AB  or  BC>  which  sides  ait  muto- 
ally  equal  to  each  other,  because  the  triangle  is  isosceles,  and  its  Verted  at 
B  =r  this  .ahgte  comprehended  bet^eeh  those  sides^  Thus^  the  diflfierence 
between  N.  b.  B.,  And  N.W.  b;  W.^  is  6  points  =i  the  angle  B  AC,  mea- 
sured by  the  arc  ai;  the  difference  between  E.  b.  N.,  and  S.E.  b.  E.  (the 
opposite  point  to  N.W.  b.  W.),  is  4  points,  measured  by  the  arc  d  e ;  and 
the  difference  between  N«  b.  E.,  and  E.  b.  N.,  is  6  poiQts,  measured  by  the 


Digitized  by 


Google 


WIKDWAIID  iAILING. 


263 


are  Ad ;  and  since  Che  distance  AC  is  ^ven  =:  50  miles^  the  side  A B^  or 
its  equal  B  C^*  may  be  readily  determined  by  oblique  angled  trigonometry. 
Problem  I.^^  page  177 1  as  thus  :— 

As  the  angle  B  '==  4  points.  Log.  co-secant  =s     10. 150515 
Is  to  the  distance  AGs  60  miles.  Log.   s       1.698970     • 
So  is  the  angle  C  s=  6  points.  Log.  sine ,  =     #      9. 965615 

To  the  distance  A  B  s  65. 33  miles,  Log.  =       1. 815 100  . 


Hence^  it  is  evident  that  the  ship  must  run  65. 33  miles  on  the  starboard 
tack^  and  65.33  miles  on  the  larboard  tack,  before  she  can  reach  her 
intended  port. 

Example  2. 

A  ship  that  can  lie  within  6  points  of  the  wind  is  bound  to  a  port  bearing 
NJB.  b.  N.,  distance  90  miles,  which  it  is  intended  she  shall  reach  on  three 
lacks,  with  the  wind  steady  at  north ;  required  the  course  and  distance  to 
be  run  upon  each  tack,  the  first  course  being  on  the  larboard  tack  ? 

<SoIti(um.<i— Since  the  wind  is 
at  north,  and  that  the  ship  can 
lie  within  6  points  thereof,  the 
course  on  the  larboard  tack  will  be 
EJI.E.,  and  that  on  the  starboard 
tackW.N.W. 

In  the  annexed  diagram,  let  the 
NJE.  b.N.  line  AB  =±  90  miles, 
represent  the  bearing  and  distance 
between  the  ship  and  her  intended 
port^  let  the  E.NJBi.  line  AD  re- 
present the  first  board  on  the  lar- 
board tack|  and,  parallel  thereto, 
the  line  B  C  =  the  second  board  on  that  ti^^k.  And,  since  the  ship  is  to 
make  her  port  in  three  tacks,  it  is  evident  that  the  board  on  the  starboard 
tack,  represented  by  the  W.N.W.  line  CD  (parallel  to  dg),  must  bisect 
the  line  A  B  in  the  point  F;  and  that,  therefore,  AF  and  FB.are  equal  to 
one  another,  each  being  equal  to  45  miles  ==  half  the  line,  or  distance- A  B. 

N0W5  since  the  straight  line  A  B  falls  upon  the  two  parallel  straight  lines 
CB  and  AD,  it  makes  the  alternate  angles  equal  to  ofie  another;  there- 


*  Since  Uie  aog^les  A  and  C  are  equal  to  one  another^  the  sides  which  subtendy  or  are 
oppotite  to  those  socles  (vii.,  B  C  and  A  B)^  are  also  equid  to  oae  anotber^<-£uclid,  Book 
I.,  Prop.  6. 


Digitized  by 


Google 


264  NAVIGATION. 

fore  the  angle  A  B  C  is  equal  to  the  angle  BAD.— -Euclid^  Book  I.,  Prop.29. 
And  because  the  straight  line  C  D  falls  Upon  the  two  parallel  straight  lines 
CB  and  AD,  it  makes  the  angle  AD  B  equal  to  the  angle  BCD,  by  the 
aforesaid  proposition.  And  because  the  two  triacigles  A  D  P  and  B  C  F 
have,  thus,  two  angles  of  the  one  equal  to  two  angles  of  the  other,  viz.,  the 
angle  F  A  D  to  the  angle  F  B  C,  and  the  angle  A  D  F  to  the  angle  B  C  F ; 
and  the  side  A  F  of  the  one  equal  to  the  sid^  B  F  of  the  other  t  therefore  the 
remaining  sides  A  D  and  D  F  of  the  one  are  equal  to  the  remaining  sides 
B  C  and  C  F  of  the  other,  each  to  each ;  and  the  third  angle  A  FD  of  the 
one  equal  to  the  third  angle  B  F  C  of  the  other. — Euclid,  Book  I.,  Prop.  26. 
Now,  since  the  two  triangles  A  F  D  and  B  P  C  are,  thus,  evidently  equal  to 
one  another,  we  have  only  to  compute  the  unknown  sides  of  one,  viz.,  of 
the  triangle  AFD,  where  the  three  angles  are  given,  and  the  side  AF,  to 
find  the  sides  At)  and  FD3  thus,  the  diJQference  between  N.E.b. N.  and 
E.NJB.,  is  3  points,  zr  the  angle  FAD,  measured  by  the  nxcab:  the  differ- 
ence between  E.N.E.  and  E.S£.  (the  opposite  point  to  W.N.W.),  is  4 
points  =  the  angle  A  DP,  measured  by  the  arc  bg ;  and  the  difference 
between  WJI.W.  and  N.E«  b.N.,  is  9  points  ==  the  angle  AF  D^  measured 
by  the  arc  ad:  hence,  by  oblique  angled  tr^onometry.  Problem  I.^ 
page  177, 

To  find  the  Side  A  D  .— 

As  the  angle  D  =  4  points.       Log.  co-secant  =10. 150515 

Is  to  the  side  AP  =  45  miles,        Log.  =5     .     1. 653213 

.  So  is  the  angle  P  =  9  points,       Log.  sine  =  9. 991574 

To  the  side  AD  =  62.42  miles,        Log.  s  1.795302 

To  find  the  Side  FD:— 

As  the  angle  D  ;=  4  points,  Log.  co-secant  =  10. 150515 
Is  to  the  side  AP  =  45  miles,  Log.  =  •  .  .  1.653213 
So  is  the  angle  A  =  3  points^     Log.  sine    =       •      9. 744739 

To.the  side  P  D  =      35 .  35  miles,  Log.  =?      •    •      1 .  548467 

Side  DC  ss.    .    .    70. 70  miles.. 

Hence  it  is  evident  that  the  ship  must  first  run  62. 42  miles  on  the  lar-* 
board  tack ;  then  70. 70  mihes  on  the  starboard  tack ;  and,  again^  62. 42 
miles  on  the  larboard  tack,  before  she  can  reach  her  intended  port. 

Example  3. 
A  ship  that  can  lie  within  6  points  of  the  wind  is  bound  to  a  port  bear- 


Digitized  by 


Google 


WINDWAED  SAILING. 


2M 


ing  N,N.W.,  distance  120  miles,  which  it  b  intended  she  shall  make  o 
four  tacks,  with  the  wind  at  N.b.  W.  The  coast,  which  is  to  the  east 
wai-d,  trends  in  a  direction  nearly  parallel  to  the  bearing  of  the  port,  so* 
that  the  ship  must  go  about  as  sopn  as  she  reaches  the  straight  line  joining 
the  two  ports  ;•  required  the  course  and  distance  to  be  run  upon  each  tack, 
on  the  supposition  th^t  ti^e  ship's  progress  is  not  affected  by  either  leeway^ 
or  currents  ? 


Solution. — Since  the  wind  is 
N.  b«  W.,  and  the  land  trends  in  a 
N.N.W.  direction,  the  first  board, 
therefore,  must  be  on  the  starboard 
tack ;  and,  as  the  ship  can  lie  . 
within  6  points  of  the  wind,  the 
course  on  the  starboard  tacks  will 
be  W.  b.  N.,  and  that  on  the  lar- 
1>oard  tacks  N.E.  b.  E. 

In  the  annexed  diagram,  let  the 
N.N.W.  line  AB,  120  miles,  re- 
present  the  bearing  and  distance 
between  the  ship  and  the  port  to 

which  she  is  bound;  let  the  W.'b.N.  S 

line  A  D  represent  the  first  board 

on  the  starboard  tack,  and  FC,  parallel  to  AD,  the  second  board  on  that 
tack ;  let  the  N.E.  b.  E.  line  D  F  represent  the  first  board  on  the  larboard 
tack,  and,  parallel  thereto,  the  line  CB  =  the  second  board  on  this  tack* 
And,  since  the  ship  is  to  make  her  port  in  four  tacks,  without  going  to  the 
eastward  of  the  line  AB,  therefore,  at  the  end  of  the  second  tack,  she 
must  reach  the  point  F,  which  bisects  or  divides  the  distance  A  B  int6  two 
equal  parts,  of  60  miles  each ;  thus  making  A  F  =  to  A  B. 

Now,  because  the  strught  line  A  B  falls  upon  the  two  parallel  straight 
lines  AD  and  FC,  it  makes  the  angle  BFC  equal  to  the  interior  and 
opposite  angle  FAD :  and,  because  the  straight  lin^  AB  falls  upon  the 
two  parallel  straight  lines  FD  and  CB,'  it  makes  the  angle  A  FD  equal 
to  the  interior  and  opposite  angle  C  B  F,< — ^Euclid,  Book  I.,  Prop.  29.  And, 
since  the  two  triangles  AFD  add  FB  C  have,  .thus,  two  angles  of  the  one 
equal  to  two  angles  of  the  other,  viz.,  the  angle  A  F  D  to  the  angle  F  B  C, 
and  the  angle  F  A  D  to  the  angle  BFC,  and  the  side  A  t^  of  the  one  equal 
to  the  side  F  B  of  the  oth^r,— ^therefore  the  remaining  sides  *A  D  and  D  F 
of  the  one,  are  equal  to  the  remaining  sides  FC  and  C  B  of  the  other,,  each 
to  each ;  and  the  third  angle  A  D  F  of  the  one  equal  to  the  third  angle 
p  C  B  of  the  other.— Euclid,  Book  1.,  Prop.  26.       . 

The  two  triangles  A  D  F  and  F  C  B,  being,  th^s,  clearly  equal  to  one 


S66  NAVlGATtOM. 

• 

another  in  «very  reipect,  we  have  only  to  compute  ttie  unknown  sides  of 
one^  vIb.,  of  thd  triangle  A  PD,  where  the  three  angles  are  given,  and  the 
side  A F  bi  60  miles^  to  find  the  sides  AD  and  D  F^  thus  the  diflPerehee 
between.N.N.W.  and  W.b.N.,  is  6  points  =3  the  angle  FAD,  measured 
by  the  arc  06}  the  difforenoe  between  W.  b.  N.  and  S.W,  b.W.  (the 
opposite  point  toN.E;b.E.)>  is  4  points  =a  the  angle  AD  P,  measured  by 
the  arc  a  e ;  and  the  difference  between  N.N.W.  and  N.E.  b.  E.^  is  7  points 
=  the  angle  A  F  D,  measured  by  the  arc  b  d. 
Hence,  by  oblique  angled  trigonometry,  Problem  L,  page  177^ 

To  find  the  Side  AD  s=  P C :— 

As  the  angle  D  =  4  points,  Log.  co«secant  ac  10. 150515 
Is  to  the  side  A  F  ==  60  miles,  Log.  «  •  .  .  1.778151 
So  is  the  angle  F  =  7  points,  Lo!g.  sine  s=      •    9. 991574 


To  the  side  AD  »  83. 22  miles.  Log.  =    .    .     L  920240 

To  find  the  Side  D  P  =  C  B  :— 

As  the  an^e  D  =  4  points,  Log.  co-secant  =  .10. 150515 
IstothesideAF=60iniles,Log.  =  ...  1.778151 
So  is  the  angle  A  =s  5  points.  Log.  sine  =      •    9. 919846 


To  the  side  D  F  &  70. 50  miles^  Log.  >=      .         1 .8485 12 

From  this  it  is  manifest,  that  the  ship  must  first  run  83. 22  miles  upon 
the  starboard  tack  j  then  70. 5$  miles  upon  the  larboard  tack ;  then  83%  22 
miles  again  upon  the  starboard  tack ;  and  70. 55  miles  upon  the  larboard 
tacki  before  she  can  reach  the  port  to  which  she  is  bound. 


SOLUTION  OF  CASES  IN  CURRENT  SAILING. 

Current  Sailing  is  the  Aiethod  of  determining  the  true  course  and  distance 
made  good  by  a  ship,  when  her  own  motion  is  affected  or  combined  with 
that  of  the  current  in  which  she  stuls. 

A  current  Is  a  progressive  motion  of  the  water,  causing  all  floating 
bodies  thereon  to  move  in  the  direction  to  which  its  stream  is  impelled. 
The  setting  of  a  current  is  that  point  of  the  compass  towards  which  the 
water  runs ;  and  the  driftot  a  current  is  the  rate  at  which  it  runs  per  hour. 

When  a  ship  saik  in  the  direction  of  a  current,  her  Velocity  will  be  equal 


Digitized  by 


Google 


CtfftRSNt  SAtLlNG.  i67 

to  the  sum  of  her  own  proper  motion  and  the  current's  drift ;  but  when 
she  sails  directly  against  a  current,  her  velocity  will  be  expressed  by  the 
difference  between  her  own  proper  motion  and  the  drift  of  the  ourrent :  in 
this  case^  the  absolute  motion  of  a  ship  will  be  a-headj  if  her  proper  velo- 
city exceeds  the  drift  of  the  current  i  but  if  it  be  leisi,  she  will  make  stern- 
way.  Whea  a  ship's  course  is  oblique  to  the  direction  of  a  current,  her 
true  eoun^  and  distance  will  be  compoutjded  of  thfl  course  tind  distance, 
given  by  the  log,  and  of  the  observcid  setting  and  drift  of  the  current. 

When  a  ship's  course  and  distance  by  the  log,  and  the  setting  and  drift 
of  the  cuRcnt  in  which  she  sails  are  given,  the  true  course  atid  distance 
made  good  may  be  found  by  a  trigonometrical  solution  of  the  triangles 
forming  the  figure }  but  the  easiest  and  most  eKpeditibus  method  of  finding 
the  course  and  distaifce  made  gobd^  particularly  when  a  ship  sails  upon 
different  courses,  is  by  resolving  a  traverse,  in  which  the  dettmg  and  drift 
of  the  cutrent  are  to  be  esteemed  as  an  additional  course  and  distance  to 
thos^  exhibited  by  the  log. 

Example  L 

If  a  ship  sails  S.W.  b,  W.>  at  the  ratfe  of  4  knots  an  hour,  in  a  current 
setting  S.SJi.1  E.,  at  the  rate  of  H  miles  ah  hour;  required  the  course 
Md  tMtaAdi  madegdod  in  24hmir8  \ 

iSbZuHoit.-^r  X  24t  =  96  miles,  the  distance  aailtd^  by  log»  in  84 
hoars} 

And  U?  X  24  =  42  miles,  the  observed  drift  of  th<  current  in  24 
hours. 

In  ^e  ume^ed  diagrani^  tet  Che 
aide  AB  of  the  tiriangle  ABC- 
represent  the  course  and  distance 
sailed  by  the  log,  and  the  ride  6  C 
parallel  to  d&  the  setting  atid  drift 
of  the  current;  then^  the  side  AC 
Will  represent  the  course  And  dis- 
tance made  good  in  the  given  time. 
Now,  in  the  triangle  A  B  6,  ^ven 
the  side  A  B  ^  96  miles,  the  side 
B  C  =  42  miles,  aiid  the  included 
angle  B  «=  8i  points,  being  the 

(fifference  bet^veen  S.S  Jl.  i  E.  and  N.E.  b.  E.  (the  opposite  point  to 
S.W.b.W.),  Pleasured  by  the  arc  a  5,  to  find  the.aiigles  A  and  C,  and  the 
true  distance  A  C.    Hence^  by  oblique  angled  trigonometry,  Problem  III., 


Digitized  by 


Google 


To  find  the  Angles  A  and  C  :— 

As  AB  +  BC  =  138  miles.  Log,  ar,  comp.  =  7. 860121 
IstoAB-BC  =  54  miles,  Log.  =  .  /' .  L732S94 
So  is  i  sum  of  the  angIes=43?35<37^T  Log.  tangent=9. 978673 


To  i  diff.  of  the  angles  -  20. 25 .  59    '  Log.  tangent=9. 57 1 188 


Angle  C=. 
Angle  A  = 


64°  1^36ir 
239  9r38if 


To  find  the  true  Distance  =  AC : — 

As  the  angle  C  =  64?l'.36|r    Log.  co-secant   =    10.046241 

'    Is  to  the  side  A  Br:  96  miles.    Log.  =   .     .     •    .     1.982271     . 

So  is  the  angle  B  =  8i  points,   Log.  sine  =     ...    9. 999477 

To  the  true  distance  =  AC  r:  106. 7  miles.  Log.  =  2. 027989 

To  find  the  Course  made  good :— - 

From  the  angle  SAB  =  S.W.  b.  W.,  or  56?  15^,  subtract  the  an^^le 
C  AB  =  23?9^38ir,and  the  remainder,  33^5^21*^  =  the  angle  SAC, 
is  the  course  made  good. 

Hence  the  course  made  good  is  S.  33? 5'  W.,  or  S,W.  b.  S.  nearly,  and 
the  .distance  106}  miles  nearly. 


To  find  the  Course  and  Distance  made  good  by  the  Traverse  Table :— 


Travbrsb  Table.                                       1 

Corrected 
Courses. 

2 

Difference  of  Latitude. 

Departure.            1 

N. 

S. 

E. 

W. 

S.W.b.W. 

Current 

S.S£fE. 

96 
42 

: 

53.  S 
36.0 

21.6 

79.8 

Diff.lat.= 

• 

89.3 

21.6 
Dejpart  = 

79.8 
21.6 

58.2 

CURRBNT   SAILING. 


269 


Now,  by  Problem  II.,  page  108, 

The  difference  of  latitude  89. 3,  and  the  departure  58. 2,  are  found  to 
agree  nearest  abreast  of  33?,  under  or  over  distance  107. 

Hence  the  course  made  good  is  S.  33?  W.,  or  S.W.  b.  S,,  and  the  dis* 
tance  107  iniles ;  which  nearly  agrees  with  the  above  result. 

Mxample  2. 

Suppose  a  ship  sails  N.W.  65  miles,  W.N.W.  70  miles^  and  N.  b.  E.  71 
miles,  in  a  current  th^t  sets  S.E.  b.  S.  36  miles  in  th^  same  time ;  required 
the  true  cQurse.  and  distance  made  good  ?         , 


Teavjihsb  Tablb 

« 

Corrected 
Courses. 

O 

S." 

• 

Difference  of  Latitude. 

Departure. 

N. 

S. 

E. 

W. 

N.W. 

65 

46.0 

—  ■ 

46.0 

W.N.W. 

70 

26.8 

— 

■—  , 

64.7 

N.b.E. 

71 

69.6 

— 

13.9 

•« 

Carfent 
SJl.b.S. 

36 

•^ 

29.9 

20.0 

— 

142.4 

29.9 

33.9 

110,7 

29.9 

33.9 

112.5. 

76. 8 

So&ifioii.— With  the  difference  of  latitude  and  departure,  thus  found,  the 
course  and  distance  made  good  may  be  djetermined  by  Problem  II.,  page 
108 ;  as  thus  : 

The  difference  of  latitude  112. 5,  and  the  departure  76. 8,  are  found  to 
agree  nearest  abreast  of  34?  under  or  over  136. 

Hence  the  direct  course  made  good  is  N.  34?  W.,  or  N.W,  b.  N.  nearly, 
and  the  distance  136  miles. 

To  find  the  Course  and  Distance  made  good  by  Calculation  : — 

This  may  be  done  by  means  of  the  5  th  analogy,  page  237  >  as  thus : 

To  find  the  true  Course :— . 


Digitized  by 


Google 


AS  ine  ain«  oi  iat«   = 
U  to  radiuf  ^    •    » 
So  is  the  depairture  *«; 


1  iz.  D  jLiOg.  af«  compx 
90?Q:  Log.  sine,  s  , 
76,8    liog.  w     .    . 


la  000000 

h  885361 


Tp  the  true  course  =  34?  l9'A2iLog.  tangent    =      9. 834209 


To  find  the  true  Distance  :— 

As  radius  =     .    .    .    90?0;     Log.  co-s^cant  =;:  10.000000 

Istodiff.oflat.  =     .112.5       Log,=     .    .     .  2.051152 

So  is  the  true  course=34?  19'  12rLog.  secant  =  ,  10. 083072 


To  the  distance  3=  136. 2  miles.  Log.  =    . 


2.134224 


Hence  the  course  made  good  is  N.  .34?  19'  W.,  or  N.W.  b.  N.  neariy^ 
and  the  distance  136  miles. 

Example  3. 

There  is  a  harbour  2  miles  broad,  in  which  the  tide  is  running  N.W*  b.  N. 
at  the  rate  of  3  miles  an  hour.  Now,  a  waterman  who  can  pull  his  boat 
at  the  rate  of  5  miles  an  hour,  wishes  to  cross  the  harbour  to  a  point  09 
the  oppoiSte  side  bearing  E.N.E.;  required  the  direction  in  whidi  he  should 
pull^  so  as  to  meet  with  the  Iqast  possible  resistance  from  the  force  of  the 
tide  in  gaining  the  intended  poinl^  Md  the  time  that  it  wiU  take  him  to 
reach  that  point } 

SpluUott^'^ince  the  principles 
of  this  Problem  are  but  little  un- 
derstood by  the  generality  of  young 
navigators,  a  brief  account  of  the 
geometrical  construction  will  be 
given,  with  the  view  of  elucidating 
and  rendering  familiar  the  nature 
of  the  corresponding  calculations. 
Thus, 

With  the  chord  of  60?  describe  the  arch  N  E  S  W  j  draw  the  porth  and 
south  line  NS,  and,  at  right  angles  thereto,  the  east  and  west  line  WP; 
make  the  arc  N  a  =  3  points,  and  draw  the  N.W.b.  N.  line  A  a  D,  which 
make  equal  to  3  ladles  (t^en  from  a.ny  scale  pf  jsqu^l  pants),  to  xepresent 
the  direction  of  the  harbour ;  perpendicular  thereto  draw  the  N.E.  b.  E.  line 
AbC,  whiA  wake  e^  tp  2  miles,  to  reprcseoC  die  bxttadth  of  Ae  har- 
bour ;  and,  from  the  point  C,  draw  the  line  C  G  parallel  to  AD,  which  lines 
will  represent  the  ctastern  ^d  w^fim  ^tmf^  ^  tiw  harbour  respectively. 


CTTRUtfT  f  AILINO.  371 

Make  N c  equal  to  6  points^  and  draw  the  E.N.E.  line  Ac¥,  cutting  C  G 
in  B;  then  will  6  represent  the  point  to  which  the  waterman  intends  to 
cross.  Take  5  miles  in  the  compasses ;  place  one  foot  on  the  point  D ;  and 
where  the  other  falls  upon  the  E.NJB.  line  A  F,  there  make  a  point,  as  at  F, 
and  draw  the  line  DF;  parallel  to  which,  draw  the  line  A  G,  and  it  will 
represent  the  distance  and  direction  in  which  the  waterman  must  pull  to 
gain  the  point  B :  for  in  the  time  that  he  would  reach  the  point  G,  by  pull- 
ing at  the  rate  of  5  mUes  an  hour,  the  tide,  running  at  the  rate  of  3  miles 
an  hour,  would  carry  him  to  the  poirft  B ;  because  B  G  bears  the  .same  pro^i 
portion  to  3  miles  an  hour  that  A O  does  to  5.  Now,  AG,  being  applied 
to  the  same  scale  of  equal  piurts  firom  which  the  other  sides  were  ti^en^ 
will  measure  2, 95  miles,  and  the  angle  G  AE^  or  eAB,  being  applied  to 
the  line  of  chords^  will  measure  13?33t ;  benee  the  direction  in  which  he 
should  puU,  is  £•  18?33  C  S^  or  E. b.  S.  ^  S«  nearly* 

Now,  in  the  triangle  A  D  F,  given  the  side  A  D  s:  3  milesy  the  side  D  F 
=t  5  miles,  and  the  angle  D  A  F  :=:  9  points  (being  the  difference  between . 
E.N.E«  and  N.W.  b.  N.,.  measured  by  the  arc  a  c),  to  find  the  angle  A  F  D. 
Hence^  by  oblique  aisled  trigonometry,  Problem  L,  page  1 779 

As  the  side  DF  =  5  miles.  Log.  ar.  comp.  =:  9.301030     ' 
Is  to  the  angle  A  ::  9  points.  Log.  sine  =:    .     9. 991574 
So  is  the  side  A  D  =  3  miles,  XiOg.  =:    ...    0. 47712 1 

To  the  angle  A  FD  =  36?2:55^  Log.  =     .    9769727 

Now^  because  the  straight  line  A  F  falls  upon  lik  two  parallel  straight 
lines  D F  and  AG,  it  makes  the  alternate  angles  equal  to  one  another ; 
therefore  the  angle  DFA  is  equal  to  the  angle  FAG,---Euclid,  Book  L, 
Prop.  29 j  but  the  angle  DFA  is  known  to  be  36?2'55'/;  therefore  the 
angle  FAG,  meosuredby  thearece,  is  also  equal  to  36?  2 '55.^;  ancUif  to 
the  ang^  FAG  weaM  the  angle  BAG  =:  U?i5'  (being  the  difference 
betwe^i  NJS.  b.  E#  and  S.N.E.,  measured  by  the  arc  j&  c)^  the  sum  ^ 
47?17-55^is  the  angle C AG, measured  by  thearc  be.    Then^ 

In  the  right  angled  triangle  A  CG,  given  the  angle  C  A  G  ==  47?  17'55r 
and  the  side  A  C  z=  2  miles,  the  breadth  of  the  harbour,  to  find  the  ride 
AG  equal  to  the  distance  which  the  waterman  must  pull  befiare  he  ean 
reach  the  pomt  B.  Hence,  by  rigbt  angled  trigonometry,  Probleip  U*, 
page  172,  making  A  C  radius^ 

As  radius  =:  .  ,  90?0;:  Log.  co-secant  =  10.000000 
IstothesideAC=  2miles,Log.  =  *  .;  .  0.301030 
So  is  the  angle  CAG=:47?17'55^Log.secant=10. 168657 

V»thedHtaice  s  AO  s  2«949  Mies.  Leg*  9  0.4aM»7 

Digitized  by  VjOOQ IC 


To  find  the  Tinje  requisite  ts  reach  the  Point  B  :*- 

M  distance  5  miles.  Log.  an  comp.  =  9.301030 
Is  to  1  hour^  or  60  minutes.  Log.  ==  .  1. 778151 
So  is  AG  =  2.949  miles,  Log.  =       .    .    0.469687 

Tothetime=35r23;. 34=35". 389    Log.=  1.548868 

To  find  the  Dii^ction  in.  which  he  should  pull  or  steer :— - 

.  From  the  angle  6Ae  =  47?17'55T,  take  away  the  angle  6AE  = 
33?45f,  and  the  remaining  angle  £  A  e  =  13?32f55r  is  the  direct  course 
which  he  should  steer .j  viz.,  E.  IS?3S'.  S.,  or.E.  b.  S.  i  S.  nearly. 

Hence  it  is  evident,  that  if  the  waterman  pulls  in  the  direction  of  E. 
IS'tSS'  S.  or  E.  b.  S.  i  S.  nearly,  he  will  reach  the  intended  point  in  the 
space  of  about  35  minutes  and  23  seconds. 


SOLUTION  OP  PROBLEMS  RB^iATlVE  TO  THE  ERRORS  OF 
THE  LOG- LINE  AND  THE  HALF -MINUTE  GLASS,  BY 
LOGARITHMS. 

The  instruments  generally  employed  at  sea,  for  finding  the  distance  run 
by  a  ship  hi  a  given  time,  are  the  log-line  and  the  hdf-minute  glass.'  Now, 
since  a  ship's  reckoning  is  kept  in  nautical  miles,  of  which  60  make  a 
degree,  the  distaiice  between  any  two  adjacent  knots  on  the  log-line  should 
bear  Ihe  same  proportion  to  a  nautical  mile  that  half  a  minute  does  to  an 
hour ;  viz.,  the  one  hundred  and  twentieth  part.  Arid,  since  a  nautical 
mile  contains  6080  feet,  the  true  length  of  a  knot  is  equal  to  6080  divided 
by  120;  that  is,  50  feet  and  8  inches  :  but,  because  it  is  advisable  at  all 
times  to  have  the  reckoning  a-head  of  the  ship,  so  that  the  mariner  may 
be  looking  out  fbr  the  land  in,  sufficient  time,  instead  of  his  making  it 
unexpectedly,  or  in  an  unpre|3ared  moment,  48  feet,  therefore,  is  the  cus- 
tomary measure  allowed  to  a  knot.  And,  to  make  up  for  any  time  that 
may  be  unavoidably  lost,  in  turning  the  half-nrinute  glass,  its  absolute 
measure  should  not  exceed  twenty^-nine  seconds  and  a  half. 

The  method  of  finding  the  hourly  rate  of  .sailing,  or  distance  run  in  a 
given  time,  by  the  log-line  and  the  half- minute  glass,  is  subject  to 
many  errors  :  thus,  a  new  log-line,  though  divided  with  the  utmost  care 
and  attention,   is  generally  found  to  contract  after  being  fii^t  used; 

■  0..:  ■       /V-    '-.    >-■    ^  ^■^'-'' 


Google 


Digitized  by 


LOG-LINB  AND  HALF-MINUTB  GLASS.  273 

and^  after  some. wear,  it  stretched  so  very  considerably  as  to  be  out  of  due 
proportion  to  the  measure  of  the  half-minute  glass.  Nor  is  the  'half- 
minute  glass  itself  free  from  error :  for  this  instrument  is. so  very  liable  to 
b^  affected  by  various  chuiges  of  weather,  from  moist  to  dry,  and  con- 
verseLy,  that  notwithstanding  its  being  perfectly  correct  when  first  taken  on 
board,  yet  it  alters  so  sensibly  at  sea,  that  at  One  time  it  will  run  out  in 
the  short  space  of  26  or  27  seconds,  and  at  another  not  till  it  has  passed 
the  half-minute  by  several  seconds.  •  Hence  it  becomes  indispensably 
necessary  to  examine  those  instruments  frequently;  and,  if  found  erso^ 
neous,  to  correct  the  ship's  run  accordingly.  .  This  may  be  done  by  means  * 
of  the  following  rules,  whioli  are  adapted  to  a  log-line  of  48  feet  to  a 
knot,  and  to  a  glass  measuring  30  seconds. 

Problem  I. 

Given  the  Distance  sailed  by  the  Log,  and  the  Number  of  Seconds  run 
by  the  Glass;  to  find  the  true  Distance,  the  lAne  being  truly  divided. 

Rule. 
To  the  arithmetical  complement  of  the  logarithm  of  the  number  of 
seconds  run  by  the  glass,  add  the  logarithm  of  the  distance  given  by  the  log, 
and  the  constant  logarithm  1. 477121*  ;  the  sum  of  these  three  logarithms, 
abating  10  jn  the  index,  will  be  the  logarithm  of  the  true  distance  sailed. 

Eaiample  1. 

Let  the  hourly  rate  of  sailing  be  1 1  knots,  and  the  time  measured  by 
the  glass  33  seconds ;  required  the  true  rate  of  sailing  I 

Seconds  run  by  the  glass  =  33,  Log.  ar.  conip.  =  8. 481486 
Rate  of  sailing,  by  log  =  11  knots,  Log.  =:  .  .  L  041393 
Constantlog.  =  .     .'. '''.     1.477121 


True  rate  of  sailing  =  1 0  knots.  Log.  =     .     .  *   .     1 .  000000 

Example  2. 

If  a  ship  sails  .198  miles  by  the  log,  and  the  glass  is  found,  on  exam- 
ination, to  runout  in  26  seconds,  required  the  true  distance  sailed  ?  ' 

Seconds  run  by  the  glass  z=  26,  Log.  ar.  comp.  zz  8. 585027 
Distan(5e  sailed  by  log  =  198  miles,  Log.  =:  •  2.296665 
Constant  log.  =  ...........     1.477121 


True  distance  sailed  =  228. 46  miles.  Log.  =    .    2. 35881*3 

*  This  is  the  logariihm  of  30  secoDds^  the  tru^  measure  of  the  half-minute  glass. 

T 

Digitized  ^by  LjOOQ  IC 


274  NAVIGATION. 

.      .  PaOBL£M    II. 

Given  the  Distance  sailed  by  the  Log,  and'the  measured  Length  of  a  Kffot; 
tofindthetnie  Distance,  the  Glass  beimg  correct. 

Rule. 

To  the  logarithm  of  the  distance  given  by  the  log,  add  the  logarithm  of 
the  measui-ed  length  of  a  knot,  and  the  constant  logarithm  8.318759*; 
the  sum  of  these  three  logarithms^  rejecting  10  in  the  index,  will  be  the 
logarithm  of  £he  true  distance  sailed. 

Example  1. 

Let  the  hourly  rate  pf  sailing  be  9  Hnot^,  by  a  log-line  which  measures 
53  feet  to  a  knot ;  required  the  true  rate  of  sailing  ? 

Hourly  rate  of  sailing  IT  9  knots.  Log,  =  .  0.954243 
Measured  length  of  a  knot  =  53  feet,  Log.  =:  .  U  724276 
Constant  log.  =       ........-•    8.318759 

True  rat^  of  sailing  =  9. 937  knots,  Log.  =  .    .    0. 997278 

Example  2. 

Let  the  distance  sailed  be  240  miles,  by  a  log-line  which  tneasures  43 
feet  to  a  knot ;  required  the  true  distance  sailed  ? 

Distance  sailed  by  Jog  e  240  miles.  Log.  »2. 38021 1 
MeaiBured  length  of  a  knot  ss  43  feet,  Log.  =  1. 633469 
Constant  log.  =    ..........     8.318759 


True  distance  sailed  s=  215  miles,  Log;   s     ,    2.338439 


PaoBLBM  IIL 

Given  the  measured  Length  of  a  Knot^  the  Numher  of  Seconds  run  hy  the 
Glass f  and  the  Distance  sailed  by  tlie  Log  ;  to  find  the  true  Distance 
sailed^ 

Rule.  ' 

To  the  arithmetical  complement  of  the  logarithm  of  the  number  of 
seconds  run  by  the  glass,  add  the  logarithm  of  the  measured  length  of  a. 
knot,  the  logarithm  of  the'  distance  sailed  by  the  log,  and  the  constant 


*  Ttils  is  the  arithmetical  complement  of  the  logarithm  of  48,  the  gcneraHy»approyed 
IcD^hof  aknot.     •  • 


Digitized  by 


Google 


LOG-LINB  AND   HAJLF-MIN'UTB   GLASS.  275 

logarithm  9. 795880*;  the  sum  of  these  four  logarithms,  rejecting  20  from 
the  index,  will  be  the  logiarithm  of  the  trae  distance  sailed 

Example  1. 

Let  the  hourly  rate  of  sailing  be  12  knots,  the  measured  length  of  a 
knot  44  feet,  and  the  time  noted  by  the  glass  25  seconds ;  required  the 
true  rat^  of  sailing  ? 

Seconds  run  by  the  glass  =  25,  Log.  ar.  comp.=8. 602060 
Measured  length  of  a  knot=44  feet.  Log.  =  1 .  643453 
Rate  of  sailing  by  log  :fs  12  knots.  Log.  =  !•  079181 
Constant  log.  = g.  795880 

True  rate  of  sailing  =  13. 2  knoU,  Log.  =     .     1 .  120574 

Example  2. 

Let  the  distance  sailed  by  the  log  be  354  miles,  the  measured  length  of 
a  knot  52  feet,  and  the  interval  run  by  the  glass  34  seconds ;  required 
the  true  distance  sailed  ? 

Seconds  run  by  the  glass  =  34,  Log.  ar.  comp.=  8.468521 
Measured  length  of  a  knot  ^  52  feet.  Log.  =  •  1. 716003 
Distance  sailed  by  log  =  354  miles,  Log.  ss  2. 549003 
Constant  log.  a •    •    •    9. 795880 


True  distance  =  338. 38  miles,  Log.  s      .    .    .    2. 529407 


Pboblem  IV. 

Gioen  the  Number,  of  Seconds  run  by  any  Glass  whatever,  to  find  the 
correiponding  Length  of  a  Knot,  which  shall  be  truty  proportional  to 
the  Measure  of  thai  Glass. 

RULB. 

To  the  logarithm  of  10  times  the  number  of  seconds  run  by  the  glass, 
add  the  constant  logarithm  9. 204120,  and  the  sum,  abating.  10  in  the 
index,  will  be  the  logarithm  of  the  proportional  length  of  b,  kndt,  in  feet, 
correqM>nding  to  the  given  glass. 

*  This  is  thesQin  of  the  two  precedioi:  constant  lo^thmt ;  thus  1.477121  .-f  8.318759 
«  9. 795880. 

T  2 


Digitized  by 


Google 


276 


NAVIGATION. 


Example  1.  *   ' 

Required  the  length  of  a  knot  corresponding  to  a  glass  that  runs  27 
seconds  ?  ■ 

Number  <}f  seconds  27  x  10  =  270  Log.  -     2.431364 
Constant  log.  =   ......    .  N    •    •    9-2^120 

True  length  of  a  knot,  in  feet,  =  43. 2  Log.  =     1 .  635484 

Example  2. 

Required  the  length  of  a  knot  corresponding  to  a  glass  that  runs  34 
seconds? 

Number  of  seconds  34  x  10  =  340    Log.  =    ?.  531479 
..Constant  log.  = 9.204120 


IVue  lengtli  of  a  knot,  in  feet,  =  54. 4    Log.  =  1 .  735599 


SOLUTION  OF  A  PROBLEM  IN  GREAT  CIRCLE  SAILING, 

Very  usefid  to  Ships  going  to  Van  Diemen's  Land,  or  to  New  South  fValeSf 
by  the  way  of  the  Cape  of  Good  Hope. 

Great  Circle  Sailing  is  the  method  of  finding  the  successive  latitudes 
and  longitudes  which  a  ship  nuist  make  ^  with  the  courses  that  she  must 
steer,  and  the  distances  to  be  run  upon  such  courses,  so*  that  her  track  may 
be  nearly  in  the  arc  of  a  great  circle,  passing  through  the  place  sailed  firom 
and  that  to  which  she  is  bound. 

The  angle  of  position  is  an  angl6  which  a  great  circle,  passing  fhrough 
two  places  on  the  sphere,  makes  with  the  meridian  of  one  of  them ;  and 
shows  the  true  position  of  each  place,  in  relation  to  the  intercepted  arc  of 
the  great  circle  and  the  respective  meridians  of  those  places. 

The  polar  angle  is  an  arc  of  the  equator  intercepted  between  the  meri- 
dians, or  circles  of  longitude,  of  two  given  places  on  the  sphere. 

On  the  sphere,  the  shortest  distance  between  two  pl^es  is  expressed  by 
the  arc  of  a  great  circle  intercepted  between  those  places:  consequently 
the  spiral,  or  rhumb  line,  passing  through  two  places  on  the  sphere,  can 
never  represent  the  shortest  distance  between  those  places,  unless  such 
rhumb  line  coincides  with  the  arc  of  a  great  circle ;  and  this  can  never 
hi^pen  but  when  the  places  are  situate  under  the  equator,  or  under  a 


Digitized  by 


oy  Google 


GRBAT  CIRCLB   SAILING.  277 

meridian.  Hence^  although  Mercator's  Sailing  resolves  correctly  all  the 
cases  incident  to  a  ship's  course  along  the  rhumb  line  passing  through  two 
places^— yet,  since  there  is  no  case  in  which  the  course,  along  the  direct 
rhumli  line  indicates  the  shortest  distance  between  those  places^  except 
when  they  both  lie  under  the  same  meridian,  or  under  the  equator,  the 
distance,  therefore,  obtained  by  that  method  of  sailing,  must  always  exceed 
tlie  truth  (the  above-mentioned  positions  excepted) ;  and  the  nearer  the 
places  are  to  a  parallel  of  latitude,  and  the  farther  they  are  removed  from 
the  equator,  the  greater  will  be  the  error  in  distance. 

Now,  since  it  is  f)-equently  an  object  of  the  greatest  importance,  to  the 
commander  of  a  ship,  to  reach  the  port  to  whi^h  he  is  bound  by  the  short- 
est route,  and  in  the  least  tiitie  possible,-^particularly  to  the  commander  of 
a  ship  bound  from  the  Cape  of  Good  Hope  to  Van  Diemen's  Land,  or  to 
His  Majesty's  Colony  at  N<ew  South  Wales,  where  the  length  of  the  voyage 
generally  occasions  a  great  scarcity  of  fresh  water, — the  following  Problem 
is,  therefore,  given^  by  which  all  the  particulars  connected  with  the  shortest 
possible  route  between  those  places  will  be  fully  and  clearly  Exhibited, 

Were  a  ship  to  sail  exactly  in  the  arc  of  ^  a  great  circle  (not  under  the 
equator  or  upon  a  meridian),  the  navigator  would  be  obliged  to  keep  con- 
tinually altering  her  course ;  but,  as  this  would  be  attended  with  more 
trouble  and  inconvenience  than  could  be  reasonably  admitted  into  the 
general  practice  of  navigation,  it  has  been  deemed  sufficiently  exact  to 
determine,  a  certain  number  of  latitudes  and  longitudes  through  which  a 
ship  should  pass,  with  the  relative  courses  and  distances  between  them ;  so 
that  the  track,  thus  indicated,  though  not  exactly  in  the  arc  of  a  great 
circle  may,  notwithstanding,  approximate  so  very  near  thereto,,  as  not  to 
produce  any  sensible  difference  between  it  and  the  true  spherical  track. 


Problem. 

Given  the  Latiiudea  and  Lmigitudes  of  two  Places  on  the  Globe,  to  deter- 
mine the  true  spherical  Distance  betwfien  iliem  ;  together  with  the  angu- 
lar Position  of  those  Places  with  respect  to  edch  other,  and  the  successive 
Positions  at  which  a  Ship  should  arrive  when  saiUng  on  or  near  to  the 
Arc  of  a  great  Circle,  agreeably  to  any  proposed  Change  of  Longitude. 

Rule.   . 

1  •  Find  the  true  spherical  distance  between  the  two  given  places,  by 
oblique  angled  spherical  trigonometry.  Problem  IIL,  page  202. 

2.  Fmd  the  highest  latitude  which  the  great  circle  touches  that  passes 
through  tl|e  two  given  places  5  th^t  is,  find  the  perpendicular  froii>  th^  pole 


Digitized  by 


Google 


878  NAVIGATION. 

tQ  that  circle  by  right  angled  spherical  trigonometry,  Problem  11.,  pag^  185 ; 
and  (ind,  also^  the  several  polar  angles  (made  by  the  proposed  alterations 
Qf  longitude^)  contained  between  the  perpendicular,  thus^  found,  and  the 
several  meridians  corresponding  to  the  successive  changes  of  longitude. 
'  .  3.' .  With  the  co-latitude  or  perpendicular,  sa  found,  and  the  several 
polar  angles,  compute  as  many  corresponding  co-latitudes  by  right  angled 
spherical  trigonometry,  Problem  IV,,  page  188. 

4.  With  {he  several  latitudes  and  longitudes  through  which  the  ship  is 
to  pass,  compute  die  corresponding  courses  and  distances  by  Mercator's 
Sailing,  Problem  I.,  page  238  ;^  and  they  will  indicate  the  path  along  which 
a  ship  must  sail|  so  as  to  keep  nearly  in  the  arc  of  a  great  circle. 

Note, — ^The  smaller  the  alterations  are  in  the  longitude,  the  nearer  will 
the  track,  thus  determined,  approximate  to  the  ti^th;  because,  in  very  small 
arcs,  the  difference  between  the  are  and  its  corresponding  chord,  sine,  or 
tangent,  is  so  very  trifling,  that  the  one  may  be  substituted  for  the  other, 
in  most  nautical  calculations,  without  producing  any  sensible  di£ference  in 
the  result.  • 

.      Example  I, 

A  captain  of  a  ship  bound  from  the  Cape  of  Good  Hope  (in  latitude 
34?24:  S.,  and  longitude  18?32^  E.)  to  New  South  Wales,  being  desirous 
of  making  the  north  point  of  King's  Island,  at  the  western  entrance  to  Bass' 
Strait  (in  latitude  39?37'  S.,  and  longitude  143?54;  E.),  by  the  shortest 
possible  route,  proposes,  therefore,  to  sail  as  near  to  the  arc  of  a  great 
circle  as  he  can,  by  altering  the  ship^s  course  at  every  5  degrees  of  longitude ; 
required  the  latitude  at  each  time  of  altering  the  ^course,  and,  alsoj  the 
respective  courses  and  distances  between  those  several  latitudes  and  longi- 
tudes made  by  the  proposed  changes  ? 

Cape  of  Good  Hope,  Latitude  =     34?24CS.    Longitude  =  18?32^E. 
King's  I^&d^  N.  point,  Latitude  =  39. 37  S.    Longitude  =  143. 54  E. 

Difference  of  longitude  =  1 25  ?  22 ! 

Stereographic  Projection* 

With  the  chord  of  60  degrees,  describe  the  primitive  circle  SENQ  on 
the  plane  qf  the  meridian,  or  circle  of  longitude  passing  through  the  Cape 
of  Good  Hope  i  dra^  the  line  E  Q  to  represent  the  equaitor,  and,  at  right 
angles  thereto,  the  line  S  N  for  the  earth's  axis ;  then  S  represents  the 
south,  or  elevated  pole,  and  N  the  north,  or  depressed  pole.  Take  the 
latitude  of  the  Cwpe  of  Good  Hope  in  the  oompassei  from  the  line  of 
ehorda  =  S4?1t4C,  and.  lay  it  off  from  Q  to  A ;  draw  the  diameter  A  C  ^ 


Digitized  by 


Google 


GREAT  CiaCLS  I{AILIN6. 


279 


and,  at  right  angles  thereto,  the  diameter  ^c  v.  Take  the  latitude  of  King's 
Island  =  39?37'  in  . 

the  compasses  from  ^ 

the  line  of  chords, 
and-  lay  it  off  from 
Q  to  r,  and  also  from 
£  to  r  ;  and,  with  the 
tangent  of  its  com- 
plement =  50?23': 
draw  the  parallel  cir-: 
cle  rr.  Take  the 
difference  of  longi- 
tude 125?  22^  from 
the  scale  of  semi- 
tangents,  and  lay  it  * 
off  on  the  equator 
from  Q  to  m:  thus 
90?  will  reach  from 
Q  to  C;  then  the 
excess  above  90?,  viz.,  35?22C,  will  r^ach  from  C  to  m.  With  the  secant 
of  the  complement  of  the  excess  of  the  difference  of  longitude  above  90? 
=  54?38'  (being  the  supplement  of  the  difference  of  .longitude  to  180?), 
describe  the  great  circle  S  m  N ;  the  intersection  of  which  with  the'  parallel 
circle  rr  at  B  shows  the  position  of  King's*  Island*  Then,  the  great  circle 
SBmN  reprcjsents  the  meridian  of  King's  Island.  Through  the  three 
points  AB  /  describe  a  great  circle,  and  then  will  the  arc  A  B  represent  the 
true  spherical  distance  between  the  Cape  of  Good  Hope  and  King's  Island ; 
in  which  A  represents  the  place  of  the  former,  and  B  that  of  the  latter. 
Through  P,  the  pol^  of  the  great  circle  A  B  f,  draw  the  great  circle  8  F P N; 
then  the  arc  S  F  will  be  perpendicular  to  the  arc  A  B.  Hence,  S  P  repre- 
sents the  least  co-latitude  at  which  the  ship  should  arrive  in  her  spherical 
passage  from  the  Cape  of  Good  Hope  to  King's  Island  ^  which,  being 
reduced  to  the  primitive  circle,  and  measured  on  the  scale  of  chords,  gives 
about  31 J  degrees.  The  arc  AB,  reduced  to  the  primitive  circle,  arid 
measured  on  the  line  of  chords,  shows  the  true  spherical  distance  to  be 
about  90|  degrees.  The  angle  S  A  B  is  the  aingle  of  position  which  the 
meridian  of  the  Cape  of  Good  Hope  makes  with  King's  Island;  and  the 
angle  SB  A  is  the  angle  of  position  which  the  meridian  of  King's  Island 
makes  with  the  Cape  of  Good  Hope.  These  angles,  being  reduced  to  the 
primftive  circle,  and  measured  on  the  line  of  chords,  give  about  39?  for  the 
former,  and  42^9  for  the  latter. 

Note.-^Tbe  remaining  parts  of  the  projection  will  be  explained  here- 
after. 


Digitized  by 


Google 


Calculntion. 

In  the  oblique  angled  spberical  triangle  A  S  B,  there  are  given  two  sides 
and  the  included  angle,  to  find  the  remaining  angl^  and  the  third  side; 
viz.)  the  side  AS  =  55?36C,  the  co-latitude  of  the  Cape  of  Good  Hope  ; 
the  side  B  S  =  50?23<,  the  co -latitude  of  King's  Island;  and  the  angle 
A  S  B  =  125^22'.,  the  difierence  of  longitude  between  those  places,  to  find 
the  true  spherical  distance  A  B,  and  the  respective  angles  of  position  SAB 
and  SB  A.  The  distance  may  be  readily  found  by  Remark  1  or  2,  to 
Problem  III.,  page  203  or  204 ;  as  thus  : 
DifF.oflong,ASB  = 

125922:  H-  2  =  62?4llTwicelog.sine  19. 897300 
Co-lat.  of  Cape  of 

Good  Hope=AS  55. 36  Log.  sine  =z      9. 9165 14 
Co-Iat.  of  King's 

l8land=BS     .    50. 23  Log.  sine  =      9.886676 

Sum  =    39. 70049a 


Diflf.ofco-lats.  =       5n3:         Half=    19.850245      .    .     19.850245 


Half  diff.  of  do.  ^     2?36:30'^Log.sine=  8. 658090 


Arch=i,    .    .    .   86?19^27':'Log,T.=  11.192155  Log.sine9. 999106 


HalfthesideAB  =  45.  13.    8|  Log.  sine  =:    9.851139 

Side  A  Be      .    .    90^26^7^;  which  is  the  true  spherical  distance 
between  the  two  given  places. 

To  find  the  Angle  of  Position  at  Cape  of  Good  Hope  s^  Angle  S  AB :-" 

This  is  found  by  Problem  I.,  page  198 ;  as  thus  : 

As  Uie  distance  AB  .  90?26n7^  Log.  co-secant=10. 000013 
Is  to' diff.  of  long.  A  SB  125.22.  0  Log.  sine  =  ,  9.911405 
So.is  the  co-lat.  -   BS     .     50.23.  0    Log.  sine  =:     •    9.886676 

To  the  ang.  of  posit.  SAB  38955  C   1  r  Log.  sine  =     .    9.  798094 

To  find  the  Angle  of  Position  at  King's  Island  ==  Angle  SB  A : — 

This  18  found  by  Problem  I,,  page  198 ;  as  thus : 
As  the  distance     A B      .    909 26^  1 7^  Log.  co-secant  =  10. 000013 
Istodiff.  oflong.  ASB     125.22.   0    Log.  sine  =      .    9.911405    . 
So  is  the  co-lat.    AS    «       55.36.    0    Log.  sine  =      .    9.916514 

To  the  ang.  of  pos.    SB  A   429l7:20irLog,rine  =     .    9.827932 


Google 


Digitized  by 


GREAT  CIRCLE  SAILING.  281 

To  find  the  Perpendicular  FS  =  the  Complement  of  the  highest  southern 
Latitude  at  which  the  Ship  should  arrive  in  the  proposed  Route : — 

Here  we  have  a  choice  of  two  right  angled  spherical  triangles,  viz.,  ASlF 
fUid  B  S  F ;  in '  each  of  which  the  hypothenuse  and  the  angle  at  the  base 
are  given,  to  find  the  perpendicular.  Thusj  in  the  triangle  ASF,  given 
the  hypothenuse  AS,  55°36'  =  the  co-latitude  of  the  Cape  of  Good 
Hope,  and  the  angle  at  the  base,  S  AF  38?55 '  K  =  the  -angle  of  position 
at  that  place^  to  find  the  perpendicular  F  S  =  the  complement  of  the 
highest  latitude  at  which  the  ship  should  arrive.  Hence,  by  right  angled 
spherical  trigonometry.  Problem  II.,  page  185, 

As  radius  = 90?  0^  OT  Log.  co-sec.=  10. 000000 

Is  to  co-lat.  C.  Good  Hope    A  S  =  55. 36.    0    Log.  sine  =       9. 9165 14 
Soistheang.of  position  .SAF=  38, 55.    1     Log.  sine  =       9.798094 

To  the  perpendicular     PS    =    .    31. 13.  13|- Log.  sine  =       9.714608 

Highest  lat.  at  which  the  ship 

should  arrive  =      .     «     .     ,     »    58?46'.46|^  south. 

*  Hence  the  true  spherical  distance  between  the  Cape  of  Good  Hope  and 
the  north  point  of  King's  Isltuid,  is  90?26.' 17^,  or  5426.3  miles  3  the 
angle  of  position  at  the  Cape  of  Good  Hope,  is  38?55'1'';  and  that  at 
King's  Island,  42?  17-20^ ;  and  the  highest  southern  latitude  at  which  the 
ship  should  arrive,  58 ?46C  46'/.  Now,  by  Mercator's  Sailing,  the  course 
from  the  Cape  of  Good  Hope  to  King's  Island  is  S.  87?r.  E.,  or  E.  i  S. 
nearly^  and  the  distance  6011.  2  miles;  whence  it  is  evident,  that  if  a  ship 
sails  on  the  direct  rhumb  line  indicated  byMercator's  Sailing,  she  will  have  to 
run  a  distance  of  no  less  than  585  miles  more  than  if  her  course  had  been 
shaped  along  the  arc  of  a  great  circle  passing  through  the  two  given  places. 
Now,  since  it  is  extremely  difficult  for  persons  unacquiunted  with  the 
doctrine  of  spherics  to  reconcile  a  route  to  their  senses,  as  the  shortest 
distance  between  two  places,  which  carries  them  nearly  22  degrees  to  the 
southward  of  the  middle  latitude  between  the  two  given  places  ;  and  since, 
in  sailing  on  the  arc  of  a  great  circle;,  the  course  ought  to  be  changing 
constantly,  with  the  view  of  keeping  the  side  of  the  polygon  on  which  the 
ship  sails  as  near  to  the  arc  of  its  circumscribing  circle  as  possible,  or  that 
the  difference  between  the  arc  and  its  chord  may  be  so  small  that  the  one 
may  be  substituted  for  the  other  without  sensibly  affecting  the  result  in 
nautical  operations,— -I  shall, .  therefore,  show  the  successive  latitudes  at 
which  the  ship  should  arrive  at  every  5  degrees  of  longitude,  as  proposed 
(which  is  sufficiently  near  to  preserve  the  desired  ratio  between  the  arc  and 
its  chord)  i  together  with  the  respective  courses  and  distances,  by  Merca-^ 


Digitized  by 


Google 


282.  NAVIGATION. 

tor's  Sailing,  between-  those  several  successive  latitudes  and  longitudes : 
then,  if  tbe  jsum  of  the  several  distances  coincide,  or  nearly  so,  with  the 
true  spherical  distance  found  as  ab^ve,  the  senses  must  become  reconciled 
to  the  propriety  of  adopting  that  high  southern  route  at  which  they 
originally  seemed  to  recoil. 

In  order  to  determine  the  several  successive  latitudes  at  which  the  ship 
muist  arrive,  we  must  previously  compute  the  vertical  or  polar  angles  ASF 
and  B  S  :  then,  if  the  sum  of  these  angles  make3  up  the  whole  difference 
of  longitude,  or  polar  angle  between  the  two  ^ven  places,  it  will  be  a 
convincing  and  satisfactory  proof  that,  for  so  far,  the  operations  will  have 
been  properly  conducted.  Now,  in  the  right  angled  sphtrical  triangle 
ASF,  given  the  hypothenuse  A  S,  55?  36'  =  the  co-latitude  of  the  Cape 
of  Good  Hope,  and  the  perpendicular  F  S,  31  °  13'  13^^  ==  th^  complement 
of  the  highest  latitude  at  which  the  ship  should  arrive,  to  find  the  vertical 
or  polar  angle  F  S  A.  And,  in  the  right  angled  spherical  triangle  B  S  F^ 
given  the  hypothenuse  B  S,  50?23'  =?  the  co-latitude  of  King's  Island,  and 
the  perpendicular  F  S,  3 1  ?  1 3^  13^r,  to  find  the  vertical  or  poliir  angle  B  S  F« 
Hence,  by  right  angled  spherical  trigonometry.  Problem  I.,  page  184, 

To  find  the  Polar  Angle  A  S  F  .-^ 

As  radius  ±5 90?  0'  0?  Log.  co-secant  =    10.000000 

Is  to  the  CO' latitude  A  S  s  S5. 36.  0  Log.  co-tangent  a  9. 83550d 
So  is  the  co-latitude  FS  =:     31.13. 13^  Log.  tangent     «      9.782550 

To  the  polar  angle  ASF  =     65-? 28 C48r  Log.  co-sine     =       9.618059 

•    .  To  find  the  Polar  Angle  BS  F:— 

As  radius  =  .  .  •  .  .  9#?  0^  07  Log.  co- secant  =  10.000000 
Is  to  the  co-latitude  B  S  =  50. 23.  0  Log.  co-tangent  =  9. 91^906 
So  isthe  co-latitude  PS  =a  31. 13. 13^  Log.  tangent  =     .    9.782550 

To  the  polar  angle  B  S  F  =  59?53 H 27  Log.  co-sine    =     .    9. 700456 

And,  since  the  sum  of  the  polar  angles,  thus  obtained,  viz.,  ASF 
65?28U8r  +  BSF59?63U2'r  =  125^22^07,  makes  up  the  whole  dif- 
ference of  longitude  between  the  two  given  places  expressed  by  the  whole 
angle  A  S  B,  it  shows  that  thus  far  the  work  is  right. 

Now,  on  the  equator,  from  Q  to  m,  lay  off  the  proposed  changes  of  Ion* 
gitude,  viz.,  5?,  10%  15?,  20?,  25%  &c.  These  are  to  be  taken  respectively, 
in  the  compasses,  from  the  scale  of  semi-tangents,  reckoning  backwardB 
from  90?  t&wards  0?,  till  the  proposed  changes,  of  longitude  reach  the 
centre  C ;  and  then  forv^ards  on  that  scale,  or  from  0?  tawards  90?,  till 
those  changes  of  longitude  meet  the  point  m;  thns,  the  extent  from  90? 


Digitized  by 


Google 


GREAT  CIRCLB  SAILING.  283 

to  85?  will  reach  from  Q  to  5? ;  the  extent  from  90?  to  80?,  will  reach 
from  Q  to  10?,  and  so  on  to  the  centre  C ;  then,  the  extent  from  0?  to-  5?, 
will  reach  from  C  to  95?  ;  the  extent  from  0?  to  10?,  will  reach  from  C  to 
100?,  and  so  on  to  the  point  m.  Through  the  points  S  and  N,  and  the 
several  points  made  by  the  proposed  changes  of  longitude  on  the  equator, 
draw  arcs  of  great  circles,  viz.,  S  1,  5?  ;  S  2, 10?  ;  S3,  15? ;  S  4,  20?,  &c. 
&c. ;  and  then  the  arcs  S  1,  S  2,  S  3,  &c.  &c.,  will  represent  the  respective 
complements  of  the  several  latitudes  at  which  the  ship  should  arrive  at  the 
given  changes  of  longitude ;  the  true  values  of  which  may  be  found  in  the 
foUoiving  manner,  viz.. 

From  the  polar  angle  ASF,  subtract  the  proposed  changes  of  longitude 
continually ;  and  the  several  polar  angles  made  by  those  changes,  and  con- 
tained between* the  perpendicular  FS  and  the-  co-latitude  of  the  Cape  of 
Good  Hope  =  S  A,  will  be  obtained.  Thus,  from  the  polar  angle  A  S  F  =± 
65?28U8r,  let  5?  be  continually *uA/racfed,  and  the  results  will  be  FS  1 
=  60?28!48r;FS.2=  55?28M8r-FS3  =;  50?28:48r,  &c.  &c.  And, 
since  the  last .  subtraction  in  this  trif^ngle  leaves  the  remainder,  or  polar 
angle,  FS  12=  5?28M8r,  which  is  28'.481  greater  than  the  proposed 
alteration  of  longitude,  therefore,  in  the  triangle  HSF,  where  the  polar 
angle  S  is  59?53n2?  (and  where  the  several  polar  angles  contained 
between  the  perpendicular  F  S  and  the  co-latitude  of  King's  Island  are  to 
be  determined  by  a  contrary  process  to  that  which  was  observed  in  the  pre- 
ceding triangle),  the  first  polar  angle  is  expressed  by  5?  —  28'48f  = 
4?31M21^  =  the  angle  FS  a;  to  which  let  the  proposed  alterations  of 
longitude  be  continually  added,  and  the  sums  will  be  PSA  =  9?31M2f ; 
FSc  =  14?81'12T,  &c.  &c.  Those  various  results  are  to  be  arranged 
agreeably  to  the  form  exhibited  in  the  first  column  of  the  following  Table ; 
and,  since  they  respectively  express  the  true  measures  of  the  several  polar 
angles  contained  between  the  meridians  of  the  given  places  and  those  of 
the  several  co^latitudes  to  which  they  correspond,  it  is,  therefore,  manifest 
that  those  results  reduce  the  two  right  angled  spherical  triangles  (ASF 
and  BS  F)'  into  a  series  of  right  angled  spherical  triangles;  to  each  of 
which  the  perpendicular  FS  is  common.  Then,  in  each  of  these  triangles, 
we  have  the  perpendiciriar  aiid  the  angle  adjacent,  to  find  the  hypothenuse 
or  co-latitude.  Thus,  in  the  right  angled  spherical  triangle  F  S  1 ,  right 
angled  at  F,  given  the  perpendicular  FS  =  31?  13 '131^,  and  the  polar 
angle  PS  1  =  60?2SM6r,  to  find  the  hypothenuse  or  co-latitude  S  1 ;  in 
the  right  angled  spherical  triangle  FS2,  given  the  perpendicular  FS  = 
31?l3nSjr,  and  the  polar  angle  FS  2  =  55?28C48r,  to  find  the  hypo- 
thenuse or  co-latitude  S  2,  &c.  &c.  Hence,  by  right  angled  spherical 
trigonometry.  Problem  IV.,  page  188, 


Digitized  by 


Google 


284  NAVIGATION. 

•    To  find  the  Hypothenuse^  or  Co-Latitude  =  S  1  :— 

As  the  perpendictirar  FS  =»  31913^  13^^  Log.  co- tangent  =  10. 217450* 
Is.  to  the  radius  =        .     •     90.   0.    0       Log.  sine  =       .     10. 000000 
So  is  the  angle  F  S  1    =  .     60. 28. 48      Log.  co-sine    =       9. 692607 


To  the  co-latitude  S  1  =      50.  53. 28       Log,  co-tangent  =  9. 910057 


'  First  latitude  ==      •     .,  ..    39?  6'32/rS.^  at  which  the  ship  should  arrive. 

To  find  the  Hypothenuse^  or  Co^Latitude  =  S  2 : — 

As  the  perpendicular  FS  =  31?13a3jr  Log.  co-tahgent  =  10. 217450* 
Is  to  the  radius  =      .     .     90.    0.   0  .    Log.  sine  =  .    .     10^  000000 
So.is  theangleFS2"   =     55.28.48      Log. co-sine  ==  .      9.753349 

To  the  co-latitude  S2  =     46.55.29      Log.  co-tangent  =  9.970799 

Second  latitude  =       .     .     43?  4'3KS.^  at  which  the  ship  should  arriye. 

Hencc^  the  first  latitude  at  which  the  ship  should  arrive^  is  39?6^32^S.; 
and  the  second  latitude  43?4'3KS. :  and',  since  it  is  the  latitude  itself^ 
and  not  its  complement,  that  is  required,  if  the  log.  tangent  of  the  sum  of 
the  three  logarithms  be  taken,  it  will  give  the  latitude  direct ;  and,  by 
rejecting  the  radius,  the  work  will  be  considerably  facilitated.  Proceeding 
in  this  manner,  the  several  successive  latitudes  corresponding  to  the  pro- 
posed alterations  of  longitude  will  be  found,  as  in  the  third  column  of  the 
following  Table. 

Npw,  let  the  several  successive  lohgi.tudes  be  arranged  (agreeably  to  the 
proposed  change,  and  to  the  measure  of  the  corresponding  polar  angles,)  as 
given  in  the  second  column  of  the  following  Table ;  cmd  find  the  difference 
between  every  two  adjacent  longitudes,  as  shown  in  the  fourth  colunan  of 
that  Table.  Find  Ihe  difference  between  every  two  successive  latitudes^ 
and  place  them  in  the  .fifth  column  of  the  Table.  Take  out  from  Table 
XLIII.  the  meridional  parts  corresponding  to  the  several  successive  lati- 
tudes, as  given  in^  column  6,  and  find  the  difference  between  every  two 
adjacent  numbers,  as  given  in  the  seventh  column.  Then  fuid,  by  Merca* 
tor's  Sailing,  Problem  I.,  page  238,  the  respective  courses  and  distances 
between  the  several  successive  latitudes  and  longitudes;  and  let  those 
courses  and  distances,  so  found,  be  arranged  as  in  the  two  last  columns  of 
the  following  Table :  viz.. 


*  The  log.  co-tangent  is  used,  so  as  to  avoid  the  trouble  of  findin|^  the  arithmetical  com- 
plement of  the  log.  tangent. 


Digitized  by 


Google 


ts 


I  I  B  u  II  n  V  u  b'b-h  B  y  y  .|t  y-tt  II  H  11  u  n  II  H  H'  It 

<9«  Off  rfh>  *.  cou  lo  ^  ^  •*•  »^>-*ioioo»CA9.uiik.c;«c;iAcn 

k?ki2S:2bJ22bifibkp 

feOMNfedlOMfeOfeOfeCMlOMOOOOOQOaOCOaDaDOOQOaDODQDoig 


^kOIOfeOfdfcCtOMfeOfSlOUpMkOtdfedkdtO^IOIOlCttvtO^ 
(Ik  • 

oooeooeoooeooooooooe  o-o  o  o  o  o  o 

w 


c*»  1^  »«  M  c;«  o»  ^  >r  r*  e>*  »d  1^  ilk.  CO  10  c;i  0*5010^50      ^  ko 

vivivi*N«ppaD^c;«p»c;i;^pp»«p^^top^<fiOa^piU 


>000000©00vji0  000© ^^^ 


Coppppppppppk^UOpppppppppppp 

oooeoeeoooo^oDooooooooeeeo 


iooo>WoD*-oow<ccnGoigo«o>-*.coto«;»'vijccj'^iowg 


oc&0Ot2o)oci^*^^u*u^uc;^poio<n~^5i)«5'<p)pv^*<^S 

oaDO»C7*ieo»u«c;«otUMCi9iutc^»-Uii^ac^09ar«^^^^ 


5I^»-<yt5o0dM>-u<<C0dp^wdQ0^ppkCaD*-.p9)C#s'^ 

D0C©rf^*^W<O^O»^<C^€»«ilfc.<OWO4^*t 


c»i»«j^2j-c»wwo>o*»*opvij-v*tj»cnQow»c;'7-7-pp 


If 


fii   s. 


ft 


r  5 


?  8 


?2 


Digitized  by 


Google 


Now,  the  sum  of  the  several  successive  differences  of  longitude  =  7522 
miles,  rAakes  up  the  whole  difference  .of  longitude  between  the  two  given 
places;  the  sum  of  the  successive  differences  of  latitude  ==  2612*  53  miles, 
is  equal  to  the  whole  difi^rence  of  latitude  comprehended  under  the  high- 
est latitude  at  which  the  ship  should  arrive,  and  the  latitudes  of  the  two 
given  places,  viz.  34?24!0r  S.,  58?46^46ir  S.,  and  39?37'0r  S.— And, 
'the  sum  of  the  several  meridional  differences  of  latitude  =  3973. 85  miles, 
coincides  exactly  with  the  whole  meridional  difference  of  latitude  corres- 
ponding to  the  highest  latitude,  and  the  latitudes  of  the  two  given  places ; 
which  several  agreements,  form  an  incontestable  proof  that  the  work  has 
been  carefully  conducted. 

The  sum  of  the  several  distances  measured  on  the  consecutive  rhumb 
lines  intercepted  between  the  successive  latitudes  and  longitudes,  as  exhi- 
bited in  the  last  column  of  the  Table,  is  5426.46  miles;— but  the  true 
spherical  distance  on  the  arc  of  a  great  circle  is  5426. 30  miles ;  the  diff- 
erence, therefore,  is  only  0' .  16 ;  or,  about  ^  of  a  mile  ;  which  is  very  tri- 
fling, considering  the  extent  of  the  arc< — ^The  distance  byMercator's  sailing 
is  6011 . 2  miles ;  which  is  583  miles  more  than  by  great  circle  sailing. 

Hence,  it  is  evident  that  the  shortest  and  most  direct  route  from  the  Cape 
of  Good  Hope  to  King's  Island  is  bythelatitude  of  58?46M6f^  S.;  and  that 
the  ship  must  make^  successively,  jthe  several  longitudes  and  latitudes  con- 
tained in  the  2nd  and  3rd  columns  of  the  Table,  in  the  same  manner,  pre- 
cisely, as  if  they  were  so  many  headlands,  or  places  of  rendezvous,  at  which 
she  was  required  to  touch.— The  first  course,  therefore,  from  the  Cape  of 
Good  Hope  is  S.  40?22^  £.  disUnce  371  miles,  which  will  bring  the  ship 
to  longitude  23?32;  £.  and  latitude  39i?6'32r  S.;--the  second  course  is  S. 
43?31'  £.  distance  328  miles,  which  brings  the  ship  to  longitude  28?32'. 
E.  and  latitude  43?4^3ir  S.;  the  third  course  is  S.  46?56:  E.  distance 
292  miles,  which  brings  the  ship  to  longitude  33?32^  E.  and  latitude 
46  ?  23' 39 'r  S  ;— and  so  on  of  the  rest.— Whence,  it  is  evident  that  if  the 
ship  sails  upon  the  several  courses,  and  runs  the  corresponding  distances 
respectively  set  forth  in  the  two  last  columns  of  the  Table,  she  will,  most 
assuredly,  arrive  at  the  several  successive  longitudes  and  latitude^  pointed 
out  in  the  2nd  and  3rd  columns  of  that  Table ;  and  thus  will  she  reach 
King's  Island,  the  place  which  it  is  intended  she  shall  make,  by  a  track 
585  miles  shorter  than  if  such  track  had  been  determined  agreeably  to  the 
principles  of  Mercator's  sailing. 

And,  in  a  long  voyage,  like  the  present,  in  which  ships  generally  expe* 
rience  a  great  scarcity  of  fresh  water,  particularly  those  bound  to  His  Ma- 
jesty's colony  at  New  South  Wales  with  troops,  or  convicts,  the  saving  of 
585  miles  run  at  sea  becomes  a  consideration  of  no  inconsiderable  import- 
ance. 

Nor  is  there  ^any  more  difficulty  in  sailing  on  the  arc  of  a  great  circle^ 


Digitized  by 


Google 


GBBAT  cmCLV  SAILING.  287 

thus  detennined,  than  there  is  in  saijifig  on  a  parallel' of  latitude ;  for,  if 
the  ahip's  compate  be  but  tolerably  good,  the  variation  thereof  carefully 
attended  to,  and  proper  attention  paid  to  the  steerage,  the  courses  and  dis« 
tances  expressed  in  the  two  last  columns  of  the  Table  will,  undoubtedly, 
cany  the  ship  direct  from  the  Cape  of  Good  Hope  to  the  north  point  of 
King's  Island,  without  ever  referring  to  celestial  observation  for  either  lati- 
tude or  longitude ;  provided^  indeed,  that  the  ship's  way  is  not  affected  by 
current? : — butj  since  the  courses  contained  in  the  8th  column  of  the 
Table,  express  the  true  bearings  between  the  several  successive  latitudes 
and  longitudes  through  which  the  ship  must  pass ;  these  must,  therefore, 
be  reduced  to  the  magnetic,  or  compass  course,  by  allowing  the  observed 
variation  to  the  right  handthereof  if  it  be  westerly,  but  to  the  left  hand  if 
easterly ;  this  being  the  converse  process  of  reducing  the  magnetic,  or 
course  steered  by  compass,  to  the  true  course. — And,  if  the  spherical 
track,  so  determined,  be  delinelEited  on  a  Mercator's  chart,  it  will,  perhaps, 
not  only  simplify  the  navigation,  but  also  point  out  to  the  mariner 
any  known  land  that  may  be  adjacent  thereto  *  ;  and  thus  enable  him  to 
alter  his  course  as  occasion  may  require. — ^The  spherical  track  may  be  rea- 
dily ddineated  on  a  chart  by  means  of  the  angles  of  meeting  made  by  the 
several  latitudes  and  longitudes,  which  show  the  places  or  points  where  the 
ship  is  to  alter  her  course  :— -Now,  those  points  being  joined  by  right  lines 
will  indicate  the  triie  courses  and  distances,  or  the  absolute  route  on  which 
the  ship  must  sail  from  the  Cape  of  Good  Hope  to  King's  Island ;  then,  if 
each  day's  run  be  carefully  measured  on  the  track,  so  delineated,  (he  navi- 
gator can  always  know  his  distance  from  the  place  to  which  he  is  bound, 
without  resorting  to  the  trouble  of  calculation. 

I  have  dwelt  at  considerable  length  upon  this  Problem  for  the  express 
purpose  of  simplifying  a  sulyect  which  is  but  very  little  understood  by  the 
generality,  of  maritime  people: — and,  with  the  view  of  rendering  it  still  more 
familiar^  another  example  will  be  given  by  which  the  approximate  spherical 
route,  as  performed'  by  I^is  Majesty's  ship  Dauntless,  under  the  command 
of  .George  Cornish  Gambier,  esq.  on  her  voyage  from  Port  Jackson  to  Val- 
paraiso, in  the  year  1822,  will  be  clearly  illustrated. 

Example  2. 

His  Majesty^s  ship  Dauntless  being  bound  from  Po^t  Jackson,  in  latitude 
33?52^  S.  and  longitude  151?  16^  E.  to  Valparaiso,  in  latitude  33^^  S. 
and  longitude  71^52'  W.,  the  captain,  G.  C.  Gambier,  Esq.,  proposed  to 
navigate  bet  as  near  to  the  arc  of  a  great  circle  as  he  could,  by  altering 
her  course  at  every  5  degrees  of  longitude ;  required  the  latitude  at  each 


*  It  Is  pres amed  that  ther^  is  not  any  laxxl  to  intercept  a  ship's  progress  in  this  track. 

Digitized  by  VjOOQ IC 


288 


NAVIGATION. 


time  of  altering  th^  course^  together  with  the  respective  courses  and  dis- 
tances between  those  severaMatitudes  and  longitudes,  occasioned  by  tlie 
proposed  changes  ? 


Port  Jackson, 
Valparaiso, 


Latitude  33?52^S.     Longitude  =  151?16'  E. 
Latitude  33.    1    S.     Longitude  =     71.52    W. 


Sum  = 


223?  8' 


Difference  of  longitude  between  the  two  given  places  =    136?S2' 
Calculation. 

Since  the  elements 
of  this  Example  are 
analogous  to  those  of 
the  last;  it  is  not, 
therefore,  deemed  ne- 
cessary to  repeat  the 
mode  of  projection ; 
the  only  difference  in  . 
the.  construction  being  *l' 
that,  in  the  preceding 
diagram,  because  the 
ship  is  bound  to  a  place 
to  the  eastward  of  that 
from  which  she  is  to 
sail ;  the  latter  is,  there- 
fore, for  the  sake  of  uni- 
formity, placed  on  the 
primitive  circle  in  the 

western  hemisphere.:-— and,  in  the  present  diagram,  because  the  ship  is 
bound  to  a  port  to  the  westward  of  that  from  which  she  is  to  sail,  the  lat- 
ter (for  the  sake  of  uniformity  also)  is  placed  on  the  primitive  circle  in  the 
eastern  hemisphere : — the  letter  Q  representing  the  western  hemisphere  in 
the  forjner  case,  and  the  eastern  hemisphere  in  the  latter. 

Now,  the  figure  being  thus  constructed  on  the  plane  of  the  meridian 
passing  through  Port' Jackson-;  let  the  point  A  represent  that  place;  the 
point  B,  the  place  of  Valparaiso,  and  the  arc  AB,  the  true  spherical  dis- 
tance between  those  places ; — then,  A  S  represents  the  co-latitude  of  Port 
Jackson  ;  B  S,  that  of  Valparaiso ;  S  AB,  the  angle  of  position  at  the  for- 
mer place,  and  S  B  A,  the  angle  of  position  at  the  latter  place. — ^The  arc 
F  S,  which  is  drawn  perpendicular  to  A  B,  represents  tbe  complement  of  the 
highest  latitude  at  which  the  ship  should  arrive ;  and  the  several  arcs  S  j  ; 


Digitized  by 


Google 


GRBAT  CIRCLE   SAILING.  289 

S2j  S3;  S4;  &c.  &c.  &c.,  represent  the  complements  of  the  successive 
latitudes  through  whith  the  ship  must  pass. -^HencCj  in  the  oblique  angled 
spherical  triangle  A  B  S,  two  sides  uid  the  included  angle  are  given  to  find 
the  third  side  and  the  remaining  angles;  viz.,  the  side  AS  =s  56? 8^  the 
co-latitude  of  Port  Jackson;  the  side  BS=  56?59'  the  co-latitude  of 
Valparaiso,  and  the  angle  ASH  =  136^^52^  the  difference  of  longitude 
between  those  places ;  to  find  the  spherical  distance  AB,  and  the  respect- 
ive angles  of  position  =  SAB  and  SB  A  : — the  distance  may  be  readily 
found  by  Remark  1,  or  2,  to  Problem  IIL,  page  203  or  204 ;  as  thus : 

Diff.long.ASB136?62C  ^-2  = 

68?26:  Twice  log.  S.=:19. 936958 

Co^atitude  of  Port 
Jackson  »  A  S  56?  8'.  Log^  sine  =       9. 919254 

Co-Iat.  of  Valpa- 
raiso :^  B  S      56?59'  Log.  sine  =       9. 923509 


Sum  ST.    .39.779721 


Diff.  of  co-lat.  s      0?5H     Half  =:  .    .19. 889860^  19. 889860} 


Halfdiff.of  ditto=  0?25:30r  Log.  S.  =    7. 870262 


Arch=±  .  •  .    .  89?25U7!rLog.tang.=12.019698iLbgS.=:9. 999979 


HalfthearcAB=50?53:56rLog.sine= 9.889881} 


Side  A  B  rs  .  •  101  ?47  '52^=  the  true  spherical  distance  between  the  two 
given  places. 

To  find  the  Angle  of  Position  at  Port  Jackson  ss  Angle  S  A  B  :'— 

This  is  found  by  Problem  L,  page  i98  ;  as  thus  : 

As  the  distance  A  B  =  101?47^52r  Log.  co-secant  =  10. 009273 
Istodiff.  long.  ASB=  136.52.  0  Log. sine  =  .  .  9.834865 
So  is  the  co-latitude  B  S=  56. 59.   0   Log.  sine  ;^  .    :    9. 923509 

Toangleofpos.=S^AB=35?50C59rLog.sine=5   .    .    9.767647 

To  find  the  Angle  of  Position  at  Valparaiso  =  Angle  S  B  A  :— 
This  is  found  by  Problem  L^  page  198 ;  as  thus  i 

V 

Digitized  by  VjOOQ IC 


290  NAVIOATIOH. 

As  the  distance  AB«  101?47^52r  Log.  co-secant  s  10.009273 

Istodiff.long.ASBa  136. &2.   0  log.  sine  a    .    .    9.834865 

Soi8theco-lat.ASs:  56.   8.   0   Log. sines    •    •    9.919254 


Toanglebfpos.  SBA  =  35?26^50r  Log.  sine  «    .    .9.763392 

To  find  the  Perpendicular  F  S  k  the  Complement  of  the  highest  Southern 
Latitude  at  which  the  Ship  should  arrive :«— ^ 

Here  we  have  a  choice  of  tWo  right  angled  spherical  triangles^  viz.  ASF 
and  B  S  F ;  in  each  of  which  the  hypothenuse  and  the  angle  at  the  base 
are  given  to  find  the  perpendicular  5 — thus,  in  the  triangle  A  S  F^  given  the 
hypothenuse  AS  =  56?8^  the  co-latitude  of  Port  Jackson;  and  the  angle 
at  the  base  S  A  B  =  35?50^59r  the  angle  of  position  at  that  place^  to 
find  the  perpendicular  F  S  » the  complement  of  the  highest  southern  la- 
titude afwhich  the  ship  should  arrive :— 

*  • 

Hence,  by  right  angled  spherical  trigonometry,  Problem  XL,  page  185> 

As  radius  =  ....  .  90^  0!  0?  Log.  co-secants  10.000000 
Is  tocp-lat.  Port  Jackson  s  AS  56.  8.  0  Log.  sine  =  .  .9.919254 
So  is  ang.  of  position  =:^  S  A  F  35. 50. 59    Log.  sine  =£  .    .  9. 767647 

To  the  perpendicular  P  S  =     29?  S^Sir  Log.  sine  =   .    .9. 686901 

Highest  lat«  at  which  the 

ship  should  arrive  =  60?54^  91  south. 

From  the  above  calculations  it  appears  evident  that  the  true  spherical 
distance  between  Port  Jackson  and  Valparaiso  is  101  ?47  '*52r,  or  6107*  87 
miles;  the  angle  of  position  at  Port  Jackson  =  35?50^59C^|  and  that  at 
Valparaiso  r=  35?26^50f,  and  the  highest  southern  latitude  at  which  the 
ship  should  arrive  =  60?54'9^. — Now,  by  Mercator's  sailing,  the  course 
from  Port  Jackson  tp  Valparaiso  is  N.  89^34  C  E.  and  the  distance  6853. 16 
miles  ;-^whence  it  is  manifest,  that  if  a  ship  sails  on  the  direct  rhumb 
line  between,  those  places,  as  indicated  by  that  mode  of  sailing,  she  will 
have  to  run  745|  miles  more  than  by  shaping  her  course  along  the  arc  of  a 
great  circle.  • 

To  compute  the  vertical,  or  Polar  Angles  ASP,  and  B  S  P  :— 

In  the  right  angled  spherical  triangle  ASF,  given  the  hypothenuse  A S 
56 ?S'  =  the  co-latitude  of  Port  Jackson,  and  the  perpendicular  FS 
29?5'51T  c:  the  complement  of  the  highest  latitude  at  which  the  ship 
should  arrive  ;  to-  find  the  vertical,  or  polar  angle  A  S  P.— And,  in  the 
right  angled  triangle  B  S  P,  given  the  hypothenuse  B  S,  56?59'  ==  the  Co- 


Digitized  by 


Google 


GRBAT  CIACUI   SAILING.  291 

latitude  of  Valparaiso^  and  the  perpendicular  FS,  29?5'51f ;  to  find  the 
polar  angle  B  S  F, — Hence^  by  right  angled  spherical  trigonometry^  Prob.  I., 
page  184, 

To  find  the  Polar  Angle  A  S  F :-« 

As  radius  = 90?0'  Or  Log.  co-secant  =     10- 000000 

Is  to  the  co-latitude  AS  =z.  .  56. 8.  0  Log.  co-tangent  =  9. 826805 
So  is  the  co-latitude  F  S  ==   .     29, 5. 5 1    Log.  tangent  =      •  9. 745493 

To  the  polar  angle  A  S  F  r:        Q8W.B1    Log.  co-sine  s:  .    .9. 572298 

■  I 

To  find  the  Polar  Angle  B  S  F  :^ 

As  radius  ==   ......    90?  0^  0?  Log.  co-«ecant  s:  10.000000 

Is  to  the  co-latitude  B  S  =  56. 59.  0  Log.  co-tangent  =  9. 812794 
So  is  the  co-latitude  FS  =         29. .  5. 5 1    Log.  tangent  =s       9. 745493 

To  the  polar  angle  B  S  F  =       68?47'55r  Log.  co-sine  =   .    9. 558287 

Now,  since  the  sum  of  the  polar  angles^  thus  obtained,  viz.  J0Ff 
68?4^5r  +  BSF,  68?47C55r  =  136?52^,  makes  up  the  whole  differ- 
ence of  longitude  between  the  two  given  places,  expressed  by  the  whole 
apgle  A  S  B,  it  shows  that,  thus  far,  the  work  is  right. 

To  find  the  several  successive   Polar  Angles  made  by  the  proposed 
changes  of  Longitude. 

From  the  polar  angle  ASF,  subtract  the  proposed  alteration  of  longi^ 
tude  continually,  as  far  as  subtraction  can  be  made ;  ^nd  the  several  polar 
angles  occasioned  by  those  alterations,  and  contained  between  the  perpen- 
dicular F  S,  and  the  co-latitude  of  Port  Jackson  =  A  S,  will  be  obtained. 
— Th^s,  from  the  polar  angle  A  S  F  =  68?4'5C',  let  5?  be  continually  sub- 
tracted, and  the  results  will  be  FS  1  =  63?4C5r  FS  2  =  68?4^6r  ; 
FS  3  =  53?4'5'r,  &c.  ic,  the  last  remainder  being  3?4'5r  r:  the  polar 
angle  F  S  13.-^Now,  the  polar  angles  contained  between  the  perpendicular 
F  S,  and  the  co«latitude  of  Valparaiso .  ==  B  S,  are  to  be  determined  by  a 
contrary  process ;  and,  since  the  last  subtraction  in  the  triangle  F  S  A,  left 
the  remainder,  or  polar  angle  F  S  13  ^  3?4^5r,  which  is  l?55'55r,  less 
than  the  proposed  alteration  of  longitude ;  therefore,  the  first  polar  angle 
in  the  triangle  FBS^  must  be  l'?55:55'r  =  the  polar  angle  F  S  a ;  to 
which,  let  5?  be  continually  added,  as  far  as  the  measure  of  the  angle  FSB 
will  allow,  lindweshallhaveFSA  =  6?55^55r3  FSc=  ll?55^55r; 
F  S  d  =;  16?55  ^55T,  and  so  on  ;  as  expressed  in  the  first  column  of  the 
folk>wing  Table. 

v2 


Digitized  by 


Google 


292  NAVIGATION. 

To  compete  the  successive  Latitudes  at  which  the  Ship  should  arrive  : — 

Since  the  several  successive  polar  angles^  obtained  as  above^  evidently 
reduce  the  two  right  angled  spherical  triangles  AFS  and  BFS,  intoa 
series  of  right  angled  spherical  triangles,  to  each  of  which  the  perpendicu- 
lar F  S  is  jcommon  ;  therefore,  in  each  triangle  of  this  series  we  have  the 
perpendicular  and  the  angle  adjacent,  to  find  the  hypothenuse,  or  co-la- 
titude.— ^Thu8,in  the  right  angled  spherical  triangle  F  S  1,  right  angled  at 
F,  given  the  perpendicular  FS  =  29?5'5K,  and  the  polar  angle  FS  1  = 
.63?4^5^  ;  to  find  the  hypothenuse,  or  co-latitude  S  1 ; — In  the  right  an- 
gled spherical  triangle  FS  2,  given  the  perpendicular  FS  =  29?5'5K, 
and  the  polar  angle  F  S  2  =  58?4'5?  ;  to  find  the  hypothenuse,  xyf  co- 
latitude  S  2,  &c.  &c.  &c. 
•  Hence,  by  right  angled  spherical  trigonom.etry^  Problem  IV.,  page  188, 

To  find  the  Hypothenuse,  or  Co-latitude  S  1  :— 

As  the  perpendicular  F  S  =       29?  5  ^  5  K  Log.  co-tang*  =    10. 254507* 
Is  to  the  radius  =  .     .     .     .     90.   0.   0    Log.  sine  =     .    .10.  OOOQOO 
So  Hthe  i[>olar  angle  FS  1  s  63.   4.   5    Log.  co-sine      .      9.656033 

To  the  co-latitude       S  1  =:     50. 5 1 .  36    Log.  co-tangent  =  9. 9 10540 

First  latitude  =  •    •    39?  8^24^  S.  at  which  ship  should  arrive. 

To  find  the  Hypothenuse,  or  Co-la(itude  S  2  :— 

As  the  perpendicular  F  S  :=      29?  5  ^  5  K  Log.  co-tang.  =     10. 254507* 
Is  to  the  radius  =  .     ...    90.   0,   0    Log.  sine  =  .     .10.000000 
So  is  the  jpolar  angle  F  S  2  =  58.   4.   5    Log.  co-sine  =  .      9. 723383 

To  the  CQ-latitude  S  2  =         46. 27. 28    Log.  co-  taifg.  =       9. 977890 

Second  latitude  =       43?32^32?  S.  at  which  ship  should  arrive. 

Hence,  the  first  latitude  at  which  the  ship  should  arrive  is  39?8'  24r  S. ; 
a^d  the  second  latitude  43?32'32^:S.— And  since  it  is  the  latitude,  and 
not  its  complement  that  is  required ;  therefore,  if  the  log.  tangent  of  the 
sum  of  the  three  logs,  be  taken,  it  will  give  the  latitude  direct ;  and,  by  re- 
jecting the  radius  from  the  calculation,  the  work  will  be  considerably  faci- 
litated.-=-Proceeding  in  this  manner,  the  several  successive  latitudes  cor- 


•  The  loff.  co-taDg^ent  is  used,  so  as  to  save  the  trouble  of  finding  the  arithmetical  ( 
plemeut  of  the  log.  taageut.' 


Digitized  by 


Google 


GRBAT  CIRCU   8AfLIN6.  293 

responding  to  the  proposed  alterations  of  longitude  will  be  found  as  shown 
in  the  3d.  column  of  the  following  Table. 

Now,  let  the  several  successive  longitudes  be  arranged  (agreeably  to  the 
proposed  change,  and  to  the  measure  of  the  corresponding  polar  angles,) 
as  exhibited  in  the  2d  column  of  the  following  Table ;  and  find  the 
difference  between  every  two  adjacent  longitudes^  as  shown  in  the  4th 
column  of  that  Table.— 'Find  the  difference  between  every  two  adjacent 
latitudes,  and  place  those  differences  in  the  5th  column.— Find  the 
meridional  parts  corresponding  to  the  several  successive  latitudes^  which 
place  in  the  6th  Column ;  and  find  the  difference  between  every  two  adja- 
cent meridional  altitudes^  as  shown  in  the  7th  column.— Then,  find^  by 
Mercator's  sailing,  Problem  I.,  page  238,  the  respective  courses  and  dis- 
tances between  the  several  successive  latitudes  and  longitudes ;  and, 
let. the  courses  and  distances,  so  found,  be  arranged  in  regular  succession, 
as  exhibited  in  the  two  last  columns  of  the  Table.^Then,  will  thiff  Table 
be  duly  prepared  for  navigating  a  ship  on  the  arc  of  a  great  circle,  a^ee- 
ably  to  the  proposed  alterations  of  longitude. — ^And,  should  the  sum  of  the 
several  successive  differences  ef  loingitude,  contained  in  the  Table,  coin- 
cide with  the  whole  difference  of  longitude  between  the  two  given'  places ; 
—the  sum  of  .th^  several  successive  differences  of  latitude  be  found  to 
agree  with  the  whole  difference  of  latitude  comprehended  under  the  mean, 
or  highest  latitude,  and  its  corresponding  extremes ; — ^tfae  sum  of  the  se- 
veral meridional  differences  of  latitude  to  be  equal  to  the  whole  meri- 
dional difference  of  latitude  corresponding  to  the  mean,  or  highest  lati- 
tude, and  its  respective  extremes, — and  the  sum  of  the  several  successive 
distances  to  make  up  the  whole  spherical  distance  (or  nearly  so,)  be- 
tween the  two  given  places;  then,  those  several  concurring  equalities 
will  be  so  many  satisfactory  proofs  that  the  work  is  right. 

JVbite.*— In  the  spherical  track  laid  down  in  the  following  Table,  it  is  pre- 
sumed that  there  is  not  any  land  to  intercept  a  ship's  progress ;  but  since 
this  track  will  take  the  navigator  into  high  southern  latitudes,  it  will  be  in- 
dispensibty  necessary  to  keep  a  sharp  look-out  at  all  times^  particularly 
during  the  night,  so  as  to  guard  against  any  of  the  ice-bergs  that  may  be 
floating  to  the  northward  of  the  Antarctic  cirele  ;— though  if  the  track  be 
made  in  .the  months  of  November,  December,  January,  or  February,  there 
will  be  no  real  night  or  darkness  to  experience ;  for  during  these  months 
there  wiH'be  a  strong  twilight  between  the  latitudes  of  53,  and  61  degrees 
south ;  and  thus  the  navigating  at  night  will  be  attended  with  very  little 
n^ore  danger  than  that  by  day. 


Digitized  by 


Google 


^J! 


I 
I. 


s 

m 


1|3 


N 


1} 


i 


H 


■9  55 


3KSi!!3S£:S8!!3a3!!iSS^Si!>&S!3Sii^£SS38&S 


-^U9C^9»«A| 


3'§SSSSSSSS.SSS8SS;SSSSSSSSSSSSSS 


l^^;;ss:^:ss$s3$^S3^8^ssss:ss^!$;$*^$i 


s 


§ 


M 

s 


H  n  H  fl  n  R  fl  n  B  I  n  I  0  R  B  IB  B  B  B  I!  Q  I  n  B  I  n  I  B  .n 

GO  («  w  oa  on  en  o)  CO  w  cfi  (ft  eft  «n  w  «  w  CO  w  w  GO  com  CO  w  CO  en  Qo  » ao  «) 


Digitized  by 


Google 


GRBAT  ClRCtV   SAILING.  295 

Nov^,  the  sum  of  the  Sereral  successive  differences  of  longitude,  viz.  8212 
miles^  coincides  exactly  with  the  whole  difference  of  longitude  between 
the  two  given  places ;  the  sum  of  the  successive  differences  of  latitude  = 
3295«  30  miles,,  agrees  with  the  whole  difference  of  latitude  comprehended 
under  the  highest  latitudeat  which  the  ship  should  arrive,  and  the  latitudes 
of  the  two  given  places;  viz.  33?52^07  S;  60?  54' 9'/ S,  and  33?  T.  Or  S: 
—and,  the  sum  of  the  sever^  meridional  differences  of  latitude  »  501 1, 80 
miles,  makes  up  the  whole  difference  of  latitude  corresponding  to  the  higft- 
est  latitude  and  the  latitudes  of  its  respective  extremes :— these  several 
concurrences  or  agreements,  form,  therefore,  the  most  satisfact<^  and  in- 
disputable proofs  that  the  work  has  been  properly  conducted. 

The  sum  of  the  several  distances,  measured' on  the  respective  rhumb-lines 
intercepted  between  the  successive  longitudes  and  latitudes,  as  given  in  the 
last  column  of  the  Table,  is  6108.  73  miles ; — but  the  true  spherical  dis- 
tance dn  the  arc  of  a  great  circle  is  6107-  87  miles ;  the  difference,  there- 
fore, is  only  0. 86,  or  a  little  more  than  three  fourths  of  a  mile ;  which  is  a 
very  close  approximation  in  the  measure  of  so  great  an  are. 

The  distance  by  Mercator's  sailing  is  6853. 16  miles ;  which  is  745. 29, 
or  about  745^  miles  more  than  by  great  circle  sailing.— HencCj  it  is  evi- 
dent that  the  shortest  and  most  direct  route  from  Po^f  Jackson  to  Valpa- 
raiso is  by  the  latitude  of  60?  54 '9^  S ;  and  that  the  ship  must  make,  suc- 
cessively, the  several  longitudes  and  latitudes  contained  in  the  2nd  and  Srd 
columns  of  the  Table,  .in  the  same  manner  precisely,  as'  if.  they  were  so 
many  ports  or  places  of  rendezvous,  at  which  she  was  directed  to  touch. 

The  first  course,  therefore,  from  Port  Jackson  to  Valparaiso,  is  S*  37^  1 8( 
E.  distance  398  miles;  which  will  bring  the  ship  to  longitude  156?16C0r 
K  and  latitude  39? 8 ^24?  S  ;— the  second  course  is  S.  40?26^  E.  distance 
347  miles  ;  which  brings  the  ship  to  longitude  161?  16  ^OfEk  and  latitude 
43?32'32r  S ;— the  third  course  is  S.  43?53^  E.  distance  304  miles,  which 
brings  the  ship  to  166?  16^0^  E.  and  latitude  47?U  'S6f  8. 

Whence  it  is  evident  that  Captain  Gambier  saved  a  distance  of  745^ 
miles  in  that  judicious  and  well-planned  route  :  And  this  saving  of  distance 
should  be  an  object  of  the  highest  consideration  to  every  captain  who  wishea 
to  recruit  the  strength  and  spirits  of  his  ship's  company  by  a  generous  sup- 
ply of  fresh  provisions  ufter  a  fatiguing  and  tedious  voyage ;  the  measure 
of  which  falls  very  little  short  of  being  equal  to  one  fourth  of  the  earth's 
circumference  as  taken  under  the  equator,  or  to  the  one  third  of  that  cir- 
cumfenence  if  taken  under  the  given  parallel  of  latitude. 


Digitized  by 


Google 


!296 


NAUTICAL  ASTRONOMY. 


SOLUTION  OF  PROBLEMS  IN  NAUTICAL  ASTRONOMY. 

Nautical  Asteonomy  is  the  method  of  findings  by  celestial  t>bservation^ 
the  latittide  and  loiigitude  of  a  ship  at  sea ;  the  variation  of  the  conipass  ; 
the  apparent  time  at  ship ;  the  fdtijtudes  of  the  heavenly  bodies,  &c.  &c.  &c. 
—Or,  it  is  that  branch  of  mathematical  astronomy  which  shows  how  to 
solve  all  the  important  Problems  in  navigation  by  means  of  spherical  oper- 
ations,, when  the  altitudes^  or  distances  of  the  celestial  objects  are  under 
consideration. 


Introductory  Problems  to  the  Science  of  Nautical  Astronomy. 
Problbm  I.  ' 

To  convert  Longitude  or  Parts  of  the  Equator  into  Time. 
Rule.     .        . 
Multiply  the  given  degrees  by  4,  and  the  product  will  be  the  correspond- 
ing time  :— -observing  that  seconds  multiplied  by  4  produce  thirds ;  mi- 
nutes multiplied  by  4  produce  seconds,  and  degrees  multiplied  by  4  pro- 
duce minutes,  which,  divided  by  60,  give  hours,  &c. 


Exampfe  !• 

Required  the  time  correspond- 
ing to  12?40U5r  ? 

Given  degrees  =  12?40M5r 
Multiply  by  •    •    •    .      4 

Corresp.time    =       0*50:43!0f 


Example  2. 

Required  the  time  corresponds 
ingto76?20^30r? 

Given  degrees  =   76?20C30r 
Multiply  by      .    .     .     .     4 

Corresp.  time    t=     5*5?22;0f 


Pro]$L£m  II. 
To  conoeri  Time  into  Longitude,  or  Parts  of  the  Equator. 

Rule. 
Reduce  the  hours  to  minutes,  to  which  add  the  odd  minutes,  if  any; 
then,  the  minutes  divided  by  4  give  <}egrees ;  the  seconds  divided  by  4  give 
minutes,  and  the  thirds  divided  by  4  gjve  seconds. 


Example  I. 
Required  the    degreea  corres- 
ponding to  0*47"36!? 

Given  time  =    .     0*47"36! 


Divide  by  . 
Corresp.  deg.  = 


4)  47^36! 

Ilf54:  or 


Example  2. 
Required  the    degrees    corres- 
ponding to  9^25^37!  ? 

Given  times?  .     .  9*25^37! 


Divide  by      .       4)  565r37! 
Corresp.  deg,  =      141?24:  ISr 


Digitized  by 


Google 


INTRODUCTORY  PROBLBlfS. 


297 


Note,f~^pie  two  preceding  Problems  are  readily  solved  by  means  of 
Table  I^ — see  explanation^  pages  1  and  2. 


PaoliLEM  IIT. 

Qioen  the  Time  under  any  known  Meridiany  to  find  ike  corresponding 
T%7ne  at  Greinwichn 

RVLB. 

Let  the  given  time  be  reckoned  from  the  preceding  noon^  to  which  apply 
the  longitude  of  the  place  in  rfme  (reduced  by  Problem  I.,  as  above,}  by 
addition  if  it  be  west,  or  subtraction  if  east;  and  the  sum,  or  difference  will 
be  the  corresponding  time  at  Greenwich. 


Example  1,  . 
Required  the  tiine  at  (}reenwich, 
when  it  is  4M0ri3!  at  a  ship  in 
Iongitudi5  80?53:i5r  W.? 

Time  at  ship  =    .     .    4*40rre: 
Long.80?53a5rW.    , 

in  times  •    .    \r  +  5.23.33 


Corresp.  time  at  Green- 
wich =       ...     10*  3?46! 


Example  2. 
Required  the  time  at  Greenwich, 
when  it  is  20*  1 1T41 !  at  a  ship  in 
longitude  98?  1 4 USrE? 

Time  at  ships:  .     .    2Q*llr4i! 
Long.  98n4U5rE. 

in  time  =s    .    .     -  6.32.59 


Corresp.  time  at  Green- 
wich =    .    .    .    .  13?38r42! 


Problem  IV.  . 

Given  the  Time  at  Greenvneh,  to  find  the  corresponding  Time  under  a 

known  Meridian, 

Rule. 
Let  the  given  time  be  reckoned  from  the  preceding  noon^  to  which  apply 
the  longitude  of  the  place  in  time  (reduced  by  Problem  I.  as  above,}  by  ad- 
dition if  it  be  east,  or  subtraction  if  west;  and  the  sum,  or  difference  will 
be  the  corresponding  time  under  the  given  meridian. 

'Example  1. 
When  may  the  emersion  of  the  first  satellite  of  Jupiter  be  obser\'ed  at 
Trincomalee,  in  longitude  81  ?22'  E.,  wifich,  by  the  Nautical  Almanac,  hap- 
pens at  Greenwich,  March  4th,  1825,  at  9*9T28!  ? 

Apparent  time  of  emersion  at  Greenwich  =    .    •    .    9t  9f28* 
Longitude  of  Trincomalee  8 1  ?  22  ^  E.,  in  time  =:     .    5 .  25 .  28 

Apparent  time  of  emersion  at  Trincomalee  =    •    •    14*34T56! 

Digitized  by  VjOOQ IC 


2d8  '  NAUTICAL  ASTRONOMY* 

Example  2. 

When  may  the  immersion  of  the  first  satellite  of  Jupiter  be  observed  at 
Port  Royal,  Jam^ca,  in  longitude  76?52'30r  W.,  which,  by  the  Nautical 
Almanac,  happens  at  Greenwich  Nov.  1st.  1825,  at  18^  17 "'45 '  ? 

Apparent  time  of  immersion  at  Greenwich  =   •     .       18^  17*45! 
Longitude  of  Port  Royal  76?52:30r  W.,  in  time  ==      5.   7.  SO 

Apparent  time  of  immersion  at  Port  Royal  r=     •    •     13*  10?  15 ' 

^     ■   I        I  ■■  ■'         «  ■         ■ ■ ■■■■  ■       ■■■■   !.■  ■'     »  ■    ■  ■    ■     ■     ■       ■       ■«.        III!        i»—    „.   m. ■     ,■ 

Pboblem  V. 
To  reduce  the  Sun^s  Longitude^  Bight  Asc^iswUf  and  DecUnation;  and^ 
alsoy  the  Equation  qf  Time,  €is  given  in  the  Nautical  Almanac,  to  any 
other  Meridian,'  and  to  any  thne  under  that  Meridian; 

RUJLB. 

Let  the  given  apparent  time  at  ship,  or  pl^ce,  be  always  reckoned  from 
the  preceding  noon;  to  which  apply  the  longitude  in  time  (reduced  by  Pro- 
blem I.,  page  296,)  by  addition  if  it  be  West,  or  subtraction  if  ea^t,  and 
the  sum  or  difference  will  be  the  corresponding  time  at  Greenwich. 

Take,  from  page  U.  of  the  month  in  the  Nautical  Almanac,  the  sun's 
longitude,  right  ascension  and  declination,  or  the  equation  of  time,  as  the 
case  may  be,  for  the  noons  immediately  preceding  and  following  the 
Greenwich  time,  and  find  their  difference;  then, 

To  the  proportional  log.  of  this  difference,  add  the  proportional  log.  of 
the  Greenwich  time  (reckoning  the  hours  as  minutes,  and  the  minutes  as 
seconds),  and  the  constant  log.  9. 1249;  *  the  sum  of  these  three  logs, 
rejecting  10  from  the  index,  will  be  the  proportional  log.  of  a  correction 
which  is  always  to  be  added  to  the  sun's  longitude,  or  right  ascension,  at 
the  noon  preceding  the  Greenwich  time  ;  but  to  be  applied  by  addition  or 
subtraction  to  the  sun's  declination,  or  the  equation  of  time  at  that  noon^ 
according  as  these  elements  may  be  increasing  or  decreasing. 

Bemark. — Since  the  daily  difference  of  the  equation  of  time  is  eiq)ressed^ 
in  the  Nautical  Almanac,  in  seconds  and.  tenths  of  a  second ;  if,,  there- 
fore, these  tenths  be  multiplied  by  6  they  will  be  reduced  to  thirds  :  hence, 
the  daily  difference  will  be  obtained  in  seconds  and  thirds.— Now,  if  those 
seconds  and  thirds  be  esteemed  as  minutes  and  seconds,  ti^  operation  of 
reducing  the  equation  of  time  wijU  become  as.  simple  as  that  of  the  sun^s 
declination ; — observing,  however,  that  the  minutes  and  seconds.  Corres- 
ponding to  the  sum.  of  the  three  logs.,  are  to  be  considered  as  seconds  and 
thirds. 

*  This  is  the  arithmetical  complement  of  the  proportional  log.  of  24  hours  esteemed  as 
winuteif 


Digitized  by 


Google 


304  .    NAUTICAL  ASTRONOMY. 


To^find  the  Moon's  Declination  :-— 

Diflference  in  12  hours  =      .     .    •    .     2?47'57^  .  Prop.  log.  =  .0301 

Greenwich  time  = 7*20r53'     Prop.  log.=  1.3891 

Constant  log.  =      ....'..•......•,..    8.8239 

Correction  of  moon's  declination  =      +  1  ?42C5 1  r     Prop.  Iog.=  0. 2431 
Moon's  declin.  at  noon  March  6th  .=       7>  58.   6      south. 


Moon's  declination^  as  required  ss  •    •    9?40^57^    soutli. 


To  find  the  Moon's  Semi-diameter : — 

DiflF.in  12  hours  =  ...    4r  ......    .     Prop.  log.. =:  3.4314 

Greenwich  time  =     ..    .     7'20r53!     ....     Prop.  log.  =   1.3891 

Constant  log.  =  .   .    .    .    .....    .    .    .    ..-.,.    8.8239 


Correction  of  the  moon's  semi-diameter  =s     —21     Prop.  log.  =:  3. 6444 
Moon's  Semi-diam.  at  noon  March  6th  =s   16^38^ 


Moon's  senii-diameter^  as  required  =    .     16C36T 


To  find  the  Moon's  Horizontal  Parallax : — 

Difference  in  12  hours  = 16^     Prop.  log.  s  2. 8293 

Greenwich  time  = 7*20r53!     Prop.  log.  =  1.3891 

Constant  log.  = .    8.8239 

Corr.  of  the  moon's  horizontal  parallax  =         10^     Prop,  log,  ^  3. 0423 
Moon's  horiz.  par.  at  noon,  March  6th       61^   2T 

Moon's  horizontal  parallax,  as  required  =  60^52^ 

Reniark.'^When  much  accuracy  is  required,  the  proportional  part  o( 
the  moon's  motion  in  12  hours,  found  as  above,  must  be  corrected  by  the 
equation  of  second  difference  contained  in  Table  ^VII.,  as  explained  in 
pages  33,  34,  and  35.  And,  in  all  cases,  the  moon's  semi-diameter,  so 
found,  must  be. increased  by  the  augmentation  given  in  Table  IV.,  as  ex- 
plained between  pages  8  and  11. 


Digitized  by 


Google 


INTRODUCTORY  I'ROBisMS,  305 

Example  2. 

Required  the  moon's  longitude,-  latitude,  right  ascensioh,  declination^ 
semi-diameter,  and  horizontal  parallax,  March  26th,  1825,  at  l*30?47!, 
in  longitude  94?15:30r  east;  the  apparent  altitude  of  that  object  being 
.24?? 

Apparent  time  at  ship  or  place  =     .........     1*30"47' 

Longitude  of  the  ship  or  place  =  94?  15  C30r  E.,  in  time  =s  .    6. 17.'  2 

Greenwich  time  past  midnight,  March  25tb,  1825  =  ..    .    •    7t  13?45 ! 

To  find  the  Moon's  Longitude  :— 

Diff.  in  12  hours  =  6?22:57^  t-  3  =     2?7^39r    Prop,  log:  =     .  1493 

Greenwich  time  =  7*  13T45!  • Prop.  log.  =  L  3962 

Constant  log.  = •    .    •    8. 8239 

One-third  of  the  proportional  part  s     1?  16C53r     Prop,  log,  a  0.3694 

Multiply  by  3 

Prop,  part  of  J 's  motiofi  in  long.    =     3?50'39? 
Equation  from  Table  XVII.  =     .    .  -  33 

Proportional  part  corrected  =       .    .    3?50'  6T 
J 's  long,  at  midnt.,  March  25th  =  2!  15?31  C56r 

Moon's  true  longHude  s    .    .    .    2!19?22:  2? 

To  find  the  Moon's  Latitude  :— 

Difference  in  12  hours  =    *..    .    .    .    33137^    Prop.  log.  =     .7287 

Greenwichtime=:7M3?45!  .    ......      Prop.  log.  =   1.3962 

Constant  log.  = •    *    .    8.8239 

Proportional  part  of  D  's  latitude   =     -  4i0'  ISr    Prop.  log.  «  0. 9488 
Equation  from  Table  XVII.  =s    .     .  •  —  6 

Proportional  part  corrected  sa     .    .     —  20'  9^ 

J 's  lat.  at  midnight,  March  25th  =       0?*46^36r  north. 

Moon's  true  latitude  =   *    •    .    .    .    0?26'.27r  north. 

X 

Digitized'by  LjOOQ  IC 


806  MAOTICAL  A8TB0MOMT. 

To  find  the  Moon's  Right  AAcension  :— 

Diff.ml2houn«6?59n:.  +  3=    2?19'40J?    Prop.  log. «    .U02 

Greenwich  time- 7*  13T45!       Prop.log.=  1.3962 

Conttunt log.  =      .    ,    . ' ^'^239 

One-third  of  the  proportional  part  =    1?24<  81      Prop.  log.  =  0. 3303 
Multiply  by  8 

Prop,  part  of  J 's  motion  in  right  asc^*?  12'24? 
Equation  from  Table  XVH.  =    .    .        -  36       . 

Proportional  part  corrected  =    .    •    4?  11 '48^ 
])'8rightasc.atmidnt./March25thas74. 1U56 

Moon's  true  right  ascension  =    .      78^  23 '  44^ 

To  find  the  Moon's  Declination  :— 

Differencein  13  hours  « I'.Or    Prop.  log.  »    2.2553 

Greenwich  time  =  7M3r45! Prop.  log.  =   .1.3962 

Constant  log.  =    . 8.8239 

Proportional  part  of  J  'a  declination  =  -  0' 36?    Prop,  log,  =     2. 4754 
Equation  from  Table  XVII.  =s     .     .    .    -  10 

Proportional  part  corrected  tt     •    .  .  —  0'26? 

))  's  dec.  at  midnt.,  March  25tii  =     23^26^53?  north. 


'    I 


Moon's  true  declination  •=       .    .    23^26:27?  north. 

To  ifind  the  Moon*s  Semi-diameter  :— 

Difference  in  13  hours  =  .    ...    .    ,    .    .6?    Prop,  log,  fai  3. 2553 

Greenwich  time ->  7M3W5!     ....     ...  Plop.  log.  a  1.3962 

Constant  log.  =. .    .    :    8.8239 

Pfopoftlowil  part  of  )  's  semUdfemeter        +  4r    Prop.  log.  »  3.4754 
D  's  semi-diam.  at  midnt.,  March  25^th.  =  151 19^ 

Moon's  apparent  semi-diameter  =B     ..    ..15128? 
Augment,  from  Tab*  IV.,  for  alt.  24  ?  p.  6 

Moon's  true  semi-idiBnietec  =::      «    ..   .1    15129^ 

Digitized  by  VjOOQ IC 


IKTHODUCTORT  PROBLXMS.  ^307 

To  find  the  Moon's  Horisontol  Ptoalliix:~ 

Diflference  ih  12  hours  =  22r Prop.  log.  =  2.6910 

Greenwich  time  c*  7M3?45;  . .  .    ,    .    .    .     .    Prop.  log.  ^  ^3962 
Constant  log.  =    ......•.•.'.•,,.,,      8.8239 


Proportional  part  of  'j  's  hor.  parallax  =     +   13f    Prop.  lo^.  s=  2. 91 U 
>  's  hor.  par.  at  midnight,  March  25th  ps  56*.  ISf    ' 


Moon's  true  horizontal  parallax  =     •    •  56' 26^ 

Note. — ^The  examples  to  the  foregoing  Problem  may  \}e  very  correctly 
solved  by  means  of  Table  XVL^JSee  explanation,  page  8O4 


Problbm  VIL 

To  reduce  the  lUghi  AscenAm,  and  Declination  of  a  Planet,  as  given  ifi 
the  Nautical  Jlmandc,  to  any  given  ^me  under  a  known  Meridian. 

Ruu« 

Let  the  apparent  time  at  the  ship  or  place  be  reckoned  from  the  preced* 
ing  Jloon,  to  which  apply  the  longitude  In  time,  (reduced  by  Problem  I., 
page  296,)  by  addition  if  it  be  west,  or  subtraction  if  east  j  and  the  $\m 
or  difference  will  be  the  corresponding  time  at  Greenwich. 

From  page  IV.  of  the  month  in  the.  Nautical  Almanac,  take  out  the 
planet's  right  ascension  and  declination  for  the  nearest  days  preoeding  and 
following  the  Greenwich  time,  and  find  the  difference;  find,  also,  the 
diflference  between  the  Greenwich  time  and  the  nearest  preceding  day; 
then, 

To  the  proportional  logarithm  of  this  diflference,  esteemed  as  minutes 
and.  seconds,  add  the  proportional  logarithm  of  the  diflference  of  right 
ascension,  or  declination^  and  the  constant  logarithm  9.9031*;  the  sum 
of  these  three  logarithms,  rejecting  10  from  the  index,  will  be  the  propor* 
tional  logarithm  of  a  correction ;  which  being  applied,  by  addition  or  subr 
traction,  to  the  right  ascension  or  declination  (on  the  nearest  day  preceding 
the  Greenwich  time),  according,  as  it  may  be  increaring  or  decreasing,  the 
sum  or  diflference  will  be  the  correct  right  ascension  or  declination. 

Example  I.  - 

Required  the  right  ascension  and  declination  of  the  planet  Mars,  Marfth 
16th,  1825,  at  4*40r  apparent  time,  in  longitude  68?12f  west  of  the 

meridian  of  Greenwich  ?        " 

--—■---■»-  I  -  I     ,  I        .  -  .  ■■    ■ 

«  Tbit  to  the  arithmetical  complement  of  the  proportkmal  logarithm  of  144  hours  »  6 
daytff  fttesmed  M  »tfiirlr« ;  and,  hie Ace>  taken  M  S  hovrt  sad  24 

X   2 


Digitized  by 


Google 


308  NAUTICAL  ASTRONOMY. 

Apparent  time  at  ship  or  place  =s     .■   .     .     March,  16  days,   4!40T  0! 
Longitude  of -ship  or  place  =  68^  12  C  W,,  in  time  =     .     .     .  4. 32. 48 

Greenwich  time  =    .    .    •..    .    .     .    ...    .    16days,  9M2T48! 

To  find  the  Right  Ascension : — 

R.  A.  of  Mars,  March.l3th=0*4ir  13f  0*  Or  0! 

Ditto  I9th=0.38Gr.time=16.   9.12.48 


Differences:      ....    0*17^    Diff.  =  3f  9M2M8!=  81M2r48! 

Piff.  oftime  =  8lM2r48r,  or  l*21?12M8f  Prop.  log.  =  .3456 
Difference  of  right  ascension  5=  0M7*  .  .  .  Prop.  log.  =  1.0248 
Constant  log.  = .    9.9031 

Correction  of  right  ascension  =  .  .  +  9T35!  Prop.  log.  =  1.2735 
Planet's  right  ascension,  March  I3th  =  OM 1  ?  0 ! 


Planet's  right  ascension,  as  required  «  0*50T35 ! 

To  find  the  Declination : — 

Dec.ofMars, March  13th=3?53^N.  13f  0*  Or  0! 

Ditto  19th=5.43N.Gr.time=16.    9.12.48 


Dlfferehc€=     \    .    .      1950:  3f  9M2r48'.— 81M2r48! 

Differenceof.timfe==81*12r48',or  H2iri2!48f     Prop.  log.  =     .3456 

Difference  of  declination  =  1?50' Prop.  log.  =     .2139 

Constant  log.  =     /........ 9.9031 

Correction  of  declinations     .     .     +  1^  2'  2r     Prop.  log.  =  0.4626 
Planet's  declinatbn,  March  13th   =       3.53.   0  north. 


Planet's  declination,  as  required  ^     •   4?55'   2?  north. 

Example  2* 

Required  the  right  ascension  and  declination  of  the  planet  Mars,  Sept. 
23d,  1825,  at  l*23ri9!,  apparent  time,  in  longitude  I00?40:30r  east  of 
the  meridian  of  Greenwich  ? 

Apparent  time  at  ship  or  place  =       ...    Sept.  23  days,     1  *23rl9! 
Lonptude  of  ship  or  place  =  100?40^30r  E.,  in  time  =       .    6. 42. 42 

Greenwich  time  (paist  noon  of  the  22d  Sept.)  =      22  days,  18U0?37' 

Digitized  by  VjOOQ IC 


INTRODUCTORY  PROBLEMS.  309 

To  find  the  Right  Ascension : — 

R.A.ofMar8,Sept.l9th=9*37r  19f  0*  Or  Of 

Ditto  25th=9. 52  Gr.  time  22, 18. 40. 37 


Difference  =       .     .     .    0*15r      Difr.=:3f  I8t40?37!  =   90*40?37! 

Difference  of  time  =  90*40?37!,  or  It30r40!37f  Prop.log.  =  .2977 
Difference  of  right  ascension  =  0*15?  .  .  .  Prop.log.  =  1.0792 
Constant  log.  = ^ 9.9031 


Correction  of  right  ascension    s     .     +  9?27!.        Prop.  log.  =  1.2800 
Planet's  right  ascension,  Sept.  19th  i=  9?37*r  0: 

Planet's  right  ascension,  as  required5=9t46r.27! 

To  find  the  Declination  :— 
Dec.ofMarsVSept.l9th=15'?30'N.  19f  0!  0?  0! 

Ditto  •     25th=tl4. 18  N.  Gr.  time  22. 18. 40'.  37 


Differences       .    .    .     l9l2^        Diff.  =   3fi8M0?37'=90!40r37! 

Difference  of  time  =  90M0?37 ',  or  1  *30?40!37  f     Prop.  log.  =     .  2977 

Difference  of  declination  =  1?12' *  Prop.  I05.  =3     .3979 

Constant  log.  =      •    .    •    • 1     •    .    .    •    9.9031 

Correction  of  declination  =        .  —  45'2K  Prop.log.  s  0.5987 

Planet's  declination,  Sept.  19th  =  15. 30.   0  north. 

Planet's  declination,  as  required.=14?44'39T  north. 


Problem  VIII. 

To  compute  tlie  Apparent  Time  of  the  Moon's  Transit  over  the  Meridian 

of  Greenwich. 

Since  the  moon's  transit  over  the  meridian  of  Greenwich  is  only  given 
to  the  nearest  minute  in  the  Nautical  Almanac ;  and,  since  it  is  absolutely 
necessary,  on  many  astronomical  occasions,  to  have  it  more  strictly  deter- 
mined :  the  following  rule  is,  therefore,  given,  by  which  the  apparent  time 
of  the  moon's  transit  over  the  meridian,  of  Greenwich  may  be  obtained  true 
to  the  decimal  part  of  a  second. 

Rule. 
From  the  moon's  right  ascension  at  noon  of  the  given  day  (converted 
into  tipie,  and  increased  by  24  hours  if  necessary,)  subt^ct  the  sun's  xi^t 


Digitized  by 


Google 


310  KAi;tIOAL  AttRONOMY. 

ascension  at  that  noon,  and  the  remainder  will  be *the  approximate  time  of 
the  moon's  transit  over  the  meridian  of  Greenwich. 

Find  the  excess  of  the  moon's  motion  in  right  ascension  over  the  son'a 
in  12  hours;  then  say,  as  12  hours,  dimitiished  by  tliis  excess, 'is  to  12 
hourSj  so  is  the  apptoxiitiate  time  of  transit  to  the  apparent  time  of  transit. 

Notfi.'^U  the  three  tetms  be  reduced  to  seconds,  the  operation  may  be 
teadily  performed  by  logarithms. 


Example  1. 

Required  the  apparent  time  of  the  moon  s  transit  over  the  meridian  of 
Greenwich,  March  26th,  1825  ? 


Moon's  R.A.  at  noon  of  given  day  =  81?10^57^  in  time  =  5*24r43\S 
Sun's  right  ascension  at  that  noon  = .^    0.  20. 24  .  (F 

ApproK..time  of  the  moon's  tr.  over  the  merid*.  of  Greenw.  =s  5 1  4T19'  •  8 

*  Son's  right  ascension  at  noon^  March  26th,  s    0^20^24! 
Son's  ditto  27th    s    0.24.   9 

Sun's  motion  in  24  hours  =s Ot  drSS! 

Sun's  motion  in  12  houi^  =     .    .    .    •    r    .    0*  1T49! 

Moon's R. A.  at  noon, March  26th,  =  81?10:57r,  in  time  =  5?24r43'.  8 
Moon's  ditto,  at  midnt,  March  26th,=88?14^  15f,  in  time=  5. 52. 5?  .0 

Moon's  motion  in  12  hours  == •  0*28T1'3'.2 

Siiti'd  nidtion  in  1^  hoUr3  3     •    .    •    •    .    ;    .    •    «    •    0.    1,49  .0 

Excess  of  the  moon's  motion  over  t)ie  sun's  in  12  hours  s        26?24\2 

As  12?  -*  26r24'  i  2 .«  1 1  *33?35  • .  8, 

in  seconds  s  «  «  ,  «  ,  .  41615.8  Log.  ar.c6.mp.s'5. 380742 
Is toa2  hours,  in  seconds  s.  .  .  43200.  Log.  s  *  ..  4.635484 
So  is  the  approximate  time  of  transit 

=  5*4?19'.8,  in  seconds  =    .     18259.8  Log.  =  .    .    4.261496 

To  the  ^parent  time  of  the  moon's 
t»uunt?s5M5:54\99iiiaecs«  a;  18954»B  Log^  =  ,    »    4.277722 


Digitized  by 


Google 


l3fTft02>UCrORy  PROBUMS.  8U 


Example  2. 

Required  the  apparent  time  of  the  moon's  transit  over  the  meridian  «f 
Greenwieh,  April  10th,  1825  ? 

Moon's  R,  A.  at  noon  of  given  day=:i94?59'lK^intime=l9!39r56\7 
Sun's  right  ascension  at  that  noon  = 1.15,1.6 


Approx*  time  of  the  moon's  tr.  over  the  merid.  of  Oreenw*  s=  1 8. 24  •  55  4 1 

Sun's  right  ascension  at  noon,  April  10th,  9    1M5?  l'«6    . 
Sun's  ditto  11th,  =    L  18.41  .6 


Sun's  motion  in  24  houra  s Ot  3r40',0 


Sun's  moti(Hi  in  12  hours  =      • .  •    •    •    .    Ot  i?50'.0 

Moon's  R.  A.  at  midnt,  April  10th  a  son  15  C6'r,  in  time  a  20 1  57  0\4 
Moon's  ditto  at  noon,  April  1 1th  =  307. 20. 12,  in  time  =  20. 29. 20  . 8 

Moon's  motion  m  12 hours  Si      .••.«..•.    0^24?20^4 
Sun's  motion  in  12  hours  = 0«   1.60  .0 


Excess  of  the  moon's  motion  over  die  son's  in  12  houn  ==      0. 22. 30  •  4 

As  12? -22r30\  4=1 1  *37r29\  6, 

in  seconds  ss 41849.6  liog.  ar.comp.sS. 378909 

Is  to  12  hours,  in  seconds  rss  .  .  4^200.  Log.  s  «  .  4.B35484 
So  is  the  approximate  time  of  transit 

ss  18^24755  M,  in  seconds  tt    6G295.1Log.«     •    •    4.821481 

To  the  apparent  time  of  the  moon's 
transits^  1 9*0734 •. 8, in  sees:  =s 68434.3  Log.  ss     .    .    4.835274 

• 
Note.^ln  strictness  the  apparent  time  of  transit,  thus  found,  should  be 
corrected  by  the  equation  of  second  difference  answering  thereto,  and  the 
mean  Second  difference  of  the  moon's  place  in  right  ascension ;  but,  at  sea, 
this  correction  may  safely  be  dispensed  with. 


Digitized  by 


•Google 


312  NAUTICAL  ASTRONOMT. 


Problem  IX. 

Given  the  Jpparent  Time  of  the  Mm! 9  Traimi  over  the  Meridian  of 
Greenwichf  to  find  the  Jpparent  Time  of  Dransit  over  any  olher 
Meridian. 

RULB. 

Takej  from  page  VI.  of  the  month  in  the  Nautical  Almanac,  the  moon's 
transit  over  the  meridian  of  Greenwich  on  the  given  day,  and  also  on  the 
day  following  if  the  longitude  be  west^  but  on  the  day  preceding  it  if  it  be 
east,  and  find  the  difference ;  which  difference  will  be  the  daily  retardation 
of  transit :  then  say. 

As  the  sum  of  24  hours  and  the  daily  retardation  of  the  moon's  transit, 
thus  found,  is  to  the  daily  retardation  of  transit ;  so  is  the  longitude  of  the 
given  meridian,  in  time,  to  a  correction,  which,  being  applied  by  addition 
to  the  apparent  time  of  transit  oVer  the  meridian  of  Greenwich  on  the 
given  day,  if  the  longitude  bq  west,  but  by  subtraction  if  east  5  the  Stum,  or 
difference,  Will  be  the  apparent  time  of  transit  oVer  the  given  meridian. 

JVbte.— This  proportion  may  be  readily  performed  by  proportional  loga- 
rithms, esteeming  the  hours  and  minutes  in  the  J&*«f  and  third  terms  as 
minutes  and  seconds. 

Example  1. 

Required  the  apparent  time  of  the  moon's  transit  over  a '  meridian 
94?30^30r  west  of  Greenwich,  March  26th,  1825,  the  computed  apparent 
time  of  transit  at  Greenwich  being  5 !  15  ?54 '.  9  ? 

Moon's  transit  over  the  mend,  of  Greenwich  on  the  given  day  =:     5t  16T 
Moon's  ditto  on  the  day ybOotnn^^:  6. 11 


Daily  retardation  of  moon's  transit  ==     .    • 0^557 

As  24  hours+0*55r  (daily  retard.)=24*5.5?  Prop. log. ar. comp.±9. 1412 
Is  to  the  daily  retard,  of  transit  ==  0.55  Prop.  log.  =  .  .  0.5149 
So  is  the  Ion.  94?30:30^W;,  in  tiiAe^e*  18r2!     Prop.  log.  =       1. 4559 

■  *  f     '     ' 
To  the  correction  of  retardation  =     .     +  13T54' .  5  Prop.  log.=:  1. 1 120 
Computed  apparent  time  of  moon^s 
transit  over  the  mend,  of  Greenwich=:5  *  1 5 ?54 ' .  9 


App.  time  of  ])  *s  tn  ovet  the  given  mer.=;5  *  29T49 ' .  4 

/Google 


Digitized  by ' 


IMHODUCTOEY  PROBLBMS,  3  IS 

Example  2. 

Required  the  apparent  time  of  the  moon's  transit  over  a  meridian 
105?10M5r  east  of  Greenwich,  April  10th,  1825,  the  computed  apparent 
time  of  transit  at  Greenwich  being  19t 0T34 '  •  3  ? 

Moon's  transit  over  the  merid.  of  Greenwich  on  the  g^ven  day  =:  19 1  1? 
Moon's  ditto  on  the  day  preceding:^,  1 8. 13 

Daily  retardation  of  moon's  transit  =:     •    •    • 0!48? 

As  24  hours+0*48r  (daily  retard.) =24? 48?  Prop.  log.  ar.comp.=9. 1392 
Is  to  the  daily  variation  of  transit  =:  0. 48  Prop,  log.  =:  •  •  0. 5740 
SoisthelongJ05?  10(45 ^E., in  timez=7*  0r43!     Prop.  log.  =     1.4094 

To  the  correction  of  r^ardation  =z  13'?34'.4  Prop, log.  =  lil226 
Computed  apparent  time  of  moon's  tr. 

over  the  merid.  of  Greenwich  =   19 1  0?34 ' .  3 


App.  time  of  ])  's  tr.  over  given  men  =  18!  46T59  * .  9 

Note. — ^Thc  above  problem  may  be  readily  solved  by  means  of  Table' 
XXXVIII.— See  explanation,  page  100, 


Problem  X. 

To  compute  the  Apparent  Time  of  a  Ptanet'e  JVanrit  over  the  Meridian 

of  Greenvokh. 

Rule. 

Find  the  planet's  right  asoension  at  noon  of  the  given  day,  by  Problem 
VII.,  page  307 ;  from  which  (increased  by  24  hours  if  necessary),  subtract 
the  sun's,  right  ascension  at  that  noon,  and  the  remainder  will  be  the 
approximate  time  of  the  planet's  transit  over  the  meridian  of  Greenwich. 

Take  the  difference  of  the  sun's  and  the  planet's  daily  variations,  or 
motions,  in  right  ascension,  if  the  planet^s  motion  be  progressive,  but  the 
sum  if  it  be  retrograde*  :  then  say. 

As  24  hoursj  diminished  or  augmented  by  this  difference  or  sum  (accord- 
ing as  the  planet's  diurnal  motion  in  right  ascension  is  greater  or  less  than 


*  When  the  daily  variation  of  the  planet's  rt^bt  ascension  Is  greater  than  that  of  tl^e 
sun*8y  its  motion  i$  jnvgreitwe ;  bat  whenless;  its  mot}oo  is  retrograde^ 


Digitized  by 


Google 


the  sun's),  is  to  24  hours,  so  is  the  approximate  time  of  transit  to  the 
apparent  time  of  the  planet's  transit  over  the  meridian  of  Greenwich. 

M>le.— -If  the  terms  be  reduced  to  seconds,  the  operation  may  be  easily 
performed  by  logarithnis.  . 

Example  1. 

Required  the  apparent  time  that  the  planet  Mars  will'pass  the  meridian 
of  Greenwich,  March  l6th,  1825  ? 

Planet's  right  ascension  at  noon  of  the  given  day  =       •    •    0M9T30' « 0 
Sun's  right  ascension  at  that  noon  =      ••••.•    •23.44.  0.3 

Approx.  time  of  the  planet'^  transit  over  the  mer.  of  Greenw.=s  1 1  5*29 ' .  7 

Planet's  right  ascension  at  noon>  March  .16th,  ss  0M9T30! 
Planet's  ditto  1 7th,  =^  0. 52. 20 


Planet's  motion  in  24  hours  =      .    .    •    .    .    0?  2?50!         0*  2r50! 

Sun's  right  ascension  at  noon,  March  I6th,  sr  28M4"  O'.S 
Sun's  ditto  17th,  =  23.47.39  .3 


Sun's  motion  in  24  hours  =     .    .    .    .    .    .0?  Sr39*.*0    0*  3rS9! 


Sum  of  the  motions  s    0*  6*29! 
^To^e.^^The  sum  is  taken  because  the  planet's  motion  is  retrograde^ 

As  24ho.+6r29?=:24',i3t29?,  In  secs.s:86789  LQg.ar.comp.^5. 061535 
Is  to  24  hours,  in  seconds  =  ...  80400  Log.  =  .  .  4, 936514 
So  is  the  approximate  time  of  transit 

=  I*5?29' .  7,  in  seconds  =      .    .      3929. 7        Log.  =  S. 594359 

To  the  apparent  time  of  the  planet's    . 

transit  =  1!5712\  I,  m  seconds  s     3912. 1        Log.  a  3. 592408 

Example  2. 

Recpiired  the  apparent  time  that  the  planet  Venus  will  pass  the  meriditfi 
of  Greenwich,  Sept.  23d,  1825  ? 

Planet's  right  ascension  at  noon  of  the  given  day  s  .  »  9^34^40'  •  0 
Sun's  right  ascension  at  that  noon  =     ••••••..  12.   0. 29  •  7 


Approx.  time  of  the  planet's  tr.  over  the  mer.  of  Gieenw.  =3  2l!34?10\3 


id  by 


Googlj 


HrrROPOCTOlY  FR0BL8US.  815 

Planet's  right  ascension  at  noon^  Sept.  23d,  =  9^34^40! 
Planet's  ditto  24th,=  9.39.20 


Planet's  motion  in  24  hours  =       ....      0*  4?40!      Ot  4?40! 

Sun's  right  ascension  at  noon,  Sept.  !23d,  as    12t  0?29'  •  7 
Sun's  ditto  24th,=5    12.   4.   5  .5 


Sun's  motion  in 24 hours  K      ...    .    .    Ot  8T35\8    0.  3.35  .8 


Difference  of  motion  »    0^  ir  4'.2 

iVate.--<-The  difference,  ia  taken  because  tbe  planet's  motion  is  pro- 
gressive. 

As  24*-.ir4*.  2e23*38r55\  8,in8ec8.s:  86335. 8  Log.ar.co.=5. 063809 
Is  to  24  hours,  in  seconds  :st  .  .  .  S6400.  Log.  ^  .  4.936514 
So  is  the  approx*  time  of  ' 

traQsit=21*34riO\3Jin8ecs.=     .      77650. 3  Log.  =  .     4.g90143 

To  the  apparent  time  of  the  planet's 

trai8it=21*35?8'.  1,  in  sees.  ='     •       77708. 1  Log.  =  .     4.  890466 


Problem  XL' 

Given  the  Apparent  Time  of  a  Planet'tl  Ti'anHt  over  the  Meridian  of 

Greemdch,  to  find  the  Jpparenit  Time  qf  Tramit  over  anjf  other 

Meridian^ 

Rule. 

Take,  from  page  IV.  of  the  month  in  the  Nautical  Almanac,  the  appa- 
rent times  of  the  planet's  transits  over  the  meridian  of  Greenwich  oh  the 
days  nearest  pf eceding  and  following  the  given  day,  and  find  the  interval 
between  those  times ;  find,  also,'  the  difference  of  transit  in  that  interval : 
then  say. 

As  the  interval  between  the  times  of  transit  is  to  the  difference  of  transit, 
so  is  the  longitude,  in  time,  to  a  correction ;  which,  being  added  to  the 
computed  apparent  time  of  transit^  if  the  longitude  be  west  and  the  plilnet's 
transit  increasing,  or  subtracted  if  decreasing,  the  sum  or  difference  will  be 
the  apparent  time  of  transit  over  the  meridian  of  the  given  place;  but,  if 
the  longitude  be  east,  a  contrary  process  is  to  be  observed  3  that  is,  the 
.correctioa  is  ta  be  subtracted  from  the  approximate  time  of  transit,  if  the 
transit  be  increasing,  but  to  be  added  thereto  if  decreasing. 

Note. — ^If  the  first  and  third  terms  of  the  proportion  be  esteemed  as 
mmtes  and  seconds,  the  operation  may  be  performed  by  proportional 
logaritbnis. 


Digitized  by 


Google 


316  Nautical  astronomy. 

Example  1. 

Required  the  apparent  time  that  the  planet  Mars  will  pass  the  meridian 
of  a  place  I45?30^  west  of  Greenwich,  March  16th,  1825,  the  computed 
apparent  time  of  transit  at  Greenwich  being  lt5?12M  ? 

Time  of  preceding  transit  =    .    .    •    •     13f  1*  8? 
Time  of  following  transit  s     •    »    •    •     19.    1.   3 


Interval  between  the  times  of  transit  ss       5  f  23^55? 


Difference  of  transit  in  that  interval  s=     •  *  5  minutes. 

As  the  interval=5f23*55r=143*55'r=2*23r55!  P.l6g.ar.co.=9.9028 
Is  to  the  difference  of  transit  =     *     «  5?         Prop.  log.  =   .  1.  5563 

So  is  the  long.  145?30'.  W.  in  time  =«:     9*42r  Q!  Prop.  log.  =     1. 2685 

To  the  correction  of  transit  =    ,     .     .     -     20!  Prop.  log.  =     2. 7276 
Computed  time  of  planet's  transit  over 

the  riieridian  of  Greenwich    =        •     1  *  5  ?  1 2 ' .  1 


Apparent  time  of  planet's  transit  over 

the  given  meridian  =      ....     It  4T52M 

Example  2. 

Required  the  apparent  time  that  the  planet  Venus  will  pass  the  meridian 
of  a  place  175^40'  east  of  Greenwich,  Sept.  23d,  1825,  the  computed 
apparent  time  of  transit  at  Greenwich  being  21t35T8'.l? 

Time  of  preceding  transit   ss      .    .     .     19^21*317 
Time  of  foHowing  transit  =s  .   ..    •    .    25.21.37 


interval  between  the  times  of  transit    =s       6f  Ot  6* 


Difference  of  transit  in  that  interval  s  •  6  minutes. 

As  the  interval=6f0*6'r=t=144*6r=2*24r  6!    P.  log.  ar,  comp.=9. 9034 
Is  to  the  difference  of  transit  =  6T         Prop.  log.  =     .     1.  4771 

Soisthelong.l75?40^E.,intiroe=5ll*42?40!    Prop.  log.  =     .     1:1867 


To  the  •correction  of  transit  s=     ...    .      /^  29!  Prop.  )og.s:2. 5672 

Computed  time  of  planet's  transit  over  the 
meridian  of  Greenwich  s     .    .    .    .    21^35?  8 '.1 


Apparent  time  of  planet's  transit  over  the 
given  meridian  s      «•«,,.    21t34T39M 


Digitized  by 


Google 


INTRODUCTORY  PROBLEMS.  317 


Probijsm  Xlf. 


To  find  the  Apparent  Time  of  a  Starts  Transit,  or  Passage  over  the 
Meridian  of  any  known  Place* 

Since  the  plane  of  the  meridian  of  any  given  place  may  be  conceived  to 
be  extended  to  the  sphere  of  the  fixed  stars,— therefore,  when  the  diurnal 
motion  of  the  earth. round  its  axis  brings  the  plane  of  that  meridian  to  any 
particular  star,  such  star  is  then  said  to  transit,  or  pass  over  the  meridian 
of  that  place.    This  observation  is  applicable  to  all  other  celestial  objects. 

The  apparent  time  of  transit  of  a  known  fixed  star  is  to  be  computed  by 
the  following 

Rule. 

Reduce  the  right  ascension  of  the  star,  as  given  in  Table  XL1V.,  to  the 
given  day ;  from  which  (increased  by  24  hours  if  necessary,)  subtract  the 
sun's  right  ascension  at  noon  of  that  day,  as  given  in  ttie  Nautical  Almanac^ 
and  the  remainder  will  be  the  approximate  time  of  transit. 

Turn  the  longitude  of  the  given  meridian  or  place  into  time,  by  Problem 
I.,  page  296,  and  add  it  to  the  approximate  time  of  transit  if  the  longitude 
be  west,  but  subtract  it  if  east ;  and  the  sum,  or  difference,  will  be  the 
corresponding  time  at  Grreenwich  ;  and  let  it  be  noted  whether  that  time 
precedes  or  follows  the  noon  of  the  given  day. 

Pind^  in  the  Nautical  Almanac,  the  variation  of  the  sun*s  right  ascension 
between  the  noons  preceding  and  following  the  Greenwich  time ;  then. 

To  the  proportional  logarithm  of  this  variation,  add  the  proportional 
logarithm  of  the  difference  between  the  Greenwich  time  and  the  noon  of 
the  given  day  (esteeming  the  hours  as  minutes,  and  the  minutes  as  seconds), 
and  the  constant  logarithm  9»  1249*  *,  the  sum  of  these  three  logarithms, 
abating  10  in  the  index,  will  be  the  proportionaHogarithm  of  a  correction, 
which^  being  added  to  the  approximate  time  of  transit  if  the  Greenwich 
time  precedes  the  noon  of  the  given  day,,  or  subtracted  therefrom  if  it 
follows  that  noon,  the  sum  or  difference  will  be  the  apparent  time  of  the 
star's  transit  over  the  given  meridian. 

Example  1. 

At  what  time  on  the  2d  of  January,  1825,  will  the  star  Rigel  transit,  of 
come  to  the  meridian  of  a  place  165?30C  east  of  Greenwich  ? 


*  This  U  the  ariihoMCical  complement  of  the  proporHontl logarithm  of  24  hours,  esteemed 
as  wtwtUfit 


Digitized  by 


Google 


Right  ascension  of  Rigel^  reduced  to  the  given  day,  =     .    .    5*  6T  8! 
Sun's  right  ascension  at  noon  of  the  given  day  =s     •    .    •    .  18. 5 1  •  44 

Approximate  time  of  transit  »     ••«••••«••  10M47S4! 
Longitude  165 ?30C  east^  in  time  a     ,    .    ,    » 11.  2.  0 

Greenwich  time  past  noon  of  January  1st  so 23M2724! 

which  it  47*36!  brfpre  noon  of  the  given  day* 

Sun's  right  ascension  at  nooUj  January  1st  =;     •    •    •    •     •  18? 47*19 f 
Sun's  ditto  2d  s     .    ^    .    .    .  18.51.44 

Variation  of  right  ascension  in  24  hours  ss Oi  4T2S'. 

Variation  of  right  ascension  s  4T25 !         Prop.  log.  =  1. 6102 
Diff.  of  Gr .  time  from  noon  =  47. 36  Prop.  log.  =  2. 355 8 

Constantlog. «      •,•,..•«...•    9.1249 

Correction  of  star's  transit  s    +  0?  9!  Prop.  log.  =  3. 0909 
Approximate  time  pf  transits  lOM 4?24  ? 

Apparent  time  of  transit  =     10?  14?33!^  as  required.* 


Example  2. 

At  what  time  on  the  2d  of  January,  1825^  will  the  star  Markab  transit, 
or  come  to  the  meridian  of  a  place  140?  40'  west  of  Greenwich  ? 

Right  ascension  of  Markab,  reduced  to  the  given  day,  as     *    22!56?  3! 
Sun's  right  ascension  at  noon  of  the  given  day  =&    .    .    •    •     18.51«44 

Approximate  time  of  transit  ac       •    •    • .  •    4    .    •    •    •      4*  .4*19' 
Longitude  140? 40'.  west,. in  time  a  .    .......      9.22.40 

—  J.         ■  ■ 

Greenwchtime  =5 ,    ,    .    13i26T59! 

which,  of  course,  is  post  the  nooa  of  the  given  day« 

Sun's  right  ascension  at  noon,  January  2d,  ss      •     .    •    •     18M1T44! 
Sun's  ditto  3d,  s       .    .    •    .     18.56.   8 

Variation  of  right  Mceorion  in  24  houi«s    «...•.      Ot  4?24! 


*  If  12  hourst  dimioished  by  half  tlie  variation  of  t&e  sun's  right  ascension,  ht  added  to 
te  apparent  time  of  tnarit.  tte«  foaod*  ike  stna,  abalUic  24  hoafs  if  aacasiao*  wttl  give 

the  apparent  time  of  transit  below  the  pole. 


IVTROBUCTOEY  PROBUTMS.  319 

Variation  of  right  ascensions    4?24;      IVop.log.  s  1.6118 

Diff.ofGr.timcfjromttoonal3*26?59!      Prop.log.  «  1.1266 

Constant  log.  m      .    ^    .    ^ 9. 1249 


Correction  of  star's  transit  B=     .-2T28!  Prop.  log.  =  1.8633 
Approximate  time  of  transit  a=   4U?19! 

Apparent  time  of  titmsit  a     .    4M  rS  1  !|  as  required.* 

Noter-^Th^  correction  of  a  star's,  approximate  time  of  transit  may  be 
readily  found  by  means  of  Table  XV.,  in  the  same  manner^  precisely,  as  jf 
it  were  the  proportional  part  of  the  sun's  right  ascension  that  was  under 
consideration. — See  explanation,  page  25,  and  examples,  pages  26  and 
28. 


PnoBLXM  XUL 

Tojmd  what  Siar$  mil  be  on^ar  neate$i  to^  th^  Meridian  ai  any  gteen 

lime. 

RULB. 

To  the  sun's  right  ascension,  at  noon  of  the  given  day,  add  the  apparent 
time  at  ship,  and  the  sum  will  be  the  right  ascension  of  the  meridian  or 
mid-heaveh;  with  which  enter  Table  XLW.,  and  find  what  stars'  right 
ascensions  correspond  with,  or  come  nearest  thereto,  and  they  will  be  the 
stars  required. 

If  much  accusacy  be  required,  the  sun's  right  ascension  at  noon  of  the 
given  day  must  be  previously  reduced  to  the  given  time  anci  place,  by 
Problem  V.,  page  298 ;  at  sea,  however,  this  reduction  may  be  dispensed 
with. 

Example  1. 

What  star  1^  be  nearest  to  tiie  meridian,  April  6th,  1825,  at  9U0r20! 
apparent  time } 

Sun's  right  ascensbn  at.ndon  of  .the  given  day  =  1  ^  0?24 ! 
Given  apparent  time  at  ship  or  place  =      •    •    9. 40. 20 

Right  ascension  of  the  meridian  or  mid-heaven=  1 0M0T44 ! 
Now^  this  being  looked  for  among  the  right  ascensions  of  the  stars^  in 


*  See  Note,  page  318. 

/Google 


Digitized  by ' 


320  NAUTICAL  ASTRONOMY. 

Table  XLIV.,  it  will  be  found,  that  the  star's  right  ascension  corresponding 
neatest  thereto,  is  that  of  ij  Argds  Navis;  which,  therefore,  is  the  star 
required,  or  the  one  nearest  to  the  meridian  at  the  giren  time. 

'Example  2. 

Wliat  star  will  be  neifTest  to  the  meridian,  December  31st,  1825,  at 
10^  12r41 :  apparent  time  ?  * 

Sun's  right  ascension  at  noon  of  the  given  dayss  18?41!'4«V 
.  Given  apparent  time  at  ship  or  place  =      •    •     10. 12. 41 

Right  ascension  of  the  meridian  or  mid-heaven=  4*54'r30! 

Now,  this  being  looked  for  among  the  right  ascensions  of  the  stars,  in 
Table  XLIV,,  it  will  be  found  that  the  staf's  right  ascension  corresponding 
nearest  thereto,  is  that  of  j3  Eridani ;  which,  therefore,  is  the  star  required, 
or  the  one  nearest  to  the  meridian  at  the  given  time. 

2Vbf€«--*When  the  sum  of  the  sun's  right  ascension  and  the  apparent 
time  exceeds  24  hours,  let  24  hours  be  subtracted  therefrom  5  and  the 
remainder  will  be  the  right  ascension  of  the  meridian,  as  in  tlie  last 
example. 


Probijbm  XIV. 

Given  the  observed  Altitude  of  the  hwer  or  upper  Limb  of  the  Sun^  to 
find  the  true  Altitude  of  its  Centre.    » 

•  Rule. 

For  the  Fore  Observation. 

To  the  observed  altitude  of -the  sun's  lower  limb  (corrected  for  index 
error,  if  any,)  add  the  difference  between  its  semi-diameter  *  and  the  dip 
of  the  horizon  f  3  and  the  sum  will  be  the  apparent  altitude  of  the  sun's 
centre:  or,  from,  the  corrected  observed  altitude  of  the  sun's  upper  limb 
subtract  the  sum  of  the  semi-diameter*  and  the  dip  of  the  horizon  fj  and 
the  remainder  will  be  the  apparent  central  altitude. 

For  the  Back  Observation. 
From  the  observed  altitude  of  the  sun's  lower  limb  subtract  the  difi^r- 

*  Page  III.  of  the  month  in  the  Nautical  Almanac.  t  Table  II. 

Digitized  by  VjOOQ IC 


INT&ODUCTORT  PR0BUM8.  321 

ence  between  its  semidiameter  and  the  dip  of  the  horizon :  or^  to  the 
observed  altitude  of  its  upper  limb  add  the  sum  of  the  semi-  diameter  and 
the  dip  of  the  horizon^  and  the  sun's  apparent  central  altitude  will  be 
obtained. 

Now^  from  the  apparent  altitude  of  the  sun's  centre,  thus  found,  sub- 
tract the  difference  between  the  refraction*  corresponding  thereto,  and  the 
parallax  in  altitudef,  and  the  remainder  will  be  the  true  altitude  of  the 
sun's  centre. 

Example  \. 

Let  the  observed  altitude  of  the  sun's  lower  limb,  by  eifore  observation, 
be  16?29^,  the  height  of  the  eye  above  the  level  of  the  sea  24  feet,  and 
the  sun's  semi-diameter  16H8?^  required  the  sun's  true  central  altitude? 

Observed  altitude  of  the  sun's  lower  limb  =     16?  29^  01 

Sun's  semidiameter  =s  ,     .     ^^'^^^In'^r 

Dip  of  the  horiz.  for  24  ket^  4. 42   J^^^' -   +  1 1 .  36 

Apparent  altitude  of  the  Sim's  centre  =     •    •     16?40'36T 
Refraction  =  3^  8r  n     .^  ^     ^ 

Parallax  =::     0,8   /  DiflFerence  =     ...     -^.  3.   0 


True  altitude  of  the  sun's  centre  =      •    •    .     16?37'36r 

Kxample  2. 

Let  the  observed  altitude  of  the  sun's  upper  limb,  by  hfore  observation, 
be  18?37  •)  the  height  of  the  eye  above  the  surface  of  the  water  30  feet, 
and  the  sun's  semi-diameter  15M6''  ^  required  the  true  central  altitude  ? 

Observed  altitude  of  the  sun's  upper  limb  s     18^37*  0? 
Sun's  semi-diameter  s     .     .     15' 46^1         _9i     i         * 
Dipof  the  horizon  for  30  feet  =  5. 15    /  ^^^ ■'    * 


Apparent  altitude  of  the  sun's  centre  =     •    •     18?  15 '.591 
Refraction  =    2^5K 
Parallax  =       0.    8 


j  Differences.    .    .      —    2.43 


True  altitude  of  the  sun's  centre  :?      .    .    •    18?  13^6? 

Example  3, 

Let  the  observed  altitude  of  the  sun's  lower  limb,  by  a  bach  observation, 
be  20?  10' ,  the  height  of  the  eye  above  the  level  of  the  sea  25  feet,  and  the 
sun's  semi-diamete/  15^55^ ;  required  the  true  central  altitude  ? 

•  Table  Vni.  t  Table  Vn. 

Y 

Digitized  by  VjOOQ IC 


322  MAUTIOAL  AlTROlfOirr. 


Observed  altitude  of  the  aun's  lower  limb  =     20M0i  0^ 
Sun's semi-cUameter  ss     .    .    \^''^^'XryM—      \\    g 
Dip  of  the  horizon  for  25  feet  =  4.47   J        * 

Apparent  altitude  of  the  aun's  centre  =    .    .     19'?58'.52* 
Refractional    2?35^ 
Farbllax  =  .    0.   8 


Refractions    2?35 ^  | ^^^^^^  ^  ^    ^    ^     ^     2.27 


True  altitude  of  the  sun's  centre  =s       .    •    •     1 9? 56 '  25 ? 


Example  4. 

Let  the  observed  altitude  of  the  sun's  upper  limb^  by  a  back  obsenaiianj 
be  25?31',  the  height  of  the  eye  above  the  surface  of  the  water  27 
feet^  and  the  sun's  semi-diameter  15  M9?;  required  the  true  central 
altitude  ? 


Observed  altitude  of  the  sun's  upper  limb   s     25 "^Sl  i  Ot 
Sun's  semi-diameter  =     .     15C49T1  ^        oa  47 

Dipofthehori2onfor27feet=4.58   J^^-    +^U-47 


Apparent  altitude  of  the  sun's  centre  =s    .    .    25?51  M7^ 
Refractions    1'57^1-..^  ,    ^^ 

Parallax  =      0.   8   /I>iflference«     ,    •         -  1.49 

True  altitude  of  the  sun's  centres      .    •    •    25?49'^58f 

Semark.'^  think  it  my  duty,  in  this  place,  to  caution  the  mariner 
against  the  mistaken  rule  for  the  back  chservaHony  given  in  some  treatises 
on  Navigation ; — because,  if  that  rule  be  adopted^  the  ship's  place  will, 
most  assuredly,  be  affected  by  an  error  in  latitude  equal  to  the  full  measure 
of  the  sun's  diameter,  or  about  32  miles :  and  this,  to  a  ship  approaching 
or  drawing  in  with  the  land,  becomes  an  object  of  the  most  serious  con- 
sideration, since  it  so  very  materially  affects  the  lives  and  mlpreits  of  those 
concerned.    To  set  the  mariner  right  in  this  matter^  I  will  here  work  an 

Examfie. 

December  25th,  1825,  in  longitude  35"?  W.,  the  meridiw  altitude  of  tke 
sun's  lower  limb,  by  a  back  observation^  was  16?28C  south,  the  height  of  the 
eye  being  20  feet  3  required  the  latitude  ? 


Digitized  by 


Google 


IKT^ODUCrofiY  PROBLEMS.  d23 

Observed  altitude  of  the  sun's  lower  limb  =     16?28!  0? 

Sun's  semi-diameter  =      .     1 6  U  8^  i 

Dip  of  the  horizon  for  20  feet=4. 17   /^^^•==   ^  *2.    1 


Apparent  altitude  of  the  sun's  centre  =    .    .     16^15^59^ 

Refraction  =    3'13?1 

Parallax  =    .0.   8   /Differences  ,    .    .     -     3.   5 


True  altitude  of  the  sun's  centre  a:      .    .     i     16?12'64^ 


Sun's  meridional  zenith  distance  =:     •    i     *    73M71  6^  north. 
Sun's  corrected  declination  s     .     •     «    ;  . «    2di  24^  46    south. 


Required  latitude  c=     ........    60?22ia0r  north. 

By  the  old  rule,  the  latitude  is  only  49?50'  norths  wbieb  is  evidently 
erroneous^  it  being  32  miles  and  20  seconds  less  than  the  truth. 


Problbm  XV. 

Given  the  oSserved  Altitude  of  the  upper  or  tower  Limb  qfihe  Moon,  to 
find  ihe  trw  central  AUitude. 

RULB. 

Turn  the  longitude  into  time,  and  add  it  to  the  apparent  time  of 
observation  if  it  be  west,  or  subtract  it  therefrom  if  east,  and  it  will  give 
the  corresponding  time  at  Greenwich. 

To  this  time  let  the  moon's  semi-diameter  and  horizontal  parallajc  be 
reduced,  by  Problem  VL,  page  302,  (or  by  Table  XVI.,  as  ekplaincsd  iri 
pages  30  and  33,)  aiid  let  the  reduced  semi-dimneter  be  increased  by  the 
correction  contained  in  Table  IV.,  answering  to  it  and  the  observed  alti- 
tude;  then. 

To  the  observed  altitude  of  the  moon's  lower  limb  (corrected  for  index 
error,  if  any),  add  the  difference  between  the  true  semi-diameter  and  the 
dip  of  the  horizon ;  dr,  from  the  observed  altitude  of  the  upper  limb 
subtract  the  sum  of  the  semi-diameter  and  dip,  and  the  apparent  central 
altitude  of  the  moon  will  be  obtained ;  to  which  let  the  correction  (Table 
XVIII.)  answering  to  the  moon's  reduced  horizontal  parallax  and  apparent 
central  altitude  be  added,  and  the  sutn  will  be  the  altitude  of  the  moon's 
cetitre. 

Example  1. 

In  a  certain  latitude,  March  10th,  1825^  at3M0T2O!  apparent  time, 

y  2 


Digitized  by 


Google 


324  NAUTICAL  ASTRONOMY* 

the  observed  altitude  of  the  moon's  lower  limb  was  20?10U0T9  and  the 
height  of  the  eye  above  the  level  of  the  sea  24  feet ;  required  the  true 
altitude  of  the  moon's  centre^  the  longitude  of  the  place  of  observation 
being 35^40'.  west?      ^ 


Apparent  time  of  observation  =    • 
Longitude  35'?40'  W.,  in  time   = 


Greenwich  time  = 


Moon's  reduced  semi-diameter  =s    , 
Augmentation,  Table  IV.  s      •    • 

Moon's  true  semi-diameter  s     .     • 
Moon's  reduced  horizontal  parallax  s= 


3*40T20* 
2.22.40 


6*  3?  0! 


15^401: 
0.   6 

15<46r 
57 '32^ 


Observed  altitude  of  moon's  lower  limb  =    20?10'.40r 
Moon's  true  semi-diam.  =  15  ^46T  ' 
Dip  of  the  horiz.  for  24  feet=4. 42 


}Diff..=  +11.   4 


Apparent  altitude  of  the  moon's  centre  =    20?  2  T.  44 ^ 
Correction  to  altitude  20?21M4r,  and 

horiz.  parallax  57 '  32'r ,  Table  XVIII.  =    +  5 1 .  24 


True  altitude  of  the  moon's  centre  =     •    21  ?  13'  8r 

Example  2, 

In  a  certain  latitude,  March  26th,  1825,  at  1  ^30^47  •  apparent  time, 
the  observed  altitude  of  the  moon's  upper  limb  was  30?  17 '30?,-  and  the 
height  of  the  eye  above  the  level  of  the  sea  30  feet ;  required  the  true 
altitude  of  the  moon's  centre,  the  longitude  of  the  place  of  observation 
being  94^15 '.30r  east? 

Apparent  time  of  observation  =     .     .    .     Ii30"47* 
Longitude  94n5:30^  E.,  in  time    =     .    6.17.   2 


Greenwich  time  past  midnight,  March  25th=7  ^  13T45 ! 


Moon's  reduced  semi-diameter  =    ...         15'.23T 
Augmentation,  Table  IV.  =     .     •    •    •  0.   9 


Moon's  true  semi-diameter  s=.    •    .    •    .         15 '3  K 
Moon's  reduced  horizontal  parallax  ==    •        56'.2(>T 

/Google 


Digitized  by ' 


INTRODOCTORY  PROBLBMS*  325 

Observed  altitude  of  moon's  upper  limb  =  30?  17C30^ 
Moon's  true  semt-diaro.  =  15  '3K  1 
Dipofthehoriz.for3(>feet=5. 15    /^^"">=  -20.46 

Apparent  altitude  of  moon's  centre  =     .    29?56'44r 
Correction  to  altitude  29?56M4?,  and 
horiz.  parallax  56  \  26r,  Table  XVIII.  =   +  47. 16 


True  altitude  of  the  moon's  centre  =     .    30? 44^  Or 

iVb/6*— ^In  the  above  examples,  the  altitudes  are  supposed  to  be  taken  by 
the  fore  obseroatum;  and  since  this  mode  of  observing  is  not  only  the 
most  naturiU,  but,  also,  the  most  simple,  it  will,  therefore,  be  constantly 
made  use  of  throughout  the  subsequent  parts  of  this  work.  Hence  the 
necessity  of  making  constant  reference  to  the  particular  mode  of  observa- 
tion may,  in  future,  be  dispensed  with. 


Pbobum  XVI. 

Gicen  the  observed  AUUude  qf  a  Planet's  Centre,  to  find  its  true 

JUitude.  ^ 

Rule. 

From  the  planet's  observed  central  altitude  (corrected  for  index  error,  if 
any,}  subtract  the  dip  of  tlie  horizon,  and  the  remainder  will  be  the  apparent 
central  altitude. 

Find  the  difference  between  the  planet's  parallax  in  altitude  (Table  VI.} 
and  its  refraction  in  altitude  (Table  VIll.} ;  now,  this  difference  being 
applied  by  addition  to  the  apparent  central  altitude  when  the  parallax  is 
greater  than  the  refraction,  but  by  subtraction  when  it  is  less,  the  sum  or 
remainder  will  be  the  true  central  altitude  of  the  planet. 

JBspample  I. 

Let  the  observed  central  altitude  of  Venus  be  16?40^,  the  index  error 
2^30'  tvbtractive,  and  the  height  of  the  eye  above  the  level  ot  the  sea  28 
feet;  required  the  true  altitude  of  that  planet,  allowing  her  horizontal 
parallax  to  be  31  seconds  ? 


326  NAUTICAL  ASTRONOMY. 

Observed  central  altitude  of  Venus  =     .  16?40'  Or 

Index  error  = —     2. 80 

Dip  of  the  horizon  for  28  feet  =  .     .     .  —     5.5 

Apparent  central  altitude  of  Venus  =     .  16'?32'25T 
Refraction,  Table  VIII.,=3 '.  10^  \j^.^_  n  .  i 

Parallax,  Table  VI.,  =     0.29   J  ^^^^^  "  ^^^* 


^^/c^.  Apparent  central  altitude  of  Venus  =     .     16?29^24r 

Example  2. 

Let  the  observed  central  altitude  of  Mars  be  17'?29'40r,  the  index  error 
aUSr  additive,  and  the  height  of  the  eye  above  the  surface  of  the  water 
26  feet  \  required  the  true  central  altitude  of  that  planet,  allowing  hi8 
horizontal  parallax  to  be  17  seconds  ? 

Observed  central  altitude  of  Mars  »       .     1 7?39 '  40^ 
Index  error    =     .......     •      +3.45 

Dip  of  the  horizon  for  26  feet  =     .    .     —     4. 52 

Apparent  central  altitude  of  Mars  =     .     17°28'33'!f 
Refraction,TableVIII.,=2?59lf1_.„  _  ^    .^ 


Parallax,  Table  VI.,  =     0.16 


/yH^4^   Apparent  central  altitu4^  qf  Mars  =     •     17^25^50^ 

'Reffwvrk^ — ^In  taking  the  altitiick  pf  a  planet,  its  centre  should  be 
brought  down  to  the  horizon.  Neither  the  semi-diameters  nor  the  hori- 
zontal parallaxes  of  the  planets  are  given  in  the  Nautical  Almanac,  but  h  is 
to  be  hoped  that  they  soon  will  be.  If  the  parallaxes  of  the  plaueta  be  de- 
termined by  means  of  a  comparison  of  their  respective  distances  (from  the 
earth's  centre)  with  the  earth's  semi-diameter,  they  will  be  found  to  be  as 
follows^  very  nearly ;  viz., 

Venus'  greatest  horizontal  parallax,  about  S3  seconds  \  and  her  least 
parallax  about  5  seconds. 

Mars'  greatest  horizontal  parallax,  about  17  seconds  \  and  bis  least  paral* 
lax,  about  3  seconds. 

Jupiter's  mean  horizontal  pai^aiUKx^  i^btfiit  2  seconds  \  and  that  of  Saturn 
about  1  second. 

Thj»  pairallaxed  of  the  two  last  planets  are  subjeet  to  i»ry  iiMie  alleiatioQ, 
because  the  dislances  at  which  those  otjeels  ate  placed  froHL  tks  esvlL'b 
oentm  are  so  exQeedi^g^y  great  as  to  raidor  any  variatiMiB  id  tkisu  panal- 
laxes  almost  insensible. 


Digitized  by 


Google 


INTRODUCrOftY  PHOBLBMS.  327 

Problem  XVII. 
Gioen  the  observed  Altitude  of  a  fixed  Star,  to  find  the  true  Altihide. 

RULB. 

To  the  observed  altitude  of  the  star  apply  the  index  error^  if  any;  from 
which  subtract  the  dip  of  the  horizon,  and  the  remainder  will  be  the 
star's  apparent  altitude. 

From  the  apparent  altitude,  thus  found,  let  the  refraction  corresponding 
thereto  be  subtracted,  and  the  reminder  will  be  the  true  altitude  of  the 
star* 

Example  1. 

Let  the  observed  altitude  of  Spica  Vir^inis  be  18?30^,  the  index  error 
3!  20^  subtractive,  and  the  height  of  the  eye  above  the  level  of  the  water 
18  feet  'j  required  the  true  altitude  of  that  star  ? 

Observed  altitude  of  Spica  Virginis  =  18?30^  01 

Index  error  « —  3. 20 

Dip  of  the  horizon  for  18  feet  =     •       —  4.   4 


Apparent  altitude  of  Spica  ^rginis  s  18?  22 '36? 
Refraction  = —  2. 50 


True  altitude  of  Spica  Virynk  ar  .     18?  19M6r 
Example  2. 

Let  the  observed  altitude  of  Regulus  be  20?43',  the  index  error  1'47? 
sdcKtive,  and  the  height  of  the  eye  above  the  level  of  Ihe  sea  20  feet ; 
reqiured  the  true  altitude  of  that  star  ? 

Observed  aUtode  of  Reguhiss  20?43C  Oi 
Index  errors      .....       +1.47 
I>ipofthehori»»lbr20ieet8i   -  4.17 

Apparent  altitude  of  Reguhis  s  20?40C30r 
Refraction  s       -.  2. 29 


Tnie«ititwleofiUg«lBa«:    .    a0t38^   K 

JVbte.— The  fixed  stars  do  not  exhibit  any  apparent  semi-diameter,  nor 
any  sensible  parallax ;  because  the  immense  and  inconceivable  distance  at 
which  they  are  placed  from  the  earth's  surface  causes  them  to  appear,  at 
all  times,  as  so  many  mere  Ittwomit  indivisible  points  in  the  heavens. 


Digitized  by 


Google 


328  NAUTICAL  ASTRONOMY. 


SOLUTION  OF  PROBLEMS  RELATIVE  TO  THE  LATITUDE. 


The  Latitude  of  any  place  on  the  earth  is  expressed  by  the  distance  of 
such  place  from  the  equator,  either  north  or  south,  and  is  measured  by  an 
arc  of  the  meridian  intercepted  between  the  said  place  and  the  equator.-— 
Or, 

The  Latitude  of  any  place  on  the  earth  is  equal  to  the  elevation  of  the 
pole  of  the  equator  above  the  horizon  of  such  place ;  or  (which  amounts  to 
the  same),  it  is  equal  to  the  distance  of  the  zenith  of  the  place  from  the 
equinoctial  in  the  heavens.  The  complement  of  the  latitude  is  the  distance 
of  the  zenith  of  any  place  from  the  pole  of  the  equator,  and  is  expressed 
by  what  the  latitude  wants  of  90  degrees.  The  latitude  is  named  north  or 
south,  according  as  the  place  is  situate  with  respect  to  the  equator. 


Problem  I. 

Given  the  Sun's  Meridian  AUiiude,  to  find  the  Latitude  oftlie  Place  qf 

Observation. 

Rule. 

Find  the  true  altitude  of  the  sun's  centre,  by  Problem  XTV.,  page  320, 
and  call  it  north,  or  south,  according  as  that  object  may  be  situate  with 
respect  to  the  observer  at  the  time  of  observation ;  which,  subtracted  from 
90?,  will  give  the  sun's  meridional  zenith  distance  of  a  contrary  denomi- 
nation to  that  of  its  altitude.  -     •     • 

Reduce  the  sun's  declination  to  the  meridian  of  the  place  of  observation, 
by  Problem  V.,  page  298,  or,  more  readily,  by  Table  XV.  Then,  if  the 
meridional  zenith  distance  and  the  declination  are  both*  north  or  both  south, 
their  sum  will  be  the  latitude  of  the  place  of  observation;  but  if  one  be 
north  and  the  other  south,  their  difference  will  be  the  latitude,  and  always 
of  the  same  name  with  the  greater  term.* 


*  The  principles  upon  which  this  role  is  founded  may  he  seen  hy  referring^  to  **  The 
Youni^  Navigator's  Guide  to  the  Sidereal  and  Planetary  Parts  of  Nautical  AstroimiQj/* 
page  98 1  readiog;,  howerer,  the  word  «m  instead  of  star. 


Digitized  by 


Google 


OF  FINDING  THE  I.ATITUDB  BY  A  MBRIDIAN  ALTITUDB.  329 

Example  1. 

April  10th,  1825,  in  longitude  75?  W.,  the  meridian  altitude  of  the 
sun's  lower  limb  was  57?4U'.30^  S.,  and  the  height  of  the  eye  above  the 
level  of  the  sea  22  feet ;  required  the  latitude^ 

Observed  altitude  of  the  sun's  lower  limb  =     S7?40C30r  S. 
Sun's  semidiameter  =s    .     ^^'^S'' I  ^.^  _^ 
Dipofthehoriz.for22feet=4.30  /^»"- -   +     "-29 

Apparent  altitude  of  the  sun's  centre  =       •    57?51  C59T  S. 
Refraction      =     0^35ri^.^  •  ,,  ^^ 

Parallax         =    0.   5   /  Difference  =     .     -      0.30 

True  altitude  of  the  sun's  centre  s     «    .    .    57^5K29rS. 


Sun's  meridional  zenith  distance  =    .    .    .    32?  8C3irN.  32?8:3KN. 

Swi's  declination  at  noon,  April  10th  =   7^56 M2?  N, 
Correction  for  longitude  75?  W.  =     .     +     4. 36 

Sun's  reduced  declination  =       ...    8?  I'lSTN.  8?ia8rN. 


Latitude,  as  required  =     •    ;    •    •    •    40?9C49?N. 

Note. — ^The  meridional  zenith  distance  and  the  declination  are  added 
tc^ther,  because  they  are  both  of  the  same  name :  hence,  the  latitude  is 
40?9M9r  N. 

Example  2. 

October  24th,  1825^  in  longitude  90?  east,  the  meridian  altitude  of  the 
ran's  lower  limb  was  27?31^20^  S.,  and  the  height  of  the  eye  above  the 
surface  of  the  sea  23  feet ;  required  the  latitude  ? 

Observed  altitude  of  the  sun's  lower  limb    =      27?31  ^  20r  S. 

Sun's  semi-diameter   =     .     16'  8^1  ^ 

Dip  of  the  horiz.  for  23  feet  =  4. 36   J  *^'^-  •"     +  ^  *  •  ^^2 


Apparent  altitude  of  the  sun's  centre  =     •    .  27?42*52T  S. 

?*'*^°°=i''*f\  Differences     ...  -    1.40 
Parallax  =      0.    8   J 

True  altitude  of  the  sun's  centre  =      .    .    •  27 ?4 1 U  2r  S. 


Sun's  meridional  zenith  distance  ss     ,    .    «    62?18'48rN, 

/Google 


Digitized  by ' 


330  NAOTICAt  ASTRONOMY. 

Sun*8  declination  at  noon,  Oct  24th  =  1 1  ?45  ^  42r  S. 

Correction  for  longitude  90?  east  =    •  —  5. 15 

Sun's  reduced  declination  =     •    •    •  1 1  ?40^  27^  S. 

Sun's  meridional  zenith  distance  ss    •  62.18.48  N. 


Latitude,  as  required  ^      •    •    •    *    50?38'2KN. 

Note.— The  difference  between  the  meridional  zenith  distance  and  the 
declination  is  taken^  because  they  are  of  contrary  names :  hence^  the  lati* 
tudei8  50?38:2irN. 


Problem  IL 

Given  the  Mom's  MenSmial  AUitude,  ioJM  the  Lufiltnde  of  the  Place 

of  Observatioju 

Bulb. 

Reduce  the  moon'^  passage  over  the  meridian  of  Greenwich,  on  tb« 
given  day,  to  the  meridian  of  the  place  of  observation,  by  applying  thereto 
the  correction  in  TaUe  XXXVIIL,  by  addition  or  subtraction,  accoidiiig 
as  the  longitude  is  west  or  east;  as  explained  in  examples  1  and  2,  pages 
tOl  and  103. 

To  the  time  of  the  moon's  passage  over  the  meridian  of  the  place  of 
observation,  thus  found,  let  the  longitude  of  that  meridian,  m  time,  be 
added  if  it  be  west,  or  subtracted  if  east;  and  the  sum,  ox  differeiice,  will  be 
the  corresponding  time  at  Greenwich :  to  which  let  the  moon's  decUiuitkM^ 
horizontal  parallax,  and  semi-diameter,  be  reduced  by  Problem  VI.,  page 
302,  (or  by  means  of  Table  XVI.,  as  explained  in  page  30,)  and  let  the 
moon's  reduced  semi-diameter  be  corrected  by  the  augmentation  contained 
in  Table  IV. 

Find  the  true  altitude  of  the  moon's  centre,  by  Problem  XV.,  page  323^ 
and  call  it  north  or  south,  according  as  it  may  be  situate  with  respect  to 
the  observer  at  the  time  of  obeenration  |  which,  sobtiacted  from  90?,  will 
give  the  moon's  meridional  zenith  distance  of  a  ccmtrary  detwmBHiation  to 
that  of  its  altitude. 

Then,  if  the  meridional  zenith  distance  and  the  declination  are  of  the 
same  name,  their  sum  will  be  the  latitude  of  die  place  of  observation ;  but 
if  they  are  of  contrary  names,  their  difference  will  be  the  fattitud^  of  the 
same  name  with  the  greater  term. 

Note^-^ln  strictness,  the  moo»'s  dccliBatioo  siM>uld  be  oovreeted  by  the 
equation  of  second  difference  contained  in  Table  XVII,  as  explained 
between  pages  33  and  37* 


Digitized  by 


Google 


OF  FINDING  THB  MTITTOB  BT  A'  MBBIDIAN  ALTITUDB.  331 


Exampie  I. 

Jnnviuy  27th9 1825,  in  longitude  55"?  W.,  the  meridian  altitude  of  the 
moQp'a  lower  limb  was  58?40t  S.,  and  the  height  of  the  eye  above  the 
level  of  the  sea  26  feet )  required  the  latitude ) 

Time  of  ])  'a  passage  over  the  meridian  of  Greenwich  s=    ,    •    5^54?  0! 
Correction,  Table  XXXVIII.,  for  longitude  55?  W.  =     .    .     +7.23 

lime  of  )>  *8  pass,  over  the  merid.  of  the  place  of  observation  =  6^   1^23^ 
Iiongitude  55?  W.,  in  time  = +  3.40.   0 

Greenwich  time  = 9?4lr23! 

Moon's  horizontal  parallax  at  noon,  Jan.  27th  ss     55 '20^ 
Correction  of  parallax  for  9*41  r23t   »      •    •     +0.17 


Moon's  reduced  horizontal  parallax  s  •    «    •    .    55 '37^ 


Moon's  semi-diameter  at  noon,  Jan.  27th  =  ,     .     15^  5T 
Correction  of  semi-diameter  for  9*41  ?23!  =      .       +4 
Augmentation  of  semi-diameter.  Table  IV.  9      ,       +12 


Moon's  true  semi-diameter  = 15^31^ 


Mom's  declination  at  nooo,  Jan.  27tb  a  18?  19!  181: N. 
Cbntctioaofdeclinatioafbr9MlT2S!s:  +1. 16.  7 


Moon's  reduced  declination  ::;:      «    .    .     19?35 1 25  r  N. 


Obeenred  altitude  of  the  mooo^s  lover  Kmb  s  .    •    58?40?  OfS. 
Motn'a  true  semi-diameter    =      15!2P  l         ^ 
I>ipofthehoriz.for26feet  =    .    4.52/*^*"     +  IU.2» 


Apparent  altitude  of  the  moon's  centre  = 
CorrectioQ  of  altitude.  Table  XVHL  = 

Thie  altitude  of  the  moon's  centre  s= 

\ 

Moon's  meridional  zenith  distance  = 
Moon's  reduced  declination  =      •     • 

Latitude  of  the  place  ot  obeeivation  ^ 


..  58t50^2»*S. 
.     +  28. 12 

.    5»?I8UirS. 

.    30?4in9rN. 
.     19.35.25  N. 

•    50?16:44rN. 

Digitized  by  VjOOQ IC 


332  KAtrricAL  astronomy. 


Example  2. 

February  3d,  1825,  in  longitude  65?  E.,  the  meridian  altitude  of  the 
moon's  upper  limb  was  62?45^  north,  and  the  height  of  the  eye  above 
the  level  of  the  sea  29  feet  j  required  the  latitude  ? 

Time  of  ])  's  passage  over  the  meridian  of  Greenwich  =       «     12t25?  0! 
Correction,  Table  XXXVIIL,  for  longitude  65?  east  =    ,    .     -     9. 44 

Time  of  })  's  pass,  over  the  merid.  of  the  place  of  observations  12 1 15T16' 
Longitude65?E.,  in  time  =      .....•....—  4.20.    0 

Greenwich  time  = 7*55?16! 

Moon's  semi-diameter  at  noon,  February  3d  =  16^341^ 
Correction  of  semi-diameter  for7*55?16!   s      -f     1 
Augmentation  of  semi-diameter,  Table  IV.  =      +16 

Moon's  true  semi-diameter  = ]6^5K 


Moon's  horizontal  parallax  at  noon,  February  3d  =  60U9T 
Correction  of  parallax  for  7*55716!  =       .    .     .      +     5 


Moon's  reduced  horizontal  parallax  =     .    .    •    •    60' 54^ 


Moon's  declination  at  noon,  February  3d  s     1 2^ 52 !  50^  N. 
Correction  of  declination  for  7*55?16!  =5      -  1. 47. 29 


Moon's  reduced  declination  s 11?5!21TN. 


Observed  altitude  of  the  moon's  upper  limb  =     62?45C  OTN. 

Moon's  true  semi-diameter  =5  16'51^1 

Dip  of  the  horiz.  for  29  feet  =    5. 10  J  ^""^  =     -  22.    1 


Apparent  altitude  of  the  moon's  centre  =     .    .  .  62?22'59r  N. 
Correction  of  altitude.  Table  XVIII.  ==    .    .    .     +29.33 


True  altitude  of  the  moon's  centre  s      •    •    •    62?52^32rN. 


Moon's  meridional  zenith  distance  ^     .    •    27?  7-28?  S. 
Moon's  reduced  declination  s=      •    ,    •    .     11;  5.21  N. 


Ladtude  of  the  place  of  observation  =  •    •    16?  2C  7'  S. 

/Google 


Digitized  by ' 


OF  FINDING  THE  JLATITUDB  BY  A   MERIDIAN  ALTITUDE. 

Remark. — Although  this  method  of  finding  the  latitude  at  sea  is  strictly 
correct  when  the  longitude  of  the  place  of  observation  is  well  determined; 
yet,  in  some  cases,  it  is  subject  to  such  peculiarities  as  to  render  it  inconve- 
nient to  the  practical  navigator :  this  happens  in  high  latitudes,  and  when 
the  variation  in  the  moon's  declination  is  very  considerable ;  because,  under 
such  circumstances,  the  moon's  altitude  sometimes  continues  to  increase 
after  she  has  actually  passed  the  meridian*  To  provide  against  this,  the 
observer  should  be  furnished  with  a  chronometer,  or  other  well-regulated 
watch,  to  show  the  instant  of  the  moon's  coming  to  the  meridian  of  the 
ship  or  place  3  at  which  time  her  altitude  should  be  taken,  without  waiting 
fcr  its  ceasing  to  rise  or  beginning  to  dip,  as  it  is  generally  termed  at  sea : 
then  this  altitude  is  to  be  considered  as  the  observed  meridional  altitude  of 
that  object^  and  to  be  acted  upon  accordingly. 


PUOBLSM  IIL 

Given  the  Meridional  Altitude  of  a  Planei,  to  find  the  Latittide  qfthe 
Place  of  Observation. 

Rule. 

To  the  apparent  time  of  observation  (always  reckoning  from  the  preceding 
noon,)  apply  the  longitude,  in  time,  by  addition  or  subtraction,  according 
as  it  is  west  or  east ;  and  the  sum,  or  difference,  will  be  the  corresponding 
time  at  Greenwich,  to  which  let  the  planet's  declination  be  reduced,  by 
Problem  VIL,  page  307. 

Find  the  true  altitude  of  the  planet's  centre,  by  Problem  XVI.,  page 
325 ;  and  hence  its  meridional  zenith  distance,  noting  whether  it  be  north 
or  south  :  then,  if  tlie  meridional  zenith  distance  and  the  declination  are  of 
the  same  name,  their  sum  will  be  the  latitude  of  the  place  of  observation ; 
but  if  they  are  of  contrary  names,  their  difference  will  be  the  latitude,  of 
the  same  nam'e  with  the  greater  term. 

Example  I. 

February  3dj  1825,  in  longitude  80?  W,,  at  II*28?30!  apparent  time, 
the  meridional  central  altitude  of  the  planet  Jupiter  was  58?22'  S.,  the 
height  of  the  eye  above  the  level  of  the  sea  24  feet,  and  the  planet's  hori- 
zontal parallax  2  seconds ;  required  the  latitude? 

Apparent  time  of  observation,  February  =     3 f  1 1 1 28T30' 
Longitude  80?  W.,  in  time  =    ....     +     5.20.   0 

Greenwich  time  =5      .......      3fl6*48r30! 

Digitized  by  VjOOQ IC 


884  KAimcAL  AiTRONoiiy. 

Jupiter's  declination^  February  Ist  »     ,     19?  S'  OI'N. 
Correction  of  ditto  for  2fl6M8730!      «     +     5.51 


Jupiter's  reduced  declination  tt    ,    «    .     19?  8'5KNi 

Jupiter's  observed  central  altitude  =  58?22C  Or  S. 
Dip  of  the  horizon  for  24  feet    =       —     4.42 


Jupiter's  apparent  central  altitude  =  58?  l/'lSf  8. 

RefracUohjTab.VII 

Parallax,  Table  VI. 


RefracHoh,Tab.VIII.=0:S4r  \ 

1.=  0.  1  /^*^'=  -"-^ 


Jupiter's  trae  eentral  altitude  »     •    68?  16'. 45?  S. 

Jupiter's  meridional  zenith  distance  =3 1  ?43  M  5  '  N. 
Jupiter's  reduced  declination  ss      ;     19.   8.51   N^ 

Latitude  of  the  place  of  observation  sa  50?52'  6?  N. 

Bsampk  2. 

March  16th,  1825,  tn  lon^tude  75?  E.,  at  iU9?  apparent  time,  the 
meridional  central  altitude  of  the  planet  Venus  was  31?  IOC  N.,  the  height 
of  the  eye  above  the  level  of  the  horizon  18  feet,  and  the  planet's  horizon- 
tal parallax  23  seconds ;  required  the  latitude  ? 

Apparent  time  of  observation,  Mareb  ta    16f  2M9? 
toingltude,75?B.^in  time  =s        «    «    «     ^    5^  0 

(}reenwich  time  =     «••«•••     15  f  21^49? 


Venus' declination,  March  13th  =     17?I5(  OrN, 
Correction  of  ditto  for  2f  21*49*?=    +  1.   5. 57 


Venus*  reduced  declination  =3    •    •     18?20'57?N. 


Venus'  observed  central  altitude  =     .     3 1  ?  10 !  0?  N. 
Dip  of  the  horizon  for  18  feet  a    •     .     —     4.   4 

Venus'  i^parent  central  altitude  =     4    3 1  ?  5  C  56?  N. 
Refraction,  Table  VIII.  =  1  •  83  ^  1  p..^  _ 
Parallax, Table VI.    =     0*20  i^*"' ^'^^ 


Venus'  true  central  altitude  =    •    .     •    3 1  ?  4 '  4 1  ?N. 


Venus'  meridional  zenith  distance  =  s     58?55'  19?  S. 
Venus' reduced  declination  =  .     •     .     18.20.57  N. 


Latitude  of  the  place  of  observation  =    40?34 '  22?  S. 

/Google 


Digitized  by ' 


OF  FINDING  THB  LATITUDB  BY  A  IIBRIDIAN  ALTITUDB.  835 

Note.^-^e  principles  of  finding  the  latitude  by  the  meridional  altitude 
of  a  celestial  object  may  be  seen  by  referring  to  ^*  the  Young  Narigator's 
Guide  to  the  Sidereal  and  Planetary  Parts  of  Nautical  Astronomy/'  between 
pages  98  and  105. 


PnoBiJUlff  IV* 

Given  the  Meridional  Jltiiude  qf  a  fixed  Star,  to  find  the  Latitude  qfthe 
Place  of  Obeervation, 

RtTLB. 

Und  the  true  altitude  of  the  star^  by  Problem  XVIL,  page  327;  and 
hence  its  meridional  zenith  distance^  noting  whether  it  be  north  or  south. 
Take  the  declination  of  the  star  from  Table  XLIV.,  and  reduce  it  to  the 
time  of  observation.  Now^  if  the  star's  meridional  zenith  distance  and  its 
declination  be  of  the  same  name,  their  sum  will  be  the  latitude  of  the  place 
of  obtervadoD ;  but  if  they  are  of  contrary  names^  their  diffeience  will  bo 
the  latitude,  of  the  tame  name  with  the  greater  term. 

Example  1. 

January  Ist,  1825,  fn  longitude  85?3f  W.,  at  12f  39r26!  appartot  time, 
the  meridian  altitude  of  Procyon  was  44?49^  S.,  and  the  height  of  the  eye 
atbove  the  level  of  the  horizon  16  feet ;  required  the  true  latitude  ? 


Observed  altitude  of  Procyon  ss     • 
Dip  of  the  horizon  for  16  feet  =  • 

Procyon's  apparent  altitude  =  •    • 
Refractions:^   ....... 


Proeyon's  true  altitude 


•    •    • 


44?49i  OrS. 

-  3.50 

44?45n0rS^ 

—  0.57 

44^44: 13rS. 


Pipocyon'a  meridional  zenith  distance  =a  45?  15' 47  ^N. 
Procyon's  reduced  declination  =3   .     •      5.40.16  N. 

Latitude  of  the  place  of  observation  8    50?56C  37N* 

Example  2. 

January  2d,  1825,  in  longitude  165 ?30'  R,  at  I0M4r3d!  i4>parent 
time^  the  meridian  altitude  of  Rigel  was  30^39^  S.,  and  the  height  of  the 
eye  abo^e tiM  level  of  the  sea  21  twt}  jctfired  Ibe  true  lMiCitde7 


Digitized  by 


Google 


336  NAUTICAt  ASTRONOMY. 

Observed  altitude  of  Rigel=     .    .    .    30?39t  O^S. 
Dip  of  the  hwizon  for  21  feet  «=    .    .     —     4. 24 

Rigel's  apparent  altitude  =       .    .    .    30?34:36rS. 
Refraction  = —     1.37 

Rigel's  true  altitudes 30?32:59fS.    ^ 

Rigel's  meridional  zenith  distance   s      59?  27'   KN. 
Rigel's  reduced  declination  s=    .    .    •      8.24.35    S. 

Latitude  of  the  place  of  observation  =     5 1  ?  2  ( 26rN. 

Note. — The  principles  upon  which  the  above  rule  is  founded,  are  given 
in  ^^  the  Young  Navigator's  Guide  to  the  Sidereal  and  Planetary  Parts  of 
Nautical  Astronomy,"  between  pages  98  and  105. 


Problem  V. 

Given  tlie  Meridimal  Altiiude  of  a  Celestial  Object  observed  below  iJie 
Pole,  to  find  the  Laiiiude  of  the  Place  of  Observation. 

Rule. 

Find  the  true  altitude  of  the  object,  as  before ;  to  which  let  the  polar 
distance  of  that  object,  or  the  complement  of  its  corrected  declination,  be 
added,  and  the  sum  will  be  the  latitude  of  the  place  of  observation,  of  the 
same  name  with  the  declination. 

Example  I. 

June  20th,  1825,  in  longitude  65?  W.,  the  meridian  altitude  of  the 
sun's  lower  limb,  observed  below  the  pole,  was  9?12l,  and  the  height  of 
the  eye  20  feet;  required  the  latitude  ? 

Observed  altitude  of  the  sun's  lower  limb  =     .    9?12'  OT 

Sun's  semi-diameter  =     .     ^5'46^1     ^     ^ 

Dip  ofthe  horizon  for  20  feet=4. 17    J^^^*-^     +11.29 


Apparent  altitude  of  the  sun's  centre  =     •    .    9?23'29^ 
Refractions     5^34r 
Parallax      s     0.    9 


I  Difference  =3   ...    —    5.25 


True  meridian  altitude  below  the  pole  s     •     •    9?18^  4r 
Sun's  corrected  polar  distance,  or  co-declination =66. 32. 17  N. 

Latitude  ofthe  place  of  obeervation  =s   •    .    .  75?50C21fN« 

Digitized  by  VjOOQ IC 


OF  FINDING  THE  ULTITUBB  BT  THB  NORTH  POLAR  STAR,        337 

Example  2. 

June  1st,  1825^  in  longitude  90?  £.^  at  11^26r40t  apparent  time^  the 
observed  altitude  of  Capella^  when  on  the  meridian  below  the  pole^  was 
11?48^  and  the  height  of  the  eye  above  the  level  of  the  sea  25  feet; 
required  the  latitude  ? 

Observed  altitude  of  Capella  as 11?48<  Of 

Dip  of  the  horizon  for  25  feet  = —  4. 47 


Capella's  apparent  altitude  ==•     ••     .     ••     •     lI?43n3T 
Refraction  = —  4. 29 


Capella's  true  meridian  altitude  below  the  pole  ss     ll?d8M4f 
Capella's  corrected  polar  distance^  or  co-declination=44. 1 L  28  N. 

Latitude  of  the  place  of  observation  s     ....    55?50U2rN. 

Eemarks.^^1^  When  the  polar  distance  or  co-declination  of  a  celestial 
object  is  less  than  the  latitude  of  the  place  of  observation  (both  being  of 
the  same  name),  such  celestial  object  will  not  set,  or  go  below  the  horizon 
of  that  place  :  in  this  case,  the  celestial  object  is  said  to  be  circumpolar^ 
because  ii  revolves  round  the  pole  of  the  equator^  or  equinoctial,  without 
disappearing  in  the  horizon. 

2.  If  12  hours,  diminished  by  half  the  daily  variation  of  the  sun's  right 
ascension,  be  added  to  the  apparent  time  of  the  superior  transit  of  k  fixed 
star,  it  wU  give  the  apparent  time  of  its  inferior  transit  over  the  opposite 
meridian;  that  is,  the  apparent  time  of  its  coming  to  the  meridian  below 
the  pole. 

3.  The  least  altitude  of  a  circumpolar  celestial  object  indicates  its  being 
on  the  meridian  below  the  pole. 


Probubm  VI. 

Given  the  AltUude  of  the  North  Polar  Star,  taken  at  any  Hour  qfthe 
Night,  to  find  the  Latitude  of  the  Place  of  Observation. 

Although  the  proposed  method  of  finding  the  latitude  at  sea  is  only 
applicable  to  places  situate  to  the  northward  of  the  equator,  yet,  since  it 
can  be  resorted  to  at  any  time  of  the  night,  it  deserves  the  particular  atten* 
tion  of  the  mariner. 


Digitized  by 


Google 


388  NAUTICAL  ASTRONOMY, 

Of  all  the  heavenly  bodies^  the  polar  star  seems  best  calculated  for  find- 
ing the  latitude  in  the  northern  hemisphere  by  nocturnal  observation; 
because  a  single  altitude^  taken  at  any  hour  of  the  night  by  a  careful 
obaerveri  will  give  the  latitude  to  a  sufficient  degree  of  accuracy^  provided 
the  apparent  tinpe  of  observation  be  but  known  within  ^^few  minutes  of  the 
truth :  however,  an  error  in  the  apparent  time,  even  as  considerable  as  20 
minutes,  will  not  affect  the  latitude  to  the  value  of  half  a  minute,  when 
the  polar  star  is  on  the  meridian,  either  above  or  below  the  pole ;  nor  will 
it  ever  affect  the  latitude  more  than  about  81  minutes,  even  at  the  star's 
greatest  distance  from  the  meridian..  But,  as  it  is  highly  improbable,  in 
the  present  improved  state  of  watches,  that  the  apparent  time  at  the  ship 
can  ever  be  so  far  out  as  five  minutes,  the  latitude  resulting  from  this 
method  will^  in  general^  be  as  near  to  the  truth  as  the  common  purposes 
of  navigation  require. 

RULB. 

To  the  Mn's  right  ascension,  as  given  in  the  Nautical  Almanac,  or  in  Table 
XII.  (reduced  to  the  meridian  of  the  place  of  observation,  by  Problem  V., 
page  298,)  add  the  apparent  time  of  observation ;  and  the  sum  (rejecting 
24  hours,  if  necessary,)  will  be  the  right  ascension  of  the  meridian,  or 
mid-heaven )  with  which  eater  Table  X.,  and  take  out  the  corresponding 
correction.  Find  the  true  altitude  of  the  star,  by  Problem  XVIL,  page 
327 ;  to  which  let  the  correction,  so  found,  be  applied  by  addition  or 
subtraction,  according  to  the  directions  contained  in  the  Table,  and  the 
sum  or  difference  will  be  the  approximate  latitude.  • 

Enter  Table  XL,  with  the  approximate  latitude,  thus  found,  at  top  of 
the  page,  and  the  right  ascension  of  the  meridian  in  one  of  the  side 
columns ;  in  the  angle  of  meeting  will  be  found  a  correction,  which,  being 
applied  by  addiiion  to  the  approximate  latitude,  will  give  the  true  latitude 
of  the  place  of  observation* 

iZemorft.— Since  the  corrections  of  the  polar  star's  altitude,  in  Table  X., 
have  been  computed  for  the  beginning  of  the  year  1824,  a  reduction  there- 
fore becomes  necessary,  in  order  to  adc^t  them  to  subsequent  years  and 
parts  of  a  year.  The  method  of  finding  this  reduction  is  illustrated  in 
exampkfis  1  and  2,  pages  17  and  18. 

Example  h 

January  2d»  1825,  in  longitude  60?  west,  at  8M0r40!  apparent  time, 
the  observed  altitude  of  the  polar  star  was  52?15^20f,  and  the  hdght  of 
the  eye  above  the  level  of  the  sea  16  feet ;  required  the  latitude  ? 


Digitized  by 


Google 


OF  FINDING  THE  LATITUDB  BT  THB  NORTH   POLAR  STAR.       839 

Sun's  reduced  right  ascension  ss 18^53?58t 

Apparent  time  of  observation  = 8.10.40 

Right  ascension  of  the  meridians:     •...«.«.      3*  4?38! 

Correction  of  altitude,  Table  X.^  answering  to  3?  s».   «    •       1?94'  16f 
Proportional  part  to  4T38!  of  right  ascension  s        •    .    f       —  1.    I 
Annual  var.  of  correction  =  13^.  67 ;  which  x  by  1  year,  gives  —  0. 14 

Correcticm  of  altitude,  reduced  to  time  of  observation  =  •  1  ?  23 '  1 T ; 
which  is  iubtracHve,  because  the  right  ascension  of  the  meridian  falls  in 
one.  of  the  left-hand  columns. 


Observed  altitude  of  the  polat  star  xs    . 
Dip  of  the  horizon  for  16  feet  s     .    « 

Apparent  altitude  of  the  polar  star  ss   . 
Refraction  s       : '  • 


True  altitude  of  the  polar  star,  ss     •    • 
Correction  from  Table  X.,  ans.  to  3!4?38 

Approximate  latitude  s     ,    .    .    «    • 
Correction  of  ditto  from  Table  XI.  s  . 

Latttilde  of  the  place  of  obsenration  s 
Example  2. 


*i 


52? 15 ^20? 

-  8.50 

52?II{3df 

-  0.44 

S2?10M6f 
-  ].23.    1 

60?47'45rN, 
+  0^28? 

5Q?48U8TN. 


January  1st,  1830,  in  longitude  75?  W.,  at  9^3?  i^parent  time,  let  the 
olMerred  altitude  of  the  north  polar  star  be  19?  15',  and  the  height  ef  the 
eye  above  the  level  of  the  horizon  28  feet;  reqtured  the  latitude? 

Sun's  R.A,,  Table  XII.,  reduced  to  ^ven  times  18)49? 
Apparent  time  of  observation  =:  ,    .    .    .   \    9.  S 

Rig^t  ascension  of  the  meridian  =  .    .    .    .    8*SS? 

Correction  ofaltitudp, Table  X.,  answering  to  8*50?  s=  .    .     U\V.3K  * 
Proportional  part  to  2  T  of  right  ascension  =      ..•••—  0. 34 
Annual  var.  of  correction^  1 0*' .  06,  which  X  by  6  years,  gives      —1.1 

Correction  of  altitude,  reduced  to  time  of  observation,  =     .     1?  9' 57^  I 
which  is  subtractive,  because  the  right  ascension  of  the  meridian  falls  in 

one  of  the  left-hand  columns. 

z2 


Digitized  by 


Google 


340 


NAUTICAL  ASTRONOMY. 


Observed  altitude  of  the  polar  star  =  • 
Dip  of  the  horizon  for  23  feet  =       .    • 


Apparent  altitude  of  the  polar  star 
Refraction  ss 


True  dititude  *of  the  polar  star  =s      .     • 
Correction  of  altitude  from  Table  X.  = 

-Approximate  latitude  =  •  ..•    •     •    • 
Correction  of  ditto.  Table  XI.  s     •    • 

Latitude  of  the  place  of  observation  ss 


.    i9?i5ror 

.  .  —  4.36 


.     19?10124r 
.       -  2.43 


.    199  7'4ir 
-1.   9.57 


•    17?57U4rN. 
.       +  0. 15 


.    17?57^59rN. 


Example  3. 

Let  the  true  altitude  of  the  north  polar  star,  January  Ist,  1854,  be 
50?5Uir^  and  the  right  ascension  of  the  meridian  17 •  13?  i  required  th^ 
latitude  ? 

True  altitude  of  the  north  polar  stars 50"^  5'4K 

Cor.  from T«ai.X.,  answ.to  17M0T=0?44^24n 

Proportional  part  to  as     .    .      3=   —  1.   8    /Addit.= +0.41.36 

Annual  var.=s  -3''.  34x30  years  =    -  1.40  J 


Approximate  latitude  s 50?47-17^N. 

Correction  of  ditto  from  Table  XL,  answ.  to  17  *  13?  =    .       +  1  •  17 

True  latitude,  as  required  s  •     • .    50?48'34?N. 

Noie.^-The  true  latitude,  computed  with  the  most  rigorous  degree  of 
accuracy,  by  spherical  trigonometry,  is  50?48'13^  N.3  the  difference, 
therefore,  between  the  true  spherical  latitude,  thus  deduced,  and  that 
resulting  from  Tables  X.  and  XI.,  as  above,  is  only  21  seconds  in  the  long 
period  of  30  years :  hence  it  is  evident,  .that  the  latitude  may  1)e  always 
•determined  by  mea^s  of  those  Tables,  to  every  degree  of  exactness  desir- 
lible  in  most  nautical  operations. 

The  elementajy  principles  of  computing  the  latitude  by  an  altitude  of 
the  north  polar  star,  are  given  in  "  the  Young  Navigator's  Guide  to  die 
Sidereal  aitd  Planetary  Parts  of  Nautical  Astronomy,'"  between  pages  144 
and  156,  where  a  diagram  may  be  seen,  illustrative  of  the  star's  apparent 
fnoHon  round  its  orbit. 


Digitized  by 


Google 


OP  FIKDIM6  TttB  LATTTDDB  BT  BOVBLB  ALTITTTBBS.  341 


Paoblbm  VIL 

Given  the  Latitude  by  Account^  the  Sun's  DecUnation,  an^  two  observed 
Altitudes  of  its  lower  or  upper  Limby*the  elapsed  Time^  and  the  Course 
and  Distance  nm  between  the  Observations;  Jo  find  the  Latitude  of 
the  Ship  at  the  Ime  qf  ObservaUm  qf  the  greatest  Altitude. 

Ruus. 

'  To  reduce  the  least  Altitude  to  what  it  would  be,  if  taken  at  the  PlAce 
where  the  greatest  Altitude  was  observed :— -    ' 

'  Find  the  angle  contained  between  the  ship's  coiurse,  (corrected  for  lee- 
way, if  any,)  and  the  sun's  bearinjp  at  the  time  of  taking  the  least  altitude ; 
with  which,  if  less  than  8,  or  with  what  it  wants  of  16  points  if  it  be  more 
than  8,  ester  the  general  Traverse  Table,  and  find  the  difference  of  latitude 
corresponding  thereto  and\he  distance  made  good  between  the  observa- 
tions, which  call  the  reduction  of  altitude* 

Now,  if  the  kast  altitude  be  observed  in  the  forenoon,  the  reduction  of 
altitude  is  to  be  applied  thereto  by  addition  when  the  above  angle  is  less 
than  8  points,  but  by  subtraction  when  it  is  tnore  than  8  points  3  the  sum, 
or  difference,  will  show  what  the  less  altitude  would  be  if  observed  at  the 
same  place  with  the  greater  altitude.  Again,  if  the  less  altitude  be 
observed  in  the  afternoon,  a  contrary  process  is  to  be  observed ;  viz.,'  the 
reduction  of  altitude  is  to  be  subtracted  therefrom,  when  the  above  angle 
is  less  than  8  points,  but  to  be  added  thereto  when  it  is  greater. 


To  compute  the  Latitude:— 

Reduce  the  sun's  declination  to  the  time  and  place  where  the  greatest 
dtitude  was  observed;  then,  to  the  log.  secant  of  the  latitude  by  account, 
add  the  log.  secant  of  the  corrected  declination ;  the  sum,  rejecting  20 
from  the  index,  will  be  the  logarithmic  ratio. 

To  the  log.  ratio,  thus  found,  add  the  logarithm  of  the  difference  of  the 
natural  co-versed  sines  of  the  two  corrected  altitudes,  and  the  logarithm  of 
the  half-elapsed  time  (Table  XXX.) ;  the  sum  of  these  three  logarithms 
will  be  the  logarithmic  middle  time.  Find  the  time  corresponding  to  this 
in  Table  XXXI. ;  the  difiS^rence  between  which  and  the  half-elapsed  time 


Digitized  by 


Google 


342  NAUTICAL  A8TROVOMY. 

will  be  the  time  from  noon  when  the  grieatest  altitude  was  observed.* 
From  the  log.  rising  (Table  XXXII.),  answering  to  this  time,  subtract  the 
log.  ratio ;  and  the  remainder  wiH  be  the  logarithm  of  a  natural  number, 
which,  being  subtracted  from  the  natural  co-versed  sine  of  the.  greatest 
altitude,  will  leave  the  natural  versed  sine  of  the  sun's  meridional  zenith 
distance ;  to  which  let  the  corrected  declination  be  applied  by  addition  or 
subtraction,  according  as  it  is  of  the  same  or  of  a  contrary  name :  and  the 
sum,  or  difference,  will  be  the  latitude  of  the  ship  at  the  time  that  the 
greatest  altitude  was  taken ;  which  may  be  reduced  to  noon,  by  means  of 
the  log,  if  necessary. 

If  the  latitude,  thus  found,  differ  considerably  from  that  by  account,  the 
operation  must  he  repeated^  using  the  computed  latitude  in  place  of  that  by 
account,  until  the  latitude  last  found  agrees  nearly  with  the  latitude  used 
in  the  computation* 

Remarks. — 1.  Since  this  method  is  only  an  approximation  to  the  truth, 
it  requires  to  be  used  under  certain  restrictions ;  vis.,  the  observations  must 
bt  tak^n  between  nine  o'clock  in  the  fortnoon^  and  three  in  the  afterno<Mi. 
If  both  observations  be  in  the  forenoon,  or  both  in  the  afternoon,  the 
elapsed  time  must  not  be  less  than  the  distance  of  the  observation  of  the 
greatest  altitude  from  noon.  If  one  observation  be  in  the  forenoon,  and  the  . 
other  in  the  afternoon,  the  elapsed  time  must  not  exceed  four  lioura  and  a 
half;  and,  in  all  cases^  the  nearer  the  greater  altitude  is  lo  noon,  the  better. 

2.  If  the  sun's  meridional  zenith  distance  be  less  than  the  latitude^  the 
limitations  are  still  more  contracted.  If  the  latitude  be  double  the  meridian 
zenith  distance^  the  observations  must  be  taken  between  half-past  nine  in 
the  forenoon  and  half-past  two  in  the  afternoon ;  and  the  elapsed  time 
must  not  exceed  three  hours  and  a  half.  The  observations  must  be  taken 
still  nearer  to  noon,  if  the  latitude  exceeds  the  meridian  zenith  distance  in 
a  greater  proportion. 

Esample  1. 

At  sea,  January  9th,  1825,  in  latitude  50"?  121  N.,  by  account,  and  lon- 
gitude 30?  IOC  W.,  at  21  ^30T0!  apparent  time>  the  observed  altitude  of  the 
sun's  lower  limb  was  10?  27 -30?,  and  the  bearing  of  its  centre,  by  azimuth 
compass,  S.E.  f  S.3  and  at  23^  lOTlO!  the  observed  altitude  was  17?6C40r« 
and  the  height  of  the  eye  above  the  level  of  the  sea  20  feet ;  the  ship's 
course  during  the  elapsed  time  was  S.S.E.^  at  the  rate  of  10  knots  an  hour; 
required  the  latitude  of  the  ship  at  the  time  of  observing  the  greater  alti- 
tude? 

*  When  the  middle  time  is  §preater  than  the  half-elapsed  time,  hoth  olMsrvaUons  will  be 
on  the  same  side  of  the  meridian  \  otherwise,  on  different  sided. 


Digitized  by 


Google 


OF   FINDING  THB  LATITUDB  BY  DOUBLE  ALTITUDBS.  ii$ 

Sun's  bearing  at  let  observation  sb  S.EL  |  S.^  or  3|  points. 
Ship's  course  = S.S.E.9        or  2  points. 

Contained  angle  = 1|  point. 


Time  elapsed  between  the  observations  =  1  *40T10!.— And, 

Asl^   :   lOr  ::   lUOriOt  :    16U2r,  or  17  miles  nearly  =  the  distance 

run  between  the  two  observations. 

Now,  to  course  1|  point,  and  distance  17  miles,  the  difFerence  of  latitude 
is  16. 5  miles,  the  reduction  of  altitude;  which  is  additioe  to  the  least  alti- 
tude/because  the  contained  angle  is  less  than  6  points,  and  the  observation 
made  in  the  forenoon. 

Time  of  observing  the  greatest  altitude  =      .     23  MOT  1 0 ! 
Longitude  SOUO^W.,  in  time  =     ...      -f  2.   0.40 

Greenwich  time  past  noon  of  the  10th  Jan.  =s       1M0?50! 

Sun's  decHnatipn  at  noon,  January  10th  &       .    •    2 1  ?  57 '  50^  S. 
Correction  of  declination.  Table  XV.,  for  1  \  10?50!  ^  -<«  0*  28 

Son'a  corrected  declination  ae   • 21?£7«22?S. 

Fint  observed  altitude  of  the  sun's  lower  lunb  =  10?  27 '  SOT 

Sun's  semi-diamcter  =      .     16'  ISr  I  ^.^  , ^     , 

,4  jy  jDiftaa    +  12.    1 


Dip  of  the  horiz.  for  20  feet : 


II  ■   m 


Apparent  altitude  of  the  sun's  centre  s:     •    •  10?S9'SI T 

Refractions:  4'56TI-..^  .    .- 

Parallax   =     0.   9   }l>^«fe'«^<^'      •    •    •  '  *^*^ 

Reduction  of  altitudes      .•«#«..  'f  16*80 


Reduced  altitude  s ,    10?51M4r 

Second  observed  altitude  of  the  sun's  lower  limb  as  17?  6140? 
Sun's  semi-diameter  s=  .    •     16' 18?  I    .^_^        x  12     I 
Dip  of  the  horiz.  for  20  feet  =   4.17   P*^*-*     +  *^* 


Apparent  altitade  of  the  nm's  centre  3s      •    •    •    17!18«41? 
Refraction  =    3ar  }  ^.^  «  ko 

RiraU«c»   .    0.8   5DiflFerence=     ....       -  2S3 

True  altitude  of  the  sun's  centre  =».....     17U5'.48?  S. 

Digitized  by  VjOOQ IC 


344  KAtrriCAL  astronomy. 

Latitude  by  account  =  50?  12'  OrN.Log.secslO.  193746 
Hme  of  obfl.         Altitude.         Nat  co-v.  sine.     Red.  dec 
21?30r  0!     10^51U4r      811695.  2 l?57'22fS. Log. 8ec.=  10.032700 

23.10-10       17.15.48        703190.         Log.  ratio  =     .    .    0.226446 


lUOr  10'.  elap8.tiine.Diff.=z  108505.         Log.  =  .    .    .    .    5.035450 


Ot50?  5!  half-elapfedtime.        Log.  half-elapsed  Uine  =     0.663950 
1. 39. 43  middle  time.  Log.  middle  time  =     .    .    5. 925846  . 


0M9T38!  time  from  noon  when  the  # 

greatest  altitude  was  taken.  Log.  rising  =     •     .    •    •    4. 368450 
Nat.  co-versed  sine  of  the  greatest  alt.    =     703190.  Log.ratio=0. 226446 


Natural  number  =     .4 13868.  Log.  =     4.142004 

Sun's  mer.z^.di8.=7 1  ?  54  i  OrN.  Nat.V.S.=689322 
Do.  reduced  dec.=  21. 57. 22  S. 


Latitude  of  shipss  49 ?56C38f  north.    And,  since  this  latitude  differs  so 
much  from  that  by  account,  it  will  be  necessary  to  repeat  the  cperatian. 

Computed  latitude  =     .    .    .    49?56^38r     Log.  secant  a    10.191427 
Reduced  declination  ^      .    .    21 .  57. 22      Log.  secant  s    10. 032700 


Log.  ratio   =      0.224127 

Diff.  of  nat.  co-versed  sinesss  108505.     Log.  =s 5.035450 

Half-elapsed  time  =    .    •    Ot50r5!     Log.  half-elaps.  time  =  0. 663950 

Middle  times    .    .    .    .    1.39.9      Log.  middle  time  =    5.923527 

Time  from  noon  when  great- 
est altitude  was  taken  s    0M9?4 !     Log.  rising  a  ...    4. 358520 
Nat  co-versed  sine  of  the  greatest  alt.  s     703190.  Log.ratio=0. 224127 

Natural  number  s     13627.  Log.  s  .    4.134393 

Sun's  mer.  z.dis.=71  ?54C52rN.  Nat.V.S.=:689563. 
Sun's  red.  dec.  =  21. 57. 22  S. 


Lat.  of  the  ship  s49?57'.30r  north.  And,  since  this  latitude  differs 
only  52  seconds  from  the  last,  it  may,  therefore,  be  esteemed  as  the  true 
latitude. 

iVbte.-»The  correct  latitude^  by  spherical  trigonometry,  is  49?56^0r 
north. 


Digitized  by 


Google 


OF  FINDING  THB  LATITtlDB  BT  J>OUfiLB  ALTITUDBS.  S45 

Example  2. 

At  sea^  April  14th,  1825,  in  latitude  43?47'  S.,  by  account,  and  longi- 
tude 60?  25^  E.,  at  23^20^40!  apparent  time,  the  observed  altitude  of  the 
sun's  lower  limb  was  35?54^,  and  at  2*  lOTlO'  apparent  time,  April  15th, 
the  observed  altitude  of  that  limb  was  28?42'.  15?,  and  the  bearing  of  the 
sun's  centre,  by  azimuth  compass,  N.W. }  N. ;  the  height  of  the  eye  above 
the  level  of  the  horizon  was  24  feet,  and  the  ship's  course  during  the  elapsed 
time  S.W.,  at  the  rate  of  9  knots  an  hour ;  required  the  latitude  of  the  ship 
at  the  time  of  observation  of  the  greater  altitude  ? 

Sun's  bearing  at  2d  observation  =  N.W. }  N.,  or  ±:  3i  points. 
Ship's  course  = S.W..     or  s=  4    points. 

Contained  angle  = 8|  points. 

Time  elapsed  between  the  observations  =  2*49?30!      And, 
Asl*  :  9V  ::   2*49?30!  :  25^26?=  the  distance  made  good  between 

the  observations. 

Now,  to  course  7i  points,  and  distance  25  miles,  the  difference  of  latitude 
is  3.  7  miles,  the  reduction  ofaUUitde;  which  is  additive  to  the  least  altitude, 
because  the  contained  angle  is  less  than  8  points,  and  the  observation  was 
made  in  the  afternoon. 

Time  of  observing  the  greatest  altitude  s=    .     .     ,     23*20^40! 
Longitude  60? 25  ^  E.,  in  time  = 4.    1.40 


Greenwich  time  past  noon,  April  14th,  =     .    .    •     19M9?  0! 

Sun's  declination  at  noon,  April  14th,  s      .    .      D?24n5?N. 
Correction  from  Table  XV.,  for  \9t  19T0!  =     .     +17. 19 


Sun's  reduced  declination  = 9?4 1'34?N. 


First  observed  altitude  of  the  sun's  lower  limb  =  35?54'  OrN. 
Sun's  semi-diameter  = 
Dip  of  the  horiz.  for  24  feet  - 


.     .     15'58r).    ^ 
eet=     4.42   i^'^'^    +  "-^^ 


Apparent  altitude  of  the  sun's  centre  s  •    ,    .    36?  5n6VN. 

Refraction  =  l'18r  I  __  ,   ,, 

„     „  ^    -    {Difference  =5s     «    •    •    •     —  1.11 

Parallax,  s    0.   7    ^ 

True  altitttdeoTthe  sun's  centres     .    .    .    .    36?  4'  srN. 

Digitized  by  VjOOQ IC 


346  NAUTICAL  ASTftONOMY. 


Second  observed  altitude  of  sun's  lower  limb  =  28M2C 15? 
Sun's  semi-diameter  =     .     15^56? 
Dip  of  the  horiz.  for  24  feet=  4. 42 


JDiff.  =    +   11.16 


Apparent  altitude  of  the  sun*s  centre  =    •    •    28?5d^3lT 

Refraction  =  l'43r 

Parallax  =      0.    8 

Reduction  of  altitude  =2 +3.42 


j  Difference  =      •    •    •       —  1 .  35 


Reduced  altitude  »      « 28?55'.38r 

Latitude  by  account  »  43?47'  O^Log.sec.  =::10«  141486 

Time  of  obs.  Altitude.        Nat.  co-v.  slue.      Red.  dec. 

23*20r40*       36?  V  51      411255      9941  ^34rLog. sec,  =10, 006244 


2.10.10        28.55.38        516302    Log.  ratio  =    •    .     .    0.147730 


2. 49. 30  elaps.  time.  Diff.=105047     Log.  =       ....    5. 021384 


1 1 24T45 !  half-elapsed  time         .     .    Log.  half-elaps.  time  =:  0. 441990 
0.47.   8   middle  time      ....    Log.  middle  time  s      5.611104 


0*37  "37*  time  from  noon  when  the 

greatest  altitude  was  taken.    Log*  rising  ss    •    •      4. 128390 
Natural  co-vened  sine  of  the  greatest  alt.  ss  41 1255Log.rakio80. 147730 


Natural  number  s    9564Log.  si     3. 980660 


Sun's mer.z.  dist.=53?15'  4?S.Nat.ver.S.=401691 
Sun's  red.  dec.     =  9.41.34  N. 


Lat  of  the  ship  s  43^33^30?  S.    But^  since  this  latitude  differs  so  much 
from  that  by  accounti  it  becomes  necessary  to  repeat  the  operation. 

Computed  latitudes  .    .    43?33^30rS.    Log.  secant  sz     10.139858 
Reduced  declination  =    .      9.41.34  N.    Log.  secant  =     10.006244 


Log. ratios  .      0.146102 
Diff.  of  nat.  co-versed  sines  =    105047  Log.  s      •    •      5.021384 

Half-elapsed  time  =     .    .    1*24T45!         Log.i-el^^s.timesO.  441990 


Middle  time  »    »    .    .    ,    0,46,57  Log.  middle timesS*  609476 

/Google 


Digitized  by ' 


OF  FINDING  THB  LATITUPB  BY  TBB  ALTITUDES  OF  TWO  STARS.    347 

Time  from  noon  when  the 

greatest  alt.  was  taken  s     0^37*48  f  Log.  rising  =    .    4*  132610 

Nat.  co-vers.  sine  of  the  greatest  altitude  =  411255Log.ratio3:0. 146102 

Natural  number  =       9694Log.  =  .    3. 986508 

Sun's  meM.dis.tt33rl4^30rS.sNat.V.  S.si401561 
Sun's  red.  dec.  a:»  9.41.34  N. 


Latitude  =  •  43?32^56''  south.  And,  since  this  latitude  only  differs 
34  seconds  from  the  last,  it  may  be  considered  as  being  the  latitude  of  the 
ship  at  the  time  of  observation  of  the  greater  altitude.  The  correct  lati- 
tude, however,  by  spherical  trigonometry,  is  43?29'30^  south :  hence  the 
method  by  double  altitudes,  even  after  repeating  the  operation^  differs  from 
the  truth  by  3  minutes  and  26  seconds. 

Noie^^The  method  of  Sliding  the  latitude  by  double  altitudes,  being  a 
very  tedious  and  indirect  operation^  and  generally  a  very  inaccurate  one, 
uftless  the  llmiutidni  pointed  out  in  the  remarks  (page  342)  are  strictly 
attended  to,  no  notice,  therefore,  would  have  been  taken  of  it  in  this  work, 
w«r«  it  tiot  fcr  the  purpose  of  giving  the  most  ample  illustration  of  the 
general  use  of  the  IVbles.  And,  notwithstanding  what  has  .been  said  in 
favour  of  double  altitudes  by  ^Aeoretteol  torifer^,  this  method  of  finding  the 
latitude  at  sea  is  evidently  far  from  being  one  of  the  most  advantageous  in 
pf  actteal  navigation  :  for  the  operation,  besides  being  rather  circuitous, 
requires  a  considerable  portfoaof  time  to  go  through  with  it  correcdy; 
a^d,  after  all)  it  fVequentiy  happens,  that  although  every  seeming  precau^ 
tion  has  been  taken,  the  mariner's  hopes  are  disappointed  in  the  result. 
We  will  now  proceed  to  a  more  direct  and  universal  method  of  finding 
the  latitude^  either  at  sea  or  on  shore. 


PaoBLBM  VIII. 

GwenAke  Altitudes  of  two  known  fixed  Stars  observed  at  the  same  instant^ 
af  otiy  Time  ^ike  Night,  to  find  the  Latitude  qfthe  Place  of  Obeerv- 
tftimi,  tfidi}ieitcfett^  qfthe  LatUnde  by  Jccounif  the  Idmgitside,  or  the 
Af^fOfent  Time* 

In  tlte  preceding  problems  for  finding  the  latitude  (the  two  last  excepted), 
tlie  meridional  aHitudes  of  the  celestial  objects  were  the  principal  elements 
under  consideration  :  however,  since  it  fj^equently  happens  that,  in  conse- 


Digitized  by 


Google 


348  NAUTICAL  AfiTRONOMT, 

quence  of  the  interposition  of  clouds,  or  other  causes,  the  altitudes  of  the 
heavenly  bodies  cannot  always  l)e  taken  at  their  respective  times  of  transit, 
the-present  problem  is,  therefore,  proposed,  which  possesses  the  pecuKar 
advantage  of  enabling  the  mariner  to  determine  the  position  of  his  ship, 
with  respect  to  latitude,  by  the  altitudes  of  two  known  fixed  stars,  observed 
at  the  same  instant  and  at  any  hour  of  the  night,  either  before  or  after  their 
passing  the  meridian,  and  independent  of  the  latitude  by  account,  the  lon- 
gitude, or  the  apparent  time  of  observation.  Nor  will  the  mariner^  in  this 
method,  be  subjected  to  the  necessity  of  repeating  the  operation^  or  of  puz- 
zling himself  with  a  variety  of  cases  and  corrections,  in  finding  an  approx- 
imate latitude. 

Rule. 

Let  the  altitudes  of  two  stars  be  observed,  at  the  same  moment,  whose 
computed  spherical  distance  asunder  is  given  in  Table  XLIV. ;  and  let  those 
observed  altitudes  be  reduced  to  the  true  by  Problem  XVIL^  page  327. 
Take  the  right  ascensions  and  declinations  of  the  two  stars^  and  also  their 
computed  spherical  distance,  from  Table  XLIV.,  and  let  these  be  reduced, 
respectively,  to  the  night  of  observation.  Let  the  star  which  is  adjacent 
or  nearest  to  the  elevated  pole,  be  distinguished  by  the  letter  A,  and  that 
which  is  remote^  or  farthest,  by  the  letter  R.— Now, 

To  the  log.  sine  of  the  tabular  distance  between  the  two  stars,  add  the 
log*  secant  of  the  decl^iation  of  the  star  A,  and  the  log.  half-elapsed  time 
of  the  difference  of  right  ascension  ;  the  sum,  rejecting  20  frcmi  the  ind^x, 
will  be  the  log.  half-elapsed  time  of  arcA  thejirst. 

From  the  natural  co-versed  sine  of  the  altitude  of  the  star  A^  subtract  the 
natural  co-versed  sine  of  the  sum  of  the  tabular  distance  between  the  sUrs 
and  the  altitude  of  the  star  R,  and  find  the  logarithm  of  the  remainder;  to 
which  add  the  log.  co-secant  of  the  tabular  distance,  and  the  log.  secant  of 
the  altitude  of  the  star  R ; — the  sum  of  these  three  logarithms,  abating  20 
in  the  index,  will  be  the  log.  rising  of  arch  the  second;  the  difference 
between  which  and  arch  the  first,  will  be  arch  the  third. 

To  the  log.  rising  of  arch  tlie  third,  add  the  log.  co-sines  of  the  declina- 
tion and  altitude  of  the  star  R,  and  the  sum,  abating  20  in  the  index,  will 
be  the  logarithm  of  a  natural  number ;  which,  being  added  to  the  natural 
versed  sine  of  the  difference  between  the  altitude  and  declination  of  the  star 
R,  when  its  polar  distance  is  less  than  90?,  or  to  that  of  their  sum  \riien 
it  is  more  than  90%  the  sum  will  be  the  natural  co-versed  sine  of  the 
latitude. 


Digitized  by 


Google 


/ 
OP  PIKDIN6  THE  LATITUDB  BY  THB  ALTITUDES  OF  TWO  8TAB8.    349 

Example  L 

January  Ist,  1825^  in  north  latitude^  the  true  altitude  of  the  star 
Alphard  was  16?0H2T^  and,  at  the  same  instant,  that  of  Regulus  waa 
27?14^8T  J  required  the  latitude  of  the  place  of  observation  ? 

A,orRegulu8'red.RA=9^59r  S!«and reduceddec.=  12?49' lOr « N. 
R, or  Alphard's ditto  =   9. 18. 59  and  reduced  dec.  =:    7. 54. 13  S. 


Tabular  distance  between  the  two  stars  =  22^59 '22r* 
DiflF.  of  right  asc,  =      0*40?  4!  Log.  half-elapsed  time  =:     0.759620 
Di8t.beUhe  two  8tars=22?59'22r  Log.  sine  =      ....      9.591690 
Dec.  of  star  A  =     .     12. 49. 10    Log.  secant  =   ....  10. 010962 


Arch  the  fir$t::z     .      It42r57'  Log.  half-elapsed  time  =     0.362272 

Di8t.beUhetwo8tars=22?59^22r  Log.  co-secant  =      .    .     10.408310 
Altitude  ofthe  star  R  =  16.  0.12    Log.  secant  ==      ...     10.017165 


Sum=     ....    38?59C34rNat.co-V.S.=370778 
Altitude  ofthe  star A=  27. 14.   8  Nat.co-V.S.=54235l 


Diff.  =  171573  Log.=5. 234449 


Arch  the  second   =:      3MSr27!  Log.  rising  =     ....    5.659924 
Arch  the  first  =:      •       1.42.57 


Arch  the  third  =  .  2*  5r30r  Log.  rising  =  .  .  .  5.165010 
Dec.  of  the  star  R  =:  7^54^  13rS.  Log.  co-sine  =  .  .  .  9. 995855 
Altitude  of  ditto  =       16.   0. 12       Log.  co-sine  =:     .     .    .    9. 982835 


Sum  =  23?54'.  25rNat.  vers.  S.=  085795  

Natural  number  =     ....     139220  Log.=:5. 143700 


True  latitude  =  •    50948!  13rN.Nat.co-V.S.=225015 

Example  2. 

January  1st  1825^  in  north  latitude,  the  true  altitude  of  a  Arietis  was 
27?12'9T,  and,,  at  the  same  instant,  that  of  Aldcbaran  was  51M5!28r  5 
required  the  latitude  ofthe  place  of  observation  ? 


*  The  method  of  re duciDip  the  right  sseensioni^  dedioationsy  and  computed  spherical  dis- 
ofthe  itan,  to  a  giTea  period,  U  shown  in  the  explanation  to  Table  XLIV.,  pagell4. 


Digitized  by 


Google 


350  NAUTICAL  ASTRONOMT. 


True  spherical  distance  between  the  two  stars,  reduced  to  night  of 
observation  =  35?32:7r» 

A,or«Arietis'red.R-A.=  l»57ri9!*  reduced  dec,=:22?37'50^N.* 
R,  orAldebaran's  ditto=4 .  25 •  53      reduced  deer:  1 6.    8. 57  N. 


Diff.  of  right  asc.  =:  2^28^34!  Log.  half-elapsed  timers  0.219110 
Dist.bet.thetwo8tar8=35?32'.  7".  Log.  sine  =  ....  9.764329 
Dec.  of  star  A  =    ..    22.37.50    Log.  secant  =:       •    .    .     10,034796 


Arch  the  first  =      .      4^S4T  3!  Log.  half-elapsed  time  =:     0.018235 

Di8t.bet.thetwostars=35?32^  7?  Log.  co-secant  =      •    .    10.235671 
AltitudeofthestarR;=51.45.28    Log.  secant  ^      .    »    .     10.208S18 

Sum=:    •    .    .    .    87?17'.35^Nat.co.V.S.=001116 
AltitudeofthestarA=27. 12,   9  Natco.V.S.=:542863 


Difference  =  541747  Log.=5. 733796 

Arch  the  second    =      8*   1T33!  Log.  rising  =    •    .    .    .      6.177785 
Arch  the  first  =      .      4. 54.   3 


Arch  the  third  =  .  3 1  7"30!  Log.  rising  =  .  .  .  .  5.500250 
Dec.  of  the  star  R=  16?  8^57^  Log.  co-sine  =  .  .  .  9.982516 
Altitude  of  ditto  =      5 1 .  45 .  28    Log.  co-sine  =     .    .    ,      9. 79 1 682 

Difference  =      •    .    35  ?36  ^  3 1  TNat.  vers.  S.  =  1 86987 

Natural  number  =     188126  Log.=:5. 274448 

True  latitude  =    88?40C26^N.Nat.co.-V.S.=375113 


Example  3, 

March  Ist,  1825,  in  north  latitude,  the  true  altitude  of  Rigel 
27?9'7':',  and,  at  the  same  instant,  that  of  Sirius  28?55^391^  j  required  the 
latitude  of  the  place  of  observation  ? 

*  See  Note,  pa^  349. 

Digitized  by  VjOOQ IC 


OF  FINDING  THE  IJITITUDB  BY  THE  ALTITODES  OF  TWO  STARS,      851 


True  spherical  distance  between  the  two  given  stars,  reduced  to  night 
of  obser>'ation  =  23?40M3^  * 

A,orRiger8red.R.A.=:5*  6T  8!  reduced  dec.=  8?24^S5rS. 
R,orSiriu8'red.R,A.  =  6. 37.  26   reduced  dcc.=  16. 28. 58 


Diflf.  of  right  asc.  =     l*3iri8!  Log.  half-elapsed  time  =  0.411262 

Dist-bet.thetwostar8=23.40.40    Log.  sine  =      .    •     •     .  9.603786 

Dec.  of  the  star  A  =     8?24:35r  Log.  secant  =  ....  10. 004695 

Arch  the  first  =     .      4*5 1  r25 !  Log.  half-elapsed  time  s;  0. 019743 

Dist.bet.thctwo8tars=23?40U3r  Log.  co-secant  =  .    .    .  10.396200 

Alt.  of  the  star  R=:     28.55.39   Log.  secant  =.    .    .    ;  10.057877 

Sum    =       .    .    .     52?36:22rNatfco-V.S.=205520 
Alt.  of  the  star  A  =   27.   9.   7  Nat.co-V.S.=543648 


Difference  =  3^8128  Log.  =  5. 529081 

Arch  the  second   =     5t51'ri7'  Log.  riising  =     ....      5.988158 
Arch  the  first  =     .      4. 5 1 .  25 


Arch  the  third  =  .  0*59^52!  Log.  rising  =  ....  4.530500 
Dec.  of  the  star  R  =  16928^58^  Log.  co-sine  =  .  .  .  .  9. 981775 
Altitude  of  ditto  =     28. 55. 39     Log.  co-sine  =  .     .     .     .      9. 942123 


Sum=    ....    45?24^37^Nat.ver8.8inc=:297975 


Natural  number  ^     28471  Log.=  4. 454398 


True  latitude  =  42?20^3KN.  Nat.co.V.S.326446 


*  The  distance  between  Rigel  and  Sirius,  as  given  for  the  year  1822,  at  the  end  of  the 
Namical  Almanac  for  1825,  Table  II.,  is  23''40'35'%  aod  Uie  change  in  10  years  +  (K5". 
This  is,  evidently,  a  mistake ;  for  the  distance  between  those  two  stars,  at  the  beginning  of 
1832,  was  23«40'42'':  and,  since  th«  annual  variation  of  distance  is  —  0'^56,  the  change, 
therefore,  in  10  yean, is  —  0'  5'^6 ;  being  mbtr active  instead  of  addUwe,  A  similar  remark 
is  applicable  to  the  stars  Fonlalhaut  and  Achemar ;  for,  by  the  above-mentioned  Table,  it 
appears  that  the  distance  between  those  stars,  at  the  beginniDg  of  1822,  was  39^20",  and 
the  change  in  10  years  —  0^  1" :  whereas  the  true  distance,  at  that  period,  was  39^7'13" ;  and, 
since  the  annual  variation  of  distance  is  ^  0^^  17,  the  change,  therefore^  in  10  years,  is 
—  tf  Y\  7,  being  very  nearly  two  seconds  of  a  degree.  The  distances  aod  annual  variations 
of  the  remaining  stars  in  the  said  Table  wiU  be  found  equally  incorrect,  as  may  be  seen 
by  comparing  them  willi  those  contained  in  this  work.  Tabic  XUV. 


Digitized  by 


Google 


352  NAUTICAL  ASTRONOMY. 

Example  4. 

September  let,  1825,  in  south  latitude,  the  true  altitude  of  Fomalhaut 
was  63?6^  18r,  and,  at  the  same  instant,  that  of  Achernar  37?441  ISr  ; 
required  the  latitude  of  the  place  of  observation  ? 

True  spherical  distance  between  the  two  given  stars,  reduced  to  the  night 
of  observation,  =  89?7'13r 

A,orAchernar'sred.RA.=  l*3iri3!  reduced dec.==58e  7^27^8. 
R,or  Fomalhaut's  ditto=22. 48.   0    reduced  dec.=:30. 32. 38 


Diff.  of  right  ascensions  2U3T13!  Log.  half-elapsed  time  =  0.184770 
Dist..bet.thetwostars=39°10'.37r  Log.sine=  .  .  .  .  9.800523 
Dec.  of  the  star  A  =      58.   7.27     Log.  secant  =     .    •     .     10.277300 

Arch  the  first  =x    .     .      2*12r27!  Log.  half-elapsed  time  =   0.262593 


Dist  bet.  the  two  stars=39?  7 '  13^  Log.  co-secant  =    .     .     10. 200004 
Alt.  of  the  star  R    =     63.   6.18     Log.  secant  =    .     .     .     10.344518 


Sum= 102?13^3KNat.co.V.S.=022677 

Alt.  of  the  star  A  =     37.44. 18  Nat.co.V.S.=387944 


Difference  =  365267  Log.=5. 562610 


Arch  the  second  =     .       7^  4r59!  Log.  rising  =     .     .    .      6.107132 
Arch  the  first  =    .     .       2. 12.  27 


Arch  the  third  .  .  .  4*52r32!  Log.  rising  =  .  .  .  5.851160 
Dec.  of  the  star  R  =  30?32'38r  Log.  co-sine  =  .  .  .  9.935124 
Altitude  of  ditto  s     .    63.   6.18     Log.  co-sine  =  .     .     .      9.655481 


Difference  =s    .    .     .    32?33U0'/Natv.sine=  157182 


Natural  number  s  276544  Log.s  5. 441765 


True  latitude  =  34^29^27^  S.    Nat.co-V. S.=433726 

Note. — ^The  principles  from  which  the  above  method  is  deduced,  will  be 
found  in  ^^  The  Young  Navigator's  Guide  to  the  Sidereal  and  Planetary 
Parts  of  Nautical  Astronomy,''  between  pages  136  land  144. 


Digitized  by 


Google 


OF  FINDING  THE  LATITUBE  BY  THE  ALTITUDES  OF  TWO  STARS.     353 

Thus^  then,  is  the  mariner  proyided  with  a  direct  and  most  accurate 
m^^Aod  of  finding  the  latitude  at  sea;  and^  since  it  prevents  the  uncertainty 
and  confusion  arising  from  an  error  in  the  assumed  latitude,  or  that  by 
account,  and,  besides,  being  free  from  all  ambiguity,  restriction,  and 
variety  of  cases  whatever, — it  may,  therefore,  be  employed  with  a  certainty 
of  success,  at  any  hour  of  the  night,  whenever  two  known  fixed  stars  are 
visible.  Indeed,  if  the  altitudes  of  the  objects  be  determined  with  but 
common  attention,  the  latitude  resulting  therefrom  will  be  always  true  to 
the  nearest  second  of  a  degree,  without  the  necessity  of  rqfeating  the 
operation,  or  of  applying  any  correction  whatever  to  the  result. 

Remarks. — ^Although  it  is  at  all  times  advisable  for  two  observers  to 
take  the  altitudes  of  the  stars  at  the  same  moment  of  time,  yet,  should  one 
person  be  desirous  of  going  through  the  whole  operation  himself,  he  is  to 
proceed  as  follows ;  viz., — Let  the  altitude  of  one  star  be  taken,  and  the 
time  of  observation  noted  by  a  watch  that  shows  seconds ;  then  let  the 
altitude  of  the  other  star  be  observed,  and  the  time  noted  also ;  and  let 
the  altitude  of  the  first  observed  star  be  again  taken,  and  the  time  of 
observation  noted. 

Now,  find  the  difference  between  the  first  and  last  times  of  observation, 
and  the  altitudes  of  the  first  observed  star ;  and  find,  also,  the  difference 
between  the  first  time  of  observation  of  the  first  star,  and  the  time  of  ob- 
serving the  second  star.  Then  say,  as  the  interval  or  difference  of  time 
between  the  two  observations  of  the  first  star,  is  to  the  difference  of  altitude 
in  that  interval ;  so  is  the  interval,  or  difference  of  time  between  the  observ- 
ations of  the  first  and  second  star,  to  a  correction ;  which,  being  applied  by 
addition  or  subtraction,  to  the  first  observed  altitude  of  the  first  star, 
according  as  it  may  be  increasing  or  decreasing,  the  sum  or  difference  will 
be  the  altitude  of  that  star  reduced  to  the  time  that  the  altitude  of  the 
second  star  was  taken.  This  part  of  the  operation  may  be  readily  per- 
formed by  proportional  logarithms ; — see  example,  page  75.  The  interval 
between  the  observations  ought,  however,  to  be  as  much  contracted  as 
possible,  on  account  of  guarding  against  any  irregularities  in  the  change  of 
altitude. 

Caution. — In  order  to  guard  against  falling  into  any  error,  by  working  in 
an  imposrible  triangle,  it  will  be  advisable  to  make  choice  of  two  stars 
whose  computed  spherical  distance,  in  Table  XLIV.,  is  not  less  than  20?, 
and  difference  of  right  ascension  not  less  than  a  quarter  of  an  hour ;  and, 
since  the  Table  contains  an  extensive  variety  of  distances  and  differences 
of  right  ascension  greater  than  those  values,  the  mariner  can  never  be 
at  a  loss  in  finding  out  two  eligible  stars  for  observation.  The  distances 
in  that  Table  are  all  computed  to  the  greatest  degree  of  accuracy  3  and, 

2  A 


Digitized  by 


Google 


854  NAUTICAL  ASTRONOMY, 

notwithstanding  that  some  of  those  whieh  are  but  of  small  meoiuse 
ought  not  to  be  employed  in  the  determination  of  the  latitude  by  the 
above  method^  yet  they  will  be  found  extremely  useful  on  many  oceasions ; 
particularly  in  assisting  to  distinguish  the  stars  to  which  they  are  annexed^ 
when  the  latitude  is  to  be  inferred  from  their  meridional  ^dtitudes^  agree* 
aUy  to  Problem  IV.,  page  335. 


Problbm  IX. 

Owen  the  Latitude  by  Accmni,  the  Jltitude  of  the  Sun*8  loiper  or  upper 
Limb  observed  near  the  Meridian,  the  apparent  Time  of  Observation, 
,^nd  the  Longitiide  ;  to  find  the  true  Latitude. 

Since  it  frequently  happens  at  sea,  particularly  during  the  winter  months 
of  the  year,  that  the  sun's  meridional  altitude  cannot  be  taken,  in  conse- 
quence of  the  interposition  of  clouds,  fogs,  rains,  or  other  causes ;  and  since 
the  true  determination  of  the  latitude  becomes  an  object  of  the  greatest 
importance  to  the  mariner  when  his  ship  is  suling  in  any  narrow  sea 
trending  in  an  easterly  or  a  westerly  direction,  such  as  the  British  Channel; 
the  present  problem  is,  therefore,  given,  by  means  of  which  the  latitude 
may  be  very  readily  and  correctly  inferred  from  the  sun's  altitude  taken  at 
a  given  interval  from  noon^  within  the  following  limits  i  viz., — The  num" 
ier  of  minutes  and  parts  of  a  minute^  contained  in  the  interval  between 
the  time  of  observation  and  noon,  must  not  exceed  the  fmmber  of  degrees 
and  parts  of  a  degree  contained  in  the  object's  meridional  zenith  distance 
at  the  place  of  observation.  And^  since  the  meridional  zenith  distance  of 
a  celestial  object  is  expressed  by  the  difiference  between  its  declination  and 
the  latitude  of  the  place  of  observation,  when  they  are  of  the  same  name, 
or  by  their  sum,  when  of  contrary  names,  the  extent  of  the  interval  from 
noon,  within  which  the  altitude  should  be  observed^  may,  therefore,  be 
readily  ascertdned,  by  means  of  the  difiference  between  the  latitude  and 
the  declination,  when  they  are  both  north  or  both  south,  or  by  their  sum 
when  one  is  north  and  the  other  south  :  thus,  if  the  latitude  be  60  degrees, 
and  the  declination  23  degrees,  both  of  the  same  name,  the  interval 
between  the  time  of  observation  and  noon  ought  not  to  exceed  37  minutes; 
but  if  one  be  north  and  the  other  south,  the  interval  may  be  extended,  if 
necessary,  to  83  minutes  before  or^  after  noon.  The  altitude^  however^ 
may  be  taken  as  near  to  noon  as  the  mariner  may  think  proper ;  the  <mly 
restriction  being,  that  the  observation  must  not  be  made  without  the  above* 
mentioned  limits. 

The  interval  between  the  ^parent  time  of  observation  and  noon  must  be 
^curately  determined :  this  may  be  always  done^  by  means  of  a  chrouch- 
meter  or  any  well-regulated  watch  showing  seconds;  proper  allowanm 


Digitized  by 


Google 


LATITUDB  BY  AN   ilLTITTTDE  TAUN  MBAR  THB  MBRIDIAN.       955 

being  made  for  the  difference  of  time  answering  to  the  ehange  of  longi- 
tude^  if  any^  since  the  last  observation  for  determining  the  error  of  such 
watch  or  chronometer. 

Now,  if  the  sun's  altitude  be  observed  at  any  time  imihin  the  abcve-men" 
Honed  limits^  the  latitude  of  the  place  of  observation  may  then  be  deter- 
mined, to  every  degree  of  accuracy  desirable  in  nautical  operations,  by  the 
following  rule  ;  which,  being  performed  by  proportional  logarithms,  ren- 
ders the  operation  nearly  as  simple  as  that  of  finding  the  latitude  by  &i€ 
meridional  altitude  of  a  celestial  object. 

See  cflcplanation  to  Tables  LI.  and  LIL,  between  pages  188  and  149. 

RULB. 

Reduce  the  son's  declination  to  the  time  and  place  of  observation^  by 
Problem  V.,  pag6  298 ;  and  let  the  observed  altitude  of  the  sun's  lower  or 
upper  limb  be  reduced  to  the  true  central  altitude,  by  Problem  XIV.,  page 
320.  Then,  with  the  sun's  reduced  declination,  and  the  latitude  by 
aecount,  enter  Table  LI.  or  LIL,  (according  as  the  latitude  and  the  decUh- 
ation  are  of  the  same  or  of  a  contrary  denomination,)  and  take  out  the 
corresponding  correction  in  seconds  and  thirds,  which  are  to  be  esteemed 
as  minutes  and  seconds ^  agreeably  to  the  rule  in  page  139.     Now, 

To  the  proportional  logarithm'  of  this  correction,  add  tvsice  the  propor- 
tional logarithm  of  the  interval  between  the  time  of  observation  and  noon^ 
and  the  constant  logarithm  7  •  2730;  the  sum  of  these  three  logarithms, 
abating  10  in  the  index,  will  be  the  proportional  logarithm  of  a  correctioni 
which  being  added  to  the  true  altitude  of  the  sun's  centre,  the  sum  will  be 
the  meridional  altitude  .of  that  object :  hence  the  sun's  meridional  zenith 
distance  will  be  known ;  to  which  lei  its  declination  be  applied  by  addition 
or  subtraction,  according  as  it  is  of  the  same  or  of  a  contrary  name,  and 
the  sum  or  difference  will  be  the  latitude  of  the  place  of  observation. 

Example  1« 

At  sea,  January  1st,  1825,  at  22U5r24?  apparent  time,  in  latitude 
51?36f  N.,  by  account.  Mid  longitude  10?45^30^  W.,  the  observed  alti- 
tude of  the  sun's  lower  limb  was  I8?33fd4f  *,  and  the  height  of  the  eye 
above  the  level  of  the  horizon  25  feet;  required  the  latitude  of  the  place  of 
observation? 

Apparent'time  of  observation  =  .     •     •    22?45T24t 
Longitude  10^45^30^  in  time  =      .    .  +  0. 43.   2 

Greenwich  time  =      .......    23*28?26f 


•  This  is  the  mean  of  seversl  aMtudes  df  the  sun's  lower  limb. 
2a2 

Digitized  by 


Google 


556*  NAUTICAL  ASTRONOMY. 

Sun's  decliQation  at  noon,  January  1st  ss  23?  0'59^  S. 
Correction  of  ditto  for  28*28r2e!  =     .     -  5.   6 


Sun's  reduced  declination  =  *    ,    .    .    22? 55^  53^  S. 

The  observed  altitude  of  the  sun's  lower  limb,  reduced  to  the  true  central 
altitude,  is  13?41'24rS. 

Cor.  in  Table  LIL,  answering  to  lat.  50?N.,  and  dec.  22?S.  s  KIS*'.  8 
Difference  to  2?  of  lat.=r  -3-^.  9;  now,  3-^.  9  x  96'.  h-  120C  =  -3.1 
Diff.  to  1?  of  dec.=  -(r. 9;  now,  O-'. 9  x  55'53^  ^  60C  =     -0.8 

Cor.  to  lat.  51936C  N.  and  declination  22?53:53r  S.  =    .    .     K  9^  9 

Computed  corrections  1?9'?'.9,  Proportional log.ss  .  .  •  2.1889 
Time  of  obs.  from  noon  1  M  4  T36 ! ,  twice  the  prop.  log.  s  •  .  0. 7650 
Constant  log.  =       7.2780 


Correction  of  the  sun's  altitude  = 
True  altitude  of  the  sun's  centre  = 

Sun's  meridional  altitude  =      .    . 

Sun's  meridional  zenith  distance  = 
Sun's  corrected  declination  = 


l?46M5r  Prop.log,=0.2269 
13. 41. 24  S. 


15?28'  9rS. 

74?3K5irN. 
22.55:53   S. 


Latitude  of  the  place  of  observation  ==      5 1  ?35 '.  58TN. ;  which  differs  but 
2".  from  the  truth. 

Example  2.  - 

At  sea,  March  21st,  1825,  at  0^50^25!  apparent  time,  in  latitude  51  ?5C 
N.,  by  account,  and  longitude  35?45' W.,  the  observed  altitude  of  the  sun's 
lower  limb  was  37?55^27^*,  and  the  height  of  the  eye  above  the  level  of 
the  sea  21  feet;  required  the  true  latitude  of  jthe  place  of  observation  ? 

Apparent  time  of  observation  =       .    »    .    0^50^25! 
Longitude  35?45^  W.,  in  time  :s  •    .    .    2. 23.   0 


Greenwich  time  = 3?13'25r 

Sun's  declination  at  noon,  March  21st  =     0?  14(30^  N. 
Correction  of  ditto  for  3  ?  13T25  *  =     .    .     +3.11 


Sun's  reduced  declination  =      .    .    .    •    0?  1 7 '  4 1  r  N. 


*  See  Note,  pa^e  355^ 


Digitized  by 


Google 


J^TITUDB  fiY  AN  ALTITUDE  TAKBN  KBAR  THE  MERIDIAN.        357 

The  observed  altitude  of  the  sun's  lower  limb,  reduced  to  the  true  central 
altitude,  is  38?6'2rS. 

Cor.  in  Tabh  LI.,  answering  to  lat.  50"?  N.  and  declin.  0?  s  .  1  rSS"^*  8 
Difference  to  2?  of  lat;  =  -  6^8;  now,  6*^.8x65^-4-120'=:  -3,6 
Diff.  to  l«.  of  declin,  =  +  ^.5;  now  I'^.Sx  17MKh.60;  =  +  0  ,4 

Correction  to  lat.  5 1  ?5  ^  N.  and  declination  0°  1 7 '  4 1  r  N.  =     .     1  r35-^.  6 

Computed  correction  =7  .  1^35*'.  6,  Prop.  log.  =  •  .  •  2.0530 
Time  of  observ.  from  noon  =  0*50?  25!,  twice  the  prop.  log.  =  1. 1054 
Constant  log.  =3 ,    ,    7*2730 


Correction  of  the  sun's  altitude  »      .    .     1?  6 '40?  Prop.  log.sO,  4314 
True  altitude  of  the  sun's  centre  =s    «    «  38.   6.   2  S« 


Sun's  meridional  altitude  =     .     .    .    .39?12U2rS. 


Sun's  meridional  zenith  distance  =  •    .  50?47'  18?N. 
Sun's  reduced  declination  =s     •    •    •    .    0. 17*41  N. 


Latitude  of  the  place  of  observation  =.51?  4'59?N. ;  which  differs  but 
1  ?  from  the  truth. 

Hence  it  is  evident,  that  the  latitude  may  be  determined  by  this  method 
to  all  the  accuracy  desirable  in  nautical  purposes.  It  possesses  a  decided 
advantage  over  that  by  double  aUitudes ;  and,  since  the  operation  is  so 
extremely  simple,  the  mariner  will  do  well  to  avail  himself  thereof  on 
every  occasion ;  because  the  latitude,  thus  deduced,  will  be  equally  as  cor- 
rect as  that  resulting  from  the  observed  meridional  altitude,  provided  the 
observation  be  made  within  the  prescribed  limits.  When,  however,  the 
latitude  and  the  declination  are  of  different  names,  it  will  not  produce  any 
sensible  error  in  the  result,  if  the  altitude  be  observed  a  few  seconds  without 
those  limits,  as  may  be  seen  in  Example  1,  above. 

But  it  is  to  be  remembered,  that  the  apparent  time  of  observation  must 
be  well  determined. 


Digitized  by 


Google 


358  VAUTICAl*  ASTRONOlfV. 


Problem  X. 


Bwm  the  La^iude  by  Account ^  the  AUiUide  of  the  Mom's  lower  or  upper 
Limb  observed  near  the  MeridiOMy  the  apparent  Time  of  Ob$ervatkm, 
and  the  Longitude  ;  to  find  the  true  Latitude* 

To  the  apparent  time  of  observation  apply  the  loiigitudei  in  time,  by 
addition  or  subtraction^  according  as  it  is  west  or  east,  and  the  corre- 
sponding time  at  Greenwich  will  be  obtained ;  to  which  let  the  sun's  right 
ascension  be  reduced,  by  Problem  V.,  page  298 ;  and  let  the  moon's  right 
ascension,  declipation,  semi-diameter,  and  horizontal  parallax^  be  also 
reduced  to  that  tim^,  by  Problem  Vf.,  page  302.  Let  the  observed  altitude 
of  the  moon's  limb  be  reduced  to  the  true  central  altitude,  by  Problem  XV., 
page  323. 

To  the  apparent  time  of  observation  add  the  sun's  reduced  right  ascen- 
sion, and  the  sum  (abating  24  hours,  if  necessary,)  will  be  the  right  ascen- 
sion of  the  meridian ;  the  difference  between  which  and  the  moon's  reduced 
right  ascension,  converted  into  time,  will  be  the  moon*s  distance  from  the 
meridian  at  the  time'  of  observation.  Now,  with  the  mooji's  reduced 
declination,  and  the  latitude  by  account,,  enter  Table  LI.  or  LIf.,  according 
9s  they  are  of  the  same  or  of  a  contrary  denomination,  and  taHe  out  the 
corresponding  correction,  agreeably  to  the  rule  in  page  139 ;  with  which, 
and  the  moon's  distance  from  the  meridian,  compute  the  correction  of 
altitude ;  and,  hen^e,  the  latitude  of  the  p(ace  of  observation,  by  Problem 
IX.,  page  354. 

Note. — ^The  limits  within  which  the  altitude  of  the  moon  should  be 
observed,  are  to  be  determined  in  the  same  manner,  precisely,  as  if  it  were 
the  sun  that  was  under  consideration ;  observing,  however,  to  estimate  the 
interval  from  the  .time  of  transit  over  the  meridian  of  the  place  of  obsenra* 
tion,  instead  of  from  noon. 

See  the  explanation  to  Tables  LI.  and  LII.,  between  pages  138  ancl  143. 

Example  1. 

January  23d,  1825,  at  3*55ri7?  apparent  time,  in  latitude  51?l5i  N., 
by  account,  and  longitude  45?  W.,  the  observed  altitude  of  the  moon's 
lower  limb  was  39?  27 "30^*,  and  the  height  of  the  eye  above  the  level  of 
the  horizon  24  feet;  required  the  true  latitude  of  the  place  of  observation  ? 


*  This  is  the  mean  of  several  altitudes. 

/Google 


Digitized  by ' 


LATITUDE  BV  AN  ALTtTtoB  TAXKN   NBAR  THB  MERIDIAN.       859 

Apparent  time  of  obserration  =:  .     ....    3*55T17! 
Longitude45?  W.,  intima  =: 3.   0.   0 

Greenwich  time  = 6*55T17! 

Smi's  right  ascension  at  noon,  January  23d,  =:  20!  22*23! 
Correction  of  ditto  for  6!55ri7!  =:      ...     +1.12 

Sun's  reduced  right  ascension  =      .     .     .     ,     20!  23735! 
Apparent  time  of  observation  =      ....      3.55.17 

Right  ascension  of  the  meridian  =     ,     ,     .       0!18T52! 

Moon's  R.  A.  at  noon,  January  23d,=:349?47^55r 
Correction  of  ditto  for  6!55ri7!  =   4-  3. 6. 56 


Moon's  reduced  right  ascension  =:     352?54!51!'r:  23*Slr39f 
Right  ascension  of  the  meridian  = 0. 18. 52 

Moon's  distance  from  the  meridian  =     .    .    .    .      0!47ri3! 

Moon's  declination  at  noon,  January  23d,  =:       1?10'39!^N. 
Correction  of  ditto  for  6!55r  17!  =     .    .       +  1 .  22. 36 


Moon's  reduced  declinatioii  =••««»      2?d3  •  15f  N. 

Observed  altitude  of  moon's  lower  limb  =     .    89? 27 '30'  S. 
Scmi-diaoieter  14^52^  —  dip4C42f  =:   .    •     +  10.10 


Apparent  altitude  of  the  moon's  centre  =      .    39?37:40f  S. 
Cor.^  Table  XVIIL,  ans.  to  hot.  parallax  54 '  4f  r=     40. 36 

True  altitude  of  the  moon's  centre  =  •    .    .    40?  18!  16T  S. 

Cor.  in  Table  LI.,  answerhie  to  lat  50?N.  and  declin.  2?N.  s  1^41'^.  8 
Diff.  to  2?  of  lat.  =s  -7*.25  now,7*'.2x75:-i.l20:  =  .  -  4  is 
Diff.  to  l?of  decUn.r:  +  1^6;  now,!''. 6x33!  15^-^60!  sr  +  0  .9 

Cor.  answering  to  lat.  5 1  ?  15 ! N.  and  declin.  2?33 n5rN.  =:  .     1  r38^  2 

Computed  correction  =     1!'38*'.2,  Prop.  log.  =       ,    .    .    .    2.0413 

Moon's  mer.distance  =:  0M7*13!,  twice  the  prop.  log.  =     ,    1. 1624 

Constant  log.  =:      ••-.*••••••-•,,.    7.2730 

Carrectkm  of  the  moon's  altitude  =:  1?  0  3r  Prop»log,  :=  0.4767 
True  altitude  of  the  moon's  centre  =  40. 18. 16  S. 

Moon's  meridional  altitude  ss  ,    .    4i;i8!19!rS. 

Digitized  by  VjOOQ IC 


360  NAUTICAL  ASTRONOMT. 

Moon's  meridional  altitude  x=  •     •    41?18M9?S. 


Moon's  meridional  zenith  distance  =  48?4 1 ' 4 1  '/N. 
Moon's  reduced  declination  =       •       2. 33. 15  N. 


Latitude  of  the  place  of  observations  5 1  ?  14 ' SGI'N. ;  which. differs  but  4f 
from  the  truth. 


Example  2* 

January  30th,  11325,  at  9M5rl2!  apparent  time,  in  latitude  57"? 40^  S., 
by  account,  and  longitude  60?  east,  the  observed  altitude  of  the  moon's 
lower  limb  was  5?37'  12^*,  and  the  height  of  the  eye  above  the  level  of 
the  horizon  26  feet]  required  the  true  latitude  of  the  place  of  observation  ? 

Apparent  time  of  observation  =       •     •     •     •      9t45T12! 
Longitude  60?  east,  in  time  s=    .    .    .    4    .      4.   0*  0 

Greenwich  time  =  .    ..•..-..      5M5ri2! 

Sun's  right  ascension  at  noon,  January  30th,  =  20*5I?25! 
Correctioaofdittofor  5M5ri2!  =  .    .    .    .     +  0.59 


Sun's  reduced  right  ascension  =      .    .    •    .    20?52?24! 
Apparent  time  of  observation  =       •    •    .    «      9.45.12 

Right  ascension  of  the  meridian  s       •    .    .      6t37*36! 

Moon's  R.  A.  at  noon,  January  30tli,  =  76?2H55r 
Correction  of  ditto  for  5  M5r  12:   =     +  2.22.52 


Moon's  reduced  right  ascension  =      .    78?44'.47T  =  5n4?59! 
Right  ascension  of  the  meridian  s ^      6. 37. 36 

Moon's  distance  from  the  meridian  =.••••      1?22T37! 

Moon's  declination  at  noon,  January  SOth,  a=  .    .    23?57M6fN, 
Correctionof  dittofor5M5?12!  =        ....       —  4.   3 


Moon's  reduced  declination  ss       ..•,•,    23?53^4S?N. 


«  See  Note,  page  358. 


Digitized  by 


Google 


LATITUDE  BT  AN  JkLTTTVVM  TAKBN   NEAR  THB  MERIDIAN.       361 

Observed  altitude  of  the  moon's  lower  limb  =  •    .      5?37'  12^N. 
Semi-diameter  15'.52r- dip  4^52'/ =  .    .    .     .     +11.0 


Apparent  altitude  of  the  moon's  centre  =    •     •    .      5?48'  12rN. 
Correc.,TableXVIII.,au8.tohor.parallax,58^5r=s    +  49.   6 

True  altitude  ofthe  moon's  centre  =5       •    *    .    .      6?37'18C'N. 

Cor.  in  Tab.  LIL,  answering  to  lat  56?  S.  and  declin.  23?  N.  =:  1  r  l*'.  8 
Difference  to2?oflat.  =  - 3^6;  now, 3*\ 6 x  100^-4-120^  =  -  3  .0 
Diff.  to  1?  of  declin.  ss  -  0*'.7j  now,0^.7x53C43r-«-60^=     -0.6 


Cor.  to  lat.  57?40'.  S.  and  declination  23?53'.43r  N.  *     .    0^58^2 

Computed  correction  =  0^58*^.  2     ftop.  log.  = 2. 2685 

Moon's  merid.  distance  =  1  *22r37 '  Twice  the  prop.  log.  =  .    .     0.6764 
Constant  log.  = ...•.?!  2730 

Correction  of  the  moon's  altitude  =     .       l?48'59f  Prop. log.sO.  2179 
True  altitude  of  the  moon's  centre  =     .      6. 37. 18  N. 


Moon's  meridional  altitude  =       ...      8?26^  17^N. 


Moon's  meridional  zenith  distance  =     •    8 1  ?33 ^  43 C"  S. 
Moon's  reduced  declination  =     .    •    •    23.53. 43  N. 


Latitude  of  the  place  of  observation  =  .    57?40'  01 S.*;  which  is  exactly 
right. 

Hence  it  is  evident,  that,  by  this  method,  the  latitude  may  be  inferred 
from  the  true  altitude  of  the  moon's  centre,  to  every  degree  of  accuracy 
desirable  in  nautical  operations,  provided  the  altitude  be  observed  within 
the  proper  limits ;  which,  for  the  sake  of  assisting  the  memory,  will  be  here 
repeated, — ^viz..  The  number  of  minutes  and  seconds,  in  the  moon's  dis- 
tance from  the  meridian  at  the  time  of  observation,  must  not  exceed  the 
number  of  degrees  and  minutes  contained  in  the  meridional  zenith  distance 
of  that  object  at  the  place  of  observation.  Thus,  in  the  above  example, 
where  the  moon's  meridional  zenith  distance  is  81?34^  nearly,  the  interval 
between  the  time  of  observation  and  the  time  of  the  moon's  transit,  or 
passage  over  the  meridian  of  the  place  of  observation,  must  not  exceed' 
81T34! ;  though  the  moon's  altitude  may  be  taken  at  any  time  within  that 
interval,  or  as  near  to  the  time  of  transit  as  the  observer  may  think  proper. 


Digitized  by 


Google 


$69  NAtrncAL  astronomy. 


Problbm  XL 

Given  the  Latitude  by  Account ^  the  observed  central  Altitude  qfa  Planet 
near  the  Meridian,  the  apparent  Tiine  of  Observation,  and  the  Longir 
tude :  to  find  the  true  Latitude. 

RUIJB. 

To  the  apparent  time  of  observation  apply  the  lonptttde,  in  tamei  by 
addition  or  Bubtraction^  according  aa  it  is  west  or  east;  and  th«  tiini)  or 
difference,  will  be  the  corresponding  time  at  Greenwich*  To  this  time  let 
the  planet's  right  ascension  and  declination  be  reduced,  by  Problem  VII., 
page  307 ',  and  let  the  sun  s  right  ascension  at  noon  of  the  given  day  be 
also  reduced  to  that  time  hy.  Problem  V.,  page  298. 

Let  the  observed  central  altitude  of  the  planet  be  reduced  to  its  true 
central  altitude,  by  Problem  XVT.,  page  325.  Then,  to  the  apparent  time 
of  observation  add  the  sun*s  reduced  right  ascension,  and  the  sum  (abating 
24  hours,  if  necessary*)  will  be  the  right  ascension  of  the  meridian  }  the 
difference  between  which  and  the  planet's  reduced  right  ascension,  will  be 
that  object's  distance  from  the  meridian  at  the  time  of  observation.  Now, 
with  the  latitude  by  account,  and  the  planet's  reduced  declination,  enter 
Table  LI.  or  LII.,  according  as  they  are  of  the  same  or  of  contrary  deno-* 
minations,  and  take  out  the  corresponding  correction,  agreeably  to  the  mle 
in  page  139;  virith  which,  and  the  planet's  distance  from  the  meridian^ 
compute  the  correction  of  altitude,  and,  hence,  the  latitude  of  the  place 
of  observation,  by  Problem  IX.,  page  354. 

Note, — The  measure  of  the  interval  between  the  time  of  observation  and 
the  time  of  transit, — that  is,  the  number  of  minutes  and  seconds  contained 
in  the  planet's  distance  from  the  meridian,  must  not  exceed  the  number  of 
degrees  and  minutes  contained  in  that  object's  meridian  zenith  distance  at 
the  place  of  observation. 

See  explanation  to  Tables  Lt.  and  Lit.,  page  I3S,  and  thence  to  143. 

Example  K 

.  January  4th,  1825,  at  12t3lT30!  apparent  time,  in  65?28CS.,  by 
account,  and  longitude  60?  east,  the  observed  central  altitude  of  the  planet 
Jupiter  was  5?  14  ^35^*,  and  the  height  of  the  eye  above  the  level  of  the 
horizon  25  feet ;  required  the  true  latitude  of  the  place  of  observation  ? 

*  This  is  the  mean  of  several  altitudes. 

Digitized  by  VjOOQ IC 


LATITUDB  B7  AN   AI.TITUDB  TAXBH  NVAR  THE  MBRIDIAN.      363 


Apparent  time  of  observation  ^ 
Longitude  60?  £.,  in  time  2=    . 


Greenwich  time 


12*31T30! 
-  4.   0-   0 

8f31?30! 


Sun's  right  ascension  at  nooti^  Jan.  4tb|  =  19^  0?32! 
Correction  of  ditto  for  8?3ir30t  =s  .    »       +   1^34r 


Sun's  reduced  right  ascension  =        •  .  19?  2?  6t 

Apparent  time  of  observation  =;     ,    .  ,  12.31.30 

Right  ascension  of  the  meridian  =     .  .  7*33?36! 
Jupiter's  right  ascension  at  noon,  Jan.  Istss  8t58?  Of 

Correction  of  ditto  for  3f8*31?30r  =  .  -   1^   6r 

Planet's  reduced  right  ascension  ■«    •  .  8t56?54! 

Right  ascension  of  the  meridian  =     .  .  7*  33. 36 

Planet's  distance  from  the  meridian  =  .  1  ?  23?  1 8 ! 


Jupiter's  declination  at  noon^  January  1st  an  17^56'  OTN. 
Correction  of  ditto  for  3f8*3ir30!  =  .    .       +6.43 


Planet's  reduced  declination  ss 


18?  2MS?N. 


Correction  ip  T«ble  UL,  answ.  to  lat.  64?$.  and  dec.  I8?N.  m  Or49''.  6 
Difference  to  2?  of  lat.=:  -  3^8}  qqm^  3'*.8  x  88^  ^  120?  as  -  2  .8 
Difference  to  1?  of  de«.=  -  0^.  4;  now,  0^.  4  x  2M3r-4-  60^  =         0.1 

Correction  answ.  to  lat.  (55?28?  S.  and  dec.  18?2U3r  N.  =    .    0r46*'.  7 

Computed  correction  =  .  .  .  0?46*'.  7  Prop.  log.  =  .2. 3642 
Jupiter's dist.  fr.mer.attimeofobs.rs  1  *23?18!  Tw.  the  prop.  log.=0. 6692 
Constant  log.  =: 7*2730* 

Correction  of  Jupiter's  altitude  ==  .  .  1  ? 28 ?  54?  Prop.  log.=;0. 3064 
Jupiter's  obs«  alt^  red.  to  tnie  centr.  alt,  is  =  5  •   0. 1 0 

jupiler't  meridional  altitude  8  ....    6?29?  4?N. 

Jupiter's  meridional  zenith  distance  =:      •  83?30'56?  S. 
Jupiter's  reduced  decimation  =s    '.    .     .  18.   2.43  N. 

Latitude  of  the  place  of  observation  b     .  65?28(  13?  S.;  which  differs  13? 
from  the  truth* 


Digitized  by 


Google 


364  NAUrrcAL  astronomy. 

Example  2. 

February  4th>  1825^  at  d?36T20'  apparent  time,  the  observed  central 
altitude  of  the  planet  Venus  was  36?24^25r*,  in  latitude  52?  12'.  N.,  by 
account,  and  longitude  45?  40^  W.,  and  the  height  of  the  eye  above  the 
level  of  the  horizon  was  26  feet ;  allowing  the  horizontal  parallax  of  the 
planet,  at  that  time,  to  be  17^9  the  true  latitude  of  the  place  of  observation 
is  required  ? 

Apparent  time  of  observation  ==    .    •    .      3^36?20' 
Longitude  45?40^  W.,  in  time  =:      .      +  3.   2. 40 

Greenwich  time  =••••••«      6. 39.  0 

Sun's  right  ascension  at  noon,  Feb.  4th  =  21M1T45! 
Correction  of  ditto  for  6*39r0!  =     .     .     +   T.  7? 


Sun's  reduced  right  ascension  =  •    ,    •     21M2T52: 

Apparent  time  of  observation  =:    •    •    •      3^3fr?20! 
Sun's  reduced  right  ascension  =:  •    .     •     21. 12. 52 


Right  ascension  of  the  meridian  =  .  .  0M9T12! 
Venus'  right  ascension  at  noon,  Feb.  lst,=  23M5T  0'. 
Correction  9f  ditto  for  3f  6 ?39T0!  =     •     +   13.   6 

Venus'  reduced  right  ascension  =  •  .  23*58?  6! 
Right  ascension  of  the  meridian  =:    .    .      0. 49. 12     ' 

Planet's  distance  from  the  meridian  =:  •      0*5 IT  6! 

Venus' declination  at  noon,  Feb.  1st,    =       2?  7'  OTS. 
Correction  of  ditto  for  3f  6*39?0!  =  •     -   1. 42. 41 


Venus' reduced  declination  =       .    .    .      0?24n9fS. 

Observed  central  altitude  of  Venus  = 36?24'.25r  S. 

Dip  of  the  horizon  for  26  feet  =: ^4.52 


Apparent  central  altitude  of  Venus  =: 36?19^33rS, 

Refrac.  (Tab.Vin.)in7r-Parall.(Tab.VI.)Oa4r=     -   1.   3 


True  central  altitude  of  Venus  = 36?18:30rS. 


*  This  is  the  mean  of  several  altitudes.    The  altitude  of  Venus  may  be  taken  very  cor* 
rectly  when  the  son  is  above  the  horizon,  provided  the  atmosphere  be  fine  and  clear. 


Digitized  by 


Google 


LATITUDE  BY  AN   ALTITUDE  TAKKN   NEAR  THE   MERIDIAN^       365 

Cor.  in  Tab.  LII.,  ans.  to  lat  52?N.  and  dec.  0^  =  ...  1^32*'.  0 
Diff.to2?onat.=  -6''.45now,6^4xl2^-Hl20'=:  .  .  .  -  0  .6 
Diff.toI?ofdec.=:-l-'.2;now,1^2x24n9r^60C=     .    .    .     -  0  ,5 

Correction  answering  to  lat.  52?12'.  N.  and  dec.  0?24n9rS.=    KS(r.9 

Computed  correction  =  H30*'.9     Prop.  log.  = 2.0749 

Venus'  merid.  distance  =  0^  5 1  Te !  Twice  the  prop.  log.  r=  .  1 .  0938 
Constant  log.  = 7.2730 

Correction  of  Venus'  altitude  =  .  .  1  ?  5 ^  6r  Prop.  log.=  0. 44 1 7 
True  central  altitude  of  Venus  =:  .     .    36.18.30  S. 


Venus*  meridional  altitude  =    .    .    .    37^23'.36rS. 


Venus'  meridional  zenith  distance  =    .    52?36 '.  24rN. 
Venus' reduced  declination  =   •    .     .      0. 24. 19S. 


Latitude  of  the  place  of  observation  =     52?  12'   5rN.;  which  differs  but 
5T  from  the  truth. 


Problem  XII. 

Cfioen  tlie  Latitude  by  Account,  the  Altitude  of  a  fixed  Star  observed  near 
the  Meridtauy  the  apparent  Time  of  Observation;  and  the  Longitude,  to 
find  the  true  Latitude. 

Rule. 

Turn  the  longitude  into  time,  and  apply  it  to  the  apparent  time  of 
observation,  by  addition  or  subtraction,  according  as  it  is  west  or  east ; 
and  the  sum,  or  difference^  will  be  the  corresponding  time  at  Greenwich. 

To  this  time  let  the  sun's  right  ascension  at  n6on  of  the  given  day  be 
reduced  by  Problem  V.,  page  298. 

Let  the  star's  right  ascension  and  declination  (Table  XLIV.)  be  reduced 
to  the  night  of  observation,  by  the  method  shown  in  page  115;  and  let  the 
star's  observed  altitude  be  reduced  to  the  true  altitude,  by  Problem  XVIL^ 
page  327. 

To  the  apparent  time  of  observation  add  the  sun's  reduced  right  ascen-* 
aion,  and  the  sum  (abating  24  hours,  if  necessary,)  will  be  the  right  ascen- 
sion of  the  meridian ;  the  difference  between  which  and  the  star's  reduced 
right  ascension  will  be  that  object's  distance  from  the  meridian  at  the  time 
of  observation. 


Digitized  by 


Google 


366  NAUTICAL  ASTRONOMY. 

Now^  with  the  latitude  by  account,  and  the  star's  reduced  deeUoation, 
enter  Table  LI.  or  LII.,  according  as  they  are  of  the  same  or  of  a  contrary 
denomination  ;  and  take  out  the  corresponding  correction,  agreeably  to  the 
rule  in  page  139;  with  which,  and  the  star's  distance  from  the  meridian, 
compute  the  correction  of  altitude )  and,  hence,  the  latitude^  by  Problem 
IX.,  page  354. 

Note. — ^The  interval  between  the  time  of  observation  and  the  time  of 
transit  must  not  exceed  the  limits  pointed  out  in  the  three  preceding 
Problems ;  viz.,  the  number  of  minutes  and  parts  of  a  minute  contmed  in 
the  star's  distance  from  the  meridian,  is  not  to  exceed  the  number  of 
degrees  and  parts  of  a  degree  contained  in  that  object's  meridional  zenith 
distance  at  the  place  of  observation. 

See  explanation  to  Tables  LI.  and  Lll.,  from  page  138  to  143. 


Example  1. 

January  Ist,  1825,  at  S?52T17!  apparent  time,  in  latitude  52?46'  N., 
by  account,  and  longitude  56?  15'.  W.,  the  observed  altitude  of  the  star 
Menkar  was  39?42'.40'f,  and  the  height  of  the  eye  above  the  level  of  the 
sea  26  feet ;  required  the  true  latitude  of  the  place  of  observation  ? 

Apparent  time  of  observation  =      ....•«...      8^52"17' 
Longitude56?15' W.,  intime  =  ........      +  3.45.   0 


Greenwich  time  = 12*37*17! 

Sun's  right  ascension  at  noon,  January  1st  =     18*47*19' 
Correction  of  ditto  for  12*37"17'  =  ...       +2.19 


Sun's  reduced  right  ascension  =      ....     18*49?38! 
Apparent  time  of  observation  =      .     .     .     .       8. 52. 17 


Right  ascension  of  the  meridian  =  .    •    .    •  3*41T55' 

Menkar's  right  ascension,  January  1st,  1824  =:  2*53T  5' 
Correction  of  ditto  for  1  year  =:       .     .     .     .       +  0'  3^ 

Menkar's  reduced  right  ascension  =:     •     .    .  2*53?  8! 

Right  aseensioii  of  the  meridian  =       .    .    •  3. 41. 55 

Star's  distance  from  the  meridian  =    «    •    «  0*48?47  • 

Digitized  by  VjOOQ IC 


JLATITUDE  BY  AN  ALTITUDH  TAKfiN   N^^R  THB  MERIDIAN.       8W 

Menkar'a  declination,  January  l«t,  1824  =  .  S?23'4lrN. 
Correction  of  ditto  for  1  year  =.....  0. 15 

Menkar*8  reduced  declination  =:  .    .    .    .    ,  3?23'58rN. 


Star's  observ.  alt.,  reduced  to  ite  true  alt,  is  =  39?36'39rS. 

Correction  in  Table  LI.  answering  to  lat.  52?N.  and  dec,  3?N.  =  KSG*'.  0 
Difference  to  2?  of  lat.=  -7^  0;  now,  7^  0  x  46^  h-  120^  =  -2.7 
Differenceto  l?of  dec.rr  +1^2;  now,l-'.2x2S^56r-«-e0r=      +0.5 


Correction  to  lat,  52*4e^N.  and  declination  3?23:56rN.  =:  lr33^8 

Computed  correction  =:      K33*'.  8    Prop.  log.  =     .     .     ,     .  2.0612 

Star's  mend,  distance  =    G*48?47*  Twice  the  prop.  log.  =  1. 1340 

Constant  log.  =: 7.2730 


Correction  of  Menkar's  altitude  ==     .       1?  IHSf  Prop.  log.  =  0.4682 
True  altitude  of  Menkar  =      ...    39. 36. 39  S. 


Menkar*s  meridional  altitude  =:    .    .    40^37 '.54rS. 


Menkar's  meridional  zenith  distances:  49^22^  6rN. 
Menkar's  reduced  declination  =:  .     .      3.  23. 56  N. 


Latitude  ofthe  place  of  observation  =:  52^46'  2rN.;   which  Offers  but 
2f  from  the  truth. 

Example  2. 

September  Ist,  1825,  at  13^28^42!  apparent  time,  in  latitude  49?30^S. 
by  account,  and  kmgitude  22?I0'30'!r  E.,  the  observed  altitude  of  the  star 
fi  Pegasi,  or  Scheat,  was  1 1  ?37 '  59?,  and  the  height  of  the  eye  above  th« 
level  of  the  sea  19  feet ;  required  the  true  latitude  of  the  place  of  observa- 
tion? 

Apparent  time  of  observation  =      ....     13'28"42! 

Longitude  22?  10^30?  E.,  fai  time  =     .    .    .  -  1 .  28. 42 

Greenwich  time  =....,....     12*  0?  0! 

Sun's  right  ascension  at  noon,  Sept.  1st,  =     .     10M1T16! 
Correction  of  ditto  for  12* OTO!  =  .     .     .     .        +   l'49r 


Son's  vedueed  right  ascension  ±a     •    .    •    •     10*43?  5! 
Apparent  time  of  observation  sa      .    •    «    .     13. 28. 42 

Right  ascension  of  the  meridian  =      •    •    .      0MIT47' 

Digitized  by  VjOOQ IC 


368  NAUTICAL  ASTRONOMY. 

Scheat's  right  ascension,  January  Ist,  1824,  s=    22^55?  5 '. 
Correction  of  ditto  for  1  year  and  8  months  &=      +0^5 


'  i:'^ 


Scheat's  reduced  right  ascension  =      •    •    .  22^55?  10! 
Right  ascension  of  the  meridian  =      •    •     .      0, 1 1. 47 

Star's  distance  from  the  meridian  =     .    •     •       1M6?37  • 

Scheat's declination,  January  1st,  1 824,  s     .  27?  7  -SS^N, 
Correction  of  ditto  for  1  year  and  8  months  =:     +  0.32 

Scheat's  reduced  declination  ss        .    •    •    •  27-  8C  7^N. 

Scheat's  observed  altitude  = ll?37'59rN, 

Dip  of  the  horizon  for  1 9  feet  s      •    •    •    .       ^4,11 


Scheat's  apparent  altitude  =       .    .    .     .    .     1 1  ?33 U8rN. 
Refraction  =       •    •    •    •     • *-  4. 33 


Scheat's  true  altitude  =     •    .    .    ,    •    .    .     ll?29'15rN. 

Correction  in  Table  LII.,  answ.  to  lat.  49?S.  and  dec.  27?N.  =  K 1 1-».  0 
Difference  to  1 9  of  latitude  =  - 1*'.  8  j  now,  ^ .  8  x  30^  h-60^  =  -0.9 
Difference  to  1?  of  dec.=  -  0^.  8  j  now,  0^.  8  x  8^7r-*-60'   =     -0.1 


Correction  to  latitude  49?30CS.  and  declination  27?8!7^N.  =  "KIO^'.O 

Computed  correction  =  KIO'^.O  Prop.  log.  =  ....  2.1883 
Star's  mend,  distance  =  1  *  16r37 '  Twice  the  prop.  log.  =  .  0. 74 1 8 
Constant  log.  = 7. 2730 

Correction  of  the  star's  altitude  =s  .*  1?52U(!?  Prop.  log.  =£  0.2031 
Scheat's  true  altitude  =:      .    .     .     .     1 1 .  29. 15  N. 


Scheat's  meridional  altitude  =:      .    .     1 3  ?  22  ^   KN. 


Scheat's  meridional  zenith  distance  =    .76?37'59rS. 
Scheat's  reduced  declination  =     .     .     27.   8.    7  N. 


LAtitudeof  the  place  of  observation  s  49?29^52rS. ;   which  differs  ST 
from  the  truth. 

Remark. — ^The  latitude  may  be  also  very  correctly  inferred  from  the 
altitude  of  a  celestial  object  observed  near  the  meridian  below  the  pole. 
In  this  case,  the  meridian  distance  of  the  object  is  to  be  reckoned  from  the 
apparent  time  of  its  transit  below  the  pole ;  the  correction  answering  to  the 
latitude  and  the  declination  is  always  to  be  taken  out  of  TcJ>le  LILj  in  the 


Digitized  by 


Google 


LATITUDE  BT  AN  ALTITUBB  TAKEN  NEAR  THE  MERIDIAN*   369 

same  manner  as  if  those  elements  were  of  different  denominations ;  and  the 
correction  of  altitude  is  to  be  applied  by  subtraction  to  the  true  altitude  of 
the  object,  deduced  from  observation,  in  order  to  find  its  meridional  altitude 
below  the  pole.  Then,  with  the  meridional  altitude  below  the  pole,  thus 
found,  and  the  declination,  the  latitude  is  to  be  determined,  by  Problem  V.^ 
page  336. 

The  interval,  or  limits  within  which  the  altitude  should  be  observed,  is 
to  be  determined  in  the  same  manner  as  if  the  celestial  object  were  near  the 
meridian  above  the  pole. 

Example  1, 

June  20th,  1825,  at  11M8?30!  apparent  time,  in  latitude  71?50'N., 
by  Account,  and  longitude  65?  W.,  the  observed  altitude  of  the  sun's  lower 
limb  was  5?30^50^,  and  the  height  of  the  eye  above  the  level  of  the  sea 
20  feet  3  required  the  true  latitude  of  the  place  of  observati^? 

Interval  between  the  time  of  observation  and  midnight  =s     •      OMlTSO! 

Sun's  observed  reduced  to  its  true  central  altitude  =    5?S3C371' 

Sun's  corrected  declination  S3    23?27'36?N. 

Sun's  north  polar  distance  s  66?32'24r 

Cor.  in  Table  LII.,  answering  to  lat-  70?  and  declin.  23?  =  .  0^37*^.  1 
Diff.  to2?  oflat.=  -3^5;  now,3'".5xllO^H-120C  =  .  -  3  .2 
Diff.  to  1?  of  dec.=  -  O*'.  2;  now,  O*',  2  x  27'.36r-H60  =     .     -0.1 

Ck)rrectiontolat.71?50' anddec.  23?27'36?  =     ....    0r33''.8 

Computed  correction  =5  0^33''.  8,  Proportional  log.  =  .  .  .  2. 5045 
Sun's  dist.  from  midnight  =  0*4lr30!,  twice  the  prop.  log.  =  1. 2744 
Constant  log.  = 7. 2730 

Correction  of  altitude  =:  .  .  .  .  -  0?15:58r  Prop.log,=  1.0519 
True  central  altitude  of  the  sun  =       .    5. 33. 37 


Sun's  meridian  altitude  below  the  pole  =  5?  17 '  391^ 
Sun's  north  polar  distance  =      ...  66. 32. 24 


Latitude  of  the  place  of  observation  =   71?50'  31  N.j  which  differs  but 
3?  from  the  truth. 

2b 


Digitized  by 


Google 


370  NAUTICAL  AJTROMOMT. 

In  case  of  a  Fixed  Star  :— 

Find  the  apparent  time  of  the  star's  superior  transit  above  the  pole,  at 
the  given  meridian,  by  Problem  XIL,  page  317  >  to  this  time  let  12  hours, 
diminished  by  half  the  variation  of  the  sun's  right  ascension  on  the  given 
day,  be  added,  and  the  sum  will  be  the  apparent  time  of  the  star's  inferior 
transit  below  the  pole.  Then,  the  rest  of  the  operation  is  to  be  performed 
exactly  the  same  as  that  for  the  sun  in  example  I,  as  above. 


Example  2. 

January  1st,  182S,  at  ll^SOrOt  apparent  time,  in  latitude  71^90^  N., 
by  account,  and  longitude  84?9'S0^  W.,  the  observed  altitude  of  the  star 
Albireo  was  9?3S!,  and  the  height  of  the  eye  above  the  level  of  die 
horizon  19  feet ;  required  the  true  latitude  of  the  place  of  obaenration  ? 

Apparent  time  of  star's  transit  above  the  pole  »  •      0tS5732' 
To  which  add  12t  -  2rl2!  (halfvar.of  S.R. A,)  =  11.57.48 

Apparent  time  of  the  star's  transit  below  the  pole  =  12!33T20' 
Apparent  time  of  observation  n    ••••••     11.50.  0 

Star's  distance  from  the  meridian  a:  .    •    •    •    ;      0M3T20! 

Observed  altitude  of  the  star  Albireo  s    •    9?SS{  Of 
Dip  of  the  horizon  for  19  feet  s     •    •    •     —  4. 11 

Star's  apparent  altitude  = 9?28U9r 

Refractions     .     .    « -i-  5,83 

Star's  true  altitude  cs 9?23'17r 

Cor.  in  Table  LIL,  answering  to  lat.  70?  and  declin.  27^  =  .  0r36*',  2 
Diff.  to  2°  of  lat.=  -  3*^.4  J  now,  3''.4x90^h-120C  =  .-2.5 
Dittto  l?ofdec,»  --0^.3}  now,0^.3x35:54r-^60  a     •     -  0  •! 

Correction  to  latitude  71?30'.  and  declin.  27?35C54f  ss     .    ,    Or33*',  6 

Computed  correction  =  .  0^33*.  6,  Prop.  log.  =  .  .  2. 5071 
Star's  merid.  distance  =  ,  0*43T20 !,  twice  the  prop,  log.  =  1 .  2870 
Constant  log.  = ,      7.2730 

Correction  of  the  star's  altitude  s=    .    .-  On7n8?  Brop.k)g.aB  L0I71 

Digitized  by  VjOOQ IC 


LATITUDE  BT  AN  ALTXTUBS  TAKSN   NEAR  THB  MBRIBIAN.       871 

Correction  <^  the  atar's  altitude  a     .     -  0?  17 U87 
True  altitude  of  the  star  b    . ,    .    .    .    9. 23. 17 


Star's  meridiau  altitude  below  the  pole  =  9?  5 '59"? 
Star's  north  polar  distance  =s  •    •    .    .62.24.   6 


Latitude  of  the  place  of  observation  =  .  71?30^  5rN.  j  which  differs  but 
f  f  from  the  truths 

^ole.— From  the  abov^  examples,  the  method  of  finding  the  latitude  by 
an  altitude  of  the  moon,  or  of  a  planet,  observed  near  the  meridian  below 
the  pole,  will  appear  obvious. 

Remark. 

The  following  ingenious  problem  for  determining  the  latitude^  either  at 
sea  or  on  shore,  has  been  communicated  to  the  author  by  that  scientific 
and  enterprising  officer.  Captain  William  Fitzwilliam  Owen,  of  His 
Majesty's  ship  Eden,  who  is  so  highly  renowned  for  his  extensive  know« 
ledge  in  every  department  of  science  connected  with  nautical  subjects. 

Pboblbk. 

Given  the  Latitude  by  Jccount,  the  true  AUiiude  qfthe  Sun's  Centre,  and 
the  apparent  Time;  to  find  the  true  Latitude  qf  the  Place  qf  Qi- 
eervathn. 

RuLb. 

Find  the  mean  between  the  estimated  meridian  altitude,  and  th^  altitude 
deduced  from  observation,  which  call  the  middle  altitude;  then, 

To  the  log.  rising  of  the  apparent  time  from  noon,  add  the  log.  eo-sine 
of  the  latitude,  the  log.  co-sine  of  the  corrected  declination,  the  log.  secant 
less  radius  of  the  middle  altitude,  and  the  constant  logarithm  7»  536274  j*' 
the  sum  of  these  five  logarithms,  abating  30  in  the  index,  will  be  the  loga- 
rithm  of  a  natural  number,  which  is  to  be  esteemed  as  minutes,  and  which, 
being  added  to  the  sun's  true  central  altitude,  will  give  his  correct  meri- 
dional altitude ;  and,  hence,  the  true  latitude  of  the  place  of  observation  ? 

.    Eceample  1* 

December  22d,  1825,  in  latitude  890'  south,  by  account,  at  23*41?15: 
q)parent  time,  the  true  altitude  of  the  sun's  centre  was  74?  16'  j  required 
the  true  latitude  ? 


*  Thif  is  the  log.  sscaat  of  one  minute,  with  a  modified  index. 

2b2 


Digitized  by 


Google 


872  NAUTICAL  ASTRONOMY. 

Apparent  time  from  noon  =  •  0M8r45!  Log.  riring  =  3-524365 
Latitude  by  account  =  ...  8?  0'  O^S.  Log.  co-sinc  =  9. 995753 
Sun's  corrected  declination  =  .    23. 27.   0  S.  Log.  co-sine  =  9. 962562 

Estimated  meridian  altitude  =  74?33^  Or  Constant  log.=  7.536274 
True  central  altitude  =       .    .    74.16.0    74?16:  Or 

Middle  altitude  ==      ....    74^24 '.30r    Log,  secant  ==   0.576604 

Correction  of  altitude  = +'39^  0rLog.=  1.589558 


Sun's  correct  meridional  altitude  s  •    .    •     74?55C  Or 
Sun's  correct  declination  = 23. 27.   0  south. 


True  latitude  of  the  place  of  observations  .      8?22^  Orsouth;  which  ex- 
actiy  agrees  with  the  result  by  spherical  trigonometry. 

Jfote. — ^By  this  method  of  computation,  an  error  of  one  degree  in  the 
latitude  by  account^  in  places  within  the  tropics^  will  produce  litde  or  no 
effect  on  the  latitude  resulting  from  calculation :  thus,  if  the  latitude  by 
account  be  assumed  at  7?0^,  or  at  9?0^,  the  resulting  latitude,  or  that 
deduced  from  computation,  will  not  differ  more  than  one  minute  from  the 
truth ;  and  the  same  result  would  be  obtuned,  if  the  altitude  were  observed 
at  the  distance  of  an  hour  from  noon  :  provided,  always,  that  the  measure 
of  the  interval  from  noon  be  very  correcdy  known. 

Esdmpie  2. 

December  23d,  1825,  in  latitude  50?0^  N.,  by  account,  at  lM4ri5! 
apparent  time,  the  altitude  of  the  sun's  centre  was  13?58' ;  required  the 
true  latitude  ? 

Time  from  noon  =  .  lM4rl5!  Log.  rising  =  .  .  .  4.716200 
Latitude  by  account  =:  50?  O^N.  Log.  co-sine  =:  .  .  .  9.808068 
Sun's  corrected  dec.  =  23. 27  S.       Log.  co-sine  =:   .     .    .    9. 962562 


Estimated  merid.  alt.=  16?3d(  Constant  log.  =:       .    •    7.536274 

True  central  altitude  =  13.58    .     .       13?58! 


Middle  altitude  =     .     15?15^30r    .    .    .'.    Log.  secant  =:  0.015586 

Correction  of  altitude  =     ...       +  l?49'  =  109C=Log.=2. 038690 

Sun's  correct  meridional  altitude  =:  .     I5?47^ 
Sun's  correct  declination  =      ...     23. 27  soutii. 


Co-latitude  of  the  place  of  observation=:39?  14  C  north ;  hence  the  true  lati- 

/  Google 


Digitized  by ' 


OF  FINDINO  TH£  JLATITUDB  BT  TH£   SUN's  DIAMBTBIU  373 

tude  is  50?46' north,  which  is  2'  less  than  the  result  by  spherical  trigono- 
metry :  the  correct  latitude  being  50"? 48^  north. 

If  die  latitude  by  account  be  assumed  at  51^48 ^,  the  latitude  by  com- 
putation will  be  50^50^ ;  being,  in  this  instance,  only  two  minutes  more 
than  the  truth. 

Note.— The  above  method  of  finding'  the  latitude  is,  as  far  as  I  am 
aware,  perfectly  original ;  it  is  exceedingly  well  arranged,  and  it  affords  a 
direct  and  general  solution  to  the  problem  given,  for  the  same  purpose,  in 
page  354  :  the  apparent  time,  or  the  measure  of  the  interval  from  noon, 
must,  however,  be  very  correctly  known  ;  although,  in  places  distant  from 
the  equator,  or  where  the  sun  does  not  come  very  near  to  the  zenith  of  the 
place  of  observation,  an  error  of  a  few  minutes  in  the  time' will  not  very 
materially  affect  the  latitude  :  thus,  in  the  last  example,  an  error  6f  two 
minutes  in  the  interval  from  noon  would  only  produce  an  error  of  six 
minutes  in  the  latitude;  and  in  the  first  example,  where  the  sun  passes  nearer 
to  the  zenith,  it  would  produce  an  error  of  eight  minutes  in  the  latitude. 

As  this  method  does  not  labour  under  any  restraint,  or  since  it  does  not 
require  that  the  interval  from  noon  should  be  governed  by  the  object's 
meridional  zenith  distance,  the  observation  may  therefore  be  taken  at  any 
hour  before  or  after  the  sun's  transit ;  and  this  is  a  peculiarity  that  gives 
it  a  most  decided  advantage  over  the  method  contained  in  the  above- 
mentioned  page. 


PftOBLBM  XIII. 

Given  the  Lmgiiude  of  a  PUxcCy  the  Sun*s  Declination  and  Semi-diameter, 
and  the  Intertal  of  Time  between  the  Instants  of  hig  Limbs-being  in  th6 
Horizons  tofnd  the  Latitude  of  that  Place. 

Rule. 

Reduce  the  apparent  time,  per  watch,  of  the  rising  or  setting  of  the 
sun's  centre  to  the  corresponding  time  at  Greenwich,  by  Problem  III., 
page  297 ;  to  which  time  let  the  sun's  declination  be  reduced,  by  Problem 
v.,  page  298. 

To  the  logarithm  of  the  sun's  semi-diameter,  reduced  to  seconds,  add 
the  arithmetical  complement  of  the  logarithm  of  the  interval  of  time,  ex- 
pressed in  seconds,  between  the  instants  of  the  sun's  limbs  being  in  the 
horizon,  and  the  constant  logarithm  9. 124939;  the  sum  of  these  three 
logarithms,  rejecting  10  in  the  index^  will  be  the  logarithmic  co-sine  of  an 
arch.    Now, 


Digitized  by 


Google 


374  MAtrricAL  asthonomt. 

To  the  logarithmic  sine  of  the  sum  of  this  arch  and  the  sun'd  reduced 
declination,  add  the  logarithmic  sine  of  their  difference ;  half  the  sum  will 
he  the  logarithmic  sine  of  the  latitude  of  the  place  of  observation* 

Example  1. 

July  13th,  1824^  in  north  latitude,  and  longitude  120?  west,  the  mm's 
lower  limb,  at  the  time  of  its  setting,  was  observed  to  touch  the  horizon  at 
7t59?581  apparent  time,  and  the  upper  limb  at  8M?4! ;  required  tha 
latitude  of  the  place  of  observation } 

Apparenttimeofsun's  setting  =  7*59768!  +  8MT4:  ^2s=     8t  2T  i: 
longitude  120?  west^  in  time  s 8.  0.  0 

Greenwich  time  of  sun's  setting  =      •    .    .    •    ^    •    •    •     16.   2*    1 

Sun's  declination  at  noon^  July  ISth,  1824,  sa     21?49C5ir  N. 
Correctionof  ditto  for  16^ 2?i:  c     .    .     •    .       —  6.   0 


Sun's  reduced  declination  = 21?43^5irN. 

Sun's  semi-diameter  =  15  U5^.  8,  in  seconds  =  945*,  8  Log.s2. 975799 
Interval  of  time  between  the  setting  of  the  sun's 

lower  and  upper  limbs  s  4T6?,  or  246!  Log.  ar.  comp.  =»  7. 609065 
Constant  log.  (the  ar.  comp.  of  the  prop. 

log.  of  24  hours  esteemed  as  minutes)  =s 9. 124939 

Arch= 59?  9^39r    Log.  co-sine  =      9.709803 

Sun's  reduced  declination  s     21. 43. 51  N. 


Sumn    ......    •    80?5S^30r    Log. sine »=  «    ,    9.994489 

DifferencQ«    ,,«.,    37.25.48      Lpg«sine::«  ,    «    9.783755 

Sums    .    .    .  19.778244 

Latitude  of  the  place  of  obs.s  50? 46 ! 34 TN. Log.  sine  =  .    •    9. 889122 

Bsmmple  3. 

October  1st,  1824,  in  north  latitude,  and  longitude  105?  eastp  the  sun'a 
upper  limb,  at  the  time  of  its  rising,  was  observed  to  emerge  from  the 
horizon  at  6^3T43',  and  the  lower  limb  at  6^6T32'. ;  required  the  latitude 
of  the  place  of  observation  ?. 

Apparent  Ume  of  sun's  rising  »  6^3r43:  +  6^6732?  -^^  2  a   6t  5?  7|! 
Longitude  105?  east,  in  time  9    • ,      7,  0,  0 

Greenwich  time  pa^  noon,  September  30th  =;      «    •    •    •    11'  57  7|! 

Digitized  by  VjOOQ IC 


OP  FIimiKO  THB  APPABBMT  TIME.  375 

Sun's  declination  at  noon,  September  30th,  1824,  s  2?52U6?  8. 
Correction  of  ditto  for  11  *5?7i!  =« +10.48 

Sun's  reduced  declination  = 3?  3:341^8. 

Sun's  semi-diameter  s  16^  1'.  2,  in  seconds  =  961"".  2    Ix>g.=2. 982814 
Interval  of  time  between  the  rising  of  the 

sun's  upper  and  lower  limbs=:2?49!,  or  169!  Log.  ar.  comp.s7. 7721 18 
Constant  log.  A       9.124939 

Arch=    .••..••.    40^40'54^  Log.  co-sine  =  9.879866 
Sun's  reduced  declination   =     •      3.   3.34S. 

Sum= .    43?44^28r  Log.  sine  =  .    9.839730 

Differences 37.37.20    Log.  sine  =  .    9.785652 

Sum  =  19.625382 

Latitude  of  the  place  of  observation=40?31 '  N.  Log.  sine  =  .    9. 8I269I 

A^marfc.— -In  this  method  of  finding  the  latitude,  it  is  indispensably 
necessary  that  the  interval  of  time  (per  Mratch)  between  the  instants  of  the 
sun's  lower  and  upper  limbs  touching  the  horizon  be  determined  to  the 
nearest  eecond  ;  otherwise  the  latitude  resulting  therefrom  may  be  subject 
to  a  considerable  error,  particularly  in  places  where  the  limbs  of  that  object 
rise  or  set  in  a  vertical  position ;  which  is  frequently  the  case  in  parts 
within  the  tropics. 


SOLUTION  OP  PROBLEMS  RELATIVE  TO  APPARENT  TIME. 

Tim^,  as  inferred  directly  from  observations  of  the  sun,  is  denominated 
either  apparefii  or  mean  iolar  time.  Apparent  time  is  that  which  is 
deduced  from  altitudes  of  the  sun,  moon,  stars,  or  planets.  Mean  tme 
arises  from  a  twpposed  ufAform  motion  of  the  sun :  hence,  a  mean  solar 
day  is  always  of  the  same  determinate  length ;  but  the  measure  of  an 
apparent  day  is  ever  variable,— being  longer  at  one  time  of  the  year,  and 
shorter  at  another,  than  a  mean  day ;  the  instant  of  ^parent  noon  will, 
therefore,  sometimes  precede,  and  at  other  times  follow,  that  of  mean 
noon.  The  difference  of  those  instants  is  called  the  equation  of  time; 
which  equation  is  expressed  by  the  difference  between  the  sun's  true  right 
ascension  and  his  mean  longitude,  corrected  by  the  equation  of  the  Equi- 
noxes in  right  ascension,  and  converted  into  time  at  the  rate  of  1  minute 
to  every  15  minutes  of  motion,  &c.  &c.  The  equation  of  time  is  always 
equal  to  the  difference  between  the  times  shown  by  an  nniform  or  equable 
going  clock,  and  a  true  sun-dial. 


Digitized  by 


Google 


376  NAUTICAL  ASTRONOMY. 

The  sun's  motion  in  the  Ecliptic  is  constantly  varying,  and  so  is  his 
motion  in  right  ascension ;  but  since  the  latter  is  rendered  further  unequal, 
on  account  of  the  obliquity  of  the  Ecliptic  to  the  Equator,  it  hence  follows 
that  the  intervals  of  the  sun's  return  to  the  same  meridian  become  unequal, 
and  that  he  will  gradually  come  to  the  meridian  of  the  same  place  too  late, 
or  too  early,  every  day,  for  an  uniform  motion,  such  as  that  shown  by  an 
equable  going  watch  or  clock. 

It  is  this  retardation,  or  acceleration  of  the  sun's  coming  to  the  meridian 
of  the  same  place,  that  is  called  the  equation  of  time  ;  which  implies  a  cor* 
rection  additive  to,  or  subtractive  from,  the  apparent  time,  in  order  to 
reduce  it  to  equable  or  mean  time. 

The  equation  of  time  vanishes  at  four  periods  in  the  year,— which  hap- 
pen, at  present,  about  the  15th  of  April,  the  15th  of  June,  the  31st  of 
August,  and  the  24th  of  December ;  because,  at  these  periods,  there  is  no 
difference  between  the  sun's  true  right  ascension  and  his  mean  longitude : 
hence  the  apparent  noon,  at  those  times,  is  equal  to  the  mean  noon.  When 
the  sun's  true  right  ascension  differs  most  from  his  mean  longitude,  the 
equation  of  time  is  greatest:  this  happens,  at  present,  about  the  11th  of 
February,  the  15th  of  May,  the  27th  of  July,  and  the  3d  of  November. 
But,  since  at  those  times  the  diurnal  motion  of  the  sun  in  right  ascension 
is  equal  to  his  mean  motion  in  longitude,  or  59'  8 T,  the  length  of  the  appa- 
rent day,  at  these  four  periods,  is,  therefore,  equal  to  that  of  a  mean  day  ; 
at  all  other  times  of  the  year,  the  lengths  of  the  apparent  and  mean  days 
differ ;  and  it  is  the  accumulation  of  those  differences  that  produces  the 
absolute  equation  of  time. 

The  equation  of  time  is  additioe  from  about  the  25th  of  December  to 
the  15th  of  April,  and,  again,  from  the  16th  of  June  to  the  31st  of  August; 
because,  during  the  interval  between  those  periods,  the  sun  comes  to  the 
meridian  later  than  the  times  indicated  by  a  well-regulated  clock :  but  it 
is  subtractive  from  about  the  16th  of  April  to  the  15th  of  June,  and,  again, 
from  the  1st  of  September  to  the  24th  of  December;  because,  during  the 
interval  between  these  periods,  the  sun  comes  to  the  meridian  earlier  thaa 
the  times  indicated  by  an  equable  going  clock* 

The  equation  of  time  is  contained  in  page  IL  of  the  month  in  the 
Nautical  Almanac ;  but,  since  it  is  calculated  for  the  meridian  of  the  Royal 
Observatory  at  Greenwich,  and  for  noon,  a  correction,  therefore,  becomes 
necessary,  in  order  to  reduce  it  to  any  other  meridian,  and  to  any  given 
time  under  that  meridian.  This  correction  is  to  be  found  by  Problem  V.^ 
page  298 ;  or  by  means  of  Table  XV.^  as  explained  in  page  25. 


Digitized  by 


Google 


OF  nKPIKO  TBB  AFPAB8KT  TIMB.  377 


Problem  I. 

To  find  the  Error  of  a  Watch  or  Chronometer ^  by  equal  Altitudes  of 

the  Sun. 

Rnus. 

In  the  morning,  wh^n  the  sun  is  nearly  in  the  prime  vertical,  or  at  least 
when  he  is  not  less  than  two  hours  distant  from  the  meridian,  let  several 
altitudes  of  his  upper  or  lower  limb  be  taken,  and  the  corresponding  times 
(per  watch)  increased  by  12  hours,  noted  down  in  regular  succession.  In 
the  afternoon,  observe  the  instants  when  the  same  limb  of  the  sun,  taken 
in  the  morning,  comes  to  the  same  altitudes,  and  write  down  each,  aug- 
mented by  24  hours,  opposite  to  its  respective  altitude.  Take  the  means 
of  the  morning  and  of  the  afternoon  times  of  observation ;  add  them  toge- 
ther, and  half  their  sum  will  be  the  time  of  noon,  per  watch,  incorrect. 
The  difference  between  the  means  of  the  morning  and  afternoon  times  will 
be  the  interval  between  the  observations :  with  this  interval,  and  the 
latitude,  enter  Table  XIIL,  and  with  the  interval  and  the  declination, 
corrected  for  longitude,  enter  Table  XIV.;  take  out  the  corresponding 
equations,  noting  whether  they  be  affirmative  or  negative,  agreeably  to  the 
rule  in  page  23  :  then,  with  the  sum  or  difference  of  those  two  equations, 
according  as  they  are  of  the  same  or  of  contrary  signs,  and  the  variation 
of  the  sun's  declination  for  the  given  day,  compute  the  equation  of  equal 
altitudes,  by  th6  said  rule  in  page  23.  .  Now,  to  the  time  of  noon,  per 
watch^  incorrect,  apply  the  equation  of  equal  altitudes,  by  addition  or  sub- 
traction, according  as  its  sign  is  affirmative  or  negative,  and  the  sum  or 
difference  will  be  the  time,  per  watch,  of  apparent  noon,  or  the  instant 
when  the  sun's  centre  was  on  the  meridian  of  the  place  of  observation ; 
the  difference  between  which  and  noon,  or  24  hours,  will  be  the  error  of 
the  watch  for  apparent  time. 

If  the  watch  be  regulated  to  mean  solar  time^  such  as  a  chronometer,  let 
the  equation  of  time  (as  g^ven  in  the  Nautical  Almanac,  and  reduced  to 
the  meridian  of  the  place  of  observation  by  Problem  V.,  page  298,)  be 
applied  to  noon,  or  24  hours,  by  addition  or  subtraction,  according  to  its 
title,  and  the  mean  time  of  noon  will  be  obtained ;  the  difference  between 
which  and  the  time,  per  watch^  of  apparent  noon,  will  be  the  error  of  tiie 
watch  for  mean  solar  time. 

Esample  1. 

March  1st,  1 825,  {civil  time)  in  latitude  50948  ^  N.,  and  longitude  SO?W., 
the  following  equal  altitudes  of  die  sun  were  observed  3  required  the  error 
of  the  watch? 


Digitized  by 


Gbogle 


378' 


ItAVTttAt  AmtOMOMT. 


n'g  Lower  Limb. 

Forenoon  Ttmeti  p. 

Watel 

k°. 

11?56^      . 

•     • 

19^59?47J 

28*  0?58! 

12.   1      . 

►     • 

20. 

0.23 

» 

28.  0.22 

12.   6      . 

»     • 

20. 

0.59 

27.59.46 

12.11       . 

1    /  • 

20. 

1.35 

27.59.10 

12.16      . 

1     • 

• 

20. 

2.11 

• 

»          • 

27.58.34 

Mean  =  .    . 

20* 

0'r59! 

Mean  ae 

27i59T46'. 

Afternoon  mc 

^ans^ 

• 

27.59.46  1 

forenoon  meam 
Sum  = 

b20.  0.59 

Interval  =     < 

7^58r47'. 

48i  Or45:    . 

Time  of  noon,  per  watch,  uncorrected  s=  •    .    24^  0^22^! 

Equation,  Table  Xlli.,  ans.  to  lat. 
50?48'.  and  interval  7'58T47'  =  -*-16r59r;  negative,  because  the  sun  is 

advancing   towards    the 
Equation,  Table  XIV.,  ana.  to  dec.  elevated  pole. 

7?82'25^S.andint.7*58r47*=  —  0.55;  negative,  because  the  sun's 

'         dec.  is  decreasing. 
Sum  of  the  equations  s     »    •    •    — 17^^54^    Prop.  log.  a     .     1.0024 
Variation  of  sun's  declination  a     22 '.  46}^  *      Prop.  log.  »     .    0. 8979 


Equation  of  equal  altitudes  ss     ,     -^22r39r    Prop.  log.  »     .    0. 9003 
Timeof noon, p. watch, uncor.cs24t  07221? 


Time,  per  watch,  of  app.noon=24t  Of  0!    • 24*  0?  0! 

Apparent  noon  =       ...    24.   0.   O+Eq.  of  time  12738! 

ss  mean  noon  s     24. 12. 38 


Watch  true  for  apparent  time=sO*  07  0*    Watch  slow  for 

mean  time  s     I27S8! 

Example  2. 

August  2d,  1825,  {dvU  or  nmUcal  iiine)  in  latitude  50?48^  N,,  and 
longitude  30?  W.,  the  following  equal  altitudes  of  the  sun  were  observed  | 
required  the  error  of  the  watch  ? 


*  Since  the  morning  observationft  belong,  astronomically,  to  Febmaiy  28th,  therefore^ 
half  the  Bum  of  the  variation  of  the  sun^t  declination^  for  the  days  preceding  and  following 
the  given  one,  is  to  be  taken  for  the  true  variation  of  declination. 


Digitized  by 


Google 


OP  FIMDIN6  THU  API»ARSMT  TIME. 


379 


Alt  of  Suu's  Lower  Limb. 


Forenoon  Timesy  p.  Watch. 


32?18:  . 

.  .  20*  3T52! 

32.23   . 

.  .  20.  4.25 

32.28   . 

.  .  20.  4.57 

32.33   . 

.  .  20.  5.30 

32.38   . 

.  .  20.  6.  2 

Afternoon  Times^  p.Watch. 

.  .  28*  3743! 

•  .  28.   3.11 

.  •  28.   2.39 

.  .  28.   2.   6 

.  .  28.    1.34 


Mean«  •    •    .    20^4757^21     Means     28*  2r38!36. 
Afternoon  mean  s  28.  2.38.36Foren.niean=:20.  4.57.12 


Interval  = 


7i57'r4r.24; 


Sum=  48 1  7"35!48f 


Time  of  noon,  per  watch,  uncorrected  =  •    .    24*  3^47 '  54  f 

(Equation,  Table XIIL,  ans.  to  lat. 

50?48C  and  int.  7 '57741!  24!=  +16'r58r;  affirmative,  because  thetun 

is  receding  from  the  ele- 
Equa.»Tab,XIV.,an8.todec.l7^47 C9'r  vated  pole. 

and  interval  7*5774r.24f  =       —  2. 14  ;  negative,  because  the  sun's 

dec.  is  decrea.sing. 

Pifference  of  the  equations  s=:   ,     +  IVAiV    Prop.  log.  s    .    1.0870 
Variation  of  sun's  declination  =:     .     15i23i«  Prop.  log.  »     »     1.0685 

Equation  of  equal  altitudes  ss    •      +12^35^    Prop.  log.  =     •     1.1555 
Timeofnoon,  p.watch,uncor.ss24 1  3747 1 54 ! 


Time, p. watch, of app. noon  =  24*  4?  0!29!  ....    24*  47  0!29! 

Apparent  noon  =      ...    24.   0.   0.   0+Eq.  of  times 

5754!=mean 
noon  =    .     24.   5.54.   0 


Watch  fast  for  apparent  times      47  0\  29 !  Watch  slow  for 

mean  times       l753!31f 


Now,  since  the  equal  altitudes  in  the  two  preceding  examples  have  been 
observed  at  the  same  place,  and  the  times  of  observation  specified  by  the 
same  watch,  the  daily  rate  of  that  machine  may  therefore  be  readily  esta-' 
blished,  upon  the  assumption  of  an  uniform  motion  j  as  follows,  viz., 

March  1st,  1825,  watch  slow  for  mean  time  at  noon  s    12738! 
August  2d,  1825,  watch  slow  for  mean  time  at  noon  s      1. 53| 


Interval  s  154  days. 


Difference  s  10744|! 


Digitized  by 


Google 


380  KAUTICAL  ASTRONOMY. 

Now,  10r444!,  divided  by  154  days,  gives  4*.  185  ;  which,  therefore,  is 
the  daily  rate  gaining. 

Remarks. 

In  finding  the  rate  of  a  watch  or  chronometer,  if  it  be  too  fast  at  the 
time  of  the  first  observation,  and  the  error  increasing,  the  machine  will 
evidently  be  gaining  on  mean  time;  but  if  decreasing,  it  will  be  losing  for 
mean  time.  Again,  if  the  watch  or  chronometer  be  too  slow  at  the  first 
observation,  and  the  error  increasing,  the  machine  will  be  losing  for  mean 
time  3  but  if  decreasing,  it  will  be  gaining  on  mean  time,  as  in  the  case  or 
example  above. 

Since  the  method  of  finding  the  apparent  or  mean  time,  by  equal  alti- 
tudes of  the  sun,  does  -not  indispensably  require  that  the  latitude  of  the 
place  of  observation  and  the  value  of  th^  sun's  declination  be  strictly  deter- 
mined, as  these  elements  are  only  employed  in  taking  out  the  equations 
from  Tables  XIII.  and  XIV. ;  and  since  any  trifling  error  therein  will  not 
sensibly  affect  the  resulting  equation,-— this  method,  therefore,  is  the  best 
adapted  for  practice  on  shore,  where  the  altitudes  may  be  taken  with  a 
sextant,  by  means  of  an  artificial  horizon,  and  the  corresponding  times 
determined  with  the  greatest  exactness.  Nor  is  it  absolutely  necessary 
that  the  instrument  be  very  rigidly  adjusted,  provided  only,  that  it  shows 
the  same  altitude  at  botli  observations. 

In  taking  equal  altitudes,  it  will  be  advisable  for  the  observer  to  fix  the 
index  of  his  sextant  or  quadrant  to  some  particular  division  on  the  arch, 
and  then  wait  till  the  contact  of  the  images  takes  place. 


Problbm  JI. 

To  find  the  Error  of  a  Watch  or  Chronometer ^  by  eqml  Altitudes  of 

a  fixed  Star, 

Rule. 

Let  several  altitudes  of  a  known  fixed  star  be  observed  when  in  the  east- 
ern hemisphere,  and  the  corresponding  times,  per  watch,  noted  down  in 
regular  succession.  When  the  star  is  in  the  western  horizon,  observe  the 
instants  when  it  comes  to  each  of  the  former  altitudes,  and  write  down 
each  opposite  to  its  respective  altitude.  Take  the  means  of  the  eastern 
and  of  the  western  times  of  observation ;  add  them  together,  and  half  their 
sum  will  be  the  time,  per  watch,  of  the  star's  transit  over  the  meridian  of 
the  place  of  observation. 


Digitized  by 


Google 


OP  FINDTKG  THB  APPARBMT  TIMB. 


381 


Compute  the  apparent  time  of  the  star's  transit  over  the  given  meridian, 
by  Problem  XII.,  page  317 ;  the  difference  between  which  and  the  observed 
time  of  transit  will  be  the  error  of  the  watch,  which  will  be  fast  or  slow 
according  as  the  observed  time  of  transit  is  greater  or  less  than  the  com- 
puted time  of  transit. 

Example  I. 

April  24th,  1825,  in  latitude  50?15^  N.,  and  longitude  60?45^  W.,  the 
foUowing  equal  altitudes  of  Arcturus  were  observed ;  required  the  error  of 
the  watch  for  apparent  time  ? 


titode*  of  Aretoni 

26?  4'. 
26.19 
26.34 
20.49 
27.   4 

1*. 

Intern  Timei,  per  Watcli. 

.     7'-  6r41!    .    .    . 
.       7.   8.14      .     . 
.      7.   9.47     .    .    . 
.      7.11.21     .    .    . 
.      7.12.55      .    . 

1        1 

1             4 

Wcftero  Timet,  perWaU 

.     .     16M9r49! 
.    .    16.48.16 
,    .     16.46.43 
.     16.45.   9 
,    .     16.43.35 

Mean  =      7*  9r47^36f 
Mean  of  eastern  times  =    .    . 

Mean 
•    « 

=  16M6r42r24f 
7.   9.47.36 

23!56r30!  Of 

Time  of  star's  transit  over  the  given  mer.,  per  watch,sIlt58Tl5!  Of 

Star's  R.  A.,  reduced  to  night  of  obs.  =s   14?  7?4 1'  •  3 
Sun's  R.  A.  at  noon  of  the  given  day  =     2.   6.  55  •  3 


Approximate  time  of  the  star's  transit  ==  1 2 1  0*46 ' .  0 
Longitude  60?45 :  W.,  in  time  =  •     -f  4,   3.   0 


12t  0r46f 


Girresponding  time  at  Greenwich  =:       16*  8T46! 
Correction  of  transit  answering  to  Greenwich  timd 

1 6 13?46 ! ,  and  variation  of  sun's  right  ascension  3r45 ' .  5  s   -  2TS 1  ? 

App,  time  of  star's  transit  over  the  merid,  of  the  place  of  obs.s=  1 1 158?15 ! 
Apparent  time  of  transit,  per  watch^  = 11. 58. 15 

Wiatch  true  for  apparent  time  =5 .      0?  OT  0: 

Example  2. 

January  1st,  1825,  in  latitude  30?45  *  S.^  and  longitude  75?30C  E.,  at 
7  M7*23!  apparent  time^  per  watch,  the  observed  altitude  of  Sirius,  in  the 


Digitized  by 


Google 


882  NAUTICAL  A8TEOKOMY. 

eastern  hemisphere,  was  33?43U0^,  aiid  at  \Si47Tl9',  when  the  atar 
was  in  the  western  hemisphere,  it  was  observed  again  to  have  the  same 
altitude ;  required  the  error  of  the  watch  for  apparent  time  ? 

Apparent  time,  per  watch,  of  the  obs.  equal  alt.  in  east.  hemi8.=37M7*23; 
Apparent' time,  per  watch,  of  the  obs.  equal  alt.  in  west.  hemis.=  15. 47*  19 


Sum  =s  23?34?42! 


Apparent  time,,  per  watch,  of  star's  transit  over  the  given  mer.:^  1 1 147?21 ! 

Star's  R,  A.,  reduced  to  night  of  observ.=6t37"25'.  6 
Sun's  right  ascension  atnoon,  Jan.  lst,=sI8.47«  19  . 1 

Approximate  time  of  star's  transit    =     1 1  ?50r  6*.  5      .     ll?50r6*.S 
Longitude  75?30^  E.,  in  time  =  .     .  —  5.   2.   0 

Corresponding  time  at  Greenwich  =    .     6M8T  6'.  5 
Correction  of  transit  answering  to  Greenwich  time 

6  M8r6i?  and  variation  of  sun's  right  ascension  4724*.  8s—  lrl5! 


Apparent  time  of  star's  transit  over  merid.  of  place  of  obs.s:  1 1 148*5 1 '  •  5 
Apparent  time  of  transit,  per  watch,  = 11. 47. 21 

Watch  tfZoto  for  apparent  time  =    •    •    » 1T31',5 

Remarks — In  ascertaining  the  error  of  a  watch  by  equal  altitudes  of  a 
fixed  star,  it  will  be  advisable  to  select  one  whose  declination  is  of  the  same 
nafie  with  the  latitude,  and  which  exceeds  it  in  value.  In  high  latitudes, 
the  altitude  most  advantageous  for  observation  may  be  computed  Ky  the 
second  part  of  the  rule  in  pages  120  and  121,^  as  exemplified  in  the  second 
example  of  those  pages. 

In  this  case,  if  the  latitude  of  the  place  of  observation  be  considerably 
distant  from  the  Equator,  the  interval  between  the  times  of  taking  the 
equal  altitudes  will  be  sensibly  contracted;  and,  therefore,  any  probable 
irregularity  in  the  going  of  the  watch,  during  that  interval,  will  be  propor- 
tionably  diminished. 

Example, 

May  1st,  1825,  in  latitude  70?30^  N.,  and  longitude  35?45<  W.,  at 
9^36718!  apparent  time,  per  watch,  the  observed  altitude  of  the  star 
Kochab,  in  the  eastern  hemisphere,  was  77°33C20r,  and,  at  14?577ll! 
that  star,  in  the  western  hemisphere,  was  again  observed  to  have  the  same 
altitude  $  required  (be  error  of  the  watch  for  apparent  time  i 


Digitized  by 


Google 


OF  FINDING  THB  ^PPARBNT  TIME.  383 

Apparent  time,  per  watch,  of  obs.  equal  alt.  in  the  east.  hemis.=9?36?18! 
Apparent  time,  per  watch, of  obs«  equal  alt.  in  the  west,  hemis^^  14. 57*  1 1 


Sum=  24*33:29! 


Apparent  time,  per  watch,  of  star's  transit  over  given  merid.s:  12  ?  16T44J ! 

Star's  R,  A.,  reduced  to  night  of  observ. = 14 15 1  ri  8  * .  6 
Sun's  right  ascension  at  noon,  May  lst=  2, 33. 23  ,9 

Approximate  time  of  transit  =      •    .     12*  17:54*.  7     .    12fl7:64'.7 
Longitude  35^45 '.  W.,  in  time  =       +  2. 23.   0  . 0 

Corresponding  time  at  Greenwich  s=     14*40?54'.7 
Correction  of  transit  answering  to  Greenwich  time  . 

14M0?54\  7,  and  var.of  sun's  right  ascension  3r49!  s      -  2:20\  1 


App.  time  of  star's  transit  over  the  merid.  of  place  of  obs.  s  12*  15T34' .  6 
Apparent  time  of  transit,  per  watch,  = 1.2.16.44  .5 


Watchyiwt  for  apparent  time  =      •..."...•  1?  9*.9 

M>te.— -In  this  example,  since  the  interval  between  the  observations  is 
only  5t20?5S!  (the  star  being  in  the  prime  vertical;  that  is,  bearing  due 
east  and  due  west  at  the  equal  altitude,)  it  is,  therefore,  evident  that  any 
probable  irregularity  in  the  going  of  the  watch,  during  that  interval,  is  less 
liable  to  affect  the  resulting  error  for  apparent  time,  in  any  sensible  man- 
ner, than  if  such  error  had  been  determined  from  observations  compre- 
hending an  interv^  of  9  ^36:55!,  as  in  the  case  of  Example  1,  page  381. 


Pkoblbm  III. 

Oioen  Hie  LcAitude  qfa  Place,  and  the  Altitude  and  DecUnation  of  the 
Sunf  to  find  the  apparent  Time  of  Observation,  and,  thence,  the  Error 
of  a  Watch  or  Chronometer. 

Method  I. 

Rl7U, 

Reduce  the  sun's  declination  to  the  time  and  place  of  observation,  by 
Problem  V.,  page  298 ;  which  being  applied  to  90?,  by  addition  or  sub- 
traction, according  as  it  is  of  a  different  or  of  the  same  denomination  with 
the  latitoiky  tbt  sum  or  remainder  will  be  the  sun's  polar  distance. 


Digitized  by 


Google 


384  NAUTICAL  A9TRONO»fT« 

Reduce  the  observed  altitude  of  the  sun's  limb  to  the  true  central  alti-* 
tude,  by  Problenr  XIV.,  page  320. 

Now,  add  together  the  sun's  true  altitude,  its  polar  distance,  and  the 
latitude  of  the  place  of  observation ;  take  half  the  sum,  and  call  the  differ- 
ence between  it  and  the  son's  true  altitude  the  remcAider. ' 

Then,  to  the  log.  co-secant  of  the  polar  distance,  add  the  log.  secant  of 
the  latitude,  the  log.  co-sine  of  the  half  sum,  the  log.  sine  of  the  remain- 
der, and  the  constant  logarithm  6.301030:  the  sum  of  these  five  loga- 
rithms, abating  20  in  the  index,  will  be  the  log.  rising  answering  to  the 
sun's  distance  from  the  meridian ;  which  will  be  the  apparent  time  at  ship 
or  place,  if  the  observation  be  made  in  the  afternoon ;  but  if  in  the  fore- 
noon, its  complement  to  24  hours  will  be  the  apparent  time  ;  the  diffv- 
ence  between  which  and  the  time  of  observation,  per  watch,  will  be  the 
error  of  the  watch,  and  which  will  be  fast  or  slow  according  as  the  time 
shown  thereby  is  later  or  earlier  than  the  apparent  time. 

JRemcark. — In  practice,  it  •becomes  absolutely  necessary  to  take  several 
altitudes  of  the  sun's  limb,  and  to  note  the  corresponding  times  per  watch; 
then,  the  sum  of  the  altitudes,  divided  by  their  number,  «gives  the  mean 
altitude,-— and  the  sum  of  the  times,  so  divided,  gives  the  mean  time. 


Example  I. 

January  1st,  1825,  in  latitude  40? 27'  N.,  and  longitude  54 940 C  W.,  the 
following  altitudes  of  the  sun's  lower  limb  were  observed^  the  height  of  the 
eye  above  the  level  of  the  sea  being  20  feet ;  required  the  apparent  time 
of  observation  and  the  error  of  tiie  watch  ? 


Mean  time  of  observation,  per  watch,  :=       ...      3?  2?  0! 
Longitude  54?40^  W.,  in  time  =       ....      +3.38.40 


Greenwich  time  = 6MOT40! 


Sun's  declination  at  noon,  January  Ist,  =    •    •     •    23?  0'39?S. 
Correction  of  ditto  for  6  M0T40t  = -1.27 


Sun's  reduced  declination  =     .....;•    22?59:32rS. 


Sun's  north  polar  distance  ss    * I12?59.32r 

/Google 


Digitized  by ' 


OF  FINDING  THB  APPARBNT  TIMB.  885 

Time,  pe^  Watch.  Altitude  of  San'g  Lower  Limb, 

3*  0r30!        '  13?49M0r 

3.    1.15  .  13.44.  0 

3.   2.   0  13.38.10 

3.   2.45  18.32.30 

3.  3.30  13.26.40 


10?  0!  191 :  0? 


Mean=  3?  2T  Of     Mean=  13?38n2r 
Sun'sseini.diam.l6a8r-dip4tl7r=  4-12.    1 


Sun's  apparent  Altitude  =   •    .    .     13?50n3i: 
Refraction 3C48r-Parallax  0^9^-     —  3. 39 


Sun's  true  central  altitude  s  .  .  13?46 C34r  Constant  log. =6. 301030 
Sun's  north  polar  distance  =  .  .  1 12. 59. 32  Log.  co-sec«*sO.  035949 
Lat.  of  the  place  of  obsenration  =     40.  27.   0   Log.  secant*  =0. 1 18631 


Sum  =    .     .    •  167?  13:  6C 


Half  s{im  =  .    .    83?36133^  Log.  co-sines  9. 046534 
Remainder  =s     .    69.49.59   Log.  sine  =      9.972523 


Apparent  time  of  observation  =     .    3*   1*45 !  Log.  rising  =  5. 47466. 7 
Time  of  observation^  per  watch,  =     8.    2.   0 

Watcb^l  fpr  apparent  time  =     •  15  seconds. 


Example  2. 

.June  9th,  1825,  in  latitude  50?40^  N.,  and  longitude  47^56^  15?  E.^  the 
following  altitudes  of  the  sun's  lower  limb  were  observed,  the  height  of  the 
eye  above  the  level  of  the.  horizon  being  23  feet ;  required  the  apparent 
time  of  observation,  and  the  error  of  the  watch  ? 

Time  of  observation,  per  watch,  =  ....     19*22725! 
Longitude  47'?56: 15?  E.,  in  time  =     .    .      -3.11.45 

Greenwich  time »  ^    .    , 16M0?40! 


*  The  lOf  are  rejected  from  the  indices  of  the  lo^rithmic  secant  and  co-secant ;  and, 
with  the  view  of  facilitating'  the  future  operationa  in  this  work,  the  same  plan  will  be  pur- 
sued in  all  the  coDputations. 

2c 


Digitized  by 


Google 


886  NAUTICAL  AlTRONOMT« 

Sun's  declination  at  noon,  June  9tbji  =     ,    ,    22?56^37?N. 
Correction  of  ditto  for  16*  10?40!  =..    •    .       +3.16 


Sun's  reduced  declin«tiQa  » 22?59C53rN. 

Sun'a  north  polar  dUtance  =s      .    .    .    •    ,    67?  Of  7' 

Hme,  per  Watch. .  Altitude  of  aon'i  Lower  Uub. 

19i20T45!  29?33aor 

19.21.35  •  29.25.10 

19.22.25  29.17.30 

19.23.15  29.  9.40 

19.24.  5  29,  1.50 


112r  5!  87^207 


Mean  aq  l'9?22r25!  Mean  =  29?17;28r :  hence  the 

traa  cenbral  Altitude  is  29?27f  7? 

Sun's  true  central  altitude  s     .    89^^7-7' 

Sun's  north  polar  distance  ss    .    67<  0.   7     Log-  oo-secahtssO;  035967 

Latitude  of  the  place  of  observ.B  50. 40.  0    Log.  Meant  =    0.198026 

Sum  =   .   .   .  147?  7n4r 


Half  sums.  .  78?33C37?  Lpg.  «o^  k  9.45)797 
Remainders  .  44.  6.30  Log. sine  =  .  9.842620 
Constant  log.  k 6.  S01080 

Sun's  distance  from  the  meridian  =  4t44?l  1 '  Log.  rinng  s  5. 82944. 0 

Apparent  time  of  obaervatiou  s      19M5*49! 
Tlo^e  of  observation^  per  watch^  a  19. 22. 25 

Watehjtut  for  apparent  time  ■■  6?9e! 

Note, — SiocQ  the  log.  rising^  in  Tibbie  XXXIL,  is  only  computed  to  five 
places  of  decimals^  therefore^  in  taKing  out  (he  meridian  distance  of  a 
celestial  oBject  from  that  TaUe^  answering  to  a  given  log.  rising,*  the  sixth 
or  right-hand  figure  of  such  given  log.  rising,  is  tn  be  rej«<;ted;  observing, 
however,  to  increase  the  fifth  or  preceding  figure  by  unity  or  1,.  when  the 
figure  so  rejected  amounts  to  5  or  upwards :  thus,  in  the  preceding  example, 
where  the  log.  rising  is  5, 474667^  th^  meridiftQ  distan<:e  ia  tatoi  oiH  for 
5. 47467  i  and  so  on  of  others* 


Digitized  by 


Google 


OF  FINDING  THB  APPARBNT  TIMS,  387 

For  the  principles  on  which  the  meHdian  distance  of  a  celestial  object 
is  computed,  and  hence  the  apparent  time,  the  reader  is  referred  to  '^The 
Young  Navigatbr's  Guide  to  the  Sidereal  and  Planetary  Fterts  of  Nautical 
Astronomy,''  page  156. 

Eemarks. 

Altitudes  for  ascertaining  the  ^rror  of  a  watch  ought  to  be  taken  by 
means  of  an  artificial  horizon :  one  produced  by  pure  quicksilver  should  be 
preferred,  because  it  shows,  at  all  times,  when  placed  in  a  proper  position^ 
a  truly  horizontal  planer  and,  therefore,  the  angles  of  altitude  taken  therein 
are  always  as  correct  as  the  divisions  on  the  sextant  with  which  those 
angles  are  observed ;  whereaa,  altitudes  taken  by  means  of  the  sea  horizon 
are  generally  subject  to  some  degree  of  uncertainty,  owing  to  its  being 
frequently  broken  or  ill-defined,  by  atmospherical  haze,  at  the  time  of 
observation ;  though  such  dtitudes  are,  nevertheless,  sufficiently  correct  for 
finding  the  longitude  at  sea« 

In  taking  altitudes  by  means  of  an  artificial  horizon,  it  is  to  be  observed, 
that  the  angle  shown  by  the  sextant  will  be  donble  the  altitude  of  the  ob- 
served limb  of  the  object ;  which  is  to  be  corrected  for  index  error,  if  any  : 
then,  half  the  corrected  angle  will  be  the  observed  altitude  of  the  object's 
limb  above  the  true  horizontal  plane;  to  which,  if  its  semi-diameter, 
refraction,  and  parallax  be  applied,  the  true  central  altitude  of  the  observed 
object  will  be  obtained.  There  is  no  correction  necessary  for  dg>,  because 
the  quicksilver  shows  a  truly  horizontal  plane,  as  has  been  before  remarked.* 

The  position  of  a  celestial  object  most  favourable  for  determining  the 
apparent  time  with  the  greatest  accuracy,  b,  when  it  is  in  the  prime 
vertical ;  that  is^  when  it  bears  either  due  east  or  due  west  at  the  place  of 
observation,  or,  if  it  be  circumpolar,  when  it  is  in  that  part  of  its  diurnal 
path  which  is  in  contact  with  an  azimuth  circle ;  viz.,  when  the  log.  sine 
of  its  altitude  =  log.  sine  of  the  latitude  +  radius  —  log.  sine  of  its  declina- 
tion ;  because,  then,  the  change  of  altitude  is  quickest,  and  the  extreme 
accuracy  of  the  latitude  not  very  essentially  requisite^  The  nearer  a  celes- 
tial object  is  to  either  of  these  positions,  the  nearer  will  the  apparent  time, 
deduced  from  its  altitude,  be  to  the  truth  $  as,  then,  the  unavoidable  small 
erron  which  generally  creep  into  the  observations,  or  a  few  miles  differ- 
ence in  the  latitude,  will  have  little  or  no  eiFect  on  the  apparent  time  so 
deduced^ 

Table  XLVII.  contains  the  time  or  distance  of  a  celestial  object  from 
the  meridian  at  which  its.  altitude  should  be  observed,  in  order  to  determine 
the  apparent  time  widi  the  greatest  accuracy ;  and  Table  XLVIII.  contains 


*  Tfae  difsct  mlis  for  applying  the  necessary  corrections  to  altitudes  taken  on  shore  by 
means  of  an  aititfcial  baHcon,  wiU  be  found  at  the  end  of  tlM  Gstopcndiiai  of  Practical 
^laTigatioDy  towards  the  latter  part  of  this  Tolame. 

2c2 


Digitized  by 


Google 


388  .  NAimCAJL  ASTRONOMY, 

the  corresponding  altitude  most  advantageous  for  observation.  But,  since 
those  Tables  are  adapted  to  the  declination  of  a  celestial  object  when  it  b 
of  the  same  name  with  the  latitude  of  the  place  of  observation,  th^y  will 
not,  therefore,  indicate  either  the  proper  time  or  the  altitude  when  those 
elements  are  of  contrary  denominations  :  in  this,  case,  since  the  sun  or  other 
celestial  object  comes  to  the  prime  vertical  before  it  rises,  atid  therefore 
does  i\pt  bear  due  east  or  west  while  above  the  horizon,  the  observation 
for  determining  the  apparent  time  from  its  altitude 'must  be  made  while 
the  object  is  neax  to  the  horizon ;  taking  care,  however,  not  to  take  an 
altitude  for  that  purpose  under  3  or  4  degrees,  on  account  of  the  uncertain 
manner  in  which  the  atmospheric  refraction  acts  upon  very  small  angles  of 
altitude  observed  adjacent  to  the  horizon.-— See  explanation'  to  the  above- 
mentioned  Tables,  pages  119  and  120.. 


Mbthod  II. 

Of  computing  the  horary  Distance  of  a  celestial  Object  from  the 

Meridian. 

RULBi 

If  the  latitude  of  the  place  of  observation  and  the  declination  of  the 
celestial  object  be  of  diflFerent  names,  let  their  sum  be  taken,— otherwise, 
their  difference, — and  the  meridional  zenith  distance  of  the  object  will  be 
obtained;  to  which  apply  its  observed  zetiith  distance,  by  addition  and 
subtractioii,  and  let  half  the  sum  and  half  the  difference  be  taken ;  then, 

To  the  log.  secant  of  the  latitude  add  the  log.  secant  of  the  declination, 
the  log.  sine  of  the  half  sum,  the  log.  sine  of  the  half  difference,  and  the 
constant  logarithm  6.301630;  the  sum  of  these  five  logarithms,  abifting 
20  in  the  index,  will  be  the  log.  rising  of  the  object's  horary  distance  from 
the  meridian ;  and  if  this  object  be  the  sun,  the  apparent  time  will  be 
known,  as  in  the  last  method;  and,  hence,  the  error  of  the  watch,  if 
necessary. 

Example  1. 

January  lOth,  1825,  in  latitude  40^30^:  N.  and  longitude  59?2'30r W., 
the  mean  of  several  observed  altitudes  of  the  sun's  lower  limb  was 
14?31  '47'',  that  of  the  correspondmg  times,  per  watch,  3*  IT45!,  and  the 
height  of  the  eye  above  the  level  of  the  horizon  .18  feet;  required  the 
apparent  time  of  observation,  and  the  error  of  the  watch  ? 

Time  of  observation,  per  watch,  =     .    .  '    3?  IT45! 
Longitude  59?2^30TW.,  in  time  =  .      +  3. 56. 10 


Greenwich  time  =s    .    .    •    ..    •    .    .      6*57*55! 

/Google 


Digitized  by ' 


OF    FINDING  THB  APPARENT  TIMB,  389 

Sun's  declination  at  noon,  January  lOth^  =s    21?57 -50^  S. 
Correction  of  ditto  for  6t5i7"55!  =5     .     •         —  2,40 


Sun's  reduced  declination  s= 21?55'10^S. 

Obs.  alt  of  sun's  U  limb=14?31M7^3  hence,  iU  true  cent,  alt  isl4?41CS6! 


Sun's  true  zenith  distance  at  time  of  observation  s      .    ;    ;    75  ?  1 8 '  24  T 

Latitudes.    •    40?30'  OfN Log. secantsO.  118954 

Declinations*     21.55. 10  S.     .    •     .    •    .    Log.  secant=0. 032588 

Sun's  mer.z.dist=  62?25  C  lOT 

Obs.  zenith  dist.  =  75. 18. 24 Const  log.s  6. 301030 


Sum=    .    .    .  137M3'.34rHalf=68?51M7^  Log.  sine  =  9.969752 
Difference  =      .     I2?53a4r  Half=  6.26.37     Log.  sine  s  9.050091 


Sun's  dist  from  the  mer.=theapp.  time=3M?15!Log.risings5.47241.5 
Time  of  observation,  per  watch,  =  •    •    3. 1. 45 

Watch  ya«<  for  apparent  time  =      •    •        0?30! 

Example  2. 

January  20th,  1825,  in  latitude  37^20^8.  and  longitude  49?45'  £., 
the  mean  of  several  altitudes  of  the  sun's  lower  limb  was  26?39C  157,  that 
of  the  corresponding  times,  per  watch,  19M 1  ?45 !,  and  the  height  of  the 
eye  above  the  level  of  the  horizon  16  feet  3  required  the  apparent  time  of 
observation,  and  the  error  of  the  watch  ? 

Time  of  observation,  per  watch,  =     •    •     19*1 1?45  !• 
Longitude  49?45^  £.,  in  time  s  .    •      -  3. 19.   0 

Greenwich  time  = '  15*52T45! 

Sun's  declination  at  noon,  January  20th,=  20?  7 '  1 K  S. 
Correction  of  ditto  for  15*52?45!  =  .    .     -  8.45 


Sun's  reduced  declination  ==     •    .    •    •     19?58'26rS. 
Obs.  alt  of  sun's  1.  limb  s  26?39^  1575hence,its  truecentalt  is26?49i58^ 


Sun's  true  zenith  distance  at  the  time  of  observation  s    .    •    63?  10'  1" 

Digitized  by  VjOOQ IC 


990  NAUTICAL  ASTKONOMY. 

Latitude  =».    .    87 -20^0^8 Logr. MCiuit^O. 099567 

Declinations:.     19.58.26  S Log.  secaatsO.  026942 

Sun'smer.s.  clist.»  1 7?  21 1 34? 

Obs.  zenith  di8t.  =  63. 10.   2 Const.  1(^.=  6.301030 


Sum*    .    .    .   .80°81^36r  Halfa40?15M8r  Log.  sine  «=  9.810435 
Differences     .45.48.28    Half=22. 54. 14    Log.  sine  =  9. 590158 

Sun's  horary  distance  from  the  merid.=4M3?42!Log.rising=5.82813.2 

Apparent  time  of  observation  =  .     .     19^  16T 187 
Time  of  observation,  per  watch,  =    .     19.11.45 


Watch  *Ioto  for  apparent  time  =      .  4?8S! 


Method  UI. 

C>f  compuHng  (Ae  Aorary  Ditttance  of  a  celestial  Object  jrom  th^ 

Meridian. 

Rule. 

• 
If  the  latitude  of  the  place  of  observation  and  the  declination  of  the 

celestial  object  are  of  different  names,  let  their  sum  be  taken, — othenrise, 

their  differ ence^ — and  the  meridional  zenith  distance  of  the  object  will  be 

obtained;  the  natural  versed  sine  of  which,  being  subtracted  from  the  natural 

co-versed  sine  of  the  object's  true  altitude,  will  leave  a  remainder*    Now, 

to  the  logarithm  of  this  remainder  add  the  log.  secants  of  the  latitude  and 

the  declination,  and  the  sum  will  b^  the  log.  rising  of  the  object's  horary 

distance  from  the  meridian ;  and  if  this  object  be  the  sun,  the  apparent 

time  willbe  known,  and,  hence,  the  error  of  the  watch,  if  required,  as 

shown  in  the  first  method,  page  384. 

Eaanq^le  !• 

May  1st,  1825,  in  latitude  40?35'  S.,  md  longitode  63?  15 f  EL,  the 
mean  of  several  altitudes  of  the  sim's  lower  limb  was  19?43^581';  that  of 
the.correspondipg  times,  per  watch,  20^57*45?,  and  the  height  of  the  eye 
ahove  the  level  of  the  sea  14  feet;  required  the  apparent  time  of  observa- 
tion, and  the  error  of  the  watch  ? 

Time  of  observation,  per  watch,  =     .     .     20*  57  "45 ! 
Longitude  63?  151  £;,  in  time  s^  •    .     —  4, 13..  0 

Greenwich  time  s=      •    .         #    .    ,    ♦     16*44?45* 

Digitized  by  VjOOQ IC 


OF  FINDINO  TBB  APPARBNT  TIME.  891 

Sun's  declination  at  noon,  May  Ist,  ^     15?  4C19?N. 
Correctionof  ditto  for  16*44r45!    =+   12.84 

Sun's  reduced  declination  ss  .   •     .     •     15?16'53^N. 

Obs.  alt.  of  sun's  I  Umbial9?43^58r|  henoe^  the  true  cent,  alt  isl9?53U7^ 

Latitude  =.    .    40?85{  OTS.     .....    Log.  secantvO.  119495 

Reduced  dec.  =:     15. 16.58  N.     .....    Log. secantsO.  015634 

Sun's  mer.  z.dis.=  55?5 1  ;53rNat.V.S.  =     43885 1 
Sun's  true  alt.  »     1 9. 53. 47  Nat.co- V.S.=659680 


Remainder  =»  220829  Log.  s    5.344056 


Sun's  horary  distance  from  the  merid.s:  3^  2T45'Log.risings=5. 47918.5 

Apparent  time  of  observation  =   •     .     20*. 57*  15 ! 
Time  of  observation,  per  watch,  =  •     20. 57-  45 

Watch  y<M/ for  apparent  time  =s  .    .  0*30! 


Example  2. 

November  10th,  1825,  in  latitude  49?  13^  S.,  and  longitude  S6t50'  W.^ 
the  mean  of  several  altitudes  of  the  sun's  lower  limb  was  22?28'30?,  the 
mean  of  the  corresponding  times,  per  watch,  5M?25!,  and  the  height  of 
the  eye  above  the  level  of  the  horizon  20  feet ;  required  the  apparent 
time  of  observation,  and  the  error  of  the  watch  ? 

Time  of  observation,  per  watch,  =     •     .      5  ?  4?25 ! 
Longitude  36?50' W.,  in  time  ±=       .      +  2. 27. 20 


Greentnch  time  ss 7*3ir45f 

Sun's  deelination  at  noon,  Nov.  10th, «    •    17?  9^50r  S. 
Correction  for  7*31?45!  = +  J.  1* 


Sun's  reduoed  decimation  »  .    .    /  .    .     17^15'  5r  S. 
Ob6«  alt.  of  rail's  1.  fimbss22^?28:30r;  heoce^  its  tiroi  cent.  alt.is22?38^  17"^ 

/Google 


Digitized  by ' 


392  NAUTICAL   ASraOKOMY. 

Latitudes.    .    49^3'  OrS Log. secantsO.  184954 

Reduced  dec.  =     17. 15.   5  S Log.  8ecant=0. 019991 


Suii*8mcr.z.di8t.=s31?57^55rNat.V.S.  =     151631 
Sun's  true  alt.  =    22. 38. 17  Nat.co-V.S.=615091 


Reminder  =  463460    Logi-    5.666012 

Sun's  dist.  from  the  mer.ssthe  appar.  time=5*  0T25!Log.ri8.=:5. 87095.7 
Time  of  observation,  per  watch,  =     •    •    5.   4. 25 

Watch^o^  for  apparent  time  =  ...  4?  0! 


Mbthod  IV. 

Cf  computing  the  horary  Diiicmce  qf  a  cekitial  Object  from  the 

Meridian. 

Rule. 

If  the  latitude  of  the  place  of  observation  and  the  declination  of  the 
celestial  object  be  of  different  names,  let  their  mm  be  taken, — otherwise, 
their  dtj^erenee,— and  the  meridional  zenith  distance  of  the  object  will  be 
obtained  ^  from  the  natural  co-sine  of  which,  subtract  the  natural  sine  of 
the  object's  true  altitude,  and  to  the  logarithm  of  the  remainder  add  the 
log.  secants  of  the  latitude  and  the  declination;. and  the  sum  will  be  the 
log.  rising  of  the  object's  horary  distance  from  the  meridian. .  Now,  if 
this  object  be  the  sun,  the  apparent  time  is  known,  and,  hence^  the  error 
of  the  watch,  if  required,  as  shown  in  the  first  method,  page  384. 

Example  I. 

July  4th,  1 825,  in  latitude  39?47 '  S.,  and  longitude  60?50'  E.^  the  mean 
of  several  altitudes  of  the  sun's  lower  limb  was  ld?2'30T,  that  of  the 
corresponding  times,  per  watch,  3M0T45!,  and  the  height  oT  the  eye 
above  the  level  of  the  horizon  22  feet ;  required  the  apparent  time,  and 
the  error  of  the  watch  ? 

Time  of  observatioiij  per  watch,  =   •    .      3M0T45! 
Longitude  6Q?50^  E.,  in  tune  =    •    •      -  4.   3.20 

Greenwich  time  past  noon  of  July  3d  s   23 1  7*25' 

Digitized  by  VjOOQ IC 


OF   FINDING  THB  APPARBNt  TIME.  393 

Sun's  declination  at  noon,  July  3d,  =     22?59C30?N, 
Correction  of  ditto  for  23 ^  7*25 !  =    .       —  4. 48 


Sun's  reduced  declination  s      .    .     .     22^54^ 42rN. 
Obs.  alt.  of  the  sun's  1.  limB=  13?2^30^;  hence,  its  true  cent.  alt.  is  13?9'53r 

Latitudes.     .    39?47'  OrS. Log. sccant=0. 114373 

Reduced  dec.  as    22. 54. 42  N •    Log.  secaiits=0. 03^90 

San's  mer.  z.  dist.=:62?41  '42?Nat.  co-8ine=458727 
Sun's  true  alt.  =     13.   9.53  Nat  sine  =    2*27751 


Remainder  =  230976    Log.  =    5.363567 

Sun's  dist.  from  the  mer.sithe  appar.  time=3*  10?35 !  Log.ris.s5. 5 1363. 0 
Time  of  observation,  per  watch,  =s     .     .    3. 10. 45 

Watch /(wKor  apparent  tirtie=    .    •     .  OTIO! 

Example  2. 

July  19th,  1825,  in  latitude  40?  10^50'/  N.,  and  longitude  53920^  W., 
the  mean  of  several  altitudes- of  the  sun's  lower  limb  was  33^23^5?,  that 
of  the  corresponding  times,  per  watch,  19*47*30',  and  the  height  of  the 
eye  above  the  level  of  the  horizon  15  feet;  required  the  apparent  time, 
and  the  error  of  the  watch  ?  .  ^ 

Time  of  observation,  per  watch,  =     •    .     19*47"30! 
Longitude  53? 20 'W.,  in  time  =  .    .      +  3.33.20 


Greenwich  time  = 23*20?50! 

Sun's  declination  at  noon,  July  19th,  =  20?53nKN. 
Correction  of  dittoVor23»20?50!   =      -  10.44 


Sun's  reduced  declination  :^     .    .    .    20?42^271^N. 

Obs.  alt.  of  sun's  I.  limb=33?  23  '15^;  hence,  the  true  cent.  alt.  is  33  ?34 '.  1 Z 

Latitude  =  .    .    40?10^50?N Log.  secant=0. 1.16898 

Reduced  dec.  =     20.42.27  N Log.  secant=0. 029004 

Sun's  mer.  z.  dist.s  19?28^  23r  Nat.  co.«ne=  942798 
liun's  true  alt.  s=    33i.34.    1     Nat.  sine  s     552011 


Remainder  =  389887    Lbg.=  5. 590939 
Sun's  horary  distance  from  the  merid.s=4!  1  lr53!  Log.  rising=:5. 73684. 1 

Digitized  by  VjOOQ IC 


394  NAUTICAL  ABTRONOICT. 

Sun's  horary  distance  frdm  the  merid.=4  't  1 1  ?53 ! 

Apparent  time  of  Qbseiyation  =     •     19*48?  7' 
Time  of  observation^  per  watch^   s=     19. 47*  30 

Watch  tUno  for  apparent  time  a:     .  0737 ' 


Problek  IV.. 

Given  the  Latitude  of  a  Place,  tlw  JUitude,  Right  jtscenrion,  and 
Declination  of  a  known  fixed  Star,  and  the  Sun'i  Right  Ascension; 
to  find  the  apparent  Time,  atid,  hence,  the  Error  qfthe  iVatch, 

RULB. 

Find  the  true  altitude  of  the  star,  by  Problem  XVIL,  page  327 ;  and  let 
its  right  ascension  and  declination,  as  given  in  Table  XLIV.,  be  reduced  to 
the  night  of  observation ;  then. 

With  the  latitude  of  the  place,  the  star's  true  altitude,  and  its  reduced 
declination,  compute  its  horary  distance  from  the  meridian,  by  any  of  the 
methods  given  in  the  last  problem.  .       . 

Now,  if  the  star  be  observed  in  the  western  hemisphere,  let  its  meri- 
dian distance,  thus  found,  be  added  to  its  reduced  right  ascension,  but,  if 
in  the  eastern  hemisphere,  subtracted  from  it,  and  the  sum  or  remainder 
will  be  the  rijght  ascension  of  the  meridian ;  from  which,  (increased  by  24 
hours,  if  necessary,)  subtract  the  sun's  right  ascension  at  noon  of  the  given 
day,  and  the  remainder  will  be  the  approximate  time  of  observation, 
fleduce  this  to  Greenwich  time,  by  Problem  111.,  page  297>  and  find  the 
proportional  part  of  the  variation  of  the  sun's  right  ascension,  for  the 
given  day,  answering  thereto  and  24  hours,  •by  Problem  XII.,  page  317  ;  . 
which,  being  subtracted  from  the  approximate,  will  give  the  apparent  time 
of  observation :  hence  the  error  of  the  watch  may  be  known. 

Note. — For  the  principles  of  this  nile^  see  ^^  The  Young  Navigator's 
Guide,"  page  156. 

Example  U 

January  1st,  1825,  in  latitude  40?29'  N.,  and  longitude  59?45f  W.,  the 
mean  of  several  altitudes  of  a  Arietis,  west  of  the  mcHdian,  waa  80?29  '481| 
that  of  the  corresponding  times,  per  watch,  11*9T29!,  and  the  height  of 
the  eye  above  the  level  of  the  sea  19  feet;  required  the  apparent  time,  and 
the  error  of  the  watch } 


Digitized  by 


Google 


OF  FINBIKO  THB   APPARBNT  TIMB.  395 

Reduced  R.  A.  of  a  Arietis=l  ^57^19^  and iu  reduced dec,s22'?37'50rN. 

Observed  alt.  of  a  Arieti8=r36?29C48r;  hence,  its  true  alt.  is  36?24f  20'r 

True  zenith  distance  of  •  Arietis  s        •.••••    53  ?35U07 

LatiUides  .    .    40?29'  OrN. Log.  secant :=:0. 118847 

Star's  red.  dec.  =a  22. 37. 50  N.     ....     .     Log.  secant^O.  034796 

Star's  mer.  2.  dis.=  17?51  ClOi: 

Star's  obs  z.dist.=:53.35.40     .......    Const.  log.=  6.^1030 

Sum  =     ...  .  71?26^50r  Half=35?43:25r  Log.  sine  =  9,766321 
Differences      •    35.. 44. 30    Half=:17.52. 15     Log.  sine  =  9:486958 

Star's  horary  dist^-west  of  the  merid.^  4^  2T45 1  L(«.Tindg»5.70795. 2 
Star's  reduced  right  ascension  »  ..    •     L57«19 

Right  ascension  of  the  meridian  s=  .    6?  OT  4! 
Sun'sR.  A.  atnoon^January  Ist  =s  .  18.47.19 

Approximate  time  =     .....  ll*12?45!   ....     llM2r45! 
Longitude59?45^W.,  in  time  =     +3.59.   0 

OreennHcli  time s     ......  15Ml?457 

Correction  of  approximate  time,  ans.  to  Qreenwich  time 

15nir45!,aiidva£iatbn6fsun'sR.A.4?24'-.8,  tft    •    •       ^  2?48! 

Apparent  time  of  observation  = ';     .     •     .     11V9"57- 

Time  of  observation,  per  watch,  = 11.   9. 29 

Watch  Woio  for  apparent  time  = 0T28' 

Example  2. 

January  1st,  1 825,  in  latitude  89?20r30rS.,  and  Iqngitnde  75?40'  B., 
the  odeanof  aereral  altitudes  of  Procyon,east  of  the  meridian,  was  27?  15  ^47^> 
th&t  of  the  corresponding  times,  per  watch,  9^30T23!,  and  the  height  of 
the  eye  above  the  level  of  the  sea  19  feet ;  required  the  apparent  time  of 
observation,  and  the  error  of  the  watch  ? 

Procyon's reduced  R.  A.=7'30r8!,  and  its  reduced  dec.=5?39C58fN. 
Observed  alt.  of  Procyon=27?i5U7^,  hence,  its  true  alt.  is=27?9:46r 

Digitized  by  VjOOQ IC 


396  NAUTICAL  ASTRONOMY. 

Latitudes.    .    39?20i3<KS.     .....    Log. 8ecant=0. 1 1 1607 

Star's  red.  dec.  =     5,39.58  N Log.  secant^O.  002128 

Procyon'8m.z.dis.=45°  0^28'Nat.vers.S.=292989 
Procyon*8trueaIt.=27.   9.46Nat.c.o-V.S.=548480 

Remainder  s  250491     Logi  =   5.398792 

——^1^   I  '  I  III* 

Procyon's  horary  dbU,  cast  of  the  iner.=3i  10T20!  Log,ri8ing=55. 5 1252, 7 
Procyon's  right  ascension  =     ...    7*  30.   8 

Right  ascension  of  the  meridian  ==     •    4 *  1 9T48 ! 
Sun'sjrightasclttnoon^  Jan.  Ist^  =     18.47.19 

Approximate  time  = 9*32r29t    .    .    •    .    9i32r29: 

Longitude  75940^  K,  in  time  =  .     -  5.   2.40 

Greenwich  time  =......     4?29T49! 

Correction  of  approximate  time»  answering  to  Greenwich  time 

4t  29r49!,  and  variation  of  sun's  right  ascension  4r24' .  8  =    —  OTSO! 

Apparent  time  of  observation  =    •    •    • 9^31?39! 

lime  of  observation,  per  watch,  s     .    r 9. 30. 23 

Watch  floto  for  apparent  time  = •    •  1*16! 

fiote^ — When  the  star's  horaiy  distance  ecist  of  the  meridian  exceeds  the 
right  ascension,  the  latter  is  to  be  increased  by  24  hours,  in  order  ^  find  the 
right  ascension  of  the  meridian. 

In  finding  the  error  of  a  watch  by  sidereal  observation,  two  dr  more  stars 
should  be  observed,  and  the  error  of  the  watch  deduced  from  each  star 
separately.  And,  if  an  equal  number  of  stars  be  observed  on  different  sides 
of  the  meridian,  and  nearly  equidistant  therefirom,  it  will  conduce  to  still 
greater  accuracy ;  because,  then,  the  errors  of  the  instrument  and  the 
unavoidable  errors  of  observation  will  have  a  mutual  tendency  to  correct 
each  other.  The  mean  of  the  errors,  thus  deduced,  should  be  taken  for  the 
absolute  error  of  the  watch.  • 


Digitized  by 


Google 


OP  FINDING  THE  APPARENT  TIME.  397 


Problem  V. 

Oiven  the  JjsHiude  and  Longitude  qf  a  Place^  and  the  Altitude  of  a 
Planet,  to  find  the  Apparent  Time  of  Observation. 

Rule. 

Reduce  the  estimated  time  of  observation  to  the  meridian  of  Greenwich, 
by  Problem  III.,  page  297 ;  to  which  time  let  the  planet's  right  ascension 
and  declination  be  reduced,  by  Problem-  VI L,  page  307 ;  and  let  the  sun's 
right  ascension,  at  noon  of  the  given  day,  be  also  reduced  to  that  time,  by 
Problem  V.,  page  298.  Reduce  the  observed  central  altitude  of  the  planet 
to  its  true  central  altitude,  by  Problem  XVI.,  page  325. 

l*hen,  with  the  latitude  of  the  place,  the  planet's  reduced  .declination, 
and  its  true  central  .altitude,  compute  its  horary  distance  from  the  meridian, 
by  any  of  the  methods  given  in  Problem  III.,  pages  384  to  392.  Let 
die  planet's  horary  distance  from  the  meridian,  thus  found,  be  applied 
to  its  reduced  right  ascension,  by  addition  or  subtraction,  according  as  it 
may  be  observed  in  the  western  or  in  the  eastei?!  hemisphere,  and  the  right 
ascension  of  the  meridian  will  be  obtained;  from  which  (increased  by  24 
hours,  if  necessary,)  subtract  the  sun's  reduced  right  ascension,  and  the 
remainder  will  be  the  apparent  time  of  observation* 

Note, — ^When  the  planet's  horary  distance  east  of  the  meridian  exceeds 
its  right  ascensiopy  the  latter  is  to  be  increased  by  24  hours,  in  order  to  find 
the  right  ascension  of  the  meridian. 

Example  1.  ' 

January  34, 1825,  in.latitude  50?30C  N.,  and  longitude  48?45^W.,  the 
mean  of  several  altitudes  of  Jupicer^s  centre,  east  of  the  meridian,  was 
23?41^55?,  that  of  the  corresponding  times,  per  watch,  9Mr,  and  the 
height  of  the  eye  abov|  the  level  of  the  sea  16  feet;  required  the  apparent 
timfc  of  observation  ? 

.  .  Time  of  observation,  per  watch,  =    •    9^  1"  0! 

Longitude  48 ?45^W.,.  in  time  =     +3.15.   0 

Greenwich  time  = 12**16?  0! 

Sun's  right  ascension  at  noon,  January  3d,  =     18?56T  8! 
Correctionof  ditto  for  12  M6?  =     .    ...       +1.39 


Sun*3  reduced  right  ascension*  =5      •    •    .    •     18*57?47* 

/Google 


Digitized  by ' 


398  NAUnCAL  ASTRONOMY, 

Obs.  cent«alt.of  Jupiter=:23?41  ^55r;  hence,  its  true  cent..alU  is  23?35 C56^ 

Jupiter's  right  ascension^  Jan.  Ist,  =s  8t58T  0' 
Correction  of  ditto  for  2f  12*16T  =       --  0.50 


Jupiter's  reduced  right  ascension \s=       8*57*10! 

Jupiter's  declination,  January  Isty  =:    17^561  OTN. 
Correction  of  ditto  for  2f  12M6?  =:       +  5.    1 


Jupiter's  reduced  declination  =  •     •    '18?  IC   KN. 
Ditto  north  polar  distance  =      .     .    71?58'59f 

Jupiter's  true  central  altitude  s  23^35 .56? 

Jupiter's  north  polar  distance  rs  7 1  •  58. 59      Liog.  co-secantssO.  021836 

Lat.  of  the- place  of  observationsSO.  30.   0      Log.  sec^^nt  aa    0. 196489 

SumLd:        146?  4:551:    Constant  log.  =6.301030 

Hilf8um=      ......    73?  2;25^r  Log.  co-sine  =   9.464933 

Remainders    ..•..".    49?26:29|r  Log.  sines   .    9.880667 

Jupiter's  horary  dist,  east  of  the  mer.=  4?58T  0!Logjrisings5. 86495,5 
Jupiter's  reduced  right  ascension  =      .8. 57^  10 


Right  ascension  of  the  meridian  =:      •    3*59T10! 
Sun's  reduced  right  ascension  =      •     .  18. 57-  47 

Apparent  time  of  observation  =      .     .    9*   1?23! 

Example  2. 

January  16th,  1825,  in  latitude  34?45^  S.,  and  longitude  80?30^  E., 
the  mfean  of  several  altitudes  of  Venus*  centre,  west  of  the  meridian^  was 
22?53C25r,  that  of  the  corresponding  times,  per  watch,  7*20^45 !,  and  the 
height  of  the  eye  above  the  level  of  the  sea  18  feet  f  required  the  apparent 
time  of  observation  ? 

Time  of  obseryation,  per  watch,  =  .    7*20r45! 
Longitude  80?30!E.,  in  time  =  .     -^5.22.   0 

Greenwich  time  = I*58r45! 

Sun'sright  ascension  at  noon,  January  {6th,  ==    19^52*41  f 
Correction  of  ditto  for  l*58r45f  =  .    .     .  +0.21 


Sun^  reduced  right  ascension  =:   •    •    •    «    »  I9t53?  2! 

/Google 


Digitized  by ' 


OF  FINDINO  THB  APPAUMT  T1MB«  '  899 

Venus' right  ascension^  January  13th,  =  22^23?  0! 
Correction  of  ditto  for  8f  I  *58?45!  =     .     +   13. 52 


Flanet'ji  reduced  right  ascension  ^    •    «   221367521 


Venus'  declination,  January  13th,  =  .     1K39!  01 S: 
Correction  of  ditto  for  3  f  1  ?58T45 !  =  -  1 .  29, 24 


Planet's  reduced  declination  as      .    •    10?  9CS6r.S.^ 

Obsenwd  central  altitude  of  Venus  sob  22?53.25?;  hence,  her  true  eeih* 
tral  altitude' it  22?47'24f,  on  the  assumption  that  her  horiiontal  parallax, 
at  the  time  o^  observation,  was  1 8  seconds  of  a  degree. 

Latitudes.    .    34?45C  OfS.    .....    Log.  secant=0. 085315 

Planet's  red.  dec. =10.   9.36   S ,   .    Log.  8ecwt=:0^b06864 

Planet's m.a.di8t.=24?35:24f  Nat.  co-sine=3909309 
Planet's  true  alt.=22. 47. 24    Nat.  sine  =    387355 


Remainders     521954    Log.  =  5.717632 

Venus*  horary  dist.^west  of  the  meiid.ar4t36?S5?  L(^.  rislngs5, 809^1 . 1 
Venus'  reduced  rig^t  ascension  =  .     22. 36. 52 

Right  ascension  of  the  meridian  a     SMSr47? 
Sun'a  reduced  right  ascension  a     .    19. 53.   2 

Apparent  time  of  observation  a     •      7*20T45! 

Bemark. — Should  the  horisontal  parallaxes  of  the  planets  be  ever  given 
in  the  Nautical  Almanac,  the  mariner  may  then  deduce  the  apparent  time 
from  their  altitudes,  by  the  abpve  Problem,  to  a  very  great  degree  of 
accuracy,  provided'  the  longitude  of  the  place  of  observation  be  known 
within  a  few  minutes  of  the  truth,  or  that  there  be  a  chronometer  on  board 
to  indicate  the  time  at  Orfetawieh.  However,  even  admitting  that  those 
parallaxee  are  still  to  remain  unnoticed,  the  apparent  time,  computed  as 
abow,  wiH  always  be  sufficiently  near  the  truth  ffH*  the  purpose  of  deter* 
mining  the  longitude  at  sea. 


Digitized  by 


Google 


400  NAUTICAL  ASTRONOMY. 


Probusm  VI.      . 

Given  the  Latitude  dnd  Longitude  of  a  Place,  the  estimated  Time  at  that 
Place,  and  the  Altitude  of  the  Moon's  Lmb  ;  to  find  the  apparent  Jhne 
of  Observation. 

RULB. 

.  Reduce  the  estimated  time  of  observation  to  the  meridian  of  Greenwich, 
by  Problem  III.,  page  297;  to  which  let  the  sun^s  right  ascension  be 
reduced,  by  Problem  V.,  page  298 ;  and  let  the  moon's  tight  ascension, 
declination,  semi-diameter,  and  horizontal  parallax  be  reduced  to  the  same 
time,  by  Problem  VI.,  page  302.  Reduce  the  observed  altitude  of  the 
moon's  limb  to  the  true  central  altitude,  by  Problem  XV.,  page  323 ;  then. 
With  the  latitude  of  the  place  of  observation,  the  moon's  reduced 
declination,  and  her  true .  central  altitude,  compute  her  horary  distance 
from  the  meridian,  by  any  of  the  methods  given  in  Problem  III.,  pages 
384  to  392.  Now,  let  the  moon's  horary  distance  from  the  meridian, 
thus  found,  be  applied  to  her  reduced  right  ascension,  by  addition  or 
subtraction,  according  as  she  may  have  been  observed  in  the  western  or 
eastern  hemisphere,  and  the  right  ascension  of  the  meridian  will  be  ob- 
tained ;  from  which  (increased  by  24  hours,  if  necessary,)  subtract  the  sun's 
reduced  right  ascension,  and  the  remainder  will  be  the  apparent  time  of 
observation. 

Note. — ^When  the  moon's  horary  distance,  east  of  the  meridian,  ei^ceeda 
her  right  ascension,  the  latter  is  to  be  increased  by  24  hours,  in  order  to 
find  the  right  ascension  of  the  meridian. 

And  it  is  to' be  borne  in  mind,,  that  the  moon's  right  ascension  and 
declination  inust  be  corrected  by  the  equation  of  second  difference.  Table 
XVXI.,  as  explained  between  pages  33  and  38.* 

^Example  1. 

January  4th,  1825^  in  latitude  50?I0^  N.,  and  longitude  60? W.,  the 
mean  of  several  observed  altitudes  of  the  moon's  lower  limb,  east  of  the 
meridian,  was  29?25'23^,  that  of  the  corresponding  times,  per  watch, 
7  •28?  181,  and  the  height  of  the  eye  above  the  surface  of  the  water  17 
feet ;  required  the  apparent  time  ? 


*  For  the  eflfecU  resulting  from  the  eqaation  of  the  mean  secood  difference  of  the  moon's 
place  In  right  ascension  and  declination,  see  *'  The  Young  Navigator's  Guide  to  Ac  Side- 
real and  Plsnetary  PMrt^  of  Nautical  Astronomy/'  page  171, 


Digitized  by 


Google 


OF   FINDING  THB  APPARENT  TIME.  401 

Time  of  observation,  per  watch,  =   .     7*28ri8! 
Longitude  60?W.,  in  time  =      .     -f  4.   0.   0 


Greenwich  time  = U»28ri8! 

Sun's  right  ascension  at  noon,  January  4th,  =    I9t  0T32' 
Correction  of  ditto  for  11*28?1 8!  =    .    .    .       +  2.   6 


Sun's  reduced  right  ascension  =      ,    •    .    .     19t  2T38! 

Moon's  right  ascenuon  at  noon,  January  4t'h,=  98?  6'5df 
Corrected  prop,  part  of  ditto  for  1 1  J28ri8!  =   +7. 17. 28 


Moon's  corrected  right  ascension  =:     .    .    .  105?24'.21? 

Moon's  semi-diameter  at  noon,  January  4th,  =:  16  C  9? 
Correction  of  ditto  for  1 1  ^28T18!  =     .     .     .         +5 
Augmentation,  Table  IV.,  =: +  8 


Moon's  true  semi-diameter  =: 16^22? 

Moon's  declination  at  noon,  January  4th,  =:    22?35'39?N. 
Corrected  prop,  part  of  ditto  for  lli28?18!=  -1. 10. 18 

Moon's  corrected  declination  =       .    .    .    .    2 1  ? 25 '  2UN. 

Moon's  horizontal  parallax  at  noon,  January  4th,=  59^17^^ 
Correction  of  ditto  for  11  *28?18!  =     .    .    .    .     +   16 


Moon's  true  horizontal  parallax  =:     .    •    .    •    •   59^33^ 

Observed  altitude  of  the  inoon'sIowerlimb=:29?25'23^j  hence,  her  true 
central  altitude  is  30?  27 '  55  r. 

Latitude  =     .    .    50?10^  OrN.    .    .    ..    •      Log.  secant=0. 193442 
Moon's  corr.  dec.  =  21.25.21  N.    ....      Log.  8ecant=:0. 031091 


Moon's  mer.z,dist.=  28?44 ',  39?  Nat.  vers.  sine=  1 23225 
Moon's  tfue  alt.  =  30. 27. 55  Nat.  co-V.  S.  =  492988 


Remainder  =  369763  Log.s  5. 567923 

MoonV  horary  dist,  east  of  the  merid.:::4  ^30?41  !Log«risingr:5. 79245. 6 

2  D 


Digitized  by 


Google 


402  NAUTICAL  ASTRONOMY. 

Moon's  horary  dUt.  cast  of  the  merid.=:4t 30*41 ! 
Moon's  cor.  R.  A.  105 ?24 ^  2 H,  in  tim€=7.    1 .  87 


Right  ascension  of  the  meridian  =    .     2?80'?56! 
Sun's  reduced  right  ascension  =  .     .19.    2. 38 

Apparent  time  of  observation  =  •     «    7^28?18! 

Example  2.     ^ 

January  SOth,  1825,  in  latitude  10?20^  S.,  and  longitude J00?50^E., 
the  mean  of  several  altitudes  of  the  moon's  lower  limb^  west  of  the  meri- 
dian, was  7  ^  23 '  30'%  that  of  the  corresponding  times,  per  watch,  1 3  ^  33 T20 ' , 
and  the  height  of  the  eye  above  the  surface  of  the  water  20  feet;  required 
the  apparent  time  ? 

Time  of  observation,  per  watch;  =  13*33?i20! 
Longitude  100^50!  E«,  in  time  =1-6. 43. 20 

Greenwich  time  = 6^50^0! 

Sun's  right  ascension  at  noon,  Jan.  30th,  =     20t51T25! 
Correction  of  ditto  for  6^50?      «    .    «    •    .       ^1.10 


Sun's  reduced  right  ascension  =      •    .    «     •     20?52?35! 

Moon's  right  ascension  at  noon,  Jan.  30th,  =     76?21'55T 
Corrected  prop,  part  of  ditto  for  6t50r  =       +  4.13.38 


Moon's  corrected  right  ascension  i=     .     ...    80^35  !33? 

Moon's  semi-diameter  at  noon^  JanuatydOth/  ts     15(46? 
Correction  of  ditto  for  6*50?  zr       .....         +5 
Augmentation^  Table  IV.,  zz +2 


Moon's  true  semi-diameter  = 15(53T 

Moon's  declination  at  noon,  January  30th,  =t     23?57-46rM. 
Corrected  prop,  part  of  ditto  for  6? 60?  =         —  8.44 

Moon's  corrected  declination  tz      .         .     *    23*54!  2TN. 

Moon's  horizontal  parallax  at  noon,  Jan.  30th,  =     57'51? 
Correctioiiof  dittofor6?S0r  £2:     .....       +17 


Moon's  true  horistontal  jporallax  =:      »    <    •    *      58r  8f 

/Google 


Digitized  by ' 


OP  FINDING  THB  ALTITCTDSS  OF  THB  BBAVENLT  BODIES.         408 

Observed  altitude  of  the  moon's  lower  limb  =  7?23'30T;  hence^  her  true 
central  altitude  is  8?25^54r. 

Latitude  s=     .    .     10?20C  OrS.     ....      Log.  secantnO.  007102 
Moon's  corr.  deer:  23.54.    2.N Log.  secantzzO.  038935 

Moon's  m.  z.  dist.  ==  34  ?  14  ^  2r  Nat.  co^ine=  826748 
Moon's  true  alt.  =     8.25.54    Nat.  sine  ='    146630 


Remainder  =  680118  Log.=5. 832584 

Moon'shorarydistwestof  themerid.=  5^  3r33!   Log.  rising^: 5.  878621 
Moon's  cor.  R.  A.  80?35 ^3Sr,  in  time=5.  22. 22 


Right  ascension  of  the  meridian  =:       10^25755 ' 
Sun's  reduced  right  ascension  =     •     20. 52. 35 

Apparent  time  of  observation  =      •     13?33?20! 

RemarJe.'^lf  there  be  a  chronometer  on  board  to  indicate  die  time  at 
Greenwich^  the  apparent  time  of  observation,  at  any  given  place,  may  be 
very  correctly  ascertained  by  the  above  problem.  But,  since  the  chrono- 
meter shows  the  equable  or  mean  time  at  Greenirich,  this  time  must  be 
reduced  to  apparent  time,  by  applying  the  equation  of  time  thereto  with  a 
contrary  sign  to  that  expressed  in  the  Nautical  Almanac.  Thus,  in  the 
above  example,  if  the  chronometer  give  the  mean  time  at  Greenwich  zz 
7?3?44!,  then  the  reduced  equation  of  time,  vis.,  13T44!,  being  subtracted 
therefrom,  shows  the  apparent  time  at  that  meridian  to  be  6t50?0t. 
Hence,  when  the  equation  of  lime  in  the  Nautical  Almanac  is  marked 
additive,  it  is  to  be  applied  by  subtraction;  but  when  marked  subtracHve, 
it  is  to  be  applied  by  addition  to  the  mean  time  (per  chronometer)  at 
Greenwich,  in  order  to  reduce  it  to  apparent  time. 


SOLUTION  OP  PROBLEMS  RELATIVE  TO  FINDING  THE 
ALTITUDES  OP  THE  HEAVENLY  BODIES. 

It  sometimes  happens  at  sea,  particularly  in  taking  a  lunar  ohseroaiion, 
that  the  horizon  is  so  ill-defined  as  to  ftnder  it  impossible  to  observe  the 
altitudes  of  the  objects  to  a  sufficient  degree  of  exactness ;  or,  perhaps, 
that  one  or  bodi  of  die  objects  are  directly  over  the  land,  at  the  time  of 
measuring  ^  Imutf  distance^  and  the  ship  so  contiguous  thereto  as  to 

2i>2 


Digitized  by 


Google 


404  NAUTICAL  A8TRONOMV. 

render  the  absolute  value  of  the  horizontal  dip  uncertain :  in  such  cases, 
therefore,  the  altitudes  of  the  objects  must  be  obtained  by  computation,  as 
in  the  following  problems ;  the  principles  of  which  will  be  found  amply 
illustrated  in  "  The  Young  Navigator's  Guide  to  the  Sidereal  and  Planetary 
Parts  of  Nautical  Astronomy,"  page  237. 


Paoblbm  I. 

Gwen  the  Latitude  and  Longitude  of  a  Place,  and  the  AppareiU  Thne  at 
that  Place;  to  find  the  true  and  the  apparent  Altitude  of  the  Sun's 
Centre. 

Rule. 

Reduce  the  given  apparent  time  to  the  meridian  of  Greenwich,  by 
Problem  III.,  page  297 ;  to  which  let  the  sun's  declination  be  reduced,  by 
Problem  V.,  page  298. 

If  the  latitude  of  the  place  and  ^he  sun's  declination  are  of  different 
names,  let  their  sum  be  taken ;  otherwise,  their  differefice:  and  the  meri- 
dional zenith  distance  of  that  object  will  be  obtained.    Then, 

To  the  logarithmic  rising  answering  to  the  sun's  distance  from  the 
meridian,  (that  is,  the  interval  between  the  given  apparent  time  and  noon,) 
add  the  logarithmic  co-sines  of  the  latitude  of  the  place  and  of  the  sun's 
reduced  declination :  the  sum,  rejecting  20  from  the  index,  will  be  the 
logarithm  of  a  natural  number ;  which,  being  added  to  the  natural  versed 
sine  of  the  sun's  meridian  zenith  distance,  found  as  above,  will  g^ve  the 
natural  co-versed  sine  of  its  true  altitude. 

To  the  sun's  true  altitude,  thus  found,  let  the  correction  corresponding 
thereto  in'  Table  XIX.,  be  added.;  and  the  sum  will  be  the  apparent  altitude 
of  the  sun's  centre. 

Example  L 

Bequired  the  true  and  apparent  altitude  of  the  sun's  centre,  January  10th, 
1825,  at3Mr45!  apparent  time,  in  latitude  40?30^  N.,  and  longitude 
S9?2C30rW.? 

Apparent  time  at  ship  or  place  =    •    •     •    3*   1T45! 
Longitude  59*?  2  ^  30''  W.,  in  time  =    •     +  3, 56. 10 


Greenwich  time  22       i 6 1 57*55! 

/Google 


Digitized  by ' 


OF  FINDING  THS^  ALTITCDBS  OF  THB  HEAVENLY  BODIES.        405 

Sun's  decimation  at  noon,  January  lOth^s  21^57 -50^  S. 
Correction  of  ditto  for  6?57T55!  =   \  -  2. 40 


Sun's  reduced  declination  =      ...       21?55M0rS. 

Sun's hor. men di8t.=:3^  lr45!  ....  Log.  risings  .  5.474670 
Sun's  reduced  dec.ss21. 55. 10  S.  .  .  .  .  Log.  co-sine  =  9. 967412 
Lat.  of  the  place  =  40. 30.   0  N. .     .     .    .    Log.  co-sine  =     9. 881046 


Sun's  mer.  z.  dist.  =  62?25 '.  lO^Nat.  vers.  8ine=  537005 


Nat.  number  =  210440  Log.s  5.323128 


Truealt.ofsun'scen.=  14?37  '48TNat.co.V.S.=  747445 
Corrcc.,TableXIX.=    +  3.26 


App.alt.ofsun'scen.=14?4i:  9'r 

Example  2. 

Required  the  true  and  apparent  altitude  of  the  sun's  centre,  January 
20th,  1825,  at  19*  16ri8!  apparent  time,  in  latitude  37^20^  S.,  and  Ion- 
gitude49?45:R? 

Apparent  time  at  ship  or  place  ==      .     .     19M6T18* 
Longitude 49? 45 ^  £., in  timers    .    .      —3. 19.   0 

Greenwich  time  = Ij5*57"18! 

Sun's  declination,  January  20th,  =     •     20?  7 '  1 1  r  S. 
Correction  of  ditto  for  15*57'ri8!  =         -  8.48 


Sun's  reduced  declination  =     .    .    .     19?58f23rS. 

Sun's  hor.  dist.  fr.mer.  =  4*43r42!*  .  .  .  Log.  rising  =  5.828140 
Sun's  reduced  dec.  =  19.58.23  S.  .  .  .  Log.  co-sine=  9. 973060 
Latitude  of  the  place  =  37. 20.   OS....     Log.  co-sine=  9. 900433 

Sun's  mer.  zen,  dist.  =   1 7  ?  2 1 C  37  ''Nat.  v.  sine=045552  

Nat,  num.  =  503075  Log.=5. 701633 

True  alt.  of  sun's  cent.=  26?49'.55^Nat.co.V.S=548627 
Reduc.  of  do.,Tab.  XIX.,=  + 1 .  44 

App.alt.of8un'scentre=:  26?51  ^89r 

•  %i  hoixrs  -  19*16*18*  ^  4*43*42' ,  the  sun's  Uorwy  distance  from  the  meridian, 

Digitized  by  VjOOQ IC 


406  NAUTICAL  ASTROHOMT. 


Probijem  IL 


Given  the  apparent  Time  at  a  known  Place,  tojind  the  true  and  apparent 
Jmtude  of  a,  fi^ed  Star. 

Rule. 

Reduce  the  given  apparerit  time  to  the  meridian  of  Greenwich^  by 
Problem  III.^  page  297 1  to  which  let  the  sun's  right  ascension,  at  noon 
of  the  given  day,  be  reduced,  by  Problem  V.,  page-298.     • 

Let  the  star's  right  ascension  and  declination  (Table  XLIV.)  be  reduced 
to  the  given  period,  hy  the  method  shown  in  page  1 15.  To  the  sun's 
reduced  right  ascension  let  the  given  apparent  time  be  added,  and  the  sum 
will  be  the  right  ascension  of  the  meridian ;  the  difference  between  which 
and  the  starts  reduced  right  asQcnsion  will  b^  the  horary  distance  of  the 
latter  from  the  meridian.  Now,  with  the  star's  horary  distance  from  the 
meridian,  thus  found,  its  reduced  declination,  and  the  latitude  pf  the  place, 
compute  the  true  altitude  of  that  object,  by  the  last  problem.  Then,  to  the 
star's  true  aldtude,  thus  found,  let  the  correction  corresponding  thereto,  in 
Table  XIX.^  be  added ;  and  the  sum  will  be  the  star's  apparent  altitude. 

Example  1. 

Required  the  true  and  apparent  altitude  of  a  Arietis,  January  Ist,  1825, 
at  llt9T29!  apparent  time,  in  latitude  40? 29^  N.,  and  longitude 
59?45^W..? 

Apparent  time  at  ship  or  place  =  .    «    ,     11?  9T29t 
Longitude59?45^  W.,  in  time=     .      +  3.59.   0 

Greenwich  time  = 15t  8?29! 


Sun's  right  ascension  at  noon,  January  Ist,  =    18M7*19* 
Correction  of  ditto  for  15  ?8T29!  =:      ...       +  2.47  • 


Sun's  reduced  right  ascension  s      •    ,    ,    •     18?50?  6! 
Given  apparent  time  = 1 1 .   9. 29 

Right  ascension  of  the  meridian  s  .     •     •    «      5?59?35! 
Star's  reduced  declination  -    «    «    •    .    22?37'50?N. 

Digitized  by  VjOOQ IC 


OF  FINDING  THE  ALTITUM8  OF  THS  BEAVBNLY  BODIES.         407 

Star's  reduced  R.  A.=  l  ?57r  19! 
R.  A.  of  the  merid.  =  3*  59. 35 


Star'shor.di8fr.mer.=?4t  2ri6!  ....  Log.  rising  =  5.706360 
Star's  redii4:^d«c.»22';37*50rN.  ,  •  .  Log.  co-sine  3: 9. 965204 
Lat.  of  tbe  place  9  40. 29,   ON...,      Log.  co-sine  :«  9. 8S1 153 


Star's  mer.zen.dist.=  17?5 1 '.  10rNat.vers.sine=  048153 


Nat.  number  =  357040  Log.=  5. 552717 


True  alt.  of  the  8tar=36?29:56rNat.  co-V.  S.=  40S19S 
Reduc.of  do.TabJClX=  +  1.17 


App.'a]t.of  giv.  8tar«36?31  HSr 

Example  2. 
Required  th«  true  and  apparent  altitude  of  Procyon,  January  Ist,  1825, 
at   9^SirS9t    apparent  tiine>  in  latitude  39?20:30^S.,  and  longitude 
75?40CB.J 

Apparent  time  at  ship  or  place  =s  .     .     •    •      9*31TS9! 
Longitude73?40CB.,  intime  38      ....      5^   2.40 


Gr^nwich  time  » 4t28T59: 

Sun's  right  ascension  at  noon^  January  Ist^  =  18M7*19! 
Correction  of  ditto  for  4*28r59!  =     .     ,     .       +  0, 49 


Sun's  reduced^right  ascension  =      .     .     ,     •     18M8?  8! 
Given  apparent  time  =     •    •    •    •     .    .    .      9.31.39 


,        Right  aseensioh  of  the  meridian  a  •    •     •    •      4M9T47' 

Procyon's  reduced  declination  s      ....      5?39'587N. 

Procyon's  red.  R.  A  =  7*30?  8 ! 
R.  A.  of  the  meridian==4. 19. 47 


Star'8hor.dis.fr.mer.=3M0?21!  ....  Log.  rising  =  5.512600 
Star's  reduced  dec.  =  5?39^58'rN.  .  ,  .  Log.  co-sine  =  9. 997872 
Latitude  oftheplace=39, 20. 30  S«      .    .    .      Log.  co-sine  =  9. 888393 


Star's  mer.  z.  dist.  =  45?  0^28rNat.  vers.sine=292990 


Nat.  number  =250533  Log.=  5.398865 


True  alt.  of  giv.  star =27?  9^36'rNat.  co-V.  S.=543523 
Reducof  do.Tab  JCIX.=  + 1 .  50 


App.  alt.  of  giv.  star=27?  1 K  26r 

Digitized  by  VjOOQ IC 


408  NAUTICAL  ASTRONOMY. 


Problem  III. 

Given  the  Latitude  atid  Longitude  of  a  Places  and  the  apparent  Time  at 
that  Place;  to  find  the  true  and  apparent  Altitude  qfa  Planets 

Rule- 

Reduce  the  given  apparent  time  to  the  meridian  of  Greenwich,  by  Pro- 
blem III.,  page  297 ;  to  which  time  let  the  sun's  right  ascension  be 
reduced,  by  Problem  V.,  page  298 ;  and  let  the  planet's  right  ascension 
and  declination  be  reduced  to  the  same  time,  by  Problem  VIL,  page  307* 

To  the  sun's  reduced  right  ascension  let  the  given  apparent  time  be 
added,  and  the  sum  will  be  the  right  ascension  of  the  meridian ;  the  differ- 
ence between  which  and  the  planet's  reduced  right  ascension  will  be  the 
horary  distance  of  the  latter  from  the  meridian.  Now,  with  the  planet's 
horary  distance  from  the  meridian,  thus  found,  its  reduced  declination,  and 
the  latitude  of  the  place,  compute  the  true  altitude  of  that  object,  by  Pro- 
blem I.,  page  404.  Then,  with  the  planet's  true  altitude,  thus  found,  by 
computation,  enter  Table  XIX.,  and  take  out  the  quantity  corresponding^ 
to  the  redaction  of  a  star's  true  altitude ;  the  difference  between  which 
and  the  planet's  parallax  in  altitude.  Table  VI.,  will  leave  a  corrccdon^ 
which,  being  added  to  the  trucj  will  give  the  oppare?!^  altitude  of  the 
planet. 

Example  1. 

Required  the  true  and  apparent  altitude  of  the  planet  Jupiter,  January 
3d,  1825,  at9MT23!  apparent  time,  in  latitude  50?30'  N.,  and  longitude 
48^45  :W.? 

Given  apparent  time  at  ship  or  place  =     •     •      9*   l?23! 
Longitude  48?45^  W.,  in  time  =    .    .     .      +  3. 15.  0 


Greenwich  time  =   . 12*l6r23! 

Sun's  right  ascension  at  noon,  January  3d,  =   18^56?  8! 
Correction  of  ditto  for  12*  16T23 !  =     ,    .    ,       +   1 .  39 


Sun's  reduced  right  ascension  =       ....     18*57"47* 
Given  apparent  time  =     •     .     •    .    •    ...      9.    1 .  23 


Right  ascension  of  the  meridian  ss  ,    •    •    •      3*59T10! 

/Google 


Digitized  by ' 


OF   PlxNDlNG  THB  ALTITUDES  OF  THE   HEAVENLY  BODIES.        40& 

Jupiter*8  declination  at  noon,  January  1st,  3=     17? 56'  0?N. 
Correction  of  ditto  for  2fl2M6r23'.  =    .    .         +  5,    1 


Jupiter's. reduced  declination  s       •    .    .    •     18?   IC   KN.. 

Jup/8lLA.atnoon,Jan,l=8*58r  0*. 
Cor.ofdo.for2fl2;i6T23!=  -0.50 


Jupiter's  reduced  R.  A.  =  8*57TlOt 
R.  A.  of  the  meridian  =    3. 59. 10 


Planet's hor.  dist.  fr. mer.  =  4*58?  0'.  .  .  .  Log.  rising  =  5. 864960 
Planet's  reduced  dec.  =  Ip.  1.  1  N.  .  .  Log.  co-8ine=  9.978164 
Latitude  of  the  place  ^     50. 30.   ON...    Log.  co-sine=  9. 80351 1 


Planet's  mer.  zen.  dist.  =  3^?28^59^Nat.V.S.=  156449 


Nat.  num.=443236Log.=5. 646635 


Jupiter's  true  central  alt.=:  23?35 :  52^N.co-V.S.=599683 

Red.Tab.XIX.2nK)   jj.^_         g,      g^ 

Par.Tab.vi.=  0.    2    J 


Jupiter's  app.  central  alt.=  23  ?38 !    1  r 

Note. — Jupiter's  horizontal  parallax  is  assumed,  in  the  preseiU  instance^ 
at  2  seconds  of  a  degree. 

Example  2. 

Required  the  true  and  apparent  altitude  of  the  planet  Venus,  January 
16th,  1825,  at  7^20^45!  apparent  time,  in  latitude  34?45^  S.,  and  longi- 
tude 80?30C  £.,  admitting  her  horizontal  parallax,  at  that  time,  to  be  18 
seconds  ? 

Apparent  time  at  ship  or  place  =      ,     ,    .    .      7  *20"45  * 
Longitude  80?30' E.,  in  time  =      .    .    .    •      5.22.   0 


Greenwich  time  = 1?58?45: 

"  Sun's  right  ascension  at  noon,  January  16th,  =   19*52T41 ! 
Correction  of  ditto  for  1  *58T45!  =:     .    .    .       -  0. 21 


Sun's  reduced  right  ascension  =       •    .    •    •     19t53?  2! 
Given  apparent  time  = *  /.  20. 45 


Right  ascension  of  the  meridian  =:      ,    «    ^      3M3T47* 

/Google 


Digitized  by ' 


410  NAUTICAL  ASTEOVOMT. 

VcQua'  declination^  January  13th,  »    .    .    .    1K39'.  OrS. 
Correction  of  ditto  for  3f  1  ?S8r45 '.      .    .       ^  U  29. 24 


Veni»' reduced  declination  55 10?  9' 36*  S, 

Venus'  R-  A.,  Jan.  13th,=22*23T  0! 
Cor.  of  do.for3f  1  ?58r45 !  =  + 13. 52 


Venus'  reduced  R.  A.  =    22*36T52! 
R.  A.  of  the  meridian  =       3. 13. 47 


Planet's  hor.  dist.  fr.  iner.  ==  4  ?36T55 '  ...  Log.  rising  as  5 .  8098 10 
Planet's  reduced  dec.  =^  10?  9^36^  S.  .  .  Log.  co-8ine=  9. 993136 
Latitude  of  the  place  =     34.45.   OS.,*    Log.  CQ-sine=  9. 914685 

Planet's  mer.  zen.  dist.  =  24?35^24Wat.V.  S.=  090691  -r 


Nat.num.=  521953Log.=5.  717631 


Venus' true  central  alt.  =  22?47'24^'N,co.V.S=612644 
Red.Tab.xix.2n5r)jj.jf^^^j  59 
Par.Tab.vi.=0. 16    ) 


Venus' app.  central  alt.  =  22?49'23r 

Remark. — In  these  problems,  a  cipher  is  annexed  to  the  logarithmic 
rising  taken  from  Table  XXXII. :  this  is  done  with  the  view  of  reducing  it 
to  six  places  of  decimals ;  so  that  there  may  be  no  mistake  in  property 
applying  thereto  the  logarithmic  co-sines  of  the  latitude  and  of  the 
declination. 


PnofiLSM  rv. 

Given  the  Latitude  of  a  Place,  and  the  apparent  Time  at  that  Place, 
with  the  Longitude;  to  find  the  true  and  apparent  Altitude  qf  the 
Moofii^e  Centre. 

Rule. 

Reduce  the  given  apparent  time  to  the  meridian  of  Oraenwieh,  by  Pro* 
blem  III.,  page  297 ;  to  which  let  the  sun's  right  ascension  be  reduced,  by 
Problem  V.,  page  298;  and  let  the  moon's  right  ascension,  declination,  and 
horizontal  parallax  be  reduced  to  the  same  time,  by  Problem  VI.,  page  302. 

To  the  sun's  reduced  right  ascension  let  the  given  apparent  time  be 
added,  and  the  sum  will  be  the  right  ascension  of  the  meridian ;  the  differ- 


Digitized  by 


Google 


OF  FINDING  THB  AtTITUDBS  Q9  THV  HBAVBNLY  BODIBS.        4U 

ence  between  which  and  the  moon's  reduced  right  ascension^  will  be  the 
horary  distance  of  the  latter  from  the  meridian* 

Now,  with  the  moon's  horary  distance  from  the  meridian,  her  corrected 
declination,  and  the  latitude  of  the  place,  compute  her  true  central  altitude, 
by  Problem  I.,  page  404.    Then, 

From  the  moon's  true  central  altitude,  thus  found,  subtract  the  correction 
corresponding  thereto  and  her  reduced  horizontal  parallax,  in  Table  XIX., 
and  the  remainder  will  be  the  apparent  central  altitude. 

Note. — ^The  moon's  right  ascension  and  declination  must  be  corrected  by 
the  equation  of  second  diflPerence  contained  in  Table  XVII.,  as  explained 
between  pages  33  and  37* 

Example  1. 

Required  the  true  and  apparent  altitude  of  the  moon's  centre,  January 
4th,  1825,  at  7 ^28r  18 '.apparent  time,  in  latitude  50?  lO'.N.,  and  longitude 
60?W.? 

Apparent  time  at  ship  or  place  =    .  •   .     .     .       7 -28"  18! 
Longitude  60?  W.,  in  time  = 4.   0.   0 

Greenwich  time  = Il*28ri8!. 

Sun's  right  ascension  at  noon,  January  4th,  t=    19 1  0?32! 
Correction  of  ditto  for  11!28?1 8!  =    .    •    .       -f  2,   6 


Sun's  reduced  right  ascension  =      •     .    •     •     19*  2?38' 
Given  apparent  time  = 7*  28. 18 


Right  ascension  of  the  meridian  =s  .    .    .     .      2^30T56! 

Moon's  horizontal  parallax  at  noon,  January  4th,  =  59M7^ 
Correction  of  ditto  for  ll?28r  18:  =....+   16 


Moon's  true  horizontal  parallax  = 59!33T 

Moon's  declination  at  noon,  January  4th,  =?     22?35:391^N. 
Cortrectcd  prop,  part  of  ditto  for  Ili28ri8:=  -  1. 10/  8 

I*  111     M^— — 

Moon's  corrected  declination  =       •     •     •    •    2 1  ?  25 '.  3 1  ^N. 

Moon's  right  ascension  at  noon,  January  4th,  =  98?  6^.53^ 
Corrected  prop,  part  of  ditto  for  1 1  ?28r  18!  =  +  7. 17.  28 

Moon'8CorrectedR,A.=7*  1"37!  =   .    .    •    .    .  105?24:2H    . 

Digitized  by  VjOOQ IC 


412  NAUTICAL  ASTRONOMY. 

Moon's  corrected  R.  A.= 7  *  1  "37 ' 
R.A.of  the  meridian  =  2.30.56 


J  '8  horary dis.  fr.  mer,  =  4  ?30T4 1 !  ...  Log.  rising  =  5. 792450 
Moon's  corrected  dec.=21?25'3ir  N;  .  .  Log.  co-sine  =  9. 968909 
Latitddeof  theplace  =  50. 10.   ON...      Log.  co-sine  ==  9. 806558 


Moon'8mer.zen.di8t.=28?44^39r  NatV.S.  =  123225 


Nat.  num.  =  36975  7Log.=:5. 567917 


True  alt.  of  ])  '8centre=30?27'.55^NatiCo-V.S.=492982 
Reduc,ofdo.Tab.XIX.=  -50.   8 


App.  alt.  of  ])  's  cent.=  29?37U7^ 

Example  2. 

Required  the  true  and  apparent  altitude  of  the  moon'a  centre,  January 
30th,  1825,  at  13t33r20!  apparent  time,  in  latitude  10?20^  S.,  and  lon- 
gitude 100?50:  E.? 

Apparent  time  at  ship  or  place  r=     •    .    •    •     13^33^20! 
Longitude  100?50'  E.,  in  time  =    .    .    .       -  6. 43. 20 

Greenwich  time  =    .     . 6?50r  0! 

Sun's  right  ascension  at  noon,  January  30th,  =  20*5lT25 ! 
Correction  of  ditto  for  6?50r  =z      ....       +   1. 10 


Sun's  reduced  right  ascension  =1      ....    20t52?35! 
Given  apparent  time  = 13. 33. 20 

Right  ascension  of  the  meridian  =  •    •    •     .     10t25?55! 

Moon's  horizontal  parallax  at  noon,  January  30th,=:57^51'! 
Correction  of  ditto  for  6*  50T  =: -f   17 


Moon's  true  horizontal  parallax  =: 58^  8T 

Moon's'declination  at  noon,  January  30th,  =:     23  ? 57 '  46 ^N. 
Corrected  prop,  part  of  ditto  for  6  ?50?  =  .     .       —3.44 


Moon's  corrected  declination  =  •    .    .    .    .    23?54'  2^N. 

Moon's  right  ascension  at  noon,  January  30th,=76?21  '55^ 
Corrected  prop,  part  of  ditto  for  6?50'r  =     +  4. 13. 38 

Moon's  corrected  R.A.=5*22:22!  :;:       ,    .    .    •    80?35^33f 

Digitized  by  VjOOQ IC 


OF   FINDING  THB  LONGITUDE.  413 


Moon's  corrected  R.  A.=:5*22r22! 
R.  An  of  the  meridian  =  10. 25. 55 


})  's  hor.  dist.  fr.  mend.  =  5 1  3T33 !  .  .  .  Log.  rising  =:  5 .  878620 
Moon'*  corrected  dec.  =23^54'  21  N.  .  .  Log.  co-sine  =  9. 961065 
Latitude  of  the  places  10.20.   OS.,.    Log.  co-sine  =  9. 992898 


D  sm«r.2en.di8tence=:34?14^  2r  Nat.  V.  S.  =  173252 


Nat.  num.  =  6801 16Log.=5. 832583 


True  alt.  of  })  's  centrer:  8?25 '. 54^Nat.  co.V.S.= 853368 
Reduc.ofdo.Tab.XIX.=  -50.47 


App. alt.  of  J)  's  centre  =:  7?35 '.  T". 

Remarh^^The  natural  sines  may  be  used  in  the  solution  of  the  four 
preceding  problems,  instead  of  the  versed  sines :  in  this  case,  if  the  natural 
number  be  subtracted  from  the  natural  co-sine  of  the  object's  meridional 
zenith  distance,  the  natural  sine  of  its  true  altitude  will  be  obtained.  Thus, 
in  the  last  example,  the  moon's  meridian  zenith  distance  is  34?  14 '2?.  Now, 
the  natural  co-sine  of  this  is  826748 ;  from  which  let  the  natural  number 
680116  be  subtracted,  and  the  remainder  =:  146632  is  the  natural  sine 
of  that  object's  tru6  altitude ;  the  arch  corresponding  to  which  is  8?25  '54T. 

These  problems  are,  evidently,  the  converse  of  those  for  finding  the 
apparent  time,  as  given  in  pages  383, 394, 397,  and  400. 


SOLUTION  OF  PROBLEMS  RELATIVE  TO  THE  LONGITUDE. 

The  Longitude  of  a  given  place  on  the  earth,  is  that  arc  or  portion  of 
the  equator  which  is  intercepted  between  the  first  or  principal  meridian 
and  the  meridian  of  the  given  place ;  and  is  denominated  east,  or  west, 
according  as  it  may  be  situate  with  respect  to  the  first  meridian. 

The  ^gt  or  principal  meridian  is  an  imaginary  great  circle  passing 
through  any  remarkable  place  and  the  poles  of  the  world :  hence  it  is 
entirely  jarbitrary ;  and,  therefore,  the  British  reckon  their  first  meridian  to 
be  that  which  passes  througli  the  Royal  Observatory  at  Greenwich ;  the 
French  esteem  their  first  meridian  to  be  that  which  passes  through  the 
Royal  Observatory  at  Paris  *,  the  Spaniards,  that  which  passes  through 
Cadiz,  &c^  &c.  &c.  Every  part  of  the  terrestrial  sphere  may  be  conceived 
to  have  a  meridian  line  passing  through  it,' cutting  the  equator  at  right 
angles :  hence  there  may  be  as  many  dififerent  meridians  as  there  are  points 
in  the  equator. 


Digitized  by 


Google 


414  NAUTICAL  ASTBONOMT. 

Every  meridian  line,  with  respect  to  the  place  through  which  it  pASses, 
may  be  said  to  divide  the  surface  of  the  earth  into  two  equal  parts^  called 
the  eastern  and  western  hemispheres.  Thus,  when  the  face  of  an  observer 
is  turned. towards  the  north  pole  of  the  world,  the  hemisphere  which  lies 
on  his  right  hand  is  called  east,  and  that  on  his  left  hand  west;  and,  vice 
versay  when  the  face  is  directed  towards  tlie  south  pole  of  the  world,  the 
hemisphere  which  lies  on  the  left  hand  is  called  east^  and  that  on  the  right 
hand  west. 

The  longitude  is  reckoned  both  ways  from  the  first  meridian,  east  and 
west,  till  it  meets  with  the  same  meridian  on  the  opposite  part  of  the 
equator :  hence  the  Ibngitude  of  any  place  pa  the  earth  can  never  exceed 
180  degrees.  The  difference  of  longitude  between  two  places  on  the  earth 
is  an  arc  of  the  equator  contained  between  the  meridians  of  those  places, 
showing  how  far  one  of  them  is  to  the  eastward  or  westward  of  the  other, 
and  can  never  exceed  180  degrees,  or  half  the  earth's  circumference. 

All  places  that  are  situated  under  the  same  meridian  have  the  same 
longitude ;  but  places  which  lie  under  different  meridians  have  different 
longitudes :  hence^  in  sailing  due  north  or  due  south,  since  a  ship  does  not 
change  her  meridian,  she  keeps  in  the  same  parallel  of  longitude ;  but,  in 
sailing  due  east  or  due  west,  she  constantly  changes  her  meridian^  and  there- 
fore passes  through  a  variety  of  longitudes. 

When  the  meridian  of  any  place  is  brought,  by  the  diurnal  revolution  of 
the  earth  round  its  axis,  to  point  directly  to  the  sun,  it  is  then  noon  or  mid- 
day at  that  place. 

The  motion  of  the  earth  on  its  axis  is,  at  all  times,  equable  and  uniform; 
and,  since  it  turns  round  its  axis  eastward  once  in  every  24  hours,  all  parts 
of  the  equator,  or  great  circle  of  360  degrees,  will  pass  by  the  sun,  or 
star,  in  equal  portions  of  time :  therefore  the  twetity-fourth  part  of  the 
equator,  viz.,  15. degrees,  will  pass  by  the  sun  in  one  hour' of  time  :  for, 
24 1  X  15?  or  1  hour,  :=  360  d^ees;  and^  conversely^  360  d^ees  -i- 
24  hours  =:  15  degrees  or  1  hour. 

fivery  place  on  the  earth,  whose  meridian  is  15  degrees  east  of  the  Royal 
Observatory  at  Greenwich,  will  have  noon  and  every  other  hour  <me  hour 
sooner  than  at  the  meridian  of  that  observatoryj  if  the  meridian  be  30 
degtces  east  of  Greenwich,  it  Mrill  have  noon  and  every  other  hour  two 
hours  sooner  than  at  the  meridian  of  that  place,  and  so  on ;  the  time  always 
diflering  at  the  rate  of  1  hour  for  every  15  degrees  of  longitude,  I  minute 
of  time  for  every  15  minutes  of  longitude,  and  1  second  of  time  for  every 
15  seconds  of  longitude. .  Again,  every  place  whose  meridian  is  15  degrees 
west  of  the  Royal  Observatory  at  Greenwich  will  have  noon  and  every  other 
hoar  one  hour  later  than  at  the  meridian  of  that  observatory;  if  the  meri« 
dian  be  30  degrees  to  the  westward  of  Greenwich,  it  will  have  noon  and 
every  other  hour  tu)o  hours  later  than  at  the  meridian  of  th«t  place,  and 


Digitized  by 


Google 


OF  PIKDtNG  THB  MVGtTUDB*  415 

SO  on.  Hence  it  is  evident,  that  if  the  time  at  the  meridian  of  a  ship  or 
place  be  greater  than  the  time,  at  the  same  instant,  at  the  meridian  of 
Greenwich,  such  ship  or  place  will  be  to  the  eastward  of  Greenwich;  but  if 
the  time  at  a  ship  or  place  be  less  than  the  time,  at  the  same  instant,  at 
Greenwich,  such  ship  or  place  will  be  to  the  westward  of  Greenwich. 

Since  tlie  longitude  of  any  place  on  the  earth  it  expressed  by  the  differ- 
ence of  time  between  that  place  and  the  Royal  Observatory  at  Greenwich ; 
therefore^  to  determine  the  longitude  of  a  given  place,  we  have  only  to  find 
the  time  of  the  day  at  that  place,  and  also  at  Greenwich,  at  the  same 
mstant;  then,  the  diflerenee  of  these  times  being  converted  !nto  motion, 
by  allowing  15  degrees  for  every  hour,  &c.,  or,  more  readily,  by  Table  I.  in 
this  work,  the  longitude  of  such  given  place  will  be  obtained. 

The  readiest,  and,  indeed,  the  most  simple  method  of  findii^  the  longi- 
tude at  sea,  in  theory ,  is  by  a  chronometer,  or  other  machine,  that  will 
measure  time  so  exaetly  true  as  to  go  uniformly  correct  in  all  .places,  sea- 
sons, and  dimates :  for,  such  a  machine  being  once  regulated  to  the 
meridian  of  the  Royal  Observatory  at  Greenwich,  would  always  show  the 
true  time  under  that  meridian,  though  temoved  in  a  ship  to  the  most 
distant  parts  of  the  globe,-^even  to  the  utmost  extent  of  longitude. 

Although  such  a  perfect  piece  of  mechanism  can  scarcely  be  hoped  for 
or  expected  to  result  from  the  ablest  and  best  applied  course  of  human 
industry, — yet,  on  the  supposition  that  the  chronometers  used  at  sea  are 
sufficiently  correct  for  the  measurement  of  time  in  short  VQjfoges,  we  will 
now  proceed  to  show  how  the  longitude  is  to  be  found  by  means  of  those 
instruments. 


PaoBLBM  L 

To  convert  «tppare»it  Time  in^  mean  Time. 

Rule. 

Reduce  the  equation  of  time^  as  given  in  page  II.  of  the  month  in  the 
Nautical  Alraanai^  to  the  time  and  place  of  observation^  by  Problen^  V., 
page  298 ;  then,  let  this  reduced  equatioit  be  applied  to  the  given  apparent, 
time  of  observation,  by  addition  or  subtraction,  according  to  the  sign 
expressed  against  it  in  tfie  Bphemeris^  and  the  stam  or  difference  will  be 
the  corresponding  mean  time. 

Ejcample  1. 

'  Jffliuary  24tb,  1S25>  in  longitude  75?  W.^  the  apparent  time  of  observa- 
tion was3U0?10' ;  required  the  mean  tame  i 


Digitized  by 


Google 


416  NAITTICAL  ASTRONOMY. 

Equation  of  Ume  at  noon^  January  24th>  ==     +   12?29' .  5 
Correction  of  ditto  for  8U0r  10!  =  .     .     .     .       +5.1 


Reduced  equation  of  time  = +   12"34'.6 

Apparent  time  of  observation  sr  '  .    •    •    •     3  MO?  10! .  0 

Mean  time,  as  required,  =       3t52?44'.6 

Example  2. 

October  6th,  1825,  in  longitude  80?  E.,  the  apparent  time  of  observation 
was  20^  10T40!  ;  required  the  mean  time  ? 

Equation  of  time  at  noon,  October  6th,  ==  —  11T49|.5* 

Correction  of  ditto  for  14* 50?40!     •    .    •  .     +10.5 

Reduced  equation  of  time  =     .    .    •     .    •  —  12?  0\0 

Apparent  time  of  observation 20MO?40'.0 

Mean  time,  as  requi^d,  =s    ••••••  19*58?40'«0 


Paoblsm  IL 
To  amoert  mean  Tltne,  at  Oreenwichy  into  apparent  Time. 

RULB. 

Reduce  the  equation  of  time,  page  II.  of  the  month  in  the  Nautical 
Almanac,  to  the  given  mean  time  at  Greenwich,  by  Problem  V.,  page  298 ; 
then,  let  this  reduced  equation  be  applied  to  the  mean  time,  with  a  contrary 
sign  to  that  which  is  expressed  against  it  in  the  Ephemeris;  that  is,  by 
addition  when  the  sign  is  negative,  but  by  subtraction  when  affirmative } 
and  the  corresponding  apparent  time  will  be  obtained, 

Example  1. 

January  1st,  1825,  the  mean  titne  at  Greenwich,  per  chronometer,  was 
lot  I3r45 !  ;  required  the  appai^nt  time  ? 

Equation  of  time  at  noon,  January  lst,*=  +  3T56'  •  7 
Correction  of  ditto  for  1 0*  13?45 !  =    .    ,     +   12  .0. 


Reduced  equation  of  timer::  .     .    .    •      —  4T  8'.7 
Mean  time  at  Greenwich  =    .     .    .    •  10*  13T45  * .  0 


Apparent  time  at  Greeniwchs  •    .    .  10?  9?36'.3 

/Google 


Digitized  by ' 


OF  FINDING  THB  'LON6iTUD£  BY  A  CHRONOMETBE.  417 

Example  2. 

September  19th^  1825^  the  mean  time  at  Qreenwicb^  per  chronometer^ 
was  18M5*30! ;  required  the  apparent  time  ? 

Equation  of  time  at  noon,  Sept.  19th,  =    —  6T 14  \  3 
Correction  ofditto  for  18*45 T30!  =  •     •     +  16  .4 


Reduced  equation  of  time  =3  .    •     .    .      +  6T30'.7 
Mean  time  at  Greenwich  =    .    .    .    •  18*45T30\0 


Apparent  time  at  Greenwich  SB  •    •    •   18^52?  0\7 


PUOBLBM  IIL 

Given  the  LatHude  qf  a  Place,  the  observed  Altiiude  of  the  Sun's  Umb, 
and  its  DecUnation;  to  find  the  Longitude  of  thai  Place  by  a  Chrono^ 
meter  or  Jlme-Keeper. 

RULB. 

Let  several  altitudes  of  the  sun*8  limb  be  observed,  at  a  proper  distance 
from  the  meridian,*  and  the  corresponding  times,  per  chronometer,  noted 
down ;  of  these  take  the  means  respectively. 

Let  the  mean  attitude  of  the  sun's  limb  be  reduced  to  the  true  central 
altitude,  by  Problem  XIV.,  page  320. 

To  the  mean  of  the  times  of  observation  apply  the  original  error  of  the 
chronometer,  by  addition  or  subtraction,  according  as  it  was  slow  or  fast 
for  mean  time  at  the  meridian  of  Greenwich,  when  its  rate  was  established; 
to  which  let  its  accumulated  rate  be  applied  affirmatively  or  negatively, 
according  as  the  machine  may  be  losing  or  gaining,  and  the  result  will  he 
the  mean  time  of  observation  at  Greenwich,  which  is  to  be  converted  into 
apparent  time,  by  Problem  II.,  page  416. 

To  the  apparent  time  at  Greenwich,  thud^found,  let  the  sun's  declination 
be  reduced,  by  Problem  V,,  page  298.  Then,  with  the  sun's  true  central 
altitude,  reduced  declination,  and  the  latitude  of  the  place,  compute  the 
apparent  time  of  observiation,  by  any  of  the  methods  given  in  Problem  III., 
page  383 ;  the  difference  between  which  and  the  apparent  time  at  Green- 
wich will  be  the  longitude  of  the  place  of  observation  in  time;— east,  if  the 
former  time  be  greater  than  the  latter ;  otherwise,  west. 


*  See  remarks  on  the  most  favourable  times  for  observatioDy  page  387. 

2  s 


Digitized  by 


Google 


418  NAUTICAL  AtrROIfOlfT. 

Uote. — If  the  meridian  of  the  place  where  the  error  of  the  chronometer 
was  determined  be  different  from  that  of  Greenwich,  let  its  longitude  in 
time  be  applied  to  the  mean  time  of  observation,  per  chronometer,  by 
addition  or  subtraction,  according  as  it  is  west  or  east,  and  the  mean  time 
of  observation  at  Greenwich  will  be  obtained. 

Example  L 

April  Tthy  1825,  in  latitude  4S?43^  N.,  the  mean  of  several  altitudes  of 
the  sun's  lower  limb  was  9?  1 H42^,  and  that  of  the  corresponding  times 
9^37*55!,  by  a  chronometer,  the  error  and  rate  of  which  had.  been 
established  at  noon,  January  1st,  when  it  was  found  4?37-  fast  for  mean 
time  at  (SreMiwicb,  and  gaining  1'.75' daily;  the  height  of  the  eye  above 
the  level  of  the  horizon  was  20  feet }  required  the  longitude  of  the  place 
of  observation  ? 

I 
Mten  time  of  obsermtioii  at  Greenwich  ss    9^37755! 
Original  error  of  the  dironometer  a    •    •     —  4. 37 
Accumulated  rates  1 ' .  75  x  96  days  ss    .     ^  2. 48 


Mean  time  at  Greenwich  s        •    .    •    .    9? 30T30! 
Reduced  ecpiation  of  time  as      •    •    •    •     —  2.  5 


Af^Mrent  time  of  observation  at  Greenw.s  9t28?26! 

Sun's  declination  at  noon,  April  7th  =s    .    6?49'38rN. 
Correction  of  ditto  for  9*  28^25 !  =    .    .     +  8. 53 

Sun's  reduced  deelinatioa  ss      .    •    «    .    6?58^3KN. 
Obs.altof  the  sun'slr.limb=9?llM2r;  hence,  its  true  cent.  alt.  is9?i8C0f 

Lat.  of  the  place  =  48?43'  OfN.      .    ,    ,      liog,  secant  =  0.180599 
Sun's  re4|iQeddec.=  6.58.31.  N,      .    .    .      Log.  secant  =  0, 003226 

Sun's  mer.z.dist.  s=  41?44f29r  Nat. vers.  8»atf253843 
TrucaIt.ofsuii*8cen.=:9. 18.  0  Natco-V.S.s  838396 


Remainder  s  584553    Log.»5. 766824 

Apparent  time  at  the  place  of  observations  5?35?20!Logjis,=5.95064.9 
Apparent  time  of  observation  at  Greenw.=  9. 28. 25 

Longitude  of  die  place  of  obs<»  in  time  s    3!537  5!«s  58?16t5r  west. 

Digitized  by  VjOOQ IC 


OF  FINDING  THJI  LONOITCTDV  BT  A  CHRONOMBTBR.  419 


Example  2. 

May  Ut^  1825,  in  latitude  30?  15  ^  S.,  the  mean  ofseveral  altitudes  of  Olf 
sun's  lower  limb  was  11"^  17 -14?,  and  that  of  the  corresponding  times 
13  ViSTlO!^  by  a  chronometer^  the  error  and  rate  of  which  were  established 
at  noon,  February  Ist,  when  it  was  found  3T25?  slow  for  mean  time  at 
Greenwich,  and  losing  0*.97  daily;  the  error  of  the  sextant  was  2 '30^ 
subtractive,  and  the  height  of  the  eye  23  feet ;  required  the  longitude  ? 


Mean  time  of  observation  at  Greedmeh  n     »  13^23710! 
Original  error  of  the  chronometer  :«s    •    •    »       «f  3,25 
Accumulated  rate  =  0'.97  x  89^  days  =  •       i<    1.27 

II I    ■■■■■>■     I 

Mean  time  at  Greenwich  « 13^28?  2! 

Reduced  equation  of  time  =,••,.»       +3.8 

Apparent  time  of  observation  at  Greenwich  =s  13t31?10! 


Sun*s  declination  at  noon,  May  Ist,  ss      •    •     IS?  4<  19fN. 
Correction  of  ditto  for  13 1 3 1?10!  =  .    .    .     +  10.   9 

Sun's  reduced  declination  s       .    .    .    ,    .    16?14'287N, 

Obs. altof  the  sun's  l.limb=  1 1  ?  17'  14^;  hence,  its  true  cent  alt.\sl  1?24  ^5? 

Lat,  of  the  place  s  dO?15^  OrS.     .    .    .      Log.  secant  »  0.0685iS9 
Sun's  reduced  dec.s  15. 14. 28  N.    .    .    .      Log.  secant  s  0. 015550 

Sun's mer. z. dist.  =  45?29^28r  Nat.  co-sine=701020 
Truealt.of8un'8cen.=:lU24.   5     Nat.  sine  =5    197681 

Remainder  =  503339  Log.=s  5.701880 
Smi's  horary  dist.  from  the  merid.  or  noon=4*26r40!Log.ris.=5. 78097.9 

Appai^nttime  atthe  place  of  observation=19^33t20! 
Apparent  time  of  observation  at  Greenw«=13. 31. 10 

Long,  of  the  place  of  observ.,  in  time  =     6*   2rl0!=90?32l30?  eiwt. 

2b2 

Digitized  by  VjOOQ IC 


420  NAXmCAL  ABTROMOMT. 


Problem  IV. 

Given  the  Latitude  of  a  Place,  and  the  observed  AUUude  of  a  knowfi  fixed 
Star;  to  find  the  LongUude  of  the  Place  of  Observation^  by  a  Chrono- 
meter or  Thne-Keeper. 

Rule. 

Let  several  altitudes  of  the  star  be  observed^  at  a  proper  distance  from 
the  meridian,*  and  the  corresponding  times,  per  chronometer,  noted  down; 
of  these,  take  the  means  respectively.    • 

Let  the  mean  altitude  of  die  star  be  reduced  to  the  true  altitude,  by  Pro- 
blem XVIL,  page  327. 

To  the  mean  of  the  times  of  observation  apply  the  original  error  of  the 
chronometer,  by  addition  or  subtraction,  according  as  it  was  slow  or  fast 
for  mean  time  at  the  meridian  of  Greenwich  when  its  rate  was  established ; 
to  which  let  its  accumulatiBd  rate  be  applied  affirmatively  or  negatively, 
according  as  the  machine  may  be  losing  or  gaining,  and  the  restdt  will  be 
the  mean  time  of  observation  at  Greenwich ;  which  is  to  be  converted  into 
apparent  time,  by  Problem  IL,  page  416. 

To  the  apparent  time  at  Greenwich  let  the  sun's  right  ascension  be 
reduced,  by  Problem  V.,  page  298 ;  and  let  the  star's  right  ascension  and 
declination,  as  given  in  Table  XLIV.,  be  reduced  to  the  period  of  observa- 
tion. Then,  with  the  star's  true  altitude,  its  declination,  and  the  latitude 
of  the  place^  compute  its  horary  distance  from  the  meridian,  by  any  of  the 
methods  given  in  Problem  III.,  page  383. 

Now,  if  the  star  be  observed  in  the  western  hemisphere,  its  horary 
distance  from  the  meridian,  thus  found,  is  to  be  added  to  its  reduced  right 
ascension  ;  but  if  in  the  eastern  hemisphere,  subtracted  from  it :  the  sum, 
or  remainder,  will  be  the  right  ascension  of  the  meridian  j  from  which, 
(increased  by  24  hours,  if  necessary,)  subtract  the  sun's  reduced  right 
ascension,  and  the  remainder  will  be  the  apparent  time  at  the  place  of 
observation ;  the  difference  between  which  and  the  apparent  time  at  Green- 
wich will  be  the  longitude  of  the  place  of  observation  in  time : — east,  if 
the  computed  apparent  time  be  the  greatest ;  if  otherwise,  west. 

Example  1  • 

January  29th,  1825,  in  btitude  40?30'  N..  the  mean  of  several  altitudes 
of  the  star  Aldebaran,  west  of  the  meridian,  was  24?&7 '  OT,  and  that  of  the 

•  Sec  Note,  pa[fe  417. 

Digitized  by  LjOOQ IC 


OF  FINDING  THB  LOfGfTUBB  BT  A  CHRONOMBTBR.  421 

corresponding  times  16?56?3t,  by. a  chronometer,  the  error  and  rate  of 
which  were  determined  at  noon,  January  1st,  when  it  was  found  7*29!  fast 
for  mean  time  at  Greenwich,  and  losing  1*.53  daily;  the  error  of  the 
sextant  was  3'  10^  additive,  and  the  height  of  the  eye  above  the  level  of 
the  horizon  22  feet;  required  the  longitude  ? 

Mean  time  of  observation  at  Greenwich  =     ,     16t56?  3! 


Original  error  of  the  chronometer  =     •     • 
Accumulated  rate  =  1 '  •  53  x  28i  days  = 

Mean  time  at  Greenwich  = 

Reduced  equation  of  time  ==       •     .     •     • 


—  7.29 
+  0.44 

16?49ri8: 
-   13,38 


Apparent  time  of  observation  at  Greenwich  =  16t35T40! 

Sun's  right  ascension  at  noon,  January  29th^  =    20  M7"  1 9 ! 
Correction  of  ditto  for  16?35?40!  =  .     .     ,        +2.60 


Sun's  reduced  right  ascension  s  .  ,  .  ,  20^50?  9! 
Aldebaran's  reduced  right  ascension  =  •  •  4t25?54! 
Aldebaran's  reduced  declination  ==....     16?  8f57*N. 


Aldebaran's  north  polar  distance  =  •    •    •    •    73^51,  37 

Observed  altitude  of  Aldebaran  = 

24?57'0r;  true  altitudes-    •     24°53^387 
Aldebaran's  north  polar  distances: 73. 51.   3     Log.  co-secantsO.  017484 
Lat  of  the  place  of  observation=40. 30.   0    Log.  secant  ss     0. 118954 

Sum=     • 139?  14 Mir  Constant  105.  =  6.301030 

Half  sum  =•    ......    69?37^20§'Log.  cfo-sine  =  9.541836 

Remainders 44. 43. 42i  Log. sine  =  .    9.847417 


Star's  horary  distance^  west  of  the  mer.=4?43?10!Log.rising=5. 82672.1 
Star's  reduced  right  ascension  =     ,    .    4. 25. 54 

Right  ascension  of  the  meridian      •    .    9*  9T  4!* 
Sun's  reduced  right  ascension  s     .     .  20. 50.   9 

Apparent  time  at  the  place  of  observ.s  12M8T55! 
Apparent  time  of  observ.  at  Greenwich=  16. 35. 40 

Longitude  of  the  place  of  obs.,  in  time=  4*  16T45 !  =64?1 1 :  157  w^t 

?  The  right  ucension  of  the  meridian  is  to  be  considered  w  beiD|^  increaaed  by  24  booing 
because  if  is  lest  than  the  sun's  reduced  right  ascension. 


Digitized  by 


Google 


422 


NAUTICAL  ASTRONOMY* 


Example  2. 

January  29th,  1825,  in  latitude  39"?  15  ^  S.,  the  mean  of  several  altitudes 
of  the  star  Regulus,  east  of  the  meridian,  was  10?28'48^,  and  that  of  the 
corresponding  times  3^36*46',  by  a  chronometer,  the  error  and  rate  of 
which  had  been  established  at  noon,  December  1st,  1824,  when  it  was 
found  4T37-  slow  for  mean  time  at  Greenwich,  and  gaining  1 ' .  17  daily; 
the  error  of  the  sextant  was  1  '34T  subtractive,  and  the  height  of  the  eye 
above  the  level  of  the  sea  21  feet;  required  the  longitude  of  the  place  of 
observation  ? 


Mean  time  of  observation  at  Greenwich  = 
Original  error  of  the  chronometer  s  «    • 
Accumulated  rate=l',  17  x  59  days  a 


Mean  time  at  Greenwich  = 
Reduced  equation  of  time  = 


3*36r46* 
+  4.37 
-  1.   9 

8i40?U- 
-   13.33 


Apparent  time  of  observation  at  Greenwich  «     3*.26T41 ! 


Sun's  right  ascension  at  noon,  January  29th, 
Correction  of  ditto  for3*26r41!  = 


Sun's  reduced  right  ascension  ts 
Star's  rediictd  right  ascension  « 

Star's  reduced  declination  =: 
Star's  south  polar  distance  s: 


20»47?19; 

+  0.35 

20t47r54t 

9J59?  3! 

i2?49norN 

I02?49n0^ 

Observed  altitude  of  Regulus  = 

10?28U8r;  true  altitude  =  .     10?17M6r 
Regulus'  south  polar  distance  =    102. 49. 10    Log.  co-secant=:0. 010962 
Latitude  of  the  place  of  observ.  ==  39. 15.   0    Log.  secant  =     0.  II 1039 

I         I  r    , 

Sums: 152"?  2  H56r  Constant  log.  =  6.301030 

Half  sum  = •    76n0^58r  Log.  co-sifte  =  9.378080 

Remainders     .    .    .     .    /.    65.53.12    Log.  sine  =  .    9.960347 


Star's  horary  distance,  east  of  the  merid.a4  f  20?  0!Log.risiiigoA«  76145.8 


Digitized  by 


Google 


OP  PINDIN6  THB  LOVQITUDB  BT  A  CHRONOMBTBR.  4S3 

Star's  horary  distance,  east  of  the  inerid.=4 120?  0! 
Star's  reduced  right  ascension  =s  ,    •      9. 59.   3 


Right  ascension  of  the  meridian  as     ,      5t39"  3! 
Sun's  reduced  right  ascension  =  •    •    20. 47. 54 


Apparent  time  at  the  place  of  observ.  =  8?51?  9! 
Apparent  time  of  observ.  at  Greenwich^  3. 26. 41 


Long.of  the  place  of  observ.,  in  time  »  5{24r28!  »  81?7-0?  east. 


PaoBLBM  V. 

Given  the,]jatUude  of  a  Place,  and  the  observed  Altitude  of  a  Planet ; 
to  find  the  lAmgUude  of  the  Place  qf  Observation^  by  a, Chronometer  or 
Jlme-Keeper. 

RULB. 

Let  several  altitudes  of  the  planet  be  observed,  at  a  proper  distance  from 
the  meridian,*  and  the  corresponding  tfanes,  per  chronometer^  noted  down; 
of  these  take  the  means  respectively. 

Let  the  moMi  altitude  of  the  planet  be  reduced  to  its  true  centiial  altitude, 
by  Probiaa  XVL,  page  325. 

To  the  mean  of  the  times  of  observation  apply  the  original  error  and 
the  accumulated  rate  of  the  chronometer,  as  directed  in  the  last  Problem  : 
hence  the  mean  time  of  observation  at  Greenwich  will  be  obtained ;  which 
is  to  be  converted  into  apparent  time,  by  Problem  II.,  page  416. 

To  the  apparent  time  of  observation  at  Greenwich  let  the  sun's  right 
ascension  be  reduced,  by  Problem  V.,  page  298 1  and  let  the  planet's  right 
ascension  and  declination  be  reduced  to  the  same  time,  by  Problem  VIL, 
page  307*  Then,  with  the  latitude  of  the  place,  the  planet's  reduced 
declinatioiii  and  its  true  central  altitude,  compute  its  horary  distance  from 
the  meridian,  and,  hence,  the  apparent  time  at  the  place  of  observation, 
by  ProUem  v.,  page  397. 

Now,  the  difference  between  the  computed  apparent  time  of  observation 
and  the  apparent  time  at  Greenwich  will  be  the  longitude  of  the  place  of 
observation  in  time;— east,  if  the  former  exceed  tfie  latter;  otherwise, 
west. 

.      •SstNeiSim;e4m 

Digitized  by  VjOOQ IC 


424  NAUTICAI.  ASTRONOMY. 

Example  1. 

February  4th,  1825,  in  latitude  39?5^  N.,  the  meflln  of  several  altitudes 
of  Jupiter's  centre,  east  of  the  meridian,  was  31?25^291',  and  that  of  the 
corresponding  times  12t6?47*9  by  a  chronometer,  the  error  and  rate  of 
which  were  determined  at  noon,  January  1st,  when  it  was  found  3T7  *  fast 
for  mean  time  at  Greenwich,  and  gaining  0'.71  daily;  the  error  of  the 
sextant  was  1'30^  subtractive,  and  the  height  of  the  eye  above  the  level 
of  the  horizon  19  feet ;  required  the  longitude  of  the  place  of  observation  ? 

Mean  time  of  observation  at  Greenwich  =     •     12 1  6^47 1 
Original  error  of  the  chronometer  =z     •    .     •       —  3.   7 
Accumulated  rate  =  0\71  x  34  days  =:     •       —  0.24 


Mean  time  at  Greenwich  =:  ' •     12 1  3?16? 

Reduced  equation  of  time  r:  ....••     —14.20 


Apparent  time  of  observation  at  Greenwich  =    11  t48T56! . 

Sun's  right  ascension  at  noon,  February  4th,  ri  21 M 1 T45 1 
Corrcctionof  ditto  for  11  ?48?56!  =:    •     ...       +   1.59  ' 


Sun's  reduced  right  ascension  c       •    •    •    •    21  ?  13T44! 

Jupiter's  right  ascension  at  noon^  February  l8t,=:8M3?  0' 
Correction  of  ditto  for  3 f  1 1  *48?56 !  =    •     .  .     -  1 .  45 


Jupiter's  reduced  right  ascension  =      •    .    ^      8 14 1  ?  15 ! 

Jupiter's  declination  at  noon,  February  1st,  =:     19?  3'  07N. 
Correction  of  ditto  for  3n  1  ?48?56!  =    .    .       +7.34 


Jupiter's  reduced  declination  =       •    •    .    .     19?10C34rN. 

Observed  central  altitude  of  Jupiter =3 1  ?  25 '  29  f;  hence,  the  true  central 
altitude  of  that  planet  is     •    •    .    31?18'.16r 


Zenith  distance  at  time  of  observation  zz  .    ,    58?4 1 U4 ? 

/Google 


Digitized  by ' 


OP  FINDING  THfi'LOKGirODB  BY  A  CHRONOMETER.  425 

Lat.  of  the  place  =  .    39?  5'.  O^N.    ...      Log.  secantrrO- 110010 
Planet's  red.  dec.  =:  .     19. 10. 34  N.    .    .     .      Log.secant=:0. 024792 


Flanet'Biner.z.  di8t.=  ]9?54^26r    ....      Conat.  log.=:6. 301030 
Zenith  dist.  by  obs.  ==    58. 41 .  44 

Sum=       ....    78?36:i0^Half=39?18'  5^Log.sine=9. 801678 
Difference  =    .    .    .    38.47. 18  Half=:  19. 23.39  Log.8ine=:9. 521223 

Jupiter's  horary  dist.,  east  of  the  merid.=4^  19T  5!LfOg,rising=:5.  75873.3 
Jupiter's  reduced  right  ascension  =   •     8. 41. 15 

Right  ascension  of  the  meridian  =     •    4  *  22?  1 0 '. 
Sup's  reduced  right  ascension  =  .     •  21. 13.44 

Apparent  time  at  the  place  of  observ.  =  7^  8T26! 
Apparent  time  of  obs.  at  GFreenwich  =11. 48. 56 

Longitude  at  the  place  of  obs.,  in  time=  4*40^30!  z:  70?7*30r  west. 

Example  2. 

October  Ist,  1825,  in  latitude  26?40'  S.,  the  m^n  of  several  altitudes 
of  Saturn's  centre,  east  of  the  meridian,  was  10?25<40'',  and  that  of  the 
corresponding  times  6^36724?,  by  a  chronometer,  the  error  and  rate  of 
which  had  been  established  at  noon,  August  1st,  when  it  was  found  3?51 ! 
slow  for  mean  time  at  Greenwich,  and  losing  0' .  49  daily ;  the  error  of  the 
sextant  was  2^20^  subtractive,  and  the  height  of  the  eye  above  the  surface 
of  the  sea  18  feet;  required  the  longitude  ? 

Mean  time  of  observation  at  Greenwich  =:    6*36*24! 
Original  error  of  the  chronometer  =    .     •     +  3, 5 1 
Accumulated  rate  =  0'.  49  X  61^  days  .     +0.30 

Mean  time  at  Greenwich  =:       ....    6*40"45! 
Reduced  equation  of  time  =       .    .    •    •  +  10. 24 


Apparent  time  of  observation  at  Greenwich=:6*51T  9! 

Sun's  right  ascension  at  noon,  Oct.  1st,  =  12^29^*21 ! 
Correction  of  ditto  for  6*51  r9!  =    .     .       +   1.    2 


Sun's  reduced  right  ascension  ==  .  •  .  12*30T23! 
Saturn's  right  ascension  at  Greenwich  timers  *  25?  0! 
Saturn's  declination  at  Greenwich  time  =:  21?4H  OrN. 


Digitized  by 


Google 


426  NAUTICAL  AaraovoMy* 

Observed  altitude  of  Saturn's  centre  =  10^25  '40^;  true  alt.  zz  10?  14'.  12? 


Zenith  distance  = 79?45M8r 

Lat  of  the  place  =  •     26?40^  O^S.     .    .     .      Log.  secant=0. 048841 
Saturn's  declination  =  21.41.   ON.    .'.    •      Log.  8ecant=0. 031872 

Saturn's  men  z,  di8t.=  48?2n  0'/ 

Zenith  dist,  by  ob8.=:  79.45.48      .    •     .     .      Const.  log.=6. 301030 


Sum= 128?  6C48?Half:t64?  3:24rLog.sine=9.953869 

Differences   .     .     ^   31.24.48  Half=:  15. 42.24  Log.fiine=:9. 432508 

Saturn's  horary  dist.,  east  of  the  nierid.=:4^227l5!Log.rifting=:5. 76812.0 
Saturn's  right  ascension  13        ...    5.25 ,*»0 


Right  ascension  of  the  meridian  r:     .It  2T4S* 
Sun's  reduced  right  ascension  =  ..     •  12.30.23 


Apparent  time  at  the  place  of  observ.rz  IS  t32T22! 
Apparent  time  of  obs.  at  Greenwich  ;=  6. 5 1 .   9 


Longitude  of  the  place  of  d)6.^  in  timez:  5  Ml?13!  =  85?18'15?eaat. 


Peoblxm  VI. 

Gwen  the  Latitude  of  a  Place,  and  the  observed  JWtude  of  the^Mom^e 
.    Limb  /  to  find  the  Longitude  ofthePlace  of  Observation,  by  a  Cfcno- 
norneteror  Time-Keeper. 

RULB. 

Let  several  altitudes  of  the  moon's  limb  be.observed^  at  a  proper  distance 
frojn  the  meridian^*  and  the  corresponding  times^  per  chronometer^  noted 
down ;  of  these  take  the  means  respectively. 

To  the  mean  of  the  times  of  observation  apply  the  original  error  and 
the  accumulated  rate  of  the  chronometerj  as  directed  in  Problem  III.^  page 
417  :  the  result  will  be  the  mean  time  of  observation  at  Greenwich,  which 
is  to  be  converted  int(>  apparent  time,  by  Problem  IL,  page  416. 

To  the  apparent  time  of  observation  at  Greenwich  let  the  sun's  right 
ascension  be  reduced,  by  Problem  V.,  page  298 ;  and  let  the  moon's  right 
ascension,  declination,  semi-diameter,  and  horizontal  parallax  be,  also, 
reduced  to  that  time,  by  Problem  VI.,  page  302.  To  the  moon's  reduced 
s^mi-diameter  apply  t&e  augmentation.  Table  IV.,  and  the  true  semi- 
diameter  will  be  obtained. 

•  See  Note,  paje  417. 

Digitized  by  VjOOQ IC 


OF  FINDING  THB  LONGITUDB  BY  A  CHRONOMBTBR.  427 

Let  the  mean  altitude  of  the  moon's  limb  be  reduced  to  the  true  central 
altitude,  by  Problem  XV.,  page  323. 

Then,  with  the  latitude  of  the  place,  the  moon's  corrected  declination, 
and  her  true  central  altitude,  compute  her  horary  distance  from  the  faieri- 
dian,  and,  hence,  the  apparent  time  at  the  place  of  observation,  by  Problem 
VI.,  page  400.  The  difference  between  the  computed  apparent  time  of 
observation  and  that  at  Greenwich,  will  be  the  longitude  of  the  place  of 
observation  in  time ; — and  which  will  be  east,  if  the  computed  time  be  the 
greatest  3  if  otherwise,  west. 

Example  !• 

April  21st,  1825,  in  latitude  50? 48'  N.,  the  mean  of  several  altitudes  of 
the  moon's  lower  limb,  west  of  the  meridian,  was  29?30^26T,  and  that  of 
the  corresponding  times  12ti6?58?,  by  a  chronometer,  the  error  and  rate  of 
which  had  been  established  at  noon,  February  1st,  when  it  was  found  7*^46! 
fast  for  mean  time  at  Greenwich,  and  losing  6' .  79  daily ;  the  error  of  the 
sextant  was  2 '25?  additive,  and  the  height  of  the  eye  above  the  level  of 
the  horizon  17  feet;  required  the  longitude  of  the  place  of  observation  ? 

Mean  tfme  of  observation  at  Greenwich  =    12*  6*58! 

Original  error  of  the  chronometer  s  .     .       —  7^  46 

Accumulated  rate  S5  0\  79  x  79|  days  =    +1.3 


I 


Mean  time  at  Greenwich  =     •    •     •    •     12?  0?I5! 
Reduced  equation  of  time  s     ,    .    .    ,       +   1 .  27 


Apparent  time  of  observ.  at  Greenwich  ss    12 1  l?42t 

Sun's  right  ascension  at  noon,  April  2l8t,=  1?55'!'41 '.  5 
Correction  of  ditto  for  1 2  *  1 T42 !  =     .     •     4-1,52.3 


Sun's  reduced  right  ascension  =      •     .     •     1*57  "33 ' .  8 

Moon's  semi-diameter  at  midnt.,  April  2l8t^  15 f  14T* 
Augmentation  of  ditto.  Table  IV.,  »     *    .    .     4-  7 


Moon's  true  semi-diameter  s     •    •    •    •    •  15'2K 


*  The  apparent  time  at  Greenwich  belo^  so  rery  close  to  mldnifht,  and  the  Tariation  In 
the  nooa'e  Jeriiaatioa/  — H-dwaeter,  and  horiaontal  parallax  but  triHio;,  no  comctkui 
lor  these  elements  becomes  necessagr  in  the  present  instance. 


Digitized  by 


Google 


428  NAUTICAL  A8TRONOMT, 

Moon's  right  ascension  at  midnight,  April  2l8t    s     70?57'59^ 
Corrected  proportional  part  of  ditto  for  0MT42!  s=      -f  0. 56 


Moon's  corrected  right  ascension  =        .    .    •    .     70*?  58' 55  T 

Moon's  declination  at  midnight,  April  2l8t,  =      •    23?  9:28rN* 

Moon's  horizontal  parallax  at  midnight,  April  2l8t  7=     55'54T* 

Observed  altitude  of  the  moon's  lower  limb=2d?30'.  26^';  hence,  her  true 
central  altitude  is '30^31 '.  191. 

Lat.  of  the  place  •    .    50?48^  OfN.     •    .    ,      Log.  6ecant=0. 199263 
Moon's  corrected  dec,=23.   9.28  N.     .     .    .      Log.  secant=0. 036483 


Moon's  men  zen.dist.=27?38'32rNat.yers.S.sI  14138 
Moon's  true  cent.  alt.  =30.31.  l9Nat.co-V.S.=492131 


Remainders  377993 Log.=5. 577484 

Moon's  horary  dist,  west  of  the  mer.=  4?38?10!  Log.  rising=5. 81323.0 
Moon's  corrected  right  ascension  = 
70?58;55r,inUme=    ....    4.43.56 


Right  ascension  of  the  tneridian  =     •    9?  22?  6 ! 
Sun's  reduced  right  ascensioa  ^  •     •     1 .  57. 34 


Apparent  time  at  the  place  of  observ.=7*24T32! 
Apparent  time  of  obs.  at  Greenwich^:  12.    1. 42 


Longitude  of  the  place  of  obs.,  intime=4J37"10!  =  69'?l7'30r  west/ 


Example  2. 

September  2d,  1825,  in  latitude  40?  10'  S.,  the  mean  of  several  altitudes 
of  the  moon's  lower  limb,  east  of  the  meridian,  was  9?8'36?,  and  that  of 
the  corresponding  times  6t39T0!,  by  a  chrpnometer,  the  error  and  rate  of 
which  were  determined  at  noon.  May  1st,  when  it  was  found  4?10!  slow 
for  mean  time  at  Greenwich,  and  gaining  1'.37  daily;  the  error  of  the 
sextant  was  1 ',  20''  subtractive,  and  the  height  of  the  eye  above  the  surface 
of  the  sea  14  feet ;  required  the  longitude  ? 


*  See  Note;  pa^  427. 

/Google 


Digitized  by ' 


OF  FINDING  THB  LONGITUDE  BY  A  CHRONOMSTER.  429 

Mean  time  of  observation  at  Greenwich  =:  •    •    6t39?  0' 
Original  error  of  the  chronometer  =       .    •  '  •     +  4. 10 
Accumulated  rate  =  1'.37  x  124iday8s     •     —  2.50 

Mean  time  at  Greenwich  = 6*40r20! 

Reduced  equation  of  time  ss +  0. 33 


Apparent  time  of  observation  at  Greenwich  »=     6M0?53! 

Sun's  right  ascension  at  noon,  September  2d,  ^  10?44T53*.  5 
Correctionof  ditto  for  6*40?53!  =     .     .     •        +   1.   0  .5 

Sun's  reduced  right  ascension  =      .     .     •     .     10*45 T54 ' .  0 

Observed  altitude  of  the  moon's  lower  limb  =  9?8^36^  ;  hence,  the  true 
central  altitude  of  that  oliject  is  10?6^23r. 

Moon's  right  ascension  at  noon,  September  2d,=:S0?5i3'  8^ 
Ck)rrected  prop,  part  of  ditto  for  6?40r53!  =  +  3. 19.   9 

•  Moon's  corrected  right  ascension*=d      .     •    •    34?  17*  17^ 

Moon's  declination  at  noon,  September  2d,  =     1 6  ?  1 2 '  13  f  N. 
Corrected  prop,  part  of  ditto  for  e*40r58!  =      +  54.   3 

«     ■  111         !■■ 

Moon's  corrected  declination  =  .     .     .     •     .17?  6'16rN. 


Moon's  semi-diameter  at  noon,  September  2d2  =  14^46f 
Correctionof  dittofor  6*40T53!  =  .  .  •  .  4-1. 
Augmentation,  Table  IV.,  =  .......        +2 


Moon's  true  semi-diameter  = •   .     14^491^ 

Moon's  horizontal  parallax  at  noon,  Sept.  2d,  =     54'.  11  • 
Correction  of  ditto  for  6  M0To3'  =       ....        +4 

Moon's  true  horizontal  parallax  =    .     .     .     .     .    64' 15^ 

Lat.  of  the  place  .     .    409 10'  OlS.     .     .     .      Log.  secant=0. 1 16809 
Moon'8correcteddec.=  17.   6. 16  N.     .     .     *      Log.  secant=0. 019647 

Moon's  mer.  z.  dist.  =    57?16'.  16rNat,vers.  S.=459.336 
Moon's  true  cent,  alt.=  10.   6.23  Nat.  co.V.S.=824524 

*  Remainder  =  365 188  Log.sS.  5625 16 

Moon's  hotary  dist.,  east  of  the  merid.=4  *  OT  01  Log.  rising=5. 69897. 2 

Digitized  by  VjOOQ IC 


430  MAirncAL  astronoiit* 

Moon'8  horary  dist,  east  of  the  inerid.rs4 1  0?  0? 
Moon's  reduced  right  ascension 
349l7'l7?,intime«    •    •     .    .     2.17.   » 


Right  ascension  of  the  meridian  as    •  22  M  7*  9' 
Sun's  reduced  right  ascension  s=       •  10. 45. 54 

Apparent  time  at  the  place  of  ohsery.=l  1 1 31 715 ! 
Apparent  time  of  obs.  at  Greenwich  =  6. 40. 53 

Longitude  of  the  place  of  obs.,  in  time=4!50r22!  =  72?35  ^SOr  east- 

Remark  1. — ^The  longitude,  thus  deduced  from  the  true  central  altitude 
of  the  moon,  will  be  equally  as  correct  as  that  inferred  fipom  the  mm's 
central  altitude,  provided  the  moon's  place  in  right  ascension  and  declina- 
tion be  carefully  corrected  by  the  equation  of  second  difiference,  as  explained 
between  pages  33  aad  38.  Whatever  little  extra  trouble  may  be  attendant  on 
this  particular  operation,  will  be  infinitely  more  than  counter-balanced  by  the 
pleasing  reflection  that  it  affords  the  mariner  an  additional  method  of  find- 
ing the  longitude  of  his  ship,  either  by  night  or  by  day,  with  all  the  accu- 
racy that  can  possibly  result  from  the  established  rate  or  going  of  hia 
chronometer. 

Remark  2.— It  frequently  happens  at  sea,  that,  owing  to  clouds,  rains, 
or  other  causes,  ships  are  whole  days  without  profiting  by  the  presence  of 
the  sun,,  or  obtaining  an  altitude  of  that  object  for  the  purpose  of  ascer- 
taining either  latitude  or  longitude ;  but  it  must  be  remembered,  that  there 
are  few  tiights,  if  any,  in  which  some  fixed  star,  a  planet,  or  the  moon, 
does  not  present  itself  for  observation,  as  if  intended  by  Providence  to 
relieve  die  mariner  from  the  great  anxiety  which  the  doubtAil  position  of 
his  ship  must  naturally  excite  in  him,  particularly  when  returning  from  a 
long  voyage,  and  about  to  enter  any  narrow  sea,  such  as  the  English  Chan- 
nel. -Under  such  circumstances,  the  three  preceding  problems  will  be 
found  exceedingly  useful;  because  they  exhibit  safe  and  certain  means  of 
finding  the  true  place  of  a  ship,  so  far  as  the  going  of  the  chronometer 
used  in  the  observation  can  be  depended  upon.  In  this  case,  since  a  know- 
ledge of  the  heavenly  bodies  becomes  indispensably  necessary,  the  reader 
is  Kfeired  to  '^  The  Young  Navigator's  Guide  to  the  Sidereal  and  Planetary 
Parts  of  Nautical  Astronomy,"  where  a  familiar  code  of  practical  directions 
is  given  for  finding  out  and  knowing  al!  the  principal  fixed  stars  and  planets 
in  the  firmament. 


Digitized  by 


Google 


OF  FINDING  THX  LOMGITODB  BT  tOKAR  OBSERVATIONS.         431 


PaoblbK  VIL 

To  Jind  th0  JLongiiude  qfa  SUp  or  Place  by  celestial  ObMrvaltmiy  com^ 
mimhf  coiled  a  hmar  ObeervQtiom. 

The  direct  progressive  motion  of  a  ship  at  sea  is  so  liable  to  be  disturbed 
by  various  unavoidable  and  often  imperceptible  causes,— such  as  a  frequent 
aberration  from  the  true  course,  by  the  ship's  continually  varying  a  little, 
in  contrary  directions,  round  her  centre  of  gravity ;  high  seas  with  heavy 
swells,  sometimes  with  and  at  other  times  against,  or  in  directions  oblique 
to  the  true  course ;  storms,  sudden  shifts  of  wind,  unknown  currents,  local 
magnetic  attraction,  unequal  attention  in  the  helm^-men,  with  many  other 
casualties  which  cannot  possibly  be  properly  provided  for, — that  the  place 
indicated  by  the  dead  reckoning  is  frequently  so  erroneous  as  to  be  whole 
degrees  to  the  eastward  or  westward  of  the  actual  position  of  the  ship.  Of 
this  every  person  must  be  fully  aware,  who  has  navigated  the  short  run 
between  England  uid  the  nearest  of  the  West  Indian  Islands. 

As  the  best  account  by  dead  reckoning  is  evidently  but  a  very  imperfect 
kind  of  guess-work,  it  should  be  employed  only  as  an  amdliary  to  the 
elementary  parts  of  navigation,  and  never  confided  in  but  with  the  utmost 
<»ution.  Hence  it  is  that  celestial  observation  should  be  constantly 
resorted  to,  because  it  is  the  only  certain  \^ay  of  detecting  the  errors  of 
dead  reckoning,  and  of  ascertaining,  with  any  degree  of  precision,  the 
actual  position  of  the  ship. 

If  a  chronometer  or  time-keener  could  be  so  constructed  as  to  go 
uniformly  correct  in  all  seasons,  places,  and  climates,  it  would  immediately 
obviate  all  the  difficulties  attendant  on  a  ship's  reckoning,  atid  thus  render 
the  longitude  as  simple  a  problem  as  the  latitude ;  for,  such  a  machine 
being  once  regulated  to  the  meridian  of  Greenwich,  would  always  show 
the  absolute  time  at  that  meridian ;  and,  hence,  the  longitude  of  the  place 
of  observation,  as  has  been  illustrated  in  the  four  preceding  problems;  but 
those  pieces  of  mechanism  are  so  exceedingly  complicated,  and  sd  extremely 
delicate,  tiiat  they  are  liable  to  be  affected  by  the  common  vicissitudes  of 
seasons  and  climates,  and  also  by  any  sudden  exposure  to  a  higher  or  lower 
d^ree  of  atmospheric  temperliture  than  that  to  which  they  have  been 
ftccvstomed :  the  celestial  bodies  ought,  therefore,  to  be  consulted,  at 
all  times,  in  preference  to  machines  so  subject  tonnitabitity,  and  should 
ever  be  confided  in  by  the  mariner,  as  the  only  immutable  and  unerring 
time- keepers. 

Of  all  the  apparent  motions  of  the  heavenly  bodies,  in  the  zodiac,  with 
which  we  are  acqucdnted,  that  of  the  moon  is  by  far  the  most  rapid ;  it 


Digitized  by 


Google 


432  NAUTICAL  ASmONOMY. 

being,  at  a  mean  rate,  about  13?  10^  in  24  hours,  or  nearly  half  a  minute 
of  a  degree  in  one  minute  of  time.  Hence,  the  quickness  of  the  moon's 
motion  seems  to  adapt  her  peculiarly  to  the  measurement  of  small  portions 
of  corresponding  time;  and,  therefore,  careful  observations  of  the  angular 
distance  of  that  object  from  the  sun,  a  plan.et,  or  a  fixed  star  lying  in  or 
near  the  zodiac,  afford  the  most  eligible  and  practicable  means  of  deter- 
mining the  longitude  of  a  ship  at  sea :  for  the  true  distance  deduced  from 
observation,  being  compared  with  the  computed  distances  in  the  Nautical 
Almanac,  will  show  the  corresponding  time  at  Greenwich ;  the  difference 
between  which  and  the  apparent  time  at  the  place  of  observation  will  be 
the  longitude  of  that  place. in  time;  and  which  will  be  east  if  the  time  at 
the  place  of  observation  be  greater  than  the  Greenwich  time,  but  west  if  it 
be  less. 

The  method  of  finding  the  longitude  at  sea,  by  lunar  observations,  is 
very  familiarly  explained,  by  geometrical  construction  and  by  spherical 
calculation,  in  '^The  Young  Navigator's  Guide  to  the  Sidereal  and 
Hanetary  Parts  of  Nautical  Astronomy,"  between  pages  172  and  212^ 
where  it  will  be  seen  that  in  a  lunar  obser\'ation  there  are  two  oblique 
angled  spherical  triangles  to  work  in,  for  the  purpose  of  finding  the  true 
central  distance;  in  the  'first  of  which  the  three  sides  are  given^  viz.^  the 
apparent  zenith  distances  of  the  two  objects,  and  their  apparent  central 
distance,  to  find  the  angle  at  the  zenith,— that  is,  the  angle  comprehended 
lietween  the  zenith  distances  of  those  objects ;  and,  in  the  other,  two  sides 
and  the  included  angle  are  gi^n,  to  find  the  third  side,  viz.,  the  true  zenith 
distances  of  the  objects ;  and  their  contained  angle,  to  find  the  side  oppo- 
site to  that  angle,  or  the  true  central  distance  between  those  objects.  The 
solution  of  the  first  triangle,  falls  under  Problem  V.,  page  207>  and  that  of 
the  second  under  Problem  III.,  page  202.  This  is  the  direct  spherical 
method  of  reducing  the  apparent  central  distance  between  the  moon  and 
sun,  a  planet,  or  a 'fixed  star,  to  the  true  central  distance ;  or,  in  other 
words,  diat  of  clearing  the  apparent  central  distance  between  those,  objects 
of  the  effects  of  parallax  and  refraction :  but,  this  being  considered  by  some 
mariners  as  rather  a. tedious  operation,  the  following  methods  are  ^ven, 
which,  being  deduced  direcdy  from  the  above  spherical  principles,  will  be 
always  found  universally  correct ;  and,  since  they  are  not  subject  to  any 
restrictions  whatever,  they  are  genei^al  in  every  case  wliere  a  lunar  observa- 
tion can  be  taken.  Besides  this,  they  will  be  found  remarkably  simple  and 
concise,  particularly  when  the  operations  are  performed  by  the  Tables 
contained  in  this  work. 


Digitized  by 


Google 


OF   FINDING  THB   IX>NGITUDB   BT   LUNAR  OBSBRVATIONS. 


43? 


JMbthod  I. 

*  .  .  ....  t 

Cf  reducing  the  apparent  to  the  true  central  Distance. 

Rulb.  • 

Take  th^  auxiliary  angle  from  Table  XX.,.  and  let  it  be  corrected  for  the 
sun's,  bUht%  or  planet's  apparent  altitude,  as  directed  in  pages  41  and  45. 

Find  the  difference  of  the  .apparent  altitudes  of  the  objects,  and,  also,  the 
difference  of  their  true  altitudes. 

Then,  to  the  natural  versed  sines  supplement  of  the  sum  and  the  differ^ 
en^e  of  the  auxiliary  angle  and  the  .difference  of  the  apparent  altitudes, 
add  the  natural  versed  sines  of  the  sum  and  the  difference  of  the  auxiliary 
angle  and  the  apparent  distance,  and  the  natural  versed  sine  of  the  differ- 
ence of  the  true  altitudes :.  the  sum  of  these  five  numbers,  abating  4  in  the 
radii  or  left-hand  place^  will  be  the  ntttural  versed  sine  of  the  true  central 
distance. 

• 
Example  1. 

Let  the  apparent  central  distance  between  the  moon  and  sun  be 
6fi?48'S41f,  the  sun's  apparent  altitude  60? IS'.SSr,  the  lAoon's  ^parent 
altitude  17?  15*  151^,  and  her  horizontal  parallax  59M3^  $  required  the 
true  central  distance  ? 

Sun'^  apparent  alt.  =  60?15'.35r-Correc.  0C29^=true  alt.=:60?15C  6r 
Moon's  apparent  art.=:  17*  15.  15+Correc.  54.  0  =true  alt.=:18.   9.  15 


Diff.  of  the  app:  alts.sr43?  0^20"!^ 
Auxiliary  angle  =    .  60.    9.*  27 
Apparentcentraldist.r=66. 48, 34 


Diff.  of  the  true  alts.  =  42?  .5'5K 


Suip  of  auxiliary'  angle 

anddiff.ofap.alts.=:103?  9^47^ 
difference  of  ditto  .==    17.   9.    7 
Sum  aux.ang.&ap.dist.126.  58.    I 
Difference  of  ditto  =      6.39.    7 
Diff.  of  the  true  alts.=  42.   5.51 


Tme  central  di8tance=66?  2'20r     Nat.   versed  sine   =     *      0,593882 

2  F 

'    .  Digitiz'ed  by  Google 


Nat.  versed  sine 

sup.   r= 

0.772277 

Nat.  Versed  sine 

sup.    =s 

1.955526 

Nat.   versed   sine 

=     . 

1.601354 

Nat.  versed  sine 

•-™   •  • 

0.006730 

Nat.  versed 'sine 

"~     • 

0.257995 

434 


NAUTICAL   ASTRONOMY. 


General  Remarks. 

1.'  The  correction  of  the  moon's  apparent  altUude  is  contained  in  Table 
XVIIl.,  and  is  to  be  taken  out  therefrom  agreeably  to  the  directions  given 
in  page -39. 

2.  The  correction  of  the  sun's  apparent  altitude  is  the  difiference  between 
the  refraction  and  the  parallax  Corresponding  to  that  altitude  in  Tables 
Vlll.andVIl. 

.  3.  The  correction  of  a  planet's  apparent  altitude  is  the  difference  between 
the  refraction  and  the  parallax  answering  to  that  altitude  in. Tables  VQI. 
aUdVI.    And, 

4.  The  correciuni  (ff  a  starts  apparent  aUitude  is  the  refraction  corre- 
sponding thereto  in  Table  VIIL'  The  fixed  stars  have  not  any  sensibk 
parallax. 

Matample  2.      . 

Let  the  apparent,  central  distance  between  the  moon  and  a  fixed  star  be 
37^12'40'r,  the  star's  apparent  aUitude  H?27'50?,  thempon's  apparent 
altitude  40?55'.  15?,  and  her  horizontal  parallax  54' 10"?^  required  the 
true  central  distance  ?  •   ' 

•Star'«  apparent  alt.=  ll?27<50r.^Correc.  4f35r=truealt.=  ll?23M5r 

M'oon'sapparentalt,=s40.55. 15   +Correc.39.51   =true  alt.=41.3S.    6 

f^  II  -    -• 

Difr.oftheapp.alt8.=39?27'25?     Difference  ofthetruealts.=30?l  1^51  *: 
Auxiliary  angle=    .    60*  19, 30 
Apparent  cent.  dist.=37. 12^  40 


Sum  of  auxiliary  angle 
and  diflF.of  ap.alUi-=89?46^55? 
Difference  of  ditto  =30. 52.   5 
Sumaux.^ilg.&ap.dist.97.32. 10 
Difference  of  ditto  =23.   6.50 


Nat.  versed  sine  sup. 
Nat.  versed  ^inc  sup. . 
Nat.  versed  sine  = 
Nat.  versed  .sine  = 


Diff.of  the  true  alts.=?30. 11.51       Nat.  versed  sine  ss 


1.003806 
1.85835^2 
1.131151 
0.080274 
0. 135703 


True  central  dist.    =37°44^52?     Nat.  versed  sine  =    .     .     0.209286 

Remark  1. — InsteaH  of  the  natural  versed  aiiies  supplement  of  the  fipt 
two  terms  in  the  calculation,  the  natural  versed  sines  of  the  supplements  of 
those  terms  to   180?   may  be  taken :  for  it  is  evident  that  the  natural 


Digitized  by 


Google 


OF  FINDING  TUB   LONGITUDE  BT  LUNAR  OBSBRVATIONS.         485 

versed  sine  of  the  tupplefitent  of  an  arch  is  the  natural  versed  sine  supple- 
ment of  that  arch.  Thus^  in  the  above  example^  the  supplement  of 
89?46^55r  is  90^3^5^,  the  natural  versed  sine  of  which  is  1.003806; 
and  the  supplement  of  30?52^5?  is  149^7 -55?^  the  natural  versed  sine  of 
which  is  1. 85  835  2,  the  same  as  above.  By  this  transformation  of  the  first 
two  terms,  all  the  tabular  numbers  that  enter  the  calculation  will  become 
affirmative. 

Remafk  2.-«rWhen  the  sum  of  the  auxiliary  angle  and  the  apparent 
centra]  distance  exceeds  a  semi-circle,  or  180  degrees,  the  natural  versed 
sine  supplement  of  its  excess  labove  that  quantity  is  to  be  taken,  or,  which 
is  the  same  thing,  the  natural  versed  sine  of  its  supplement  to  360  degrees. 

Remark  3>-Instead  of  using  the  natural  versed  sines  supplement  of  the 
first  two  terms  in  the  calculation,  as  above,  or  the  natural  versed  sines  of 
their  supplements  to  ISO?,  as  mentioned  in  Remark  1,  the  natural  versed 
sines  of  those  terms  may  be  employed  directly  3  as  thus  : — Let  the  sum  of 
the  natural  versed  sines  of  the  first  two  terms  be  subtracted  from  the  warn 
of  the  natural  versed  sines  of  the  last  three  terms,  and  the  remainder  will  be 
the  natural  versed  sine  of  the  true  distance.  "^ 


Example. 

Let  the  apparent  central  distance  between  the  moon  and  sun  be 
119?53^58?,  the  sun's  apparent  altitude  22?10C35T,  the  moon's  apparent 
altitude  15^51:22^,  and  her  horizontal  parallax  58U0r ;  required  thf  true 
central  distance  ? 

Sun'sapparentalt.  «22?10'.3.5r--Corr^.   2'!  K«  true  alt =22?  8C24r 
Moon'sapparentalt.=  15.51.22  +Corree;53,   7  Fstruealt.sl6.44.29 


Diff.ofapparentalts.=i  6?19n3r    Diff.  of  true  altitudes    s    5?23^6Sr 
Auxiliary  angle  =  .    60.   8.25 
Apparent  cent.  diat.=s  1 19. 53. 58 


Bum  of  the  aux.  angle 

anddiff.ofap.alts.=66?27'38r  Nat.V.S.^0.600620>  «      _,  01029« 

Difference  pf  ditto  ^  53. 49. 12  Nat.V.S.=0. 409676)  ^°*^ *'       ^^ 
Sum  of  auxiliary  angle 

andapp.dist.  =  180.   2.23  Nat.V.S.=2.000000l 

Difference  of  ditto=  59. 45. 33  Nat.V.  S.==0. 496365  >Sum=2. 500800 

Diff.  of  true  alts.   =     5.23.55  Nat.V.S. =0.004435) 

True  cential  dist.  =  1  ig*?  22  i  25  r  Nat.  versed  sine  =    .    .    .    1 .  490504 

2f2 


Digitized  by 


Google 


436 


NAUTICAL  ASTRONOMY. 


Mbthob  II. 
Of  redttdi^  tA6  opporcnt  to  tAe  !rtt6  centra!  IKfiance. 

RUJLB. 

Take  the  auxiliary  angte  from  Table  XX.,  and  let  it  be  corrected  for  the 
ftun's,  star's^  or  placet's  apparent  altitude^  as  directed  in  pages  44  and  43. 

Find  the  sum  of  the  apparent  altitudes  of  the  objects,  and,  ako,  the  sum 
of  their  true  altitudes ;  then, 

To  the  natural  versed  sines  of  the  «um  and  the  difference  of  the  auxiliary 
angle  and  the  sum  of  the  apparent  altitudes,  add  the  natural  versed  sines  of 
the  sum  and  the  difference  of  the  auxiliary  angle  and  the  apparent  distance, 
and  the  natural  versed  sine  supplement  of  the  sum  of  the  true  altitudes ;  the 
sum  of  these  five  terms,  abating  4  in  the  radii  or  left-hand  place,  will  be 
the  natural  verised  sine  of  the  true  central  distance. 


Example  1. 

Let  the  apparent*  central  distance  between  the  moon  and  Venus  be 
53?49j:54r3the  apparent  altitude  of  Venus  19?10<40r,  and  her  horizontal 
parallax  23T ;  the  moon's  apparent  altitude  37?40^20^,  and  her  horizontal 
parallax  59^47?  ;'reqiiired  the  true  central  distance  7 

Venus'  apparent  alt.=:19?10:40^-Correc.   2^2K=truealt.=  19?  8^9^ 
Moon's  apparentalt.=37. 40. 20  +Ck>rrec. 46.   5  ^true  alt.=38. 26. 25 


Sum  of  the  app.  alts.s  36?  5 1 '  Or     Sum  of  the  true  alts,  s 
Auxiliary  angle  =      60.20.14 
Apparent  cent.  dist.=  53. 49. 54 


57^34  U4r 


Sum  of  auxiliary  angle 

&sumofap.alts.=:117?lIM4r 
Difference  of  ditto  =     3.  29. 14 
SumauxAng.&ap.dis.ll4. 10.   8 
Difference  of  ditto  =:     6. 30.  20 
Sum  of  the  true  alts.s=:57. 34. 44 


Nat.  versed  sine  = 
Nat.  versed  sine  = 
Nat.  versed  sine  = 
Nat.  versed  sine  = 
Nat.  versed  sine  sup< 


1.456899 
0.001851 
1.409423 
0.006439 
1.536138 

0.410755 


True  central  dist.  =    53?53M8r      Nat.  versed  sine  s  . 

Note.'^'For  the  correctioas  of  the  apparent  altitudes  of  the  objects,  see 
remarlbs,  pa^  434. 


.    Digitized  by 


Google 


OF  FINDING  THB    L0N6ITUDB  BY  LtTNAR  OBSERVATIONS. 


487 


Example  2. 

Let  the  apparent  central  distance  between  the  moon  an4  sun  be 
119?57'56^,  the  sun's  apparent  altitude  18?10'50'',  the  moon's  apparent 
altitude  10?30nOf,  and  her  horizontal  parallax  60^37^;  required  the 
true  central  distance  ? 


Sun's  apparent  alt.=  18?10^50r-Correc.   2M4'r=truealt.=  18?  8'.  6t 
Moon's  app^ent  alt.s  10. 30. 10  +Correc.  54. 35  =strue  alt=  1 1 .  24. 45 

Sumoftheapp.aIts.=2.8?41^  Oir     Sum  of  the  true  alts.  =    29?32'51^ 
Auxiliary  angle  a  •    60. .  5. 34. 
Appar.  central  dist.=  119.57-56 


Sum  of  auxiliary  angle 

&  sum  of  app.  alts.=:88?46^34r 
Difference  of  ditto  =  31.  24. 34 
Sum  aux.ang.&ap.dis.  1 80.   3. 30 
Difference  of  ditto  =  59. 52.  22 
Sum  of  the  true  alts.=29.32.51 


Nat.  versed  sine  =3  •  . 
Nat.  versed  sin^  =  •  . 
Nat.  versed  sine  ^  •  « 
Nat.  versed  sine  =  •  < 
Nat.  versed  sine  sup.  == 


True  central  dist.  =  1 19?33C  4r      Nat  versed  sine 


0.978640 
0.146535 
1.999999 
0.498076 
1.869948 

1.493200 


Remark  1. — Instead  of  using  the  natural*  versed  sine  supplement  of  the 
sum  of  the  true  altitudes,  the  natural  versed  sine  of  that  term  may  be 
employed :  in  this  case,  if  from  the  sum  of  the  natural  versed  sines  of  the 
first  four  terms  in  the  calculation,  the  natural  versed  sine  of  the  last  term 
be  taken,  the  remainder,  abating  2  in  the  radii  or  left-hand  place^  wiU  be 
the  natural  versed  sine  of  the  true  central  distance. 

Hemark  2.  When  the  sum  of  the  auxiliary  angle  and  the  apparent  cen-* 
tral  distance  exceeds  a  semi-circle,  or  180%  the  natural  versed  sine  supple- 
ment of  its  excess  above  that  quantity  is  to  be  tak^n,  or,  which  amounts  to 
the  same,  the  natural  versed  sine  of  its  supplement  to  360?,  as  in  the  above 
example.  The  same  is  to  be  observed  in  the  event  of  the  aggregate  of  the 
auxiliary  anglQ  and  the  sum  of  the  apparent  altitudes  exceeding  180 
degrees :  this,  however,  will  but  very  rarely  happen,  . 


Digitized  by 


Google 


498  IIAUTICAL  ASTRQNOinr* 

mbthod  hi. 

Of  reducing  the  apparent  to  the  true  central  Distance. 

Rule. 

Take  the  logarithmic  difference  from  Table  XXIV.,  and  let  it  be  cor- 
rected for  the  tun's,  star's,  or  planet's. apparent  altitude,  aa  directed  in 
pages  49,  51,  and  52. 

Find  the  difference  of  the  apparent  altitudes  of  the  object^,  and,  also, 
the  difference  of  their  true  altitudes. 

.Then,  from  the  natural  versed* sine  of  the  apparent  distance,  subtract 
the  natural  versed  sine  of  the  difference  of  the  apparent  attitudes  j  tp  the 
logarithm  of  the  remainder  let  the  logarithmic  difference  be  added,  and  the 
sum  (abating  10  in  the  index^  wil)  be  the  logarithm  of  h  natural  number; 
which,  being  added  to  the  natural  versed  sine  of  the  difference  of  the  true, 
altitudes,  will  give  the  Natural  versed  sine  of  the  true  central  distance. 

£4faf^ple  1. 

Let  th9  apparent  central  distance  between  the  moon  and  Mara  be 
83'?10'.23r,  the  apparent  altitude  of  Mars  17?10^20r,  and  his  horizontal 
pi^all^x  15? ;  the  moon's  apparent  altitude  31  ?20'.30T,  and  her  horizontal 
parallax  58'  531f  |  required  the  true  central  distance  ? 

Mara'  apparent  alts  l7?10'20?-^Ck>rrec.   aUfir=true  alusl??  7'3ir 
Moon's  appt.  alt.  =  31. 20. 30  +Correc.  48. 44  stru«  alt,:=:82,   9. 14 

Diff.ofappar.alts.=  14?10M0r^  Diff.  of  true  altitudes  =:      15?  lM3r 

Apparent  disuneet:  83?10^28rNat.V.S.=:881129 

Diff«  of  appar.  alto.:3  14. 10. 10  NatV. 8.2^030436  tog.  diff«=39. 996299 

Remainder  =:  850693  Log.  =s      5. 929773 

Natural  number  = 843476  Log.  =:      5.926072 

Diff.  of  the  true  alts.=15?  lUSrNat.V.S.=034204 


True  central  dist  =  82?58^26rNat.V.S.=877680 

Nofe.— For  the  corrections  of  the  apparent  altitudes  of  the  objects,  see 
remarks^  page  434. 


Digitized  by 


Google 


OF  FINDING  THg  tONGITyBB  BY  LUNAR  OBSBRVATIONS.        439 

Example  2. 

Let  the  apparent  central  distance  between  the  moon  and  sun  be 
I18?56'40^,  the  sun's  apparent  altitude  16?40flOf^  the  moon's  apparent 
altitude  9?39'50f^  and  her  horizontal  parallax  59'  19r  i  required  the  true 
centra]  distance  ? 

Sun's  apparent  alt*  a  16? 40n or -Correc.  S^  Ors:truealt.sl6?37nOr 
Moon's  apparent  altss  9. 39. 50  +Correc.  S9.   3  «:trae  alt,ss  10. 32. 53 


Diff.  of  the  app.  alts.=  7?  0' 20r    Diff.  of  the  true  alts.  »   .    6?  4^  17r 

Apparent  distance  =U8?56^40r  N.V.S.=  1.483961 

Diff.  of  appar.  alts.  =    7.   0,20   N.y.S.=  .007466Log.diff.=9. 998919 


Remainder  =  1. 476495  Log.  =    6. 169232 


Natural  number  = ^    .     1.472825  Log.  a»   6. 168151 

Diff.  of  the  true  alt8.:ce*  4M7?Nat.V.S.=  .005609 


True  central  dist.  =118°35C   lrNat.V.S.=  1.478434 


Mbthod  IV. 
Of  reducing  the  apparent  to  the  true  central  Distance. 

RULB.  ' 

Take  the  logarithmic  difference  from  Table  XXIV.,  and  let  it  bo  cor-i 
racted  for  the  sun's,  star's,  or  planet's  apparent  altitude,  as  directed  in 
pages  49|  51,  and  52. 

Find  the  sum  of  the  apparent  altitudes  of  the  objects,  and,  also,  the  sum 
of  their  true  altitudes  3  then, 

From  the  natural  versed  vine  supplement  of  the  sum  of  the  apparent 
altitudes,  subtract  the  natural  versed  sine  of  the  apparent  distance;  to  the 
logarithm  of  the  remainder  let  the  logarithmic  difference  be  added,  and 
the  sum  (abating  10  in  the  index,)  will  be  the  logarithm  of  a  natural  num-i 
ber;  which,  being  subtracted  from  the  natural  versed  sine  supplement  of 
the  sum  of  the  true  altitudes,  will  leave  the  natural  versed  sine  of  the  true 
central  distance. 


Digitized  by 


Google 


440  NAUTICAL  ASTRONOMY. 


Example  1. 

Let  the  apparent  central  distance  between  the  moon  and  sun  be 
1 10?53^34'r,  the  sun's  apparent  altitude  38?1 1  '59?^  the  moon's  apparent 
altitude  15?51'22'r^  and  her  horizontal  parallax  58 MO'';  required  the 
true  central  distance  7 

Sun's  apparent  alt.=38?  1 1^59^  -  Cofrec.    1 '  IS^^-true  alt=38?10tS4r 
Moon's  appar.  alt.=:  15. 5 1 .  22    -^  Correc.  53.   7  =tnie  alt.==  16. 44. 29 

Sumoftheap.alts.=:54?  3^2K     Sum  of  the  true  altitudes  =  54?5S: 23 r 

Sumofap.aIt8.=54°  3 :2KNat.V.S.suj).=  1.586997 

Appandist.  =  110.53.34  Nat.  vers.  S.=  1. 356620  Log.diff.=^9. 998150 

Remainders:     .  230377  Log.  =     5.362439 

Natural  number  =  . 229398  Log.  =     5.360589 

Sum  of  true  alts.  54?55^23rNat.V.Su8up.=:l. 574676 

Truecent.di8,=  110?ir.56rNat. vers.  S.=  1.345278 

^o^e.— See  remarks^  page  434,  relative  to  the  corrections  of  the 
apparent  altitudes  of  the  objects. 

Example  2. 

Let  the  apparent  central  distance  between  the  moon  and  a  fixed  star  be 
4^11^7^,  the  star's  apparent  altitude  43?10'.20r,  the  moon's  apparent 
.altitude  56?48'16^,  and  her  horizontal  parallax  59'.25T;  required  the 
true  central  distance  ? 

Star's  apparent  alt.=43?  10^  20r-.  Correc.    1'.   ir=true  alt=439  9U9^ 
Moon's  appar.  alt.s:;56. 48. 16   +  Correc.  31, 56  strue  alt.=57. 20. 12 

Sumoftheapp.alt8.=99958'36^'    •Sum  of  the  true  altitudes=!00?29^3ir 

Sumoftheapp.alt8.=99?58'36r  N.V.S.8up.= 826753 
App.centraldist.^  41. 11.   7  N.  vers.  S.=2474 16  Log,diff.=9. 998895 

Remainder  =  579337  Log.  =     5.762931 

Natural  number  = .    571250  Log.  =     5.756826 

Sumoftruealt8.=100^29^3KNat.V.S.fiup.=^817902 


True  cent.  dist.  =  41°  7-  8^Nat,  vers.  S.=  246652 

/Google 


Digitized  by ' 


OP   FINDING  TM£  L0N6ITUDB  BV  LUNAR  OBSBRVATIONS*         441 

Method  V. 
Ofredudng  tlie  apparent  to  the  true  central  Distance. 

Rule. 

To  die  logarithmic  sines  of  the  sum  and  the  difference  of  half  the 
apparent  distance  and  half  the  difference  d(  the  apparent  altitudes^  add  the 
logarithmic  differencie,  Table  XXI V.^  and  the  constant  logarithm  6. 301030: 
the  sum  of  these  four  logarithms  (rejecting  30  in  the  index,)  will  be  the 
logarithm  of  a  natural  number;  which,  being  added  to  the  natural  versed 
sine  of  the  difference  of  the  true  altitudes^  wiH  give  the  natural  versed  sine 
of  the  true  central  .distance; 

Example  1. 

Let  the  apparent  -distance  between  the  moon  and  a  fixed  star  be 
37?56'43^,  the  star's  apparent  altitude  19?32'^  th^  moon's  apparent  alti- 
tude 56?33',  and  her  horizontal  parallax  61 '  161}  required  the  true  cen- 
tral distance  ? 

Star's  apparent  alts  19?32:  Or-Corrcc.   2U0?=true  alt=19?29'20r 
Moon'sappar.alt.^  56.33.   0  -t-Correc.  33.   9  ^tmetiu^SJ.   6.   9 


Diff.oftheapp.alt8.=?37?  V.  Or    Diff.  of  the  true  altitudes^:  37^36^49? 


Half  diff.  of  ap.  alts.=:  18?30:30r 
Half  the  appar«  dist.=  18.58.21^ 

Log.  diff.  =      ....  9.993713 

Sum=     •    .    .    .    3r?28'51jr  Log.^sine  =       ....  9.784259 

Differences     ,    .      0.27.51^    Log.^sine  =       .    ,    •    .  7.908677 

Constant  log.  ±:     .    .    •  6.301030 

Natural  number  s=   «•.......«    9720  Log,s3. 987679 

Diff.of  thetruealts.=^37?36:49r  Nat.  vers.  S.=207855 


True  central  dist.  =s  38?31'.   ir  Nat.  vers.  S.=2 17575 

^Urample  2. 

Let  the  apparent  central  distance  between  the  moon  and  sun  be 
L06?22M8r^  the  sun'e  d^iparent  altitude  39'?25^^  the  moon's  apparent 
altitude  19?56^^  and  her  horizontal  parallax  58^0^;  required  the  true 
central  distance  ? 


Digitized  by 


Google 


442  VAxmcih  Awnmour. 

Sun's  apparent  alt.  =  39?25^  0^— Correc.    1^  3r=trucalt,=39?23:57^ 
MooD'sapparentaIt.=  19.56.   0  +Conrec.  51.56  =:truealt.=20.47.56 


Diff.  of  the  app.  alu.=s:  19?29'  Or    Diff,  of  the  true  altitudes^^lS^SG^   i  '. 

Half  diff.  of  app;  alt8.=  9944C30r 
Half  the  app.  di8t«  =  53.11.24 

Liog,diflF.  =;      .    •  ,    •    9,997672 

Sums    .    .    .    .    62?55^541f    I^.aines^      ^    .  .    ,    9.949616 

Differences:     .    .    43.26,54      Log. sine  =5      .    ,  •    ,    9/837399 

Constant  log.  =:    .  .    ,    6,30103U 

Natural  number  =s •     1. 218196  Log.=6^ 085717 

Diff.  of  the  true  alts.=  18?36' '  1  ^Nat.  vers.  S.=  052234 


True  central  dist.=105?41^24rNat.ver8.S.=  l.  270430 

Mbthod  VI. 
Of  reducing  the  apparent  to  the  true  central  Distance. 

Rule. 

To  the  logarithmio  cocaines  of  tho  sum  and  the  diffarenca  of  half  the 
apparent  distance  and  half  the  sum  of  the  apparent  altitudes^  add  the 
logarithmie  diffnence^  Table  XXIV.,  and  the  constant  logarithm  6.  SOI  090: 
the  sum  of  these  fouir  logarithms  (rejecting  30  in  the  index,)  will  be  the 
logarithm  of  a  natural  number;  which,  being  subtracted  from  the  natural 
versed  sioe  supplement  of  the  sum  of  the  true  altitudes,  will  leave  the' 
natural  versed  sine  of  the  true  distance. 

.  Example  1. 

Let  the  apparent  central  distance  between  the  moon  and  a  fixed  star  be 
69?21'.25r,  the  star's  apparent  altitude  27tS2{S7?,  the  moon's  apparent 
altitude  22?28'56r,  and  her  horizontal  parallax  56 M 7^;  required  the 
true  central  distance  ?  • 

Star's  apl)arent  alt.=;27?32:37?-Corwc.    1  U9r=true  alt.=27?30:48T 
Moon's  appar.  alt.=  22. 28. 56  +Correc.  49.43  =true  alt.=:23. 18.39 

Sttmoftheap.alts.=  50?  1^33?    Sum  of  the  tfue4ltitudes^50?49C27r 


Half8umofap.alt8,=25?  0M6ir 

/Google 


Digitized  by ' 


OF  FINDING  THB  liONQITUDB  BT  LUNAR  OBSERVATIONS.         443 

U«lf9iimofap<alt8.c:a5?  0'A^1 
Halfiip.coiit.di8t,:;:  34.40.42^  . 

Log.  diflF.  =:       •  •  .    ,  9. 997468 

Sum=.    .    ,    ,    59M1C291:     Log.  co-sine  =    ,  ,  .    ,  9.702997 

PiflFerence  =    ^    .      9.39.56      Log.co-6ine=    ,  .  ,.,  9.993791 

Constant  log.  =  .  ,    .  6. 301030 


Natural  number  z=: .    ,    .    989204  Log.=:5^995286 

Sumof  truealte.=50?49',27*  Nat.V,S.iup,=:l.  631703 

True  cent.  di8t.=69?  3'  1 1  jrNat.  vers.  S.  =    .  642499 

Example  2. 

Let  the  apparent  central  distance  between  the  moon  and  Jupiter  be 
116'?40'.28^,  Jupiter's  apparent  altitude.  10^40' 20r,  and  his  horisontal 
parallax  2*!,  the  moon's  apparent  altitude  15?10'30'!f,  and  her  horizontal 
parallax  59^  13? ;  required  the  true  central  distance  7 

Jupitey's  appar.  aU.=  10?40'.20':-Correc.   4:54'/fctnie  alt,=  10.?35^26r 
Moon's  appar.  alt.  =  15. 10. 30  +Correc.  53. 41   =true  alt.^^16.   4. 1 1 


Sum  c^theap.  alts,  s  2^5?50'.50r  Sum  of  the  true  altitudes  sc  26?S9f37r 

Halfsum  of  ap.  alts.=12?55:25? 
Half  app.  ccfnt.  dist.s  58. 20. 14 

— i».*~^      Logidiff.  « 9.998220 

Sum  =    .    .    ..    71^15^39?  Log.  cQ-sine  =i  .     .    •    •  9.506857 

Difference  ='    .     .     45.  24. 49  Log.  co-sine  ?=....  9. 846327 

Constant  log.  »     ....  6.301030 


Natural  number  s     ^    ......    .    «    449194  Log.=5. 652434 

Sum  of  true  alts. =26?  39: 37^  Nat.V.  Si  sup.ss  1 .  893683 

True  cent.  dist=;  U6?23 '  26?  Nat.  vers.  S.  s=  1 .  444489 


Method  VIL 
Cf  reducing  the  apparent  io  the  true  central  Distancei 

JlULB. 

To  the  apparent  central  distance  add. the  apparent  altitudes  of  the 
cribjects^  and  take  half  the  ^um;  the  difference  between  which  and  the 
apparent  distance>  call  the  remainder  ^  then. 


Digitized  by 


Google 


444  KAtJTicAL  ASTRoyomr. 

To  the  logarithmic  difference^  Table  XXIV.,  add  the  logarithmic  co-aines 
of  the  above  half  sum  and  remainder :  the  amn  of  these  three  logarithms 
(rejecting  20  in  the  index,)  will  be  the  logarithm  of  a  natural  number. 
Now,  twice  this  natural  number  being  subtracted  from  the  natural  versed 
sine  supplement  of  the  sum  of  the  true  altitudes,  will  leave  the  natural 
versed  sine  of  the  true  central  distance. 

Smtmrks, — If  the  remaining  index  of  the  three  logarithms  (after  20  is 
rejected)  be  9,  the  natural  number  is  to  be  taken  out  to.  six  places  of 
figures;  if  8,  to  five  places  of  figures ;  if  7^  to  four  places  of  figures ;  if  6, 
to  three  places  of  figure8,^and  so  on. 

The  logarithmic  difference  is  -  to  be  corrected  for  the  sun's,  star's,  or 
planet's  apparent  altitude,  as  directed  in  pages  49,  51,  and  52  ;*-this,  it  is 
presumed,  need  not  l>e  again  repeated. 

Example  1. 

« 

Let  the  apparent  distance  between  the  moon   and  a  fixed,  star  be 

48?20'2K,  the  star's  apparent  altitude  11  ?33' 29^,  the  moon's  apparent 

altitude  U?10'35?,  and  her  horizontal  parallax  55 '32^;  required  the 

true  central  distance  ? 

Star's  apparent  alt  =  ll?33^29r-Correc.  4^33^=:truealt.=;ll?28^56r 
Moon's  appar.  alt.  ss  11.10.35   -f  Correc.49.46  =truea]t.=3l2.   0.21 

Appar.  central  dist.=  48. 20. 21       Sum  pf  the  true  altitude»=23?29^  17^ 

Sum=    .     .     .     .     71?  4^25r 

Log.  diff.  =      ....  9. 998827 

Half  sum  =  .    .    .    35?32n2ir  Log.  co-sine  = .    .    ,     .  9.910487 

Remainder  es    .     .     12*48.   8|     Log.  co-sine  = .    .    .    .  9.989067 


Natural  number  = 791372  Log.=9. 898381 


Twice  the  natural  number  s;     •     •     .     .     .     1. 582744 
Sum  of  true  alts.=23?29M7^  Nat.V.  S.  sup.=  1 .  917143 


True  cent,  dist.=  48?  16^  17^  Nat  vers.  S.  =    .  334399 


Example  2. 

Let  the  apparent  distance  between  the  moon  and  sun  b6  108?42'  Sf, 
the  sun's  apparent  altitude  6? 28%  the  moon's  appairent  altitude  54?12r, 
and  her  horizontal  parallax  56 H9?;  required  the  true  central  distance? 


Digitized  by 


Google 


OF   FINDING  THB  LONGITUDE   BY  LUNAR  OBSSRVATIONS.         445 

Sun's  apparent  alt.  =  6?28'  Or-Correc.    7'.45r=:truealt=  6^20^5^ 
Moon's apparcntalt.=54, 12.  0  +Correc,3l.40  sstrue  alt.=54. 43. 40 

Appar.  central  dists  108. 42.   3      Sum  of  the  true  altitude8=6 1  ?  3'55T 

Sum=  .    .    .    .     169?22'.  3f 

Log.  difr.= 9.994507 

Half  sum  =  .   ,    .      84?4n   Ijr  Log.  co-sine  =  .    ...  8.966858 

Remainders  .    .      24.    1.    1^    Log. co-sine  =  •    •    •    .  9.960673 

Natural  i)umber  =  .    ..........    83568  Log.=8. 92203d 

u 

Twice  the  natural  number  =:     ......  167136 

Sumoftruealts.=  61?  3^55?  Nat.V.S.sup.=  l;483813 

True  cent  dist.=sl08?27M3r  Nat.  vers.  S.=  1.316677 


Method  VIIL 
Of  reducing  the  apparent  to  the  true  central  Distance.   . 

Rule. 

To  the  logarithmic  sines  of  the  sum  and  the  difference  of  half  the 
apparent  distance  and  half  the  difference  of  the  apparent  altitudes^  add  the 
logarithmic  difference :  half  the  sum  of  these  three  logarithms  (10  being 
previously  rejected  from  the  index,)  will  be  the  logarithmic  sine  of  an  arch. 
Now,  half  the  sum  of  the  logarithmic  co^sines  of  the  sum  and  the  differ- 
ence of  this  arch  and  half  the  difference  of  the  true  altitudes,  will  be  the 
logarithmic  co-sine  of  half  the  true  central  distance. 

Example  1. 

Let  the  apparent  central  distance  between  the  moon  and  a  fixed  star  be 
41?24'22^,  the  star's  apparent  altitude  12?  4 '27^,  the  moon's  apparent 
altitude  7^47^47^,  and  her  horizontal  parallax  57'24f  ;  required  the  true 
central  distance  ?  ^  ' 

Star's  apparent  alt.=  .12?  4'27?-Correc.   4122^s=true  ait.=12?  O'.SI 
Moon's  appar.  alt.  =    7. 47. 47  +Correc.  50. 13  strue  alt.=  8. 38. 0 


Diff.  of  the  app.  alt8.=  4  ?  1 6  f  40^     Diff.  of  the  true  altitudes  =  3?  22  ^  5  ^ 


Half  diff.  of  app.  alt8.=  2?  8  C20r    Half  diff.  of  the  trtte  alts.=  1  ?4 1 C  2jr 

/Google 


Digitized  by ' 


446  HAUTICAL  A8TR01C0MT. 

Halfdiff.ofap.altt.«   2?  8;20r 

Half  theBp.cent.di8.s20. 42. 1 1 

Log.  diff.  =     ...    .    9.999201 

Sams     ,    .    .    •    22. 50.  SI      Log.  sine  s     ...    .    .    9.589045 
Differences      .    .     18.33.51       Log.  sine  s     «...    9.502927 

Sum  =19.091173 

Arch=    ....    20?33!'44|f  Log.  sines     ....    9.545586} 
Half  diff.  of  true  alts.s  1.41.   2| 

Bums     ....    22?14U7?     Log.  co-sines     .    .    .    9.966406 
Differences     .    .     18.52.42      Log.  co-sine  s     .    .    .    9.975987 

Sums  19.  942393 

Half  the  true  dist.  s  20?38^  l5r     Log.  co-sine  s     .    .    .    9. 9711961 

True  central  dist.  =  4m6'30r 

Example  2. 

Let  the  apparent  central  distance  between  the  moon  and  Satmn  be 
110?l4'34r,  Saturn's  apparent  altitude  9?40f48r^  and  his  horizontal 
parallax  1%  the  moon's  apparent  altitude  15^40^6?^  and  her  horizontal 
parallax  58'.43T.;  required  the  true  central  distance  ? 

Saturn's  apparent  alt.as9?40:48r-Correc.   5'24rstrue  aIt.s9?35C24r 
Moon's  apparentalt.s  15. 40.    6  +Correc.53.  Ilstruealtsl6.33. 17 

Diff.  of  the  app.  alts.s  5  ^59 '  1 8r     Diff.  of  the  true  altitude8s6?57 '  53r 

Half  di£of app.  alu.s^?59^39r  Half  diff.  of  the  true  alto.=:8e28C56|r 
Half  app.  cent  dist.s55,   7«  17 

Log.  diff.  s.    ....    9.998176 

Sums  ....    58?  6(S61    Log.  sine  s 9.928966 

Differences     .    .    52.   7.38      Log.  sine  = 9.897284 

19.824426 

Aiehs      .    .    .    54?47'  21     Log.  sine  s   .....    9.91221S 
Half  diff.of  true alts.«3.  28. 56i 

Sums        .    .     .    58?  15 :58ir  Log.  co-sine  s    ....    9.720963 
Differences    .    .    51. 18.   5^    Log.  co-sine  s    ....    9.796035 

§um=  19.516998 

Half  the  truedist  s,  55?  0'30jr  Log.  co-sines    ....    9.758499 

True  centra! 'dist.=  nor  IM* 


Digitized  by 


Google 


OF  FINDING  THB  LONGItUDB  BY  LUNAR  OBSSRVATIONS.         447 

Method  IX. 
Cff  reducing  the  apparent  to  the  true  central  Distance, 

RULK. 

To  the  logarithmic  co-sines  of  the  sum  and  the  difference  of  half  the 
apparenl  distance  and  half  the  sum  of  the  apparent  altitudes^  add  the 
logwithmie  difference :  half  the  sum  of  these  three  logarithms  (10  Seiof 
previously  rejected  from  the  ihdex^)  will  be  the  logarithmic  ico-sine  of  a& 
acch.  No%  half  the  sum  of  the  logarithmic  sines  of  the  sum  and  differ- 
ence of  this  arch  and  half  the  sum  of  the  true  altitudes,  will  be  the  loga- 
rithmic sine  of  half  the  true  central  distance. 

Example  I.  - 

'  Let  the  apparent  central  distance  between  the  moon  and  a  fixed  star  be 
41?!!9'58r,  the  star's  apparent  altitude  ll?3l'2r,  the  moon's  apparent 
altitude  8?44<35T,  and  her  horizontal  parallax  57^247 ;  required  the  true 
central  distance  ? 

Star's  apparent alt.=rll?31(   27-Correc.    4^34r=truealt.=  ll?2jSf28: 
Moon's  appar.  alt.  a=  8.44.35  ;-f€k>rrec.50.46  struealt.as:  9.35.21 

Sumoftheapp.a]ts.=:20?  15  ^37^     Sum  of  the  true  altitudes^21  T  1  '49r 

Halfsumofap.alts.»10?  7U8^r  Half  sum  of  the^«ealts.=:10?30^54i? 

Half  ap.  cent  dist.^  20.  44. 59 

Log.  diflF.  = 9.  999083 

Sum  =       ...     30?52M7ir  Log.  co-sine  =    .....  9.933612 

Difference  ss    •    •     10. 37*  10}    Log!  co-sine  =   ....   '9.d92497 

19.925192 

Arch  a  .  .  .  23?26:23^  Log.  co-sine »  t  •  .  .  9.962596 
Halfsumoftr.alU.=s  10.30.541    .  . 

Sum.r::  .    .    .    .    33?57n7|^Log.  sitic  =   .....    9.747053 
Differences:    .'    .     12. 55. 28}  Log.  sine  = 9.349604 

.       ^  Sums  19.096657 

Halfthetraedist.a8  20?4lC54^r  Log.  sine  £=  .    .    .    .    .    9.548328| 

Truecentraldist.=  41?23'49r 

Example  2. 
JjBt  the    apparent    central   distance  between  the  moon  and  sun  be 
101?54:51?»  the  #un's  apparent  altitude  39?34^35r,  the  moon's  apparent 
altitude  29?23C2%  and  her  horiaontal  parallax  58^53^^ ;  requured  die  true 
central  distance  ? 

Digitized  by  VjOOQ IC 


448  NAUTICAJ.  ASTRONOMY. 

Sun's  apparent  alt.a  39?34'35r-Correc.    1^  3r=traealt.s39?33^32r 
Moon's  appar.  alt.  =  29.23.   2  -f  Correc.49.38  =truealt.=30. 12.40 


Somof  the  app.alts.s68?57'37"     Sum  of  the  true  aldtudes=  69?46'.  12^ 


Halfsumofap.alts.=:34?28'.48ir  Halfsumof  thetnie  alt8.=:  34?53^  6? 

Halfapp.cent.dist.=  50.57.25i  . 

.-.    ■   •  —  Log.diff.  =  .    ....    9.996517 

Snin=   .    .    .         85?26'14r  Log.  co-sine  =  .    .    .    .    8.900647 

Differences     .    .     16.28.37    Log.  co-sine  =  .    .    .    .    9-981789 


18.878953 


Areh=.    .    .    .    74?  1'57'    Log.  co^sine  =     .    .    .    9.439476^ 
Halfsumoftruealt8.=:34.53.   6        . 


Sum=   .    .    .    .  108?55^  3r    Log.  sine  s  \.    .    .    .    9.975885 
Differences    .    .    39.   8.51      Log. sine  =    '.    .    .    .    9.800249 


Sum=  19.776134 


Half  the  true  di8t.=  50?36(22r    Log.  sine  =     ....    9.888067 
True  central  dist.=:  101?  12M4r 

.  MEtHOD  X. 

*    Of  reducing  the  apparent  to  the  true  central  Distance. 

'   Rule. 

To  the  logarithmic  sines  of  the  sum  and  the  difference  of  half  the 
apparent  distance^  and  half  the  difference  of  the  apparent  altitudes^  add 
the  lit^arithmic  difference^  its  index  being  increased  by  10 :  from  half  the 
sum  of  these  three  logarithms  subtract  the  logarithmic  sine  of  half  the 
difference  of  the  true  akitudes,  and  the  remainder  will  be  the  logarithmic 
tangent  of  an  arch ;  the  logarithmic  sine  of  which,  being  subtracted  from 
the  half  sum  of  the  three  logarithms,  will  leave  the  logarithmic  sine  of 
half  the  true  central  distance. 

Example  1. 

Let  the  apparent  central  distance  between  the  moon  and  b  fixed  star  be 
55^4'.531f  the  star's  apparent  altitude  10?8'.6f,  the  moon's  apparent 
altitude  8?  1^25^,  and  her  horizoatal  parallax  58^?;  required  the  mte 
central  distance  ? 


Digitized  by 


Google 


or  riNDINO  THB  LON6ITUDB  B7  LTTNAR  OBSERVATIONS.        449 

Star's  Bpparenfalt=  10?  8'  6r-Correc.   5ni7=trueitlt.s=10?  2.55? 
Moon's  apparoit  alt.s=8.    1.25  +CoiTec.  50. 58  =:truealt.=  8.52.23 

Diff.oftheapp.aIts.=2?  6'4ir    Diff.  of  the  true  altitudes  =  1°10'327 

Halfdiff.ofapp.alts.sl?  3t20^t  Half  diff.  of  true  aldtudes  =  0?35n6^ 
Halftheap.cent.dis.=27. 32. 26i 

•Log.diff.sa  19. 999162 

Sums.    ...    28?35'.47?LQg.8iiie=>  9.680006 
Difference  a    .    .    26.29.  6  Log.8ine=*9. 649299 

Sum= 39:328467 


Half  sum  3  .    . 19.664233^     .     19.664233| 

Half  diff.  of  true  alts.=0?35n6rLog.Bme=  8. 011083 

ArchB  .     ....    88?43^367Log.tan.sll.653150iLog.d.9. 999893 


Half  the  true  distances    •    .    ..  .    27?29M4r  Log.  8ine=9.664340i 

True  central  distance  s    .    .    .    .    54?59'28; 

ExanqtU  2. 
Let  the  apparent  central  distance   between  the  moon  -  and  sun  be 
91?26'8',  the  sun's  apparent  altitude  WHS'AMy  the  moon's  apparent 
altitude  53?41  H 7,  and  her  horizontal  parallax  58C 297 ',  required  the  true 
central  distance  ? 

Sun's  ^parent  alt.=  U?45M17-Correc.    3:267strue  alt.=  14?42'157 
Moon's  appar.alt.=  53.41.    1   -{-Correc.  33. 56  =true  alt=54. 14)57 

Diff.of  the  ap.  alts.=38?55^2p7    Diff.  of  the  true  alts.  =s  .    39?32'.427 

Half  diff.of  ap.aite.= 19?27 M07    Half  diff.  of  thefpie  alts.=  19?46'. 217 
Halfap.  Cent. (Ust.=  45.43.  4 

Log.diff.= 19. 994220 

Sums  ....    65:?lO<447Log.sines 9. 957905 
Differences   .    .    26^15.24  Logcsine  =  9. 645809 

Sum  s 39.597934 


Half8nm=s    .........     19.798967    .    .    19.798967 

Half  diff.oftrue  «lto.l9?46^217Log.sine=  9.529285 

Archs      .    .    ,    61?44^48?Log.tanU=  10. 269682  Log. sine 9. 944902 

Halflhe  true  distances  •,    .    .    .    45?36:38i7  Log,  8ine=9. 854065 


True  central  distance  a     .'v   .    .    91?13''17' 

2  o 


Digitized  by 


Google 


450  HAimCAL  ASTRONOMT* 

Method  XI. 
Of  reducing  the  apparent  to  the  true  cmUral  Distance. 

Rule. 

To  the  logarithmic  difference  (its  index  heing  {ncreaaed  by  10,)  add  the 
logarithmic  co-sines  of  the  stmi  and  the  difference  of  half  the  apparent 
distance  and  half  the  sum  of  the  apparent  altitudes ;  from  half  the  sum  of 
these  three  logarithms  subtract  the  logarithmic  co^sine  of  half  the  sum  of 
the  true  altitudes,  and  the  remainder  will'  be  the  logarithmic  sine  of  an 
arch ;  the  logarithmic  tangent  of  which^  being  subtracted  from  the  half 
sum  of  tlie  diree  logarithms^  will  .leave  the  logarithmic  sine  of  half  the 
tnie  central  distance^ 

Example  I. 

Let  the  apparent  central  distance  between  the  moon  and  a  fixed  star 
be  68?5i2M0r,  the  starts  apparent  altitude  10?52U7^  the  moon's 
apparent  altitude  6*39^28^,  and  her  horizontal  parallax  58<3ir}  required 
&e  true  central  distance,? 

Star's  apparent  a]t.=10?52<  17^-Correc.    4^507=  true  ait=10?47^27^ 
Moon's  appar.  alt.«  6. 89. 26  +Correc.  50. 26  a  true  alt.s  7. 29. 54 

Sum  of  the  ap.  alts.=  1 7?d  1 '  45  r    Sum  of  the  true  dtitndes  =  1 8?  1 7 '  2 1  r 


HaIfsumofap.alts.=:  8?45^52ir  Half  sum  of  the  true  alts.  =:  9?8M0ir 
Half  ap.  cent.  dist.  =  in.  26. 20 

Log.diff;  19. 999826 

Sam=  .    .    .    .    48?12a2irLog.co-si.9. 862684 

Difference  =    •     •    25. 40. 27^'  Log.co-si.9. 954856 

■III    I    1^ 

Sum=  . 39.816866 


Half  mm  3= 19.908433    .    .     19.908433 

Half8umoftruealts.s9?  8'40irLog.co-n.9.994445 


Ai«h«  ....    S5T  7'.  4'.  ho^jiae»9,9l»998hag.T.=:l0.\i6679 
Half  the  tniedutances   ....    34?22^34r  Lo^.tkio  :s9,7S1758 

TVue central  distance  ts     ,■  ',    .    .    68?45(  87 

Digitized  by  VjOOQ  IC 


OF  FINDING  THB  LOM6ITUDB   BY  LUNAR  OBSBRVATIONS.  451 

Example  2. 

Let  the  apparent  central  distance  between  the  moon  and  sun  be 
120?I0'4(K,  the  sun's  apparent  altitude  13?30^0r,  the  moon's  apparent 
altitude  6?10;0r,  and  her  horizontal  parallax  61(12^5  required  the  true 
central  distance  ? 

Sun's  apparent  alt,c:ql3?30'  Or-Cofrec.    8M5?==  true  alt,«13?26I15f 
Moon's  appar.  alt,??  6.10.   0  +Correc.  52.36  =  true  alt.=  7.    2.36 

Sumoftheap,alts.=:19?40C  0?    Sum  of  the  true  altitudes  «  20?28'64r 

Halfsum  of  ap,alts.=9?50^  0?     Half  spm  pf  the  true  ftlt8.=  109l4'25ir 
Half  ap,  cent.  dist.= 60.   5.20 

-. Log,  diff.a  19. 999345 

Sums.    .    ,    .    69?55  i20r  Log.  co-sine  9.535668 
Difference  =    .    .    50, 15. 20  Log.co-sine  9. 805749 

Sum^i. . /.    .39.340762 


H^lfsums    .    .........  19.670381     .      19,67088! 

Half8umoftrtieaIu.l0?l4^25ji:Log,cor6iiie  9.993026  • 

Arch==     .    .    .     28M4'.23r  Log.  sine  =  9.677355  Log.T.  9. 738070 

Half  the  true  distance  =      .    •    .    .     59? 56  r  59'r  Log.  sine=9: 9378 1 1 

True  centnd  distance  an  .    .    .  U9?53C58r 


AterHoo  Xljr    • 
Of  redwing  the  apparent  to  the  true  central  Distance. 

RuLB. 

From  the  natural  versed  sine  supplement  of  the  sum  of  the  apparent 
altitudes^  subtract  the  natural  versed  sine  of  their  difference^  and  call  the 
remainder  arch  first.  Proceed  in  a -similar  manner  with  the  true  altitudeb^ 
and  call  the  remainder  arch  second;  and  from  the  natural  versed  sine 
supplement  of  the  sum  of  the  apparent  altitudes,  subtract  the  natural  versed 
sine  of  the  apparent  distance,  and  call  the  remainder  aroA  tlArd, 

Now,  td  the  arithmetical  complement  of  the  logarithm  of  arch  first  add 
the  logarithms  of  arches  second  and  third,  and  the  sum  (rejecting  10  from 
the  index,)  will  be  the  logarithm  of  a  natural  number ;  which,  being  sub- 
tracted from  the  natural  versed  sine  supplement  of  the  sum  of  the  true 
altitudes,  will  leave  the  natural  versed  sine  of  the  true  central  distance. 

2a2 


Digitized  by 


Google 


452  '  NAUTICAL  ASTRONOMY. 

Example  1. 

Let  the  apparent  central  distance  between  the  moon  and  a  fixed  star  be 
83?  15 '19",  the  star's  apparent  altitude  7? 39' 4 r,  the  moon's  apparent 
altitude  10?57'36%  and  her  jiorizontal  parallax  58(53';  required  the 
true  central  distance  ? 

*'sap.alt.=  7?39'  4?-Cor.  6M5?=Truealt.  7^32119^ 
])'sap.«lt=10.57.36  +Cor.53.   3  <=iTruealt.ll.50.39 

Sum  =     .     ISeseUO'^iJ^Jl.  947707  Sum  =  I9?22(58r••;:i^}1..943322 
Dlff.  «    .      8.18.32N.V.S..001668Diff.  =    4. 18.20N.V^..002822 


Arch  first  =   1 .  946039  Areh  second  =  1 .  9405QO 

Sumofaplalt8.=sl8?36M0r  N.V,S. sup.  =  1. 947707 
Ap.  cent.  di8t.s=83?  15 a9r  Nat.  V.  S.  =      .  882554 


Arch  third  = 1. 065 153  Log.  s;     6.027432 

Arch  seconds 1. 940500 Lqg.  =     6.287914 

Arch  firsts     .........     1.946Q39Log.ar.co.3. 710848 

Natural  number  s     .    . 1. 0621^9  Log.  s    6i  026194 

Sum  of  true  alts.  19?22(58:  N.V.S.  sup.  =  L  943322 

TVue  cent.  dist.  83?10(28?  Nat. vers. sine-  .881153      . 

Example  2. 

Let  the  apparent  distance  between  the  moon  and  sun  be  II  1?27' If, 
the  sun's  apparent  altitude  24^40' 16?,  -the  moon's  apparent  aldtude 
16?52.3K,  and  her  horizontal  parallax  54'56r j  required  the  tone  central 
distance  ? 

©'sap.alt.=24?40n6r-Cor.  l'56r=Truealt.24?38(20r 
])'sap.alt.=16.62.31   +Cor.49.28  =TrueaIt.l7.41.59 


Sums   ,    41?32U7rY,f}l. 748419  Sum  =  42?20<19r'UV}l. 739177 
DiflF.  =    .      7.47.45N,V.S..009242  Diff.=    6. 56. 21N.V.S. . 007325 

Aroh  first  =  1. 739177  Arch  second  =  V.  731852 

Sumofap.alts.=4 1  ?32  U7?N.V.S.8up.=  1 .  7484 19 
App.  central  dist.  U 1  °  27^  ?  Nat.V.S.=  1 .  365694 

Arch  thirds        .......      .382725  Log.  s     .    5.582887 

Archseconds      ..-.■....    1.731852  Log.  =    .    €.238511 

Arch  firsts     ........    1.739177  Log.  ar.co.=3. 759638 

Natural  number  = 381097  Log.  s     .    5.581036 

Sumoftruealts.42?20(  19f  N.V.Sjup.sl.  739177 
True  central  dbt.  U0?58(56rN.V.S.5=l. 356080 

Digitized  by  VjOOQ  IC 


OF  FINDING  TUB  LONGITUDE  BT  LUNAR  OBSERVATIONS.         453 

Method  XIII. 

To  the  apparent  distance  add  the  apparent  altitudes  of  the  objects ;  take 
half  the  sum,  and  call  the  difference  between  it  and  the  apparent  distance, 
the  remainder.    Then, 

To  the  logarithmic  difference  (its  index  being  augmented  by  10,)  add  t^e 
logarithmic  co^sines  of  the  half  sum  and  the  remainder ;  from  half  the  sum 
of  these  jthree  logarithms  subtract  the  logarithmic  co-sine  of  half  the  sum 
of  thq  true  altitudes^  and  the  remainder  .will  be  the  logarithmic  sine  of  an 
arch.'  Now,  the  logarithmic  co-sine  of  this  arch,  being  added  to  the 
logarithmic  co-sineof  half -the  sum  of  the  true  altitudes  (rejecting  10  from 
the  index),  will  give  the  logarithmic  sine  of  half  the  true4:entral  distance* 

Example  !• 

Let  the  apparent  central*  distance  between  the  moon  and  Spica  Virginia 
be  37^1 2 UOr,  the  stat's  apparent  altitude  ll?27'50r,  the  moon's 
apparent  altitude  40^55 '.  15  T,  and  her  horizontal  parallax  54C'10? ;  required 
the  true  central  distance  ? 

Star's  apparent  alt.= 11? 27 '50!r^Correc.    4^35r=-:true  alt.=ll?23n5r 
Moon's  appar.alt.=40. 55. 15   -f-Correc,  39.51  =strue  alt,=41.35.   6 
Appar.  cent,  dist,  =  37. 12. 40 

Sum  =       .    .    .    89^35  USr 

— rLog.aiff.=r  19. 995703 

Half  sum  =:     .    ,    44?47 '  52i:Log.co-sin,  9. 85 1012 
Remainders      •      7.35. 12*  Log.co-sin.9.996\81 

Sum=     .     .    39.842896 


Half  sum  =       19.921448 
Halfsumoftruealts.26?29'.10irLog.co-sin.9.951844      •    .    9.951844 

Arch  s      .    .    .    68?48'.45r  Log.  sine=:9.969604Log.co-8i.9. 558014 

Half  the  true  disunce  :=^    .    •    .    •     18?5a|26ir  Log.  8ine=9. 509858 

True  central  distance  =3     .    .    .    •    37^44(53^1: 

Example  2. 

'  Let  the  apparent  central  distance  between  the  moop  and  sun  be 
U7?42'28^,  the  sun's  apparent  altitude  10?  19^  19^,  the  moon's  apparent 
altitude  42?55'.1%  and  her  horizontal  parallax  60C2Cj  required  the  true 
antral  distance? 


Digitized  by 


Google 


454  WAOTIGAL  AtTAONOMY. 

Sun'Bapparentalt=10?19a9r-Correc.    4^56r=  true  alt.=10?14^23^ 
Moon's  app.  alt.  =  42.55.    1   +Currec.  42. 57  =  true  alt=43.37.S8 
Appar.cent.dist.=:117.42. 28    . 

Stttns    •    .    •     170?56^48r 

^ — Log.diff.=  19. 995005 

Half  turn  »  .    .      85^28124rLog.co-Mii.  8. 897204 
Remainder  z=     •      32*  14*  4  Logxo-sin*9.927305 


Suin'=:    38.819514 


Half  sum  19.409757 
Half8umoftruealt8.26?S6nOi*Log.co.8.=9. 950127  .    •    .    9.950127 

Arch=     .    •    .     16?44^52^  Log.  8me=9.459630Log.co.8i.8.981l77 


Half  the  true  distance  s     ....    58?36'58r  Log.  8ine=9. 931304 

True  central  Stance  — 117?13'.56.r 

Note. — There  are  some  curious  properties  peculiar  to  the  limar  obsenra^^ 
tions,  with  which  the  mariner  ought  to  be  acquainted^  but  which  the 
general  tenor  of  this  work,  will  not  allow  of  being  touched  upon  here  :— 
these  properties  or  peculiarities  may,  however,  be  readily  seen,  by  making 
reference  to  the  General  Remarks  contained  between  pages  208  and  212 
of  '^  The  Young. Navigator's  Guide  to  the  Sidereal  and  Planetary  Parts  of 
Nautical  Astronomy." 


Probum  VIIL 

Given  the  apparent  Thne  and  the  true  central  Distance  between  the  Moon 
and  Sun,  a  fixed  Siar,  or  a  Planet;  to  determine  the  Longitude  qf  the 
Place  of  Observation. 

Rule.        . 

If  the  true  central  dbtance  can  be  found  in  the  Nautical  Almanac,  the 
corresponding  apparent  timflr  at  Greenwich  will  be  seeii  standing  drer  it 
at  the  top  of  the  page ;  bat  if  the  true  central  distance  cannot  be  exactly 
found,  which  in  general  will  bt  the  case,  take  o^t  the  tiiro  distaaoee  firom 
the  Nautical  Almanac,  one  of  which  is  next  greater  and  the  other  next  less 
than  the  true  central  distance,  And  find  their  difference ;  find,  also,  the 
difference  between  the  true  central  distance  and*  the' ^irf  of  .tiie'two 
distances  so  taken  from  the  Nautical  Almanac ;  then,  froip  the  proportional 
logarithm  of  this  difference,  subtract  the  proportional  logarithm  of  the. 
former  difference,  and  the  remainder  will  be  the  proportional  lcs;aritlun  of 


Digitized  by 


Google 


OF  FINDING  THB  LONGITUDB  BY  LUNAR  OBS£RVATION(S.         455 

B  portion  of  time,  which,  being  added  to  the  time  corresponding  to  the 
first  of  the  two  distances  taken  from  the  Nautical  Almanac,  will  give  the 
apparent  time  of  observation  at  Greenwich*  Now,  the  differeniee  between 
the  apparent  time  at  Greenwich,  thus,  found,  and  the  apparent  time  at  the 
place  of  observation^  being  turned  into  degrees,  will  be  the  longitud^of 
the  latter  place ;— and  which  will  be  east,  if  the  time  at  the  ship  be  greater 
th^p  that  at  Greenwich ;  if  otherwise,  west. 

Example  I. 

At  sea,  January  9th,  1825,  in  longitude  (by  account)  54?48^.  east,  at 
23t40?47'  apparent  time,  the  true  central  distance  between  the  moon 
and  sun  was  107^  19 '56^;  required  the  corresponding  apparent  time  at 
Greenwich,  and  the  longitude  of  the  place  of  observation  ? 

Traeccntdtst.atship=:107?l9^56^{  Djff^io  7/32*;  Prop.  log.=:4258 

Distance  at  18  hours  =  108. 27, 28    > 

Distance  at  21  hours  =  1Q6. 48. 12    j  Diff.=:  1. 39. 16    Prop.  log.=:2585 

Portion  of  time  =      .......      2*2^27*.     Prop,  log.zi  1673 

lime  corresponding  to  first  distance  =:    »    18.   0.  0 

Apparent  time  of  observ.  at  Greenwich  =  20 1  2T27' 
Apparent  time  oJP  observation  at  ship  =     23. 40. 47 


Longitude  of  the  ship,  in  time  =  .    .    .      3*38r20*  =54^85  (  east. 

Example  2. 

At  sea,  MaiehSd,  1825,  in  longitude  (by  account)  47?55^  west^  at 
10M2T43!  apparent  time,  the  true  central  distance  between  the  moon  and 
Spica  Virginis,  was  50?3^2S^;  required  the  corresponding  apparent  time 
at  Greenwich,  and  the  longitude  of  the  place  of  observation  ? 

True  central  disUince=50?  3 '  23?  I  l)iff.=0?53 '  20r    Prop.  log.  =^5288 

Distance  at  12  bourse  50. 56. 43   <   . 

Di8tanceatl5houn=49.   2.48  jDiff.=  1.54.   0      Prop.  log.  =  1984 

Portion  of  time  =    .• l*24rlS!     Prop.  log.  =  3299 

Time  corresponding  to  first  distance  =     12.  0.   0 

Apparent  time  of  observ.  at  Greenwich  :=  lS^24ri3' 

Apparent  time  of  obaervation  at  ship  =     10.12.43  *     . 

Longilttde  of  the  ship,  in  time  =    .    .  ^  3!  1  lT30;;=47?52130r  west. 

Digitized  by  VjOOQ IC 


^56  NAUTICAL  A8TR0K0MY. 

Problbm  IX. 

Given  the  LatUude  of  a  Place  and  Us  Longitude  by  accdtmt^  the  observed 
Distance  between  the  Moon  and  Sun,  a  fixed  Star,  or  a  Planet,  and 
i^  observed  JUiiudes  of  those  Objects  ^  to  find  the  true  Longitude  qf 
the  Place  of  Observation. 

Rule. 

Reduce  the  apparent  time  of  observation  to  the  meridian  of  Greenwich, 
by  Problem  III.,  page  297 ;  to  this  time  let  the  moon's  horizontal  parallax 
and  semi-diameter  be  reduced,  by  Problem  VI.,  page  302,  and  let  the 
moon's  reduced  semi-diameter  be  increased  by  the  augmentation  (Table 
IV.)  answering  to  her  observed  altitude.  * 

Find  the  apparent  and  the  true  altitude  of  each  object's  centre,  by  the 
respective  problems,  for  that  purpose,  contained  between  pages320and  327* 

To  the  observed  distance  between  the  nearest  limbs  of  the  moon  and 
sun,  corrected  for  index  error,  if  any,  add  their  respective  semi-diameters^ 
and  the  sum  will  be  the  apparent  central  distance.  But,  if  the  distance 
be  observed  between  the  moon  and  a  fixed  star  or  planet,  then  the  moon's 
true  semi-diameter  is  to  be  applied  to  that  distance  by  addition  when  it  is 
measured  from  the  nearest  limb,  but  by  subtraction,  when  it  is  measured 
from  the  remote  limb :  in  either  case,  the  result  will  be  the  apparent 
central  distance.  With  the  apparent  and  the  true  altitudes  of  the  objects, 
and  their  apparent  central  distance,  compute  the  true  centra]  distance^  by 
any  of  the  methods  given  in  Problem  VII.,  between  pages  433  and  454  ; 
and  find  the  apparent  time  at  Greenwich  corresponding  to  this  distance, 
by  Problem  VIII.,  page  456. 

Now,  the  difference  between  the  apparent  times  of  observation  at  the 
ship  and  at  Greenwich,  being  converted  into  degrees,  will  be  the  longitude 
of  the  place  of  observation ;  which  will  be  east  or  west>  according  as  the 
time  at  the  ship  is  greater  or  less  than  the  Greenwich  time* 

Remarks* 

It  the  watch  be  not  well  regulated  to  the  time  of  observation,  the 
apparent  time  may  be  deduced  from  the  true  altitude  of  the  sun,  moon, 
star,  or  planet,  used  in  the  computation,  provided  the  object  made  choice 
of  for  this  purpose  be  sufficiently  far  from  the  meridian  at  the  time  of 
measuring  the  lunar  distances;  if  not,  the  error  of  the  watch  must  be 
inferred  from  the  true  altitude  of  one  of  those  objects^  when  in  a  more 
favourable  position  with  respect  to  the  meridian :  then  the  error  .of  the 
watch,  thus  found,  being  applied  to  the  mean  time  of  measuring  the  lunar 
distances,  by  addition  or  subtraction,  according  as  it  is  slow  or  fast,  the 


Digitized  by 


Google 


OF  FINDING  THR  LONGmiDB  BY  LCNA&  OBSBRVATIONS.        4&7 

sum  or  difference  will  be  the  apparent  time  of  taking  the  lunar  observa« 
tion,  agreeably  to  the  meridian  under  which  the  error  of  the  watch  was 
obtained.  The  error  of  the  watch  is  to  be  found  by  Problems  III.,  IV.^  V., 
or  VL,  between  pages  383  and  400,  according  as  the  object  may  be  the 
sun,  a  fixed  star,  a  planet,  or  the  moon. 

In  taking  a  lunar  observation,  it  is  necessary  that  several  distances  be 
meaauredj — that  the  corresponding  times,  per  watch,  be  carefully  noted 
down, — and  that  the  altitudes  of  the  objects  be  observed  at  the  same 
instants  with  the  distances :  then,  the  respective  fiujns  of  the  times  (per 
watch)  of  the  observed  distances  and  of  the  altitudes,  being  divided  by  their 
common  number^  will  give  the  mean  time  of  observation,  the  mean 
observed  distance,  and  the  mean  observed  altitude  of  each  object* 

Example  L       . 

January  9th>  1825,  in  latitude  19?30'.  N;,  and  longitude  5?45^  E.^  by 
account,  the  following  observations  were  taken ;  the  index  error  of  the 
sextant  by  which  the  distances  were  measured  Was  2^S0lf  subtractive,  and 
the  height  of  the  eye  above  the  level  of  the  sea  20  feet ;  required  the 
longitude  of  the  place  of  observation  ? 


Apparent  time  of 
observfttion. 


Obsenred  distance  between 

nearest  limbs 

of.  Mooh  and  Sun. 


Altitude  of  Sun's 
lower  limb. 


Altitude  of  Moon's 
lower  limb. 


19* 10-45 • 

•  11.  50 

•  12.  55 

•  14.    0 
.15.    5 


1070  48'  30'' 
.  47.  50 
.  47.  15 
.  46.  40 
.  46.    0 


70  9'  0* 
7.22.  30 
7.36.  0 
7.49.30 
8.    3.    0 


530  13'  30" 
53.  2.  0 
52.  50.  30 
52.  39.  0 
52.  27.  20 


Mean..».  19M2-55' 

Longitude  r     23.   ^ 
in  time  f     ^*  ^ 


Red.  time  18M9-55« 


Mean 10r»  47'  15" 

Index  error..  —  2.  30- 
D  'a  semi-diam.  +16.  25  ■ 
Q'i  semi-diam.  +16.  18 


Metti..7^36'    0"lMean  52°50' 28'' 


Moon's  semi-tdiameter         +16.  25 
Dip  of  the  horizon  •  •  —4.17 


Appar.di8t,..108<'  17'  28" 


Moon's  appar.  altitude      53°  2' 36" 
Correction  of  ditto. ...        +35.    1 


Moon's  true  altiiude,  •     53®  37'  37" 


Observed  altitude  of  sun's  lower  limb  =     7^361  01 
Sun's  semi-diameter  =    .     .     •    •    •    »    •+16.18 
Dip  of  the  horizon  =      ..,.••       —  4. 17 

Sun's  apparent  altitude  =    ..••••      7?48!   K 
Gorrection  of  ditto  «=     •    •    •    •    .    »       —  6.29 


Sun's  true  altitude  == 


7Mr.32: 


Digitized  by 


Google 


458 


KAtrnCAL  ASTRONOMY. 


Moon's  rednoed  horizontal  parallax  » 

Diff.  of  the  app.  alts.=45  ?  1 4  ^  35 'r 
Auxiliary  angle  =  .     60.  26. 25 
App.  central  dist.  =.  108. 17. 28 


59C25? 


Sum  of  aux.  ang.  and 

difl:oNpp.alts.^l05?4K  0^ 
Difference  of  dittos  15.11.50 
Samaax.ang.&ap.di8J68. 4S.  53 
Differenoeofdittoa47.5.1.   3 
Diff.  of  tnie  alts«  s     45. 56.   5 


Nat.  versed  sine  sup*  a= 
Nat.  versed  sine  sup.  ^ 
Natural  versed  sine  s  . 
Natural  versed  sine  =  . 
Natural  versed  sine  s=  .. 


Natural  versed  sine  = 


True  central  di8t=107^59^32r  ) 

JDi 


.729680 

1.965029 

1.980722 

.328937 

.3045122 

1. 308890 


Dist  jai  1 8  hours  =  108. 27.  28    ]  Diff.=0?  27  i  56r       Prop.  log.  =  8091 
Dist,  at21  hours  =106.48. 12    }  Diff.=  L39. 16        PropJc^.  =  2585 


Portion  of  time  = •    .      Ot50?39!  Prop.  log.  =  5506 

Hme  corresponding  to  first  distance  =       18.    0.    0 


Apparent  time  of  observ.  at  Greenwich  =   1 8  tSOTSS ' 
Apparent  time  at  the  place  of  observ.  =;;     19. 12.  55 


Longitude  at  the  place  of  obs.,  in  time  a     Ot22ri6!  =  5?34'  east. 

Example  2. 

February  1st,  1825,  in  latitude  45?40'.N.,  and  longitude  59?10^W.,  by 
account,  the  following  observations  were  taken ;  the  height  of  the  eye  above 
the  level  of  the  horizon  was  22  feet,  and  the  index  error  of  the  sexftmt  by 
which  the.  distances  were  measured  K30?  additive  j  required  the  true 
longitude? 


Apparent  time  of 
obserration. 


Obgerved  dUtance  of 
Moon'g  remote  limb. 


Altitucle  of 
Reguktt. 


Altitude  of  Moon's 
lower  limb. 


8*50«10» 
.51.  25 
.  52.  40 
.  53.  55 
.  55.  10 


350 


7' 40" 
6.  50 
6.  10 
5.  20 
4.  40 


28^50'  50" 
29.  4.  30 
29.  18.  20 
29.  32.  Q 
29.  45.  50 


Mei^n....     S*52«40« 


Mean 35o    6^    8' 

Index  error. .        +.  1.  30 
]) 'b  semi-diam.   —16.26 


Mean  29o  IS'  18' 


10«44'  40^ 
11.40.    0 

12.  35.  20 

13.  30.  50 

14.  26.  10 

Mean  12o35'  24" 


Red.  time   12H^»20» 


Apparent  dist.  34'?  51'  12^' 


Moou'bsemi^ameter*.      4-16.  26 
Dip  of  the  horizon  ....     —  4.  30 

Moon's  appar.  altitude..  12^  4r  20* 
Correction  of  ditto...  ..     •h54.  27 


Moon's  true  altitude.,..  13«  41'  47" 


Digitized  by 


Google 


OF  FINDING  THB  J^MGITUDB  BY   LUNAR  OBSERVATIONS. 


4S9 


Obsenred  altitude  of  B^guluii  = 
Dip  of  the  horisQH  for  22  feet  ^ 

•  Apparent  altityde  of  Regulut  == 
Correction  of  ditto  =      .     •     • 


True  altitude  of  Regulus  =: 


29? 18' 18? 

-  4.30 

29?  ISMS'? 

-  1.41 

29?12'.  7" 


Moon's  reduced  horizontal  parallax  = 


60^3: 


Sumoftheapp.ftlt8:=42?  V.  61 
Auxiliary  angle  ==      fiO.   6.49    ' 
App.  central  dist.  =  34.51. 12 

Sum  of  aux,  ang.  and 

sum  of  app.  dts.=  1029  7^57?  Natural  versed  sine  rs 

Difference  of  do.  =     18.   5.41  Natural  versed  sine  = 

S4UBaiii|4uig:&ap.dist.94.58.    1  Natural  versed  sine  = 

Difference  of  do.  =r    25. 15. 37  Natural  versed  sine  = 


1.210173 
.049456 

1.086581 
.095622 


Sum  of  the  true  alti«s:42. 53. 54      Natural  versed  sine  sup.  =  1 .  732563 

Natural  versed  sine  =     .       .  174395 

True  central  dist.  =  34?2r.  0?  ) : — 

Dist.atl2hours  = -34.51. 16  j   Diff.  =  6?30n6r     Prop.  log.  =  7743 
Di»t..at  15  hours  =  33.   2.35    }  Diff.=  1.48.41       Prop.  log.  =  2191 


Portion  of  time  r=     ••...•     . 
Hme  corresponding  to  first  distance  :r 


0*50r  8!Prop.iog.=5552 
12.   0.   0 


Apparent  time  of  observation  at  Greenwichz:  12^50?  8! 
Apparent  time  ait  the  place  of  observation  =    8. 52. 40 


Longitude  at  the  place  of  obsery.,  in  time  ;=    3^57^28!  ={9?22'01' west. 


EteampU  3.  •     • 

March  9th,  1825^  in  latitude  43n7'  S.,  and  longitude  57?55:  E.,  by 
account,  at  20M4T  per  watch,  not  regulated^  the  mean  of  several  observed 
distances  between  the  moon  and  sun  was  107?  28'  17'' ;  at  the  same  time 
the  mean  of  several  altitudes  of  the  sun's  lower  limb  was  26?39'40^,  that 
of  the  moon's  upper  limb  39?30'45^,  and  the  height  of  the  eye  above  the 
level  of  the  sea  19  feet ;  required  .the  longitude  of  the  place  of  observation  ? 


Digitized  by 


Google 


460 


NAiniCAL  ASTHONOMY. 


Time,  per  watch,  =     20*14r  0'. 
Long.57?55CE.=     -3.51.40 

Reduced  time  =     .     16t22r20' 


Alt  sun's  lower  limb=  26?39C40r 
Sun's  semi-diameter=;     +16..  8 
Dip  of  the  horizon  =     —  4. 1 1 

Sun's  apparent  alt.  =  26'?51f37r 
CorrecUonz:     .    .         —  1.44 


Sun's  true  alt.  =  .      26'>.49'.53'. 


Dist  of  nearest  limbs 

of  moon  and  sun=107?28'.  17? 
Sun's  semi-diameter  =    +16.   8 
Moon's  semi-diam.  =     +16.   0 


Apparent  distance  =  108*!  0'.25r 

Alt.  J 'slower  limb  =  39?30'45r 
Moon's  8emi-diameter= +16.   0 
i)ip  of  the  horizon  ==     —  4. 1 1 

Moon's  apparent  alt.:=39?42^34? 
Correction  =     .    .       +43.33 


Moon's  true  altitudes  40?26t  7? 


Moon's  reduced  horizontal  parallax  =    58'47 

App.  cent.  dist.=  l08?  0<25rN.V.  S.=:l. 309132 

Diflf.  of  ap.  alts.  =  12. 50. 57  N.V.  S.=  .  025041  Log.  diff.  =  9. 995483 

Remainders  1.284091.   Log.s  .    6.108596 

Natural  number  = 1.270809    Log.=  .    6.104081 

DifiF.oftruealts.=  13°36n4?N.V.S.=  .028054 

Truecent.di8.=  107?23;2ir  »N.V.S.=  1. 298863 

Dist.  at  15  hrs=  108.   6.55    jDiff.  =    0?43'.34r    Prop.  log.  =     6161 

Dist.atl8hr8=106.32.42   jDiff.  =     1.34.13      Prop.  log.  =     2811 

Portion  of  time  = ^_     1^23T14!     Prop.  log.  =     335a 

Time  corresponding  to  first  distance  —  15.   0.   0 

Apparent  time  of  obs.  at  Greenwich  =  16?23?14; 

To  find  the  apparent  Timfe  at  the  Place  of  Observation  t— 
Lat.oftheplaceofob8.=43»17'  OrS.  .    .    .    Log.  8ecant=0. 137885 
Sun's  reduced  declinations  4. 13. 10  S.  .    .    .    Log.8ecant=0.001179 
Sun's  toerid.zen.  dist.  =39?  3^50rNat.V.S.=22.^556 
Sun's  true  central  alt.  =   26'.  49. 53  N.co-V.S.=548634 

Remainders  325078Log.s5.5U98S 
Sun's  horary  distance  from  noon  s  3  MSToS :    Log.  rising  =  5. 65 105.2 


Apparent  time  at  the  place  of  ob6.s20*  14T  5 ! 


Digitized  by 


Google 


OV  FINDING  THE  LONGITU0B   BY  LUNAR  OBSERVATIONS. 


461 


Apparent  thne  at  the  place  of  obs.=20M4?  5 ! 
Appar.  time  of  obs.  at  Greenwich  =161  23. 14 

Lwig,  of  the  place  of  obs.,  in  tiine=:3*50T51 !  =  57?42'45r  east. 

Example  4. 

April  1st,  1825,  in  latitude  49?30^  S.,  and  longitude  61?30^  E.,  by 
account,  at  1 1  t29T  per  watch,  not  regulated,  the  mean  of  several  distances 
between  the  moon's  remote  limb  and  the  star  Antares  was  76948' 27'',  and, 
at  the  same  time,  the  mean  of  an  equal  number  of  altitudes  of  the  moon's 
lower  limb  was  39"?  10' 12?,  and  that  of  the  star,  east  of  the  meridian, 
37?56^3? ;  the  height  of  the  eye  above  the  level  of  the  horizon  was  23 
feet;  required  the  longitude  of  the  place  of  observation  ? 


Time,  per  watch,  =     ll?29r  0'. 
Long.61?30^B.=  -  4.   6.   0 


Reduced  time  :r:  .  7^23?  0! 
Altitude  of  Antares  =  37?56^43r 
Dip  of  the  horizon  =:      —  4. 36 


Apparent  altitude  =    37?52^   7? 
Correction  =:       .    .       —   1;  14 


Star's  true  altitude  =  37?S0^53r 


Dist.])  's  remote  limb=:76?48^  27? 
Moon's  semi-diam.  ^     — 16. 50 


Apparent  distance  =    76^31 137^ 

Alt.  ]>  's  lower  limb  =  39?  lOU 2? 
Moon's  semi-diam.  =     + 16. 50 
Dip  of  the  horizon  r:     —  4. 36 


Apparent  altitude  =     39^22'.  26! 
Correction  =       .  '.       H-46.    9 


Moon's  true  altitude=  40?  8^35? 


Moon's  reduced  horizontal  parallax  =  61<  11? 

Sum  of  app.  alts.=:77?  14^33?N.V.  S.  sup.rr  1 .  220825 

App.  cent  dist  =  76. 31. 37  Nat.V.  sine  =  .  767012Log.diff.=:9. 995271 

^  Remainder  ==  .453813      Log.=5. 656877 

Natural  number  = 448898      Log.=5. 652148 

Sum  of  true  alt8.=:77?59^28?N.V.  S.  8up.=  1. 208063 

Truecentraldi8t.=76?  3^55?  ?  Nat.V.  8.=  .759165 

Dist.  at  6  hours  =  76. 57.   9   f^'^-  =  0?53M4?     Prop.  log.  =    5291 

Dist  at  9  hours  =  75.  3.  29   J  D^^-  =  ^^  53. 40      Prop.  log.  =    1996 

Portion  of  time  =    .......     It24ri7!     Prop.  log.  =    3295 

Time  corresponding  to  first  distance  =:     6.   0.   0 

Apparent  time  of  obs.  at  Greenwich  =:     7  J24r  1 7 ! 


Digitized  by 


Google 


468 


NAimCAL  ASTRONOMY. 


To  find  the  apparent  Tune  at  the  Place  of  Observation : — 


Lat.  of  the  place  of  obs.  =  49?S0'  OrS.  . 
Star's  reduced  declin.  =    26.    2.   2  S.  . 


Log.  secantrrO.  187456 
Log.  8ecant=:0. 046465 


Star's  merid.  zenith  dist.=23?27-581'Nat.co-sin.  917296 
Star's  true  altitude  =    .    37. 50. 53  Nat.  sine  =613570 


Remainder  ^  303726 Log.=::5. 482482 


Star's  horary  distance,  east  of  the  mer.=4?  5?23!Lbg.  rising=5. 71640.3 
Star's  reduced  right  ascension  ^      »     16. 18. 42 


Right  ascension  of  th6  meridian  = 
Sun's  reduced  right  ascension  =  . 


12M3ri9! 
0.43.19 


Apparent  time  at  the  place  of  obsarv.ss  1 1 130*  0! 
App.  time  of  observ.  at  Greenwich  =    *  7. 24. 17 


Longitude  of  tfie  place  of  obs.,  in  time=  4t  5?48;  ^  61?25U5f  eatt. 

Example  5. 

April  22d,  1825,  in  latitude  40?10<  N.,.  and  longitude  55?  17'  W.,  by 
account,  at  Ot23?  per  watch,  not  regulated^  the  mean  of  several  observed 
distances  between  the  nearest  limbs  of  the  sun  and  moon  was  48?47-46fy 
and,  at  the  same  time,  the  mean  of  an  equal  number  of  altitudes  of  the 
sun's  lower  limb  was  61?26'44^,  and  that  of  the  moon's  upper  limb 
48<?46'32r  I  the  height  of  the  eye  above  the  level  of  the  borismi  was  21 
feet;  required  the  longitude  of  the  place  of  observation  ? 


Time,  per  watch,  =     0*23r  0! 
Long.  55?17'.W.=:   +3.41.   8 


Reduced  time  = 


4i  4?  8! 


Alt.  sun's  lower  limb=  61^26^4: 
Sun's  semi-diameter  =    -h  15. 56 
Dip  of  the  horizon  =       —  4.24 


Sun's  app.  altitude  =  61?38'16r 
Correction  =s       .     .       —  0. 27 


Sun's  true  altitude  =  6l?37'49r 


Dist.  nearest  limbs  of 

mooti  and  sun    s     48?47  •  46f 
Sun's  8emi*diameter  s    -f  15. 56 

Moon's  semi-diameter=:  + 15. 32 

— » 

Apparent  distance  s   49?19a4f 

AlL  of  J  's  upp.  limb=:48?46^32r 
Moon's  semi-diameter=  — 15. 32 
Dip  of  the  horizon  =      —  4.  24 


Moon's  apparent  alt.=48?26^36f 
Correction  =      .     .       +36. 29 ' 


Moon's  true  altitude  ;;s49?  3'  5' 


Digitized  by 


Google 


OF  FINDING  THB  LOKOITVDB  »»  LJWAR  OBSERVATIONS.         408 

Moon's  reduced  horisoDtal  parallax  =  56'  16? 

Half  the  apparent  central  di>tance=s24?39'37r 
Half  difference  of  the  app.  alts.  =    6. 35 .  SO 

_  Log.diff.  =    9.994848 

°™n= 31?15i27?    Log.  sine  =5     9.715071 

Differences 18.   3.47      Log.  sine  =    9.491451 

Const,  log.  =  6,301030 

Natural  number  =    ...    .    .    .    317980    Log. «  .    .    5.502400 

Diff.  of  true  alts.=  12?34<  44  rN.V.  sine  024008 

True  cent  di8t.=48?61i  4r  )  N.V.siiie341983 

Di8t,at3h«iirB«48. 19.26  JDiff.  ~    0?3H38r    Prop.  log.   s»     7551 

I>ist.«t6h(mr9=49.47.57   jDi£^  :*     1.28.31      Prop.  log.   s    3082 

Portion  of  time  =s      ......     li  4?19!     Prop,  log,    =     4469 

Time  corrosponding  to  first  distance  «  3,   0.   0 

Apparent  time  of  obs.  at  Greenwich  ^     4 1  4?  1 9 ! 

To  find  the  apparent  Time  at  the  Place  of  Obaervation  i^ 

.  Aote.— Since  the  apparent  time  cannot  be  inferred  with  sufficient 
aocaracy  from  the  sun's  altitude^  on  aiseonat  of  the  proximity  of  that  object 
to  the  meridiap^  \t  must  be  deduced  from  the  moon's  true  central  altitude ; 
88  thus  I-* 

Lat.  o/ place  of  obs.=40n0<  O^N.  .    .    ,     .    .  Log.8ecant=.  116809 
Moon's  correct  dec.= 23. 13.46  N Log. secants:,  036716 

J'smer.zen.dist.  =    16?56n4r    Nat.  V.  8»043376 
Moon's  true  cent.  alt,«49.   3.   5  Nat. co-V.S= 244702 

Remainder  s  201326  L<^.s5. 308900 

Moon's  horary  dist.,  east  of  the  mer.  =  2?57"59?  Log.ri8ing=5. 45742.5 
Moon's  eorrected  right  ascension  ss    .     2.51.    1 

Right  ascension  of  the  meridian  =      .     2t23T  2! 
Sun's  reduced  right  ascension  =r     •     .     2.   0.   4 


Apparent  time  at  the  place  of  observ.=  0*22r58! 
Apparent  time  of  obsenr.  at  Greenwich=4,   4. 19 


Longitude  of  the  place  of  ohs.,  in  time=3?4i?21 !  =  55?20(  15f  west. 

/Google 


Digitized  by ' 


464 


NAUTICAL  ASTRONOMY. 


Example  6. 
May  6th,  1825,  in  latitude  34?45^  S.,  and  longitude  33?30^  E-,  by 
account,  at  21  ?30T  per  watch,  not  regulated^  the  mean  of  several  observed 
distances  between  the  nearest  limbs  of  the  sun  and  moon  was  11 9? 50' 38 T; 
and,  at  the  same  time,  the  mean  of  an  equal  number  of  altitudes  of  the 
sun's  lower  limb  (imperfeetly  observed,  owing  to  an  obstructed  horizon,) 
was  27?  13^27^,  and  that  of  the  moon's  upper  limb  (also  imperfectly 
observed,)  19?24M2^ ;  the  index  error  of  the  sextant  used  in  measuring 
the  distances  was  2^25?  subtractive,  and  the  height  of  the  eye  above  the 
level  of  the  horizon  19  feet ;-  required  the  true  longitude  of  the  place  of 
observation  ? 

Time,  per  watch,  =     21*30r  0! 


Long,33?30'.  E.,  =  -2.14.   0 
Reduced  time  =      .     19M6r  0! 


Alt  of  sun's  low.  limb=27?  13^  27^ 
Sun's  semi-diam.  =        +15.52 
Dip  of  the  horizon  =     —4.11 


Sun's  apparent  alt.  =  27^25^  8* 
Correction  =      .     .       —   1.41 


Dist.  nearest  limbs 

of  moon  and  sun=  119?50^3S1! 
Index  error  =     •    .       —  2. 25 
Sun's  semi-diameter  =    +15.52 
Moon's  semi-diameter =s  + 15. 30 


Apparent  distance  =  120M9^35r 

Alt.  of  ])  's  upp.  limb=  19?24C  12f 
Moon's  semi-diameter=:--15.30 
Dip  of  the  horizon  =     —4.11 


Moon's  apparent  alt=19?  4131^ 
Correction  =8      .    .       +50. 44 


Sun's  true  altitude  =  27^23^27'^      '    Moon  s  true  altitude^  19?55;  15f 

Moon's  reduced  liorizontal  parallax  ==  56^34T 
Half  the  app.  central  distance  =  60e  9'.47i^ 
Half  sum  of  the  appar.  altitudes—  23. 14. 49^ 

Log.difF.  =    9.997841 

Sum  =        83?24^37^       Log.  co-sine=9. 059787 

Difference  == 36. 54. 58        Log.  co-sine=:9. 902827 

Constant  log.=6. 301030 

Natural  number  = 182593     Log.sS.  261485 

Slim  of  truealte.=47n8M2r  N.V.S. sup.  =  1.678010 

True  cent  dist=  1 19?41  ^ 50r  )  NatV.  S.=:  1 .  495417 
Distatl8hours=120. 19.47    jDiff.  =  0?37^57r     Prop.  log.  =    6761 
Distat21hours=U8.50.22    JDiff.  =  1.29.25       Prop.  log.  =   3039 

Portion  of  time  = 1  *  16r24^  Prop.  log.  =t   S722 

Time  corresponding  to  first  distance  =     18.   0.   0 


Apparent  time  of  observ.  at  Greenwich  =  19^  16T24! 


Digitized  by 


Google 


OF  FIKBINO  THS  IjONGITUBE  BT  LUNAR  OfiSBRVATIONf.         465 

Since  the  obstruction  of  the  horizon  prevented  the  altitudes  of  the 
objects  from  being  taken  to  that  degree  of  accuracy  which  is  so  essentially 
necessary  to  be  observed  when  the  apparent  time  is  to  be  inferred  from 
their  altitudes  (though  sufficiently  exact  to  be  employed  in  the  reduction  of 
the  apparent  to  the  true  central  altitude), — ^therefore^  in  the  afternoon^ 
that  is,  on  May  7th,  at  1^53*  per  same  watch,  the  sun's  altitude  was 
again  observed,  and,  when  reduced  to  the  true,  was  found  to  be31°44^28^; 
at  that  time  the  latitude  of  the  ship  was  34?50'  S.  Hence  the  apparent 
time,  the  error  of  the  watch,  and  the  longitude  of  the  place  of  observation, 
are  obtained  as  follows  : — 


Time  of  observing  the  sun's  altitude,  per  watch,  =   1^53T  0! 
Hme  of  observing  the  lunar  distance,  per  watch,  =  21. 30.   0 

Interval  = 4»23r  0! 

Apparent  time  of  lunar  observation  at  Greenwich  =  19. 16. 24 


App.  time  at  Greenwich  of  observing  the  sun's  alt.=:  23?39r24! 

Sun's  declination  at  noon.  May  6th,  ss  .    •    .    .     16?3lC5KN. 
Correction  of  ditto  for  23*39?24!  =    •    .    •     •       +16.27 


Sun's  reduced  declination  =s 16948a8rN. 

Lat.  of  place  of  obs.:=34?50'  0?S Log.  secant^O.  085754 

Sun's  reduced ^dec.  =  16.48. 18  N Log.  seeant=0. 018955 

Sun's  mer.  zen.  dist.  =  5 1  ?38^  18rNat.V.  S.=379376 
Sun'strue  altitude  =  31.44.28N.co.V.S.=473918 


Remainder  =    94542    Log.  =  4. 975625 

App.  time  of  observing  the  sun's  altitude  =  1  ?53?35  7Log.rising=5. 08033.4 
App.  time  at  tjreenw.  of  obs.  sun's  alt.=  23. 39. 24 

Longitude,  in  Ume  = 2*14Tll!  =  33?32M5reast. 

JVbf^.— Hiis  is  the  longitude  of  the  meridian  where  the  sun's  altitude 
was  observed  for  the  purpose  of  finding  the  apparent  time. 


Remark, — In  place  of  finding  the  interval  between  the  time  of  observing 
the  lunar  distance  and  that  of  taking  the  sun's  altitude,  as  above,  this  part 
of  the  operation  may  be  performed  as  follows ;  which,  perhaps,  may  be 
more  intelligible  to  those  who  are  not  very  conversant  with  this  subject. 

2h 


Digitized  by 


Google 


466 


NAUTICAL  ASTILONOlfT. 


Apparent  time  of  observing  the  sun's  altitudes:  1  ^  5S?35  * 
Time  of  observing  dittOj  per  watch,  s     .     •     1  •  53.   0 


Watch  slow  for  apparent  time  c=  .    .     «    «  0T3$'. 

Timei  per  watch,  of  gbs.  the  lunar  distances  21. 30.   0 


Apparent  time  of  observing  the  lunar  dist.  =  21  ?30T35  ! 
Apparent  time  of  ditto  at  Greenwich  =      •     1 9. 1 6. 24 


2*14?li:=33?32M5rE. 


Longitude  of  the  place  where  the  error  of 
the  watch  was  found,  in  time  =  •     .     • 

JExample  7* 

June  22d,  1825,  in  latitude  SOMO^N.,  and  longitude  45?7^W.,by 
account,  at  3^0T5!  apparent  time,  the  mean  of  several  observed  distances 
between  the  moon's  remote  limb  and  the  planet  Venus  was  118M1'48T, 
and,  at  the  same  time,  the  mean  of  an  equal  number  of  altitudes  of  the 
moon's  upper  limb  was  30?  18^  25  T,  and  that  of  the  planet's  centre 
15?  1 1 M7^;  the  index  error  of  the  sextant  used  in  measuring  the  distances 
was  2'  10?  additive,  and  the  height  of  the  eye  above  the  level  of  the  sea  18 
feet;  required  the  true  longitude  of  the  place  of  observation  ? 


Apparent  time  ss     .      3^  0"  5! 
Long.  45?7' W.  =       3.   0.28 


Reduced  time 


6?  0r33! 


Alt  of  Venus'  centre=5l5?l  1  U7t 
Dip  of  the  horizon  ss     -*  4.   4 


Venus'  apparent  alt.=  15?  7'43r 
G)rrection  ^s      .    «       ^  3.   9 


Venus' true  altitude=  15?  4C34r 


Dist.  of  Moon's  remote 

limb  from  Venus=  118?41 :481 
Index  error  ss    .     .       +  2. 10 
Moon's  semi-diamet^s  -*16. 16 


Apparent  distance  s  11 8?  27 '42? 


Alt.  of  3) 'sup. limb  =  30?  18'. 25? 
Moon's  8emi-diameter=  ^16. 16 
Dip  of  the  horixon  s&      —  4.   4 


Moon's  apparent  alt.=29?58C  5? 
Correction  =      .    •       +49, 40 


Moon's  true  altitudes  30? 47^45? 


Moon's  reduced  horizontal  parallax  s=  59n3? 

Apparent  distance  =    .    ^     .  118?27'42? 
Venus' apparent  altitude  =     •     15.   7*43 
Moon's  apparent  altitude  =    •     29. 58.   5 


Sum 


163?SS:30? 


Digitized  by 


Google 


OP   FINDING   THB    LONOITUOB  BY  hVVATL  OBSBRVATIOKS.         46/ 

Sam  =       163^33  ^aO'r 

Log,  diff.    =2    9. 996430 

Half8um= 81?46U5r  Log.  co-sine  =  9. 155302 

Remainder  =     ^ 36. 40. 57  Log.  co-sine  =  9. 904152 

Natural  number  =    ..••;....     113/32  Log.=:9. 055884 

Twice  the  natural  number  s=  .     .    •     .     •     .     227464 
Sum  of  true  alt8.=45?52^  l9i:Nat,V.  S,  8up.=  1. 696264 

True  cent  di8t=117?57'23T  )  Nat.  V.  sine=l,468800 
Di8t.at6hour8=117-36.42    *DiflF.»:  0?  O'All    Prop. log.  =  2. 4206 
Dist.  at  9  bourse  119. 39.    7   i^^'"^  ^•*2-25       Prop,  log.  ^    .244» 

Portion  of  time  « Ot   1?12!  Prop.  log.  ^k  2, 1757 

Time  corresponding  to  first  distance  »      6.   0.   0 

Apparent  tiioe  of  obs.  at  Greenwich  sb     6  ?  1  ?  1 2  ? 
Apparent  time  at  the  place  of  observ,  =s    3.   0,  5 

Longitude  of  the  place  of  obs.^  in  time  =  3i  1?  7-  ==  45?I6'45T  west. 

i2^mar&.— In  taking  a  lunar  observation,  it  is  customary  to  have  three 
assistants,  two  of  whom  are  to  observe  the  altitudes  of  the  objects  at  the 
moment  that  the  principal  observer  measures  the  distance  ;  the  third  is  to 
be  provided  with  a  watch^  showing  seconds^  and  to  note  down  carefully  the 
respective  times  of  observation,  with  the  corresponding  distances  and  alti- 
tudes. But,  since  it  sometimes  happens,  particularly  in  small  ships,  that 
the  necessary  assistant  observers  cannot  be  in  readiness,  or  at  liberty  to 
attend,  the  following  example  is  given,  by  which  it  will  be  seen  how  one 
person  may  take  the  whole  of  the  observations  himself,  without  any  other 
assistant  than  merely  a  person  to  note  down  the  times  of  observation^  par 
watch,  with  their  respective  distances  and  altitudes. 

Example  8. 

July  6th,  1825,  in  latitude  49M3:  N.,  and  longitude  42?22C  W.,  by 
aoeo^Bt,  the  following  observations  were  made,  in  order  to  determine  the 
true  longitude ;  the  index  error  of  the  sextant  used  in  measuring  the  dis-* 
tances  was  1  '40?  subtractive,  and  the  height  of  the  eye  above  the  level  of 
the  sea  17  feet. 


Appar.  Time.  Mean  Time.     Meao  Altitude. 

21  ^  8-32?  Alt.of  sun's  low.  limb=46?5S^  01-] 
21.  9.32  Ditto  47.   7.  0    S21f  9T32?     47?  7^  0 

21.10.33  Ditto  47.16.  0  J 

2h2 


i^ 


Digitized  by 


Google 


468  NAUTICAL  ASTRONOMY. 

Appar.  Time.  Mmui  Time.        Mean  Altitude. 

21 M  ir37'  Alt.  of  J 's  upp.  limb=23";51130r-| 

21.12.37  Ditto                23.42.20  ^21M2?37^      23?42:20r 

21.13.37  Ditto                23.33.10  J 

21.14.50  Observed  distance  s  98.58.50  '^ 

21.16.  0  Ditto                98.58.20  I 

21.17.10  Ditto               98.57.50  >21. 17. 10      98.57.50 

21.18.20  Ditto       •         98.57.20 

21.19.30  Ditto                98.56.50  ^ 

21.20.43  Alt.of  )>'8upp.limb=20.   9.40  -j 

21.21.43  Ditto                20.   0.   0  J>21. 21.43       20.  0.   3 

91.22.43  Ditto                19.50.30  J 

21.23.48  Alt.of8un'8low.limb=49. 14.   0  ^ 

21.24.48  Ditto                49.22.40  J>21.24.48       49.22.40 

21.25.48  Ditto                49.31.20  J 


To  find  the  Sun's  Altitude  at  the  Time  of  talcing  the  mean  Distance  :— 

l8ttime21»  9r32!  l8talt.47?  T-  07  1sttimc21*  9r32' l8talU7?  7'  Or 
2dtime21.24.48   2d  alt.49. 22. 40    .^r4it}21. 17- 10 

As      0»15?16^  isto    2?15U0r    so  is    0^  7 "38 f  to  +  1.   7.50 

Reduced  observed  altitude  of  the  sun's  lower  limb  =       .    .    48?14'50r 

Sun's  semi- diameter  = +15.46 

Dip  of  the  horizon  = —  3.57 

Sun's  apparent  central  altitude  =s       48?26'397 

Correction  of  the  sun's  apparent  altitude  = —  0. 44 

IVue  altitude  of  the  sun's  centre  = 48?25'55' 

To  find  the  Moon's  Altitude  at  the  Time  of  taking  the  mean  Distance  : — 

I8ttime21*12r37^  lstalt.23?42'20?  l8ttime2lM2r37t  l8talt.23?42C20r 
2dUme21.21.43   2dalt.20.   0.  3    J2:Vl}21. 17. 10 

As      0?  9T  6:  isto    3?42n7^     so  is    0*  4r33:  to  -  1.51.   8 

Reduced  observed  altitude  of  the  moon's  upper  limb  =     .    .    21?51'12f 

Moon's  true  semi-diameter  = ,    —14.52 

Dip  of  the  horizon  = —  3. 57 

Moon's  apparent  central  altitude  s 21?32'23T 

Correction  of  the  moon's  apparent  altitude  = +48.   4 

True  altitude  of  the  moon's  centre  s= 22? 20' 27' 

Moon's  reduced  horizontal  parallax  ss  .    .    .    .    541 147 

Digitized  by  VjOOQ  IC 


OF  FINDING  THB  LONGITUBB  BY    LUNAR  OB8BRVATIOH8.         469 


Obs.di8t.betw.  J  &  ©=s98?57'50r 
Index  error  of  8extant=  —  1.40 
Sun's  semi-diameter  =   + 15. 46 
Moon's  8emi-diameter=  +14.52 


Appar.  central  dist.  =  99?26'43! 


App.  time  of  ob8erv.=21*17"10! 
Longitude42?22'W., 

in  time  ^     .    .     +  2. 49. 28 


Reduced  time  past  noon, 

July  7th,  =      .    .    0?  6r38! 


Half  the  app.  central  di8tance=  49^43'.  24"^:   • 
Half  the  diflF.  of  the  app.  alts.  =r  13.27.   8 

Log.  diflF.  =?  .    9. 997660 

Sum  = 63?10'32'r     Log.  sine  =    .    9.950556 

DiflFerence  = 36. 16. 16       Log.  sine  =    .    9. 772033 

Sum  =:  19. 720249 


Archss 46?26^2ir     Log.  sine  =    .    9.860124i 

Half,  the  diflF.  of  the  true  alu.=:  13.   2. 44 


Sum  =     •    . 
DiflFerence  = 


59?29^  51    Log.co.8ine=     9.705665 
33. 23. 37      Log.  co-sine  =    9. 921639 


Sum=  19.627304 


Half  the  true  distance  —      ..    49?22130?     Log.  tso-sine  s     9. 813652 


True  central  distance  ==  .     .     98?45!  Or? 

Distance  at  0  hour,  or  noon,  =  98. 48.   3   iDiff-O^  3^  3rP.log.=  1.7710 

Distance  at  3  houra  =     .    .    97.26.  27    j^*ff-^- 21-36  PJog.=  .3436 

Portion  of  time  = 0?  6r44!  Prop.log.=5l.4274 

Hme  corresponding  to  first  distance  =s     .    0.   0.   0 


Apparent  time  of  observ.  at  Green^ch  =  0*  6T44* 
Apparent  time  at  the  place  of  observations 21. 17*  10 

Longitude  of  the  place  of  observ.,  in  time  =  2*49^34!  =42?  23^ 30r  west 


Note, — Proportional  logarithms  will  be  found  very  convenient  in  the 
reduction  of  the  altitudes  of  the  objects  to  tlie  time  of  taking  the  mean 
lunar  distance :  thus,  to  the  arithmetical  complement  of  the  proportional 
logarithm  of  the  first  term,  add  the  proportional  logarithms  of  the  second 
and  third  terms ;  and  the  sum,  abating  10  in  the  index,  will  be  the  propor- 
tional logarithm  of  the  reduction  of  altitude.--See  Example,  page  75  or  76. 


Digitized  by 


Google 


4^0  HAUTICAT.  ASTRONOMY. 

Remarlci.-^ln  taking  the  means  of  the  several  observations^  those  which 
are  evidently  doubtful  or  erroneous  ought  to  be  rejected.  A  doubtfiil 
altitude  or  distance  may  be  readily  discovered^  by  observing  if  the  successive 
differences  of  altitude  or  distance  be  proportional  to  those  of  the  times  of 
observation.  If,  however,  the  time  (which  is  supposed  to  be  accurately 
noted,)  and  two  of  the  observations  be  correct,  the  erroneous  observation 
may  be  easily  rectified  by  the  rule  of  proportion. 

In  order  to  attain  to  the  greatest  accuracy  in  deducing  the  mean  from  a 
series  of  observations,  these  ought  to  be  taken  at  equal  intervals  of  time,  as 
nearly  as  possible ;  such  as,  one  minute,  one  minute  and  a  hatf,  or  two 
minutes. 


Problsm  X. 

Owen  the  apparetit  Time,  the  observed  Distance  between  the  Moon  and 
Sun,  a  fixed  Star,  or  a  Planet,  the  Latitude,  and  the  Longitude  iy 
accctmt;  to  find  the  true  Longvtudx. 

RULB. 

Compute  the  true  and  the  apparent  altitude  of  each  object's  centre,  by 
Problem  I.,  IL,  III.,  or  IV.,  between  pages  404  and  410,  according  as  the 
mood  is  compared  with  the  sun,  a  fixed  star,  or  a  planet. 

Reduce  the  observed  to  the  apparent  central  distancei  by  the  rule  to 
Problem  IX.,  page  456 ;  with  which,  and  the  computed  altitudes  of  tlie 
objects,  let  th^  true  central  distance  be  determined,  by  any  of  the  methods 
given  in  Problem  VII.,  betweeki  pages  433  and  453 ;  and  find  the  apparent 
time  at  Greenwich  answering  to  the  true  central  distance,  thus  computed, 
by  Problem  VIIL,  page  454.  Then,  the  difference  between  the  apparent 
times  of  observation  at  the  ship  and  at  Greenwich  will  be  the  longitude  of 
the  ship  or  place,  in  time ;  which  is  to  be  called  east  or  west,  according  as 
the  apparent  time  at  the  place  of  observation  is  greater  or  less  than  that  at 
Greenwich. 

Example  1. 

August  4th,  1825,  in  latihide  40^25^  N.,  and  longitude  56?S6;  W.^  by 
account,  at  19M0T35!,  apparent  time,  the  mean  of  several  observed 
distances  between  the  nearest  limbs  of  the  sun  and  moot)  was  107^3' 47^; 
requited  the  true  lon^^tude  of  the  place  of  observation } 


Digitized  by 


Google 


OF    FINDING  THB  LOKOITUDB  BT  LUNAR   OBSBRVATION8.        471 


Appar*  time  of  obs.s:  19t  10735 ! 
Long.56?36^W.,in 

times       .    •    .      3.46.24 


lUduced  time  = 


22*56759: 


Obs.  dist.  between 

moon  and  sun  =    107°  S'ATI 
Sun's  8emi-diameter=     +15.48 
Moon's  semi-diam.  ==    +14.57 


Appar.  central  diBt=  107^34^321! 


To  find  the  Sun's  true  and  apparent  Altitude  :^ 

Sun^s  dist.  frommerid.=4*49725!  •  •  .  •  Log.  rising  s  5.843150 
Lat.ofplaceofob8.  s=;40?25:  OIN.  .  .  .  Log.  co-sine  =  9.881584 
Stto's  reduced  dec.  ^    17«    1.43  N.    .    .    .    Log.  co-sine  =  9. 980530 


Sun*s  mer.  aen.  dist.=  23?23^  17^  Nat.  V.  S.=082163 

Natural  number=507299  Log.=5. 705264 


Sun's  true  central  alt.=:24?14'.  197N.  oo-V.  S.s589462 
Correction  of  altitudes    +   1.59 


Sun's  apparent  alt.  s>  24?16U8^ 


To  find  the  Moon's  true  and  apparent  Altitude  &— 


App.  time  of  observ.s:19^  10*35! 
Sun's  reduced  R.  A.  =  9.   0.28 


R.  A.  of  the  merid.  = 
Moon's  red.  R.  A.  = 

Moon's  dist.  from  mer. 
Lat.  of  place  of  obs.  =r 
Moon's  reduced  decs 


4Mir  3! 
1.28.35 

=2*42T28! 
40?  25!  O-fN. 
13.44.  4  N. 


Moon's  red.horiz.  par.  s.  54!  167 

Moon's  red.  semi-diam.s  14!  47  7 
Augmentation  of  ditto  =     +10 


Moon's  true  8emi-diam.=14!577 
.  .  Log.  rising  «  5.381870 
.  .  Log.  co«sine3s9. 881584 
•    .      Log.  co-sine=s9. 987402 


Moon's  mer.  «en«dist.s26?40!567  NatV.  S.a  106489 

Natural  number  %:  178174  Log.s5. 250856 


Moon's  true  cent,  alt.: 
Correction  of  altitude^ 


=45?40!  157  N.co-V.  S.C284663 
s   ^37.   0 


Moon's  apparent  alt.^  45?  3!  157 


Digitized  by 


Google 


472  NAUTICAL  ASTRONOMY. 

To  find  the  true  central  Distance,  and,  hence,  the  Longitude  of  the 
Place  of  Observation  :— 

Half  the  app.  cent.  dist.=53?47'  I6r 
Half  sum  of  the  ap.  alts.=34. 39. 46| 

Log.diff.  =    .    .    .    fl.  995327 

Sum= 88?27'  2ir  Log,  co-sine  =    .    .     8.431961 

DiflFerence  =      ...     19.   7. 29f    Log.  co-sine  =    •     .     9. 975343 


Sum  =  18.402631 


Aiy;h= 80?5in0f    Log.  co-sine  =     .    .    9.2013151 

Half  sum  of  true  alts.si:  34. 57. 17 


Sum= 115?48:27^     Log.  sine  :?=     .    .     .    9.954369 

Difiference  =     .    «    r    45.53.53       Log.  sine  ^     .    .    .     9.856186 

Sums:  19.810555 


Halfthe  true  distances  53  ?3K  3f    Log.  sine  ==     .    .    .    9.905277| 

True  central  distaiice=  107?  2'.  6r  } 

Dist.  at  21  hours  =     107.55.    2    iDiff.=0?i2^56rProp.log.=  .5315 

Dist.at24hours,ornoonl06.33.30   }DiflF.=  1.21.32  Prop. log.=  . 3439 

Portion  of  time  = 1^56r52!Prop.log.=  .  1876 

Time  corresponding  to  first  distance  =    •     21.   0.   0 


Apparent  time  of  observ.  at  Greenwich  =  22^56?52! 
Apparent  time  at  the  place  of  observations  19. 10. 35 

Longitude  of  the  place  of  observ.,  in  time  =  3*46?17*  =  56?34i  15r  W. 

Example  2. 

September  27th,  1825,  in  latitude  36?  151  S.,  and  longitude  47?301  K, 
by  account,  at  14?58?10!  apparent  time,  the  mean  of  several  observed 
distances  between  the  moon's  remote  limb  and  the  star  Aldebaran  was 
55?17^36f  ;  required  the  true  longitude  of  the  place  of  observation  ? 

Apparent  time  of  observation  z=      .     .    •     •     14t58?10! 
liongitude  47?30'  E.,  in  time  s       .    .      —  3. 10.   0 


Reduced  times 11M8?10! 

Observed  distance  of  moon's  remote  limb  s     55?17'36? 
Moon's  true  semi-diameter   ss       •    •    •    •       —14.49 


Apparent  central  distance  =      •    •    •    •    «    55?  2C47? 

/Google 


Digitized  by ' 


OF  FINDING  THE  LONGITUDB   BY   LUNAR   OBSERVATIONS.  473 

Sun's  right  ascension  at  noon,  Sept.  27th,  =     12^  14T54! 
Correction  of  ditto  for  11  *48r  10!  =    ...       +1.47 

Sun's  reduced  right  ascension  s      .     .     •     •     1 2  M  6T4 1 ' 
To  find  the  true  and  apparent  Altitude  of  the  Star  Aldebaran  :— 


Apparent  time  of  observation  =       .    •    « 
Sun's  reduced  right  ascension  =     •    •     • 

Right  ascension  of  the  meridian  =  .     •     . 
Aldebaran's  right  ascension  =:'    ;    .     .     . 

Aldebaran's  distance  from  the  meridian  = 


14*58-10! 

12.16.41 
»  > 

3*14?51! 
4.'25;55 

mir  4! 


Aldebaran's  dist.  from  merid.s  1 M 1  r  4 !  .  .  Log.  rising  =  4. 67S460 
Lat.  of  the  place  of  observ.=36?  15 '.  0':rS.  .  Log.  Qp-sine=9. 906575 
Aldebaran's  reduced  dec.  =   16.    9.    1  N.    .       Log.  co-sine=:9. 982513 


Aldebaran's  mer.  zen.  dist.=52?24!   KN.V.S.=389859 


Natural  number  =    36944  Log.as4. 567548 


Star's  tniealtitude=:34?58'24r  Nat.co.V.S.=  426803 
Correction  of  alt.  =     +   1.21 


Star's  appar.  alt  =  34?59'.45r 

To  find  the  true  and  the  apparent  Altitude  of  the  Moon :« 

Moon's  reduced  semi-diameter  rs       .     •     •'    14^42? 
Augmentation  of  ditto  = +7 


Moon's  true  semi-diameter  =: 14!49T 

Moon's  reduced  horizontal  parallax    =     53'55T 

R.  A.  of  the  meridian  =      .    3*  1475 1 ! 
Moon's  reduced  right  ascen.s  0. 42. 28 


Moon's  dist.  fi-om  the  merid.=2^32r23!  .  .  Log.  rising  =  5. 328420 
Lat.  of  the  place  of  ob8erv.=36?  15 '.  OrS.  .  Log.  co.sine=  9, 906575 
Moon's  reduced  declination^  9. 19. 30  N.     .    Log.  co-sine=  9. 994223 

Moon's  merid.  zenith  di8t.=  45?34!30rN.V.S.=300025 

Natural  number  =  1 695 19Log.= 5 .229218 

Moon's  true  altitude  =  32?  2^10!' N.co.V.S.=469544 
Correction  of  altitude  =  —  44. 32 


Moon's  apparent  alt.  =  3 1  ?  17 '  38r 

/Google 


Digitized  by ' 


474  NAOTICAL  AtTRONOMT. 

To  find  the  true  central  Distance,  and,  hence,  the  Longitude  of  the 
Place  of  Observation  : — 

Half  the  ap.cent.di8t.=27?31 '  23^? 
Halfdiff.ofapp»iUts.  3*  1.51.   3f 

Log.diff.l9. 996651 

Sum= 29?22'27rLog.si&e  9.690648 

Differences     .    .    .    25.40.20  Log. sine  9.636711 


Sum= 39.324010 


Half  sum  a 19.662005  .    .     19.662005 

Half  diff.  of  true  alts.  =  l?28f  7?Log.sine  8.408737 


Arehor e6?48'20rLog.taa.ll.253868Log.sin.9.9993S5 

Halfthe  true  distances  27?22^54ir    ....      Log.  sinesd.  662680 


TVtt6  ctotral  distance  *»  54  M5  'A91  \ 

Di8tenceat9hou«.=s56.    8.    7    iDiflF.=:l?22:i8r  Prop.  log,=  •  3399 

Di8t.atl2hr8,orm{dnt.54.40.   8  J  I>lff.«1.87.59    Prop. log.* . S109 

Portion  of  time  = 2M8r  32!  Prop,  log.s.  0290 

Time  corresponding  to  first  distance  =       9.    0.    0 


Apparent  time  of  obsehr.  at  Greenwich  se  1 1  M8?22!  ^ 

Apparent  time  at  the  place  of  observ.  =     14. 53.  10 

Longitude  of  the  plabe  of  observ.^  in  time—  3^  9*48!  3±  47? 27' east. 

Example  3.; 

December  25th^  1825,  in  latitude  39?  13^  N..  and  longitude  42?55^W., 
by  account,  at  14M9T27'  apparent  time,  the  mean  of  several  obsenred 
distances  between  the  moon's  nearest  limb  and  the  planet  Mars  was 
94?40^28^ ;  required  tl^e  longitude  of  the  place  of  observation  ? 

Apparent  time  of  observation  =     ....     14M9T27I 
Longitude  42? 55  ^  W.,  in  time  =    •    .    •       +  2. 5 1.  40 


Reduoad  times 17-4lr  7! 

/ 

Observed  distance  of  moon's  nearest  limb    =:  94?40^  22? 

Moon's  true  semi-diameter  =: -fi5«41 


Apparent  central  distance  =      »    •    <    •    .    94?56!  3? 

/Google 


Digitized  by ' 


OF  FINDING  THB  LONGlTODt  BY  LtTtlAR   OBSBRVATIONS.         475 

Sun's  right  ascension  at  noon, December  25th,=  18?  15T13! 
Correction  of  ditto  for  17 Mir? '  «     •    .    •       4  3. 16 


Sun's  reduced  right  ascension  =      •     .     .    •     18M8r29! 
Apparent  time  of  observation  =      •     •     *     •     14.49.27 

Right  ascension  of  the  meridian  ::£.••      9?  7*56! 


To  find  the  true  and  the  apparent  Altitude  of  the  Planet  Man  i« 

Horizontal  parallax  of  Mars  ^  5  seconds. 

R.  A.  of  the  meridian  a   9t  7^56? 
Reduced  R.  A.  of  Marss  13.12. 23 


Mars' dist.  from merid.=  4?  4r32!  .  .  .  Log.  rising  =  5.7136S0 
Lat.  of  place  of  obs.  ==  39?  13'.  OrN.  .  •  Log.  co-sines  9. 889168 
Reduced  dec.  of  Mars=:  8. 52. 53  S.      .     .      Log,  co-sine=  9. 994761 

Max«'mer.zen.dist.  -48?  5^53r  Nat.V.S.=i:332142 

Natural  number  =  395921  Log.=  5«  597609 

IVii6  altitude  of  Mars£3l5?46U6rN.co.V.  S.=728063 
Correction  of  altitudes*    +  3. 14 


App.  alt.  of  Mars   =     15^50^  0* 


To  find  the  true  and  the  apparent  Altitude  of  the  Moon's  Centre  :— 

Moon's  reduced  horizontal  parallax  s= 56^48C 

Moon's  reduced < semi-diameter  15C28^  +  augmentation  13T  =   15 '4 It 

R.A.ofthemerid.=9i  7^56! 
Moon's  fed.  R.  A.S  7.    1.30 


])'8di8t.frommerid.2*  6r26! Log.  rising  =  5. 171280 

Lat.ofplaceofobs.39?13^  0?N Log.  co-sine  =  9.839168 

Moon's  red. dec.=  1 9. 49. 23  N-     ....    Log.  co-sine  =  9. 9734/2 

>  *8  mer. zen. dist.  19?23'S7^Nat.  vers.  sine=056740 

Natural  number  =  108123     Log.=5. 033920 

J's true  cent.  alt.=56937'48r  Nat.co.V.S.=  164863 
Correction  of  alt.  ==  — 3 1 .   7 


Moon's  ap.  alU«  56?  6:41? 

/Google 


Digitized  by ' 


476  NAUTICAL  A8TAONOMT. 


To  find  the  true  centrid  DiBtance,  and,  hence,  the  Longitude  of  the 
Place  of  Observation  :— 

Half  ap.  cent.  di«t.=47?28:   Hi 
Half  sumof  ap.alt8.=:35. 58. 2o| 

Log.diff.  19.994221 

Sum=    ....     83?26122rLog.co-8in.  9. 057868 
Differences     .     .     11.29.41  Log.co-Bin.  9. 991201 

Sum  s 39. 043290 


H^lfsums: 19.521645    .    .     19.521645 

Half  sumof  true  alt8.=36?  12'  17^Liog.co.sin.  9. 906826 


Ai^ch=   ....    24?19:33rLog.  8ine=9.614819Log.tang.9.655197 

Half  the  true  dist  =  47?  19U8ir         Log.  sine  =      ...    9. 866448 

True  central  dist.  =  94?39'.37^  ) 

Dist.  at  15  hours  =    96.    4.    2  /Diff.=  l?24<25r  Prop.log.=  .3288 

Dist.  at  18  hours  =    94.30.    7  (l^^ff-^   1-83.55     Prop.  log.  =  .2825 

Portion  of  time  = 2Mir47*  Prop.  log.  =  .0463 

Time  corresponding  to  first  distance  ==     15.    0.    0 


Apparent  time  of  observ.  at  Crreenwich  =17-41T47' 
Apparent  time  at  the  place  of  observ.  =:  14. 49.  27 

Long,  of  the  place  of  observ.^  in  time  =     2  t52r20!  «  43?5  ^  west. 


Example  4. 

December  30th,  1825,  in  latitude  46?30^S.,  and  longitude  84?15CE., 
by  account,  at  21  MO?  15'  apparent  time,  the  mean  of  several  observed 
distances  between  the  nearest  limbs  of  the  sun  and  moon  was  107?2'7^9 
and,  at  the  same  time,  the  mean  of  an  equal  number  of  altitudes  of  the 
moon's  upper  limb  was  15?40'24^ ;  but,  for  want  of  the  necessary  assist* 
ants,  the  sun's  altitude  could  not  be  taken ;  the  height  of  the  eye  above  the 
level  of  the  horizon  was  IS  feet;  required  the  true  longitude  of  the  place 
of  observation? 

Apparent  time  of  observation  = 21M0T15! 

Longitude  84?  15' E.,  in  time  = 5.3/.   0 


Reduced  time  = ; 15t33T15! 

/Google 


Digitized  by ' 


OF  FmDfNO  THB  IX>NGIT0I>B   BY  LUNAR  OBSERVATIONS.         477 

Observed  distance  between  the  moon  and  8un  =  .  107?  2'  7^ 

Sun's  semi-diameter  = •       +16.18 

Moon's  semi-diameter  = +16.5 


Apparent  central  distance  s: 108?34'30^ 

Observed  altitude  of  the  moon's  upper  limb  ==      .  15?40^24! 

Moon*s  true  semi-diameter  = .  —16.   5 

Dip  of  the  horizon  =: —4.4 


Moon's  apparent  altitude  = 15?20C15r 

Correction  = +53. 16 

True  altitude  of  the  moon's  centre  =      •    ...     15?  IS^Sir 
Moon's  reduced  horizontal  parallax  =     •    •     •     •     •     58' 47^ 

To  find  the  true  and  the  apparent  Altitude  of  the  Sun's  Centre : — 

Sun's  horary  dist.  from  mer.=:2*49r45 !     .    .    Log.  rising  =  5. 418280 
Lat.ofthe  place  of  observ.=46?30!  OTS.     •    Log.  co-sinen  9.837812 

Sun's  reduced  declination  =23.    8. 14   S.      •    Log.co-sine=  9.963583 

t ___ 

Sun's  mer.  zenith  di8tance=:23?21U6rN.V.S.=081988 

Natural  number  =  165835  Log.=:5. 219675 


Sun's  true  cent.  alt.=:48?46'46r  Nat.  co-V.  S.=247823 
Correction  of  ditto=     +  0.43 


Sun's  apparent  alt.=:  48?47'29^ 


To  find  the  true  central  Distance,  and,  hence,  the  Longitude  of  the 
Place  of  Observation  :— 

O's  ap.alt.48?47^29r       ©'s  true  central  alt.48?46'46r 
})'sap.alt.l5.20. 15         D 'strue  central  alt.  16. 13.31 


Sum  =      64?  7'.44r  ^tif"}    1.436349   Sum64?59^59r  '^:*}.  1.422622 
DiflF.  =     33.27.14    N.V.S.  .165671   Diff.32.33.15  N.V.S.  .  157117 


Arch  first  =  .    .     1 .  270678  Arch  secondzz  1 .  265505 

/Google 


Digitized  by ' 


478  VAVncAM.  asteonoiiy. 

Sum  of  app.  alto.=:64?  7  •  44  rNatV.  S.=  1 .  436349 
App.  cent.  di8t.=  108?34'30rNAt.V.S.=  1.318546 

»         ■  ■ 

Arch  third  = 117803 Log.  =       5.071156 

Arch  seconds 1.265505  Log.  =       6.102264 

Arch  first  = 1. 270678  Log.ar.co=:  3. 895965 


Natural  number  ==  . 117323  Log.  X      5.069385 

Sum  of  true  aIt8.=64?59'59rN.V.S.8up.l.  422622 

True  cent.  di8t.=  107M6'.34r  J  N.V.sinel.  305299 
Di8t.atl5hour8=108.   4.31    jDiflF.  =  0^17^57^     Prop.  log.=  1.001 2 
Dist.  at  1 8  hour8=  106.  27. 28   }  Diff-  =  1  -  37.   3      Prop.  log.=  .  2683 

Portion  of  time  =: 0?S3?17'  Prop.  log.=:  .7329 

Time  corresponding  to  first  distance  =     15.   0.   0 

Apparent  time  of  observ.  at  Greenwich^   15 133?17  • 
Apparent  time  at  the  place  of  observ.  zz    21. 10. 15 

Longitude  of  the  place  of  obs.^  in  time  =  5?36r58!  =  84?14^30r  east. 

Remark.-^ln  Problem  XXIX,  page  320,  of  «  The  Young  Navigator's 
Guide  to  the  Sidereal  and  Planetary  Parts  of  Nautical  Astronomy,"  there 
is  an  interesting  method  given  for  reducing  the  apparent  central  distance 
between  the  moon  and  sun,  or  a  fixed  star,  to  the  true  central  distance,  by 
an  instrumental  operation ;  it  being  a  correct  mechanical  mode  of  working 
the  lunar  observaUons  by  Gunter's  scale  and  a  pair  of  compasses. 


PaoBLBM  XI. 
To^ftd  the  LmgUude  of  a  Place  by  the  Eclipses  of  JupUer's  Satellites. 

First, 
To  know  if  an  Eclipse  wiU  be  visiffle  at  a  gieen  Place. 

Rule. 
f 
Convert  the  mean  time  of  the  eclipse  at  Greenwich  (as  given  in  page  ITI. 
of  the  month  in  the  Nautical  Almanac,)  into  apparent  time,  by  Problem 
II.,  page  416;  and  let  this  time  be  reduced  to  the  meridian  of  the  place 
of  observation,  by  Problem  IV.,  page  297. 


Digitized  by 


Google 


OF  FINDING  THB  JLONOITUDl  BY  BCUFSES.  479 

Now^  if  at  this  reduced  time  Jupiter  be  not  less  than  6  degrees  above 
the  horizon  of  the  given  place,  and  the  sun  be  as  many  below  it^  or  stars 
of  the  third  $j(Uignitude  be  visible  to  the  naked  eye,  the  eclipse  may  be 
observed  at  that  placet  this,  it  is  presumed,  does  not  require  to  be 
illustrated  by  an  example. 

SmoNP, 

To  find  the  Longitude  of  the  Place  of  Observation  of  an  Eclipse. 

RULB. 

Reduce  the  mean  time  of  the  eclipse  at  Greenwich  into  apparent  time, 
by  Problem  IL,  page  416.  Then,  to  the  observed  time  of  the  eclipse,  at 
the  given  place^  ^PP^y  ^he  error  of  the  watch  for  apparent  time,  deduced 
from  obaervationa  of  the  sun's  altitude,  or  from  those  of  a  fixed  star,  a 
planet,  or  the  moon  :  hence  the  apparent  time  at  the  place  of  observation 
will  be  known.  Now,  the  difference  between  this  time  and  the  apparent 
time  at  Greenwich  will  be  the  longitude  of  the  place  of  observation  in 
time ;  which  will  be  east  or  west,  according  as  the  former  is  greater  or  less 
thun  the  latter* 

Example  I. 

January  8th,  1825,  in  latitude  39?5(  N.,  and  longitude  28?3f  W.,  by 
account,  an  immersion  of  the  first  satellite  of  Jupiter  was  observed  at 
8M2t59!,  by  a  watch  which  was  1T46!  fast  for  apparent  time;  required 
the  true  longitude  of  the  place  of  observation  ? 

Mean  time  of  the  eclipse  at  Greenwich  =  10M0T29! 
Equation  of  time  ss _  7, 16 


Apparent  time  of  the  eclipse  at  Greenwichss  10?  3713! 

Time  of  observation,  per  watch,  a:      ...    8*  1 2759 ! 
Watch  fast  = p    •    .    .     -  1.46 


Apparent  time  at  the  place  of  observation  ^     8M 1 7 1 3 1 
Apparent  time  at  Greenwich  =      ,    .     «,  •  10.   3. 13 

Longitude  of  the  place  of  observ.,  in  time  =     I  ?527  0!  =5;  28?0'0r  west. 

Hote. — If  Jupiter  be  far  enough  from  the  meridian  at  the  time  of 
observing  an  immersion  or  an  emersion  of  one  of  his  satellites,  the 
apparent  time  of  observation  may  be  inferred  directly  from  his  altitude;  and. 
If  the  altitude  be  taken  at  the  same  instant  of  observing  the  immersion  or 
emersion  of  the  satellite,  the  use  of  a  watch  will  then  become  unnecessary. 


Digitized  by 


Google 


480  NAtrriCAL  astronomy. 

Example  2. 

January  2d,  1825,  in  latitude  39?5i;i0r  N.,  and  longitude  4?15:  E., 
by  account,  an  emersion  of  the  first  satellite  of  Jupiter  was  observed;  and, 
at  the  same  instant,  the  altitude  of  that  planet's  centre,  east  of  the  meri- 
dian, was  found  to  be  28?49'30T  ;  the  height  of  the  eye  above  the  level  of 
the  sea  was  20  feet ;  required  the  true  longitude  of  the  place  of  observation  ? 

Mean  time  of  the  emersion  at  Greenwich  s= 
Equation  of  time  = 


Apparent  time  at  Greenwich  =  •     •  . 

Observed  altitude  of  Jupiter's  centre  = 

Dip  of  the  horizon  for  20  feet  s      •  . 

Jupiter's  apparent  altitude  s       •     .  • 

Refiraction=  l'43r) 

Parallax  =  -0.   2  JDiffe'^»<^e  =  • 

True  altitude  of  Jupiter's  centre  = 


7V3?50: 

-14.31 

6f49ri9! 

28»49^80'r 
-  4.17 

28?45n3r 
-  1.41 

28?43'.32r 


Lat.ofplaceofobs.=39?5in0rN.     ....    Log.  8ccant=0. 114812 
Jupiter's  red.  dec.  =3   19.    5.48  N Log.  secant^  0. 024583 

Jupiter's  mer.z.  dist.  20?45'22r  Nat,V.  S.  =  064902 
Jupiter's  true  cent.  alt28. 43. 32  N.co-V.S.=  519385 

Remainder  =  454483     Log.s  5.  6575 1 8 

Jupiter's  horary  dist.,  east  of  the  merid.=:4'32rl6!Log.rising=5.  79691.3 
Jupiter's  reduced  right  ascension  =     .     8. 43. 39 

Right  ascension  of  the  meridian  ==     .    4*  1 1 T23? 
Sun's  reduced  right  ascension  =     •     »  21.   4.  52 

Apparent  time  at  the  place  of  ob^erv^  =7-   6T31 1 
Apparent  time  at  Greenwich  ==      ••    6.49.  19 


Longitude  of  the  place  of  obs.,  in  umes:0^17"12!  =  4?18'.0f  east. 

ttemarlcs. 

An  immer$ion  of  a  sateUUe  is,  the  instant  of  its  entrance  into  the  shadow 
of  Jupiter;  and  an  emersion  is  that  of  its  re-appearance  out  of  the  shadow. 
The  instant  of  an  immefsion  is  known  by  the  last  appearance  qf  the 
satellite;  that  of  an  emersion,  by  its^r^f  appearance. 

Tlie  eclipses  of  Jupiter's  satellites  afford  the  readiest,  and,  for  general 
practice,  the  best  method  of  determining  the  true  Icmgitudes  of  places  on 


Digitized  by 


Google 


OF  FINDING  THB  LONGITUDE  BT  LUNAR  BCLIPSBS.  481 

shore :  but,  since  those  eclipses  cannot  be  distinctly  observed  except  by 
means  of  telescopes  of  a  high  magnifying  power, — and  since  these  cannot 
possibly  be  used  at  sea,  on  account  of  the  incessant  motion  of  the  vessel, 
which  continually  throws  the  object  out  of  the  field  of  view, — ^this  method, 
therefore,  though  the  very  best  at  land,  will  be  but  of  little,  if  any 
advantage  to  the  mariner.  It  is  to  be  observed,  however,  that  this  method 
of  finding  the  longitude  is  not  always  available ;  because  Jupiter  passes  so 
apparently  close  to  the  sun  at  certain  intervals,  that,  for  about  six  weeks 
in  every  year,  both  himself  and  his  satellites  are  entirely  lost  in  the  superior 
splendour  of  the  solar  rays. 

Problem  XIL 
To  find  the  Jjmgitude  of  a  Place  by  an  Eclipse  qfthe  Moon. 

Rulb. 

Observe  the  times,  per  watch,  (regulated  to  apparent  time,)  of  the 
beginning  and  the  end  of  the  eclipse :  the  mean  of  these  times  will  be 
the  apparent  time  of  the  middle  of  the  eclipse  5  the  difference  between 
which  and  that  given  in  the  Nautical  Almanac,  will  be  the  longitude  of  the 
place  of  observation  in  time ;  which  will  be  east  or  west,  according  as  it 
is  greater  or  less  than  the  time  at  Greenwich. 

Note. — If  only  the  beginning  or  the  end  of  the  eclipse  be  observed,  the 
apparent  time  of  observation  must  be  compared  with  the  time  answering  to 
the  corresponding  phase  in  the  Nautical  Almanac ;  but,  it  must  be  remem- 
bered, that  it  will  always  be  conducive  to  greater  accuracy  to  observe  the 
instants  of  both  phases. 

Example  1. 

May  Slst,  1825,  in  latitude  38?24'  N.,  and  longitude  26^0^  E.,  by 
account,  the  beginning  of  the  lunar  eclipse  was  observed  at  13^35?32! 
per  watch^  and  the  end  at  14t4T47'  j  the  error  of  the  watch  was  2T13! 
slow  for  apparent  time;  required  the  true  longitude  of  the  place  of 
observatioit  ? 

Beginning  of  the  eclipse,  per  watch,  =      .    .     13!35r32! 
End  of  ditto  ditto        ^      .    .     14.   4.   7 


Middle  of  the  eclipse,  per  watch,  =      .     .    .     13  *50r  9^ ! 
Error  of  the  watch  s +2.13 


Apparent  time  of  the  middle  of  the  eclipse  =  13?52?22i! 

Apparent  time  of  ditto  at  Greenwich  s    .     .  12.    8. 30 

Longitude  of  the  place  of  observ.,  in  time  =  lM3T52i!s25?58^7irE. 

2  I 


Digitized  by 


Google 


482  NAUTICAL  AITMNOlfy. 


EsMtnple  2. 

November  25th,  1825,  ia  latitude  J6?40'  N»»^nd  IqPgiMe  54°40^  E., 
by  account,  the  beginning  pf  the  lunar  ellipse  was  observed  at  7*6"40!, 
an4  the  end  at  9J0T55;  per  wat«b,  not  regulate,d^  la  order  to  ftud  the 
error  of  the  w^tch,  the  altitude  of  Aldebiirau,  ea^t  of  th?  meridlao,  was 
taken  at8?7T30:,  and  found  to  be-28'?42:30r5  the  height  of  the  eye 
above  the  level  of  the  horizon  waa  22  feet;  required  the  true  longitude  oi 
the  place  of  observation  ? 

Time,  per  watch,  of  observing  the  star's  altitude  =    8*  7^0* 
Longitude  54?40^E.,  in  time  MBS 3.38.40 

Reduced  times .    .    .      4^28750! 

Sun^s  right  ascension  at  noon  = 16^  3T44! 

Correction  of  ditto  for  4^28750'  «■      .    .     .     .       +0.45 


■irw 


Sun's  reduced  right  ascenaipu  v^    •    ,    ,    ^    »    ,    16 1  4729 1 

Observed  altitude  of  Aldebaran  =?.....    28? 42 'SOT 
Dip  of  the  horizon  for  22  feet  =  , -^^  4. 30 

Star's  apparent  altitude  ?=   •    ^    •    .    •    •    »    ,    28?86^  07 
Refraction  SF    »««•*»•«•#••      -»  L44 

Star's  true  altitude  =     •    .    •    , 28?36'167 

Lat.ofthe  place  of  obs.=16?40^  O'N.   ,    .    .    Log.  secant— 0.  OI80S9 
Aldebaran's  red.  dec.  =   16.   9.   5  N.    .    •    .    Log.  secant=sO.  017489 

AldebMan'si»er,z.dist.=  0?30iWr  N»t,V. 8,=3000040 
Aldebarw'struealt*  P9   28.36.  }6N.cp-V.8»»i$3l240 

Remainder  =  521200Log.;=5. 717004 

Star's  horary  distance,  east  of  the  mpr.=54  M  7713 '  hof,  ris)in;^5, 7531341 
Star's  reduced  ri^ht  ascension  =     •      4. 25. 57 

Right  ascension  of  the  inusridian  =  .      0  *  8744  J 
Sun's  reduced  rigbt  ascension  =.      .     16,   4. 2^ 

Apparent  time  of  obs.  the  star's  alt,  m   8!  4716! 
Time  of  observation,  per  watch,  :s   ,       6.   7«80 

Watch  fast  tor  apparent  time  as       ,  8?  IS ! 

Digitized  by  VjOOQ IC 


OF  FINDING  THB  VAEIAtlOH  Q?  rHB  COMPASS. 

Begmuing  Qf  th^  eclipse,  per  watph,  5=   7t  6-4Q5 
Bncl  of  ditto  ditto      =9    9-  0.56 


Middle  of  the  eplipse,  pef  watch,  =       8*  3?47i- 
Errorof  thn  watch  =     .    .    .    ,    .       -  3,15 


App.  time  of  the  middle  of  the  eclipse  =  8?  (iT32i'. 
Apparent  time  of  ditto  j|t  Greenwich  =  4. 22.   0 


Longitude  of  the  place  of  obs.,  in  timer=8;38782f !  =  54?3817i?  east 

Hemarks. 

From  the  two  preceding  examples^  it  is  evident  that  the  beginning  )vnd 
the  end  of  the  eclipse  are  the  principal  phases  frqm  which  the  longjtpde  is 
to  be  found.  If  the  observer  be  provjded  with  a  sextant,  those  phases  may 
be  observed  to  a  tolerable  degree  of  accuracy^  by  inean^  of  the  largest 
telescope  belonging  to  that  instrument;  or  they  may  be  observed  with  a 
good  night  telescope. 

This  method  of  finding  the  longitude  at  sea  is  evidently  the  most  simple 
of  any  of  the  astronomical  methods  that  have  been  proposed  for  that 
purpose ;  however,  since  the  lunar  eclipses  happen  so  very  seldom^  there 
are  but  few  opportunities  of  carrying  it  into  practice  :  nevertheless,  when- 
ever such  eclipses  take  place,  the  prudent  mariner  will  do  well  to  avail 
lumself  thereof,  and  to  determine  his  longitude  by  them  accordingly. 


SOLUTION  OP  PROBLEMS  RBLATIVB  TO  THE  VARIATION 
OP  THE  COMPASS. 

The  variation  of  the  eomposs  is  the  deviation  of  the  points  of  the 
mariner's  compass  from  the. corresponding  points  of  the  horizon,  and  is 
denominated  east  or  west  variation  accordingly. 

East  variatum  is,  when  the  north  point  of  the  compass  is  to  the 
eastward  of  the  true  north  point  of  the  horizon ;  west  variation  is,  when 
the  north  point  of  the  compass  is  to  the  westward  of  the  true  north  point 
of  the  horizon.  « 

The  variation  of  tfie  compass  may  be  found  by  various  methods,  such  as 
amplimdes,  azimuths,  transits,  equal  altitudes/  rising  and  setting  of  the 
celestial  objects,  &c. 

2i2 


Digitized  by 


Google 


484  NAtrricAL  astronomy. 

The  true  ampUluie  of  any  celestial  object  is,  an  arch  of  the  horizon 
intercepted  between  the  trae  east  or  west  point  thereof,  and  the  object  a 
centre  at -the  time  of  its  rising  or  setting. 

The  magnetic  amplitude  of  an  object  is,  the  arch  of  the  horizon  that 
is  intercepted  between  its  centre,  and  the  east  or  west  point  of  the  com- 
pass, at  the  time  of  its  rising  or  setting;  or,  it  is  the  compass  bearing  of 
the  object  when  in  the  horizon  of  the  eastern  or  western  hemisphere. 

The  true  amplitude  of  a  celestial  object  is  found  by  calculation;  and  the 
magnetic  amplitude  is  found  by  an  azimuth  compass. 

The  true  azimuth  of  a  celestial  object  is,  the  angle  Contained  between 
the  true  meridian  and  the  vertical  circle  passing  through  the  object's 
centre. 

The  magnetic  azimuth  is,  the  angle  contained  between  the  magnetic 
meridian  and  the  azimuth,  or  vertical  circle  passing  through  the  centre  of 
the  object;  or,  in  other  words,  it  is  the  compass  bearing  of  the  object,  at 
any  given  elevation  above  the  horizon. 

The  true  azimuth  of  a  celestial  object  is  found  by  calculation;  and  the 
magnetic  azimuth  by  an  azimuth  compass. 


Problem  I. 

Given  the  Latitude  of  a  Places  and  the  Sun'§  magnetic  AmpUtude;  to  find 
the  Variation  of  the  Compose. 

Rule. 

Reduce  the  apparent  time  of  the  sun's  rising  or  setting  to  the  meridian 
of  Greenwich,  by  Problem  III.,  page  297 ;  to  which  time  let  the  sun's 
declination  at  noon  of  the  given  da%be  reduced,  by  Problem  V.,  page  298. 
Then,  to  the  logarithmic  secant  of  the  latitude,  add  the  logarithmic  sine 
of  the  sun's  reduced  declination;  and  the  sum,  .abating  10  in  the  index, 
will  be  the  logarithmic  sine  of  the  true  amplitude, — to  be  reckoned  north 
or  south  of  the  true  east  or  west  point  of  the  horizon,  according  to  the 
name  of  the  declination.  Now,  if  the  true  amplitude,  thus  found,  and 
the  ma^etic  amplitude,  observed  per  azimuth  compass,  be  both  north  or 
both  south,  their  difference  is  the  variation;  but  if  one  be  north  and  the 
other  south,  their  mm  is  the  variation : — and  to  know  whether  it  be  east 
^or  west,  let  the  observer  look  directly  towards  that  point  of  the  compass 
representing  the /m«  amplitude;  then,  if  the  magnetic  amplitude  be  to 
the  left  hand  of  this,  the  variation  is  easterly ;  but  if  to  the  righi  hand,  it 
is'westeriy. 


Digitized  by 


Google 


OF  FINDING  THB  VARIATION  OF  THB  COMPASS,  485 

Example  1.    .. 

May  20th,  1825,  in  latitude  48?50^  N,,  and  longitude  6?30:  W,,  ajk 
about  7*40T,  the  sun  was  observed  to  set  W.  56?42^  N,;  required  the 
variation  of  the  compass  ? . 

,  Estimated  time  of  observation  ^ 7^40? 

Longitude  6?30' W.,  in  time  =   .••...     +26 

Redweddmess     • •    8^  6? 

Sun's  declination  at  noon,  May  20th,   .     •    .     19?58M3rN. 
Correction  of  ditto  for  8*6?  = +4.11 

Sun's  reduced  declination  s  ••••.•    20?  2^54fN. 

Latitude  of  the  place  of  observ.=  48?50'N.        Log.  secantslO.  181608 
Sun's  reduced  declination  =     .     20?  2'54rN.  Log.  sine  =     9.535057 

True  amplitudes    .    •    .    W.  31?23<  8rN.  Log.sine  =    9.716665 
Magnetic  amplitude  =       .    W.  56. 42.   0  N. 

Variation  =?..«.:     •      25?  18^.52? ;  which  is  weet,  because  the 
magnetic  amplitude  is  to  the  right  hand  of  the  true  amplitude. 

Example  2« 

July  10th,  1829;  in  latitude  18?40'.  N.,  and  longitude  73?45f  W.,  at 
about  17^29T,  the  sun  was  observed  to  rise  E.  30?12^  N.;  required  the 
variation  of  thcf  compass  ? 

Estimated  time  of  observation  ==       •    .    •    •    •     17^29? 
Longitude73?45' W.,  intime  =     .    .    •    .      +4.55 

Reduced  time  = 22*24? 

Sun's  declination  at  noon,  July  10th,     .    .    22?16n6?N.     * 
Corrcetionofditto  for  22^24?=     .     .    .    .       -  7. 13 

Sun's  reduced  declination  =s 22?  9 C  3fN, 

Digitized  by  VjOOQ IC 


.^ 


486  NAttxiCAt  AistltONbttYi 

Latitude  of  the  place  of  observ.ac  18°40'.  OrN.  Log.  seCant=  10. 023468 
Sun's  reduced  declination  =     .    22.  9;   3  N.  Log.  sine  =     9. 576395 

true  amplitude  =     .    .    .    E.  23927'   7^1*:  Log.  Sine  =s     9.599863 
Magnetic  amt)litu(ie  =       .    fi,  30.  ll   b  N. 

Variations    ......      6?44C53r;  which  is  e(M<,  because  the 

magnetic  kmplitu^e  is  to  the  l^  Kdfid  ttf  the  trtie  Aniplitudci 

Example  3. 

October  17th,  1825,  in  latitude  42?  lO:  N.,  and  longitude  14?30^  W., 
at  about  5!277,  the  sun  was  observed  to  set  W.  7'?33'.  N.j  required  the 
variation  of  the  compass  r 

EsUniated  time  of  observation  =   .   .    •      5*27"  0*. 
Longitude  14 ?30'.  W.,  in  time  =      .     .        +58.0 

kediiced  tiine  = 8^25?  0! 

Sun's  declination  at  noon>  October  17th,  =  9?  15^9:  S. 
Correction  of  ditto  for  6!  26 T  »  •    •    •       +  5.5^ 

Sun'srednceddeclmatiohs     .    .    .    .      '9^2inKS. 

Latitude  oif  the  place  olf  observ.  =  42?  10  •  O?^.  Log.  secahkrs  lO.  1S0067 
Sun's  reduced  declination  =     .      9. 2 1 . 1 1  S.   Log.  sine  =     9.2 10901 

True  ampKtude  =     .    .    .    W.  12?39:57'S.  Log.  sine  =    9.340968 
MHghetic  wni)llW(fc  *      .    W.    ?.3l  0  N.        • 

Variations 20?12'.57?;  #hM*  isioert,  beonne  the 

magnetic  aqaplitude  is  to  the  right  hand  of  the  troe  amplitude. 

In  finding  the  variation  of  the  compass  by  this  method,  the  sun's 
amplitude  shduld  hk  tHketi,  With  an  azimuth  compa^,  Wheh  HM  altitude- 
of  his  lower  limb  is  equal  to  the  sum  of  his  semi-diameter  and  the  dip  of 
the  horiibn.  Thus,  if  the  stin's  GiehiUdiamet^  be  16'5fj  and  *e  dip  of 
the  hor&in  4<17^  (for  20  feet),  the  khtik  st  20^22^  is  Ae  hqgbt  which 
the  lower  hmb  of  that  object  should  be  above  the  horizon,  at  the  time 
of  obsefrVing  its  'Amplitude. 


Digitized  by 


Google 


OF  FINDING  TBS  VAtUATlOK  OF  TB£  COMPASS.  467 

If  the  index  of  the  quadrant  be  set  to  the  altitude^  thus  determined^  the 
sun's  magnetic  amplitude  may  be  taken  when  his  lower  limb  attains  that 
altitude,  either  at  rising  or  setting ;  for,  although  the  sun  is  apparently  so 
el^vatied)  f^ty  m  nctOMtit  lof  tile  atoid^beritd  reftticUoii,  his  centre  is 
MMdlythi^niiithehorfauiii^fiiieplftMoroltaemrtioiu   * 

Note. — For  the  principles  of  finding  the  variation  .of  the  compass  by 
the  amplitude  of  a  celestial  object,  see  '^  The  Young  Navigator's  Guide  to 
the  Sidereal  and  Planetary  Parts  of  Nautical  Astronomy,''  page  261. 


Probj^m  II. 

Given  the  Laiktideofa  Placey  the  Sim's  Altitude^  and  his  magnetic 
JzimtUhj  to  find  tke  Variation  of  the  Compass. 

RuLfi, 

Reduce  the  appattnt  time  of  observation  to  the  meridian  of  Greenwich, 
by  Probleni  III.,  page  297 ;  to  which  time  let  the  sun's  declination,  at  noon 
of  the  given  day,  be  reduced,  by  PhiUem  V.^  page  298,   ' 

Find  the  true  central  altitude  of  the  sun,  by  Probkm  XtV*,  pi^SSO) 
now. 

To  the  sun's  polar  distance,  add  its  true  ceitfral  altitude  and  the  latitude 
of  the  jplace  of  observation ;  take  half  their  sun^  and  call  the  differenc# 
Wtween  it  and  the  jpolar  distance  the  remainder. 

llien,  to  the  logarithmic  secanls,  less  radius,  of  the  tcift  central  altitude 
and  the  latitude,  add  the  logarilimiic  co^^sines  of  the  half-sum  and  the 
remainder :  half  the  sum  of  theS6  four  logarithifis  will  be  the  logarithmic 
€o-^Qe  of  an  arch ;  which,  being  doubled,  will  be  the  true  azimuth^  to  be 
redumed  from  the  northin  north  latitude,  but  from  the  south  in  south 
latitude;  towards  the  east  in  the  forenoon,  and  towards  the  west  in  the 
t(teni«Diii 

Now,  if  the  true  azimuth,  th^is  found,  and  the  magnetic  azimuth, 
bb^retv^d  per  iKzMnfth  eompttss,  k^  oh  t)ie  "same  side  of  the  mimiian,  their 
difference  is  the  variation;  but  if  on  different  sides,  their  sum  is  the 
variation : — and  to  know  whether  it  be  east  or  west,  let  the  observer  look 
directly  towards  that  point  of  the  compass  which  represents  the  true 
^aimiiUhj  tfiei^  if  the  ttii^etic  azittiutli  be  to  the  l^  hand  ef  thts^  the 
variation,  is  ewteidly  j  but  if  t#  irtie  right  Aond,  it  is  wescerlf  • 


Digitized  by 


Google 


488 


KATTTICAL  AtTBOMOICT. 


Example  1. 

April  15th,  1825,  in  latitude  39^40^  N.,  and  longitude  14?0^  W.,  at 
4  M  0?  per  watch,  the  observed  altitude  of  the  sun's  lower  limb  was  27  ?  1 1  - » 
and  the  bearing  of  his  centre,  by  azimuth  compass,  N,-80?37-30T  W. ; 
the  height  of  the  eye  above  the  level  of  the  sea  was  24  feet ;  required  the 
variation  of  the  compass  ? 


Time  of  observation,  per  watch, : 
Longitude  14?0'  W^  in  time  as 


Reduced  time  = 


4*10r  0! 
+56.  0 

5*  6?  01 


Observed  altitude  of  sun's  lower  limb  ss    27?11  •  0! 


Sun's  semi-diameter  =       •     J     .     • 
Dip  of  the  horizon  for  24  feet  ^     « 

Sun's  apparent  altitude  =s  .     .    •    . 
Refraction  l(50r  -  Parallax  81  = 

Sun's  true  central  altitude   =    •    , 


+  15.57 

-  4.42 

27^22'.  15r 

-  1.42 

27?20^33r 


Sun's  dec.  at  noon,  April  15th,=9?45'46rN. 
Correction  of  ditto  for  5 ?6r  =     +  4. 32 


Sun'a  reduced  declination  =       9?50a8rN. 


Sun's  north  polar  distonce  =     80?  9M2r 

Sun's  true  central  altitude  =     27.20.33      Log.  secant   =     0.0514S1 

Latitudeof  the  place  of  obs.=  39.40.   0      Log.  secant   b     Q.  113638 

Sum  = \i7nO'.l51 


Half  sum  = 
Remainder  =5 


Arch  =  •    .    •    •• 

True  azimuth  =     • 
Magnetic  azimuth  =: 

Variation  =       •    . 


.    73?35C  7V  Log-  co-sine  =     9.451150 
•      6.34.^4^    Log.  co-sine  s    9.997133 

Sums  19.613872 


.    50?  9^  9r    Log.  co-sine  =    9.806686 

N.100?18;i8rW. 
N.  80.37.30  W. 


19?40U8r;  which  is  wett,  because  the 


magnetic  azimuth  is  to  the  right  band  of  the  true  azimuth. 


Digitized  by 


Google 


OF  FINDINO  THB  VARIATION  OF  THB  COMPASS.  489 


Example  2. 

March  lOUi,  1825,  in  latitude  42?4i:  S.»  and  longitude  14895^  E.,  at 
19^25?  per  watch,  the  observed  altitude  of  the  sun's  lower  limb  was 
18?3' ,  and  the  bearing  of  his  centre,  by  azhnuth  compass,  S.I08?37  -SOrE. ; 
the  height  of  the  eye  above  the  level  of  the  sea  was  19  feet ;  required  the 
variatioa? 


Tim%of  observation,  per  watch,  = 
Longitude  148?5  i  K,  in  time  =:    • 

Reduced  time  =:        •     •     •     •     • 


Observed  altitude  ofthe  sun's  lower  limb=:  18?  3'  Or 

Semi-diameter  = ' 

Dip  of  the  horizoh  :=      *••••. 

Apparent  altitude  =      ••.••• 
Refraction  2^5ir  -  Parallax  8 r  =  .    . 

Sun's  true  central  altitude  =  .     «     •    . 

Sun's  dec.  at  noon,  March  10th,  =  4?  5'43r  S. 
Correction  of  ditto  for  9i32T40!=     -  9.21 


19?25?  o: 
9.52.20 

9^32T40! 

:18»  S'.   or 
+  16.  7 
-  4.11 

18?l4<56r 
-  2.43 

I8?i2n3r 

Sun's  reduced  declination  =  .    .      3?56'.22rS. 


Sun's  south  polar  distance  =     .     86?  3^38r 

S.un's  true  central  altitude  =     .    18. 12. 13      Log.  secantr:  0.022298 

Latitude  of  the  place  of  observ.  =  42.41.   0      Log.  secant=  0.133647 

Sum=: 146?56fTir 


Half  sum  ;= .    73?28^25ir  Log.  co-siner:  9.454014 

Remainders      .    .    •    •    .    •     12.35. 12^    Log. co-sine=  9. 989435 

Sum=  19.599394 


Archr: 50?54'42r     Log. co^inei:  9. 799697 

TVue  azimuth  =     .    •    •    .  S.  101  ?49^  24?  E. 
Magnetic  azimuth  =  .     .    .  S.  108.37.30   £. 

Variations 6M8'7  6! ;  which  is  ecue,  because  the 

magnetic  azimuth  is  to  the  left  hand  of  the  true  azimuth. 


Digitized  by 


Google 


490  IIAVTICAIi  AflmOIiOMT. 

Note. — After  this  manner  may  the  variation  be  deduced  from  the  true 
altitude  and  magnetic  bearing  of  a  fixed  etar,  a  planet,  or  the  moon,  as 
will  be  seen  by  referring  to  ^^  The  Voung  Navigator's  Guide  to  the  Sidereal 
attd  Pl&il^tA^  Pdrtt  tdt  Ndtotlcal  AdtfrOH«my/'  jMige  263 ;  whtM^  the  prb- 
eiples  of  this  methed  Are  fimiiliaHy  explained  hf  %  ttemogniphia 
proj^tiob» 


^  •-'•  -- 


J  new  MMod  oft&mputing  ike  CfM  JMmyHh  «(^(i  todoBtUl  Otjecty  andy 
ikm^y  finding  the  Fi»Uaim  ^Ifo  Ctoni|MM« 

RctLB. 

From  the  natural  versed  «ine  supplement  •f  the  stun  of  the  latitude  and 
the  true  altitudei  subtract  the  natural  versed  sine  of  the  olgect's  polar 
distance :  to  the  logarithm  of  the  remainder  add  the  logarithmic  secants 
of  the  latitude^  and  the  true  altitude :  the  sum  of  these  three  logarithms, 
rejecting  20  ftt)M  til(  index,  will  be  the  logariihai  «lf'  the  MsUiral  versed 
sine  supplement  of  the  true  azimuth;  to  ^e  ret^k^ti^d  (torn  thfe  north  in 
north  latitude,  but.  ttom  the  south  in  south  latitude ;  the  difference  between 
which  and  the  magnetic  azimuth  will  be  the  variation  ot  the  compass^  as 
before* 

Example  1. 

October  17th,  1825,  in  IfttitXnfe  42?10;  N.,  imd  hmgitwl*  U^dO'.  W;, 
at  3^2?  per  watch,  the  menn  of  several  observed  altitudes  of' the  sun's 
lower  limb  was  23  939 '34^,  And  the  miean  of  an  equal  ttttmb^r  of  his 
central  bearings,  by  *tiiftiuth  compass^  N.  lO$?28!56?  W.;  the  height  of 
die  eyh  above  th«  level  x^t  the  hroHzmi  \^as  17  feet}  leqtltted  thfe  ^iffatioti 
of  the  compass  ? 

^Tlttie  Of  obseiVation,  pet  tjlitch,  =    ;    .      3?  2?  0^ 
Longitude  14^30  J  W,,  iti, time  ==    .    •       +58.  t) 


Redtrced  time  = 4t  0?  0! 

Sun's  declin&tibn  at  noon,  October  1 7th,  =  9?  15  '19?  S. 
Correction  of  dilftofi*  4  *0*«!   ds     .    .       -f  S.«9 


Sun's  reduced  declination  zz     .    .    .    .      9?  18'58r  S« 


Sun's  north  poUrdistMice  a  .    ^    >    %    Mn8<i8?  . 

/Google 


Digitized  by ' 


OP  FINDING  TUB  V^UItAtlOlC  W  TBB  COMPASS.  491 

Ob«erV«d  ftltittide  of  Sun's  lotin^r  limb  =     2S?89C34? 
Semi-diatneter  =      ;    •     .     i     •    ^    •       +16.   5 
Dip  of  tht  horizon  r:     i    4    v    *    .    •       —  3^57 


Appfrretil  altitude  =:       ....*.     23^51 :  49^ 
Refraetion  2C9?  -  Parallax  %:    .    .    ,       -  2.    1 


Sun's  t^ue  central  altitude  =    .    .    ••    .    23?49Ui? 

Lat.of  placeofobs.=:42?10'  Or     ...     .    Log.  secant  =  10. 130067 
Sun's  true  cent.  alt.=:23. 49. 4 1       .    .    .     .    Log.  secant  =:  10. 038692 

Sum=.    .    .    .    65?59Ulrrl.V.S,8up.=  i.  406821 
Sun's N.  polar  dist.=99. 18. 58  NatV.  sine=l.  161881 


Remainder  =   .  244940  Log.=:5. 389060 


Thiea^muth=:N.12»?41-63rW.  N.V.SvStip.2:3612o9  Li«.=:a.  557819 
Magnetic  do^z  N>109-.  28. 56  W. 

Variations:       .     20?12'57rj    which   is  west,  because   t\\e  magnetic 
azimuth  is  to  the  right  hand  of  the  true  or  computed  azimuth. 

*£!»r(itnjMv  fi. 

Pecember  9th|  18255  in  latitude  i9?40^  N.,  Ihe  true  attitucle  of  ^he 
star  Capella  was  20*?  10'^  and  his  bearings  by  azimutli  compass, 
N.  41  ?0'  E. ;  required  the  variation  ? 

Latiifplaceof  obs.=:l9?40^  0*  .    .     .     .    Log.  af^cant  =:  It).  0261(18 
dapelfe's  true  ilt.  =  20. 10.  X)    .    .    .    .'  Log.  ^ckdt  =  10.0fi747» 

Sum=    .     .    .     .    39*50^  OrN.V.S.sup.zz  1.767911 
Capella'sN.pol.dis.=:44. 11.24  N.Vwsin'd  =     .282968 


ReteiuAder  :=  I-.  4949ii!iiogA9.  mHO 

Trueazimuthr:N.47?  9U5rE.  N.V.S.sup'.=  1.67«»28L^.3«»225a8ft 
Magnetic  do.=  .N.  41.   0.   0  E. 


Variation  =t       .      6?  9 '43 1'';   which  is   east,    because   the   magnetic 
aan^mith  ts  1»  the  le/t  hakd  tf  iHe  true  or  counted  wHBmth. 

Remarks — Instead  of  finding  the  natural  versed  sine  supplement  of  the 
aiitt  tif  the  tlire«  k^garithnn^  that  mem  may  lie  considered  as  a  l<^garithmie 


Digitized  by 


Google 


492  NAUTICAL  ASTRONOMY. 

rising.  In  this  case^  if  the  supplement  of  the  time  corresponding  thereto 
be  taken  from  Table  XXXII.^  and  converted  into  degrees,  by  Table  L  or 
otherwise,  the  result  will  be  the  true  azimuth.  Thus,  in  the  last  example, 
the  sum  of  the  three  logarithms  is  6. 225289 ;  the  time  corresponding  to 
this,  in  the  Table  of  Logarithmic  Rising,  is  8t5lT21',  which,  taken  from 
12  hours,  leaves  3*8T39V;  and  this,  being  converted  into  time,  gives 
47^9 '45^  for  the  true  azimuth,  which  is  precisely  the  same  as  above. 


Problem  IIL 

To  find  the  Variation  of  the  Compois  by  Observations  of  a  circunqH>lar 

Star. 

Rule. 

From  the  log.  co-sine  of  the  star's  declination,  (the  index  being  increased 
by  10,)  subtract  the  logarithmic  co-sine  of  the  latitude :  the  remainder  will 
be  the  logarithmic  sine  of  the  star's  greatest  eastern  or  western  azimuth 
(according  as  it  may  be  situated  with  respect  to  the  meridian) ;  to  be 
reckoned  from  the  north  in  north  latitude,,  but  from  the  south  in  south 
latitude.    Then, 

From  the  logarithmic  sine  of  the  latitude,  (the  index  bring  increased  by 
10,)  subtract  the  logarithmic  sine  of  the  star's  declination,  and  the 
remainder  will  be  the  logarithmic  sine  of  the  star's  true  altitude  when  at 
its  greatest  eastern  or  western  azimuth.  Set  the  index  of  the  quadrant  -Co 
this  altitude,  and,  when  the  stir  has  attained  it,  let  its  bearing  be  taken  by 
the  azimuth  compass ;  the  difference  betweefn  which  and  the  computed 
azimuth,  when  they  are  of  the  same  name,  or  their  sum  when  of  contrary 
names^  will  be  the  variation ;  which  will  be  east^  if  the  observed  or  magnetic 
azimuth  be  to  the  left  of  the  computed  azimuth ;  otherwise,  west. 

Example  1. 

January  1st,  1825,  in  latitude  41^53'.  S.,  the  greatest  eastern  azimuth 
of  the  star  Canopus,  by  azimuth  compass,  was  S.  72?50^  E. ;  Required  the 
variation  of  the  compass  ?       *  - 

To  find  the  Star's  Altitude  when  at  its  greatest  Azimuth  :— 

Latitude  of  the  place  of  observ.  =  41  ?53 '  Or  S.    Log.  sine=:9. 824527 
Reduced  declination  of  Canopus  =:  52. 36. 10  S.    Log.  sinez=9. 900063 


Star's  alt.  at  greatest  azimuth  =  57?10UOr        Log.  Btne=9t  924464 

/Google 


Digitized  by ' 


OF   FINDING  THB  VARIATION  OF  tHB   COMPASS.  493 


To  find  the  Star's  greatest  eastern  Azimuth : — 

Reduced  declination  of  Canopu8=52?36^  lOr  S.  Log.  co.8ine=9. 783430 
Lat.  of  the  place  of  observation  =  41. 53.   0   S.  Log.  co-8ine=:9.  871868 


Greatest  eastern  azimuth  =     S.  54?39'45rE.  Log.  sine  =    9.911562 
Magnetic  azimuth  =     •     .      S.  72.50.   0  E. 


Variation  ==    ......       18?  11/. 45 ^ ;  which  is  ea«^  because  the 

magnetic  azimuth  is  to  the  left  hand  of  the  computed  azimuth. 

Example  2- 

December  31st,  1825,  in  latitude  43?45<  N.,  the  greatest  western 
azimuth  of  the  star  Dubhe,  by  azhnuth  compass,  was  N.  16?56^  W. ; 
required  the  variation  of  the  compass  ? 

To  find  the  Star's  Altitude  when  at  its  greatest  Azimuth:— 

Latitude  of  the  place  of  observ.  =r  43?45'.  OrN.    Log.  sine= 9. 839800 
Reduced  declination  ot  Dubhe  =     62. 4 1 .  19  N.     Log.  sine=9. 948670 

Star's  altitude  at  greatest  azimuth=:51?  6^  8r        Log.  8ine=9.891130 

To  find  the  Star's  greatest  western  Azimuth  :— 

Reduced  declination  of  Dubhe  =   62^1^  19?N.  Log.  co-8iue=9. 661648 
Lat.  of  the  place  of  observation  =  43. 45.   0  N.  Log.  co-8ine=9. 858756 

Greatest  western  azimuth  =     N.  39?25 ^58rW.  Log.  sine  =     9. 802892 
Magnetic  azimuth  =     .     .      N.  16.56.   0  W. 

Variation  = '•       22?29'58^ ;  which  is  west,  because  the 

magnetic  azimuth  is  to  the  right  hand  of  the  true  or  computed  azimuth. 

-  Itemarks* 

In  the  above  method  of  finding  the  variation  of  the  compass,  the  star's 
declination  must  be  greater  than  the  latitude  of  the  place  of  observation, 
and  of  the  same  name. 

A  star,  or  other  celestial  object  is  said  to  be  circumpolar  when  its 
^distance  from  the  elevated  pole  is  less  than  the  latitude  of  the  given  place 
(the  declination  and  latitude  being  of  the  same  name) ;  because,  under 


Digitized  by 


Google 


494  WAUTICAL  ASffftONOMr. 

such  circumstances,  the  object  comes  within  the  circle  of  perpetoal 
apparition,  and  r-evolves  round  the  celestial  pole  without  erer  settidg^  or 
going  belpw  the  horizoi^  of  t^at  place. 

The  variation  of  the  coinpass  may  be  found  by  equal  altitudes  of  the 
fixed  stars  J  as  thus  :— 

Let  the  star'^  altitude  be  observed  i|i  the  eastern  hemispher^j  wb^V)  it  is 
at  least  two  hours  distant  from  t\\e  meridian }  and,  at  the  s^m^  ivistant,  l^t 
its  bearing  be  taken  with  an  azimuth  compass :  then,  when  the  star  comes 
to  the  same  altitude  in  the  western  hemisphere,  }et  its  azimi|th  be -again 
taken.  Npw»  half  the  di^ience  between  the  eastern  a^d  western 
azimuths  will  be  the  variation ;  which,  when  the  observations  are  reckoned 
from  the  south  point  of  the  compos,  will  be  east  or  west  according  as 
the  eastern  or  western  azimuth  is  the  greatest ;  but  if  they  be  reckoned 
fropi  the  nortl^  point  of  the  copip^ss,  a  poptrary  prqpf  ss  is  to  be  ol^^rv^ : 
that  is,  the  variation  js  tp  be  called^^o^^  if  the  western  aaiimth  be  th^ 
greatest ;  but  west  if  the  eastern  azinmfh  be  th^  ^eatest.  The  v^at^qii 
also  may  be  found,  by  observing  the  points  of  the  compass  upon  which  a 
fixed  star  rises  and  set^ ;  then,  half  the  difference  between  those  points 
will  be  the  variation  of  the  compass,  as  before. 

Nof^.— The  above  met^iod  of  finding  the  variation  of  the  compass  by 
observations  of  a  circumpolar  star,  is  clearly  illustrated  in  *'  The  Young 
Navigator's  Guide  tp  the  Sidereal  and  Planetary  P^ts  of  Nautical  ^strpr 
nomy,''  between  pages  267  and  27 1  • 


PRO^LEIf  IV, 

Tojind  tJie  Variation  of  the  Compass  by  the  magnetic  Bearing  ofajixei 
SlaTf  or  Planet,  taken  at  tfie  Tim^  of  its  Transit,  pr  Passage  over  any 
known  Meridian. 

Rnd  the  apparent  time  of  the  star's  transit  or  passage  over  the  meridian 
of  the  given  place,  by  Problem  XII,,  p^ge  317  ;  but  if  the  object  selected 
for  observation  be  a  planet,  its  apparent  time  of  transit,  as  given  in  the 
Nautical  Almanac,  is  to  be  reduced  to  the  meridian  of  the  place  of 
ob&ervation,  by  Problem  XI.,  page  S15.  Let  the  watch  be  well  regulated 
to  apparent  time  under  the  meridian  of  the  given  place,  and  it  will  show 
the  instant  of  the  star's  or  planet's  transit  over  that  meridian ;  at  which 
instanjt  its  bearing,  by  asimuth  compass^  ia  to  be  carefully  taken :  the  diftr- 
f  )ice  betw^n  wb^ck  md  tbe.noidi  or  SQUtb  point  of  the  fiompats  (aefiordiog 


Digitized  by 


Google 


OF  FINDING  TRX  VAftUTIOV  69  VHB  COMPASS.  495 

to  the  hemisphere  In  whioh  the  star  may  be  posited),  wilLshow  the  deviation 
of  the  needle  from  the  true  corresponding  point  of  the  hoiizon  $  then,  if 
the  observed  or  magnetio  aaimuth  be  to  the  left  hand  qf  the  meridian,  the 
^aviation  is  easterly  y  but  if  to  the  figki  hmd,-  it  is  westerly.* 


Example  \. 

January  2d,  1885,  In  latitude  20?)0f  N.,  and  longitude  IMtaOf  S.,  at 
IP 28?  15!  apparent  time,  the  star  Canopus  was  on  the  meridian,  and 
bore,  by  azimuth  compass,  S.  9?30'  E. ;  required  the  variation  ? 

Sokdion.— The  observed  or  magnetic  bearing  of  the  star  9?30'  is  the 
variation ;  and  is  easterly,  because  it  is  to  the  left  hand  of  the  meridian. 

JExqmpk  2. 

January  1st,  1825,  in  latitude  34?25'  S.,  and  longitude  18?52'  E.,  at 
I4?8TI5 !  apparent  time,  the  planet  Jupiter  was  on  the  meridian,  and  bore, 
by  aoimuth  compass,  N.  S5f  Sd'  B. ;  required  the  variation ) 

Sobitum^^The  observed  bearing  pf  t|^f  planetj^  ?5?36^  is  the  variation; 
which  is  west,  because  the  magnetic  bearing  or  azimuth  is  to  the  right 
hand  of  the  meridian. 

The  lesr  the  altitude  of  die  star  or  planet^  and  the  greater  its  dedinatioSi 
the  more  accurately  will  the  variation  be  obtained.  When  ^e  north  polaf 
star  is  in  (be  sam^  vertical  circle  wi(h  the  star  Aliofh  or  the  9^  Cor  Caroli, 
it  will  be  on  the  meridian,  or  nearly  so ;  and  if  its  azimuth  be  observed 
at  that  time,  the  variation  will  be  pbt^ned  as  before.  If  two  stars  be 
observed  to  be  vertical,  whose  right  ascensions  are  either  equal  or  differ 
180  degrees,  they  will  be  on  the  meridian:  the  azimuth  of  either  may 
then  be  taken,  but  that  which  is  nearest  to  the  elevated  pole  should  be 
preferred ;  whence  the  variation  may  4)e  inferred,  in  the  same  manner  as 
if  i^  apparent  tin^e  of  transit  ha4  been  computed. 

*  In  like  manner  may  the  yarlatioa  h$  d$$^rfalntA  at  nopfi ;  vis.,  I^y  Qbferving  tb^ 
mapietic  bearing  of  the  sun  at  the  time  of  its  being  on  the  meridian :  and,  if  the  place 
of  ffbfepr^tiaD  If^  canai4erably  distant  fvofn  th^  equ^r^  ^  very  rig^  degree  of  accuracy  is 
no^  necessary  in  the  moment  of  observing  the  sun's  bearing  ;  since,  in  such  a  place,  an 
error  of  5  minutes  in  the  time,  before  or  after  noon,  wHl  only  produce  ao  error  of  about 
kai/a  fuarier  of  apMii  in  the  variation  $  which  comes  suftcicntly  peer  the  triftb  for  mosf 
9Atttic^  ptti|^%»f  f  If  finpe,  tfiis  piieth9d  va^y  often  prove  useful  ip  ques  where  the  fnariner 
is  prevented,  by  clouds  or  other  unavoidable  causf 8|  from  ascertaining,  fn  thf  forenoon^  the 
tnie  value  of  the  magnetic  variation. 


Digitized  by 


Google 


496  NAUTICAL  ASTRONOMY. 

The  variation  may  also  be  deduced  from  the  magnetic  azimuth  of  a 
fixed  star  at  the  apparent  time  of  its  transit  below  the  pole :  this  time  may 
be  always  known,  by  adding  12  hours,  diminished  by  halfthe.variaiian  qf 
the  mm'g  right  asceruiion  an  the  given  day,  to  the  computed  apparent  time 
of  the  star's  superior  transit  above  the  pole. 

The  number  of  brilliant  stars  which  pass  over  the  meridian  of  a  ship  at 
night,  and  the  readiness  with  which  their  respective  times  of  transit  may 
be  found,  render  the  above  method  of  finding  the  variation  of  the  compass 
at  sea  both  desirable  and  convenient  to  the  practical  navigator. 


Problem  V. 

Given  the  true  Course  between  two  Places^  and  the  Fariaiionqfthe 
Compass;  to  find  the  Magnetic  or  Compass  Course. 

Rule. 

'  When  the  variation  is  westerly,  let  it  be  allowed  to  the  right  hi^id  of  the 
true  course ;  but  when  easterly,  to  the  left  hand :  in  either  case,  the 
magnetic  or  compass  course  will  be  obtained. 

Example  1. 

Required  the  course,  per  compass,  from  Scilly  to  Cape  Gear,  the  true 
course  being  N.  52?55^  W.,  or  N.W. }  W«  nearly,  and  the  variation  2i 
points  westerly  ? 

Solution.— The  variation  2|  points,  being  allowed  to  the  right  hand  of 
the  true  course,  because  it  is  westerly,  shows  the  magnetic  course  to  be 
N.N.W.  i  W.;  which,  therefore,  is  the  course  which  a  ship  must  steer  by 
compass  from  Scilly  to  Cape  Clear,  provided  the  variation  be  a3  above. 

Example  2. 

Required  the  course,  per  compass,  from  Port  Royal,  Jamaica,  to  Santa 
Martha,  Columbia;  the  true  course  being  S.  21?42^  E.,  or  S«S£.  nearly, 
and  the  variation  about  half  a  point  easterly  ? 

Solution, — ^The  variation  i  a  point,  being  allowed  to  the  left  hand  of  the 
true  course,  because  it  is  easterly,  shows  the  magnetic  course  to  be 
S.S.E.  i  E. ;  which,  therefore,  is  the  course  which  a  ship  must  steer  by 
compass  from  Port  Royal  to  Santa  Martha,  provided  the  variation  be  as 
above,— and  independent  of  currents. 


Digitized  by 


Google 


\ 

OF  FINDING  THB  VARIATION  OF  THE  COMPASS.  497 

Problem  VI. 

Given  the  magnetic  Course,  or  that  steered  hy  Compass,  and  the 
Variation;  to  find  the  true  Course. 

Rule. 

If  the  variation  be  westerly,  it  is  to  be  allowed  to  the  left  hand  of  the 
course  steered  by  compass;  but  if  easterly,  to  the  right  hand:  in  either 
case,  the  true  course  will  be  obtained. 

Example  1. 

Let  the  magnetic,  or  course  steered  by  compass,  be  E.  by  N.  |  N.,  and 
the  variation  1^  point  westerly ;  required  the  true  course  ? 

Solution, — ^The  variation  1|  point,  being  allowed  to  the  left  hand  of  the 
compass  course,  because  it  is  west,  shows  the  true  course  to  be  N.E. 
byRJE. 

Example  2. 

Let  the  course  steered  by  compass  be  N.W.  f  W.,  and  the  variation  one 
point  and  three-quarters  easterly ;  required  the  true  course  ? 

Sobition, — ^The  variation  1|^  point,  being  allcwed  to  the  right  hand  of 
the  magnetic  or  compass  course,  because  it  is  easterly,  shows  the  true 
course  to  be  N.W.  by  N. 


AZIMUTH  COMPASS; 

The  card  being  graduated  on  an  improved  principle,  so  as  to  be  more 
particularly  adapted  to  the  taking  of  amplitudes  and  azimuths,  the  measur- 
ing of  horizontal  angles,  &c.  &c. ;  being  thus  rendered  far  more  applicable 
to  nautical  purposes  in  general  than  that  which  is  now  in  common  use  at  sea. 


The  azimuth  compass,  as  well  as  the  mariner's  compass,  is  an  artificial 
representation  of  the  horizon  of  any  place  on  the  terrestrial  globe :  it 
consists  of  a  circular  card,  divided  into  32  equal  parts,  called  points  or 
rhumbs;  and,  since   the  circle  contains  360?,  each  point  is  equal  to 

2  K 


Digitized  by 


Google 


NAUTICAt  A6TR0N0MT« 

11915' :  for  360?  h-  32  =  ll?15' .♦  The  four  principal  points  of  the 
compass,  viz.,  N,,  E.,  S,,  and  W.,  are  called  cardinal  points;  the  others 
are  compounded  of  these,  and  are  named  according  to  the  quarter  in  which 
they  ore  situated. 

To  the  under  side  of  the  card,  and  in  the  direction  of  its  north  and 
south  line,  a  bar  of  hardened  steel  is  attached,  called  the  needle,  which, 
being  touched  by  a  load-stone,  acquires  the  peculiar  property  of  pointing 
north  and  south,  and  thus  directs  the  different  points  on  the  card  to  the 
correspondent  points  of  the  horizon.  In  the  centre  of  the  needle  there  is 
a  small  socket,  by  means  of  which  it  is  placed,  with  its  attached  card,  on 
an  upright  pin  called  the  pivot  or  supporter,  which  is  fixed  in  the  bottom 
of  a  circular  or  conical  brass  box  :  on  this  pin  the  needle  turns  freely,  and, 
by  its  magnetic  property,  the  several  points  of  the  compass  card  keep 
always  in  the  same  <Urection,  very  nearly;  though  these  do  not  always 
indicate  the  true  correspondent  points  of  the  horizon,  beeause  of  the 
aberration  which  the  needle  suffers,  owing  to  that  secret  and  unknown 
agency  which  causes  its  north  and  south  poles  to  deviate  more  or  less  from 
the  respective  correspondent  poles  of  the  world. 

However,  since  the  compass  is  an  instrument  with  which  mariners  are 
well  acquainted,  it  is  not  deemed  necessary,  in  this  place,  to  enter  any 
farther  into  its  description.  Hence,  I  shall  merely  point  out  some  of  the 
many  advantages  which  a  compass  card,  graduated  on  the  above  principle, 
possesses  over  those  now  in  general  use  at  sea.  In  this  card,  the  circular 
ring  of  silvered  brass  is  to  be  sufficiently  broad  to  admit  of  four  concentric 
Bpaees*  The  outer  edge  of  the  ring  is  to  be  graduated,  nvnthematicaUy 
correcHy^  to  every  20th  minute  of  a  degree  (though,  for  vrdnt  of  room,  the 
present  card  is  only  graduated  to  every  30th  minute  of  a  decree),  to  which 
a  vernier  is  to  be  adapted,  containing  20  divisions  on  each  side  of  its  nonius 
for  the  purpose  of  subdividing  the  divisions  on  the  card  into  minutes  of  a 
degree. 

The  interior  surface  of  the  vernier  should  be  ground  concave  to  the 
segment  of  a  circle,  whose  radius  is  equal  to  that  of  the  card.  The  remote 
edge  of  the  inner  concentric  space,  on  the  silvered  brass  flat  ring,  may  be 
graduated  similarly  to  that  of  the  outer  edge,  so  as  to  render  it  more 
convenient  in  reading  off  amplitudes  according  as  they  may  be  reckoned 
from  the  prime  vertical,  or  from  the  meridian. 

The  first  space  on  the  broad  ring  of  silvered  brass,  viz.,  that  next  the 
points  of  the  compass,  is  particularly  adapted  to  taking  amplitudes  when 
the  observations  are  reckoned  from  the  east  or  the  west  points  of  the 


*  Tabki  XXXIII.  contains  the  different  ang^les  which  ereiy  point  and  quniter^peiDt'of 
the  compass  makes  with  the  meridian;  and  Table  XXXiV.  contains  tfaeiogaithaucaiBflB> 
taofent^  and  secants  of  ev€i;y  point  and  quartor-poiAt  of  the  compass* 


Digitized  by 


Google 


OF  FINDING  THB  YABIATIOM  OP  THE  COMPASS.  499 

horizon;  and^  therefore,  it  b  numbered  both  ways,  from  those  points, 
towards  the  meridian :  that  is,  from  0?  to  90?.  The  second  space  being 
adapted  to  Jiorizontal  azimuihSf  viz.,  to  amplitudes  reckoned  from  the 
meridian,  is  therefore  numbered  both  ways,  from  the  north  and  south 
points  of  the  horizon  towards  the  east  and  west  points  thereof:  that 
is,  from  0?  to  90?,  in  a  contrary  order  to  the  last.  The  third  space  is 
intended  for  the  accommodation  of  an  azimuth  when  the  obserratioQ  is 
reckoned  from  the  south  in  north  latitude,  or  from  the  south  la  south 
latitude :  hence,  it  is  numbered  both  ways  from  the  south  to  the  north 
point  of  the  compass,  or  from  0?  to  180?.  The  fourth,  or  outer  space,  is 
designed  for  azimuths  reckoned  from  the  north  in  north  latitude,  or  from 
the  north  in  south  latitude,  according  to  the  will  of  the  observer}  an^ 
therefore,  it  is  numbered  both  ways  from  the  north  to  the  south^  or  from 
0?  to  180?,  &c.— See  the  Frontispiece  to  this  volume. 

Besides  the  evident  uses  of  a  compass  card,  graduated  after  this  maimer^ 
in  observing  amplitudes  and  azimuths,  it  will  also  be  found  of  the  greatest 
utility  in  taking  correct  surveys  of  coasts  and  harbours,  and  in  settling  the 
true  positions  of  places  on  shore  from  a  knoum  position  at  sea.  It  may, 
moreover,  be  applied  successfully  to  many  astronomical  purposes ;  nay,  it 
may  even  be  applied  to  the  determination  of  the  longitude  by  Imar 
observations,  as  thus :— Let  two  observers,  with  two  good  compasses  of 
the  above  description,  take  the  azimuths  of  the  moon  and  sun,  or  a  fixed 
star,  &c.,  at  the  same  instant ;  then,  if  those  two  azimuths  be  reckoned 
from  the  same  point  of  the  horizon,  their  sum,  subtracted  from  360?,  will 
be  the  angle  at  the  zenith  comprehended  between  the  zenith  distances  ci 
the  objects ;  with  which,  and  the  true  zenith  distances  of  the  objects,  the 
true  central  distance  may  be  found  by  oblique  angled  spherical  trigoao* 
metry,  PirobTem  III.,  Remark  I  or  2,  page  203  or  204 ;  and,  hence,  the 
longitude  of  the  place  of  observation,  by  Problem  VIIL,  page  454. 

An  azimuth  compass  of  this  description  would  be  of  real  advantage  to 
the  practical  navigator ;  whereas,  the  one  now  in  common  use  at  sea  is  so 
very  ill  adapted  to  the  important  purposes  for  which  it  is  designed,  that 
it  is  very  seldom  resorted  to  for  those  purposes;  and,  therefore,  it  is 
scarcely  ever  seen  upon  deck,  except  for  the  simple  purpose  of  comparing 
its  parallelism  with  that  of  the  binnacle,  or  steering  compass. 


2k2 

Digitized  by  VjOOQ IC 


500  NAUTICAL  ASTROMOMT. 

SOLUTION  OP  PROBLEMS  RELATIVE  TO  PINDING  THE 

APPARENT  TIMES  OF  THE  RISING  AND  SETTING 

OF  THE  CELESTIAL  BODIES. 

Problem  L 

Given  the  Day  of  the  Mmth,  the  Latitude  of  a  Place,  and  the  Height  of 
the  Eye  above  the  Level  of  the  Horizon :  to  find  the  apparent  Times  qf 
the  &m*s  Rising  and  Setting. 

RULB. 

Let  the  sun's  declination^  at  noon  of  the  given  day^  be  reduced  to  the 
meridian  of  the  given  place,  by  Problem  V.,  page  298 ;  then,  to  the 
logarithmic  tangent  of  this  reduced  declination,  add  the  logarithmic 
tangent  of  the  latitude ;  and  the  sum  (abating  10  in  the  index,)  will  be 
the  logarithmic  co-sine  of  an  arch ;  which,  being  converted  into  time,  will 
be  the  approximate  time  of  the  sun's  rising,  and  its  supplement  to  12 
hours  will  be  that  of  the  sun's  setting,  the  latitude  and  the  declination 
being  of  the  same  name ;  but  if  these  elements  be  of  contrary  names,  the 
above  arch,  reduced  into  time,  will  be  the  approximate  time  of  the  sun's 
setting,  and  its  complement  to  12  hours  that  of  the  sun's  rising. 

Reduce  the  approximate  times  of  rising .  and  setting,  thus  found,  to  the 
correspondent  times  at  Greenwich,  by  Problem  III.,  page  297 ,  to  which 
times,  respectively,  let  th6  sun*s  declination  be  reduced,  by  Problem  V«, 
page  298;  then, 

To  the  aggregate  of  90  degrees,*  the  horizontal  refraction,t  and  the 
dip  of  the  horizon,  diminished  by  the  sun's  horizontal  parallax,^  add  the 
sun's  polar  distance,  and  the  co-latitude  of  the  place  of  observation  :  take 
half  the  sum  ;^  the  difference  between  which  and  the  first  term,  call  the 
remainder. 

Now,  to  the  logarithmic  co-secants,  less  radius,  of  the  polar  distance, 
and  the  co-latitude,  add  the  logarithmic  sines  of  the  half  sum,  and  of  the 
remainder :  half  the  sum  of  these  four  logarithms  will  be  the  logarithmic 
sine  of  an  arch ;  which,  being  doubled,  and  converted  into  time,  will  be 
the  apparent  time  of  the  sun's  rising.  In  the  same  manner  the  apparent 
time  of  the  sun's  setting  is  to  be  computed ;  but,  in  this  case,  the  half  sum 
of  the  four  logarithms  is  to  be  considered  as  a  logarithmic  co-sine. 

Example  1. 
Required  the  apparent  times  of  the  sun's  rising  and  setting,  July  13th, 
1824,  in  latitude  50?48'  N.,  and  longitude  120?  W.,  the  height  of  the  eye 
above  the  level  of  the  sea  being  30  feet  ? 

«  The  sun's  distance  from  the  zenith  when  hU  centre  is  in  the  horizon, 
t  The  horizontal  refraction  of  a  celestial  object  is  33  minutes  of  a  de|^e. 
t  The  sun's  horiaontal  parallax  is  about  9  seconds. 


Digitized  by 


Google 


APPARENT  TIMB  OF  RISING  OR  SETTING  OF  A  CBLBSTIAL  OBJECT.  501 

Sun's  dec.  at  noon,  July  13th,  =  2IM9^5HN. 
Con  of  do.  for  long.  120?  west  =     --  2. 59 

Sun's  reduced  declination     =     21  ?46'.52rN.  Log.  tangent=  9.601613 
Lat.  of  the  given  place  =      .     50.48.   0  N.  Log.  tangents  10. 088533 

Arch= 60^39  U7r       Log.  co-sine  =  9.690146 

Approx.  time  of  sun's  rising  =s     4 1  2T39 ',  Appr.  time  ©'s  set  7  ?  57  ?2 1 ! 

To  find  the  apparent  Time  of  the  Sun's  Rising :—         c.       '^ 

Approximate  time  of  the  sun's  rising   =       4*  2T39f 
Longitude  120?  W.,  in  time  =s       .    .     +  8.   0.   0 

Greenwich  time  past  noon  of  given  day  =     Ot  2?39! 

Sun's  declination  at  noon,  July  13th,  =     21?49'5irN. 
Correction  of  ditto  for  0?2?89!  =    .     .       —  0.    1 


Sun's  dec.,  reduced  to  Greenwich  time  =s  21?49'50fN. 

Sun's  north  polar  distance  =    .    •    •    .    68?  10. 10? 

90?  +  33^  +  5'.15r  -  9r  =  90?38:  6? 

Sun's  polar  distance  =     .    .    68. 10. 10      Log,  co-secant=  0, 032317 

.    39.12.   0      Log.  co-secantss  0. 199263 


Co-latitude  = 
Sums 


•     •    • 


Half  sum  =  • 
Remainder  ^ 


198?  on6r 


99?  0<  8r      Log.  sine  =.    9.994617 
8.22.   2        Log. sines.    9.162914 


Sum  =  19.389111 


Archs 29?39^57i?    Log. sine  ==      .    9.694555^ 

Arch  doubled  =  .    .    .    .    59?19^55r  =  3J57?20t  j  which,  therefore, 
is  the  apparent  time  of  the  sun's  rising. 

To  find  the  apparent  Time  of  the  Sun's  Setting :~ 

Approximate  time  of  the  sun's  setting  =     7^57*21! 
Longitude  120?  W.,  in  time  ==:  .    .    .     +  8.   0.   0 

Greenwich  time  past  noop  of  given  day  =5  15  f  57T21 J 

Digitized  by  VjOOQ IC 


-      K' 


508  NAtmCAL  ABTROKOMY. 

Sun's  declination  at  noon,  July  13th^  s     2 1  ?49 1 5 KN. 
Correction  of  ditto  for  15!57"2n  =s     .       —  5. 58 


Sun's  dec.^  reduced  to  Greenwich  time  =s    21?43^53fN. 


Sun's  north  polar  distance  =s  ....    68?  16'  Ti 
90?  +  33^  +  5n5r  —  9r  =  90?38'  6? 

Sun's  polar  distance  s     .    .    68. 16.   7       Log.  co-secant^  0. 032017 

•    39.12.   0      Log.  co-secant=  0. 199263 


Co-latitude  = 
Sum  =     •    • 

Half  sum  =s 
Remainder  b 


198?  6n3r 

-' 

99?  S:  6jr  Log.  sine  =3    .    9.994558 
8,25.  Oi    Log. sines:    .    9.165461 


Sums  19.391299 


Arch= 60?15^  6r      Log.  co-sine  =     9.695649$ 

Arch  doubled  =3      .    .     .  120?30^  12r  =  8*2?  1 !  j  which,  therefore,  is 
the  apparent  Ume  of  the  sun's  setting. 

Example  2. 
Required  the  apparent  times  of  the  sun's  rising  and  setting,  October  1st, 
1824,  in  latitude  40?30^  N.,  and  longitude  105?  E. ;  the  height  of  the  eye 
aboye  the  level  of  the  sea  being  29  feet  ? 
Son's  declination  at  noon,  Oct.  lst,=:3?  16^  6rS. 
Correc.  of  ditto  for  long.  105?  E.  =  -  6. 48 

Sun's  reduced  declination  =      •      3?  9M8rS.   Log.tang.s:  8.741316 
Latitndeof  the  given  place  ==  •    40.30.  0  N.  Log.tang.=  9.931499 

Ai«h  m     • 87?18f  6r      Log.  co^sine  8.672815 

Appr^Xf  time  of  the  sun's  setting  =    5 149712! Appr.time  O'sris.6M0T48! 

To  find  the  apparent  Time  of  the  Sun's  Setting  :— 
Approximate  time  of  sun's  setting  »    .    .    .      5  *  49?  1 2 ! 
Longitude  105?  B.,  in  time  »»....-  7.  0.  0 

Greenwich  time  past  noon,  September  30th,  s  22t49?12! 

Sun's  declination  at  noon,  September  30th^    =r     2?52U6?  S. 
Correction  of  ditto  for  22U9?12!  s      .  v .       +22. 11 


Sun's  declination,  reduced  to  Greenwich  time  :&  3?  14C57?  S. 


Sun's  north  polar  distance  as      •    .    •    •    •    98?  14^57? 

/Google 


Digitized  by ' 


APPARBNT  TIMB  OF  RISING  OR  SITTING  OF  A  CELESTIAL  OBJECT.  S03 

90?  +  381  +  snor  -  91ss  90988^   K 


Sun's  polar  distance  = 
Co-latituda  c3    •    •    • 


Sum  = 


•    •    •     • 


Half  sum  =  . 
Remainder  = 


98. 14. 57      Log.  co-secantsO.  000699 
49.30.  0      Log.  co-8ecaat«0«  U8954 


233^22158r 


116Ml^29r    Log.  sine  =  .    9.951065 
26,  3,28      Log.  sine  =   .    9.642739 

Sum=  19.713457 


Archs= 44?  IMl^?  Log.  co-sine  =    9.856728| 

Arch  doubled  =     ....    88?  3^23?  =  5*52?13i! ;  which,  there- 
fore^ is  the  apparent  time  of  the  sun's  setting. 

To  find  the  apparent  Time  of  the  Sun'sfiising  :— 

Approximate  time  of  sun's  rising  =     .    ,     •       6M0T48? 
Longitude  105?  E.,  in  time  =     ....      —  7*   0.   0 


Greenwich  time  past  noon^  September  SOth^s:  1 1 !  10?48! 

Sun's  declination  at  noon,  September  30th,  a     2?52'46f  S. 
Correction  of  ditto  for  UU0T48!  ss   .    .    «       +10.52 


Sun's  decIinaUon,  reduced  to  Greenwich  time,  ==  3?  3'38?  S. 


Sun's  north  polar  distance  = 93?  3^38r 

90?  +  331  +  5'lOr  -  9?  =  90?38'   1? 

Sun's  polar  distance  s     .    •    93.   3.38      Log.  eo-secantssO.  000620 

Co-latitudes 49.30.   0      Log.  co-secant sO.  118954 

Sum»    ..<...    «  23S?1U39? 


Half  sum  = 116?35U9i?  Log.  sine  »    ,    9.951423 

Remainders 25.57.48^    Log.  sine  =    .    9.641274 


Sum=  19.712271 


Archas 45?53<28r    Log.  sine  a   «    0«  8561351 

Arch  doubled  =      ....    91?46^56f  =  6*5T8! ;  which,  therefore, 
is  the  apparent  time  of  the  sun's  rising. 

See  Examples  1  and  2,  page  125j  and,  also,  the  Example,  pagesl26,1274 

Digitized  by  LjOOQ IC 


504  NAUTICAL   A8TROKOMT* 

Remark, — ^If  the  equated  or  mean  times  of  the  sun's  rising  and  setting 
be  required,  then,  to  the  apparent  times,  found  as  above,  apply  the  reduced 
equation  of  time,  as  directed  in  Problem  I.,  page  415 ;  and  the  result  will 
be  the  mean  times  of  that  object's  rising  and  setting. 


Probijbm  IL 

Given  the  Latitude  of  a  Place,  and  the  Height  of  the  Eye  above  the  Level 
of  the  Horixon  ;  to  find  the  apparent  Times  of  the  Rising  and  the  Setting 
of  a  fixed  Star. 

RtJLB. 

Compute  the  apparsnt  time  of  the  star's  transit,  or  passage  over  the 
meridian  of  the  given  place,  by  Problem  XII.,  page  317 ;  then. 

To  the  aggregate  of  90  degrees,*  the  horizontal  refraction,t  and  the 
dip  of  the  horizon,  add  the  star's  polar  distance  and  the  co-latitude  of  the 
place  of  observation :  take  half  the  sum ;  the  difference  between  which  and 
the  first  term,  call  the  remainder. 

Now,  to  the  logarithmic  co-secants,  less  radius,  of  the  polar  distance, 
and  the  co-latitude,  add  the  logarithmic  sines  of  the  half  sum  and  of  the 
reminder :  half  the  sum  of  these  four  logarithms  will  be  the  logarithmic 
co-sine  of  an  arch ;  which,  being  doubled  and  converted  into  time,  will  be 
the  star's  semi-diurnal  arc,  or  half  the  time  of  its  continuance  above  the 
horizon ;  which  is  to  be  reduced  to  apparent  solar  time,  by  subtracting 
therefrom  the  proportional  part  corresponding  to  it  and  the  variation  of 
the  sun's  right  ascension  for  the  given  day :  this  is  done  by  Problem  V., 
page  298.  Now,  the  apparent  semi-diurnal  arc,  thus  found,  being  applied 
by  subtraction  and  addition  to  the  apparent  time  of  the  star's  transit  over 
the  given  meridian,  will  give  the  respective  apparent  times  of  its  rising  and 
setting  at  that  meridian :  these  may  be  reduced  to  the  mean  times  of  rising 
and  settings  by  Problem  L,  page  415. 

Example  1. 

'  Required  the  apparent  times  of  the  rising  and  setting  of  the  star  «  Arietis, 
January  1st,  1824,  in  latitude  50^48' N.,  and  longitude  30?0!  E. ;  the 
height  of  the  eye  above  the  level  of  the  sea  being  16  feet  ? 


*  This  is  the  star's  distance  from  the  zenith  when  its  centre  is  in  the  horizon, 
t  The  horizontal  refraction  of  a  celestial,object  is  33  minutes  of  a  degree«  The  fixed  start 
have  no  sensible  paraUaxt 


Digitized  by 


Google 


APPARBNT  TIMB  OP  RISING  OR  SETTING  OP  A  CELESTIAL  OBJECl*.    505 

*'8dec,,red.togivenday,  22?S7C33^N.,&it8R^.=  l  JSr^lG! 
Sun's  right  ascension  at  noon  of  the  given  day  s=       18. 43. 58 


Approx.  time  of  star's  transit  over  the  meridian  =     7*13?18'..  7M3T18! 
Longitude  of  the  given  place  30?0^  E.^  in  times  —  2.   0«   0 

Corresponding  time  at  Greenwich  =      •    •     •     •    5M3?18! 
Reduc.oftrans.aiis.  to  5M3T18!&4T24!  the  var.  of  sun's  R.A.s  -  0.57 


Apparent  time  of  star's  transit  over  merid.  of  the  given  place  ss  7^  12721 ! 

90  degrees  +  33^  +  3^50r  =  90?38^50r 

Star's  north  polar  distance  =    67. 22.  27      Log.  co-secant=0. 034781 

Co-latitude  of  the  given  place=39. 12.   0      Log.  co-secant=0. 199263 


Sum  = 197?13n7C 


Half  sum  = 98?36:38jr  Log.  sine  =    .    9.995077 

Remainders 7«57.48i    Log.  sine  =    •     9.141580 


Sum  =19.370701 


Arch= 61?  0C58r     Log.  co-sine  =    9.685350^ 

Star's  semi-diurnal  arc  =      .  122?   lC56r,  in  time  =     .     .       8?  8?  8! 
Prop,  part  of  variation  of  sun's  R.  A.  4T24  ?  ans.  to  8^  8?8 !  =      —   1 .  30 

Star's  apparent  semi-diurnal  arc  = 8i  6T38! 

Apparent  time  of  the  star's  transit  over  the  given  meridian  =     7«  12. 21 

Apparent  time  of  the  star's  rising  past  noon,  Dec.  3Isty  1823^=:  23  *  5?43 ' 
Apparent  time  of  the  star's  setting  past  noon  of  the  given  day=15M8T59! 

Example  2. 

Required  the  apparent  times  of  the  rising  and  setting  of  the  star  Sinus, 
January  1st,  1824,  in  latitude  40?30:  N.,  and  longitude  120?01  W.;  the 
height  of  the  eye  above  the  level  of  the  horizon  being  46  feet  ? 

*'8dcc.,red.togivenday,16?28^53^S.,&it8R.A.=6*37?23! 
Sun's  right  ascension  at  noon  of  the  given  day  =     1 8. 43. 58 

Approximate  time  ofstar's  transit  over  the  merid.=  1H53T25!   Ilt53t25! 
Long,  of  the  place  of  observ.  120?W.,  in  time=:  4-  8.   0.   0 

Corresponding  time  at  Greenwich  =    .     .     •     .     19* 53? 25! 
Reduc.oftrans.ans.tol9?53r25!&4T24!thevar.ofsun'sR.A.=  -  3r39! 


Apparent  time  of  the  star's  transit  over  the  given  meridian  =    1 1  *49T46 ! 

/Google 


Digitized  by ' 


506  NAUTICAL  ASTRONOMY. 

90  degrees  +  33J  +  6C30r  a  90?39f30? 

Star's  north  polar  distance  »  106. 26. 53       Log.  co^secants  0. 018220 

Co-latitude  of  the  place  =    •    49.30.   0      Log.  co-8ecant=:  0.118954 

Sum= 246?38^23r 


Half  sum  = 123?  19Ul|r  Log.  sine  »      .    9.922008 

Remainders 32.39.41§    Log.  sine  a      .    9.732132 


Sum  ;r=  19.791314 


Arch  = 38?  8^501'/  Log.  co-sine  =s     9.895657 

Star's  semi-diurnal  arc  2=      .     76 ?  1 7 '  4 11^,  in  time  =    .     .      5  *  5  T 1 1 ! 
Prop,  part  of  var.  of  sun's  R.  A.  4  ?24 !  answering  to  5  ?5  T II !  =    —  0. 56 


Star's  apparent  semi- diurnal  arc  = 5?  4?15! 

Apparent  time  of  the  star's  transit  over  the  given  meridian  =   1 1. 49. 46 


Apparent  time  of  the  star's  rising  past  noon  of  the  given  day  =  6^45T31 ! 
Apparent  time  of  the  star's  setting  past  noon  of  the  given  day=  16^54?  1 ! 

See  Example  2^  page  129 ;  and^  also^  the  Example^  page  130. 


Problem  IlL 

Ghen  the  LatUude  of  a  Place,  and  the  Height  of  the  Eye  above  the 
Level  of  the  Horixon  ;  to  find  the  apparent  Times  qfa  Planet's  Bisuig 
and  Setting. 

RULB. 

Compute  the  apparent  time  of  the  planet's  transit  over  the  meridian  of 
the  given  place,  by  Problems  X.  and  XI.,  pages  313  and  315  ;  reduce  Ais 
time  to  the  meridian  of  Greenwich,  by  Problem  IIL,  page  297 ;  to  which 
let  the  planet's  declination  be  reduced,  by  Problem  VII.,  page  307  J  then. 

To  the  logarithmic  tangent  of  the  latitude  add  the  logarithmic  tangent 
of  the  planet's  reduced  declination,  and  the  sum  (abating  10  in  the  index,) 
will  be  the  logarithmic  sine  of  an  arch ;  which,  being  converted  into  time, 
and  added  to  6  hours  when  the  latitude  and  the  declination  are  of  the  same 
name,  but  subtracted  from  6  hours  when  of  contrary  names,  the  sum  or 
difference  will  be  the  planet's  approximate  semi-diurnal  arc^  or  half  the 
time  of  its  continuance  above  the  horizon. 


Digitized  by 


Google 


APPARENT  TIME  OF  RtSISG  OR  SETTING  OF  A  CELESTIAL  OBJECT.  507 

Liet  this  time  be  applied^  by  subtraction  and  addition,  to  the  apparent 
time  of  transit ;  and  the  approximate  times  of  the  planet's  rising  and  setting 
will  be  obtained. 

Reduce  the  approximate  times  of  rising  and  setting,  thus  found,  to  the 
correspondent  times  at  Greenwich,  by  Problem  IIL,  page  297  j  to  which 
times,  respectively,  let  the  planet's  declination  be  reduced,  by  Problem. 
VII.,  page  307;  then. 

To  the  aggregate  of  90  degrees,*  the  horizontal  refraction,t  and  tiie  dip 
of  the  horizon,  diminished  by  the  planet's  horizontal  parallax,^  add  the 
planet's  polar  distance  at  the  approximate  time  of  rising  or  setting,  and 
the  co-latitude  of  the  ^ven  place  :  take  half  the  sum;  the  difference 
between  which  and  the  first  term,  call  the  remainder. 

Now,  to  the  logarithmic  co-secants,  less  radius,  of  the  polar  distance 
and  the  co-latitude,  add  the  logarithmic  sines  of  the  half  sum  and  of  the 
remainder  :  half  the  sum  of  these  four  logarithms  will  be  the  logarithmic 
co-sine  of  an  arch ;  which,  being  doubled  and  converted  into  time,  will  be 
half  the  time  of  the  planet's  continuance  above  the  horizon,  or  its  semi- 
diurnal arc. 

Find  the  proportional  part  of  the  variation  of  the  planet's  transit  over 
the  meridian,  answering  to  half  its  continuance  above  the  horizon,  by 
Problem  XI.,  page  315,  in  the  same  manner  as  if  it  were  the  reduction  of 
transit  to  a  different  meridian  that  was  under  consideration.  Now,  this 
proportional  part  being  added  to  half  the  time  of  the  planet's  continuance 
above  the  horizon  when  the  planet's  transit  is  increasing,  but  subtracted 
therefrom  when  decreasing,  the  sum  or  difference  will  be  the  planet's 
apparent  semi-diurnal  arc ;  which  being  applied  by  subtraction  to  the 
apparent  time  of  transit,  the  remainder  will  be  the  apparent  time  of  the 
planet's  rising.  In  the  same  manner  let  the  apparent  semi-diurnal  arc  for 
the  time  of  setting  be  computed ;  which,  being  added  to  the  apparent  time 
of  transit,  will  give  the  apparent  time  of  the  planet's  setting.  The  apparent 
times  of  rising  and  setting,  thus  found,  may  be  reduced  to  the  mean 
times  of  rising  and  setting,  if  necessary,  by  Problem  I.^^  page  415. 

Example  1. 

Required  the  apparent  times  of  Jupiter's  rising  and  setting,  January 
4th,  18^4,  in  latitude  36*?  N.,  and  longitude  135?  W.j  tiie  height  of  the 
eye  above  the  level  of  the  horizon  being  23  feet  ? 

The  apparent  time  of  Jupiter's  transit  over  the  meridian  of  the  given' 


*  This  is  the  zenith  distance  of  a  planet  when  its  centre  is  in  the  horizon, 
t  The  horizontal  refraction  of  a  celestial  object  is  33  minutes. 
X  For  the  parallaxes  of  the  planets,  see  pa^e  326. 


Digitized  by 


Google 


508  NAimCAL  ASTRONOBCY* 

place  18  1I!20T30!  ;  his  declination^  being  reduced  to  this  time  and  the 
given  longitude^  is  23?  18^55?  north. 

Latitude  of  the  given  place  =  36?  0^  O^N.    Log.  tangent  =  9. 861261 
Jupiter's  reduced  declination=23.  18. 55  N.    Log.  tangent  =  9.634461 

Arch= 18?14152i:    Log.  sine  =     .    •    9.495722 

Arch,  converted  into  time^  =     1  ^  12T59 ! ;  this^  being  added  to 

6  hours^  gives  the  approximate  semi-diurnal  arc  =  •■  •  7- 12T59! 
Apparent  time  of  Jupiter's  transit  over  the  given  meridians  •     11. 20. 30 

Approximate  time  of  Jupiter's  rising  =.••-..••      4^  7*31'. 
Approximate  time  of  Jupiter's  setting  == 18?33?29! 

The  longitude^  in  time,  being  applied  to  those  times  by  addition,  because 
it  is  west,  shows  the  approximate  time  of  the  planet's  rising  at  Greenwich 
to  be  13*7*31 '  past  noon  of  the  given  day,  and  that  of  its  setting  3^33729! 
past  noon,  January  5th«  The  declination  reduced  to  these  times  respectively, 
is  23?18:46rN.  at  the  time  of  rising,  and  23?I9UrN.  at  the  time  of 
setting. 

To  find  the  apparent  Time  of  Rising  :— 

90?  +  33r-»-  4^36r  «  2r  =  90?37'34r 

Jupiter's  polar  distance  =s     •    66.41.14     Log.  co-secant  ==  0.036988 

Co-latitude  of  the  place  3=  .    54.   0.  0    Log.  co-secant  =  0.092042 

Sum=: 211?18'48r 

Half  sum  =  ......  105?39:24r  Log.  sine  =  .    .    9.983580 

Remainder  =     •    .    •    •    •     15.    1.50    Log.  sine  =  .-    .     9.413860 

Sum  =  19.526470 

Archs 54 ?34 ^  3r  Log.  co-sine  a  .    9.763235 

Semi-diurnal  arc  =     .    .    .  109?  8^  6r,  in  time  =     .    .    7M6T32! 
Variation  of  transits=30?  decreasing;  the  proportional  part  of 

which,  answering  to  7*  16r32^  is  = -1.31* 

Apparent  semi-diurnal  arc  = 7M5?  1! 

Apparent  time  of  Jupiter*s  transit  = 1 1 .  20. 30 

Apparent  time  of  Jupiter's  rising  =  ' 4t  5T29! 

*  If  the  transit  had  beeo  pro^^ressivei  or  increaslDi^y  the  proportional  part  would  be 
addUivtM 


Digitized  by 


Google 


APPARBKT  TIME  OF  RISING  OR  S£TTINQ  OP  A  CBLBSTUL  OBJECT.  509 

To  find  the  apparent  Time  of  Setting  :— 

90?  +  33<  +  4^36r  -  2r  =  90?37^24r 

Jupitcr'i  polar  distance  =     .    66. 40. 56     Log.  co-secant  =:  0.  OS7004 

Co-latitude  of  the  place  =s     .    54.   0.   0    Log.  co-secant  =  0.092042 

Sums 211?18'.20r 


Half  sum  s 105?39n0r  Log.  sine  =  .    .    9.983588 

Remainders 15.    1.46    Log.  sine  =  .    .    9.413829 

Sums  19.526463 


Arch  s 54?34^  5r  Log.  co-sine  s     9.763231} 

Semi-diurnal  arc  =      .     .     .  109?  8:iOr,  in  time  =  .     .     7*16?33! 
Variation  of  transit  =  30?,  decreasing;  the  proportional  part 

ofwhich,  answering  to  7 M6T33!,  is  = —   l?3l!» 


Apparent  semi'-diumal  arc  s 7M5T2! 

Apparent  time  of  Jupiter's  transit  = •    •     •  1 1 .  20. 30 


Apparent  time  of  Jupiter's  setting  == 18?35?32! 

Example  2. 

Required  the  apparent  times  of  the  rising  and  setting  of  the  planet  Mars, 
January  16th,.  1824,  in  latitude  40?  N.,  and  longitude  140?  E.;  the  height 
of  the  eye  above  the  level  of  the  sea  being  26  feet  ? 

The  apparent  time  of  Mars'  transit  over  the  meridian  of  the  given  place 
is  16M3T37^  ;  now,  his  declination,  being  reduced  to  this  time  and  the 
given  longitude,  is  0?55M5?  south. 

Latitude  of  the  given  place=40?  0^  OrN.     Log.  tangent  =    9.923814 
Mars' reduced  declination  =    0.55.45  S«     Log.  tangent  =    8.210009 

Arch= 0?46M71'        Log.  sine  =     .    8.133823 


Arch,  converted  into  time  =     0^  3?  7  •  ;  this,  being  subtracted 

from  6  hours,  leaves  the  approximate  semi-diurnal  arc  =       5756T53! 
Apparent  time  of  Mars'  transit  over  the  given  meridian  =     •     16. 43. 37 

Approximate  time  of  Mars' rising  = 10M6T44! 

Approximate  time  of  Mars' setting  s 22*40T30! 


•  See  Note,  pa^  508. 


Digitized  by 


Google 


51.0  NAUTiCAX.  AST&ONOMT. 

The  longitude,  in  time,  being  applied  to  those  times  by  subtraction, 
because  it  is  east,  shows  the  approximate  time  of  the  planet's  rising  at 
Greenwich  to  be  1*26T44!  past  noon  of  the  given  day,  and  that  of  its 
setting  13*20T30!  past  the  same  noon.  The  planet's  declination  reduced 
to  these  times,  respectively,  is  0?54^20?S.  at  the  time  of  risix^,  and 
0?57'9^  S.  at  the  time  of  setting. 

To  find  the  Planet's  apparent  Time  of  Rising  :— 

90?  +  38:  +  4^52*  -  lOr  =r  90?37M2r 

Planet's  polar  distance  =      •    90. 54. 20      Log.  co-secant=:0. 000054 

Co- latitude  of  the  place  =     .    50.   0.   0       Log.  co-secant=:0. 115746 

Sum  =: 231?32^  21 

Half  sum  =: I15?46:   It    Log.  sine  =  •    9.954517 

Remainder  =: 25?  8'19r    Log.  sine  =  .    9.628195 

Sum=  19.698512 

Aiths:     ..,,,,.    45?  VA9t    Log.  oo-sim  =  9. 849256 

Semi-diiimal  arc  =:      .    .     .    90?  3C3dr,  in  times    •    .    6^0715! 
Variation  of  transit  =:  20T,  decreasing  ;  the  proportional  part 

of  which,  answering  to  6*0?  15!,  is  = —0.50* 

Apparent  semi-diurnal  arc  = 5t59r^! 

Apparent  time  of  transit  =: •    •    •    •    *  16. 43. 37 

•    ■  ■  ■     ■■ 
Apparent  time  of  Mars' rising  = 10!44rl2! 

To  find  the  Planet's  apparent  Time  of  Setting  v^ 

90?  +  33'.  +  4:52r  -  lOr  =  90?37'42r 

Planet's  polar  distance  =:       .    90. 57.   9      Log.  co-seeant=0. 000060 

Co-latitude  of  the  place  ==    .    50.   0.   0      Log.  co-secant=:0. 1 15746 

Sum= 231?34^5K 

Half  sum  =.    .    ;    ,    •    .  115?47'.25ir  Log.  sine  =:    .    9.954432 

Remainder  = 25?  9 M3i?  Log.  sine  =5    .    9.628573 

Sums  19.698811 
Arch=:     .......     45?  0^38r     Log.  co-sine  =:    9.8494051 

•  SeeNo«e>pa{:e508. 

Digitized  by  VjOOQ IC 


APPABBNT  TIMB  OF  ItlSlNO  OA  SETTING  09  A  CBJLBSTIAL  OBJECT.  Sll 

Arch= 45?  0'.38i: 

Semi-diurnal  arc  =     ...    90?   i:  16'/,  in  time  =   .    •    6?  0?  5! 
Variation  of  transit  =:  20?,  decreasing;  the  proportional  part 

of  which,  answering  to  6  tOT5!,  is  =z —  0.50  * 

Apparent  semi-diurnal  arc  =: 5^59^15! 

Apparent  time  of  transit  =  • 16.43.37 

Apparent  time  of  Mars' setting  = 22*42?52! 


^       Problem  IV. 

Given  the  Latitude  of  a  Place,  and  the  Height  of  the  Eye  above  the 
,  Level  of  the  Horizon  ;  to  find  the  apparent  Times  of  the  Moon's 
Bising  and  Setting. 

Rule. 

Compute  the  apparent  time  of  the  moon's  transit  over  the  meridian  of 
the  given  place,  by  Problems  Vill.  and  IX.,  pages  309  and  312;  and 
reduce  it  to  the  meridian  of  Greenwich,  by  Problem  IIL,  page  297 )  to 
which  let  the  moon's  declination  and  horizontal  parallax  be  reduced,  by 
Problem  VI.,  page  302 ;  then. 

To  90  degrees,  diminished  by  the  difference  between  the  moon's  hori- 
zontal parallax  and  the  sum  of  the  horizontal  refraction  and  the  dip  of  the 
horizon,  add  the  moon's  polar  distance  and  the  co-latitude  of  the  giveB 
place.  Find  the  difference  between  half  the  sum  and  the  first  term,  which 
call  the  remainder. 

Now,  to  the  logarithmic  co-«secants,  less  radius,  of  the  polar  distance, 
and  the  co4atitude,  add  the  logarithmic  sines  of  the  half  sum  and  of  the 
remainder:  half  the  sum  of  these  four  logarithms  will  be  the  logarithmic 
co-sine  of  an  arch;  Which,  being  doubled,  and  converted  into  time,  will 
be  the  moon's  approximate  semi-diamal  arc  :  this  being  subtracted  from 
and  added  to  the  apparent  time  of  the  moon's  transit,  the  respective 
approximate  times  of  her  rising  and  setting  will  be  obtained. 

Reduce  the  approximate  times  of  the  moon's  rising  and  setting,  thm 
found,  to  the  correspondent  times  at  Greenwich,  by  Problem  III.,  page 
297 ;  to  which  times,  respectively,  let  the  moon's  declination  and  hori- 
zontal parallax  be  reduced,  by  Problem  VI.,  page  302;  and  let  the  moon's 

•  See  Nolt,  imge  909. 

Digitized  by  VjOOQ IC 


512  NAUTICAL  ASTRONOMY. 

declination^  at  each  tioie^  be  corrected  by  the  equation  of  second  difference; 
then, 

M^th  90  degrees^  ditnimshed  as  before,  the  moon's  respective  polar 
distances,  and  the  co-latitude,  compute  the  approximate  semi-diurnal  arcs 
corresponding  to  the  times  of  rising  and  setting. 

Find  the  proportional  part  of  the  daily  variation  of  the  moon's  transit 
answering  to  each  semi^diurtidl  arCy  and  24  hours  augmented  by  the 
variation  of  transit,  by  Problem  IX.,  page  312,  in  the  same  manner  as  if  it 
were  the  reduction  of  transit  to  a  different  meridian  that  was  under  consi- 
deration. Now,  these  proportional  parts,  being  cMed  to  their  corre- 
sponding semi-diurnal  arcs,  will  give  the  apparent  semi-diurnal  arcs  at 
the  times  .of  the  moon's  rising  and  setting:  the  former  being  subtracted 
from  the  apparent  time  of  transit,  and  the  latter  added  thereto,  the 
respective  apparent  times  of  the  moon's  rising  and  setting  will  be  obtained. 
These  may  be  reduced  to  the  mean  times  of  rising  and  setting,  by  Problem 
!•>  page  415,  if  necessary. 

Example  1. 

Required  the  apparent  times  of  the  moon's  rising  and  setting,  January 
17th,  1824,  in  latitude  51?28M0?N.,  and  longitude  75?W*;  the  height 
of  the  eye  above  the  level  of  the  horizon  being  30  feet  ? 

The  computed  apparent  time  of  the  moon's  transit  over  the  meridian  of 
the  given  place  is  13*43r46!  ;  now,  her  declination,  being  reduced  to 
this  time,  and  the  given  longitude,  is  10?  29' 27  TN.,  and  her  horizontal 
parallax  60^  59r. 

90?-60:59r+33'  +  5n5r=89?37:i6f 

Moon's  north  polar  distance  =  79.30.33       Log.  co-secant=:0. 007321 

Co-latitude  of  the  given  place  ==  38. 3 1 .  20      Log.  co-secant=:0. 205639 

Sum  = 207^39:  9". 


Half  sum  =: 103^49^34^^  Log.  sine  =    .    9.987230 

Remainder  =:       .    .    •    ,     .     14.  12.  18i    Log.  sine  =    .     9.389864 

Sum=:  19.590054 


Arch= 51?24^28r    Log.  cosine  =     9.795027 

J's  approx.  semi-diurnal  arcn  102?48^56r,  in  time  =:  .     .     6t51?16! 
Moon's  apparent  time  of  transit  over  the  given  meridian  =:    •  13. 43. 46 

Approximate  time  of  the  moon's  rising  = 6J52T30! 

Approximate  time  of  the  moon's  setting  =:     •    .    •    •    .    •20^357  2*. 


Digitized  by 


Google 


APPARBNT  TIMB  OF  RISING  Oft  SETTIK6  OF  ▲  OBLBSTIAL  OBJECT,  513 

The  longitude,  in  time,  being  added  to  those  times,  because  it  is  west^ 
shows  the  approximate  time  of  the  moon's  rising  at  Greenwich  to  be 
ll?52r30!  past  noon  of  the  given  day,  and  that  of  her  setting  1  t35T2! 
past  noon  of  the  1 8th.  Now,  the  moon's  declination  and  horizontal 
parallax,  reduced  to  these  times  respectively,  (the  former  being  corrected 
by  the  equation  of  second  difference,)  gives  the  declination  at  the  time  of 
rising  12?12'0lfN.,  and  the  horizontal  parallax  61  ^7^;  and  the  declina-^ 
tion  at  the  time  of  setting  8?46^  8f N.,  and  the  horizontal  parallax  60'49r« 

To  find  the  apparent  Time  of  Rising : — 

90?-.61'7^+33: +5a5r=89?37'  8r 

Moon's  polar  distance  ==     •     77*48.    0      Log.  co-secants  0.009921 

Co-latitude  = 38.31.20      Log.  co-secants  0.205639 

Sums 205?56:28r 

Half  sums 102?58'14r     Log.  sine  s     .    9.988775 

Remainders 13.21.   6      Log.  sine  s      .    9.363475 

Sum=  19.567810 

Archs 52?33a7i^  Log.  co-sine  s     9.783905 

Semi-diurnal  arc  s      ...  105?  6:35r,  in  time  s     .     .     7?  0r26! 
Variation  of  transit  s  53?,  the  proportional  part  of  which, 

answering  to  7 *0T26!,  is +14.55 

Apparent  semi-diurnal  arc  s 7*15 ?2l! 

Apparent  time  of  moon's  transit  s  .     . 13. 43. 46 

Apparent  time  of  the  moon's  rising  s  ««.««.     k     •    6 .  28T25 ! 

To  find  the  apparent  Time  of  Setting  :— 

90?-60M9r+33^+5U5'/s89^37^26r 

Moon's  polar  distance       .    .    81 .  13. 52      Log.  co.»ecant;=0. 005 106 

Co-latitudes 38.31.20      Log.  co-secantsO. 205639 

Sum  = 209?22:38r 

Half  sums 104?41U9'r    Log.  sines    .    9.985570 

Remainders 15.   3.53      Log. sines    .    9.414824 

Sums  19.611139 

Archs 50?  16:30§i:  Log, co-sines    9, 805569* 

2  L 

Digitized  by  VjOOQ IC 


514  NAOTICAL  ASnONOMT. 

Aroh« 60?16J30i^ 

Semi-diurnal  uc »      «    .    .  100?33M%  in  time  s     •    •    6!42?12! 
Variation  of  transit  a  53T,  the  proportional  part  of  which, 

aniweringto6*42Tl2:,  ias ;    .    .    .     +14.17 

Apparent  semi-diurnal  arc  s      «    .    »    «    •    b    •    •    •    •    6?56729! 
Appairent  time  of  moon's  transit  a      •••..•••  13. 48»  46 

Apparent  time  of  the  moon's  setting  =      ^ 20?40?15! 

Example  2. 

Required  the  apparent  times  of  the  moon's  rising  and  setting,  January 
20th,  1824,  in  latitude  40?30^N.,  and  longitude  80?B.5  the  height  of  the 
eye  above  the  level  of  the  horixon  being  30  feet  ? 

The  computed  apparent  time  of  the  moon's  transit  over  the  meridian  of 
tlie  given  place  is  15*55?5! ;  now,  her  declination,  being  reduced  to  this 
time,  and  to  the  given  longitude,  is  5?55'.35rS.,  and  her  horizontal  parallax 
58^53f. 


>*^ 


90?-58J53r  +  33C+5a5*'=89?39122* 

Moon's  north  polar  distance  a  95.55.35      Log.  co-s6cant=±  0.002329 

Co-latitude  of  the  given  place=:49. 30.  0      Log.  co-secant=>  0.118954 

Sum  = 235^  4^57^ 

Half  sums: 117?32C28ir  Log.  sine  s      .    9.947767 

Remainders 27.53.   6$    Log.  sine  =      •    9.669968 

Sum=  19.739018 

•     J  I  -T  -^ 

Airchssa    «««•..,    42?13!42?    Log.  co-sine  as      9.869509 

}) 's  approx.  semi-diurnal  arc  =  84?27' 24r,  in  time  =     .    •    S!37"50! 
Apparent  time  of  the  moon's  transit  over  the  given  meridian  sl5. 55.  5 

Approximate  time  of  the  moon's  rising  = 10'  17*15* 

Approximate  time  of  tiie  moon's  setting  = 21  i32?55! 

The  longitude,  in  time,  being  subtracted  from  those  times,  because  it  b 
east,  shows  the  approximate  time  of  tiie  moon's  risbg  at  Greenwich  to  be 


Digitized  by 


Google 


APPARBNT  TIMB  OF  KIIINO  OR  SBTTINQ  Of  A  CfilJBSTIAL  OBJBCT.  515 

4*57"15!  past  noon  of  the  given  day,  and  that  of  her  setting  16M2?65! 
past  the  same  noon.  Now^  the  moon's  declination  and  horizontal  parallax, 
reduced  to  these  times  respectively^  (the  former  being  corrected  by  the 
equation  of  second  difference,)  gives  the  declination  at  the  time  of  rising 
4?32^3rS.,  and  the  horizontal  pafallftx  59'6?  ;  and  the  declination  at  the 
time  of  setting  7?  18'  12rS.,  and  the  horizontal  parallax  58'40r. 


To  find  the  apparent  Time  of  Rising  :— 

90?- 59^6^4- ssc  +  snsrssg^sg^  97 

Moon's  polar  distance  cs      «    94.32.   3      Log.  co-secantaO.  001361 
Co-latitudes 49.30.   0      Log,  co-secant»0, 118954 

Sums    ......    •  233?4ia2? 

Half  sums 116?50^36?    Log.  sine  =    .    9.950484 

Remainders 27.11.27      Log.  sine  =    .    9.659874 

Sum=5 19.730673 

Archs 42?49'.42?    Log.  co-slnc  =  9. 8658861 

Semi-diurnal  arc  =:     .    .    .    85?39:24^  in  time  s     .    .    5M2r38:. 
Variation  of  transit  s  49T,  the  proportional  part  of  which^ 
answering  to  5*42r38!,  Is  «       +11.16 

Apparent  semi'*diunial  arc  a      •    •    « 5^53?54! 

Apparent  time  of  the  moon's  transit  as       .••••••  15.55.  5 

Apparent  time  of  the  moon's  rising  ^ •    •    •  lOt  ITli; 


To  find  the  apparent  Hme  of  Setting  i^ 

90?-58U0r+33^  +  5n5r=89?39^35r 

Moon's  polar  distance  =  •    .    97.18.12      Log.  co-secant*  0.003538 

Co-latitude  =     .    .    • 

Sum  =3     .    .    •    .    • 


Half  sum  =  • 
Remainder  ^ 


Arch: 


^9.30.   0      Log.  co-secant=  0,118954 


236°27M7r 

1 1 8?  13 :  53if  Log.  sine  =s     .    9. 944998 
28.34. 18J    Log.  sines:     .    9.679664 

Sum  =  19.747154 


41*S7'5ir    Log.co-sine  =    9.873577 
2l2 


Digitized  by 


Google 


516  NAtrriCAL  astronomV. 

Arch=r ;    .    41^37'.5ir 

Semi-diurnal  arc  =     .    .    .    83^5 '42^  in  time  =      .     .    5*33r  3! 
Variation  of  transit  =  49?,  the  proportional  part  of  which^ 

answering  to  5i33T3!,  is  SB +10.57 

Apparent  semi-diurnal  arc  = 5*44T  0! 

Apparent  time  of  the  moon's  transit  = 15.55.   5 


Apparent  time  of  the  moon's  setting  = 21*39?  5! 

See  Examples  1  and  2,  pages  134  and  136 ;  and^  also,  the  example  or 
work,  pages  137  vid  138. 


Problem  V. 

Gken  the  Latitude  and  Longitude  of  a  Place,  and  the  Day  of  the  Month; 
to  find  the  Time  qfthe  Beginning  and  of  the  End  of  TwiUgkt,  and  the 
Length  of  its  Duration. 

Rule. 

Reduce  the  sun's  declination,  at  the  midnights  preceding  and  following 
the  noon  of  the  given  day,  to  the  meridian  of  the  given  place,  by  Problem 
v.,  page  298;  then. 

Add  together  the  constant  quantity  108  degrees,*^  the  sun's  polar  distance, 
and  the  co-latitude  of  the  given  place :  take  half  the  sum ;  the  difference 
between  which  and  the  constant  quantity  call  the  remainder:    Now, 

To  the  logarithmic  co-secants,  less  radius,  of  the  polar  distance,  and  the 
co-latitude,  add  the  logarithmic  sines  of  the  half  sum  and  of  the  remainder: 
half  the  sum  of  these  four  logarithms  will  be  the  logarithmic  sine  or 
logarithmic  co-sine  of  an  arch ;  which,  being  doubled,  and  converted  into 
time,  will  be  the  apparent  time  of  the  beginning  or  of  the  end  of  twilight 
accordingly. 

Compute  the  apparent  times  of  the  sun's  rising  and  setting,  by  Problem 
L,  page  500 ;  then,  the  interval  between  the  time  of  the  cdiiimencement  of 
twilight  and  that  of  sun  rising,  will  be  the  duration  of  the  morning  twilight; 
and  the  interval  between  the  time  of  sun  setting  and  the  end  of  twilight, 
will  be  the  duration  of  the  evening  twilight. 

Note. — If  much  accuracy  be  required,  the  sun's  declination  must  be 
reduced  to  the  meridian  of  the  given  place,  at  the  respective  times  of  the 

•  90<>  +  18°  »  108^    See  Remarks,  page  518, 

Digitized  by  VjOOQ IC 


OF   FINDING  THB  BEGINNING  OR  END  OF  TWILIGHT. 


517 


commencement  and  of  the  end  of  twilight^  found  as  above  |  then^  the 
operations  being  repeated,  the  correct  apparent  times  of  the  beginning 
and  of  the  end  of  twilight  will  be  obtained.  This  degree  of  accuracy  may, 
however,  be  dispensed  with, — unless  in  cases  of  mere  speculative  inquiry^ 
or  where  some  philosophical  object  is  under  consideration. 

Example^ 

Hequired  the  apparent  times  of  the  beginning  and  of  the  end  of  twilight, 
and  its  duration,  October  1st,  1824,  in  latitude  40?30C  north^  and  longitude 
105?  east? 

To  find  the  Beginning  of  Twilight  :— 


Sun's  declination  at  midnt.  September  30th,=:3?  4C26r  S. 
Reduction  of  ditto  for  longitude  105?  E.  ^     —  6.48 


Sun's  reduced  declination  = 


2?57'38rS. 


Constant  quantity    =     108?  0'  Or 
Sun's  polar  distance  =     92. 57. 38 
Co-latitudes     .    .    .    49.30.   0 


Sum  =     .    • 

Half  sum  = 
Remainder  = 


.  250?27^38f 

.  125?13U9r 
.    17.13.49 


Log*  co-secant 
Log.  co-secant 


Log.  une  s 
Log.  sine  = 


=    0.000580 
=    0.118954 


•  9.912137 

•  9.471604 

19.503275 


Arch 


34?2r.54ir    Log.  sine  =      .    .    9.751637J 


Beginning  of  twilight  =     68?43M9r,  in  time  = 
Apparent  time  of  sun-rbing  on  the  given  day  =: 


4*34?55! 
6.   5.   8 


Duration  of  morning  twilight  = 1^80?13t 


To  find  the  End  of  Twilight  :— 

Sun's  declination  at  midnight,  October  Ist^  s  3?27'45r  S. 
Reduction  of  ditto  for  longitude  105?  E.  =      —  6. 48 


Sun's  reduced  declination  = 


•    •    •    • 


3?20:57rS. 

/Google 


Digitized  by ' 


518 


NAUTICAL  ASTEONOMYf 


Constant  quantity    s  108?  Ot  Or 

Sun's  polar  distance  =  93. 20. 57        Log*  co-aecaat 

Co-latitude  ^s    ...  49. 30.   0        Log,  co-secant 

Sum  = 250?50:57r 


Half  sum  =  . 
Remainder  = 


0.000743 
0.U8954 


.  125?25^28|r     Log.  sine  =    .^.    .    9.911093 
.     17.25,284      Log.  sine  =    .     .    .    9.476324 


Archss 55?27U0r       Log.  co-sine  =   • 


End  of  twilight  =  .     .  110?55:20r,  in  time  =     .     .    . 
Apparent  time  of  sun-setting  on  the  given  day  a     .    .    • 


Duration  of  evening  twilight 


•    •    •    •    • 


19.507114 
9.753557 

7*23T41! 
5.52.13 


1*31T28! 


Remarks. 

Twilight,  technically  called  the  crepusculumj  is  that  faint  light  which 
we  perceive  before  the  sun  rises  and  after  he  sets.  It  is  produced  by  the 
rays  of  light  being  refracted  in  their  passage  through  the  earth's  atmo- 
spherci  and  reflected  from  the  different  particles  thereof. 

The  morning  twilight  commences  when  the  sun  wants  18  degrees  of 
appearing  in  the  horizon  of  the  eastern  hemisphere,  and  the  evening 
twilight  ends  when  he  is  depressed  18  degrees  below  the  horizon  of  the 
vi^tem  hemisphere. 

When  the.  sun's  deolinatifMi  exoeeds  the  difference  between  the 
co-latitude  of  any  given  place  and  18  degrees,  there  will  be  no  real 
doribteit  or  night  at  that  place,  but  continual  day  and  twilight;  as  is  the 
case  at  London,  from  the  22d  of  May  to  the  21st  of  July. 

When  the  sun  is  on  the  same  side  of  the  equinoctial  with  the  derated 
polcj  the  duration  of  twilight  will  constantly  increase  as  he  approaches 
that  pole,  till  he  enters  the  tropic ;  at  which  time  the  duration  of  twilight 
will  be  the  longest.  It  will  then  decrease  until  some  time  after  the  sun 
passes  the  ec^uinox,  but  vrill  increase  again  before  he  arrives  at  the  opposite 
tropic  :  hence,  there  must  be  a  point  within  the  tropics  where  the  duration 
of  twilight  b  the  shortest.  This  point  may  be  found  by  the  following 
problem. 


Digitized  by 


Google 


OF  FINDINO  TOB  TIMB  OF  THB  9IIOETB8T  TWILIGHT.  (18 

PftOdLBM  VI. 

Cfiveti  the  LoUtude  qfa  Place  ;  tojind  the  Time  of  the  ehorte$t  Tmligla, 

and  ite  Duration. 

Rule. 

To  the  logarithmic  tangent  of  the  half  of  18  degrees^  add  the  loga« 
rithmic  sine  of  the  latitude ;  and  the  sum  (abating  10  in  the  index,)  will 
be  the  logarithmic  sine  of  the  sun's  declination  at  the  time  of  the  shortest 
twilight,  of  a  contrary  name  to  the  latitude :  the  day  corresponding  to  this 
declination  will  be  that  required. 

Again,  to  the  logarithmic  sine  of  the  half  of  18  degrees,  add  the  loga- 
rithmic secant  of  the  latitude ;  and  the  sum  (abating  10  in  the  index,)  will 
be  the  logarithmic  sine  of  an  arch,  which,  being  doubled  and  converted 
into  time,  will  be  the  duration  of  the  shortest  twilight; 

Example. 

Required  the  time  of  the  shortest  twilight,  and  its  duration,  in  the  year 
1824,  in  latitude  50?48^N.? 

Half  of  18  degrees   =    9?  OC  Of        Log.  tangent=:9. 199713 
Latitude  of  the  placezzSO.  48,  0  Log.  sine  =     9. 88927 1 


Sun's  declination    =:      7?  3^   K        Log.  sine  s     9. 088984  j 
which  is  south,  of  a  contrary  name  to  the  latitude. 

Half  of  18  degrees  =z     9?  0^  01        Log.  sine  =     9. 194332 
Latitude  of  the  place=  50. 48.   0  Log.  secants  10. 199263 

Aichzi    •    .    .    .    14?19C497        Log«  sine  =    9.393595 


Duration  of  twilights  28^39^38?^  in  time  s  1^54?39;. 

The  days,  in  the  Nautical  Almanac,  corresponding  to  the  sun's  decUna^-. 
tion  7?3a?  S«,  are  March  2d  and  October  1 1th,  which,  therefore,  are  the 
days  of  the  shorteat  twiHght  in  the  year  1824,  in  latitude  50?48C  north  | 
and  the  duration  of  the  twilight^  on  those  days,  is  l!54T39!* 


Digitized  by 


Google 


520  nautical  astronomy* 

Problem  VI^. 

Given  the  Latitude  of  a  Place  between  48'!32:  and  66  ?32^  (the  lAmiis 
of  regular  TbnUght) ;  to  find  when  real  Night  or  Darkness  ceases, 
and  when  it  commences.    • 

RULB^ 

The  complement  of  the  latitude,  diminished  by  18  degrees,  will  be  the 
declination  of  the  sun,  of  the  same  name  as  the  latitude,  at  the  time  when 
it  ceases  to  be  real  night,  and  also  when  real  night  commences. 

Example. 

Required  the  interval  of  time,  in  the  year  1824,  during  which  there  will 
be  no  real  darkness  or  night,  in  latitude  50?48^  north  ? 

Solution.— The  complement  of  the  latitude  39^21  N.-18?=21?12'N, 
=  the  sun's  declination.  Now,  the  days  answering  to  21?  12'  of  north 
declination  are.  May  26th  and  July  17th.  Upon  the  first  of  these  days, 
therefore,  real  night  ceases,  and  it  commences  upon  the  last.  During  this 
interval  there  is  no  real  darkness,  because  the  sun  is  less  than  18  d^rees 
below  the  horizon ;  and  so  on  for  any  other  latitude  within  the  limits. 


pROBf^M  VJIL 

Gioen  the  Sun*s  DecUnation  and  Semi-diametet ;  to  find  the  Interval 
between  the  Instants  of  his  lower  and  upper  lAmbs  being  in  the  Horizon 
of  a  known  Place. 

Rule. 

Rnd  the  appronmate  time  of  the  sun*s  rising  or  setting,  by  Problem  I., 
page  124;  to  which  time  let  the  sun's  declination  be  reduped,  by  Ph>bleia 
v.,  page  298. 

To  the  logarithm  of  the  sun's  semi-diameter,  expressed  in  seconds,  add 
the  constant  logarithm  9. 124939,  and  call  the  sum  a  reserved  logarithm; 
then. 

To  the  logarithmic  co-sine  of  the  sum  of  the  latitude  and  declination, 
add  the  logarithmic  co-sine  of  their  difference :  half  the  sum  of  these  two 
logarithms,  being  subtracted  from  the  reserved  hgariihm,  will  leave  the 
logarithm  of  the  interval  of  time,  in  seconds,  between  the  instants  of  the 
sun's  lower  and  upper  limbs  being  in  the  horizon  of  the  given  place. 


Digitized  by 


Google 


OF  THB  TIME  THAT  THE  SUN's  DIAMETER  TAKES  TO  RISE  OR  SET.  521 

Example  1. 

Required  the  interval  between  the  instants  of  the  sun's  lower  and  upper 
limbs  being  in  the  horizon,  at  the  time  of  its  setting,  July  ISth^  1824,  in 
latitude  50?48C  N.,  and  longitude  120?  W.  ? 

Apparent  time  of  setting  in  Table  L,,  to  latitude  50?48'  N., 

and  declination  2 1?49^ 51  rN.=    .     .     .   * 7*57"12'. 

Longitude  120?  west,  in  time  s= 8.   0.   0 

Greenwich  time  of  sun's  setting  = 15*57T12! 

Sun's  declination  at  noon,  July  13th,  1824,  =    21  ?49^ 5 1  ?N. 
Correction  of  ditto  for  15*57T12!=     ...       —5.58 

Sun's  reduced  declinatioa  = 21?43'53?N. 

Sun's  semi-diameter  15M5\8=945\.8  Log.=2.975799 
Constant  logarithm  = 9.124939 

Reserved  logarithm  s 12. 100738  .  .  12. 100738 

Sun's  red.  dec.  =  21?43^53rN. 
Lat.ofthcplace=50.48.   ON. 

Sum=    .    .    .    72?3K53r  Log.  co-sine  =  9.477387 
Differences    .     29.   4.   7     Log.  co- sine  =  9.941531 

Sum  a 19.418918 

Half  sums    « 9.709459  ..  .9.709459 

Interval,  in  seconds,  s=  246. 195  = Log.  s  2.391279 

Hence,  the  interval  between  the  instants  of  the  sun's  limbs  touching  the 
horizon^  is  4  minutes  and  6  seconds. 

Sxample  2. 

Required  the  interval  between  the  instants  of  the  sun's  upper  and  lower 
limbs  touching  the  horizon,  at  the  time  of  rising,  October  Ist^  1824,  in 
latitude  40?30C  N.,  and  longitude  105?  E.  ? 

Apparent  time  of  rising,  in  Table  L.,  to  latitude  40?30'N., 

and  declination 3?  16C6r  S.  = 6M0?48! 

Longitude  105?  east,  in  time  = 7.   0.   0 

Greenwich  time  past  noo%  September  SOth^ 11M0T48! 

Digitized  by  VjOOQ IC 


523  FEAGTICAL  ASTRONOIfT. 

Sun's  declination  at  noon^  Sept.  SOth,  1824^=  2?52U6?  S. 
Correction  of  ditto  for  1 1  ?  10T48  ?  =    .    .       + 10. 53 


Sun's  reduced  declination  = 3?  3'39rS. 

■  • 

Sun's  8en)i-diameter«=:  16'.  1 ''.  2=^961 '.  2  Log.=:2. 982814 
Constant  logaritbm  » «    •    .    9.124939 


Reserved  logarithm  = 12. 107753  •.  12. 107753 

Sun's  red.  dec.  =  3?  3^39rS. 
Lat.oftbeplace=:40.30.   0  N. 

Sum  as     •    .    43?3SC39r    Log.  co-sine  a  9.860124 
Differences  .    37.26.21       Log.  co-sine  s  9.899820 

Sum  as 19. 759944 


Half  sum  =  .    ; 9.879972  ..  .9.879972 


Interval,  in  seconds,  =  168. 958  =        Log.  =  2. 227781 

Hence,  the  interval  between  the  instants  of  the  sun's  limbs  touching  the 
horizon,  is  2  minutes  and  49  seconds. 

Note* — ^The  constant  logarithm  made  use  of  in  thk  problem  is  the  arith* 
metical  complement  of  the  proportional  logarithm  of  24  hours  esteemed 
as  minutes.  If  the  sun's  diameter  be  used,  instead  of  its  semi-diameter,  it 
must  be  expressed  in  minutes  and  decimal  parts  of  a  minute :  in  this  caie^ 
the  same  result  will  be  obtained  by  employing  the  constant  logarithm 
8. 823909 1  vi£.,  the  arithmetical  complement  of  the  oommon  logarithm 
of  15  degrees,  or  the  motion  corresponding  to  one  hour  of  time. 


SOLUTION  OF  PROBLEMS  IN  GNOMONICS  OR  DIALLING. 

Dialling,  or  GnomonicSf  is  a  branch  of  mixed  mathematics,  which 
depends  partly  on  tiie  principles  of  geometry  and  partiy  on  those  of 
astronomy;  and  it  may  be  defined  as  being  the  method  of  projecting  on 
the  surface  of  any  given  body,  whether  plane  or  otherwise,  a  figure  called 
a  sun-dialj — the  different  lines  of  which  indicate,  by  the  shadow  of  a  s^le 
or  gnomon,  when  the  sun  shines  thereon,  the  apparent  time  of  the  day. 

The  upper  edge  of  the  style  or  gnomon,  which  projects  the  son's  shadow 
on  the  plane  of  the  dial,  must  be  parallel  to  the  earth's  axis :  hence,  it  k 
sometimes  called  the  axis  of  the  dial. 

The  plane  of  the  gnomon  must  be  perpendicular  to  that  of  the  dialf 


Digitized  by 


Google 


GNOMONICS^  OR  PIAiXtNG.  523 

The  plane  on  which  it  is  erected  is  called  the  sub-style  i  in  horizontal  dials 
it  may  be  called  the  meridian,  or  12  o'clock  line. 

The  angle  comprehended  between  the  style  and  the  sulhstyle,  is  called 
the  elevation  of  the  style:  this  angle,  in  horizontal  dials,  is  always  equal  to 
the  elevation  of  the  pole,  or  the  latitude  of  the  place  for  which  it  is  com- 
puted; but,  in  erect  direct  north  or  south  dials^  it  is  equal  to  the  comple- 
ment of  the  latitude  of  such  place. 

Those  dials  whose  planes  are  parallel  to  the  plane  of  the  horizon,  are 
called  horizontal  diab  i  but  such  as  have  their  planes  perpendicular  to  the 
plane  of  the  horizon,  are  called  vertical  or  erect  dials. 

Those  vertical  dials  whose  planes  are  either  parallel  or  perpendicular  to 
the  plane  df  the  meridian,  are  called  direct  erect  dials.  One  of  these  must 
always  face  one  of  the  cardinal  points  of  the  horizon,  according  as  it  may 
be  a  north,  south,  east,  or  west,  erect  dial. 

All  other  erect  dials  are  called  declining  dials*  Those  dials  whose  planes 
are  neither  parallel  nor  perpendicular  to  the  plane  of  the  horizon,  are  called 
reclining  dials. 

In  this  place,  however,  we  shall  only  show  the  method  of  constructing  a 
horizontal  dial,  and,  also,  that  of  a  north  or  south  erect  direct  dial ;  these 
being  by  far  the  most  useful,  and,  indeed,  the  most  common  of  all  the 
varieties  in  dialling. 

PaoBLEM  I. 

Given  the  Latitude  of  a  Place;  to  find  the  Angles  which  the  Hour  Lines 
make  uAth  the  Sub- Style  or  Meridian  Line  of  a  Horizontal  Sun-Dial. 

Gbneral  Proposition. 

In  every  right  angled  spherical  triangle,  radius  is  to  the  sine  of  one  of 
the  legs  containing  the  right  anglcy  as  the  tangent  qf  the  angle  adjacetit 
to  that  side  is  to  the  tangent  of  the  other  containing  side  ^  tlie  triangle. 
This  is  merely  a  variation  of  the  equation  for  finding  the  leg  BC,  in 
Problem  IV.,  page  189  :  bence  the  following 

RULK. 

To  the  logarithmic  sine  of  the  latitude,  add  the  logarithmic  tangent  of 
the  sun's  horary  angle  iWtm  noon ;  and  the  sum  (abating  10  in  the  index,) 
will  be  the  logarithmic  tangent  of  the  angle  comprehended  between  the 
correspopding  hour  line  and  the  sub^style,  at  the  centre  of  the  dial. 

Note, — Sbce  the  sun's  apparent  motion  in  the  ecliptic  is  at  the  rate 
of  15  degrees  to  an  hour,  therefore  at  one  hour  from  noon  the  sun's  horary 
angle  is  15? ;  at  Iwo  hours  from  noon  it  is  30:^  J  and  so  on. 


Digitized  by 


Google 


524 


PRACTICAL  ASTRONOMY. 

Example. 


Required  the  angles  which  the  hour  lines  make  with  the  sub-style,  or 
meridian  line  of  a  horizontal  dialj  in  a  place  situated  in  50?48'  15 f  north 
latitude  ? 

To  find  the  Angle  at  one  Hour  from  Noon : — 

Latitude  of  the  place  =         .     .     50?48'.  15r     Log.  sine  =     9. 889296 
Sun's  horary  ang.  at  1  *  from  noon=  15.    0.    0       Log.  tangent=:9. 428053 

Hourlineofl,  or  11  o'clock  =  .     ll?43C52r     Log.  tangent=9.317349 

To  find  the  Angle  at  two  Hours  from  Noon : — 

Latitude  of  the  place  =     •    .     .    50?48: 15^     Log.  sine  =     9. 889296 
Sun's  horary  angle  at  2?  from  noon  =15.    0.   0       Log.  tangent^  9.  761439 

Hour  line  of  2,  or  10  o'clock  =       24?  6 :  20^     Log.  tangent=9. 650735 

Proceeding  in  this  manner,  the  several  angles  which  the  respective  hour 
lines  make  with  the  meridian  will  be  found  to  be  as  follows ;  viz.^ 

Hour  lines  of  L  and  XL  =:  ll?43^52r 

Ditto  IJ.  and  X.  =  24.   6.^0 

Ditto  in.  and  IX.  =  37.46.31 

Ditto  IV.  and  VIIL  =  53.18.53 

Ditto  V.  and  VU.  =  70.55.39 

Ditto  VI.  and  VI.  =  90.   0.  0 


Digitized  by 


Google 


GNOMONICS^  OR  DIALLING.  525 

The  hour  lines  of  VII.  in  the  evening  and  V.  in  the  morning,  make  the 
same  angles  with  the  meridian,  on  the  opposite  Hde  of  the  VL  o'clock  hour 
line^  as  the  hour  lines  of  VII.  in  the  morning  and  V.  in  the  evening.  In 
the  same  manner  the  hour  lines  of  VIII.  in  the  evening  and  IV.  in  the 
morning  make  the  same  angles  with  the  meridian  as  the  hour  lines  of  VIII. 
in  the  forenoon  and  IV.  in  the  afternoon;  and  so  on. 

The  angles  for  the  halves,  quarters,  or  other  subdivisions  of  the  hours, 
are  to  be  determined  in  the  above  manner. 

The  angles  which  the  diiferent  hour  lines,  &c.  make  with  the  meridian, 
being  thus  determined,  the  dial  may  then  be  very  readily  constructed,  by 
means  of  a  pair  of  compasses,  and  the  line  of  chords  on  a  common  Gunter's 
scale,  or  of  that  on  a  Sector :  the  latter,  however,  should  be  preferred, 
because  the  degrees  thereon  are  generally  divided  into  halves,  and  some- 
times quarters,  which  gives  it  a  decided  advantage,  in  point  of  accuracy, 
over  that  on  Gunter's  scale. 

Construction. 

On  the  proposed  plane  draw  the  meridian,  or  XII.  o'clock  hour  line,  a  b ; 
parallel  to  which,  at  a  distance  equal  to  the  intended  thickness  of  the 
gnomon  or  style,  draw  the  line  c  d :  perpendicularly  to  these  draw  the  VI. 
o'clock  hour  line  e/.  Open  the  Sector  to  any  convenient  extent,  and 
ti^e  the  transverse  distance  60?  to  60?  (on  the  line  of  chords)  as  a  radius 
in  the  compasses,  and,  from  a  as  a  centre,  describe  the  arc  g  h :  with  the 
same  radius,  and  from  c  as  a  centre,  describe  the  arc  i  ft ;  and,  since  the 
hour  lines  are  less  distant  from  each  other  about  noon  than  in  any  other 
part  of  the  day,  it  is  advisable  to  have  the  centres  of  those  quadrants  or 
iarcs  at  a  little  distance  from  the  centre  of  the  plane  of  the  dial,  on  the  side 
opposite  to  XII.,  so  as  to  allow  of  the  hour  distances  being  enlarged  near 
the  meridian  under  the  same  angles  in  the  plane  of  the  dial :  thus,  the 
centre  of  the  plane  is  at  A ;  but  the  centres  of  the  quadrants  or  arcs  are 
taken  a  little  below  it,  at  the  points  a  and  c. 

Take  the  transverse  distance  1 1  ?43 ^ 52?  to  1 1  ?43 ^  52^,  in  the  compasses, 
from  the  line  of  chords,  and  set  it  off  from  ^  to^l,  and,  also,  from  i  to  6  : 
take  the  transverse  distance  24?6^20?,  in  the  compasses,  and  set  it  off 
from  ^  to  2,  and  from  i  to  7 ;  and  proceed  in  the  same  manner  with  the 
remaining  horary  angles. 

Now,  from  the  centre  a  draw  the  forenoon  hour  lines  a  1  XI.,  a  2  X., 
a  3 IX.,  a  4  VIII.,  a  5  VII. ;  and,  from  c  as  a  centre,  draw  the  afternoon 
hour  lines  c6I.,  c7II.,  cSIII.,  c9IV.,  cOV.  :  produce  .a  5  VII.  and 
a  4  VIII.  for  the  hour  lines  of  VII.  and  VIII.  o'clock  in  the  evening;  and 
produce  c9IV.  and  cOV.  for  the  hour  lines  of  IV,  and  V.  in  the 
morning.    In  the  same  manner  may  the  quarter  and  half-hour  lines  be 


Digitized  by 


Google 


526  lHACriCAL  ASTltOKOMY. 

drawn  (and  minutes  if  necessary),  by  setting  off  the  compnted  corre- 
sponding angles  from  the  meridian :  these,  however,  have  been  omitted 
in  the  above  diagram,  with  the  view  of  preventing  embarrassment. 

Take  the  latitude  50?48n5T  in  the  compasses,  vis.,  the  transverse 
distance  50^48^  ISr  to  50^48'  151f,  and  set  it  off  from  g  to  L,  and  draw 
the  hypothenuse  line  a  L  P  for  the  axis  of  the  style  or  gnomon. 

The  style  may  have  any  shape  the  artist  pleases,  provided  its  edge  a  L  P 
be  a  perfectly  straight  line.'  It  should  be  a  metallic  substance,  and  must 
be  of  an  equal  thickness  with  the  breadth  of  the  space  comprehended 
between  t^e  two  parallel  straight  lines  ab  and  cd;  in  which  space  it  must 
be  erected  truly  perpendicular  to  the  plane  of  the  dial :  then,  since  the 
angle  B  a  P  is  equal  to  the  latitude,  the  straight  edge  of  the  style  =  a  L  P 
will  be  directed  to  the  elevated  pole  of  the  world,  and,  hence,  parallel  to 
the  earth's  axis  when  the  dial  is  truly  set ;  the  shadow  of  which,  when  the 
sun  shines,  will  indicate  the  hour  of  the  day. 

Note. — Since  the  hour  of  the  day  indicated  by  a  sun-dial  is  expressed  in 
apparent  solar  time,  it  must  be  reduced  to  mean  time,  by  Problem  I., 
page  415,  so  as  to  make  it  correspond  with  that  shown  by  a  well-regulated 
watch  or  clock. 


Probubm  II. 

To  Jind  the  Jngles  on  the  Plane  qfan  erect  direct  sciUh  Dial  for  aiiy 
proposed  north  Latitude,  or  on  that  qf  an  erect  direct  north  JXal  for 
any  proposed  south  Latitude. 

RVLE. 

To  the  logarithmic  co-sine  of  die  latitude,  add  the  logarithmic  tangent 
of  the  sun's  horary  angle  from  noon ;  and  the  sum  (abating  10  in  the  index,) 
will  be  the  logarithmic  tangent  of  the  angle  comprehended  between  the 
corresponding  hour  line  and  the  sub-style,  at  the  centre  of  the  dial« 

Example. 

Required  the  angles  which  the  hour  lines  on  an  erect  direct  south 
dial  make  with  the  sub-style  or  12  o'clock  line,  in  latitude  50?48:15? 
north  ? 


Digitized  by 


Google 


QNOMOKlCSi  OR  DlALLtKQ. 


5i7 


To  find  the  Angle  at  one  Hour  from  Noon  :— 

Latitude  of  the  placer:     .     .    .    50^48^5'     Log.  co-sinezz  9. 800699 
Sun's  horary  ang. at  1^  from noon=:  15.   0.   0      Log.  tangent=: 9. 428053 

Hour  angle  of  1^  or  1 1  o'clock  =:     9?S6M0r    Log.  tangentr:9. 228752 


To  find  the  Angle  at  two  Hours  from  Noon  :*-* 

Latitude  <^  the  place  =     .     .    .    50?48n5r     Log.  co-sine=:  9. 800699 
Sun'8horaryangleat2?fromnoon=30.   0.   0      Log*  tangent=::9. 761439 

Hour  angle  of  2^  or  10  o'clock  =:  20?  2M4r    Log.  tangent=9. 562138 

Proceeding  in  this  manner^  the  several  angles  which  the  respective  hour 
lines  make  with  the  meridian  will  be  found  to  be  as  follows  {  viz.. 


Hour  lines  of 

I. 

and 

XI. 

;^ 

9?36M0r 

Ditto 

II. 

and 

X. 

^ 

20.  2.44 

Ditto 

III. 

and 

IX. 

^Z 

32.17.30 

Ditto 

IV. 

and 

VIII. 

*"* 

47.35.10 

Ditto 

V. 

and 

VII. 

ZZ 

67.   1.25 

Ditto 

VI. 

and 

VI. 

zz 

90.  0.  0 

Digitized  by 


Google 


528  MSNSURATION  OF  HBIGHTS   AND  DISTANCES. 


Construction, 

On ^ the  proposed  plane  draw  the  XII.  o'clock  hour  line  ab;  parallel  to 
which,  at  a  distance  equal  to  the  intended  thickness  of  the  style,  draw  the 
line  c  d :  at  right  angles  to  the  sub-style,  or  XIL  o'clock  line,  draw  the  VI. 
o'clock  hour  line  «/•  Open  the  sector  to  any  convenient  extent,  and  take 
the  transverse  distance  60^  to  60?  (on  the  line  of  chords)  as  a  radius  in 
the  compasses,  and,  from  a  as  a  centre,  describe  the  arc  gh;  with  the 
same  radius,  and  from  c  as  a  centre,  describe  the  arc  i  k.  Take  the  trans- 
verse distance  9?36'40^  to  9?36'40r  in  the  compasses,  and  set  it  off  from 
g  to  1,  and,  also,  from  i  to  6.  Take  the  transverse  distance  20?2U4^  to 
20? 2' 44  f  in  the  compasses,  and  set  it  off  from  g  to  2,  and  from  i  to  7 ; 
and  proceed  in  the  same  manner  with  the  remaining  horary  angles.  Theiv 
from  the  centre  a,  draw  the  forenoon  hour  lines  a  1  XI.,  a  2  X.,  &c.  &c. ; 
and,  from  c  as  a  centre,  draw  the  afternoon  hour  lines  c  6  I.,  c  7  II*^  &c.  &c. 

Take  the  complement  of  the  latitude  in  the  compasses,  viz.,  the  trans- 
verse distance  39?  1 1 M5 T  to  39?  1 T. 45 f  ;  set  it  off  from  ^  to  L,  and  draw 
the  hypothenuse  line  aLP  for  the  axis  of  the  style  or  gnomon. 

Now,  when  the  dial  is  placed  vertically,  with  its  plane  duly  facing  the 
south,  the  VI.  o'clock  hour  line  e/will  be  parallel  to  the  plane  of  the 
horizon ;  and  the  style  B  a  L  P,  directed  downwards,  making  an  angle  with 
the  sub- style  or  XIL  o'clock  hour  line  equal  to  the  complement  of  the 
latitude,  will  be  truly  parallel  to  the  earth's  axis. 

Since  the  sun  cannot  shine  any  longer  on  a  dial  of  this  description  than 
from  VI.  in  the  morning  until  VI.  in  the  evening,  it  is  not  necessary  to 
describe  hour  lines  upon  it  before  or  after  those  periods  of  time. 

Note. — ^An  erect  direct  north  dial  for  a  place  in  north  latitude,  is  con- 
structed  exactly  in  the  same  manner  as  an  erect  direct  south  dial ;  but  the 
position  of  the  dial  must  be  reversed :  that  is,  the  VI.  o'clock  hour  line 
must  be  at  the  bottom  instead  of  the  top  of  the  dial ;  and  the  style  or 
gnomon  must  be  directed  upwards  instead  of  downwards. 


SOLUTION  OF  PROBLEMS  RELATIVE  TO  THE  MENSURA- 
TION  OP  HEIGHTS  AND  DISTANCES, 

Since  it  is  frequently  of  the  greatest  importanpe  to  the  mariner,  but  at 
all  times  to  the  engineer  or  other  military  officer^  to  be  able  to  ascertain 
the  heights  and  distances  of  remote  objects  with  precision,  the  following 


Digitized  by 


Google 


MBN8URATIOM  OF  HEIGHTS  AND  DISTANCES^ 


529 


problems  are  given  for  their  general  guidance  in  such  cases.  In  solving 
these  problems,  it  is  the  logarithmical  mode  of  calculation  that  will  be 
attended  to,  with  the  view  of  showing  the  direct  application  of  the  prin- 
ciples of  plane  trigonometry  to  such  cases.  .  To  the  imagination  of  the 
ingenious,  however,  many  other  modes  of  obtuning  an  approximate  value 
for  the  heights  and  distances  of  remote  objects  will  soon  present  themselves : 
such  as,  by  means  of  shadows,  mirrors,  unequal  vertical  staves,  &c.  &c. ; 
but,  since  these  methods  entirely  depend  upon  the  principles  of  similar 
triangles  (as  demonstrated  in  Euclid,  Book  VI.,  Prop.  4),  they  admit  of 
direct  sdutions  without  the  assistance  of  trigonometrical  tables :  hence^  no 
notice  can  be  taken  of  them  in  this  work. 


Problem  L 
To  find  the  Hetght  qfan  accessible  Object. 
Rule. 

Let  A  B,  in  the  annexed  diagram,  be  the 
object :  from  B  measure  any  convenient  dist- 
ance to  C ;  take,  at  C,  with  a  quadrant  or  other 
instrument,  the  angle  ADE;  then,  in  the 
triangle  ADE,  given  the  side  D E  s=  B  C,  and 
the  angle  at  D ;  to  find  the  side  A  E :  to  which 
let  the  height  of  the  observer's  eye  above  the 
horizontal  plane  ==  CD  or  B  E  be  added,  and 
the  sum  will  be  the  true  height  of  the  object 

AB. 

Example. 

Let  the  horizontal  distance  &  C  be  250  feet,  the  angle  of  elevation  ADE 
s  41945C,  and  the  height  of  the  eye  CD  =5  feet;  required  the  height 
of  the  object  AB? 

This  comes  under  Problem  II.  of  right  angled  plane  trigonometry,  page 
172 ;  and,  by  making  D  E  radius,  it  will  be 

As  radius  =: 90?         Log.  co-secant  =  10. 000000 

Is  to  the  distance  DE:sCB=  250  feet    Log.  =s      •    .    «    2.397940 
So  is  the  angle  of  elevation  A  D  E=:4 1  ?45 '.  Log.  tangent  =  .    9. 950625 


To  the  part  A  E  =  .    . 
Height  of  the  eye  C  D  =3 

Height  of  the  object  A  B : 


223. 13 
5. 


Log.ss 


2.348565 


228. 13  feety  as  required. 
2  M 


Digitized  by 


Google 


5S0  MBN8URATION  OF  HBIOSVS   AND  B18TAVOB9. 

JZemorfc.— By  removing  either  tQwavds  or  from  the  objeet,  uatil  the 
quadrant  shows  the  angle  of  altitude  to  be  45  degrees,  the  measure  of  the 
distance  between  the  toot  of  the  observer  and  that  of  the  objeel,  aug^ 
mented  by  the  height  of  the  eye,  will  become  the  altitude  or  height  of  that 
object. 


Probj^m  II. 

Owen  the  Angle  of  Elevation,  and  the  JSeighi  ofm  Okfee$;  t^fi^  Ifce 
Observer's  horizontal  Distance  from  thai  Object* 

At  any  convenient  distance,  as  at  C,  let  the  angle  of 
elevation  A  D  E  be  taken ;  then,  in  the  triangle  A  D  E, 
given  AE  s:  the  height  of  the  object  AB,  dipciinished 
by  the  height  of  the  eye  CD,  or  iU  equal  BE,  and  the 
angle  at D j  to  find  the  horizonUl  ({istaiice  DE  =  Cfi. 


Example. 

Let  the  height  of  the  object  AB  be  175  feet,  the  angle  of  elewHioa 
A  D  E  37?20' ,  and  the  height  of  the  observer's  eye  C  D  »  5  feet>  leqiured 
the  horizontal  distance  B  C  ? 

This  f^ls  under  Problem  IL,  Qf  rigl^t  ^ngM  pl«Be  trigoaoiii?t(j,  jM^e 
172  >  and  by  ipokuig  AB  radius,  tbe  proportiop  wiU  be 

As  radius  =.    ......    90?         Log.  co-secant  =  10.000000 

Is  to  height  of  the  object  AB  175  ft.^BE  5  ft.=  17Q  Log,  =     2.230449 
So  is  the  angle  of  elevation  A  D  E=37?20^  Log.  co.tangent=  10. 1 17637 

T9thehQrizontaldist.l)£==CB=^22.89  I^g[.  ^     .    ,    .    8.348086 


Digitized  by 


Google 


msm8uration  ov  hbi6hts  and  distancat*  581 

Paoblbm  IIL 
To  find  the  Height  of  an  inaccessible  Object^  asAB^ 

Rule. 

At  any  convenient  points,  as  C  and  D  (these 
being  in  the  same  vertical  plane  with  A  B), 
observe  the  angles  of  elevation  AFE  and 
AGE;  and  measure  the  distance  C  D :  then^ 
because  the  exterior  angle  AFE  is  equal  to 
the  two  interior  and  opposite  angles  A  G  F 
and  G  A  F  (Euclid,  Book  I.,  Prop.  32),  if  from 
the  angle  AFE  the  angle  AGE  be  subtracted,  the  remainder  will  be  the« 
angle  GAP.  Now,  in  the  oblique  angled  triangle  AGP,  given  the  side 
G  F  =  D  C,  and  the  angles  A  and  G ;  to  find  the  side  A  F :  and,  in  the 
right  angled  triangle  AFE,  given  tlie  hypothenuse  AF,  found  as  above, 
and  the  angle  AFE;  to  find  the  perpendicular  AE:  to  which  let  the 
height  of  the  observer's  eye  above  the  horizontal  plane  be  added^  and  the 
sum  will  be  the  height  of  the  ob)ect  AB. 

Example. 

In  the  above  diagram  let  the  angle  of  elevation  at  C  =  A  FE  be  49?2&^ , 
and,  after  receding  200  feet  in  the  same  vertical  plane,  to  the  point  D,  let 
the  angle  of  elevation  AGEbe31?20^;  now,  admitting  the  height  of  the 
observer's  eye  above  the  horizontal  plane  =  DGor  BEtobeS  feet^  it  is 
required  to  determine  the  height  of  the  object  A  B  ? 

The  angle  AFE  49?28C   -  the  angle  AGF  31?20:  a  the  aif^ 

OAF  18?8<. 

Now,  in  the  oblique  angled  triangle  AGF,  since  the  angles  and  one 
side  are  given,  the  side  A  F  is  found  by  oblique  angled  plane  trigonometry. 
Problem  I.,  page  177;  and,  in  the  right  angled  triangle  AEF,  since  the 
hypothenuse  A  F  is  now  kno\vn,  and  the  angle  at  F  given,  the  perpen- 
dicular AE  is  found  by  right  angled  plane  trigonometry,  Problem  L,  page 
171.    Hence, 

To  find  the  Side  A  P  :— 

Asthe  angle  GAP  SB  .    «    .    .     18?  &C     Log.  co^secants  10. 506919 
IstothesideGF  =  DC=  .     .    200  Log.  =  .    .    .    2.301030 

So  is  the  angle  AGF  =     •    .     .    31?20:     Log.  sine  =      .    9,716017 

TothestdeAFs 834,17    Log-=  .    .    .    2.52396& 

2m2 


Digitized  by 


Google 


583  IIBMSURATION   OP  HBI6BT8  AND  DI8TAMCB8. 

To  find  the  Perpendicular  AB  :— 


As  radius  ss  •  •  •  •  . 
Is  to  the  hypothenuse  AF  = 
So  is  the  angle  A  FE  =      • 

To  the  perpendicular  AE  zs 
Height  of  the  eye  B  E  =      . 

Height  of  the  object  A  B  = 


90^  Log.  co-8ecant=  10.  OOOOOO 
334.17  Log.=  •  .  .  2.523966 
49?28^  Log.  sine  =     .    9.  880830 

253.98  Log.  =    '  .    .    2.404796 
5. 


258. 98  feet|  as  required. 


Remark^-lf  it  be  required  to  know  the  horizontal  distance  B  C  or  B  D, 
it  may  be  readily  determined  by  means  of  the  last  problem. 


PROBLBM  IV* 

To  Jind  the  Distance  of  an  inaccemble  Object  tohkh  the  Observer  can 
ndther  advance  towards  nor  recede  from  in  Us  vertical  Line  of  Direction. 

Rule. 

Let  the  point  A  be  any  inaccessible  object, 
and  B  and  C  two  stations  from  which  the 
<fistance  of  that  object  is  to  be  determined : 
measure  the  distance  B  €»  and,  with  a  sextant 
or  other  instrument,  observe  the  horizontal 
anglesABC  and  AC  B;  then,  in  the  triangle  (*4 
ABC,  the  angles  and  the  side  BC  are  given;  to  find  the  other  two  sides^ 
viz.,  ABand  AC» 


JExatnpIe* 

Let  the  horizontal  angle  ABC,  measured  with  a  sextant,  be  59?  15 C, 
the  angle  ACB  42?45^,  and  the  measured  baseline  BC  350  yards; 
required  the  respective  distances  A  B  and  AC? 

The  angle  ABC  59915^  +  the  angle  ACB  42945^  =  102?;  and 
180?  -  102?  =;  78?,  the  ang)le  CAB. 

Now,  the  angles  and  one  side  being  thus  known,  the  remaining  sides 
are  to  be  determined  by  oblique  angled  trigonometry,  Problem  L,  page 
177*    Hence,  the  following  proportions :-?-  . 


Digitized  by 


Google 


MSNStJRATION  OF  BB1QHT8  AND  DI8TANCB8.  SS3 


To  find  the  Distance  A  C  :'— 

As  the  angle  C  A  B  =  •  .  .  «  78?  Log.  co-secant  s^  10. 009596 
Is  tothesideBC^s  «...  350  Log.  »  •  «  «  2.544068 
So  isthe  angle  ABC  s     •    ,.    .    59?  15^  Log. sine  =»  •    *    9.934199 

To  the  distance  A  C  =      •    .    •    307.51  Log.  =s    «    •    •    2.487863 


To  find  the  Distance  A  B : — 

As  the  angle  CAB  =  ....  78?  Log.  co-secant=  10. 009596 
Is  to  the  side  BC  =  •  •  •  «  350  Log.  =s  .  .  .  2.544068 
So  is  the  angle  A  CB=:       .    .    .    42?45^  Log.  sines      .    9.831742 

To  the  distance  A  B  =  .    •    .    .    242.89  Log.  =  •    «    •    2.385406 

Remark. — This  problem  will  be  found  of  very  essential  service  to  His 
Majesty's  ships  and  vessel^  of  war,  on  many  hostile  occasions :  for,  when 
it  is  intended  that  a  squadron  of  those  ships  should  cannonade  a  fort  to 
eifect,  or  batter  a  breach  in  the  sea-defences  of  a  town,  the  distance  at 
which  the  ships  should  be  placed,  abreast  of  such  fort  or  town,  with  the 
view  of  opening  their  fire  to  the  greatest  advantage,  may  be  readily  deter- 
mined in  the  above  manner.  Thus,  let  two  competent  persons,  provided  with 
sextants,  in  two  ships,  observe  the  angles  subtended  between  the  fort  and 
each  ship  respectively;  and  let  the  distance  between  the  two  ships  be  care- 
fully ascertained,  which  is  readily  done  by  Problem  II.,  page  530,  provided 
the  height  of  the  masts  be  known ;  or  it  may  be  found  by  means  of  a  boat 
sent  from  one  ship  to  the  other,  with  instructions  to  pull  at  an  uniform  rate : 
then,  if  the  interval,  per  watch,  be  noted  between  the  time  of  the  boat's 
pulling  off  from  one  ship  and  that  of  her  arrival  at  the  other,  and  her 
velocity  or  hourly  rate  of  sailing  be  duly  determined  by  the  log,  the  distance 
between  those  ships  may  be  easily  obtained  by  the  rule  of  proportion. 

Now,  with  the  distance  between  the  two  ships  as  a  base  line,  thus  found, 
and  the  angles  subtended  between  the  fort  and  each  ship,  the  respective 
distances  of  those  ships  from  the  fort  may  be  very  readily  comput^d^ 
agreeably  to  the  principles  of  the  present  problem. 

Note. — The  most  convenient  distance  for  commencing  a  cannonade,  is 
about  300  yards ;  that  is,  about  a  cable  and  a  quarter's  length  from  the 
object  at  which  the  guns  are  directed.  On  such  occasions,  however,  the 
captains  of  His  Majesty's  ships  of  war  always  make  choice  of  a  much 
closer  position,  provided  there  be  a  sufficient  depth  o|  water.^ 


Digitized  by 


Google 


S34  1ISN8UBATI0N  OF  HBIGHTS  AND  DI6TAMCBS. 

This  problem  is  also  extremely  useful  in  military  movements:  because, 
when  a  general  is  determined  on  the  reduction  of  a  to^n  or  garrison,  his 
engineer  is  thus  enabled  to  apprize  him  of  his  absolute  distance  from  any 
point  of  the  enemy's  defences  against  which  he  may  be  desirous  of  com- 
mencing eperetiens,  and  of  the  most  advantageous  position  for  throwing 
up  batteries  which  may  produce  the  greatest  possible  eflfect  on  the  fortified 
works  pf  the  besieged.  » 

In  military'  operations,  the  battering  guns  are  generally  placed  at  about 
375  paces  (312J  yards)  from  the  works  intended  to  be  breached. — 
A  military  pace  is  reckoned  at  30  inches. 


Problem  ¥• 
To^nd  the  Distance  between  two  inaccessible  Objects. 

Rule. 

Let  A  and  B  be  any  two  inaccessible 
olgects,  the  distance  between  which  is 
required.  Measure  any  base  line,  as  C  D ; 
at  the  point  G  observe  the  angles  A  C  B, 
BCD;  and,  at  the  point  D,  observe  the 
angles  B  D  A,  A  D  C.    Now,  in  the  triangle 

A  C  D,  in  which  the  angles  and  the  side  C  D      t"^^— ^D 

are  given,  compute  the  side  AD,  by  oblique  angled  trigonometry,  Problem 
I.,  page  177.  In  like  mumer,  in  the  triangle  BCD,  where  the  angles  and 
the  side  C  D  are  given,  compute  the  side  B  D  by  the  above-mentioned 
problem.  Now,  in  the  triangle  ABD,  the  sides  AD  and  BD,  and  the 
included  angle  ADB,  areghren;  with  which  the  distance  A  B  is  to  be 
computed,  l^  oblique  angled  trigonometry.  Problem  III.,  page   179. 

Example. 

Wanting  to  know  the  distance  between  the  two  inaccessible  objects  A 
and  B,  in  the  above  diagram,  I  measured  a  base  line  C  D  of  360  yards :  at 
C,  the  horizontal  angle  A  C  B  was  observed  with  a  sextant,  and  found  to 
be  53^30'.,  and  the  angle  BCD  38?45' ;  at  D,  the  horizontal  angle 
BD  A  was  67?20^  and  the  angle  ADC  44?30^  $  required  the  distance 
between  A  and  B> 

Angle  ACB  53?30'.  +  angle  BCD  38?45^  «  angle  ACD  92?15!; 
aiid^e  ACD  92?15'.  +  angle  ADC  44?S0^  »  136?45:.  Nov, 
180?  -  136?45;  ^  the  angle  CAD  43n&f. 


Digitized  by 


Google 


MUfSllRATlOK  or  BJBieHTS  AND  DI8TANCBS.  6SS 

Again:  Atigle  BDA  67?80:  +  angle  ADC  44?30:  =  ahgleBDC 
lll^SOCj  and  angle  BDC  111°50C  +  BCD  38?45f  =  160?85';. 
Now,  180?  -  150?35:  =^  the  angle  CBD  29?25:. 

In  the  Triangle  A  CD,  to  find  the  Side  AD:— 

As  the  angle  CAD  =  .  .  .  43915^  Log.  co-secant  =  10. 164193 
Is  to  the  side  CD  =  •  ...  360  Log.  =  •  .  .  2.556303 
So  is  the  angle  A  C  D  =     .    •    .    d2?  IS  ^  Log.  sine  =:  «    .    9.  ^9^665 


To  the  side  AD  =: 5iS.b    Log.  =f     •    •    .    1720161 

In  the  Triangle  BCD,  to  find  the  Side  BD:— 

As  the  angle  C  BD  =  .  •  .  29?25?  Log.  co-secant  =5:  10. 306^79 
Is  to  the  »ide  CD  ==  ...  360  Log.  =:  .  .  .  2.5563^3 
Soistheaftgi^BCDzr    .    •    .    38^45 C  Log.  sine  =i  .    .    9.796521 

To  the  side  BD  =1     .     •     .     .    45^,78  Log.  =     .    .    .    ld6l60S 

In  the  Triangle  ABD,  to  find  the  Angle  DAB  or  DBA,  and 

the  Side  AB:— 

180^  -  the  angle  BD  A  67?20^  =  112?40'  -♦-  2  =  569205  =  half 
the  sum  of  the  angles  DBA  and  DAB. 

Ai»  the  sum  of  the  sides  A  D  and  D  Br:988. 78  Log.  an  eoini^«=a7^  007 102 
Is  to  their  difFerenee  =  ....  66.28  Log.  =i:  .  .  1.821388 
So  is  ^  sum  of  tfBglesDBAand  DAB=s56?20:  Log.  tangents:  10. 176476 

To  half  di£ferenee  of  ditto  =:         •    .    5  ?46 ^  83?  Log.  tang.::  9<  00496 1 


Angle  DBA  = 62?  6^33r 

Angle  DAB  = S0?33:27? 

To  find  the  Distance  ABs— 

As  the  angle  DAB  =  «  .  .  50933C37?  Log  co.secant=lO.  112218 
Is  to  the  side  BD  r:  .  .  .  458.78  Log.  r:  .  .  2.661603 
Soistheangle  ADBn      .    .    67^20^  0?  Log.  sine  =     .    9.965090 

Totheside  ABist         .    .    .      54^.10      Log.fi^    ^    ,    2*788911 
wl^b^  tbefefote,  is  lli«  f eqiilred  dfittaacii^ 


Digitized  by 


Google 


536  MBN8URATI0N  OF  HXI6HT8  AMD  DI8TANCB8. 

^ote.— This  problem  is  very  useful  in  taking  surveys  of  coasts^  harbours^ 
bays,  islands^  &c. 


Problem  VJ, 

Given  the  Distances  between  three  Objects,  and  the  angular  Distances 
between  these  Objects  taken  at  any  Point  in  the  same  horizontal  plane  j 
to  find  the  Distance  between  that  Point  and  each  of  the  ObjectSm 

RULB» 

Let  Ay  B,  and  C  be  any  three  objects  whose 
distances  from  each  other  are  given,  and  E  the 
place  of  the  observer :  at  E,  observe  the  angles 
C  E  A  and  C  E  B ;  connect  the  points  A,  B,  and  C, 
by  right  lines;  make  the  angle  AB  D  equal  to  the 
observed  angle  C  E  A,  and  make  the  angle  DAB 
equal  to  the  angle  C  E  B :  hence  the  point  D  is 
found;  then,  through  the  three  points  A,  D,  and 
B,  describe  the  circle  AD  BE;  join  CD,  and 
produce  this  line  till  it  meet  the  circle  at  the  point 
E,  tlie  place  of  the  observer ;  join  E  A  and  E  B. 

Now,  in  the  triangle  ABC,  of  which  the  three  sides  are  given,  find  the 
angle  B  A  C.  In  the  triangle  A  B  D,  in  which  the  angles  and  the  side  A  B 
are  known,  find  the  side  A  D.  In  the  triangle  A  C  D,  of  which  two  sides, 
AC  and  AD,  and  the  included  angle  CAD  are  known,  find  the  angle 
A  C  D.  In  the  triangle  A  E  C,  of  which  the  angles  and  the  side  A  C  are 
given,  find  the  sides  E  A  and  EC.  And  in  the  triangle  ABE,  the  sides 
AB,  AE,  and  the  angles  AEB  and  EAB  are  given;  to  find  the  sideEB. 

Exanq^le. 

Let  the  points  A,  B,  and  C,  in  the  above  diagram^  be  three  known 
objects :  the  distance  between  A  and  B,  290  yards ;  between  B  and  C,  195 
yards ;  and  between  A  and  C,  240  yards :  let  E  be  the  place  of  an  observer, 
where  the  angle  C  E  A  was  measured  with  a  sextant  and  found  to  be  30?5', 
and  the  angle  CEB'25?45^  ;  required  the  distances  E A,  EC,  and  ED ? 

In  the  triangle  ABC,  the  three  sides  are  given ;  to  find  the  angle  B  A  C. 
Hence^  by  oblique  angled  plane  trigonometry^  Problem  lY.j  page  180, 


Digitized  by 


Google 


MBN8URATI0N  OP  RBIGHTft  AND  DISTANCES. 


5S7 


Side  BC  (opposite  the  required  angle) =195 
Side  A  C  (containing  the  required  angle) = 240 
Side  A  B  (containing  the  required  angle) = 290 


Log.  ar.co.=:7. 619789 
Log.  ar.co.= 7. 537602 


Sum=: 


725 


Half  sum  =:    . 
Remainder  =: 


Arch  = 


362.5    Log.  =: 
167.5    Log.= 


2.559308 
2. 224015 


Sum=  19.940714 
20?55<46r  Log.  co-sine  =9.970357 


Angle  BAC= 41?51^32f 

Angle  BAD  =  the  angle  CEB=  25.45.   0 


Angle  DAC  = 


16?  6^32r 


In  the  triangle  A  B  D,  the  angles  and  the  side  A  B  are  given ;  to  find 
the  side  AD  :  thus,  the  angle  ABD  (=  the  angle  CEA)  =  30?5'.  + 
the  angle  DAB  (=  the  angle  CEB)  =  25?45^  =  the  angle  AEB 
55?50C ;  and  180?  -  55?50C  =  the  angle  ADB  =*124?10: :  for,  the 
angle  A  DB  is  evidently  the  supplement  of -the  angle  AEB;  because  the 
opposite  angles  of  every  quadrilateral  figure  described  in  a  circle  are  equal 
to  two  right  angles. — Euclid,  Book  III,,  Prop.  22.  Hence,  by  trigo- 
nometry. 

As  the  angle  A  D  B  =  124?  10'  Log.  co-seeant  =  10. 082281 
Is  to  the  side  A  B  =  290  Log.  =  .  .  .  2.462398 
So  is  the  angle  A  B  D=  30?  5  C     Log.  sine  =       .    9. 700062 

To  the  side  AD  =     .175.69      Log.  =     •    .    •    2.244741 

In  the  triangle  ADC,  the  two  sides  AC,  AD^  and  the  included  angle 
DAC,  are  given;  to  find  the  angle  ACD:  hence,  by  oblique  angled 
trigonometry.  Problem  III.,  page  179, 


Log.ar.co.7.381230 

Log.  =      1,808279 

DAC  16?6^32r  J    81?56:44TLog.tang.lO,  849213 


As  side  AC  240  +  side  AD  175. 69  =     415. 69 
Is  to  side  AC  240-  side  AD  175. 69  =      64. 31 
So  is  180?  -  angle  DAC  16?6^32f 
=  163?53:28f 


Tohalfdiflf.ofanglesADCandACDzz  47.33.   3   Log.tang.lO.  038722 
Angle  ACD  = 34?23Mi: 


Digitized  by 


GooQle 


S88  ftCBNSUEATIOIf  Of  BVtOHTS  AlTD  StlTAKCtt. 

In  the  triangle  AEQ  the  unglet  and  the  tide  AC  lu«  gWdn  j  to  And 
the  sides  EAand  ECt  thus^  the  angle  CEA  dO?6^  +  the  angle  ACfi 
d4?23MI?  2  64?28UK  ;  and  180?  -  64^28^1''  a  the  atlgle  E  AC 
115?31 '  19?.  Hence^  by  oblique  angled  trigonometry.  Problem  I.^  page 
177, 

To  find  the  Side  E  A  >— 

As  the  angle  AEG  =     30?  5 C  Or     Log,  co-secant  =  10.299938 
Is  to  the  side  AC  =         240  Log.  =     .    .    .    2.380211 

SoistheangleACEs:  34?2dMir     Log.  sin^  =  .    .    9.751905 

TothesideEA=     .       270.47        Log.  =:     .    .    .    I4d21l4 


To  find  the  Side  EC:-- 

As  the  angle  AEC  =     30?  5 C  Or    Log.  =     .  .  .  10.299938 

I*  to  the  side  AC  =         240  Log.  :±     .  .  •    2.3S02tl 

Si  is  the  angle  EAC  =:115?3lU9r     Lbg.Sine  =:  .  .    $.955407 


To  the  side  fiC   r:  432.07        Log.  r:     .     .     .    2.685356 

In  the  triangle  ABE,  the  sides  AB,  AE,  and  the  angles  A EB,  EAB, 
are  given ;  to  find  the  side  E B :  thus,  from  the  angle  E  A C  li5?3U  19r, 
take  the  angle  BAC  41?5H32r,  and  the  remainder  is  the  angle  EAB 
=:  73?39'47r.     Hence,  by  trigonometry. 

As  the  angle  A  EB  <=    55?50^  Or    Log.  oo-seeant  =:  10.082281 
Is  to  the  side  A  B  =         290  Log.  =     .    .     .     2.462398 

So  is  the  angle  E  AB  =  73?39U7^     Log.  sine  =  .    .    9. 982101 


To  the  ride  EB»  336.34        Log.  »     «    •    ,    2^526780 

Hence  the  distance  of  the  object  A  from  the  observer  lit  E!^  is  270*47 
yards ;  that  of  C,  432. 07  yards ;  and  that  of  B,  336. 34  yards. 


Digitized  by 


Google 


MBNSURATION  OF  HBIGMTS  AND  DISTAKCBS. 


580 


Problem  VII. 

Given  the  Distances  between  three  Objects ,  and  the  angular  Distances 
between  these  Objects^  taken  at  any  Point  within  the  Triangle  formed 
by  the  right  Lines  connecting  the  Objects ;  to  find  the  Distance 
between  that  Point  and  each  of  the  Objects. 

Rule. 

Let  A,  B,  and  C  be  any  three  objects  whose 
distances  from  each  other  are  given,  and  E  the 
place  of  the  observer:  complete  the  triangle  ABC; 
at  E,  observe  the  angles  AEC,  AEB,  and  BEC; 
make  the  angle  BAD  equal  to  the  supplement  of 
the  angle  B  E  C ;  in  like  manner,  make  the  angle 
A  BD  equal  to  the  supplement  of  the  angle  A  E C  : 
hence  the  point  D  is  found.  Through  the  three 
points  A,  B,  and  D,  describe  a  circle;  join  D  C,  and 
it  will  cut  the  circle  in  E,  the  place  of  the  observer ; 
connect  the  points  A  E,  BE,  and  the  construction  will  be  completed;  the 
calculations  in  which  will  be  nearly  similar  to  those  in  the  preceding 
problem. 

Example. 

Let  A,  B,  and  C,  in  the  above  diagram,  be  any  three  known  objects 
whose  distances  from  each  other  are  as  follow :  viz.^  A  B,  620  yards ;  A  C^ 
570  yards ;  and  B  C,  460  yards.  At  a  point  E,  within  the  triangle  formed 
by  those  objects,  the  angle  A  E  C  was  measured  with  a  circle,  and  found  to 
be  125?151 ;  the  angle  AEB,  I24?15'.  ;  and  the  angle  BEC,  I10?80'. ; 
required  the  distances  E A,  EC,  and  EB ? 

In  the  triangle  A  B  D,  the  angles  and  the  side  A  B  are  given ;  to  find 
the  side  AD:  thus,  the  angle  BAD  69^30 C  sf  the  supplement  of  the 
angle  BEC;  the  angle  ABD  54°45C  =  the  supplement  of  the  angle 
AEC;  and  the  angle  AD  B  55M5'.  =:  the  supplement  of  the  angle  AEB. 
Hence^  by  trigonometry. 


As  the  angle  ADB  =     55?45C 
Is  to  the  side  AB   =       620 
So  is  the  angle  ABD  =  54?45C 


Log.  co-secant=10. 082710 
Log.  =  .  .  .  2. 792392 
Log.  sine  =     .    9.912032 


To  the  side  AD 


612.54    Log.  =  .    .    .    2.787134 


In  the  triangle  A  B  C,  all  the  sides  are  given ;  to  find  the  angle  B  A  C : 
which,  being  added  to  the  angle  BAD,  will  give  the  obtuse  angle  CAD. 
Henca^  by  trigonometry^  Problem  IV.^  pi^  180. 


Digitized  by 


Google 


540  MENSURATION  OF  HBIOIITS  AND  DI8TANCB8. 

Side  BC  =    460 

Side  AC  =     570    Log.  ar.  comp.=  7. 244125 

Side  AB  =     620    Log.  ar.  comp.=  7. 207608 

Sum  s=      .    1650 

Half  sums     825    Log.  =      .    .    2.916454 
Remainder  =  365     Log.  =       .    .    2. 562293 

Sums  19.930460 

''  Arch=      .    22  ?37  ^  9'Log.co-8ine=9. 965240 

Angle  CAB=45?14n8?  +  angle  BAD=  69?30'  s  angle 
CAD114?44a8r. 

In  the  triangle  A  C  D,  the  sides  A  C,  A  D,  and.the  included  angle  CAD 
are  given ;  to  find  the  angle  A  C  D :  hence,  by  oblique  angled  trigtmometiy, 
Problemlll.,  page  179, 

As  the  side  AD  612. 54+the  side  AC  570=1182. 54  Log.ar.co.6. 927IS4 
Is  to  the  side  AD  6 12. 54 -the  side  AC  570=  42. 54  Log.  =  1.628798 
Sois  180?- angle  CAD  H4?44n8r  J 32,37,5,,  Log. tang. 9. 806374 

To  half  diff.  of  angles  A  C  D  and  A  D  C=  1.19.10    Log.  tang.  8. 362356 

AngleACD=       •••...•    33?57^   K 

In  the  triangle  ADC,  all  the  angles  and  the  side  A C  are  gitren ;  to 
find  the  sides  AE  and  EC :  thus,  the  angle  A  EC  125?15'.  +  angle  ACE 
33?57n'/  =  159?12'K;  and  180?  -  I59?12!ir  =  the  angle  CAE 
20? 47  •59?.     Hence,  by  oblique  angled  trigonometry.  Problem  I.^  page 

177, 

To  find  the  Side  A  E  :— 

Astheangle  AEC=  125?15^  0*  Log.  co-secant  =10. 087968 
Is  to  the  side  A  C  =     570  Log.  =  •    .     •     2. 755875 

So  is  the  angle  A  C  E=  33? 57 '   K  Log.  sine  =     •    9. 747002 

Tothe8ideAE=      389.80        Log.  =  .    .    .     2.590845 

To  find  the  Side  EC:— 
As  the  angle  A  E  C  =  1 25  ?  1 5 '  0^  Log.  co-secant = 1 0. 087968 
Is  to  the  side  AC  =     570  Log.  =  •    .    .     2. 755875 

So  is  the  angle  C  A  E=  20? 47 '  59f  Log.  sine  =     •     9. 550359 

To  the  side  EC  =       247.86        Log*  =  ,    .    «    2.394202 


Digitized  by 


Google 


In  the  triangle  BEC^  given  the  sides  BC,  CE,  and  the  angle  EEC; 
to  $nd  the  angle  BCE,  and,  thence,  the  side  BE :  the  angle  BC  E  is 
found  by  oblique  angled  trigonometry.  Problem  IL,  page  178;  and  the 
side  BE  by  Problem  L,  page  177«    Hence, 

To  find  the  angle  B  C  E  :— 

As  the  side  B  C  =r      460  Log.  ar.  comp.  =  7. 337242 

Is  to  the  angle  B  E  C  s=  1 10?30:     Log.  sine  =r      •    9. 971588 
So  is  the  side  EC  =     247. 86        Log.  «...    2. 394202 

To  the  angle  C BE  =     30?18f4KLog.  sine  =      •    9.703032 
Angle  B£C=     .    .  110.30.   0 


Sum= 140?48Uirj  and  180?  ^   140?48'41f  = 

the  angle  B  C  E  z  39m  n9f 

To  find  the  Side  BE  :— 

As  the  angle  3  ECs  110?30^  Or  Log.  co-secants  10. 028412 
Is  to  the  side  B  C   =       460  Log.  =  .     .     .    2. 662752 

So  is  the  angle  BCEr:39?lin9r  Log.  sines:     .    9.800631 

TothesideBE=        310.31       Log.  =s  .    .    .    2.491795 

Hence  the  required  distances  are,  EA,  389.80  yards;  EB,  310.31 
yards;  and  EC,  247. 86  yards. 


Problem  VIIL 

Given  the  Distances  between  three  Objects  situated  in  a  straight  Line^ 

and  the  angular  Distances  of  these  Objects  taken  at  any  Point  in  the 

same  horizo7ital  Plane;  to  find  the  Distance  between  that  Point  and 

each  qfthe  Objects » 

RuiJs. 

Let  the  points  A,  B,  and  C  be  any  three 
objects  situated  in  a  straight  line :  make  the 
angle  ACD  equal  to  the  observed  angle 
AEB,  and  make  the  angle  DAC  equal  to 
the  observed  angle  B  EC :  hence  the  point  D 
is  found.  Through  the  three  points  A,  D, 
and  C  describe  a  circle ;  join  D  B,  and  produce 
it  till  it  cuts  the  circle  in  E ;  then  E  will  be 
the  place  of  the  observer,  and  EA,  EB,  and 
£  C  the  required  distances. 


Digitized  by 


Google 


542  IISNSUEATION  OF  HXIOHTS  AND  DItTAKCXS. 

Example. 

Let  A,  B,  and  C,  in  the  above  diagram,  be  any  three  knofvm  chjeets 
situated  in  a  straight  line,  whose  distances  from  each  other  are  as  follow : 
viz.,  A  B,  490  yards ;  B  C,  300  yards ;  and  A  C,  790  yards :  at  a  point  E, 
the  angle  BBC  was  observed^  and  found  to  be  43"^,  and  the  angle  BE  A 
33?45'.  I  required  the  distances  £A^  EB,  and  SC? 

In  the  triangle  A  D  C,  all  the  angles  and  the  side  A  C  are  given ;  to  find 
the  side  AD:  thus,  the  angle  D  AC  43?  =  the  observed  angle  BEC; 
the  angle  ACD  33?45^  =  the  observed  angle  BE  A;  and,  consequently^ 
the  angle  AD  C  =:  103?  IS '  :  hence  the  Me  A  D  may  be  found. 

As  the  angle  AD  C    =     lOSnSC  Log.  co-secant=10.011718 
Is  to  the  side  A  C  =         790  Log.  =  .    .    ,    2. 897627 

SoistheangleACD  »    d3?45:  Log.  sine  s     .    9.744739 

TothesideADs  450.94    Log. »  .    «    .    2.654084 

In  the  triangle  A  B  D,  given  the  sides  A  B,  A  D,  and  the  included  angle 
DAB;  to  find  the  angle  ABD:  hence,  by  trigonometry.  Problem  III., 
page  179, 

As  the  side  AB  490+the  side  A  D  450. 94=940. 94  Log.ar.co.7. 026438 
Is  to  the  side  A  B  490-  side  A  D  450. 94=:  39. 06  Log.  =  1 .  591732 
Soisl80e-aBgleDAB43?  =  137?^2=68?30^  Or  Log.  tang.lO.  404602 

Tohalfdiir.ofanglesADBandABD=  6?  0'57rLog.tang.  9.022772 

Angle  ABD  = 62?291  3rj  and,  since  the  two 

straight  lines  AC  and  DE  intersect  each  other  in  the  point  B,  the  opposite 
angles  are  equal  to  one  another  (EucKd,  Book  I.,  Prop.  15)  :  therefore  the 
angle  E B  C  is  62?29^3?,  equal  to  the  angle  ABD.  In  like  manner,  the 
angles  D  BC  and  ABE  are  equal  to  one  another;  and  because  DBC  is 
the  supplement  of  the  angle  D  B  A,  it  is  equal  to  U  7 ?S0^ 57' :  hence  the 
angle  ABE  is  also  equal  to  117?30C57r. 

In  the  triangle  ABE,  all  the  angles  and  the  side  AB  are  given ;  to  find 
the  sides  EA  and  EB :  thus,  die  an^e  BE  A  33?45(  +  the  angle  ABB 
117?30:57r  =  151?l5^57f ;  aod  180?  -  l51?lSC57r  =  the  aagle 
B  A  E  28?44  ^ 3r.    Hence,  by  trigonometry^  Problon  i,  page  177> 


Digitized  by 


Google 


To  find  the  Side  EA:— 

AstheangleBEA  =  33?45f  01  Log.  co-secant=  10. 255261 
Is  to  the  aide  A  B   =;      490  Log.  =»  «    ,    ,    2, 690196 

Soi8theangleABE=117?30:57^  Log.  sine  =     .    9.947867 

To  the  side  £A  =3  782.21       Log.  =:  .    .    .    2.893324 


To  find  the  Side  EB^^ 

As  the  cuigW  B£A  s  3SHi'.  01  Log.  eo-ieeontss  10. 255261 
Jltptb^«i4^AB  s?      490  Log.>R.    .    •    2,690196 

So  b  the  angle  BAE=:  28^44'  3?  Log,  aioe  pi      .    9.681917 

To  the  side  EBs        424.01       Log.  =  .    .    .    2.627374 

^  In  th^  triangle  SBC,  given  the  sides  £ B,  B C^  and  all  the  angles ;  to 

i  find  the  side  EC. 


As  the  angle  B EC  »  43?  Oi  OZ  Log.  oo-sfioant^ia  166^17 
Is  to  the  side  BC  =  300  '  Log.  =  .  .  •  2.477121 
So  is  the  angle  E B  C  =62^^29'  31  Log.  sine  =     .    9. 947866 

TathesideEC^.  390.19    Log. »  .    .    .    2.591204 


Hence  the  required  diatanees  are,  EA^  782.21  yards;  EB,  424.01 
yardft;  wi  EC,  390. 13  yards. 

JZemorilr.— The  above  problem,  together  with  that  given  in  page  536, 
wiU  be  found  exceedingly  useful  to  a  general  or  other  officer  employed  in 
conducting  the  military  operations  of  a  iiege ;  because,  if  he  cati  only 
preeure  a  correct  map  of  the  town  or  garrison  which  he  may  have  occasion 
to  invest,  so.  aa  to  ascertaia  the  relative  dietances  between  any  three 
desirable  positions,  the  above  problems  will  enable  him  to  find  his  absolute 
distance  from  those  positions  without  the  trouble  of  measuring  a  base  line : 
nor  is  it  necessary  to»  resort  ta  trigonometrical  calculation  for  this  partieular 
purpose,  since  the  distances  may  be  readily  determined  by  geometrical 
projection,  to  every  degree  of  accuracy  desirable  in  such  operations. 


Digitized  by 


Google 


544  MENSURATION   OP  HBI6HTS  AND  DI8TANGB8. 

Problem  IX. 
Given  the  Height  qfthe  Eye;  to  find  the  Distance  of  the  visiile  Horizon. 

.      RULB. 

Let  the  earth's  diameter,  in  feet,  be  augmented  by  the  height  of  the 
eye ;  then,  to  the  logarithm  thereof,  add  the  logarithm  of  the  height  of 
the  eye ;  from  half  the  sum  of  these  two  logarithms  subtract  the  constant 
logarithm  3.783904,*  and  the  remainder  will  be  the  logarithm  of  the 
distance  in  nautical  miles ;  which  is  to  be  increased  by  a  twelfth  part  of 
itself,  on  account  of  terrestrial  refraction. 

Esample. 

Chimbora$o,  the  highest  part  of  the  Andes,  is  said  to  be  20633  feet 
above  the  level  of  the  sea:  now,  admitting  that  an  observer  be  placed  upon 
its  summit,  at  what  distance  can  he  see  the  visible  horizon^  allowing  a 
twelfth  part  of  that  distance  for  the  effects  of  refraction  ? 

Diameter  of  the  earth,  in  feet,  =    .     •     •     4 1 804400 
Elevation  of  Chimbora9o  20633+5  feet,?  20638    Log.=  4.314668 

the  height  of  the  observer's  eye  =3 

Sum= 41825038    Log.=  7.621436 

Sum  s= 11.936104 

Half  sum  = 6.968052 

Constant  log.  =5 •    «    •    .      3.783904 


Distance  uncorrected  by  refraction  ^  •     •     «     152.81    Log.=2«  184148 
Allowance  for  terrestrial  refraction  =  •    .    .       12. 40 


Dist.  at  which  the  visible  horizon  may  be  seenss  165. 21  miles# 


•  This  U  the  logarithm  of  6080,  the  number  of  feet  in  a  nantiGal  mile. 


Digitized  by 


Google 


MBNSURATION  OF  HBI6HTS  AMD  DISTANCES. 


545 


Problem  X. 

Given  the  measured  Length  qf  a  base  Line  ;  to  find  the  Allowance  for  the 
Curvature  or  spherical  Figure  of  the  Earth. 

Rule, 

Let  £  B  F  represent  the  arc  of  a  great 
circle  on  the  earth ;  C^  the  earth's  cen- 
tre;  C  B  its  semi-diameter ;  and  A  B  the 
measure  of  a  base  line^  on  an  apparent 
level  or  horizontal  plane  on  the  earth's 
surface :  join  C  A,  and  it  will  cut  the  arc 
of  the  great  circle  in  D ;  then  A  D  will  be  the  excess  of  the  apparent  level 
of  the  horizon  above  its  true  leveK 

Now,  in  the  right  angled  plane  triangle  ABC,  given  the  perpendicular 
B  C  and  the  base  A  B ;  to  find  the  hypothenuse  A  C :  which  is  readily 
determined  by  Euclid,  Book  I.,  Prop.  47.  Then,  the  difference  between 
A  C,  thus  found,  and  C  D  =  C  B,  will  be  equal  to  D  A,  or  the  absolute 
value  of  the  true  level  behm  the  apparent  level :  and,  if  this  value  be 
expressed  in  miles  and  decimal  parts  of  a  mile,  it  may  be  reduced  to  inches, 
if  necessary,  by  being  multiplied  by  63360  =  the  number  of  inches  in  an 
English  mile. 

Example. 

Let  the  base  line  A  B,  in  the  above  diagram,  be  1  English  mile,  and 
the  earth's  semi-diameter  BC  =  3958.75  miles;  required  the  allowance 
for  the  earth's  curvature  answering  to  that  base  line,  or.  the  difference 
between  the  true  and  apparent  levels  on  the  earth's  surface  expressed  by 
the  measure  of  the  line  AD? 

BC3958. 75  xB  03958.75:^15671701.5625 
AB=  1  X  AB=  1=      .  1. 


Sum  of  the  squares  =     .     .     15671702. 5625 ;  the  square 

root  of  which  =  C  A,  IS •    •    •    3958.7501263 

Subtract  C  D  =  C  B,  the  earth's  semi-diameter,  =     .    .    3958. 7500000 

Remainder  ==  the  line  A  D,  the  allowance  for  curvature,=  0000. 0001263 
Multiply  by  the  number  of  inches  in  an  English  mile,  =     63360. 

Number  of  inches  which  the  true  level  is  below  the 

apparent  level  in  one  mile  =s 8. 0023680 

2n 


Digitized  by 


GooQle 


546  MBNSURATION  OF   HBIOHTS  AND  DI8TAMCB8. 

^Now,  since  the  curvature  answering  to  A  B  is  known,  that  corresponding 
to  any  other  base  line  on  the  earth's  surface  may  be  readily  determined  by 
the  following  proportion: — 

As  the  square  of  A  B,  is  to  AD;  soisthesquareofBG,  to  GH:  whence 
it  is  manifest,  that  the  curvature  answering  to  any  given  distance^  as  B  G^ 
is  in  the  duplicate  ratio  of  that  distance  to  A  B  : 

And,  since  AB  is  expressed  by  unity  or  1,  and  that  AD  is  a  constant 
quantity,  the  proportion  may  be  reduced  to  a  logarithmical  expression  ;  as 
thus : — 

To  twice  the  logarithm  of  the  given  base  line,  expressed  in  miles  and 
decimal  parts  of  a  mile,  add  the  constant  logarithm  0. 903219  (the  log.  of 
8. 002368  inches) ;  and  the  sum  will  be  the  logarithm  of  the  number  of 
inches  and  decimal  parts  of  an  inch  which  the  true  horizontal  level  at  sea 
is  below  its  apparent  level. 

Example  L 

Required  the  curvature  of  the  earth  answering  to  a  distance  of  2  miles 
on  its  surface  ? 

Distance  =  2  miles ;  twice  the  log.  =  «    .    •    0. 602060 
Constant  log*  s       . ;    •    0.903219 

Curvature,  in  inches,  =  32.009  Log.  =     1.505279 

Hence,  the  curvature  answering  to  a  distance  of  2  miles  on  the  surface 
of  the  earth,  is  32. 009  inches ;  or  2|  feet,  nearly. 


Example  2» 

Required  the  curvature  of  the  earth  answering  to  a  distance  of  15  miles  ? 

Distance  =  15  miles ;  twice  the  log.  =       .     .     2. 352182 
Constant  logarithm  =    ........    0.903219 

Curvature,  in  inches,  =  1800.533       Log.  =     3.255401 

Hence,  the  curvature  answering  to  a  distance  of  15  miles  on  the  earth's 
surface,  is  1800§  inches ;  or  150  feet  and  half  an  inch. 

Remark. — If  to  twice  the  logarithm  of  the  ^ven  base  line,  in  mile%  tbe 
constant  logarithm  9.  824037  be  added,  the  sum  (abating  10  in  the  index,} 
will  be  the  logarithm  of  tha  excess  of  the  apparent  above  the  true  lefe^  m 
feet* 


Digitized  by 


Google 


M&NSt7itATIDN  DP  HtilCHTS  AND  DiSTANCSd.  54^ 

Example. 

Required  the  curvature  of  the  earth,  or  the  excess  of  the  apparent  above 
the  true  level,  answering  to  a  base  line  of  15  English  miles  in  length  ? 

Given  base  line  =15  miles;  twice  the  logarithm  =r      •     •     .     !2. 352182 
Constant  log.  =z  log.  of  8. 002368  inches,  diminished  by  the 

log.of  12  inches,  =       9.824037 

Bxcess  of  the  app«  above  the  true  level^  in  ft.,s:  150. 044  hog.tsi.  176S19 

Note, — ^This  problem  will  be  found  useful  to  land-surveyors,  engineers, 
and  others  employed  in  the  art  of  levelling,  cutting  canals,  and  conducting 
water  (by  means  of  pipes,  &c.)  from  one  place  to  another. 


Problbm  XI. 

QiDen  the  meoiured  Length  of  a  Base  Line  on  any  deoaJted  Level  $  to 
find  its  true  Measure^  when  referred  to  the  Level  of  the  Sea. 

Rule. 

In  the  annexed  diagram,  let  the  arc  AB  represent  the  A^ 
measured  length  of  a  base  line,  at  any  given  elevation 
above  the  level  of  the  sea  expressed  by  the  arc  D  E ;  let 
C  D  be  the  radius  of  the  earth,  or  the  distance  from  its 
centre  to  the  surface  of  the  sea ;  and  let  C  A  be  the  earth's 
radius  referred  to  the  level  of  the  measured  base  line  A  B< 
Now,  because  the  arcs  A  B  and  D  E  are  concentric  and  similar,  and  that 
similar  arcs  of  spheres  are  to  each  other  as  their  radii,  we  have  the  following 
analogy  j  viz,, 

As  the  radius  C  A,  is  to  the  radius  C  D ;  so  is  the  arc  A  B,  to  the  arc  D  E: 
that  is,  as  the  earth's  semi -diameter,  augmented  by  the  height  of  the  base 
line  above  the  level  of  the  sea,  is  to  the  earth's  true  semi-diameter ;  so  is 
the  measured  length  of  the  given  base  line,  to  the  true  measure  of  that 
line  at  the  surface  of  the  sea. 

Example* 

Given  a  base  line  of  36960  feet  in  length,  measured  on  a  horizontal  plane 
which  is  elevated  1 20  feet  above  the  level  of  the  sea  j  required  the  measure 
of  that  base  line  at  the  surface  of  the  sea  ? 

2n2 


Digitized  by 


Google 


548  MENSURATION   OF   HEIGHTS   AND   DISTANCES.  . 

As  CD  =  20902200  +  DA  =  120  =  C  A  20902320=AB  36960  :: 
CD  20902200  :  DE  =  36959.787813.  Hence  the  given  base  line, 
reduced  to  the  level  of  the  sea,  is  36958.  787813  feet;  which  is  about  2^ 
inches  less  than  the  measure  on  the  elevated  horizontal  plane. 

But,  since  the  probable  elevation  of  ^ny  horizontal  plane  on  the  earth, 
above  the  level  of  the  sea,  can  bear  but  a  very  insignificant  proportion  to 
the  earth's  semi-diameter,^ — if,  therefore,  the  product  of  the  measured  base 
line  by  its  height  above  the  level  of  the  sea  be  divided  by  the  earth's  radius, 
the  quotient  will  be  the  excess  of  the  measured  base  above  the  correspond- 
ing arc  at  the  surface  of  the  sea.  This  may  be  reduced  to  a  logarithmical 
expression,  in  the  following  manner  5  viz.,  to  the  constant  logarithm 
2. 679808,  add  the  logarithms  of  the  base  line  and  of  its  elevation  above 
the  level  of  the  sea,  both  expressed  in  feet :  the  sum  will  be  the  logarithm 
of  a  natural  number,  which,  being  taken  from  the  measured  base  line,  will 
leave  the  measure  of  that  line  at  the  surface  of  the  sea,  sufficiently  near  the 
truth  for  all  practical  purposes.    Thus,  to  work  the  last  example. 

Constant  log.=ar.  co.  of  the  log.  of  the  earth's  semi-diam.  in  fit.=2. 679808 
Elevation  of  given  base  line  above  level  of  sea=  120  feet.  Log.=2. 079181 
Measured  length  of  the  given  base  line  =    36960  feet.     Log.=4. 567732 

Excess  of  the  given  base  line  above  the 

arc  at  the  surface  of  the  sea  =     ...     —  0. 212188  Log.  9. 326721 


Given  base  line,  reduced  to  level  of  sea,  =  36959.  7878 1 2 ;  which  approx- 
imates so  very  closely  to  the  true  result  by  the  direct  method  of  computa- 
tion, as  scarcely  to  admit  of  any  sensible  difference. 

Remark. — In  consequence  of  the  spherical  figure  of  the  earth,  no  two 
points  on  its  surface  can  be  situated  exactly  on  the  same  horizontal  plane; 
for  it  is  the  chord  of  the  arc,  and  not  the  arc  itself,  that  measures  the 
horizontal  distance  between  two  points.  Hence,  when  philosophical 
inquiries  are  under  consideration,  it  becomes  necessary  to  apply  a  small 
correction  to  the  measured  base  line  on  a  horizontal  plane,  so  as  to  reduce 
it  to  the  corresponding  terrestrial  arc ;  though,  in  general,  this  correction 
is  so  very  inconsiderable,  that,  even  in  the  most  extensive  trigonometrical 
surveys,  it  may  be  safely  disregarded.  If,  however,  it  be  deemed  necessary 
to  find  its  value,  or  (which  amounts  to  the  same  thing)  if  the  excess  of 
the  arc  over  its  chord  be  required,  it  may  be  very  readily  determined  by 
the  following  rule,  to  every  desirable  degree  of  accuracy ;  viz.. 

From  thrice  the  measured  'length  of  the  base  line,  in  feet,  subtract  the 
constant  logarithm  16. 020595  :  the  remainder  will  be  the  logarithm  of  the 
excess  of  the  arc  over  its  corresponding  chord,  expressed  by  the  given  base 
line. 


Digitized  by 


Google 


Let  it  be  required  to  find  the  excess  of  the  terrestrial  arc  over  its  chords 
answering  to  a  measured  base  line  of  36960  feet  in  length,  or  seven  English 
miles? 

Given  base  line  =  36960  feet ;  thrice  its  logarithm  =     .    ♦     13. 703196 
Constant  log.  =  log.  of  24  times  the  square  of  the  earth's 

radius,  in  feet,  = 16. 020595 

Excess  of  the  arc  over  its  chord,  in  feet,=0. 004815  Log.=    -  7. 682601 

Hence  it  is  evident,  that  the  extent  by  which  a  terrestrial  arc  of  36960 
feet  exceeds  the  chord  of  the  same  arc,  is  only  the  small  decimal  fraction 
.  004815  of  a  foot, — an  excess  so  very  trivial,  as  to  be  scarcely  worth  taking 
into  account,  even  where  the  greatest  accuracy  is  required :  for,  in  the 
present  instance,  though  the  base  line  is  7  English  miles  in  length,  it 
amounts  to  no  more  than  the  two  hundred  and  sixteenth  part  of  an  inch. 

Note. — ^The  index  of  the  logarithm  of  the  excess  comes  out  a  negative 
quantity,  because  the  index  of  the  constant  logarithm,  is  greater  than  that 
of  the  term  from  which  it  is  subtrac*ted. 


Problem  XII. 
To  find  the  Height  and  Distance  qfa  IBll  or  Mountain. 

Rule. 
Let  the  point  A,  in  the  annexed 
diagram,  be  the  summit  of  a  hill, 
the  height  of  which,  A  B,  is  to  be 
determined;  and  let  the  point  C 
be  the  place  from  which  its  dist- 
ance is  to  be  found :  at  C,  observe 
the  vertical  angle  ADE;  then 
measure  any  convenient  distance 
for  a  base  line,  as  C  F ;  at  the  point 
C,  observe  the  inclined  angle  ACF, 
and,  at  F,  observe  the  angle  AFC.  Now,  in  the  inclined  triangle  ACF, 
given  the  angles  and  the  side  F  C ;  to  find  the  side  A  C,  which  may  be 
considered  as  being  essentially  equal  to  the  side  A  D :  and,  in  the  vertical 
or  right  angled  triangle  A  ED,  the  angle  ADE  and  the  hypothenuse  or 
side  A D  are  given ;  to  find  the  perpendicular  AE:  to  which,  the  height 
of  the  eye  B  E  =:  C  D  being  added,  gives  the  required  height  A  B. 


S$Q  MB^rSURATIOH^  OF  II9I6HT3  At^B  0I3TAKCB6. 

Wanting  to  know  the  height  of  the  hill  ^  P,  and  the  distance  of  its 
summit  A  from  the  poipt  C^  the  vertical  angle  A  D  E  was  ohsefved,  afid 
found  to  be  14?30^ ;  at  the  points  C  and  F,  500  feet  asunder,  the  inclined 
angles  ACF  and  AFC  were  measured,  and  found  to  be  80?5',  and 
73?30'  respectively;  Required  the  distance  of  the  point  A  from  the 
pbserver  at  C,  and  its  height  above  the  level  of  the  horizontal  plane  C  B  ? 

Ii^  the  inclined  trij^ngle  ACF,  the  angles  are  given,  and  the  side  FC  =s 
500  feet ;  to  find  the  side  or  distance  A  C  :  thus,  the  angle  A  C  E  80?5  ^  + 
the  angle  AFC  73^80^  =  158?35^ ;  and  180?  -  153^85'*  =  26?25^ 
the  measure  of  the  angle  C  A  F.     Hence,  by  trigonometry, 

To  find  the  Distance  AC  s  the  Side  AD :— 

A^  the  a^gje  C  AF  =;     26?25;     Log.  co-seeaut=  10. 351742 
Is  to  the  side  C  F  =    .     500  Log.  =      .     .       2. 698970 

So  is  the  angle  CF A  =   73?30^'     Log.  sine  =   .       9. 981737 

To  the  distance  A  C  =     1077. 5^5  hog.  ^      .    .      3. 032449 ; 

and,  since  A  C  and  A  D  are  essentially  equal,  tha  aide  A  D  is  also  IQ77«5a 
feet. 

To  find  the  Height  A  B  :— 

As  radius  =      ...     90?  Log.  co-secant=l 0.000000 

Is  to  the  side  ADnAC  1077.5$  Log.  =       .     .     3.032449 
So  is  the  angle  A  D  E  =  14?30:     Log.,  fine  s:      .9. 39860Q 

To  the  perpendicular  AE=269. 84  Log.  =  .    .    ,    2.431049 
Height  of  the  eye  BE=C  D    5. 

Height  of  A  3=      .    .    274.84  feet 


PROBLEM  XIIL 

To  find  the  Height  of  a  Mountain^  hy  means  of  two  Barometers  and 

Thermometers. 

RULB. 

Let  two  observers  (pravided  with  barometers  and  thermpqiet^rs  of  equal 
constructio^i,)  carefully  note  dpwn^  at  the  sain^  in^tanty  the  r?^>ective 


Digitized  by 


Google 


MBNAtnUTlON  OF  HSIGHTS  AKD  DI8TAKCS8.  551 

heights  of  the  barometers  at  the  top  and  bottom  of  the  mountain,  or  other 
eminence  intended  to  be  measured^  with  the  temperature  of  thewquicksilver 
in  each  instrument  by  means  of  attached  thermometers,  and  also  the  tem- 
perature of  the  air,  in  the  shade,  hj  means  of  detached  tliermoitietei% ; 
then. 

Find  the  diflference  of  the  logarithms  of  the  observed  heights  of  the 
barometers,  the  first  four  figures  of  which,  besides  the  index,  are  to  be 
considered  as  whole  numbers.  To  this  difference  apply  the  product  of 
0. 454,  by  the  difference  of  the  altitudes  of  the  two  attached  thermometers.^ 
by  subtraction  if  the  temperature  of  the  quicksilver  at  the  bottom  station 
exceed  that  at  top ;  otherwise,  by  addition :  and  the  sum  or  difference  will 
be  the  approximate  height,  in  fiatboms,  English  measure. 

Now,  to  the  logarithm  of  the  approximate  height,  thus  found,  add  the 
logarithm  of  the  difference  between  the  mean  of  the  two  temperatures  of 
the  detached  thermometers  and  32?,  and  the  constant  logarithm  7. 387390: 
the  sum  of  these  three  logarithms  will  be  the  logarithm  of  a  correction, 
which  being  added  to  the  approximate  height  when  the  mean  temperature 
exceeds  32"?,  but  subtracted  if  it  be  less>  the  sum  or  difference  will  be  the 
true  elevation  of  the  mountain,  expressed  in  fathoms;  which  may  be 
redueed  to  feet,  if  necessary,  by  being  multiplied  by  6. 

JVbto. — ^This  m)e  is  deduced  firom  that  given  by  Dr.  HutCon,  in  the 
second  volume  of  his  ^^  Course  of  Mathematics/'  page  255. 

Example  1. 

Let  the  observations  at  the  top  and  bottom  of  a  mountain  be  as  follow ; 
required  its  height  ? 

Attschftd       Detadked 
thermometer*    thermom.    Barometer. 

Obs-  at  bottom=57  57     29-  68  Log.r:  1 .  472464 

Ditto  at  top  -     43  42    25. 28  Log.=:  1 .  402777 

Differtnee  =r       14  S«Bn  =  99    Difference  =  0.069&87 

Multiply  by      .454  Mean=:  49|  Product  =i        -  6.36 

Product  = -6.356  32    Approx.  alt>  n  690. 51  Log.2«  839170 

Diff.sr     \7ior\7.5  hog.zz       .    .    .     1.243038 
CfHMKtant  log.  =r 7.387390 

Correction  of  the  approximate  altitude  =:  •  •  •  +  29.48  Log.l.  469598 
True  altitvte  of  tbe  momtaiD^  in  fotbooae,  =:     .     719. 99,or 4319. 94  h. 

Digitized  by  VjOOQ IC 


552  MENSURATION   OF   HSI6HT8  AND  DI8TANCB8. 


Example  2. 

Let  the  observations  at  the  top  and  bottom  of  a  mountain  be  as  follow ; 
required  its  height  ? 

Attached       Detached 
thermometer,    thermom.    Barometer. 

Obs.  at  bottom=38  31     29. 45  Log.=  1 .  469085 

Ditto  at  top  =     41  35     26. 28  Log.=  1 .  428459 

Diflference  =         3  Sum  =  66    Difference  =0.0406.26 

Multiply  by     .454  Mean=  33     Product  =         +1-36 

Product  =  + 1 .  362  32    Approx.  alt.  =  407. 62  Liog.2. 610255 

Diff.  =     1     Log.= 0.000000 

Constant  logarithm  = 7.387390 

Correction  of  the  approximate  altitude  =      ,    •     +  0. 99  Log.9. 997645 
True  altitude  of  the  mountain^  in  fathoms,  =:      •     408. 6 1,  or  245 1 .  66  ft. 


Problem  XIV. 

To  find  the  Distance  of  an  Object,  by  observing  the  Interval  of  Time 
between  seeing  the  Flash  and  hearing  the  Report  of  a  Gun  or  of  a 
Thunder-Cloud. 

RULB. 

To  the  logarithm  of  the  number  of  seconds  elapsed  between  seeing  the 
flash  and  hearing  the  report,  add  the  constant  logarithm  9. 273762* ;  and 
the  sum  (abating  10  in  the  index,)  will  be  the  logarithm  of  the  distance  in 
nautical  miles ;  or,  if  the  constant  logarithm  9. 335032t  be  made  use  of, 
it  will  give  the  distance  in  English  statute  miles. 


*  This  is  the  sum  of  the  arithmetical  complement  of  the  logarithm  of  6080,  the  nnmher 
of  feet  in  a  nautical  mUe,  and  the  log^thm  of  1 142,  the  number  of  feet  which  sound  travels 
in  one  second  of  time. 

t  This  is  the  sum  of  the  arithmetical  complement  of  the  los;arithm  of  5280,  the  mimber 
of  feet  in  an  English  mile,  and  the  logarithm  of  1142  feet,  the  established  velocity  of  aound. 


Digitized  by 


Google 


OF  THE  COURSE  8TEBRBD  BY  A   SHIP   8BBN   AT  A  DISTANCE.      553 


Example  1. 

A  ship  at  eea  was  observed  to  fire  a  gun,  and  43  seconds  afterwards  the 
report  was  heard ;  required  the  distance  of  the  ship  in  nautical  miles  ? 

Interval  betw.  seeing  the  flash  and  hearing  the  gun=43?  Log.=:l.  633469 
Constant  logarithm  = 9. 273762 


Distance  in  nautical  miles 


8.076    Log.=0. 907231 


Example  2. 

A  flash  of  lightning  was  seen,  and  after  a  lapse  of  18  seconds  the  repdrt 
reached  the  ear  of  the  observer ;  required  the  distance  of  the  thunder- 
cloud in  English  miles  ? 

Inter,  betw.  seeing  the  flash  and  hearing  the  thunder=  1 8 !  Log.=  1 .  255273 
Constant  logarithm  = 9. 335Q32 


Distance  of  the  thunder-cloud  in  English  miles=3.894  Log.ssO.  590505 


PaOfiLBM  XV. 

Given  three  Bearings  of  a  Ship  sailing  upon  a  direct  Course,  and  the 
Intervals  of  Time  between  those  Bearings ;  to  find  the  Course  steered 
by  that  Ship,  and  the  Time  of  Iter  nearest  Distance  from  the  Observer. 


Describe  the  circle  N  E  S  W, 
and  from  the  centre  C,  th6 
place  of  the  observer,  draw  the 
lines  CA,  CB,  and  CD,  to 
represent  the  three  bearings 
of  the  ship :  through  the  centre 
C  draw  the  line  FG,  at  right 
angles  to  the  second  bearing 
CB;  make  CF  equal  to  the 
first  interval,  and  C  G  equal  to 
the  second,  each  being  ex- 
pressed in  minutes,  and  taken 
firom  any  convenient  scale  of 
equal  parts;  from  the  points  F 


Rule. 


Digitized  b' 


GooqIc 


554  IfEKSUBATlON  OP  HBI6HT8  AND  DI8TANCJK8. 

and  G  draw  the  lines  FA,  GD,  parallel  to  the  second  bearing  CB^  and 
meeting  CA  and  CD  in  the  points  A  and  D;  join  AD,  and  it  will 
represent  the  ship's  track ;  through  C  draw  C  K,  parallel  to  A  D,  and  the 
arch  S  K  will  be  the  measure  of  the  ship's  course.  From  C  let  fall  the 
perpendicular  C  H  upon  the  line  A  D,  produced  if  necessary ;  and  from  H 
let  fall  the  perpendicular  H I  upon  the  line  F  G^  produced  also^  if  necessary  | 
then  the  measure  of  CI  will  give  the  interval  between  the  dme  of  the 
second  bearing  and  that  when  the  ship  was  nearest  to  the  observer. 

Make  A  L  equal  to  the  difference  betweeh  the  perpendiculars  A  F  and 
D  G ;  then,  in  the  right  angled  triangle  A  L  G,  given  the  perpendicular 
AL  and  the  base  LD;  to  find  the  angle  LAD,  which  is  evidently  equal 
to  the  angle  a  C  K ;  to  this  let  the  inclination  of  C  B  to  a  parallel  be 
applied,  and  the  result  will  be  the  apparent  course  of  the  ship. 

Example. 

At  1  ^20T  past  noon  a  ship,  sailing  upon  a  direct  course,  was  observed  to 
bear  N.W.  b.  N. ;  at  2M0r,  she  bore  N.  ^  W. ;  and  at  3*25?,  the  bearing 
w«3  N.E.  b.  EL;  required  ttie  apparent  course  steered  by  that  ahip>  and  the 
time  when  she  was  nearest  to  the  observer  ? 

Sobition. — ^The  circle  being  described  and  quartered,  and  the  three  given 
bearings  laid  down  as  above  directed^  through  C  draw  F  G  perpendicular 
to  the  second  bearing  C  B ;  make  F  C  equal  to  50  minutes,  the  interval 
between  the  first  and  second  bearings,  and  C  O  equal  to  75  mkiQleey  Ae 
interval  between  the  second  and  third  bearings  :  tfiese  may  be  taken  from 
any  scale  of  equal  parts.  Then  proceed  with  the  other  parts  of  the  con- 
struction, agreeably  to  the  rule ;  now,  the  ship's  apparent  course,  repre- 
sented by  the  angle  S  C  K,  being  applied  to  the  line  of  chords,  will  be 
found  to  measure  72j  degrees :  hence  the  course  is  &  73?SO^  Bb^  or 
E.  b.  S.  I  S.  nearly.  The  perpendicular  GD,  being  sfqplied  to  the  scale 
of  equal  parts  from  which  the  intervals  were  taken,  will  be  fomd  to 
measure  40,  and  the  perpendicular  F  A  93^ ;  tbe  diffetence  between  wliich 
=  53},  is  the  measure  of  A  L.  Then  C  I,  measured  upon  the  same  scale, 
gives  26  minutes ;  which  is  evidently,  by  the  construction,  past  tbe  tnne  of 
the  second  bearing :  hence  the  time  of  the  ship's  nearest  approach  to  tbe 
observer  at  C,  is  2M0T  +  26r  =  2^36?  past  noon.  Now,  the  figare 
being  thus  completed,  the  required  parts  may  be  obtained  by  trigono- 
metrical calculation,  in  the  following  manner  : — 

In  the  right  angled  triangle  AFC,  given  the  an^es  and  the  base  FC  50 
minutes ;  to  find  the  perpendicular  FA.  Thus,  since  the  sln^ht  line  AC 
falls  upon  the  two  parallel  straight  lines  C  B  and  PA,  it  midkea  the  ahemate 
angles  equal  to  one  another  (Euclid,  Book  I.,  IVop^  29) :  therefbie  the 


Digitized  by 


Google 


OP  THB  COUBSB  STKBRBB  BY  A  SHIP  SBBN  AT  A  DISTANCB.    558 

angle  FAC  ia  equal  to  the  angle  ACB;  but  the  angle  ACB  is  given, 
being  equal  to  2^  points,  viz.,  the  difference  between  N.W.  b.  N.  and 
N.  I W. :  henee  the  angle  F  A  C  is  also  equal  to  2^  points. 

In  the  same  manner  it  may  be  shown  (in  the  right  angled  triangle  DOC, 
where  the  angles  and  the  base  C  G  75  minutes  are  given ;  to  find  the 
perpendicular  G  D,)  that  the  angle  G  D  C  is  equal  to  the  angle  BCD; 
and ^  since  BCD  is  given,  being  equal  to  5^  points,  viz.,  the  sum  of 
N.E.  b«  E.,  and  N.  |  W.,  therefore  the  angle  G  D  C  is  also  equal  to  5^ 
points.     Hence, 

To  find  the  Perpendicular  GD  :— 

As  radius  ^  •  •  .  •  90?  Log.  co-secant  =  10.000000 
Is  to  the  base  CG»  .  75?  Log.  =i  .  .  .  1.875061 
So  is  the  angle  G  D  C^H  poinU,  Log.  eo-tangent  ^  9. 727957 


Tothep6i7endicularGD=40.09Log.=      .    .    .     1.603018 

To  find  the  Perpendicular  F  A : — 

As  radius  =      ....     90?       Log.  co-secant    10.000000 
Isto  the  base  FC      .     .     50r       Log.  =  .     .     .     1.698970 

So  is  the  angle  F  A  C  =x  2^  points,  Log.  co-tang.  =  10. 272043 

I        .... 

1^  the  perpendicular  FA=s93. 54    Log.  »  .    .     .     1.971013 
Perpendicular  QD    a      40.09 

Difference  =;«.,,    53*  45,^  which  is  equal  tQ  the  part  A  L* 

In  the  ri^ht  angled  triangle  ALP,  given  the  base  LD  =  FG  125 
minutes,  and  the  perpendicular  AL  53.45;  to  find  the  angle  LAD: 
therefore, 

M  the  perpendicular  A  L  =^  53. 45  winytes     Log.  ar^  co(np.=:8. 272052 
IstoMiwB==     .    .    «    .      90.0  Log.  sine   *=i     10.000000 

Soiatbe  baseLD  =  .     .       125   minutes,    Log.  =      .    .    2.096910 

To  the  angle  LAD  =     .    66^50^54'/  Log.  tangent  55^10,868962 

Now^  ttpce  CK  is  paraUel  to  AD,  and  Ca  to  AL,  the  angle  aCK  is 
equal  to  the  angle  L  AD ;  but  the  angle  L  ADis  found,  by  computation, 
to  b^  $Q?50'54'' ;  wherefore  the  angle  a  C  K  is  al^o  equal  to  66?50'.54^  : 
to  thij^  let  the  angle  a  C  S  ==^  the  angle  N  C  B  Oi  point,  or  5?37  '30r,  be 
added;  and  the  sum  72^28"  24'/  =  the  angle  S  C  K  is  the  apparent  course 
of  the  ship  between  the  south  and  the  east,  vLz.^  S.  72'^28'24^'E,  or 
E.  b.  S.  i  S.  nearly. 


Digitized  by 


Google 


556  Mj^SURATION   OF  HBfGHTS  AND  DISTANCES. 

We  have  now  to  determine  the  measure  of  the  base  C I^  in  the  right 
angled  triangle  C I H ;  to  do  which,  we  must  first  find  the  value  of  the 
hypothenuse  A  C  in  the  right  angled  triangle  AFC,  and  that  of  the  base 
CH  in  the  right  angled  triangle  A  H  C.    Thus, 

To  find  the  Hypothenuse  AC: — 
As  radius  =       ....    90?       Log.  co-secant  =10. 000000 
Is  to  the  base  FC=    .    .    SOT       Log.  =  .     .    .     1.698970 
So  is  the  angle  FAC  =  2?  points.  Log.  co-secant=  10. 326613 

To  the  hypothenuse  A  C  =  106. 07  Log-  =s  •    •    •    2. 025583 

Tofind  the  Base  CH:— 
As  radius  ^      ....     90?       Log.  co-secant=  10. 000000 
Is  to  the  hypothenuse  AC- 106. 07  Log.=  •    •     .     2.025583 
So  isLAD-FAC=CAH=38?43^24rLog.sine  =  9.796269 


To  the  base  CH=     .     .    66.35  Log.  =  .     .     .     1.821852 

Now,  in  the  right  angled  triangle  C I  H,  given  the  hypothenuse  C  H  = 
66. 35  minutes,  and  the  angle  CHI;  to  find  the  base  C  I.  The  measure 
of  the  angle  C  H I  is  thus  determined.  In  all  quadrilateral  or  four-sided 
figures,  the  sum  of  the  four  angles  is  equal  to  four  right  aisles,  or  360 
degrees.  Now,  in  the  quadrilateral  figure  A  H  I F,  since  three  of  the 
angles  are  given,  the  remaining  or  obtuse  angle  A  H I  is  known  by  sub- 
tracting the  sum  of  the  given  angles  from  360  degrees :  thus,  the  angle 
HIF90?  +  I  FA  90?  +  FAH  66?50^54r  =  246?50:54r5  and  360? 
-  246?50^54r  =  113?9^6r,  is  the  measure  of  the  angle  A  HI;  from 
which  take  away  the  right  angle  AHC  90?,  and  the  remainder  = 
23?9^6T  is  the  absolute  measure  of  the  angle  CHI.  Hence  CI  may  be 
readily  found ;  as  thus  :-* 

As  radius  = 90?  Log.  co-8ecant=l 0.000000 

Is  to  the  hypothenuse  C  H  s=  66. 35  minutes,  Log.  =  .     .    .     1 .  821852 
So  is  the  angle  C  H I  =     .     23?9'.6r  Log.  sine  =      .    9.594572 

To  the  interval  or  base  C 1=  26. 09  minutes.  Log.  rs  .     .    .     1. 416424 
Time  of  second  bearing  =  2M0T.0 

Sum= 2*36r.09;  which  is  the  time  of  the  ship's 

nearest  approach  to  the  observer. 

Note. — ^This  interesting  problem  is  thus  worked  at  length,  trigono- 
metrically,  with  the  view  of  adapting  it  to  the  use  of  mariners  in 
general  j  though,  indeed,  in  such  cases,  calculation  need  not  be  resorted 
to,  since  the  solution  deduced  from  geometrical  construction  will  always 
be  sufiiciently  near  the  truth. 


Digitized  by 


Google 


PRACTICAI,  GUMNBRT.  557, 


SOLUTION  OP  PROBLEMS  IN  PRACTICAL  GUNNERY. 

Gonnery  is  the  art  of  projecting  balls  and  shells  from  great  guns  and 
mortars ;  of  finding  the  ranges  and  times  of  flight  of  shot  and  shells ;  and 
of  determining  the  different  degrees  of  elevation  at  which  those  bodies 
should  be  projected,  so  as  to  produce  the  greatest  possible  effect. 


Problbm  L 
Qiven  the  Diameter  of  an  iron  Ball;  to  find  its  Weight. 

RULS. 

The  diameter  of  an  iron  ball  of  9  lbs.  weight  is  4  inches,  very  nearly; 
and,  since  the  weights  of  spherical  bodies,  composed  of  the  same 
materials,  are  as  the  cubes  of  their  diameters,  (Euclid,  Book  XII., 
Prop.  18,)  it  wil^  be, — as  the  cube  of  4,  is  to  9  lbs. ;  so  is  the  cube  of  the 
diameter  of  any  other  iron  ball,  to  its  weight.  Hence  the  following  rule : — 

To  thrice  the  logarithm  of  the  diameter  of  the  given  ball,  add  the  con- 
stant logarithm  9. 148063 ;  and  the  sum  (abating  10  in  the  index,)  will  be 
the  logarithm  of  the  required  weight  in  lbs. 

Example  1. 

Required  the  weight  of  an  iron  ball,  the  diameter  of  which  is  6. 7  inches  ? 

Given  diameter  =  6.  7  5  thrice  its  log.  =     2. 478225 
Constant  log.  = 9.148063 

Weight  in  pounds  =  42. 295     Log.  =     .     1. 626288 

X  Example  2. 

Required  the  weight  of  an  iron  ball,  the  diameter  of  which  is  5. 54 
inches  ? 

Given  diameter  =:  5. 54 ;  thrice  its  log.  ==:   2. 230530 
Constant  log.  = 9.148063 

Weight  in  pounds  =  28.91      Log.  =     .     1.378593 

Note. — ^The  constant  logarithm  used  in  this  problem  is  expressed  by  the 
arithmetical  complement  of  the  logarithm  of  the  cube  of  4,  added  to  the 
logarithm  of  9. 


Digitized  by 


Google 


558  PllACtlCAL  fitNNERY, 

Problbm  II. 
Givefi  the  Weight  of  an  iron  Ball;  to  find  its  Diameter. 

This  problem  being  the  converse  of  the  last^  we  obtain  the  following 
logarithmieal  expression : — 

To  the  logarithm  of  the  weight  of  the  given  ball^  add  the  constant 
logarithm  0. 851937  ;  divide  the  sum  by  3^  and  the  quotient  1^11  be  the 
logarithm  of  the  required  diameter. 

Note. — The  constant  logarithm  given  in  this  rule  is  expressed  by  the 
arithmetical  complement  of  the  logarithm  of  9,  added  to  the  logarithm  of 
the  cube  of  4. 

Example  1. 

Required  the  diameter  of  a  42  lb*  iron  baU  ? 

Given  weight  sis  42  lb.     Log.  =     .    •    •    1 .  623249 
Constant  log.  = 0.85 1 937 


Divide  by  3)  2.475186 


Diameter  in  inches  ^  6. 685      Log.  ^    0. 825062 

Example  2. 
Required  the  diameter  of  a  24  lb.  iron  ball  ? 

Given  weight  =  24  lb.     Log.  =     •.,!.  3802 1 1 
Constant  log.  = 0.851937 


Divide  by  3)  2. 232148 


Diameter  in  inches  =  5.547     Log.  =:     0.744049^ 

Problbm  III. 
Given  the  Diameter  of  a  leaden  Ball;  to  find  Ue  Wdghi. 

RULB. 

A  leaden  ball  of  1  inch  in  diameter^  weighs  -r\  of  a  lb.;  wfaiclt^  reduced 
to  a  decimal  fraction,  is  .  2143,  very  nearly :  and,  as  the  weights  of  spherical 
bodies  are  as  the  cubes  of  their  diameters,  it  will  be,— as  the  cube  of  1,  is 
to  .  2143 ;  so  is  the  cube  of  the  diameter  of  any  other  leaden  ball,  io  its 
weight  in  lbs.    Whence  the  following  logarithmieal  rule  :*- 


Digitized  by 


Google 


BALLS  AND  SHSLLS.  559 

To  thrice  the  logarithm  of  the  diameter  of  the  given  leaden  ball,  add 
the  constant  logarithm  9. 331022 1  and  the  sum  (abating  10  in  the  index^} 
will  be  the  logarithm  of  the  required  weight. 

Example  I. 

Required  the  weight  of  a  leaden  ball>  the  diameter  of  which  is  6, 68 
inches  ? 

Given  diameter  :=  6. 68 ;  thrice  its  log.  =  2. 474331 
Constant  log.  = 9.331022 


Weight  in  pounds  =  63.88      Log.  =     1.805353 

Example  2. 

Required  the  weight  of  a  leaden  ball^  the  diameter  of  which  is  5, 32 
inches  ? 

Given  diameter  =  5.32;  thrice  its  log.  =  2. 177736 
Constant  log.  = 9.331022 


Weight  in  pounds  b  82.26      Log«  a     1.508758 

JNb/e.— The  constant  logarithm  used  in  this  problem  is  the  logarithm 
of  the  decimal  fraction  •  2143. 


Problbm  IV. 
Qioen  the  Weight  of  a  leaden  BaU;  to  find  its  Diameter. 

RtLB. 

Since  this  problem  is  merely  the  converse  of  the  last,  we  obtain  the 
following  logarithmical  expression;  viz.,  to  the  logarithm  of  the  weight  of 
the  given  leaden  ball,  add  the  constant  logarithm  0. 668978 ;  divide  the 
sum  by  3,  and  the  quotient  will  be  the  logarithm  of  the  required  diameter. 

Example  1. 

Required  the  diameter  of  a  64  lb.  leaden  ball  ? 

Given  weight  =  64  lb.      ...    Log.  j=  1. 806180 
Constant  log.  = 0.668978 

Divide  by  3)  2. 475 158 


Diameter  in  inches  a  6*68      Log*  ^    0.82505^1 

/Google 


Digitized  by ' 


500  PRACTICAL  GUMNBRT. 

Example  2. 

Required  the  diameter  of  a  32  lb.  leaden  ball  ? 

Given  weights  32  lb.      .    .    .    Log.  =  1.505150 
Constant  log.  = 0.668978 

Divide  by  3)  2. 174128 
Diameter  in  inches  =  5.305      Log.  =    0.724709^ 

I 

Note. — ^The  constant  logarithm  made  use  of  in  this  rule  is  the  arith- 
metical complement  of  that  used  in  the  preceding  rule. 


Probi^m  V. 

Given  the  intemal  and  estemal  Diameters  of  an  iron  Shell;  to  find 

Us  Weight. 

RULB. 

Find  the  difference  of  the  cubes  of  the  intemal  and  external  diameters 
of  the  shell,  to  the  logarithm  of  which  add  the  constant  logarithm 
9. 148063 )  and  the  sum  (abating  10  in  the  index,)  will  be  the  logarithm 
of  the  required  weight  in  pounds. 

Note. — ^The  constant  logarithm  used  in  this  rule  is  the  same  as  that 
given  in  Problem  I.,  page  557. 

Example  1. 

Let  the  external  diameter  of  an  iron  shell  be  12, 8  inches,  and  its 
internal  diameter  9. 1  inches ;  required  its  weight  ? 

12. 8  X 12. 8  X  12. 8=2097. 152,  cube  of  the  external  diameter. 
9. 1  X  9. 1 X   9. 1=  753. 571,  cube  of  the  internal  diameter. 

Differences     .     ,     1343.581         Log.= 3.128264 

Constant  log.  = , 9. 148063 


Weight  in  pounds  ^    188.94         Log.s 2.276327 

/Google 


Digitized  by ' 


BALLS  AND   8HBLL8.  561 

Example  2. 

Let  the  external  diameter  of  an  iron  shell  be  9. 8  inches^  and  its  internal 
diameter  7  inches ;  required  its  weight  ? 

9. 8 X  9. 8  X  9. 9=?94L  192^  cube  of  the  external  diameter. 
7     X  7     X  7     =343.  cube  of  the  inteitial  diameter. 

Difference  =..    698.192        Log.  = 2.776841 

Constant  log.  = 9. 148068 


Weight  in  poundss84. 12         Log.  = 1.924904 

Problem  VL 
To  find  how  tmich  Powder  will  fill  a  Shells 

RULB. 

To  thrice  the  logarithm  of  the  internal  diameter  of  the  shelly  in  inches, 
add  the  constant  logarithm  8.241845;  and  the  sum  (abating  10  in  the 
indeX;)  will  be  the  logarithm  of  the  pounds  of  powder. 

ExcmvpU  1. 

How  much  powder  will  fill  a  shell,  the  internal  diameter  of  which  is  9. 1 
inches  7 

Internal  diameter,  9. 1  inches ;  thrice  its  log.  =  .    •    •         .2. 677123 
Constant  log.  = 8.241845 

Powder,  in  pounds,  s  13, 15        Log.  =      ••••'••     1. 118968 

Example  2. 

How  much  powder  will  fill  a  shell,  the  internal  diameter  of  which  is  7 
inches  ? 

Internal  diameter,  7  inches ;  thrice  its  log.  = 2.535294 

Constant  log.  a= 8.241845, 

Powder,  in  pounds,  =s  5.986        Log.  =     .....    .0.777139 

Note.— The  constant  logarithm  made  use  of  in  this  problem  is  the  arith- 
metical complement  of  the  logarithm  of  57.3,  the  established  divisor  for 
filling  shells. 

2  o 


Digitized  by 


Google 


5fl2  PRACTICAL   OIJMIBRY. 

PaOBtBM  VII. 

To  find  iht  Size  of  a  SheU  to  contain  a  given  Weight  of  Powder. 

RULB. 

Thi8  problem  being  the  convene  of  the  last,  we  obtain  the  following 
logarithmical  eaipression : — 

To  the  logarithm  of  the  given  weight  of  powder,  in  pounds,  add  the 
constant  logarithm  1.758155;  divide  the' sum  by  3,  and  the  quotient  wiU 
be  the  logarithm  of  the  internal  diameter  of  the  shell,  in  inches. 

Example  1. 

Required  the  internal  diameter  of  a  shell  that  will  hold  13. 15  lbs.  of 
powder  ? 

Given  weight  =  13. 15  lbs.        Log.  =s      .    .     1 .  U8926 
Constant  log.  = 1.758155 

Divide  by  3)  2*  877081 
Internal  diameter,  in  inches,  =  9. 1        Log.  =  0. 959027 
Example  2. 

Required  the  internal  diameter  of  a  shell  that  will  hold  5. 986  lbs.  of 
p6wder? 

Given  weight  =  5. 986  lbs.        Log.  =     .    .    0.777137 
Constant  log.  =a 1.758155 

Divide  by  3)  2. 535292 
Internal  diameter,  in  inches,  =  7. 0        Log.  =0. 845097i 

Problbm  VIII. 
To  find  how  much  Powder  will  fill  a  rectangular  Bos. 

Ruus* 

To  the  logarithms  of  the  length,  breadth,  and  depth  of  the  box,  in 
inches,  add  the  constant  logarithm  8.52287.93  and  the  sum  (abatiqg  10 
in  the  index,)  will  be  the  logarithm  of  the  pounds  of  powder. 


Digitized  by 


Google 


POWDBR  AND  BOXUf.  56S 

Example  1. 

How  much  powder  will  fill  a  box,  the  length  of  which  is  15,  the  breadth 
1 2,  and  the  depth  10  inches  ? 

Length    s=   15  inches.        Log.  es       ...  1. 176091 

Breadth  ^  12    do.  Log.  b=      .    .     ,  1.079181 

Depth      =  10    do.  Log.  =       .    .    •  1.000000 

Constant  log.  s= 8.522879 

Powder,  in  pounds,  =  60. 0        Log.  =:•:!.  77815 1 

Example  2. 

Required  the  quantity  of  powder  that  will  fill  a  cubical  box,  the  side  of 
which  is  12  inches? 

Side  of  the  cubical  box  =12  inches.       Log.  ss    1. 079181 

•     Multiply  by  3 

Logarithmicalcubeof  12=      •     .    ,    .     .    .    3.237543 
Constant  log.  =    .     .•   . 8. 522879 

Powder,  in  pounds,  «  57. 6        Log.  as     «    .     1, 760422 

No^e.— The  constant  logarithm  made  use  of  in  this  problem  is  the  arith- 
metical complement  of  the  logarithm  of  30,  the  established  divisor  for 
filling,  rectangular  powder  boxes.         * 

Problem  IX. 
To  find  the  Size  of^  ciMcal  Box  to  contain  a  gio^  Weight  of  Powder. 

RULB. 

To  the  logarithm  of  the  given  weight  of  powder,  in  pounds,  add  the 
constant  logarithm  1.477121 ;  divide  the  sum  by  3,  and  the  quotient  will 
be  the  logarithm  of  the  side  of  the  box,  in  inches. 

Example  1. 
Required  the  side  pf  a  cubical  box  that  will  hold  60  lbs.  of  gunpowder  ? 

Given  weight  s  60  lbs.        Log.  s^  •    •    »    •     1.778151 
Connant  log.  = 1.477121 


Divide  by  3)  3. 255272 

Side  of  the  box,  in  inches,  12. 16        Log.  =     1. 085090f 

2o2 


Digitized  by 


Google 


564  PRACtlCAI.   GUNNBRY. 

Example  2. 

Required  the  side  of  a  cubical  box  that  will  hold  120  lbs.  of  gun- 
powder ? 

Givenweight=5  1201b8,        Log,  =     .    •    .    2.079181 
Constant  log.  =5    . 1.477121 

Divide  by  3)  3. 556302 


Side  of  the  box^  in  inches,  15.32      Log.  =     .     1.1 85434 

Note, — Since  this  problem  is  the  converse  of  the  last,  the  constant  loga- 
rithm made  use  of  is  the  logarithm  of  30,  the  established  divisor  and 
multiplier  for  filling  rectangular  boxes. 


Problem  X. 

.  Tojind  how  much  Powder  wiUJlU  a  Q/linder» 

RuL^. 

To  twice  the  logarithm  of  the  diameter  of  the  cylinder,  add  the  logarithm 
of  its  length  and  the  constant  logarithm  8.417937;  the  sum  (abating  10 
in  the  index,)  will  be  the  logarithm  of  the  pounds  of  powder. 

Example  1.' 

How  much  powder  will  a  cylinder  hold,  the  diameter  of  which  is  13 
inches,  and  the  length  26  inches  ? 

Diameter  of  the  cylinders  13  inches  $  twice  its  log.  =  2. 227886 
Length  of  ditto  =  .  .  26  ditto.  Log.  =  .  1.414973 
Constant  log.  = 8.417937 


Powder,  in  pounds,  =s     •     115.02        Log.  =     .    *2. 060796 

Example  2. 

How  much  powder  will  a  cylinder  hold,  the  diameter  of  which  is  4 
inches,  and  the  length  12  inches  ? 

Diameter  of  the  cylinder  =  4  inches  j  twice  its  log.  er  1. 204120 
Length  of  ditto  =  .  .  12  ditto.  Log.  .=  .  1.079181 
Constant  log.  =       . 8.417937 


Powder,  in  pounds,  =  5. 026        Log.  =  .    .    .    .    0.701288 


Digitized  by 


Google 


POIjVDBR   AND  CYLINDBES.  565 

Note. — The  constant  logarithm  made  use  of  in  this  problem  is  the 
arithmetical  complement  of  the  logarithm  of  38. 2y  the  established  divisor 
for  filling  cylinders  with  gunpowder. 


Problbm  XI. 

Tojind  what  Length  of  a  CyUnder  vM  be  filled  with  a  given  Weight 

of  Gunpowder. 

Ruj-B. 

To  the  arithmetical  complement  of  twice  the  logarithm  of  the  diameter 
of  the  cylinder^  or  caliber  of  the  gun,  add  the  logarithm  of  the  ^vei)  weight 
of  powder  iii  pounds,  and  tW  constant  logarithm  1.582063:  the  sum 
(abating  10  in  the  index^)  will  be  the  logarithm  of  the  length  of  the 
cylinder,  in  inches. 

Example  1. 

What  length  of  a  24-pounder  gun,  of  5. 66  inches  caliber,  will  be  filled 
with  8  lbs.  of  gunpowder  ?      .         . 

Caliber  of  the  gun  ±s  5.  66        Ar.  comp,  of  twice  its  log.  =  8.  494368 

Given  weight  of  powder=8  lbs.  Log.  =s •    0.903090 

Constant  log.  s  •••... 1.582063 

Length,  in  inches,  =  9.539     Log.  s     ,    ....    .    .    0.979521 

Example  2. 

What  length  of  a  42-pounder  gun,  of  S.  23  inches  caliber,  will  be  filled 
with  1  Of  lbs.  of  gunpowder  ? 

Caliber  of  the  gun  =  6. 23  *  Ar.  comp.  of  twice  its  log.  ss  8. 411024 
Given  weight  of  powder=  10. 666  Log.  =  .....  1. 028002 
Constant  Jog.  =:  ......•».• 1.582063 

Length,  in  inches,  =  10.497  Log.  =      .    •    .    ^    .     1.021089 

Note, — ^This  problem  being  the  converse  of  the  last,  the  constant  loga- 
rithm is  the  logarithm  of  38. 2,  the  established  divisor  and  multipiur  for 
filling  cylinders  with  gunpowder. 


Digitized  by 


Google 


586  PRACTICAL  OtTVNBRY* 

Problbm  XII. 
Tojmd  the  Numbisr  of  Balk  in  a  triangular  Pile, 

RUJLB« 

To  the  logarithm  of  the  number  of  balls  in  the  bottom  row,  add  the 
logarithm  of  that  number  increased  by  1,  and  also  its  logarithm  increased 
by  2y  and  the  constant  logarithn>  9.  221849  :  the  sum  (rejecting  10  in  the 
index,)  will  be  the  logarithm  of  the  required  number  of  balls. 

Example  !• 

Required  the  numWr  of  balls  in  a  triangular  pile,  each  side  of  its  base 
containing  30  balls  ? 

Balls  in  one  side  of  the  base  =  30  Log.  =  1.477121 
Ditto,  increased  by  1,  =  •  .  31  Log.  =  1.491362 
Ditto,  increased  by  2,  =  .  .  32  Log.  ^  1.505150 
Constant  log.  33 9.221849 


Number  of  balls  =t  .    .    .    4960        Log.  =  3.  695482 

Example  2. 

Required  the  number  of  balls  in  a  triangular  pile,  each  side  of  its  base 
c6ftUuiiin|(20baU8}      ' 

Balls  in  one  side  of  the  base  =  20  Log.  s  L  301030 
Ditto,  increased  by  ),  =  •  .  21  Log.  =  1.322219 
Dittoi  inertesed  by  2,  e=  .  .  22  Log.  ^  1.34S42S 
G>nstant  log.  =     .......»»•    9.221849 


Niimfaecofb«IIa»    .    .    .    1540-     Logi  »  S.  187521 

^^•^^The  constant  logarithm  employed  in  this  problem  is  the  aritli- 
metical  complement  of  the  logarithm  of  6,  the  established  divisor  for 
triangular,  square,  and  rectangular  piles  of  shot 


Digitized  by 


Google 


To  find  the  Number  of  Balk  in  a  square  Pikr 

Rule. 
To  the  logarithm  of  the  number  of  balls  in  one  side  of  the  bottom  row, 
add  the  logarithm  of  that  number  increased  by  I,  the  logarithm  of  twice 
the  same  number  increased  by  1,  and  the  constant  logarithm  9.  221849  : 
the  sum  (abating  10  in  the  index,)  will  be  the  logarithm  of  the  required 
number  of  balls. 

Example  1. 

Required  the  num*ber  of  balls  in  a  square  pile,  each  side  of  its  base 
containing  30  balls  ?  . 

Balls  in  one  side  of  the  base  :z  30  Log.  =:  1. 47f  121 
Ditto,  increased  by  1,  =^  31  Log,  z::  L  491362 
Twice  ditto,  increased  by  1,  z:  61  Log,  =.  1.785330 
Constant  log.  =    .     • 9.221849 


Number  of  balls  =     .     .        9455         Log,  ;=:  3.  975662 
Example  2. 

Required  the  number  of  balls  in  a  square  pile,  each  side  of  its  base 
containing  20  balls  ? 

Balls  in  one  dde  of  the  base  =  20  Log.  ==  1.301030 

Ditto,  increased  by  1,  =  .     .     21  Log.  z=  1.322219 

Twice  ditto,  mcreased  by  1,  ss  41  Log.  =:   1. 612784 

Constant  log.  =  .     .    .     ...  .     .     .     .    9.221849 

Mimberofbftll*-:   •    ,    •    2870        Log.  =;  i3.457S82 

Probwm  XIV, 
To  find  the  Number  of  Balls  in  a  rectangular  Pile. 

RULB. 

From  three  times  the  nnmber  of  balls  contained  in  the  loigtfa  of  the 
bottom  row,  subtract  the  number  of  balk,  less  by.  1,  contained  in  the 
breadth  of  that  row ;  then,  to  the  logarithm  of  the  remainder,  add  the 
logarithm  of  the  number  of  balls  eontained  in  the  breadth  of  the  bottom 
row,  the  lognrtthm  of  l^at  number  increased  by  1,  and  the  constant  lo|;a- 
rithm  9. 221849*:  the  sum  (rejecting  10  in  the  index,)  will  be  th^e  logarithiQ 
of  the  required  number  of  balls. 


Digitized  by 


Google 


568 


PRACTICAL   GUNMBRy. 


Example  1. 

Required  the  number  of  balls  in  a  rectangular  pile,  which  contains  46 
balls  in  the  base  row  of  its  longest  side,  and  15  balls  in  that  .ofi  cs  shortest 
side? 

Balls  in  length    46  x  3=  138 

Balls  in  breadth  15—1=   14 


Remainder  =  .  •  .  124 
Balls  in  breadth  row  =t  15 
Ditto,  increased  by  1,  =  16 
Constant  log.  !=..«•. 

Number  of  balls  =     .    4960 


Log. 
Log. 
Log. 


2. 093422 

1. 176091 

1.204120 

.9.221849 


Log.  ==  3.695482 


Example  2. 
Required  the  number  of  balls  in  a  rectangular  pile,  which  contains  59 
balls  in  the  base  row  of  its  longest  side,  and  20  balls  in  that  of  its  shortest 
side  ? 

Balls  in  length    59*x3=:177 
Balls  in  breadth  20-1=  19 

Remainder  =     .    .    .     158        Log.  =  2.  198657 

Balls  in  breadth  row  =     20        Log.  =  1.301030 

Ditto,  increased  by  1,  =  21         Log.  =  1.322219 

Constant  iog.  = 9.221849    . 


Number  of  balls  =      11060        Log.  =    4.048755 

Problem  XV. 

To  find  the  Number  qf  Balls  in  an  incomplete  triangular  PUe. 

Rule. 
Find  the  number  of  balls  in  the  whole  pile,  considered  as  complete,  by 
Problem  XII.,  page  566 ;  and  find  also,  by  the  same  problem,  the  number 
of  balls  answering  to  the  triangular  pile,  the  side  of  whose  base  is  repre- 
sented by  the  number  of  shot  in  the  side  of  the  top  course  of  the  incom- 
plete pile  diminished  by  1 ;  then,  the  difference  of  the  two  reiults  will  be 
the  number  of  shot  remaining  in  the  pile. 

Example. 

.  Required  the  number  of  shot  in  an  incomplete  triangular  pile ;  each  side 
of  its  bottom  course  containing  40  balls,  and  each  side  of  its  top  course 
conUuning  20  balls  ? 


Digitized  by 


Google 


PJtlNG  PF  BALLS,  569 

To  find  the  Number  of  Balls  in  the  complete  Pile : — 

Balls  in  one  side  of  bottom  course  =  40         Log.  =  1 .  602060 

Ditto^  increased  by  1^  ^      .*'   .    .,     41         Log.  =  1.612784 

Ditto,  increased  by  2,  =      ....    42         Log.  =  1.  623249 

Constant  log.  =±       .     . 9«^221849 


.  Number  of  balls  for  the  whole  pile  ^  11480  Log.  =  4.059942 
To  find  the  Number  of  Balls  deficient  :-^ 

Balls  in  each  side  of  top  course=20- 1  =  19  Log.= 1 .  ^78754 
Diminished  course,  or  19,  increased  byl,=20  Log.zs  ] .  301030 
Ditto,  increased  by  2,  =  ....  22  Log.=  1.322219 
Constant  log.  =      .     • 9.221849 

Number  of  shot  wanting  =    •    .     .      1330      Log.=3.  123852 
Now,  11480— 1330=10150  is  the  number  of  shot  in  the  incomplete  pile. 

Problem  XVL 
To  find  the  Number  of  Balls  ui  an  incomplete  square  Pile. 

Rule. 

Find  the  number  of  balls  in  the  whole  pile,  considered  as  complete,  by 
Problem  XIII.,  page  567  ;  and  find  alsoj  by  the  same  problem,  the  number 
of  balls  answering  to  the  square  pile,  each  side  of  whose  base  is  represented 
by  the  number  of  shot  in  each  side  of  the  top  course  of  the  incomplete  pile 
diminished  by  1 }  then,  the  difference  of  the  two  results  will  be  the  number 
of  shot  remaining  in  the  pile. 

Example. 

Required  the  number  of  shot  in  an  incomplete  square  pile ;  each  side  of 
its  bottom  course  containing  24  balls,  and  each  side  of  its  top  course  8 
balls? 

To  find  the  Number  of  Balls  in  the  complete  Pile : — 

Balls  in  one  side  of  the  base  =  .24  Log.  =  1. 38021 1 
Ditto,  increased  by  1,  =  ...  25  Log.  =  1.397940 
Twice  ditto,  increased  by  1,  =  .  49  Log.  =  1.690196 
Constant  log.  =       «... 9.221849 

Number  of  balls  for  the  whole  pile  =  4900    Log.  =  3. 690196 


Digitized  by 


Google 


570  PRACrtCAL  OlfKKBRY. 

To  find  the  Number  of  Balk  deficient  :-*- 

Balls  in  each  side  of  top  course  =  8—1  =7  Log.=0. 845098 
Diminished  course,  or  7^  increased  by  1^=8  Log.=0.  903090 
Twice  ditto,  increased  by  1,  =  .  .  15  Log.=l.  176091" 
Constant  log.  =   ............       9.221849 

Number  of  balls  wanting  =     .  .140         Log. = 2.  1 46 1 28 

Now,  4900— 140  =  4760  is  the  number  of  shot  in  the  incomplete  pile. 

PBOBtBM  XVII. 

To  find  the  Nuvuber  o/BqIIs  in  an  tncomplete  rectangular  Pile. 

RULB. 

Find  the  number  of  balls  in  the  whole  pile,  considered  as  complete^  by 
Problem  XIV.,  page  567 ;  and  find  also,  by  the  same  problem,  the  number 
of  balls  answering  to  the  rectangular  pile,  whose  ?ides  are  represented  by 
the  respective  sides  of  the  top  course  of  the  incomplete  pile,  the  number  of 
shot  in  each  side  being  diminished  by  1  ;  then,  the  difference  of  the  two 
results  will  be  the  number  of  shot  remaining  in  the  pile. 

Example, 
Required  the  number  of  shot  in  an  incomplete  rectangular  pile;  the 
length  of  its  bottom  course  being  40  balls,  its  breadth  20,  and  the  length 
of  its  top  course  29  balls,  and  its  breadth  9  > 

To  find  the  Number  of  Balls  in  the  complete  Pile  :~ 
Bottom  course,  40   X   3  »  120 
Breadth,  20  --   1  =     19 

Remainder  s      ....     101  Log.^  2, 004321 

Balls  in  breadth  row  =      .       20  Log.=  1.301030 

Ditto,  increased  by  1,  :^    .      21  Log-=:  1.322219 

Constant  log.  =: \  .    .    .    9.221849 

Number  of  balk  fiw  whole  piles7070  Log.s3»  8494 19 

To  find  the  Number  of  Balls  deficient  :— 
Top  row,  29-1  =  28  X  3  ='84 
Breadth,    9-1  =    8  -  1  sb    7 

Remainder  = 77      Log.s:  1.886491 

Balls  in  breadth  row  =  .  •  6  Log.=  0. 903090 
Ditto,  increased  by  1,  =  .  .  9  Log.=  0. 954243 
Constant  log.  s=      ».••«,,»    9.221849 

.  Number  of  balls  wantmg  s  924   .  Log.ss  2. 965673 


Digitized  by 


Google 


VBLOC1TIB8   OF.  SHOT  AND  SHELLS.  571 

Now,  7070  —  924  =  6146  is  the  number  of  shot  in  the  incomplete 
pile.  . 

Note.^n  triangular  and  square  piles,  the  number  of  horizontal  rows  or 
courses  is  always  equal  to  the  liumber  of  balls  in  one  side  of  the  bottom 
row ;  and,  in  rectangular  piles,  the  number  of  horizontal  rows  is  equal  to 
the  number  of  balls  in  the  breadth  of  the  bottom  row.  In  these  piles,  the 
number  of  balls  in  the  top  row  or  edge  is  always  one  more  than  the  differ- 
ence between  the  number  of  balls  contuned  in  the  length  and  the  breadth 
of  the  bottom  row. 


PfiowsM  XVIIL 

To  find  the  Felocity  of  any  Shot  or  Shell, 

RULB. 

.  From  the  logarithm  of  twice  the  weight  of  the  charge  of  powder,  in 
pounds,  subtract  the  logarithm  of  the  weight  of  the  shot :  to  half  the 
remainder  add  the  constant  logarithm  3.  204120,  and  the  sum  (rejecting 
5  in  the  index,)  will  be  the  logarithm  of  the  velocity  in  feet,  or  the 
number  of  feet  which  the  shot  or  shell  passes  over  in  a  second. 

Example  1. 
With  what  velocity  will  a  24-poandB  ball  be  projected  by  8  lbs.  of 
powder? 

Twice  the  charge   s     16  lbs.        Log.  s  1. 2D4120  • 
Weight  of  the  shot  =  24  lbs.     '  Log.  s  1.380211 

Remainder  = 9. 823909 

.Half  the  remainder  ^      .     .    ...     ...     4.9119544 

Constant  log.  =      .     .     .    .     .     .     .     .    3.204120 

Velocity  of  shot,  in  feet,i:  1306        Log.  f=  3. 1 16074| 

Example  2. 
With  what  velocity  will  a  13-inch  shell,  weighing  196  lbs.>  be  discharged 
by  9  lbs.  of  powder  ?  ' 

Twice  the  char^  s     18  lbs.        Log.  s  1.255273  • 

*Weightofthe  shell  =19(5  lbs.        Log.  =  2.292256      ' 

Remainders 8.963017 

Half  the  remainder  ==       ......    4.481508^ 

Constant  log.  = 3.204120 

Velocity  of  sheH,  in  feet,  485        Log.  a  2.685626^ 


Digitized  by 


Google 


572  PRACTICAL  GUNNBAY. 

Noie. — ^The  constant  logarithm  made  use  use  of  in  this  problem  is  the 
logarithm  of  1600  feet,  which  is  the  velocity  ifcquired  by  a  1  lb.  ball,  vrhen 
fired  with  8  ounces  of  powder. 


Probj^m  XDC 

To  find  the  terminal  Velocity  of  a  Shot  or  Shell;  that  »,  the  greaiegt 
Velocity  it  can  acqukre  in  descending  through  the  Air  by  its  cum 

Weight. 

RULB. 

For  Balls.-^-To  half  the  logarithm  of  the  diameter  of  the  ball,  in  inches, 
add  the  constant  logarithm  2.  244277  ;  and  the  sum  will  be  the  logarithm 
of  the  terminal  velocity  of  tUe  ball. 

And,  for  Sliells. — ^To  half  the  logarithm  of  the  external  diameter  of  the 
shell,  in  inches,'  add  the  constant  logarithm  2. 168203  3  and  the  sum  will 
be  the  logarithm  of  the  terminal  velocity  of  the  shell. 

Example  1. 

Required  the  terminal  velocity  of  a  24  lbs.  ball,  its  diameter  being  5. 6 
inches?  '  .  . 

Diameter  of  the  ball    =    5.6        Log.  s  0.748188. 


Half  the  log.  = 0.874094 

Constant  log.  =^      «.......»    2. 244277 


Terminal  velocity  r±     .    415        Log.  ==  2.618871 
Example  2. 

Required  the  terminal  velocity  of  a  shell  weighing  196  lbs.,  its  external 
diameter  being  1 2^  8  inches  ? 

Diameter  of  the  shell  =^  12. 8         Log.  c   1. 1Q7210 


Half  the  log.  =r       ....:...    0. 553605 
Constant  log.  = .2. 168203 


Terminal  velocity  =    527     .    .    Log.  =  2.721808 

Note. — The  constant  logarithms  made  use  of  in  this  problem  are  the 
respective  logarithms  of  175.5  and  147.3,  the  established  multipliers  for 
shot  and  Shells.  It  is  by  this  problem  that  the  terminal  velocities  contained 
in  Tables  A  and  B,  following^  have  been  computed. 


Digitized  by 


Google 


VBLOCITIBS  OF   SHOT  AND   SDBLLS.  573 


PilOBLBM  XX. 

To  find  the.  Height  from  which  a  Body  mustfaUf  in  vacuo^  in 
.   order  to  acquire  a  given  Velocity. 

.     R(7LB. 

Since  the  spaces  descended  by  falling  bodies  are  as  the  squares  of  the 
velocities,  and  as  a  fall  of  16tV  f®«^  produces  a  velocity  of  32}  feet, — 
therefore,  as  the  square  of  32 J  feet,  is  to  16^^  feet;  so  is  the  square  of 
any  other  given  velocity,  to  the  altitude  from  which  it  must  fall,  to  acquire 
such  velocity.     Hence  the  following  logarithmical  expression : — 

To  twice  the  logarithm  of  the  given  velocity,  in  feet,  add  the  constant 
logarithm  8;  191564  ;  and  the  sum,  (abating  10  in  the  index^)  will  be  the 
logarithm  of  the  required  altitude,  or  height 

Example  1. 

From  what  height  must  a  body  fall,  in  order  to  acquire  a  velocity  of 
1340  feet  per  second? 

Given  velocity  =  1340^  Iwice  its  log.  =   6.254210 
Constant  log.  = 8.191564 


Altitude,  or  height,  = '  2791 1         Log..  =  4. 445774 

Example  2. 

From  what  height  must  a  body  fall,  in  order  to  acquire  a  velocity  of  1670 
feet  per  second? 

Given  velocity  s:   1670;  twice- its  log.  =  6.445434 
Constant  log.  r=      ........    8. 191564 

Altitude,  or  height,  =  43352        Log.  =  4. 636998 

Note — It  is  by  this  problem  that  the  altitudes  in  Tables  A  and  B, 
following,  have  been  computed;  but,  since  the  fractional  parts  beyond  J 6 
and  32  were  omitted,  and  the  constant  logarithm,  in  consequence  thereof, 
assumed  at  8. 193820,  the  respective  altitudes,  in  these  Tables,  are  some- 
thing beyond  the  truth. 


Digitized  by 


Google 


574 


PRACTICAL   60NNBRT. 


CONCISE  TABLES 

FOR  DETERMINING  THE  GREATEST  HORIZONTAL  RANGE  OF  A  SHOT  OR  SHELL,  1IFHE>' 
PROJECTED  IN  THE  AIR  WITH  A  GIVEN  YELOCITY; 

TOG&TBER  WITH  THE  ELEVATION  OF  THE  PIECE  TO  PRODUCE  THAT  RANGE. 

Table  A. — For  Great  Guns. 


Weight  of 
Shot. 

Diameter,  in 
inches. 

Terminal 
Velocity. 

Log^tbm. 

Altitude. 

Logarithm. 

1. 

1.94 

244 

7.612610. 

930 

2.^69483 

2 

2.45 

275 

7.  560667 

.     1182 

3.072618 

3. 

2.80 

294 

7.531653 

1360 

3. 133539 

4 

3.08 

308 

7.511449 

1482 

3. 170848 

6 

3.53 

330 

7.481486 

1701 

3.230704 

9 

4.04 

353     . 

7. 452225 

1958    . 

3.291613 

•  12 

4.45 

370 

7.431798- 

2139 

3.330211 

18 

5.09        < 

396 

7.  402305 

2450 

3.389166 

24 

5.60 

415 

7.381952 

2691 

3.429914 

32 

6.17. 

436 

7.  .360514 

2970 

3,472756 

36 

6.41 

444 

7.352617 

3080 

3.48KMi 

42 

6.75 

456 

7, 3*41035 

3249       • 

3.511750 

Table  B. — For  Mortars. 


SizeofSlien. 
in  inches. 

Weight  of 
Shells  filled. 

Diameter,  in 
inches.' 

Terminal 
Velocity. 

Logarithm. 

Altitude. 

Logarithm. 

f 

10 
13 

9 

18 

47 
•91§ 
201 

4.53 
5.72 
7.90 
9.84 
12. 80 

314 
352 
414 
i62. 

%2r 

7.503070 
7.453457 
7.383000 
7. 335358 
7.278189 

1541 
19S6 
2678 
3335 
4340 

3. 187803  , 
3.286905 
3.427811 
3. 523096 
3.637490 

Table  C^For  Great  Guns  and  Mortars. 


Initial  Velocity, 

divided  by 
TerminalVclocity 

Lpgarithm. 

Elevation. 

Range  divided  by 
Altitude. 

Legarithm. 

0.6910 

9. 839478 

44<>  0' 

0.3914 

9.592621 

0.9i45 

.  9.975202 

43.15 

0.&850 

0.767166 

1. 1980 

0.  078457 

42.30 

0.  7787 

9.  891370 

L4515 

0.161817 

41.45 

0.9724 

'9.987845 

1.7050 

0.231724 

.     41.   0 

1. 1661 

0.066736 

1.  9585 

0.  291924 

40.15 

1.3590 

0. 133475 

2.2120 

0.344785 

39.30 

1.5535 

0.191311 

"2.4655 

0. 391905 

38.45 

.     1.7472 

0.24234a 

.  2.7190 

0. 434409 

38.  0 

1.^09 

0.288003 

2.9725 

0.473122 

37.15 

2. 1346 

0,329317 

3.2260 

0.508664 

36.30 

2. 3283 

0.367039 

3.4795 

0.541517 

35.45 

.   2.5220    ' 

0.401745 

.    3.733a 

0.572058 

35.  0 

2.  7157 

0.433882 

3. 9865 

0.  600592 

34.15 

2.9094 

0.463S03 

4.2400 

0.  627:^66 

33.30 

3.1031 

0.  491796 

4.4935 

0.652585 

32.45      . 

^.2968 

0. 518093 

4. 7470 

.  0.676419 

32.  0 

3.4905 

0.542888 

5.0000 

0.  698970 

31.15. 

3.6842 

0.566343 

JVb^e.— These  Tables  are  deduced  from  those  given  id  the  third  voltime 
of  Dr.  Huttoh's  "  Course  of  Mathematics.'* 


Digitized  by 


Google 


RANGES   OF  BAU.8  AND   8HBtI^.  575 


Problem  XXI. 


Tojind  the  greatest  Range  of  a  Ball  or  Shell,  and  the  Eleoafion  of  the 
.      Piece  to  produce  that  Range. 

Rule. 

Enter  Table  A  or  B^  and  take  out  the  logarithm  of  the  tenninal  velocity 
answering  to  the  given  ball  or  shell,  as  the  case  may  be^  and  also  the 
logarithm  of  th6  corresponding  altitude ;  then. 

To  the  logarithm  of  the  velocity  with  which  the  ball  or  shell  is  projected, 
add  the  logarithm  of  its  terminal  velocity ;  and  the  sum  (abating  10  in  the 
index,)  will  be  the  logarithm  of  the  quotient  of  *the -initial  velocity  of  the 
ball  or  shell,  divided  by  its  terminal  velocity.  With  this  logarithm,  enter 
the  second  column'  of  Table  C,  and  iji  the  adjoining  or  middle  qolumn  will 
be  .found  the  corresponding  degree  of  elevation  to  produce  the  greatest 
range ;  abreast  of  which,  in  the  last  column  of  the  same  table,  will  be  found 
the  logarithm  of  the  range  divided  by  the  altitude.  Now,  to  this  logarithm 
add  th^  logarithm  of  the  altitude  taken  from  Table  A  or  B,,  as  above' 
directed;  and  the  sum  will  be  the  logarithm  of  the  greatest  range. 

Note^-^K  great  accuracy  be  required,  proportional  parts  must  be  taken 
for  the  excess  of  the  given  above  ^e  next  less  tabular  numbers  in  Table  C. 

Example  1. 

Let  it  be  required  to  find  the  greatest  range  of  a  24  lb.  ball,  when 
discharged  with  a  velocity  of  1640*  feet,  and  the  elevation  of  the  piece  to 
produce  that  range  ?  : 

Log«  of  terminal  velocity  of  a  24  IK  ball/  Table  A,  ss  7. 381952 
Given  velocity  of  the  balls:  1640     .     .     /    Log.s:3. 214844      • 

Ans>«reririg to  which,  in  Table  C,  is  34«  \5\      .  Log.=0. 596796 

« 
Log.  of  corra^onding  altitude.  Table  A,  =    •    «    .-  3. 429914 
Abreast  of  34?  15  C,  in  last  column  of  Table  C,  stands  0. 463803 

Greatest  range,  in  feet^  =  7829 .     .    •    .    .    Log.B3. 8937 17 

Hence  the  |;reatest  range  of  a  24  lb.  ball,  when,  projected  with  a  velocity 
of  1640  feet,  is  7829  feet^  which  is  nearly  an  English  mile. and  a  half]  and 
the  elevation  to  produce  that  range,  is  34  ?  1 5  ^ 


Digitized  by 


Google 


576  PRACTICAL   GUNNSRT. 

Example  2. 

Let  it  be  required  to  find  the  greatest  range  of  a  13-inch  shell,  when 
projected  with  a  velocity  of  2000  feet  per  second,  and  the  eleTation  to 
produce  that  range  3  the  diameter  of  the'sl^ell  being  12.^0  inches  ? 

Log.  of  terminal  veloitity  of  a  13^  inch  shell.  Table  B,=s7. 278189 
Given  velocity  of  the  shell  =  200(r     .     .     .    Log.=3. 301030 

Answering  to  wliich,  in  Table  C,  is  34^49:        Log.=0. 579219 

Log.. of  corresponding  altitude,  Table  B,  =    •     .     .    3. 637490 
Corresponding  to  34?49',  in  Table  C,  is  .     .     •     .    .0. 441196 

Greatest  range,  in  feet,  =  1 1986     ....     Log.=4.t)78686 

Hence  the  greatest  range  of  a  13*inch  shell,  when  projected  with  a 
velocity  of  2000  feet,  is  1 1986  feet,  which  is  2i  miles  and  106  feet;  and 
the  elevation  to  produce  that  range,  is  34?  49'. 

-   Not^. — In  this  example,  proportion  is  made  for  the  excess  of  the  given 
above  the  next  less  numbers  in  Table  C. 


Problbm  XXII. 

Gioefi  the  Range  at  one  Eletaiion;  to  find  the  Range  at  another 

Elevation. 

RaLB.' . 

As  the  logarithmic  sine  of  twice  the  first  elevation,  is  to  the  logarithm 
of  its  corresponding  range  $  so  is  the  logarithmic  sine  of  twice  the  other 
elevation,  to  the  logarithm  of  its  correspondii^  range. 

Example  I.,  - 

If  a  13-i;ich  shell  be  found  to  range  1 1986  feet,  when  discharged  at  an 
elevation  of  34?49%  how  far  will  it  orange  when  the  elevation  is  45  degrees; 
the  charge  of  powder  being  the  same  at  both  elevations  ? 

As  twice  34?49^   =   69?38r        Log.  co-secantn  10. 028036 

Is  to  its  ranges  11986  feet,         Log.  =     .    •    .    4.078674 

.  Soi8twice45?0'   =  90?  0'        Log.  sine  =  .    .  10.000000 


To  the  required  range,  in  feet,  =  12785        Lojj.  =  4. 106710 

/Google 


Digitized  by ' 


.  RANGES   OF   SHBLtS.  577 

Example  2. 

If  a  shell  be  found  to  range  .4760  feet;  when  discharged  at  an  elevation 
of  45  d^rees^  how  far  will  it  range  when  the  ^elevation  is  30^5'  |  the 
charge  of  powder  being  the  same  at  both  elevations  I 

As  twice  45?= 90?  O:-    Log.  co-secant=s  10. 000000 

Is  to  its  range  =       ....     .    4760  feet, Log.  =       .    .    3.677607 
So  is  twice.30?45;  =    ....    61?30^     Log.  sine  =     .    9.943899 

To  the  required  range,  in  feet,  =     4183        Log.  s       ,  *  •    3. 621506 


Problem  XXIIL 

Given  the. Elevation  for  one  Range;  to  find  the  Eletmtionfor  another 

Range.  • 

Rule.  • 

As  the  logarithm  of  the  first  range,  is  to  the  logarithmic  sine  of  twice.its 
corresponding  elevation ;  so  is  the  logarithm. of  any  other  given  range,  to 
the  logarithmic  sine  of  an  arch.  Now,  the  half  of  this  arch  will  be  the 
elevation  required.  " 

Example  1. 

If  a  shell  be  found  to  range  11986  feet. when  projected  at  an  elevation 
of  34?49'.,  at  wh^t  elevation  must  it  be  discharged  to  strike  an  object  at 
the  distance  of  1378.5  feet,  with  the  same  .charge  of  powder  ? 

As  the  first  range  =:  11986  Log.  ar.comp.=5. 921326 
Is  tA  twice  34?49^  =  69?38C  Log.  sine  =  .  9.971964 
So  is  the  other  range=12785    .Log.  =     .    .    4,106710 

Toanchrr    •    •    .    90?  Oi     Log.  sine  a  .  10. 000000 

Half  the  arch  =:      •    45?  0^,  the  elevation  required. 

Example  2.  ' 

If  a  shell  be  found  to  ranjge  4760  feet  when  discharged  at  an  elevation 
of  45?,  at  what  elevation  must  it  be  projected  to  strike  an  object  at  the 
distance  of  4 1 83  feet,  with  the  same  charge  of  powder  ? 

2  p 


Digitized  by 


Google 


578  PRACnCAi;  GUNNSR7* 

As  the  first  range  =     4760        Log.  ar.  t:oinp.=6. 322393 
Is  to  twice  45?  =:         90?  Log.  sine  s      10. 000000 

So  is  the  other  Taiige=s4183        Log.  sine  s  « .  3. 621488 


To  arch  =     •    .    61?29:45r  Log.  tifte  »  •    9.943881 


Half  the  arch  =  .    30?44^52^^9  the  elevation  required, 

.  Problem  XXIV. 
Qioen  the  Charge  for  one  Range;  to  find  the  Charge  for  awAher  Range. 

RULK. 

Since  th^  ranges  at  the  same  elevation  are  nearly  proportional  to  the 
charges^  therefore — ^As  the  logarithm  of  the  first  range,  is  to  the  logarithm 
of  its  corresponding  charge ;  so  is. the  logarithm  of  the  other  range^  to  the 
logarithm  of  the  charge  corresponding  thereto. 

Exaifnpte  1* 

If,  with  a  charge  of  12  lbs.  of  powder,  a  shell  range  5334  feet»  what 
charge  will  be  sufficient  to  throw  it  2667  feet ;  the  elevation  being  45?  in 
both  cases  ?         - . 

As  the  iirst  tange  ss  «  «  5334  Log.  ar.  eoinp.sc6. 272947 
Istoitscha^eto  ,  «  .*  12  lU  Log. »  .  .  1.0791S1 
So  is  the  odier  range  ss    .    2667    Log.  a     .    .    3. 426023 


To  the  required  charge^  in  lb8.=:6«  0  Log.  =?      «    «    0. 77815 1 

* 

If,  wtthacharge.of  dibs,  of  powder,  a  shell  range  4000  feet,  what 
charge  will  be  sufficient  to  throw  it  8000  feet ;  the  elevation  being  45?  in 
both  cases  ?        •  .  • 

As  the  first  range  =  •  .  4000  Log.  ar.  comp.=6. 397940 
Is  to  its  charge. =  •  .  .9  lbs.  Log.  =  .  .  0.954243 
So  is  the  other  range  s    ,3000    Log.*      ,    ,3*477121 

To  the  required  charge,  in  Ib8.ss6. 75  Log*  »      «    t    0. 829304 


Digitized  by 


Google 


KAM6B8  OF  SHBIXk:  579 

Problbm  XXV. 
Gwen  the  Range  for  one  charge;  tojwd  the  Range  far  another  Charge. 

Ruis. 

As  the  logarithm  of  the  first  charge^  is  to  the  logarithm  of  its  corre- 
sponding ranges  so  is  the  logarithm  of  the  other  charge^  to  the  logarithm 
of  its  coitesponding  r^g^ ;  the  elevation  being  the  same  in  both  ease6« 

Example  K 

If  a  shell  be  projected  5334  feet  by  a  charge  of  12  lbs,  of  powder^  at 
what  distance  will  it  strike  an  object  when .  discharged  with  6  lbs.  of 
powder;;  the  elevatiaH  being  the  same  in  both  cases  ? 

As  the  first  charge  =:  '  •  12  lbs.  Log.  ar.  comp.==8. 920819 
Is  to  its  range  ==  •  .  .  5334  Log.  =  .  .  3.727053 
So  is  the  other  charge  ?s      6  lbs.    Log.  as    «    •    0.  .778 15 1 

T^the  required  range,  in  ft.,= 2667     Log.  =     .    •    3*  426.023. 

Example  2, 

If  a  shell  be  projected  4000  feet  by  a  charge  of  9  lbs.  of  powder^  at  what 
distance  will  it  strike  an  object  when  discharged  with  6i  lbs.  of  powder; 
the  elevation  being  the  same  in  both  cases? 

As  the  first  charge  =  .  •  9  lbs.  Log.*at.corop.=9. 045757 
Is  to  its  range  =•.••■  4000  Log.  s  ,  .  8. 602060 
So  is  the  other  charge  s     .    6. 75     Log.  =     .    .    0. 829304 


To  the  required  range,  in  ft,,  =  3000    Log.=     •    •    3.477121 

— 

•  PaoBiBM  XXVL 
Given  the  Range  and  tlie  Elevation;  to  find  the  Impetus. . 

Rule. 

As  the  logarithmic  sine  of  twice  the  angle  of  elevation,  is  to  the  loganthm 
of  half  its  corresponding  range ;  so  is  radius,  or  the  logarithmic  sine  of 
90?,  to  the  impetus^ 

2p2 


Digitized  by 


Google 


S8Q  PKAtrriCAL   flUNNBRT. 

■ 

Example  L 

With  Vfh9t  impetus  must  a  shell  be  discharged  at  an  elevatidn  of  34?49^^ 
to  strike  an  object  at  the  distance  of  2986  feet  ? 

As  twice  34^49^  =  .  69?38^  Log.  co-secants  10. 028036 
Is  to  half  the  range  =s  1493  Log.s:.  .  ..  3.1740^ 
So  is  radius,  or  ...       90?       Log.sine=      .  10.000000 

To  the  impetos,  in  feet,  ss  1592      Log.  s  •    .    .    3. 202096 

Example  2. 

With  what  impetus  must  a  shell  be  discharged  at  an  elevation  of  25?,  to 
strike  an  object  at  the  distance  of  2760  feet  ? 

As  twice  25?  S3  ....  50?  Log.co-secants:  10. 115746 
Is  to  half  the  ranges  .  .  1380  Log.  s  .  .  3.139879 
Sow  radius,  or     ....    90?     Lojj.sines     .10.000000 

To  the  impetus,  in  feet>    s    1804    Log.s.     .    .    3.255625 


Problem  XXVII. 
Given  the  Elevation  and  the  Bange;  to  find  the  Time  of  the  tVght. 

.Rule. 

As  radius,  is  to  the  logarithmic  tangent  of  the  elevation ;  so  is  the  loga« 

rithm  of  the  rahge,  in  feet,  to'  a  logarithmic  number  j  which,  bein^  divided 

by  2,  will  give  the  logarithm  of  4  times  the  number  of  seconds  taken  up  in 

the  flight. 

•       •     *  ' 

Es^mple  I. 

In  what  time  will  a  shell  range  1 1986  feet,  at  an  elevatbn  of  34?49'.  ? 

Asrwtiuss  .  .  .  .  90?  OC  L6g.  co^secants  10*  000000 
Is  to  the  elevation  =  .  34?49f  Log.  Ungent  =  9. 842266 
So  is  the  ranges    .    .     11986      Log.=:  .    .    .    4.078674 

Divide  by  2)  3. 920940 

■■■■I '■■  1 1       '■ 
Four  times  the  flight  s=     91.30      Log.  s  .    .    ;     1.960470 

Number  of  seconds  a  •    22. 825,athe  time  of  flight. 

Digitized  by  VjOOQ IC 


JLxample  2. 

In  what  time  will  a  afaell  range  3250  feet,  at  an  eievation  of  32  degrees  ? 

Asradiwas    .    .    •    ,    .    90?     Log.  co-secantslO.OOOOOO     • 
Is  to  the  elevation  =#   .    .    32?     Log.  tangent  =     9. 795789 
So  18  the  range  =     .    .    .3250    Log.  »  «    ,    .    3.511883 


Divide  by  2  J  3. 307672 


Four  times  the  flight  s  ,    45.06   Log.  »  .    «    •    1.653836 


Number  of  seconds  =    «     II.  26^=s  the  time  of  flight 

Note. — ^From  this  it  b  manifest  that  when  the  elevation  of  the  piece  u 
45?,  half  the  logarithm  of  the  range  will  be  the  logarithm  of  4  times  the 
number  of  seconds  taken  up  in  the  flight.  . 


Probusm  XXVIII. 

Given  the  Range  and  the  Elevation :  tojlnd  the  greateet  Altitude  qfthe 

,    SheU. 

RVLBs 

As  radius,  is  to  the  logarithmic  tangent  of  the  elevation ;  so  is  the  loga- 
rithm of  one-fourth  of  the  range,  to  the  logarithm  of  the  required  altitude. 


Example  1.  . 

If  a  shell  range  11986  feet,  when  projected  at  an  elevation  of  34?49^ ; 
required  the  greatest  altitude  which  it  acquires  during  its  flight  ? 

As  radius^  .«  .  .  .90?  0'  Log.  co-sec.=  I  0.000000 
Is* to  the  elevation  =  34 ?49  ^  Log.  tangent  =  9. 842266 
So  ia  i  ot  the  range  =  2996. 5     Log.  =     4.3, 476614 


To  the  altitude,  in  feet,=20S4    Log.  =: 


3.318680 


Example  2. 

If  a  shell  range  4760  feet,  when  projected  at  an  elevation  of  45?;, 
required  the. greatest  attitude  which  it  aqquires  during  its  flight?    . 


Digitized  by 


Google 


582  mucrtCAL  otrnvBRT. 

As  radius  =  .  .  90?  0'.  Log.  co-sccantnl  0.000000 
Is  to  the  elevatioVi  =  45?  0^  Log.  tangent  =  10. 000000 
So  is  i  of  the  ranges  1190      Log.=   .    .    .    3.075547 


Tothe  altitude,  in  fcet,=  1190  Log.  :^   ,    .    .•    3.075547 

.  Note^r^TOOi  this  it  is  manifest,  that  when  the  elevation  of  the  mortar 
is  45  degreesj  one-fourth  of  the  range  will  .be  equal  to  the  greatest  altitude 
at  which  the  ahell  can  amve. 


Probum  XXIX. 

Gwenfhe  Inclbuiiion  qf  the  Plofie,  the  Elevaiian  of  the  Piece,  and 
the  Impetus  f  to  find  the  Range* 

Rule. 

.  To  twice  the  logarithmic  secant  of  the  inclination'  of  the  plane,  add  the 
logarithmic  sine  of  the  elevation  of  the  piece  above  the  plane,  the  loga- 
rithmic co-sine  of  the  elevation  of  the  piece  above  the  horizon,  and  the 
logarithm  of  4  times  the  impetus :  the  sum  of  these  four  logarithms 
(rejecting  40  iii  the  index,)  will  be  the  logarithm  of  the  required  range. 

tiow  far  will  It  shell  range  on  a  jplane  which  ascends  10?  15',  and  also 
en  another  plane  which. descends  10?  15^  ;  the  impetus  being  SOOO  feet  in 
both  cases,  and  the  elevation  *of  the  mortar  3 1  ?45 '  ? 

.    fSb&ilion.    31  ?45 C  - 10?  15 '.  =21  ?30^  the  elevation  of  the  piece  above 

the  aflcending  plane  ; 
and,  31?45^  +10?  15^  =42?  0',  the  elevation  of  the  piece  above 

the  descending  planew 

To  find  the  Range  on  the  ascending  Plane : — 

Inclination  of  tba  plane  =     10^15':  Twice  the  log.  secant=SO.  013974 

Elevation  above  the  plane  f=  21. 30  Log.  sine  =      .    .  * .     9. 564075 

Elevation  above  the  hofizon=z3 1.45  Log.  co-sine  =     .     .    9.929599 

tour  times  the  impetus  =     8000  Log.  =   ,    ...    .     ,    3«.  903090 

Range,  in  feet,  =s     •    •    •    8575       Ug.  sb  ^    «    ,    ^    ,    3.410788 

Digitized  by  VjOOQ IC 


RANGB8  ^VD  iimrros  or  srblls.  583 

To  find  the  Range  on  the  descending  Plane  :~ 

Inclioailion  of  the  plane  :=     10?I5C  Twice  the  log.  flecant=20. 018974 

Bleyation above  the  planes   42.  0  Log.aines      .    •    •    9.82S511 

Elevalion  above  the  horizoa=S  1. 45  Log.  co-sine  =:     .    •    9. 929509 

Four  times  the  impetus  ^      8000  Log.  =:  .    •    .    •    •    3.90S090 


Range,  in  feet,  =:    •    .    %    4701        Log«  =   .    ^  .•    .,>    8.672174 


PROBLSM  XXX, 

Gwm  the  belinqfion  qf  the  Piane^  the  Elevation  qfthe  Pieci,.and  the 
.  Range  J  to  Jind  the  Impetus. 

RULB. 

To  twice  the  logarithmic  co-sine  of  the  inclination  of  the  plane,  add  the 
logarithmic  co^secant  of  the  elevation  of  the  piece  above  the  plane,  the 
logarithlnic  secant  of  the  elevation  of  the  piece  above  the  horizon,  and  the 
logarithn^  of  the  one-fourth  of  the  range :  the  sum  of  these  four  logarithms 
(abating  40  in  the  index,)  will  be  the  logarithm,  of  the  impetus*- 

EaAmple. 

With  what  impetus  mttsi  a  shell  be  dischatgsd  to  strike  an  object  at  the 
distance  .of  2575  feet,  on  an  inclined  plan^  which  ascends  10^15^,  and, 
also,  another  olgeet  at  the  distance  of  4701  feet,  on  an  inclined  plane 
which  descends  10?  15^ ;  the  elevation  of  the  piece  being.  31?45'  in  both 
cases  ? 


Solution.    31?45!-.10?15;=21?30i;  is  the  elevation  of  the  piece, 

abbver  the  ascending  plane; 
and,  Sl?45'  +  10?15^r:42?  0',  is  the  elevation  of  the  piece 

above  the' descending  ]dane» 

To  find  the  Impetus  on  the  ascending  Plane: — 

Inclination  of  the  plane  =     10?  15!  Twice  thelog.co-sine=  19. 986026 

Elevation  above  the  plane  =r  21 .  80^  Log;  cor-secant  :i     ,     10. 435925 

Elevation  above  the  horizohzrd  1 .  45  Log.  secant  =:     .    .     10. 070401 

One-fourth  of  the  range  =  043: 75  Log.  =  •,.,.      2. 808717 

Impetus, in  feet, c:*    i»  •  %    8000        Log,  c  *   /    ;    .    .      3.301069 

Digitized  by  LjOOQIC 


584  PRACTICAL  GUKMBAy. 

To  find  the  Impetus  on  the  descending  Plane  :— 

Inclination  of  the  plane  =     10?15<  Twice  the  log.  co-siner:  1 9.966026 

Elevation  above  the.plane  =  42.   0  Log.  co-secant  =i     ,     10. 174489 

Elevation  above  the  horizon=3 1 .  45  Log.  secant  =      •    .    10. 070401 

One-fourth  of  the  range  =  1 1 75 .  25  Log.  =       ....      3. 070 130 

Impetus,  in  feet^  =.    .    •      2000       Log.  =  .  .  ,    .    .    .      3.301046 


Problem  XXXI. 

Gix^m  the  Weight  of  a  Ballj  the  Charge  of  Powder  twlfc  which  it  itjtfed, 
and  thekfioum  Velocity  of  that  Ball;  to  find  the  FelocUy  ofaSheli, 
when  projected' with  a  gioen  Charge  qf  Powder. 

RlJLB. 

To  the  arithmetical  complement  of  half  the  logarithm  of  the  weight  of 
the  ^hell)  add  half  the  logarithm  of  twice  the  weight  of  the  charge,  in 
pounds,  and  the  constant  logarithm  3. 204120 :  the  sum  (abating  10  in  the 
ind^x,}  will  be  the  velocity  of  the  shell  answering  to  the  given  charge. 

JExample, 

If  a  ball  of  1  lb.  \^eight  acquire  a  velocity  of  1 600  feet  per  second,  when 
fired  with  8  ounces  of  powder,  it  is  required  to  find  with  what  velocities 
the  several  kinds  of  shells  will  be  projected  by  the  respective '*chaige8  of 
powder  expressed  against  them  in  the  following  table  ? 

For  the  Ifl-inch  Shell  :— 

Weight  of  the  shell  =  .     .    .196    Ar.  comp.  of  i  its  log.=8. 853872 
Twice  the  weight  of  the  charge  =:  18    Half  its  log.  =     ...    0. 627636^ 
Constant  log.  =      •    .'   .    .    .    .    .    .    .    .    .'    .     .    .    3.204120 

Velocity,  in  feet,  =; .  ..    .    .    485     Log.  =:      •    •    •    •    2.685628| 

For  the  10-inch  Shell :— 

Weight  of  the  shell  .  .  .  .90  Ar.  comp.  of  |  its  log.s:  9. 022879 
Twice  the  weight  of  the  charge  =:  8  Half  its  lo^.  =  •  .  .  0. 45 1545 
Constant  log.  =    •.....,. 3,204120 

Velocity,  in  feet,  =  •    •    •    •    477    Log,c?    ...•...•    .    2,678544 


Digitized  by 


Google 


IMPBTUS^  VELOGITY^  AND*  CHARGSB  OF  POWDER. 


585 


For  the  8-inch.  Shell  2— 

Weig;ht  of  the  shell  =  •  .  .  .  48  Ar.  confj).  of  4  its  log.sQ.  159380 
Twice  Ifce  weight  of  the  charge  =  4  Half  its  log.  =  .  .0. 301030 
Constant  1(^.  = 3.204120 


Velocity,  in  feetj  =  .    ...       462    Log.  s      .    .    .    .    2.664530 

Nofe.— The. same  results  will  be  obtained*  by  computing  agreeably  to 
th^  rule  in  Problem  XVIIL,  page  571. 

Table  D^Showhig  the  Felocities^ofthe  diffjerenl  sized  SheUs,  when 
projected  with  given  Charges  qf  Powder i 


SizeofSbeU, 
in  inchM. 

WeiphtofShclI, 
in  pounds. 

Charge  of 
Powder, 
in  lbs. 

Loga/itbpa. 

V€iIocity, . 
in  feet. 

1 

Logarithm. 

13 

10 

8 

196 

90 

48 

16 

8 

-    9 
4 

2 

1 

0.477124* 
0.301030 
0. 150515' 
0. 000000 
0.^9485 

485 
477 
462 
566 
566 

7.314258t 
7.321482 
7.335368   . 
7.247184 
7.247184 

Problem  XXXII. 

Given  tlie  JSI^a<ian  and  the  Range ;  to  find  the  Impetus^  Velodty,  and 

Charge  of  Pjowder. . 

Rule. 

Find  the  impetus^  by  Problem  XXVI.,  page  579 ;  to  the  logarithm  of 
which  add  the  constant  logarithm  1.  206376 ;[  :  take  half  the  sum,  and  it 
will  be  the  logarithm  of  a  natural  number ;  which,  being  doubled,  will  be 
the  required  velocity.  Now,  to  the  logarithm  of  the  velocity,  thus  found, 
add  the  logarithms  from  Table  D  answering  to  the  charge  and  the  velocity 
of  the  givcfn  shell:  the  sum  of  these  three  logarithqis  (abating  10  in  the 
index,)  being  doubled,  will  give  the  logarithm  of  the  required  charge  of 
powder,  in  pounds.  • 

*  The  numbers  in  tim  column  are  the  logarithms  of  .the  square  voots  of  the  respective 
charges.  .  • 

t  The  numbers  in  this  column  are  the  aiithmetical  complemeots  of  the  logarithms  of  the 
lespectiye  velocities.  . 

t  This  is  the  logarithm  of  l^  feet,  the  descent  of  a. falling  body  in  the  first  second  of 
time.  .        •  .       •        - 


Digitized  by 


Google 


696  MAcncAL  mjvtmBY^ 

Example* 

With  what  impetiiSy  felocity,  and  charge  of  pofwder^  miMt  a  IS^itich 
shell  be  fired  ataneleratioii  of  34?49^^to  strike  an  object  at  the  distance 
of  11986  feet? 

.     .  To  find  the  Impetiu  and  the  Velocity  t~      ' 

Twiee  the  elevatbn  =     .    .    ,    ,    69?38:  Log.  co-secants  10. 028036 
Half  the  range  =        ......      5993     Log.  =s  .    .    •3.777644 

Impetus,  in  feet;  =     .....      6392     Log.  =  .     .    .    3.805680 
Constant  i0g.  »  ................    1.206376 

V     ^  Divide  by  4)5.012056 

Nataral.namber  s     ...'•.    321        Log.=  .    •    .    2.506028 

Velocity,  in  feet,  =    :....    642 

To.  find  the  Charge  of  Powder :  —    * 

Velocity,  in  feet,  = 642        Log.  ==  .     .     .     2. 807535 

Log.  of  charge  for  a  13-ineh  shell,  from  Table  D,  =    .     .     .    0.477121 
Log.  of  velocity  for  a  IS-inch  shell,  from  Table  D,^^  ...    7. 314258 

Sum  IS  0.598914 


Charge,  in  pounds,  <s      ,    .    .     19.77        Log.  :s  .    ,    .     K 197828 

Hence  the  impetus  is  6392  feet,  the  velocity  642  feet,  and  the  charge  of 
powder,  15. 77  Ib'B.,'  or  15  lbs.  12§  oz.  nearly. 


PROBLKM  XXXIIL 

Gioen  the  /ncKnatiofi  of  the  Plane^  the  Eleoaiwn  of  the  Piece,  and  the 
Range;  tojbad  the  Charge  t^ Powder. 

RlTLB. 

Find  the  impetus,  by  Problem  XXX.,  page  583 ;  with  which  proceed  as 
directed  in  the  last  problem. 

Example  1. 

How  much  powder  will  throw  a  10-inch  shell  6760  feet,  on  an  inclined 
plane  which  ascends  7?30C ;  the  elevation  of  the  mortar  being  33?  14!  } 


Digitized  by 


Google 


above  the  ascending  plane. 

inclination  of  the  plane  =  .      7?30'  'IVice  the  log.  co-8irie=  19. 992538 

Elevation  above  the  pkne  =     25. 44  Log.  co-secant  =»      .     10. 362327 

Elevatioti  ab6ve  the  horizon  =  33. 14  Log.  secant  =     .     .     10. 077562 

One-fourth  of  the  range  =        1690  Log.  =       .    .    .     .      3.227887 


ImpetUA^  in  fe«t^ »    4    .    /  4574      Log.  =       .    .    .     .      3.660314 
Constant  log.  =      ..•..•.... L?06376 


Divide  hy  2)4.86(5690 


Naturd  number.  = 


271 


Log.  s= 


2.433345 


Velocity  =  ...     •     .     .     542  feet. 

To  find  the  Charge  of  Powder  :— 

Velocity  = .542  feet,  Log.  -     ....     2. 733999. 

Log.  of  charge  for  10-inch  shell,  from  Table  D,  =r  .     .     .     .     0. 301030 
Lo^.  of  velocity  for  10-inch  shelly  from  Table  D,  ^  .    .    .    .     7. 3214t>2 

,  •  Sum.=  0.356511 

Charge,  in  pounds,  =3    •    .    .    5.164    Log.  =     ....    0.713022 
Hence  the  charge  of  j^owder  is  5. 164  lbs.,  or  5  lbs.  2i  oz. 

Example  2..    * 

How  much  powder  will  throw  a  10-inch  shell  6760  feet,  cm  an  inclined 
plane  which  descends  7?30%  the  deration  of  the  morCar  being  33^  14'?    - 

SoluUon.    33^41  +  7^30C  qs  40?44^  is  the  elevation  of  the  mortar 
above  the  descending  plane. 

Inclination  of  the  plane   =     7^30^  Twice  the  log.  co-8ine=19. 992538 

Elevation  above  the  plane=2  40. 44  '  Log.  co-secant  =       .     10. 185393 

Eleyationabovethehorizon=33. 14  Log.  secant  =3       .    •     10.077562 

One-fourth  of  the  ranges:     1690  Log.  = ••      3.227887 


Impetus,  in  feet,  =      .    .    3044        Log.  = .3.483380 

Constant  log.  =      .  '.    .    .    .     .     .     .    .  ' 1.206376. 


Natural  number' = 
Velocity,  in  feet,  = 


221        Log.  = 


Divide  by  2)  4. 689756 
..    .    ,    .      2.344878 


442 


Digitized  by 


Google 


$88  j^jurricAL  GimMBRY. 

To  find  the  Charge  of  Powder  :— 

Velocity- 442     Log.  =  .    •    •     .    2.645422 

Log.  of  charge  for  lO-^inqh  shell,  from  Table  D,  =  .     .    ..    .    0. 301030 
Log.  of  velocity  for  10-inch  shelly  from  Table  D^  =»      .    .    .    7«  321482 

/Sum  =  0.267934 

Charge^  in  pounds,  =    .    •    .    3.434    Log.  =     •     .     .'  ;    0.53586fi 

Hence  the  ehal-ge  is  3. 434  lbs.,  or  3  lbs.  7  oz.  nearly*. 


Problbm  XXXIV, 

Gtoeh  the  Inclinatum  of  the  Plane,  the  Elevation  of  the  Piece,  and  the 
ImpetU9  ;  to  find  the  Time  of  Flight. 

Rule.         * 

To  the.  logarithmic  secant  of  the  inclination  of  the  plane  a^d  the  idge^ 
rithmic  sine  of  the  elevation  above  the  plane,  and  half  the  logarithm  of  the 
impetus :  the  sum  (abating  20  in  the  index,),  will  be  the  logarithm  of  twice 
the  time  of  flight,  in  seconds. 

Exfmple.  ♦ 

In  what  time  wilt  a  10-inch  shell  strike  an  object  on  an  inclined  plane 

.  which  ascends  7?30^,  when  discharged  with  an  impetus'of  4574  feet,  the 

elevation  of  the  mortar  being  33?14<  ;  and  in  what,  time  will  it  strike 

^  another  object  on  a  descending  plane,  with  the  same  impetus  and  elevatkm? 

Solution.    33?14:-7?30C  =25  M4^  is  the  elevation  of  the  mortarabove 

the  ascending  plane ; 
and,  33?14f  +7?301=:40?441  is  the  elevation  of  the  mortarabove 

the  descending  plane. 

To  find  the  Time  of  Flight  on  the  ascending  Plane  :«<- 

Inclination  of  the  plane  =  .  .  ,  8?30:  Log.  secants  10. 004797 
Elevation  above  the  plane  =  .  .  .  25.44  Log.  sine  =  9.637673 
Impetus  =      .     .    .     .     ...     .     4574        Half  its  log.  =  1.830148 


Twice  the  time  of  flight  =5     .     .    .     29.69       Log.  =   .    .     1.472618 


Time  pf  flight  =      •    •    •    •    .'    .     14. 845  seconds. 


Digitized  by 


Google 


HORIZONTAL  EAN6BS   OP.SHBLLS.  589 


To  find  the  Time  of  Flighl  on  the  descending  Plane : — 


Inclination  of  the  plane  =  . 
Elevation  above  the  plane  s 
Impetus  ss     •    •    •'    •    • 

Twice  the  time  of  flight  =' 


8?30^  Log.  secants  10. 004797 

40.44  Log.  sines     9:814607 

4574  Half  its  log.  ^  1 .  830148 

44.62  Log.=  ,  '.     1.649552 


Time  of  flights       ••••..     22.31  seconds. 

—  -    —  -    '         .  ---  -  ■  ■   — 

problbm  xxxy. 

CTtoai  the  Impetus  and  the  Elevation  ;  tojind  the  horizontal  Range. 

RULB. 

To  the  logarithm  of  the  impetus  add  the  logarithmic  si^e  of  twice  the 
angle  of  elevation  :  the  sum  (abating  20  in  the  index^)  will  be  the  logarithm 
of  a  natural  number ;  which^  being  doubled,  will  give  the  required  range 
on  the  horizqnial  plane. 

JExample  1. '     '         * 

Let  a  shell  be  discharged  with  an  impetus  of  1592  feet,  at  an  elevation 
of  34?49V;  required*its  range  on  the  horizontal  plane  ? 

Impetus  :=     •    .    .     1592      Log.  =  .    3.201943 
Twice  the  elevations  69^38^    Log,  sines  9. 97 1964 


Natural  number  =  .     1492.5  Log.  5=  •    3.173907 
Horizontal  range  =      2985  feet* 

Example  2.  .  : 

Let  a  shell  be  discharged  with  an  impetus  of  1804  feet,  at  an  elevation 
of  25?  ;  required  its  range  on  the  horizontal  plane }  ' 

Impetus  =     .     .    -.    1804       Log.  =  .    3.25^237 
Twice  the  elevation  =    50?        Log.  sine  s  9. 884254 

Natural  number  s  .     1382      Log.  =r  .    3. 140491 

■ 

Ifiorizontal  range  ^    2764  feet.. 


Digitized  by 


Google 


590  PRACTICAT.   GUNNBRT. 


Problem  XXXVL 


Oioen  the  Impetus  and  the  Eleeatim  ;  to  find  the  lime  ofFBght  an 
the  horizontal  Plane. 

Rule. 


With  the  impetus  and  the  elevation  compute  the  horizontid  range,  by 
the  last  problem ;  then>  with  the  horizontal  range,  thus  found,  and  th& 
elevatioijk  of  the  piece,  compute  the  time  of  flight,  by  Problem  XXVII^ 
page  580.  Or,  the  time  of  flight  maybe  computed  directly,  by. Problem 
XXXIV.,  page  58& 

Example. 

In  what  time  will  a  13-inch  shell  strike  an  object  on  a  horizontal  pUkie, 
when  discharged  with  an  impetus  of  6392-  feet,  the  elevation  of  the  mortar 
being  34  ?49:? 

Impetus  =     •    ;    •     .    6392        Log.  =•    .    8.805637 
IVice  the  elevation  =    69938^       Log.  sine  =  9. 971964 


.  Natural  number  =      •    5992.4    Log.  =     •3.777601 

Horizontal  range  s     .  il984. 8  feet.    Log.  a  4,  078631 
Elevation  =s     .    ,   .34749^       Log.  tangent  a  9. 842266 


Divide  by  2)  3. 920897 


Fourtimesthe.flight=:  91.30      Log.s     .    •     1.960448| 
Flights    ....    22. 825 seconds. 

To  find  the  Time  of  Flight,  by  Problem  XXXIV.,  page  588  :— 

Inclination  of  the  plane  =  ....  0?  0'  Log.  secant=  0.000000 
Elevation  above  the  plani  of  the  ftom.=34. 49.  Log.  sine  =  9. 756600 
Imf)etus=  ,    ,    .    •-    •    -    •    •.6392       Jiatfiulog.s  1.90281 84 

Twice  the  time  of  flight  =s      ^    .    ,45.64      Log.  a     •   •1.659418| 

Time  .of  flight  = 22. 82  seconds  j  which  agrees  with 

the  time  of  flight  found  by  the  last  rule,  as.above* 


Digitized  by 


Google 


BUMU4  AND  FUZBa«  591 

.     Pbobum  XXXVII.     . 
.  Given  the  Ttme  of  Flight  of  a  Shell ;  to  find  the  Length  of  the  Fuze. 

RULB. 

To  the  logarithm  of  the  time  of  $ght  add  the  constant  logarithm 
9;  342423,  for  13  and  10-inch  shells,-!-^  9*380211/ for  8,  5^,  and  41- 
inch  shells  :*  and  the  sum  (abating.  10  ia  the  index^)  will  be  the  logarithm 
of  the^length  of  the  fuze^  in  inches; 

.        "• 

Example  L 

Let  the  time  of  flight  of  a  13-iiich  shell  be  31. 75  seconds ;  required  the 
length  of  the  fiize  ? 

• 
Time  of  flight,  in  seconds,  =3     .     31. 75        Log.  =  1. 501744 
Constant  log«  e      •    •    .    •    .    ...  •    ....    ^    9.342423 


Length  of  the  fuze,  fai  inches,  =    6. 985      Log.  =  0. 84416?       * 

Example  2. 

Let  the  time,  of  flight  of  an  8-inch  shell  b^  21.. 5  seconds ;  required  the 
length  of  the  fiize  ? 

Time  of  flight,  in  seconds,  8    •    21.  5        Log.  s  1. 3S2438 ' 
.Constant  log.  =     ...•.*...,.'      9.380211 

Length  of  the  fiize,  in  inches^  ss    5. 16        Log.'s  0. 712649 

Jfotej^The  fuzes  for  a  13  and  a  10-inch  shell  are  so  constructed  as  to 
bum  •  22  of  an  iiich  in  one  second  $  and  those  £br  the  smaller  kind,  viz.,  8^ 
5|,  and  4|-inch  shells,  •  24  of  an  inch  in  the  same  Apace  of  time.  Now, 
the  logarithms  of  these  two  ikcimal  iiumbers,  viz.,  9.342423  and 
9.38021 1,  are  therefore  the  constant  logarithms  made  use  of  in  the  above 
rule.  •  .  *  *• 

Fuzes  are  generally  marked  off,  by  circular  lines,  into  seconds  and  frac- 
tional parts  of  a  second,  so  that  no  time  n\ay  be  lost  in  measuring  and 
adapting  them  to  tlie  shells  for  which  they  are  intended. 


Digitized  by 


Google 


592  MBN8URAT10N  OF   PLANBS. 

SOLUTION  OF  PROBLEMS  IN  THE  MENSURATION  OP 
PLANES,  &c. 

Problem  I.        * 

Given  the  Base  and  perpendicular  Height  of  a  Plane  Triangle;  to  find 

its  Area. 

RUJLE. 

To  the  logarithm  of  the  base  add  the  r^gariUim  of  half  the  perpendicidar 
height,  and  the«iim  will  be  the  logarithm  of  the  area,  or  superficial  content 
of  the  triangle. 

Example. 

Let  the  base  of  a  plane  triangle  be  37. 6  yards,,  fmd  its  perpendicular 
height  29. 8  yards ;  required  its  area,  or  superficial  content  ? 

Base  of  the  triangle  :s  .-  .•  .     .     .    37.6yard«        Log,  =  1.575.188 
Eerpei^.  height  ==  29. 8  yards  -h  2  =   14.9  yards      •  Log.  ^  1. 173186 

Area,  or  superficial  content,  s^  •     .  560. 24  • .  . .      Log.  ==  2. 748374 


Problbm  IL 

Giten  two  ^dee  and  the  contained  Anffle  of  a  plane  TViangle;  to  find 

.  if<  Jrea*    ... 

RULB. 

To  the  logarithmic  sine  of  the  contained  angle,  add  the  logarithms  of  the 
containing  sides :  and  the  sum  (abating  10  in  the  index,}  will  be  the  loga- 
rithm of  twice  the  area  of  t1ie  triangle.* 


Let  the  two  given  sides  of  a  triangle  be  109. 5  yards  and-.  168. 2  yards 
respectively,  and  the  contained  angle  79?  16^ ;  required  the  area,  or  super- 
ficial content  of  that  triangle? 

*  Or,  to'  the  logarithmic  sine  of  the  contained  angle,  add  the  logarithm  of  one  of  the 
conUining  sides  and  the  logarithm  of  haif  ilu  other  emitamitig^  tidt :  the  turn  of  these  tfuee 
logarithms  (abating  10  in  tiie  index,)  will  be'the  logarithm  of  the  area  of  the  triangle. 


Digitized  by 


Google 


MBNSURATION  OF  PLANB8.  593 

Contained  or  included  angle  s  .  •  .  79?  16^  Log.  sine  =  9. 992335 
One  of  the  containing  sides  3  .  .  .  109.  5  Log.  =  .  2.039414 
The  other  containing  side  =       ...  168.   2      Log.  s  .    2. 225826 

Twice  the  area  of  the  triangle  ss     .    18095.   7      Log.  s  .    4.25757S 


Area  of  the  triangle  s      .    •    •    •      9047. 85  yards^  as  required. 

Note. — ^The  above  problem  will  be  found  exceedingly  useful  in  the 
practice  of  land-surveying. 


Problem  IIL 

Oivm  the  three  Sides  of  a  THar^le)  to  find  its  Area,  or  superficial 

.  .   Content. 

RULB. 

Add'  the  three  sides  together,  and  take  half  their  sum;  subtract  each 
side  severally  from  that  half  sum,  noting  the  remainders :  then. 

To  the  logarithm  of  the  half  sum  add  the  logarithms  of  the  three 
remainders;  now,  the  sum  of  these  four  logagthms,  being' divided  by  2^ 
will  give  the  area  of  the  triangle.   * 


Example. 

Let  the  three  sides  of  a  triangle  be  433, 312,  and  205  yards  respectively; 
required  its  area  ? 

First  »de  :s  433  First  remainder  =  42  Log.  s  1.623249 
Second  side  =  ^12  Second  remainder  zct  163  Log.  r=  2.212188 
Third  side  =     205      Third  remainder*  =     270      Log.  =  2.431364 

Sum  s     .    .    950 

Half  sum  =       A15      \og.  ^ .    2.676694 


Divide  by  2)  8. 943495 


Area  of  the  triangles  29631.08      Log.  =     .....    4.471747i 

2a 


Digitized  by 


Google 


504  MBNsumAnoN  oi  planes. 

Problem  IV.  ^ 

Given  the  Diameter  of  a  Grcle }  to  find  its  Ckcumjerence,  trnd 
•  conwrsely.^ 

Rule. 

To  the  logarithm  of  the  diameter  add  the  constant  logarithm  0. 497150, 
and  the  mm  will  be  the  logarithm  of  the  circumference.  And^  to  the 
logarithm  of  the  circumference  add  the  constant  logarithm  9. 502850^  and 
the  sum  will  be  the  logarithm  of  the  diameter. 

Esfamipl^  1.      .       . 

The  earth's  diameter  is  7917*  5  miles ;  required  its  circumference? 

Diameter  of  the  earth  =  ....    7917.5  miles    Log.  =.  3.898588 
Constant  log,  =      •    •    •    ••»...«.•••.•      0.497150 

Circumference  of  the  earth  =    .    .      24873.miles    Log.  =  ' 4. 395738 

Example  2. 

If  the  circumference  of  the  earth  be  2500O.mned,  what  is  its  difuheter  ? 

Circumference  of  the  earth  s     .    .    .,    25000  miles    Log.  =?  4. 397940 
Constant  log.  :e      .    .    ..«    •    ^    ..«•....    .      9.502850 


Diameter  of  the  earth,  in  miles,  =      .  7957. 7  Log.  =  3. 900790 

Note. — The  diameter  in  the  first  example,  viz.,  7917. 5  miles^  appears  to 
be  the  true  diameter  of  the  earth,  on  the  spherical  hypothesis.  The  con- 
stant logarithms  used  in  thia  problem  will  be  found  in  tl|e  TMe  of  Miscel- 
laneous Numbers,  at  the  end  of  the  second  volume. 


Probi^m  V. 

Given  the  Dicmetery  or  the  Circumference  of  the  Earth;  to  find  the  whole 

Area  of  its  Surface. 

Rule.  '   \  ^' 

To  twice  the  logarithm  of  the  earth's  diameter  add  the  constant  loga« 
rithm  0.497150,  and  the  sum  vrill  be  the  logarithm  of  the  area  of  the 
earth's  surface,  in  square  miles.     Or, 


Digitized  by 


Google 


MBNSUftATtON  OF  PtAKXS.  595 

To  twice  the  logarithm  of  the  earth's  circumference  add  the  constant 
logarithm  9. 502850,  and  the  sum  will  he  the  logarithm  of  the  earth's 
surface,  in  square  miles. 

Example  I. 

Required  the  area  or  superficial  measure,  in  square  miles,  of  the  whole 
of  the  earth's  surface,  allowing  its  diameter  to  be  7917<  5  English  miles  ? 

Diameter  of  the  earth  =  7917. 5     Twice  its  log.  =?  7. 797176 
Constant  log.  ^      . «    «    g    0. 497150 

Area,  in  square  miles,  =:  196936545. 5     Log.  =  «    .8. 294326 

Example  2. 

Required  the  area  or  superficial  measure  of  the  whole  of  the  earth's 
surface,  in  square  miles,  allowing  its  circumference  to  be  24873  English 
miles? 

Circumference  of  theearthat  24873    Twice  itslpg.aS.  791476 
Constant  log. »      ..    ^    ...*«..    .    9.502850 

Area,  in  square  miles,  s  196936545. 5    Logi  s   .    8. 294326 


■lUM   mm 


Problem  VI. 
To  Jind  ib$  Length  of  any  Arc  qfaCbreU, 

Rule. 

To  the  logarithm  of  the  degrees  in  the  given  arc,  considered  as  a  natural 
number,  add  the .  logarithm  of  the  radius  of  that  arc,  and  the  constant 
logarithm  8. 241878;  the  sum  will  be  the  logarithm  of  the  length  of  the 
arc. 

ExomvplB, 

Required  the  length  of  an  arc  of  45  degrees,  the  radius  being  9  inches  ? 

Length  of  the  arc,  in  degrees,  s    45  Log.  s  1. 653213- 

Radius  of  the  arc,  in  inches,    =      5  Log.  =  0. 954243 

Constant  log,  = .  .*    •    •    8.241878 

Length  of  the  arc,  in  inches,  ^  ^      7. 0686  Log.  a  0. 849334 

2q2 

Digitized  by  VjOOQ IC 


596  PRACTICAL   OAtJGINO. 


SOLUTION  OF  PROBliEMS  IN  GAUGING. 


Gauging  is  the  art  of  finding  the  number  of  gallons^  &c^  contained  in 
any  vesseL 

By  a  recent  Act  of  Parliament,  therie  i3  to  be  but  one  general  standard 
gallon  throughout  His  Majesty's  dominions  of  Great  Britain  and  Ireland; 
which  gallon  is  to  contain  10  lbs.  (avoirdupois  weight)  of  distilled  water, 
each  pound  of  which  is  to  weigh  7000  granis  (troy  weight) :  hence  the 
new  standard  gallon  is  to  contain  70000  grains  (troy  weight)  of  distilled 
water.  Now,  since  a  cubic  inch  of  distilled  water  weighs  252. 458  grains 
(troy  weight),  the  contents  of  the  new  standard  gallon  may  be  readily 
reduced  to  cubic  measure,  by  the  following  proportion;  viz..  As  252.458 
grains  :  1  inch  H  70000  grains  :  277. 27384357  inches ;  which,  there- 
fore, is  the  number  of  cubic  inches  in  the  new  stiuidard  gallcm.  And 
because  the  mieasure  of  the  present  ot  old  standard  wine  gallon  is  231 
cubic  inches,  and  that  of  the  old  standard  ale  gallon  282  sudi  inches^  we 
have  sufficient  data  for  obtaining  proper  multipliers  for  the  reduction  of 
the  old  standard  wine  and  ^e  measure  into  the  new  general  standard 
measure,  and  conversely.    Hence, 

277.27384357-1-231  =  1.200319671  is  tiie  general  multiplier  for  reducing 
Log.ssO.  079297  r     the  new  standard  measure  into  the 

J      old  Handard  mne  tl^easure  ;  and, 
231h-277.  27384357=^0. 8331 1 140  "j  is  tiie  general  multiplier  for  reducing 
Log.=9. 920703  ^     the  old  standard  wine  measure  into 

J      the  new  standard  measure. 
277. 27384357-^282=0. 98324058  ^  is  the  general  multiplier  for  reducing 
Log.=s  9.992660  y     the  new  standard  measure  into  the 

J  •     old  standard  ale  measure;  and> 
282H-277. 27384357=1. 01704508^  is  Oie  general  multiplier  for  reducing 
Log.ssO.  007340  I     the  old  standard  ale  measure  into 

j     the  new  standard  measure. 


Now,  the  respective  multipliers  and  their  corresponding  logarithms  being 
thus  obtained,  the  reduction  of  the  old  standard  wine  and  ale  measure  into 
the  new  general  standard*  measure,  and  conversely,  may  be  very  readily 
performed^  by  means  of  the  foUpiving  problems. 


Digitized  by 


Google 


PRACTICAL  6AU6IK6.  597 

Problem  L 
To  reduce  the  old  standard  fFine  Measure  into  thenew  Imperial  Measure. 

Rule. 

To  th^  logarithm  of  the  old  standard  wine  gallons  add  the  constant 
logarithni  9. 920703^  and  the  sum  will  be  the  logarithm  qf  the  new  standard 
gallons. 

Example  1. 

Reduce  400  gallons  of  the  old  standard  wine  measure  into  the  new 
general  standard  measure. 

Given  number  of  gallons  =  400        Log.  =  2.602060 
Constant  loe.  =  .     .    .     .     , 9.920703 


New  standard  gallons  s  333.245        Log.  =  2.522763 
ExampUQ^ 

Reduce  9864  gallons  of  the  old  standard  wine  measure  into  the  new 
general  standard  measure- 
Given  number  of  gallons  =  9864        Log.  =  3.994053 
Constant  log.  =5  .« ,    ,    9.920703 

New  standard  gallons   =    8217.8        Log.  s:  3.914756 

Probi^m  II.- 
To  reduce  the  new  Imperial  Measure  into  the  old  stamdard  wine  Measure^ 

Rule. 

To  the  logarithm  of  the  new  standard  gallons  add  the  constant  logarithm 
0. 079297^  and  the  sum  will  be  the  logarithm  of  the*  old  standard  wine 
gallons. 

Example  1, 

Reduce  400  gallons  of  the  new  general  standard  measure  into  the  old 
standard  wine  measure. 

Given  number  of  new  standard  galloris    =    400        Log.  s  2. 602060 
Constant  log.  =   • 0^079297 


p}d  standard  wine  gallons  =  ,    ,    ,       480.128        Log.  =  2*681357 


Digitized  by 


Google 


Example  2. 

Reduce  9S64  gallons  of  the  new  general  standard  measure  into  the  old 
fttf^ndar4  wipe  m^fwire* 

Given  number  of  new  standard  gallons  =  9864        Log.  =  3. 994053 
Constant  log.  = 0.079297 

Old  standard  wine  gallons  «...     11889.95        Log.  s  4.073850 
Note. — ^From  the  above  problems  it  appears  that  the  new   general 
standard  gallon  is^  very  nearly^  ovae-Jifih  greater  than  the  present  or  old 
standard  wine  gallon. 

-        I-     ■       —  •      ' 

Problbm  liL 
To  reduc0  tk$  M  itandard  Ale  Measure  into  the  new  Iwferial  iileosure. 

RULB, 

To  the  logarithm  of  the  old  standard  ale  gallons  add  the  constant  loga- 
rithm 0.007340^  and  the  buiq  will  be  the  logarithm  of  the  new  general 
standard  gallons. 

Exaimiple  1. 

Reduce.  400  gallons  of  the  old  standard  ale  measure  into  the  new 
general  standard  measure. 

QtvQQ  mmber  of  ale  gallons  =  400        Ifig.  9  2, 602060 
Constant  log.  :;s    . 0.007340 

New  standard  gallons  =      406.82        Log.  =  2.609400 

.  JBsampk  3^ 
Reduce  9864  gallons  of  the  old  standard  ale  measure  into  the  new 
general  standard  measure. 

Given  number  of  ale  gallons  s;  9864        Log.  =  3.994053 
Constant  log.  =;    ...*.,.....    Q.  007340 

New  standard  gallons  s  10032. 13        Log.  s.  4. 001398 

PaolitBM  IV. 
To  reduce  the  new  Imperial  Measure  into  the  old  strndair^  4k  Measw^ 

Rum. 

To  the  logarithm  of  the  new  standard  gallons  add  the  eonstaiit  logarithm 
9.992660,  and  the  sum  will  be  the  logarithm  of  the  old  standard  ale 
gallons. 


Digitized  by 


Google 


Reduce  400  gallons  of  the  new  general  standard  measure  into  the  old 
standard  ale  measure. 

Given  number  of  new  standard  gallons  ^  400        Log.  =  2.602060 
Constant  log.  aa      «    <    «    •    « 9.902660 

Old  standard  ale  gallons  =     .    .    •       393.296        Log.  =  2.594720 

Example  2. 

Reduce  9864  gallons  of  the  new'  general  st^dard  measure  into  the  old 
standard  ale  measure. 

Given  number  of  new  standard  gallons  =  9864        Log.  =  3. 994053 
Constant  log.  := 9.992660 

Old  standard  ale  gallons  ac    .    •    .    .   9698.7        hog.  sa  3*986713 

No<e.-»From  the  two  laat  problems  it  appears  that  the  new  general 
standard  gallon  is^  very  nearl]|r/  one-sisiieih  less  than  the  present  or  old 
standard  ale  gallon. 

Problem  V.        • 

Cfiten  the  DiinensionB  of  a  drcutar-headed  Cash;  tojtn^  its  Content  in 
Ale  and  in  fTine  Gallons^  and  alto  agreeabhf  tf)  the  new  general 
standard  or  Imperial  GaUon. 

RULK. 

Divide  tlie  bead  diameter  by  the  bung  difimeter,  to  two  places  of  deci- 
mals in  the  quotient;  then, 

Add  together  the  logarithm  for  ale  or  wine  gallons  corresponding  to 
this  quotient  in  the  first  part  of  Table  LVIL,  the  logarithm  of  the  bun^ 
diameter  in  the  second  part  of  that  table,  and  the  common  Icq^arithm  of 
the  length  of  the  cask;  iVe  .sum  (abating  10  in  the  index,)  mil  be  the 
logarithm  of  the  content  of  the  cask  in  ale  or  wine  gallons.  Now,  to  the 
logarithm, .  thus  found,  add  the  constant  logarithm  0. 007340  for  ale 
gallons,  or  9. 920703- for  wine  giallons ;  and  the  sum  will  be  the  logarithm 
of  the  true  content  of  the  cask  in  gallons^  agreeably  to  the  new  general 
standard  or  imperial  measure. 

Example  I, 

Let  the  bung  diameter  of  a  cask  be  25  inches,  its  head  diameter  19. 5 
inches,  and  length  31  inches;  required  its  content  in  ale  and  wine  gallons^ 
and  also  in  gallons  agreeably  to  the  new  general  standard  measure  ) 


Digitized  by 


Google 


600  PAACTICAL  GAUGING. 

19. 50  -I-  25  =s  0. 78j  quotient  of  the  head  diameter  divided  by  the  bong 
diameter. 

First,— For  Ale  Gallons  :— 

Quotient  = 0. 78    Log.  for  ale  gallons  =  7. 362671 

Bung  diameter  =      ...     25  inches  Corresponding  log.  =  .  2.  795880 

Length  of  the  cask  =     •    •    31  inches  Common  log.  =  .     .1. 491362 

- 

Content  in  ale  gallons  =     .     ;  44.66     Log.  =     ....    .1.649913 
Constant  log.  = 0.007340 


Content  in  imperial  gallons  =:  45'.  42    Log.  <=      •     .     .    .     L  657253 
Second, — ^For  Wine  Gallons  :— 

Quotient  = 0.  78     Log.  for  wine  gallons  =.  7.  449340 

Bung  diameter  =      •    .      25  inches  Corresponding  log.  a?       2.  795880 
Length  of  the  cask  ==    • .     31  inches  Common  log.  =      .    .     1. 491362 


Content  in  wine  gallons  =3     54.52    Log.  =      .    •    .    .    •     1.736582 
Constant  log.  =  .....*...' 9.920703 


Content  in  imperial  gallons=45.  42    Log.  =      .....     1. 657285 

See  the  example  for  illustrating  the  use  of  Table  LVIL,  page  153,  and 
also  page  154. 

NotoiP— In  gauging  a  cask,  it  is  to  be  remembered  that  the  dimendons 
of  the  bung  diameter,  the  head  diameter,  and  the  length  of  the  cask,  be-all 
takjsn  within  the  cask.  In  measuring  these  dimensions,  it  must  be  care- 
fully observed  that  the  bung-hole  be  in  the  middle  of  the  cask,  and  that 
the  bung-stave  and  the  stave  directly  opposite  thereto  be'  both  regular  and 
even  within  the  cask  j  also,  that  the  heads  of  the  cask  be  equal  and  truly 
circular :  if  so,  the  distance  between  the  inside  of  (he  chimb,  to  the  outside 
of  its  opposite  stave  %vill  be  the  head  diameter  intKn  the  cask,  very  nearly. 

.  Example  2. 

Let  the  bung  diameter  of  a  cask  be  31. 25  inches,  its  head  diameter 
23.  75  inches,  and  length  39  inches ;  required  its  content  in  ale  and  wine 
gallons,  and  also  in  gallons  agreeably  to  the  new  general  standard  or 
imperial  measure? 

23. 75  -«-  31.  25  rs  0. 76>  quotient  of  the  head  diameter  divided  by  the 
bung  diameter. 


Digitized  by 


Google 


#racti6al  gauging.  601 

•First,— For  Ale  Gallons  :— 

Quotient  =..•..•  0.  76  Log.  for  ale  gallons  =  7. 355087 
Bung  diameter  =  .  .  31.  25  inches  Corresponding  log.  =  2. 989699 
Length^of  the  cask  =s:    .  •         39  inches  Common  log.  =     .     1. 591065 

Content  in  ale  gallons  =  .  86.268  Log.  =  .  4  .  .  1.935851 
Constant  log.  = .    0.0073*40 

Content  in  imperial  gallons  s  87.  .74      Log.  =     .    •    .    .     1.943191 

Secqnd. — ^For  Wine  Gallons  :-^ 

ftuotient  =  .  .  .  .  '.  .  0, 76  *  Log.  for  wine  gallons  =  7. 441742 
Bung  diameter  =  .  .  31.  25  inches  Corresponding  Tog.  =  2.989699 
Length  of  the  cask  =      '  39  inches  Common  log.  =     .     •     1.591065 

Content  in  wine  gallons  =  105. 32  Log.  =s  .....  2.  022506 
Constant  log.  =  . 9. 920703 

Content  in  imperial  gallons^  87.  74     Log.  =     .....     1.943209 

Remark,— The  above  problem  will  be  found  exceedingly  useful  to  Pursers 
in  the  Royal  Navy,  to  Commissaries  in  the  Army,  aiid  to  other  officers  in 
charge  of  gpvernment  stores,  who  may  have  occasion,  to  purchase  beer, 
wine,  or  spirits,  on  His  Majesty's  account^  in  foreign  countries ;  because  it 
enables  them  to  ascertain,  in  a  few  minutes,  the  absolute  number  of  gallons 
contained  in  any  given  quantity  of  liquor,  of  the  old  measure,  agreeably  to 
the  newly-established  standatd^  or  imperial  measure. 


Problem  VI. 

Given  the  Content  of  a  XJask  lying  in  a  horizontal  Posiiiony  its  Bung^ 
Diameter,  and  the  Depth  of  the  Ullage  or  wet  Inches;  to  find  the 
Quantity  oflAqaor  in  the.  Cask. 

Rule. 

Conceive  the.  bung  diameter  to  be  represented  by  urfity  or  1  inch,  and 
that  it  be  dividedinto  10000  equal  parts ;  then  the  half  of  this,  viz.^  .  5000, 
is  to  be  considered  as  a  constant  decitnal. 

Divide  the  wet  inches,  or  depth  of  the.  ullage,  by  the  bung  diameter,  to 
four  places  of  decimals  in  the  'quotient ;  find  the  difference  between  this 
quotient  and  the  constant  decimal.  Now,*  one-fourth  of  this  difference 
being  subtracted  from  the  quotient,  if  the  latter  be  less  than  th6  constant 
decimal,  or  adde<i^  thereto  if  it'be  more  than  that  decimal,  the  difference  or 
sum  will  be  the  multiplier. 


Digitized  by 


Google 


602  PRACTICAL  OAUGlNOi 

Then,  to  the  logarithm  of  the  mukipliefi  thus  founds  add  the  logarithm 
of  the  content  of  the  cask,  in  wine  measure;  and  the  sum  Mall  be  the  loga- 
rithm of  the  ullage,  or  number  of  gallons  of  liquor  in  the  cask,  in  wine 
measure.  And  if  to  this  logarithm  the  constant  logarithm  9.  920703  be 
added,  the  sum  will  be  the  logarithm  of  the  ullage^  agreeably* to  the 
imperial  measure. 

Note. — If  the  content  of  the  cask  be  given  in  ale  measure,  the  constant 
logarithm  will  be  0. 007340. 

Example  1. 

Let  the  buAg  diameter  of  a  cask  be  31. 25  inches,  its  content  in  wine 
measure  105.32  gallons,  and  the  depth. of  the  ullage,  11.5.  inches; 
required  the  quantity  of  liquor  in  the  cask  ? 

Depth  of  the  ullage,  or  wet  inches,  1 1  •  5  -f-  31 .  25 

inches  (B.  D.)  =  .  3680  quotient, .  3680,  which  is  less  than  the  constant 
Constant  decimal  =  .  5000  decimal. 


Difference  =  .     .     .  1320  -h  4   =5  .  .330,  subtractivc. 


Multipliers  .......      .3350  Log.   »:     9.52504S 

Content  of  cask,  in  wine  measure,s£  105. 32  gallons    Log.  »    2«  02250*6 

Content  of  ullage,  in  wine  gallons,  s=  35 .  28  .  Log.   ^     1  •  54755 1 

Omstant  log*  «     9.020703 

Content  of  ullage,  in  imperial  galls.:=  29. 39  Log.  =     1 .  468254 

Note. — If  the  content  of  the  cask  be  given  in  imperial  measure,  let  the 
logarithm  thereof  be  added  to  the  logarithm  of  the  multiplier ;  and  die 
sum  will  be  the  logarithm  of  the  ullage.. 

Thus,  in  the  above  example,  let  the  content  of  the  cask  be  given  agree- 
ably to  the  new  general  standard  or  imperial  measure  J  vis.,  87. 74  gallons; 
then. 

Multiplier,  aa  abcAre,  3s: 3350  Lo|;.  a     9.625045 

Content  of  the  cask,  in  ioipl.  Bieas.8s87 •  74  galldnt    Lqg.  »     1  •  943209 

Content  of  ullage,  in  imperial  galls.  =  29. 39  Log.  =     1.468254 

Example  2.     . 

Let  the  bung  diameter  of  a  cask  be  25  Inches,  its  content  hi  vrine 
measure  54. 52  gallons,  and  the  depth  of  the  ullage  15.  f$  inches ;  required 

the  quantity  of  liquor  in  the  cask  ? 


Digitized  by 


Google 


PRACTICAL  GAUGING. 


60» 


Depth  of  the  ullage,  or  wet  inches,  15.  75  -»-  25 

inches  (B.  D.)  ^  .  6300  quotient,  •  6300,  which  is  more  than  the  constant 
Contaot  decimal  =  .  5000  decimal. 


Difference  == 


.  1300  -4-  4  =    . .  325,  additive.  . 


Multiplier  =       .......     .6625                Log.     =  9.821186 

Content  of  the  cask,  lawine  meas.=54. 52  gallons     Log.     =  1. 736582 

Content  of  ullage,  in  wine  gallons,»36. 12                Log.     ==  1 .  557768 

•  Constant  log.  =  9. 920703 


Content  of  ullage,  in  imperial  gall^.  ss  30. 09  Log.     =      1 .  47847 1 

But  if  the.  content  of  the  cask  be  given  agreeably  to  the  imperial  standard 
measure,  yiz,^  45.42  gallons,  then  the  latter  part  o/the  operation  will-be 
as  thus  :-r- 

Muitiplier,  as  above,  =     ,    .     .     .     .  6625  Log.  =  9. 821 186 

Content  of  the  cask,  in  impl.  meas.  =  4'5. 42  gallons     Log.  =   1 .  657285 


Content  of  ullage^  in  imperial  galls.  =  30. 09 


Log.  =  1.478471 


Remark. — If  the  dry  inches  of  the  bung  diameter  be  made  use  ot  instead 
of  the  wet,  the  result  of  the  operation  will  express  the  vacuity  in  the  cask ; 
and  if  thb  vacuity  be  ^ded  to  the  ullage,  the  sum  will  be  the  contest  of 
the  cask,  which  will  be  a  proof  that  the  work  is  right. 

Thus^  in  the-  last  example,  where  the  bung  diameter  is  25  inches,  and 
the.  depth  of  the  ullage  15.75  bches,  the  difference  of  these  is  9.25, 
which,,  therefore,  is  the  number  of  dry  inches. 


Then,  dry  inches  9. 25  -h  25 

inohes  (B.  D.).  sx  ,  3700  quotient,^  •  3700,  which  is  less  than  the  constant 
Constant  decimals  .5000  decimal.. 


Difference  =  .    .     .  1300  •»-  4  a  .  .325,  subtracUve. . 

MultipKe?=  •....•    .     ,3375  Log.  =;     9.528274 

Content  of  the  cask,  in  impl.  meas.^  45. 42  gallons    Log.  s      1 .  657285 


Vacuity  ia  the  cask  r: 
Content  of  the  ullage  9 

Content  of  the  caskss 
1* 


15.33 
30.09 


Log.  s      1. 185559 


45. 42 ;  wbich  proves  the  work  is  right. 


Digitized  by 


Google 


604  PRACTICAL   GAUGING. 


Problbm  VII.  • 

Owen  the  Content  qfa  Cask  standing  in  a  vertical  or  upright  Position,  its 
Lengthy  and  the  Depth  ofth^  Ullage  or  wet  Indies  s  to  find  the  Quan- 
tify of  Liquor  in  tlie  Cask. 

RULB, 

Conceive  the  lengthof  the  cask  to  be  represented  by  unity  or  1  inch^  and 
that  it  be  divided  into  lOOOO  equal  parts  -,  then  the  half  of  this^  viz«^  •  5000^ 
is  to  be  considered  as  a  constant  decimal. 

Divide  the  wet  inches,  or  depth  of  the  ullage,  by  the  length  of  the  cask, 
to  four  places  of  decimals  in  the  quotient;  find  the  difference  betwreen  this 
quotient  and  the  constant  decimal :  now,  one-tenth  of  this  difference  being 
subtracted  from  the  quotient,  if  the  latter  be  less  than  the  constant  deeimal, 
or  added  thereto  if  it  be  more  than  that  decimal,  the  difference  or  sum  will 
be  the  multiplier. 

Then,  to  the  logarithm  of  the  multiplier^  thus  founds  add  the  logarithm 
of  the  content  of  the  cask,  in.  wine  measure;  and  the  sum  will  be  the 
logarithm  of  the  ullage,  or  number  of  gallons  of  liquor  in  the  cask,  in  wine 
measure.  And  if  to  this  logarithm  the  constant  logarithm  9. 920703  be 
added,  the  sum  will  be  the  logarithm  of  the  ullage  agreeably  to  the  imperial 
standard  measure. 

Note. — If  die  content  of  the  cask  be  given  in  ale  measure,  the  constant 

logarithm  will  h^  0. 007340. 

».  •    * 

Example  L 

Let  the  length  of  a  cask,  between  the  heads,  be  39  inches,  its  content  in 
^vine  measure  105.32  gallons,  and  the  depth  of  the  ullage  16.5  inches; 
required  the  quantity  of  liquor  in  the  cask  ?  ' 

Depth  of  ullage,  or  wet  inches,  16. 5  -*-  39 

inches  (length)  =  .4231  quotient^  .4231,  which  is  less  than  the  conatant 
Constant  decimal  3=.. £000  decimal* 

Difference  =  -    •     . .  769  -«-  10  =  . .  77,  subtractive. 

Multiplier  = 4154  Log.  =     9.618467 

Content  of  the  cask,  in  wine  meas.=  105. 32  gallons    Log.  s     2. 022506 


Content  of  ullage,  in  wine  gallons,sr  43. 75  .  Log.  »     1. 640973 

Constant  bg.  s    9.920703 

Content  of  the  ullage,  in  imperial  galls.=36. 45  Log.  s     1  •  56 1 676 

f. 


Digitized  by 


Google 


PRACTICAL   GAUQINO.  605 

Nbt^. — If  the  content  of  the  cask  be  given  agreeably  to  the  imperial 
standard  measure,  let  the  logarithm  thereof  be  added  to  the  logarithm  of 
the  multiplier;  and  the  sum  will  be  the  logarithm  of  the  ullage.  Thus,  in 
the  above  example,  let  the  content  of  the  cask  be  given  in  imperial 
treasure ;  viz.,  87*  74  gallons  j  then, 

MulUpliers::       4154  Log.  s  9*.  618467 

Content  of  the  cask,  in  impl.  meas.  =  87. 74  gallons    Log.  s  1. 943209 

Content  ofullage,  in  imperial  galls,  s  36.45  Log.  =  1.561676 

Example  2. 

Let  the  length  of  a  cask,  between  the  heads,  be  31  inches,  its  content 
in  wine  measure  54. 52  gallons,  and  the. depth  jof  th6  ullage  18. 5  inches; 
required  the  quantity  of  liquor  in  the  cask  ? 
Depdi  of  ullage,  or  wet  inches,  18. 5  -h  31 

inches  (length)  s  .5968  quotient, .  5968,  which  is  more  than  the  constant 
Constant  decimal  =  .  5000  decimal. 

Difference  s  •    .     . .  968  -f-  10  =  .  .97,  additive. 

Multiplier  =£  ........     .6065  Log.    =     9.782831 

Content  of  the  cask,  in  wine  galls.^:  54. 52  Log.    =:     1 « 736582 

Content  of  ullage,  in  wine  galls,  a    33. 07       '         Log.    =     1 . 5 1 9413 

:       Constant  log.   =     9. 920703 


Content  of  ullage,  in  imperial  galb.=27. 55  Lo^.    =     1 .  4401 16 

But  if  the  content  of  the  cask  be  given  in  imperial  measure,  viz.,  45, 42 
gallons,,  then  the  latter  part  of  the  operation  will  be  as  thus : — 

Multiplier,  as  above,  =       . 6065  Log.  =  9.782831 

Content  of  the  cask,  in  imperial  meas.=45. 42  gallons    Ldg.  —  I.  657285 

Content  of  ullage,  in  imperial  galls.  =  27. 55  Log.  =2  1. 4401 16 

Remarks — If  the  dry  inches  of  the  length  of  the  cask  be  made  use  of 
instead  of  the  wet,  the  result  of  the  operation  will  express  the  vacuity  in 
the  cask ;  and  if  this  vacuity  be  added  to  the  ullage,  the  sum  will  giv^  the 
content  of  the  cask :  but  this,  it  is  presumed,  does  not  need  to  be  elucidated 
by  an  example. 

Note.— The  following  Table,  which  is  particularly  adapted  to  the  reduc- 
tion of  the  old-established  wine  and  ale  measure  into  the  new  general 
standard  or  Imperial  measure,  and  the  contrary,  will,  be  found  of  very  con- 
siderable use  in  the  event  oi*  purchasing'  wine  or  spirits  in  places  out  of 
His  Majesty's  dominions. 


Digitized  by 


Google 


606 


PftAOnCAL   OA.06INO. 


A  Tablb 

For  readily  finding  the  Number  of  Wine  or  Ale  OdUon$  tohieh  is  adualif 
equivalent  to  any  given  Number  of  OaUcm  of  the  newly^e8taUi$hed 
general  standard  or  Imperial  Measure,  and  conversely. 


n 

r 

Wine  Measure 

Ale  Measure. 

H 

Imperial  Meas.  . 

4 

Imperial  Meas.   1 

G. 
0 

Q 

0 

p. 

0 

GiUs. 

Ss 

G. 

a 

p. 

GUU. 

^s. 

G. 

2 

GUU. 

•^s 

G. 

fi 

P. 

Gills. 

1.200 

Iffil) 

0 

0 

0.983 

if^ii 

0 

0 

.0.833 

T 

0 

0 

0 

1.017 

0 

0 

0 

2.401 

2<o. 

0 

0 

1.966 

2do. 

0 

0 

1.666 

0 

0 

0 

2.034 

Ido. 

0 

0 

0 

3.601 

3do. 

0 

0 

2. 950 

ido. 

0 

0 

2.499 

Mo. 

0 

0 

0 

3.051 

Lpt. 

0 

0 

1 

0.801 

lpt. 

0 

0 

3.933 

lpt. 

0 

0 

3.332 

lpt. 

0 

0 

.1 

0.068 

Iqt. 
2qt8. 

0 
0 

1 

2 

0 
0 

1.603 

Iqt. 

0 

0 

- 

3.866 

Iqt. 

0 

0 

1 

— 

2.665 

iqt. 

0 

1 

0 

0.136 

3.20.5 

2qts. 

0 

1 

3.732 

2qts. 

0 

1.330 

2qts. 

0 

2 

0 

0.273 

'Tf 

0 

3 

1 

0.808 

3qts. 

0 

2 

3.598 

% 

0 

2 

3.1595 

)qt8. 

0 

3 

0 

0.409 

1 

0 

1 

2.410 

G.1 

0 

3 

3.464 

0 

3 

2.660 

G.1 

1 

0 

0 

0.645 

2 

2 

1 

1 

0.820 

2 

1 

3 

2.927 

2 

1 

2 

1.319 

2 

2 

0 

0 

1.091 

3 

4 

3 
4 

2 
3 

0 

3.231 

3 

2 

3 

2.391 

3 

2 

1 

3.979 

3 

3 

0 

0 

1.636 

0 

1.641 

4 

3 

3 

1.855 

4 

3 

1 

2.638 

4 

4 

0 

0 

2.182 

5 

6 

tf 

0 

0.051 

5 

4 

3 

1.318 

5 

4 

0 

1.298 

5 

5 

0 

0 

2.727 

6 

7 

0 

1 

2.461 

6 

5 

3 

0.782 

6 

4 

3 

3.957 

,    6 

6 

0 

0 

3.273 

7 

8 

] 

1 

0.872 

7 

6 

3 

0.246 

7 

5 

3 

2.617 

7 

7 

.0 

0 

3.818 

8 

Si 

9 
10 

2 
3 

0 
0 

3.282 

8 

7 

3 
3 

3.710 

8 

6 

2 

1 

1.277 

« 

,  8 

0 
0 

1 

0.364 

1.692 

9 

8 

3.173 

9 

7 

3.936 

9 

9 

1 

0.909 

10 

12 

0 

0 

0.102 

10 

9 

3 

2.637 

10 

8 

1 

2.596 

10 

TO 

0 

1 

1.454 

20 

24 

0 

0 

0.205 

20 

19 

2 

1.274 

20 

16 

2 

1 

1.191 

20 

20 

1 

0" 

2.909 

30 

36 

0 

0 

0.307 

30 

29 

1 

.3.911 

30 

24 

3 

3.787 

30 

30 

2 

0 

0.363 

40 

48 

0 

0 

0.409 

- 

40 
50 

39. 
"49^ 

0 
0 

— 

2.548 

40 

33 

1 

2.383 

40 

40 

2 

1 

1.818 

50 

60 

0 

0 

0.511 

1.185 

50 

41 

2 

0.978 

50 

5U 

3 

0 

3.272 

60 

72 

0 

0 

0.614 

60 

58 

3 

3.822 

60 

49 

3 

3.574 

60 

61 

0 

0 

0.727 

70 

84 

0 

0 

0.716 

70 

68 

3 

2.459 

70 

58 

1 

2. 170 

70 

n 

0 

1 

3.181 

80 

96 

0 

0 

0.818 

80 

78 

2 

1.096 

80 

66 

2 

0.765 

80 

81 

1 

0 

X635 

SO 
100 

108 
120 

0 
0 

0 
0 

0.921 

90 

8» 
98 

1 
1 

- 

3.733 

90 

74 

3 

1 

3.361 

90 

91 

2 

0 

1.090 

1.023 

100 

2.370 

100 

83 

1.956 

100 

roi 

2 

I 

'2.544 

200 

240 

0 

0 

2.046 

200 

196 

2 

0.740 

.200 

166 

2 

3.91.3 

200 

203 

1 

1 

1.0B9 

300 

360 

0 

0 

3.069 

300 

294 

3 

.3.110 

300 

249 

3 

1.869 

300 

305 

0 

0 

3.633 

400 

480 

0 

1 

0.092 

400 

393 

1 

1.479 

400 

33S 

0 

3.826 

400 

406 

3 

0 

2,177 

500 
600 

600 
.720 

0 

7 

1 

1.U5 

500 

491 

2 

3.849 

500 
«00 

416 
499 

2 
3 

1.782 

500 

m 

2 
0 

0 

1 

0.721 

1 

2.138 

€00 

589 

2^219 

3.739 

600 

610 

3.166 

700 

840 

0 

1 

3.161 

700 

688 

1 

0.589 

700 

583 

0 

1.695 

700 

711 

3 

1 

hSlO 

800 

960 

1 

0 

.0. 184 

800 

786 

2 

2.959 

80^ 

666 

1 

3.652 

800 

813 

2 

1* 

0.354 

900 

1080 

1 

0 

1.807 

900 

884 

3 

i.329 

900 

749 

3 

1.608 

900 

915 

1 

0 

2.898 

1000 

1200 

1 

0 

2*229 

1000 

983 

0 

3.699 

1000 

833 

0 

0 

3.5^5 

1000 

1017 

0 

0 

1.443 

Note. — ^In  using  the  above  TaUe,  if  the  given  number  of  gaHon^  csnnot 
be  exactly  found,  or  if  it  fall  without  the  limits  of  the  Table,  tl»e  sum  ^ 
the  different  quantities  corresponding  to  the  several  terms  wbieh  mtke  wp 
the  given  number  of  gallons  is,  in  such  eases,  to  be  taken ;  as  in  tbi 

following  jexamples : — 


Digitized  by 


Google 


BnaCBLLANBOlTfl  PROBLEMS.  607 

'Example  L 
In  1736  gallons^  imperial  measure^  how  many  gallons  of  wine  measure  ? 

G.      Q.  P.  Gills. 
1000  galls.,  impl.  meas.,  are  equal  to  1200. 1. 0. 2. 229  W.  M. 
700  ditto  ditto  840.0. 1.3.  161  ditto. 

30  ditto  ditto  36.0.0.0. 307  ditto. 

6  ditto. '  ditto  7.  0.  1. 2. 461  ditto. 


Hence,  1736  galls.,  impl.  irieas.,  artf  equal  to  2083. 3. 0. 0. 158  W.  M. 

•    Example  2. 
In  1839  gallons,  \nne  measure,  how  many  gallons  imperial  measure  ? 

G.   2.  P.  GiUi. 
1 000  galls,  wine  meas.^  are  equal  to   833. 0.  Q-  3. 565  impl.  meas. 
800  ditto  clitto        666. 1. 1.3. 652     ditto. 

30  ditto  ditto  24. 3. 1. 3. 787     ditto. 

9  ditto   .  ditto  7.1.1.3.936    ditto. 


Hence,  1839  galls.^  wine  meas'.,  are  equal  to  1532. 0. 0. 2.  940  imjil.  meas.* 

SOLUTION  OF  MISCELLANEOUS  PROBLEMS. 

Problem  I. 

Qioen  the  Grcumference  of  a  Cablcy  and  its  Length;  to  find  its  Weight, 

Rule. 
To  twice  the  logarithm  of  the  circumference  of  the  given  cable,  add  the 
logarithm ^f  its. length,  and  the  constant  logarithm  9. -734967:  the  sum  of 
these  three  logarithms  (abating  10  in  the  index,)  will  be  the  weight  of  the 
given  cable,  in  pounds,  avoirdupois. 

.    Example. 
Let  the  circumference  of  a  cable  be  21  inches,  and  its  length  110 
fathoms ;  required  its  weight  ?  . 

Circumference  or  girt  of  given  cable=21  inches  Twice  its  log. =2.  6^4438 
Length  of  ditto,  in  fathoms,  =:     •     110  Log.  =  .    ,    2.041393 

Ckmstant  log.  =  •    .  '.    .    .    .  ..     .    .    '.     .    ....    9.734967 

Weight  of  the  given  cable,  in  pounds,=:2635 1 ,  Log.  =:  .    «    4. 420798 

*  A  general  VictuaUing  Table  is  ^ven  at  the  end  of  the  gecood  volume,  which  wiU  be 
found  of  considerable  utility  to  the  Pureers  of  the  Naval  service,  in  mal^ing^  out  their  annual 
accoiuits,  or  in  contpletiBg^  the  pnyvidioDs  of  their  ships  to  any  given  time. 


Digitized  by 


Google 


608  MISCBLIJINBOUS   PROBLBM8* 

Remark. — It  has  been  found,  by  actual  experimeit,  that  1  fiitlioni  of  a 
hemp  cable  which  measuret  9  inches  in  circumference  weighs  44  lbs.  avoir- 
dupois. Now,  since  cylinders  of  equal  lengths  are  as  the  squares  of  their 
circumferences, — therefore,  as  the  square  of  9  inches  (the  circumference 
of  the  experimented  cable),  is  to  the  weight  of  1  fathom  thereof,  viz^ 
44  lbs. ;  so  is.  the  square  of  the  circumference  of  any  other  cable*  to  the 
weight  of  1  fathom  of  such  cable:  which,  multiplied  by  the  length  of  the 
cable,  will  give  ite  whole  weight  The  constont' logarithm  9.734967  is 
found  by  adding  the  arithmetical  complement  of  twice  the  logarithm  of  9 
inches  to  the  logarithm  of  44  pounds. 


Problbm  II. 

Given  the  Diameter  of  a  Circle  i  to  find  Us  Circun^er€nce* 

Rule. 

To  the  logarithm  of  the  diameter  of  the  given  circle  add  the  constant 
logarithm  0. 497150,  atid  the  sum  will  be  the  logarithm  of  the  circumfer- 
ence of  that  circle. 

Example  1. 

Let  the  diameter  of  a  circle  be  78. 41  yards ;  required  its  circumference? 

Diameter  of  the  given  circle  =     .     •    78. 41  yards    Log.  =  1.  894372 
Constant  log.  = .    .0.497150 

Circumference  of  the  given  circle,  in  yards,  =  246. 33  Log.  =  2. 391522 

No/«.^— The  circumference  of  a  circle  whose  (Uameter  is  wiity  or  1,  is 
3. 14159265  ;  and,  since  the  circumferences  of  circles,  are  to  each  other,  as 
their  diameters,,  or  radii,— therefore,  as  the  diameter  1,  is  to  its  circum- 
ference 3. 14159265 ;  so  is  the  diameter  of  any  otlier  circle,  to  its  circum- 
ference :  and  hence  the  above  rule.  The  constant  logarithm  is  expressed 
by  the  logarithm  of  3. 1 4 1 59265. 

Example  2. 
If  the  diameter  of  the  earth  be  79 17*  .5  miles,  what  is  its  circumference? 

Diameter  of  the  earth  q=     .    .    .    .    7917.5  miles    Log. «  3.898588 
Constant  log.  =s  . •    .     0.497150 

Circumference  of  the  earth,  in  miles,=: 24873. 5  Log.  =  4. 395738 

The  converse  of  this  problem,  viz.,  deduchig  the  diameter  from  the 
circumference,  is  obvious.— See  Problem*  IV.,  page  594.    * 


Digitized  by 


Google 


MISCELLANEOUS   PROBLEMS,  01151 

PROBJUkM   in. 

Gioen  the  Diamei^  of  a  Grcle  ^  to  find  its  Jrea,  or  wpeftftdal  Omient 

Hulk. 

All  circles  are  to  one  another,  as  the  squares  of  their  diameters;  and  as 
die  area  of  a  circle  whose  diameter  is  unity  or  1,  is  •  7853982,  the  loga- 
rithm of  which  is  9. 895090, — therefore,  to  twice  the  logaridim  of  the 
given  circle,  add  the  constant  logarithm  9.  895090 ;  and  X\ie  sum  (abating 
10  in  the  index,)  will  be  the  logarithm,  of  the  area^  or  superficial  contept 
of  that  ciTcIe. 

Example.  • 

If  the  diameter  of  a  circle  be  78. 41  yrf^ds,  what  is  its  area  or  superficial 
content? 

Diameter  6(  the  given  circle  s±  78. 41  yards    IVnce  its  log.  ==  3. 788744 
Constant  log.  =  ..,    ....•., 9.895090 

Area  of  the  g^ven  circle,  in  yards,  s  4828. 8    Log.  =      .    •    3. 683834 


Problem  IV. 

Given  the  Area  or  supeificial  Content  of  a  Circle :  to  find  Us  Diameter. 

Rule. 

As  this  problem  is  evidently  the  converse  of  the  last,— therefore,  to  the 
logarithm  of  the  area  of  the  given  circle,  add  the  constant  logarithm 
0. 104910  (the  arithmetical  complement  of  9. 895090). :  divide  the  sum  by 
2,  and  the  quotient  will  be  the  logarithm  of  the  diameter  of  the  given 
circle. 

Example. 

Let  the  area  of  a.  circle  be  48^8. 8  yards;  required  its  diameter  ? 

Area  or  superficial  content  of  given  crrcle;=4828. 8  yards    Log.=s3. 683834 
Constant  log.  =  .    .    ............    .0.104910 

Divide  by  2)  3. 788744 

Diameter  of  the  given  circle,  in  yards>  s  78. 4 1    .Log.  =s  •    •     1  •  894372 

2  R 


Digitized  by 


Google 


610  VISfSLLANBOUS  PAOBUUCS* 

PfU>BJLBM  V. 

Given  the  Diameler  tffa  Grcfe;  to  jmd  I/m  Siilfi  qfa  Square  equal  m 
Area  to  thai  .CSrcfe. 

RULB. 

To  the  logarithm  of  the  diameter  of  the  given  cifble^  add  the  conrtant 
logarithm  9. 947545  (the  logarithm  of  the  square  root  of  .  7853982)  ;  and 
the  sum  (abating  10  in  the  index,)  will  be  the  logarithm  of  the  §ide  of  a 
square  equal  in  area  or  eaperficial  content  to  that  circle. 

Example*  " 

If  the  diameter  of  a  circle  be  78. 41  yards^  what  is  the  side  of.a  square 
equal  in  area  to  that  circle  ?  * 

Diameter  of  the  given  circle  =n     •    •    •     78.41  yards  Log.  ==  1.894372 
Constant  log.  =a  ...'..    ^    ........    .     9.947545 


Side  of  the  required  square^  in  yards^  =  69. 49  Log.  =  1. 841917 


Problem  VI. 

Given  the  Diameter  ofaGrcle;  to  find  the  Side  of  a  Square  mecribed 

in  that  CSrcle. 

RviB. 

To  the  Ic^arithm  of  the  diameter  of  the. g^ven  circle^  add  the  constant 
logarithm  9;  849485 ;  and  the  sum  (abatiiig  10  in  the  index,)  will  be  the 
logarithm  of  the  side  of  a  square  inscribed  in  that  circle. 

Example.' 

If  the  diameter  of  a  circle  be  78. 41  yards,  what  is  the  side  of  a  square 
inscribed  in  that  circle  ? 

Diameter  of  the  ^ven  circle  =  .     .     .     78*41  yards  Log.  =   1.894372 
Constant. log.  s      ••.......     ^    «...    .      9.849485 


Side  of  the  .inscribed  square,  in  yards,  =  55. 44  Log.  =  1. 743857 


Digitized  by 


Google 


MUCJUXANXOns  PR0BIJIM8.  611 

pRomsM  VII. 

Gken  the  iran$verse  and  the  conjugate  JHameten  qfon  EOipsiet  tofiiui 

.itiJrea. 

RULV. 

To  the  logarithms  of  the  longer  and  the  shorter  diatneters  of  the  ellipsis, 
add  the  constant  logarithm  9, 895080 :  the  sum  (abating  10  in  the  index,) 
will  be  tlfe  area  of  that  ellipsis.  ^ 

Example. 

'  Let  the  transverse  diameter  of  an  ellipsis  be  616  yards,  and  its  conjugate 
diameter  445  yards ;  requiredthe  area  or  superficial  content  of  that  ellipsis? 

Transverse  diameter  s  «.  •  •  616  yards  Log.  =:  2*789581 
Conjugate  diameter  :s  •  .  ,  445  yards  Log.  s  2. 648360 
Constant  log.  =     ..•••••,...       9.895090 


Area  of  the  given  ellipsis,  inyards,32l5294  Log.  s  5.833031 


Probjjbm  VIII. 

Given  the  transverse  and  the  conjugate  Diameters  of  an  Ellipsis  ;  to  find 
the  JMameter  of  a  Circle  equal  in  Area  to  that  EOipeis* 

Rule. 

To  the  logarithm  of  the  longer  diameter,  add  the  logarithm  of  the 
shorter  diameter;  divide  the  sum. by  2,  and  the  quotient  will  be  the  loga- 
rithm of  the  diameter  of  a  circle  equal  in  area  to  the  ellipsis. 

'Example* 

Let  the  transverse  diameter  of  an  ellipsis  be  616  yards,  and  its  conjugate 
diameter  445  yards ;  required  the  diameter  of  a  circle  equal  in  area  to  that 
ellipsis? 

Transverse  diameter  of  the  given  ellipsisst  616  yards  Log.  s=  2.789581 
Conjugate  diameter  of  ditto  =s     «     .    •    445  yards  Log.  =s  2.648360 


Divide  by  2)  5.437941 


Diameter  ofa  circle  equal  in  area=:52. 356  yards  Log.  =  2.718970^ 

2b2 


Digitized  by 


Google 


612  mSCBLLAMSOUS  PBOBUUIS. 


Problbm  DC. 

Given  the  trantvene  and  the  angugaie  Diameten  qfan  EOifme^  to  fied 

Us  drcumference. 

Rule. 

Square  the  two  diameters ;  add  those  squares  together :  take  half  the 
sum,  and  find  the  logarithm  correspondhig  thereto.  Now,  the  half  of  this 
logarithm  will  be  Uie  logarithm  of  a  natural  number,  which,  being  added 
to  half  the  sum  of  the  two  diameters,  will  give  die  cdtreeted  mean  diameter. 

To  the  logarithm  of  the  corrected  mean  diameter,  thus  found,  add  the 
constant  logarithm  0. 1915121 ;  and  the  sum  will  be  the  logarithm  of  the 
circumference  of  the  ellipsb. 

Example. 

Let  the  transverse  diameter  of  an  ellipm  be  616  yards,  and  its  conjugate 
diameter  445  yards ;  required  its  circumference  ? 

Tr.diam.616x616i=379456,  the  square. 
Conj.do.445  x  445= 198025,      ditto. 


Divide  by  2 )  577481,  sum  of  the  squares. 
Halfsuinofsquarte=:288740iLg.5.4605077 

Half  the  logarithms^      .    ..     2.7302588iNtJVoJ>37.3519 
Half  the  sum  of  the  two  diameters  =      ...    530.5 


Corrected  mean  diameter  =  .    .    .    .    .     .    .  1067. 851 9Lg^.  0285 11 

Constant  log.  =  . 0.196121 

Circumference  of  the  ellipsis,  in  yards,  =  1677.38    Log.  s  ,    3.224632 


PaoBLmtf  X. 

I  •    •  •  ■        • 

Giventhe  Diameter  of  a  Sphere,  or  Globe:  to  find  iti  SoluKiy. 

To  thrice  the  logarithm  of  the  diameter  of  the  given  sphere,  add  the 
constant  logarithm  9.  718999  j  and  the  sum  (abating  10  in  the  index,)  will 
be  the  solid  content  of  such  sphere. 


Digitized  by 


Google 


MISC&LLANBOUS   PROBLEMS.  613 

Example^  • 

If  the  diameter  of  the  earth  be  ^[9 1  /•  5  miles^  what  is  its  solidity  ?  , 

Diameter  of  the  earth  =s  7917:5  miles    Thrice  its  log*  s  IL  695764 
Constant  log.  = 9.718999 

Solidity  of  the  earth,  in  miles,  ^  259874059701 . 5    Log.  s  1 1 .  414763 

Note. — It  has  been  found  that  the  solidity  or  solid  content  of  a  sphere, 
whose  diameter  is  unity  or  1,  is  .  5235988 ;  and  sincje  spheres  are  to  one 
another, .  as  the  cubes  of  their  dian)eters,-^therefore,  as  the  cube  of  the 
diameter  1,  is  to  its  solidity  .  5235988 ;  so  is  the  cube  of  the  diameter  of 
any  other  sphere  or  globe,  to  the  solidity  of  such  sphere  or  globe :  and 
hence  the  above  rule.  The  constant  logarithm  is  expressed  by  the  loga- 
rithm of .  5235988. 

Remark. — ^For  the  method  of  finding  the  number  of  square  miles  con- 
tained in  the  earth's  superficies,  see  Problem  V.,  page  594. 


Prqblbm  XI. 

Given  the  Earih'i  Diameter;  to  find  the  Height  to  which  a  Person  should 
be  raised  to  see  onC'third  of  its  Surface. 

RuLB.  .;        . 

From  twice  the  logarithm  of  the  earth's  semi-diameter,  subtract  the 
logarithm  of  its  one-third :  the  remainder  will  be  the  logarithm  of  the 
height  to  which  a  person  should  be  raised  above  the  earth's  centre,  to  see 
one-third  of  its  surface ;  from  which  let  the  earth's  radius  or  semi-diameter 
be  taken,  and  the  remainder  will  be  the  required  height  abore  its  surface. 

Example. 

How  high  above  the  earth  must  a  person  be  raised,  that  he  may  see  one- 
third  of  its  surface  ? 

Earth's  semi-diameter  =    .    .    3958. 75  miles  Twice  its  log.=:7. 195 1 16 
One-third  of  ditto  =     .    .    .     1319.5833  miles      Log.  =     3.120437 

Height  above  the  earth's  centre  =  1 1876. 25  miles    Log.  s    4. 074679 
Deduct  the  earth's  semi-diameter  s  3958. 75  miles. 


Remainder  = 7917^^50  miles;   which  is  the  true 

height  to  which  a  person  should  be  raised  above  the  earth,  to  see  one- 
third  of  its  stirface.  ' 


Digitized  by 


Google 


614  MISCSU^NBOUS  PR0BLBM8. 

Rrobubm  XIL 

GiveA  the  Etirth*s  SemUDiaimetery  and  the  SufCi  fnean  horizonial 
ParalUuf;  to  find  the  Earth's  Distance  from  the  Sm. 

RtJLE. 

To  the  lofpurithm  of  the  eartVa  6emi«diameter^  add  the  logarithi 
co-tangent  of  the  sun's  mean  horizontal  parallax;  and  the  sum  (abating 
10  in  th^  index^)  will  \>e  the  logarithm  of  the  sun's  m^an  distance  from 

the  earth. 

Example, 

By  the  traosito  of  Veutis  over  the  sun  s  disk  in  the  yetfr^  1761  end  17695 
the  aun'a  me^u  borisontal  parallax  appears  to  be  about  8. 65  seconds  of  a 
degree;  now^  if  the  earth's  semi- diameter  be  3958. 7&  mites^  its  mean 
distance  from  the  sun  is  required  ? 

Semi-diameter  of  the  earth  ^     .    3953. 75  nules    Log«  =?  3^  5975£81 
Mean  horizontal  parallax  of  the  sun=8^.  65    L(>g.  cp-tang.=  14. 3730860 


Earth's  mean  distance  from  the  8im^94546196  miles  Log.  =  7. 9756441 


Problem  XIIL 

Gioen  the  Sun's  mean  Distance  frtm  the  Earth,  and  his  apparent  Semi^ 
JHameieir,  at  a  mean  Rater  tojind  the  true  Measure  ^fkis  Diameter, 
in  EngUsk  JIMles. 

RULB. 

"Te  the  logarithm  of  the  sun's  mean  distanee  from  the  earthy  add  Ae 
logarithmic  tangent  of  his  seni-dlameter  |  and  the  sum  (abating  10  tn  the 
index,)  will  be  the  logarithm  of  the  sun's  semi-diameter,  in  English  miles  ; 
the  double  of  which  will  be  the  measure  of  his  whole  diameter. 

Example. 

If  th^  sun's  mean  disUwce  from  the  earth  be  94546196  English  nules^ 
and  his  mean  Apparent  semi-diameter  I6in^.65,  the  true  measure  of  his 
diameter  is  required  ?  * 

Sun's  mean  distance  from  the  eartK=9454619e  miles     Log.  =  7.  9756441 
Sun's  apparent  semi-diamct»r  =       16f  1  *.  63  Lqg.  tangent=:7. 6683950 

Sim's  tfue  semi-diameter  »     .    .    440797. 5  miles    Log.  a  5. 6442391 

■ '        .III 
True  measure  of  the  sun's.diameter=  88 1 595    English  veilm* 

/ 


Digitized  by 


Google 


PftOBLBM   XIV. 

Oioen  ike  Diameters  of  the  Earth  and  the  Sun;  to  find  the  Batio  af 

thrir  Magniiudee*  . 

RULB. 

.Since  the  magnitudes  of  all  spherical  bodies  are  as  the  cubes^  or  tripli- 
cate ratio,  of  their  diameters  (Euclid,  Boole  XII.,  Prop.  18), — therefore, 
from  thrice  tiie  logarithm  of  the  sun's  diameter  subtract  thrice  the  lo^- 
rithm  of  the  earth's  diameter,  and  the  remainder  will  be  the  logarithm  9f 
the  ratio  of  their  magnitudes. 

Esampte. 

If  the  earth's  diameter  be  7917.&  Bnglkh  miles,  and  that  of  the  sun 
88 15  95  such  miles,  required  the  ratio  of  their  magnitudes  ? 

Sun's  diameter,  in  English  miles,  =  881595  Thrice  its  log.  =17.8358076 
Earth's  diameter.  In  ditto,  =  .    .    7917. 5  Thrice  ita  log.  =  11. 6957643 

Ratio  of  the  magnitudes  of  the  earth  and  sua»(  1380532  Log.ss6. 1400433 


Problsm  XV. . 

Owen  the  Circumference  qf  the  Earth;  to  find  the  Rate,  per  HauTf  at 
<   which  the  Inhabitants  under  the  JSquator  are  carried,  in  consequence  of 
the  Earth's  diumtil  Motion  round  Us  Axis. 

;  Rule. 

To  the  arithmetical  ooniplement  of  the  logarithm  of  24  hours,  add  the 
logarithm  of  the  earth's  circumference,  and  the  logarithm  of  1  hour:-  the 
sum  of  these  three  logarithms  (abating  10  in  the  index,)  will  be  the  loga- 
rithm of  the  rate  per  hour  at  .which  the  inhabitants  under  the  equator  are 
carried  by  the  earth's  diurnal  motion  on  its  axis. 

Example. 

'  Let  the  circumference  of  the  earth  be  24873. 5  miles  ;  required  the  rate 
per  hour  at  which  the  inhabitants  under  the  equator  are  carried,  in  con- 
sequence of  the  earth's  diurnal  motion  ? 

One  day,  or  24  hours,  Arith.  comp.  of  its  log.  =  8.6197888 
Earth's  circumference  s±  24873. 5  miles  Log.  =  4. 3957369 
Given  tittle,  or    ....     I  hour  Log.  =    0.0000000. 

Rate  per  hour,  in  miles,  =     1036.396, '  Log/  ss    3.0155257 


616  ICIflCBLLILNBOnS  FEOBLBMS. 

Pkoblbm  XVI. 

To  find  the  Bate  ai  whieh  the  InhabUanis  tmdtr  amf  gioem  ParaBel  of 
Latitude  are  carKed,  in  consequence  of  the  Earth's  dkamal  MoUtm  em 
its  Axis. 

Rule. 
The  circumference  of  the  earth  under  the  equator  is  24873. 5  miles ; 
and  since  the  circumference  under-  any  parallel  of  latitude  decreaaes  in 
proportion  to  the  co-sine  of  the  latitude  of  such  parallel, — therefore,  to 
the  logarithm  of  the  earth's  circumference,  under  the  equator,  add  die 
logarithmic  co-sine  of  the  latitude  of  the  given  parallel ;  and  the  sum 
(abating  10  in  the  index,)  will  be  the  logarithm  of  the  earth's  drcumfcr- 
ence  under  that  parallel :  with  whifeh  proceed  as-  directed  in  the  last 

problem* 

Example. 

Let  the  circumference  of  the  earth  be  24873. 5  miles;  required  the  rate 
per  hour  at  which  the  inhabitants  under  the  parallel  of  London  are  carried 
by  the  earth's  motion  on  its  axis  ? 

Circumference  of  the  earth  =  24873. 5  miles    Log.  =  .     .    4. 3957369 
Latitude  of  the  parallel  of  London  =  51  ?3 1 C     Log.  co-sine  =  9. 793990? 

Circumference  under  given  parallels:  15478. 45  Log.  =  .     .     4. 1897276 
One  day,  or  24  hours,    Arith.  comp.  of  its  log.  =      .    .     .     8. 6197888 

Rate  per  hour,  in  miles,  as  required,  =  644. 93  Log.  =  •    •    2. 8095164 


PaoBLBM  XVn. 
To  find  the  Length  of  the  tropkdl  or  solar  Year. 

Rule. 

It  has  been  found,  by  observation,  that  the  sun  apparentlgf  advances  in 
the  ecliptic  59^8^.33  of  a  degree  every  day  at  a  mean  rate;  thatis,  from 
the  time  of  his  leaving  any  given  meridian  to  the  time  of  his  returning  to 
the  same  meridian.  Now,  since  the  ecliptic  is  a  great  circle  of  360  degrees, 
— ^therefore,  as  the  sun's  apparent  diurnal  motion  in  the  ecliptic,  is  to  I 
day,  or  24  hours ;  so  is  the  great  circle  of  360  degrees,  to  the  true  length 
of  the  tropical  or  solar  year ;  that  is,  to  the  time  of  the  sun's  periodical 
revolution  round  the  ecliptic  from  any  equinoctial  or  solstitial  point  to  the 
same  point  again;    Hence,  by  logarithms. 


Digitized  by 


Google 


MISGBLLANBOUS   FROBLBMS.,  617 

Example, 

The  sun's  daily  motion  in  the  ecliptic  is  59' 8^.  33  in  every  natural  day, 
or  24  hours,  at  a  mean  rate;  required  the  length  of  the  tropical  or  solar 
year? 

Sun's  app.  diur.  motion  59 'S"".  33,  in  8ecs.=3548. 33  Log.ar.co.6. 4499760 
One  day,  or  24  hours,  in  seconds  =  .  .  86400  Log.  =  4. 9365 1 37 
Ecliptic,  or  great  circle  of  360?,  in  secs.=  1 296000    Log.  =     6.11 26050 

Length  of  the  tropical  year,  in  seconds=:3IS56928  Log.  =  7. 4990947 
Hence  the  tro)iical  or  solar  year  con^sts  of  365  '5 !  48T48 ' ,  as  required. 


Problbm  XVIIL 

To  Jtnd  the  Rate  at  winch  the  Earth  tnoves  in  the  EcUptk  during  the 
Time  of  its  annual  or  periodical  Revolution  round  the  Sun» 

Rule. 

Since  the  earth's  mean  distance  from  the  sun  .is  94546196  miles 
(Problem  XII.,  page  614),  the  diameter  of  the  orbit  in  which  it  moves 
round  that  great  luminary  is  189092392  miles;  and  since  the  diameter  of 
a  circle  is  to  its  circumference  in  the  ratio  of  unity  or  1,  to  3. 14159265, 
the  circumference  of  the  earth's  orbit  is  594031320  mWei.  Now,  as  the 
earth  describes  this  circumference  in  36if5*48T48!  (last  problem),  or 
8766  hours  nearly,  we  have  the  following  computation  by  logarithms : — 

As  the  length  of  the  year,  in  hours,  =^  8766     Log.  ar.  comp.=6. 0571985 
Is  to  the  circumf.  of  the  f^atth's  orbits=59405 1320  miles  Log.3=8. 7738239* 
Sois *    Ibour  Log.=:0. 0000000 

To  the  earth's  hourly  motion  in  i|»  orbit  =  67768  miles  Log.=4. 8310224 


Problem  XIX. 

Given  the  Moon's  mean  Distance  from  the  Earth,  and  her  apparent 
Semi-diameter,  at  a  mean  Rate;  to  find  the  true  Measure  of  her 
Diameter,  in  English  MUes. 

RuLBt 

it  b  shown  in  page  9,  under  the  head  ^'Augmentation,  of  the  Moon's 
Semi'diameter/'  that,  the   moon's-  mean  distance   from  the   earth   is 


Digitized  by 


Google 


618  MI8CBXXAKBOUS  PR0^JE1C8« 

236692.35  miles.  Now,  since  her  apparent  semi-diameter  is  15 M3?  at 
a  mean  rate, — ^therefore,  to  the  logarithm  of  her  mean  distance  from  the 
earth,  add  the  logarithmic  tangent  of  h^r  apparent  semi^diameter^  and 
the  sum  (abating  10  in  the  index,),  will  be  the  logarithm  of  the  moon's 
semi-diaAeter  in  English  miles  :  the  double  of  which  will  be  the  measnxe 

of  her  whole  diameter.  . 

Example. 

• 

Let  the  moon's  distance  from  the  earth  be  236692. 35  miles,  and  her 
semi-diameter  15  M3^;  required  the  true  measure  of  her  diameter  in 
English  miles  ?       . 

Moon's  mean  distance  from  the  earths 236692. 35  miles  Log.ft=5. 374IS42 

Moon's  apparent  semi-diameter  =    15'43?     Log.  tangent  =  7*  6600896 

Moon's  true  semi-diameter  =  • .  •  1082. 1  miles  Log.=3. 0342738 
True  measure  of  the  moon's  diameter=2164. 2  English  miles. 


Problem  XX. 

CUcen  the  Diameters  of  the  Earth  and  the  Moon  ;  to  find  the  Ratio 
of  their  Magnitudes, 

Nofe.^This  is  performed  by  Problsm  XTV.,  page  615. 

*    Example^    , 

If  the  earth's  diiimeter  be  7917*  5  English  miles,  and  that  of  the  moon 
2164.  2  such  miles,  required  the  ratio  of  their  magnitudes  ? 

Diameter  of  the  earth  :a       .    .    7917.5    Thrice  its  log.=r II.  6957643 
Diameter  of  the  moon  ^      •    .    2164.2    Thrioe  its  log.s  10. 0058922 


Ratio  of  the  magnitndes  of  the  earth  and  moon=48. 96  LfOg.=I.  6898721 


t^ROBLBM  XXI* 

To  find  haw  wmeh  larger  the  Earth  appears  to  the  hmar  InhaBUasiis 
than  the  Moon  appears  to  the  terrsstrial  Inhabkanis. 

RULB. 

Since  the  distance  between  the  earth  and  the  moon  is  such  as  to  cause 
their  opposing  hemispheres  to  appear^  reci]meal)y  from  each  other,  like  flat 


Digitized  by 


Google 


circles  \  and  Binee  circles  are  to  one  another  as  the  squares  of  their  diameters 
(Euclid^  Book  XII.,  Prop.  2^)  or,  which  is  the  same  thbig,  since  spherical 
surfaces  are  to  each  other,  as  the  squares  of  their  radii,-«*therefore,  from 
twice  the  logarithm  of  the  earth's  diameter,  subtract  twice  the.  logarithm 
of  that  of  the  moon ;  and  the  remainder  will  be  the  logarithm  of  the  number 
of  times  that  the  earth  appears  larger  to  the  inhabitants  of  the  moon  than 
the  moon  does  to  the  inhabitants  of  the  earth. 

Example. 

The  diameter  of  the  earth  is-  79 17. 5  miles,  and  that  of  the  moon  2164. 2 
miles;  requited  how  much  larger  the  earth  appears  from  the  moon  than  the 
moob  does'from  the  earth  ?  « 

Diameter  of  the  earth  ==  ,  ,  .  79l7. 5  Twice  its.  lQg.=:7. 7971762 
Diameter  of  the  moon  =     .     .     .     2164.  2     Twice  its  log.=6. 6705948 

Number  of  times  the  earth  is  larger  than  the  D  =«  13. 88  Log.»  1 .  1265814 


PkoBLBM  XXII. 

Tojind  the  Rate  at  which  the  Mbon  revolver  round  her  Orbit. 

Note^r^This  is  performed  by  Problem  XVIIL,  page  617  S  as  thus :-« 

Since  the  moon's  mean  distance  from  the  earth  is  236692. 35  miles,  the 
diameter  of  her  orbit  must  be  twice  that  distance,  or  473384.  70  miles  : 
hence  its  circumference  is  1487182  miles.-  And  since  the  moon  goes 
through  this  circuit,  or  orbit,  in  27^7*43r5!,  her  hourly  motion  may  be 
determined  \n  the  Ibllowing  manner ;  viz., 

As  one  luiiation=27'7*43r5!,  in  secs.= 2360585  Log.ar.co.=:3. 6269804  • 
Is  to  the  circumference  of  the  moon's  orbits  1487182    Log.=6. 1723641 
So  is  one  hqur^  in  seconds  =.,.,.    3600       .  I40g.=:3. 5563025  < 

To  the  moon^s  hourly  motion  in  her  orbit  ==  2268  miles  LfOg.s=3. 3556470 


Problbm  XXIIL 

To^nd  ike  mean  Distance  4^  a  Planet  fiom  the  Sun. 

Rule. 

It  has  been  demonstrated,  by  the  celebrated  Kepler,  that  if  two  or*niore 
bodies  move  round  another  body  as  their  common  centre  of  motion,  the 


620  MI'8C£LLAN£0US   l»ROBLBMS« 

squares  of  their  periodical  times  ^ill  be  to  each  other  in  the  same*  propor- 
tion as  thectthfs  of  their  mean  distances  from  the  central  body;  and  hence 
the  following  rule  ;-*- 

As  die  square  of  the  earth's  periodical  or  annual  motion  round  the  sun, 
is  to  the  cube  of  its  mean  distance  from  that  luminary ;  so  is  the  square  of 
any  other  planet's  periodical  revolution  round  tlie  sun,  to  the  cube  of  its 
mean  distance  therefrom ;  the  root  of  which  will  be  the  distance  sought. 

Example. 

Tlie  earth's  periodical  or  annual  motion  round  the  sun  is  completed  in 
365  days,  5  hours,  48  minutes,  ^8  seconds,  and  that  of  Venus  in  224  days, 
16  hours,  49  minutes,  1 1  seconds. «  Now,  if  the  earth's  mean  distance  from 
the  sun  be  94546196  miles,  what  is  Venus'  distance  from  that  luminary  ? 

Earth'3  periodical  revolution 

365f5t48T48!,  in  sees.  31556928  Ar.  cb.  of  twice  its  log.  5.0018106 
Earth's  mean  dist.from  sun,  inmiles,=94546196  Thrice  its  log.23. 9269323 
Venus' per.rev.224M6M9ril!,insecs.l94l4l51  Twice  log.  14. 5762368 

Reject  20  from  the  index;  and,  to  extract  the  root,  divide by3) 23. 5049797 

Venus'  mean  distance  from  sun,  in  miles,  «  68390098  Log.=7. 8349932^ 


PftOBLBM  XXIV. 

To^nd  how  much  more  Heat  and  Light  the  Planets  adjacent  to'the  Sun 
receive  from  that  Jjuminary  than  those  tohich  are  more  remote. 

Rule. 

Since  the  effects  of  heat  and  light  are  redprocally  proportional  to  the 
squares  of  the  distances  from  the  centre  whence  they  are  generated,— 
therefore,  from  twice  the  logarithm  of  the  remote  planet's  distance  from 
the  sun,  subtract  twice  the  logarithm  of  the  adjacent  planet!s  distance 
therefrom ;  and  the  remainder  will  be  the  logarithm  of  the  number  of  times 
that  the  planet  adjacent  to  the  sun  is  hotter  and  more  luminous  than  that 
which  is  more  remote. 

.  Exainple. 

If  the  earth's  distance  from  the  sun  be  94546196  miles,  and  that  of 
Venu9  68390098  miles,  required  how  much  more  heat  and  light  the 
kites  planet  receives  from  the  sun  than  the  former  ?  . 


Digitized  by 


Google 


JJflSCXUJkNBOUS  PftOBLSMS.  621 

Earth's  mean  di^t.  from  8un=;94546196  miles  Twice  its  Iog.= 15. 95 12882 
Venus'  ditto  s68390098  mUes  Twice  its  log.=  15. 8699865 

Heat  and  light  VeHus  receives  more  than  the  earths  1 .  205  Log.  0. 08 1301 7 


Probliim  XXV. 

Given  the  apparent  Diameter  of  a  Planet;  to  find  the  Measure  of  its 

true  Diametei\ 

Find  the  difference  between  the  earth's  and  the  planet's  mean  distances 
from  the  sun,  and  it  will  show  the  planet's  mean  distance  from  the  earth ; 
with  which  and  the  planet's  apparent  semi-diameter,  compute*  her  true 
diameter,  by  Problem  XIX.,  page  617- 

Example*'        .      '        ^ 

Let  the  earth's  distance  from  the  sun4>e  94546196  English  miles,  that 
of  Venus  68390098  such  miles,  and  her  apparent  diameter  58''.  79; 
required  the  true  measure  of  her  diameter,  in  English  miles  ?  ' 

Earth's  distance  from  the  sun  =  94546 196'mile8 
Venus'  ditto  =  68390098  miles 


Venus'  mean  dist.  from  earth  =  26I5609S  miles  Log.  =    .    7*  4175729 

Venus'  apparent  semi-diameter  =  29"^.  395  Log.  ung.=6. 1537885 

'    ■         ■ 

Venus^  true  semi-diameter  =:   3727  miles  Log.  =    .    3. 5713614 

True  measure  of  Venus'  diam.=  7454  English  miles. 

'  IVoftfw— If  the  ratio  of  the  magnitudes  of  the  earth  and  a  planet  be 
required,  it  may  be  determined  by  Problem  XIV.,  page  615;  thus  in  the 
case  of  Venus  :— 

Diameter  of  the  earth  s      .     7917. 5  miles  Thrice  ito  log.s  1 1 .  6957643 
Diameter  of  Venus  =      .    .    7454      miles  Thrice  its  log.=  1 1 .  6171682 


Ratio  of  the  magnitudes  of  th^  earth  and  Venus=l.  198  Log. =0.0785961 

The  velocity  or  rate  at  which  a  planet  moves  round  its  orbit  may  be 
determined  by  Problem  XVIIL,  page  617. 


Digitized  by 


Google 


622  msGBLUiiiBpvs  paoBUOCs, 

PaoBuuc  XXVI. 
Tojinidthe  Tims  thai  the  Sm  iaket  to  titrn  romd  iU  Jms. 

RULB. 

If  the  bright  face  of  the  sttn  be  carefully  observed  through  a  good  tele- 
scope,  large  black  spots  will  be  found  to  make  their  appearance  on  its 
eastern  limb ;  from  this  they  gradually  advance  to  the  middle  of  the  diak, 
and  thence  to  the  western  limb)  where  they  disi^pear.  After  being  absent 
for  nearly  the  same  period  of  time  that  they  were  visible,  they  will  be 
observed  to  appear  again  on  the  eastern' limb  as  at  first;  thus  finishing 
their  career  in  27  days,  12  hours,  and  20  minutes*  Call  this  time  the 
observed  intervaL 

Find  the  number  of  degrees  and  parts  of  a  degree  that  the  earth  has 
moved  eastward  in  the  ecliptic  during  that  interval^  in  the  following  man- 
ner; viz., 

• 

As  the  earth's  annual  motion 

round  the  sun:is36$  f  5 1 48748 ! ,in secs.3 1556928  Log.  ar.  co.2. 5009053 
Is  to  eclip.,  or  great  circle  of  360^,  in  sees.  1 296000  Log,  s  6. 1 1 26050 
Soistheob8.interv.==27fl2^20!,insecs.  2377200  Log.  =s    6.3760657 

Earth's  advance  in  ecHptic  during  obs.  interv.  =  97628r  Log.=  4. 9895760 
Ditto,  in  degrees  and  parts  of  a  degree,  aa     27"? 7 -8? 

Now,  as  360  degrees,  augmented  by  the  earth's  advance  in  the  ecliptic 
during  the  observed  interval,  thua  found,  is  to  the  observed  interval ;  so  is 
the  great  circle  of  360  degrees,  to  the  absolute  time  of  the  sun's  rotatoiy 
motion  on  its  axis ;  thus  :«- 

As  360?  +27?7'8rss387?7'8r,  in  secs.=  1393628  Log.  at.  co.3. 8558532 
Is  to  the  obs.  int.  «  27'  I2t20r,  in  secs.«2377200  Log.  «  6«  3760657 
So  is  the  great  circle  of  360?    in  secs.s  1296000    Log..»  6. 1126050 

To  the  time  of  the  sun's  rotatory  motioh=2210670f  Log.  s  6. 3445239 

Ditto,  in  days  and  parts  of  a  day  =  .  25f  14M*30t ;  which^  therefore, 
is  the  true  time  that  the  sun  takes  to  turn  round  once  upon  its  axis^  as 
required. 


Digitized  by 


Google 


PaoBLRM  XXVIL 

Tofnd  ih$  Length  of  a  Pendulum  far  Vibra^g  Seconds  in  the  Latitude 

of  London.  ; 

RUUB. 

It  has  been  found  by  actual  experiment  that  a  heavy  body  let  fall  in  the 
latitude  of  London^  will  descend,  by  the  force  of  gravity,  16tV  ^^^  ^^  ^ 
second  of  time  ;-^now,  since  the  circumference  of  a  circle,  whose  diameter 
is  unity  or  1,  is  found  by  computation  to  be  3. 14.159265  ;  and  since  the 
pendulum  vibrates  in  the  arc  of  a  cicele,  or  cycloid,  the  radius  of  which  is 
equal  to  the  length  of  the  pendulum  from  the  centre  of  oscillation ;  there^ 
fore  if  twice  the  space  piass^d  through  by  a  falling  body  in  one  second  of 
time,  be  divided  by  the  square  of  the  computed  circumference  of  a  circle, 
as  above,  the  quotient  mU  be  the  length  of  the  pendulum  for  vibrating 
seconds  in  the  parallel  of  London. 

Thus.  16^  feet  =  193  inches  x  2  =  386  inches,  Log.  =  2. 5865878 
Circumf.  of  circle  to  diam.  I  =3. 14159265,  twice  its  Log.=  0. 9942998 


Length  of  thepend.  in  inches  =  89. 1 1  Logarithm  =:1. 5922875 
Note. — By  actual  experiment  the  length  of  a  pendulum  for  vibrating 
seconds  in  London  is  39^  inches,  or  39. 125, 


Problem  XXVIII. 

Tojind  the  Length  qfa  Pendulum  for  vibrating  Sdlf-Seconds. 

Rule.  * 
To  the  arithmetical  complement  of  twice  the  logarithm  of  120  (the 
number  of  vibrations  in  a  minute  for  the  half-seconds'  pendulum),  add 
twice  the  logarithm  of  60  (the  number  of  vibrations  in  a  minute  for  the 
seconds'  pendulum),  and -the  logarithm  of  the  length  of  the  latter  pendu- 
lum :  the  sum  of  these  three  logarithms  (abating  10  in  the  index,}  will  be 
the  logarithm  of  the  length  of  the  pendulum  for  vibrating  half-seconds. 

Example. 
Let  the  length  of  a  pendulum  for  vibrating  seconds  be  39. 1-25  inches  ; 
required  the  length  of  a  pendulum  that  will  vibrate  half- seconds  ? 
Vibrations  for  i-secs.'  pendulum  =  120  Ar.  co.  of  twice  its  log.=5. 841638. 
Ditto  for  the  seconds'  pendulum  =    60  Twice  its  log.  =  .     .3. 556302 
Length  of  the  pendulum  for  sees.  ^  39. 125  inches    Log*  r=     1*  592454 

Length  of  half-seconds'  pendulum  »  9. 781  inches    Log.  ^    0. 990394 
Hence  the  length  of  a  pendulum  for  vibrating  half-seconds^  is  9}  mches. 


Digitized  by 


GooQle 


624  A  COMPBMDIUM' OF  PBACnCAL  NAVIGATION. 

A. COMPENDIUM  OP  PRACTICAL  NAVIGATION;  including  the 
direct  maimev  of  making  out  a  Day*s  fFork  at  sea ;  intended  for  the  use 
of  persons  unacquainted  with  the  elements  of  Geometry  and  Trigonometry. 

Problem  I. 

To  reduce  the  Sun's  DecUnatioriy  as  given  in  the  Nautical  Jlmanac,  to  the 
Thne  of  apparent  Nom  under  any  known  Meridian. 

Rule. 

From  page  II.  of  the  month  in  the  Nautical  Almanac  take  out  the  son's 
declination  for  noon  of  die  given  day,  and  note  whether  it  is  increasing  or 
decreasing ;  and,  at  the  same  time,  take  out  ^the  variation  of  the  aun's  de- 
clination between  the  noons  of  the  given  and  preceding  days  if  the  longi- 
tude be  east,  but  between  those  of  the  given  and  following  days  if  the  lon- 
gitude be  west.  Then,  with  this  variation,  or  difference  of  declination, 
enter  Table  XV,  at  top,  and  the  longitude  of  the  given  meridian  in  the 
right  hand  column  i — in  the  angle  of  meeting  will  be  found  a  correction, 
which  being  applied  to  the  declination,  taken  from  the  Nautical  Almanac, 
agreeably  to  the  directions  expressed  at  the  bottom  of  the  Table^  will  give 
the  sun's  correct  declination  at  the  noon  of  the  given  place.  *   • 

Note. — ^When  the  longitude  of  the  given  meridian,  and  the  variation  of 
declination  cannot  be  exactly  found  in  the.  Table  ;  then,  the  sum  of  the 
proportional  parts,  corresponding  to  the  several  terms  which  make  up  the 
whole  longitude  and  the  whole  variation,  will  be  the  correction  of  declina- 
tion required. 

Ej^ample  I. 

Required  the  sun's  declination  at  noon,  August  lOth.,  1825%  in  longi- 
tude 100?30C  East  ? 

Variation  of  declination  between  given  And  preceding  noons 
•  (the  longitude  being  east)  is  17-26^ 

Sun's  declination  at  noon  of  the  given  day  per  Nau- 
tical Almanac   (decreasing)  :=:       .     .    .     /  .     .  15?36'30TN. 

Pro.  pt.  to  Ion.  90?  0^  and  var.  17^  Or  =  4^15^  OT 

Ditto    .    .    .  90..  0  ditto         0.26    =0.   6.30 

Ditto    .    .    .  10.30  ditto        IJ.   0   ^0.29.45 

Ditto    •    .    .  10.30  ditto         0.26   n  6.  0.45.30 


Correction  of  dec.  additive  =  .    .    •     .    4152r  OTSOV   +     4C52r 


Sun's  reduced,  or  corrected  declination  = ;15'^4l!22rN. 

*  It  is  the  nautical  or  tea  day  that  is  made  use  of  in  this  and  the  foUowin;  Examples  :— 
this  day,  like  the  civil,  hegios  at  midnifht,  and  ends  at  the  foUowini;  miiifaiii^ht:— Itis 
divided  into  two  parts,  of  12  hours' each ;  the  first  pitrt  or  that  contained  between  nidai^t 
and  noon  is  called  A.M.  or  ante  taieridiemy  and  the  other  part,  or  that  between  noon  aod 
midaiffhty  P.M.  or  post  meridiem. 


Digitized  by 


Google 


A   COMPBNBIUll  OF  PRACTICAL   NAVIGATION.  625 

JSxample  2. 

Required  the  sun's  declination  at  noon,  April  3d.,  1825  *,  in  longitude 

75°45^  West? 
Variation  of  declination  between  given  and  foUcnving  noons 

(the  longitude  being  west)  is  22'54f 

Sun's  declination  at  noon  of  the  given  day,  per  Nau- 
tical Almanac,  (increamg)  = S?18U0C' N. 

Pro.pt.  tolon.  75?  OC  andvar.  22:  0r  =  4^35r  (T, 

Ditto  0. 45    ditto       22.   0    =  0.    2. 45 

Ditto  75.   0    ditto        0.54    =0.11.15 

Ditto  0.45    ditto        0.54    =0.   0.   6.45 


Correction  of  declination,  additive  zr      .    4:49r  6r45V  +     4M9r 


Sun'a  reduced,  or  corrected  declination  =:   .     .    .         .    .  5?23:29f  N, 


Problem  II. 

Given  the  Sun's  Meridian  Altitude,  tojini  the  Latitude  of  the  Place  of 

Observation* 

RULB. 

Reduce  the  sun's  declination  to  the  meridian  of  the  given  place  by  the 
preceding  Problem. 

Then,  to  the  observed  altitude  of  the  sun's  lower  limb  add  the  difference 
between  its  semi-diameter  (page  III.  of  the  month  in  the  Nautical  Alma- 
nac,) and  the  dip  of  the  horizon,  (Table  II.)  and  the  sum  will  be  the  appa- 
rent altitude  of  the  sun's  centre  ;  .from  which,  let  the  difference  between 
the  parallax  and  refraction  answering  thereto  (Tables  VII.  and  VIII.)  be 
subtracted,  and  the  remainder  will  be  the  sun's  true  central  altitude ;  which 
being  taken  from  90  degrees  will  leave  the  sun's  meridional  zenith  distance 
of  a  contrary  denomination  to  that  of  its  observed  altitude.     Now, 

If  the  sun's  meridian  zenith  distance  and  its  reduced  declination  are 
both  north,  or  both  south,  their  sum  will  be  the  latitude  of  the  place  of 
observation :  but  if  one  be  north  and  the  other  south,  their  difference  will 
be  the  latitude,  and  always  of  the  same  name  with  tiie  greater  term. 

JExample  I. 

April  10th,  1825,  in  longitude  75?  west,  the  meridian  altitude  of  tlie 
8un*s  lower  limb  wa»  57?40'30^  south,  and  the  height  of  the  observer's 


See  Note,  page  624. 
2  S 


Digitized  by 


Google 


626  A  COMPENDIUM  OF   PRACTICAL  NAVIGATION. 

eye  above  the  level  of  the  sea  22  feet ;  required  the  latitude  of  the  place 
of  observation  i 

Variation  of  the  sun's   declination  between  the  given   and  ft^lowing 

noons,  (the  longitude  being  west,)  is  22'6r . 

Sun's  declination  at  noon  of  the  given  day  per  Nau- 
tical Almanac  (increcwin^)  =:      7?56U2^  N. 

Propl.parttolong.75?0^  andvar.  22^  0^=4^35^  OT 

Ditto    .    .    .     .75.0    ditto        0.   6  =0.    1.15 

Correction  of  declination,  additive  =     •    .   4^86^15^  +  4^36r 


Sun's  reduced,  or  corrected  declination  =    .     .     .     .     8?   I'lS'T  N. 

Observed  altitude  ofthesun's  lower  limb  =       .    .     .  57*?40'SO?  S. 
8nn*8semi.diameter=:     •     *  *^^59r-i     ^^^^^^    ^^^       jj,29r 
Dip  of  the  hor.  for  22  feet  =    4. 30   J 


Dip 

Apparent  altitude  of  the  sun's  centre  = 
Parallax  0^5?  refrac.  0^35 r,  difF.  =  0'.30 

Sim'fl  true  central  altitude  =.•••• 

Sun's  meridional  zenith  distance  =:  •     • 
Sun'^s  reduced  declination  =:    «    •     •    • 

Latitude  of  the  place  of  observation  =   . 


,    .    .    57^51C59r  S. 
subtractive  =:     0'30^ 


57^5r.29f  S. 

32?  8^3ir  N. 
8.    1.18    N. 


40?  9<49r  N. 


Example  2« 

September  21st.,  1825,  in  longitude  60?  cast,  the  meridian  altitude  of 
the  sun's  lower  limb  was  56?26^  north,  and  the  height  of  the  obaerver's 
eye  above  the  level  of  the  sea  26  feet ;  required  the  latitude  of  the  place  of 
observation  ? 

Variation  of  the  sun's  declination  between  the  given  and  preceding 
noons,  the  longitude  being  east,  is  2^^.22^. 
Sun's  declination  at  noon  of  the  given  day  per  Nau' 

tied  Almanac  (decreflwing)  = 0?43C34r  N. 

*  Propl.pirttolong.60?0:«advar,23'.  0r=3^50T  Cf 
Ditto    .    •    .    .    60.0    ditto         0.22  =0.   3.40 


Correction.of declination, additive  =     .    .    3'53r407  +  3:54* 


Sun's  reduced  or  corrected  declination  =     •    .    .     .    0?47'28.  N. 


Digitized  by 


Google 


A  COMPtiIlt>iUM   OF  PRACTICAL  MAVIQATION.  627 

Observed  altitude  of  the  sun's  lower  limb  =    .     •     .    56? 26^  0?  N. 
Sun's  semi-diameter  15  '58^  dip  of  the  horizon  for  26 

feet  =  4'52r  difiference  r= +   11?  6lf 


Apparent  altitude  of  the  sun's  centre  =       ....     56?37'  6f  N. 
Parallax  O'.SI  refrac.  0:37^  diff.  =  0'32r  subtractive  =s      0^327 


Sun's  true  central  altitude  =:  .    . 

Sun's  meridional  zenith  distance  =: 
Sun's  reduced  declination  = 


Latitude  of  the  place  of  observation  = 


66?36:34f  N. 

88^33^26?  8. 
•  0.47.28  N. 


•  •  • 


32?35:58f  S. 


pROBLBBf  III. 

Given  the  difference  of  Longitude  between  two  Places^  both  under  the 
same  Parallel  of  Latitude,  tojind  their  Distance^ 

RCLB. 

To  the  logarithmic  co-sine  of  the  latitude,  add  the  logarithm  of  the  diff- 
erence of  iQngitude,  in  miles ;  and  the  sum,  abating  10  in  the  index,  will  be 
the  logarithm  of  the  distance. 

Example. 

Required  the  distance  between  Portsmouth,  in  longitude  1?6'  wes{,^and 
Green  Island,  Newfoundland,  in  longitude  55  ?35^  west,  their  common 
latitude  being  50?47'  north  ? 

Long,  of  Portsmouth  =s  .    .    •    *     1?  6i  W. 
Long,  of  Green  Island  =     .    •     •     55.35    W. 


Difference  of  Longitude  =      .     .    54?29^  ==  3269  ms.  Log.  3. 514415 
Latitude  of  the  parallel  =  50? 47?  N.    Log.  co-sine  tz  .    .  9. 800892 

Distance, in  miles  =      •    -    •    2(966.8  Log.-   .    .3.315307 


PaoBUSM  IV. 

*  Gitrm  the  Distance  between  two  Places,  both  under  the  same  Parallel  of 
Latitude,  tojind  their  Difference  ofLomgitude. 

Rui^. 
To  the  logarithmic  secant  of  the  latitude,  add  the  logarithm  of  the  dis-  * 
tance,  and  the  sum,  abating  10  in  the  index,  will  be  the  logarithm  of  the 
difference  of  longitude* 

282 


Digitized  by 


Google 


628  A   COMFBMBIUM   OF   PRACTICAL  NAVIGATION. 

Example. 

A  ship  from  Cape  Gear,  in  latitude  51?25^  north,  and  longitude  9?29! 
west,  sailed  due  west  1040  miles ;  required  the  longitude  at  which  «he 
then  arrived  ? 

Lut.  of  the  parallel  =s  .    .  5 1  ?25 '  Log.  secant  =  ...     10. 205037 
Distance  sailed  =  ,.    .    •     1040    miles,  Liog.  =  •     *     .      3.017033 


Difference  of  long.  =  .    .  27?48^  W.=:  1667. 6  mUes,  Log.  3. 222090 
Longitude  sailed  from  =       9. 29    W. 

Longitude  arrived  at  =        37?  17'  W. 

No/e.— The  ahove  two  Problems  are  essentially  Useful  when  a  ship  sails 
upon  a  parallel  of  latitude ;  that  is,  when  she  steers  either  due  east,  or  due 
west. 


PROBJ.BM  V. 

Given  the  LatUudei  and  Longiludes  of  two  Places,  to  find  the  Omne 

and  Distance. 

RULK. 

From  the  logarithm  of  the  difference  of  longitude,  the  index  being  aug- 
mented by  10,  subtract  the  logarithm  of  tlie  meridional  difference  of  lati- 
tude ;  the  remainder  will  be  the  logarithmic  tangent  of  the  course  : — then, 
to  the  logarithmic  secant  of  the  course,  thus  found,  add  the  logarithm  of 
the  difference  of  latitude,  and  the  sum,  abating  10  in  the  index,  will  be  the 
logarithm  of  the  distance. 

£rampZe. 

Required  the  course  and  distance  between  CapeBajdli,  in  latitude  40?3' 
north,  longitude  3?52r  east,  and  Cape  Sicie,  in  latitude  43?2'  north,  and 
longitude  5?58'  east  ? 

Lat.  of  C.  Bajoli  40?  Z'.  N.    Merid.  pts.  2626. 6,    Longitude  3. 52  E. 
Lat. of  C.  Sicie  =43.   2   N.    Merid.  pts.  2865. 8,     Longitude  5. 58  E. 

Diff.  of  latitude      2?59^  Merid.  diff  lat.    239: 2.    Diff.  long.  2?  6^ 


=  179  miles.  =  126  miles. 

To  find  the  Course  :— 
Difference  longitude  126  miles,    .     .    Logarithm  =  2. 100371 
Merid.  difference  of  latitude  239  miles    Logarithm  =  2. 378398 


Course  N.  27^47 :53rE.=    ...    Log,  tang,  =  9.721973 

/Google 


Digitized  by ' 


A   COMPENDIUM   OF  PRACTICAL   NAVIGATION.  629 

To  find  the  Distance  : — 

Course  27  M7^  53 r     ....     Log.  secant  =.     10.053254 
Difference  latitude  =  179  miles.  Logarithm  =     .       2. 252853 


Distance  in  miles  =  202. 3  •     .     Logarithm  =  .     .    2.306107 

Hence  the  true  course  is  N.  27^47 '•53r  R,  or  N.  N,  E;i  E.  nearly,  and 
the  distance  202^  miles. 


Problem  VL 

Given  the  Latitude  and  Longitude  of  the  Place  sailed  fiom,  with  the 
Course  and  Distance  ;  to  find  the  Jjatitude  and  Longitude  of  the  Place 
come  to. 

Rule. 

To  the  logarithmic  co-sine  of  the  course,  add  tlie  logarithm  of  the  dis- 
tance ;  the  sum,  abating  10  in  the  index,  will  be  the  logarithm  of  the  differ- 
ence of  latitude  5  which  being  applied  to  the  latitude  left  by  addition  or 
subtraction,  according  as  the  latter  is  increasing  or  decreasing,  the  sum,  or 
difference  will  be  the  latitude  come  to.  Now,  to  the  logarithmic  tangent 
of  the  course,  add  the  logarithm  of  the  meridional  difference  of  latitude; 
the  sum,  abating  10  in  the  index,  will  be  the  logarithm  of  the  difference  of 
longitude  ;  which  being  applied  by  addition  or  subtraction  to  the  longitude 
left,  according  as  the  latter  is  increasing  or  decreasing,  the  sum  or  differ- 
ence will  be  the  longitude  come  to. 

Example  1.     . 

A  ship  from  Cape  Ortegal,  in  latitude  43947'  N.  and  longitude  7?49^ 
W.,  sailed  N.  W.  i  N.  560  miles;  required  the  latitude  and  longitude  of 
the  place  come  to  ? 

To  find  the  Difference  of  Latitude  :— 

Course  steered  =:  31  points  .    .    .     .     •    Log.  co-sine  =  9. 888185 
Distance  sailed  560  miles Logarithm  =    2. 748188 

Difference  of  latitude  432. 8  miles      .    .    JiOgarithm  =    2. 636378 

« 

To  find  the  Latitude  come  to  :— 

Latitude  of  Cape  Ortegal  43^47'  N.  Meridional  parts  .     .  2927.8 
Diff.  of  lat.  432. 8  N.  =       7. 13    N. 

Latitude  come  to  .     .     .51?  0'  N.  Meridional  parts       .    35B8. 8 

*  _ 

Meridional  difference  of  latitude  =       . .641.0 


Digitized  by 


Google 


6ao 


A  COMPBNDIUM  Of  PRACTICAL  MAVIOATION, 


To  find  the  Difference  of  Longitude,  and  hence  the  Longitude  come  to  :— 

Course  steered  = 3|  points.     Log,  tang.  •     .    9.914173 

Meridional  difference  of  lat,  =:   ,  641  miles.    Logarithm  •     •     2.  806S58 

Diflferencc  of  long.  =  8?46'  W.=  526  miles.    Logarithm  .     .    2. 721031 
Long.of  C.  Ortegal  =  7. 49  W. 

Long,  come  to  sx  .    16?35^  W. 

Remarks. — ^When  a  ship  decreases  her  latitude  ;  that  is,  when  the  diflFer- 
ence  of  latitude  made  good  is  pf  a  different  name  to  the  latitude  sailed 
from  ;  then,  if  the  difference  of  latitude,  expressed  in  degrees,  be  greater 
than  the  latitude  left,  their  difference  will  be  the  latitude  come  to ;  which 
will  be  of  a  contrary  denomination  to  that  sailed  from ;  beeaute,  in  this 
case  it  is  evident  that  the  ship  must  have  crossed  the  Equator. 

And,  when  a  ship  decreases  her  longitude ;  that  is,  when  the  difference 
of  longitude  made  good  is  of  a  contrary  name  to  the  longitude  sailed  from  ; 
then,  if  the  difference  of  longitude^  expressed  in  degrees,  be  greater  than 
the  longitude  left,  their  difference  will  be  the  longitude  eome  to ;  which 
will  be  of  a  eontrary  name  to  that  sailed  from ;  because^  in  this  case  the  ship 
will  have  crossed  the  meridian  whence  the  longitude  is  reckoned. 

Agaln.-^When  a  ship  increases  her  longitude ;  that  is,  when  the  differ^ 
ence  of  longitude  made  good,  expressed  in  degrees,  is  of  the  same  name 
with  the  longitude  sailed  from,  their  sum  will  be  the  longitude  .come  ta  ; 
but,  if  this  sum  exceeds  180  degrees,  then,  its  differance  to  360  degrees  wUl 
express  the  longitude  cojne  to,  which  will  be  of  a  contrary  denomioalioii  to 
that  sailed  from ;  for,  in  this  case,  also,  the  ship  will  have  crossed  the  me- 
ridian that  the  longitude  was  reckoned  fi-oni :— see  Problems,  Rules,  and 
Remarks,  between  pi^s  211  and  217*   . 

Example  2. 

A  ship  from  the  Island  of  Annabona,  in  latitude  l?23f  S.,  and  longitufle 
5?34^'%.,  sailed  W.N,  W.  546  miies;  i^wed  the  latitude  and  longitude 
of  the  place  at  which  she  arrived  ? 

To  find  the  Difference  of  Latitude ; — 

Course  steered  =:  6  points     .  •     .     .    L(^.  co-sine  .     .    9. 582840 

Distance  sailed  S4Q  mka   ....    Logarithm     «    .    2.737198 


Difference  of  latitude  208, 9  miles  zz    Logarithm     . 

T(>  find  the  Li^titude  come  to  :^^ 
Latitude  sailed  from  =  .     .     1?23'  S.     Merid.  piMfts 
Piff.tet.  =    208. 9  miles  =  3.29    N. 


2.320033 


83,0 


Latitude  come  ta  =      .    .    2?  6;  N.    Merid.  pvts=   •    .126.0 
Meridional  difference  of  latitude  =      ..,..,    ^    ,    209.0 

Digitized  by  VjOOQ IC 


A   COMPENDIUM   Of  PRACTICAL  NAVIGATION.  631 

To  find  the  Difference  of  Longttude^  and  hence  the  Longitude  come  to:-— 

Course  steered  =     .     •    •    .    6  points.    Log.  t»ng.  =  10.  S82776 
Meridional  diff.  of  lat.  =  .    .209  miles.      Logarithm  =    2. 320146 


Diff.  of  long.  =    .    8?  25  C  W.  =  504. 6  miles      Log.  =  2. 702922 
Long,  sailed  JErom      5.34     E. 


Long,  come  to  =  .   2?5l<  W» 
Hence^  the  latitude  come  to  is  2?6^  N.  and  the  longitude  2?5I  C  W. 

Example  3. 

A  ship  from  Pitt's  Island,  in  latitude  2?541  N.  and  longitude  174?30C 
E.  sailed  S.  £.  by  E.  ^  E.  760  miles ;  required  the  latitude  and  longitude  of 
the  place  at  which  she  arrived? 

To  find  the  Difference  of  Latitude  : — 

Course  steered  =:  .    •    5^  points.     Log.  co-sine  :=:  •    ,    9*  673387 
Distance  sailed  760  miles.  Logarithm  =     ,    ,    2*  880814 

Diff.  of  lat.  =  358. 4  miles.  Logarithm    ...    2. 5S4201 

To  find  the  Latitude  come  to  :-— 

Latitude  sailed  from  =  «    .    2?54C  N.    Mend,  parts  =:    •     174. ) 
Diff.  of  lat.  358. 4  miles  =     5. 58   S. 


Lat.  come  to  =    .    •    •    •    3?  4^  S*    Merid.  parts  x     •     184.1 
Meridional  difference  of  latitude  = 858. 2 

To  find  the  Difference  of  Longitude,  and  hence  the  Longitude  come  to  :-— 

Course  steered  =:  «     .     .     5|  points.     Log.  tangent  =    10.  272043 . 
Meridional  diff.  of  lat.  =  358. 2  miles.    Logarithm  :=      .  2. 554126 


Diff.  of  long,  made  good  =  II ?  10^  E.  =  670  miles  Log.  2. 826169 
Longitu4e  sailed  from  :::    174.30    & 

Sum  =  185?40'.  E. 


Longitude  come  to  =    .     174°20'.  W. 

Hence,  the  latitude  come  to  is  3 ?4^  S.,  and  the  longitude  174?20C 


west. 


Digitized  by 


Google 


632 


A   COMFENBiUM  OF  PRACTICAL  KAVIGATION. 


Problem  VIL 

Given  both  Latitudei  and  the  Course;  to  fnd  the  Dutance  Sailed  and 
the  Longitude  come  to. 

RULB. 

To  the  logarithmic  secant  of  the  course,  add  the  logarithm  of  the  differ- 
ence of  latitude  ;  the  sum,  abating  10  in  the  index,  will  be  the  logarithm 
of  the  distance.— Then, 

To  the  logarithmic  tangent  of  the  course,  add  the  logarithm  of  the  me- 
ridional difference  of  latitude;  the  sum,  abating  .10  in  the  index,,  will  be 
the  logarithm  of  the  difference  of  longitude ;  which  being  applied  to  the 
longitude  left  by  addition  or  subteaction,  according  as  it  is  increasing  or 
decreasing,  the  sum  or  difference  will  be  the  longitude  come  to. 

Example. 

A  ship,  from  a  place  in  latitude  3?4'  S.,  and  longitude  174?20t  W^ 
esuled  N.  W,  by  W.  ^  W.  until  she  was  found,  by  observation,  to  beiu 
latitude  2?54C  N.;  required  the  distance  sailed,  and  the  longitude  at  which 
the  ship  arrived  ? 


Lat-  sailed  from  iz  3?  4'.  S. 
Lat.  come  to  =  .    2. 54    N. 


Mer.  parts  =   .    184. 1  miles. 
Mer.  parts  ==  .    174.  i  miles. 


Diff.  oflat.  =  . 


Course  =  . 
Diff.  of  lat. 


.  5?58<=  358  ms.    Mer.  diff.  lat.  =  358. 2  miles. 

To  find  the  Distance  Sailed : — 
.     5|  points.    Log.  secant  =:    • 


.    .    358  miles.    Logarithm    =   .    . 
Distance  sailed  =:  759. 4  miles.    Logarithm    =   .    . 

To  find  the  Difference  of  Longitude  :— 


Course  =  .    .    . 
Merid.  diff.  lat  . 

Diff.  of  long.  =  • 


51  points.     Log.  tang.  = 
358. 2         Logarithm  = 

670. 1  ras.  Logarithm  r= 


10.326613 
2.553883 

2.880496 


10. 272043 
2.554126 

2.826169 


Long,  sailed  from  =     •.••••• 
Difference  of  long,  made  good  670  miles  = 


174?20^  W. 
11.10    W. 


Sum  =  . 


185?30^  W. 


Longitude  come  to  zz 


174?30t  E. 


Digitized  by 


Google 


A   COMPENDIUM   OF  PRACTICAL  NAVIGATION.  633 

Note.— The  three  last  Problems  comprehend  all  the  cases  that  usually 
occur  in  the  practical  part  of  Mercator*8  sailing ; — for  the  speculative 
cases^  see  pages  from  236  to  248,  inclusive. 


Problbm  VIIL  . 

To  find  the  Course^  Disiancey  Difference  of  Latitudej  and  Difference  of 
Longitude  made  good  upon  compound  Courses,  and  also  the  Bearing 
and  Distance  from  a  Ship  to  the  Place  to  which  she  is  ^^ound,  viz  : — 
To  make  out  a  Day's  Work  at  Sea, 

Rule. 

Make  a  Table  of  any  convenient  size,  and  divide  it  into  six  columns  :-* 
in  the  first  of  these  place  the  several  courses,  taken  from  the  log  board 
(corrected  for  lee-way,  if  any,  and  also  for  variation),  and  in  the  second 
place  their  corresponding  distances.— The  third  and  fourth  columns  are  to 
contain  the  differences  of  latitude,  and,  therefore,  to*  be  marked  N.  S.  at 
top  5  and  the  fifth  and  sixth  the  departures,  or  meridian  distances,  which 
are  to  be  marked  at  top,  also,  with  the  letters  B.  W. — Now, 

Enter  the  general  Traverse  Table,  and  take  out  the  difference  of  latitude 
and  departure  answering  to  each  corrected  course  and  distance,  and  place 
them  in  their  respective  columns  :— then,  the  difference  between  the  sums 
of  the  N.  and  S.  columns  will  be  the  whole  difference  of  latitude  made 
good,  of  the  same  name  with  the  greater ;  and  the  difference  between  the 
sums  of  the  E.  and  W.  columns  will  be  the  whole  departure  made  good,  of 
the  same  name  with  the  grieater  term.    ' 

Remark. — ^The  courses,  taken  from  the  log  board,  are  to  be  corrected 
for  variation,  and  lee-way,  if  atiy,  in  the  following  manner,  viz. 

If  the  variation  be  easterly,  it  is  to  be  allowed  to  the  right  hand  of 
the  course  steered  by  compass ;  but  to  the  left  hand  if  it  be  westerly: — 
And, 

If  the  larboard  tacks  be  aboard^  the  lee-way  is  to  be-  allowed  to  the 
right  hand  of  the  course  steered  by  compass }  but,  to  the  left  hand  if  the 
starboard  tacks  be  aboard. 

To  find  the  Course  and  Distance  made  good  :-— 

From  the  logarithm  of  the  departure,  the  index  being  increased  by  10/ 
subtract  the  logarithm  of  the  difference  of  latitude }  the  remainder  will  be 
the  logarithmic  tangent  of  thie  course. — Then, 


Digitized  by 


GooQle 


634  A  COMPBNDIUU  Of  PRACTICAL  NAVIGATION, 

To  the  logarithmic  secant  of  the  course^  thus  found,  add  the  logarithm 
of  the  difference  of  latitude,  and  the  eum,  abating  10  in  the  indexj  will  be 
the  logarithm  of  the  distance. 

To  find  the  Latitude  in,  by  Account,  or  Dead  Reckoning: — 

If  the  difference  of  latitude,  and  the  latitude  of  the  place  from  which 
the  ship's  departure  was  taken,  or  the  yesterday *s  latitude,  be  of  the  same 
name  ;  their  sum  will  be  the  latitude  in,  by  account :  but  if  of  contrary 
names,  their  difference  will  be  the  latitude  in,  of  the  same  name  with  the 
greater  term. 

To  find  the  Difference  of  Longitude  ;  and  thence  the  Longitude 

come  to : 

To  the  logarithmic  tangent  of  the  course  made  good,  add  the  logarithm  of 
the  meridional  difference  of  latitude  (by  observation)  3  the  sum,  abating  10 
in  the  index,  will  be  the  logarithm  of  the  difference  of  longitude. — ^Now, 
if  the  difference  of  longitude,  and  the  longitude  of  the  place  from  which 
the  ship's  departure  was  taken,  -or  the  yesterday's  longitude  be  of  the  same 
name ;  their  sum  will  be  the  longitude  in,  by  account,  when  it  does  not 
exceed  180  degrees ;  otherwise  it  is  to  be  taken  from  360  degrees,  and  the 
remainder  will  be  the  longitude  in,  of  a  contrary  name  to  that  left : — but,  if 
the  difference  of  longitude,  and  the  longitude  left  be  of  contrary  names, 
their  difference  will  be  the  longitude  come  to,  of  the  same  native  with  the 
greater  term. 

To  find  the  Bearing  and  Distance  of  the  Ship  to  the  Port,  or  Place  to 
which  she  isi  Bound  :^ 

From  the  logarithm  of  the  difference  of  bngitude  between  the  ship  and 
the  place  to  which  she  is  bound,  the'  index  being  increased  by  10,  subtract 
the  logarithm  of  the  meridional  difference  of  latitude ;  the  remainder  will 
be  the  logarithmic  tangent  of  the  course.  Then,«-To  the  logarithmic  secant 
of  the  course,  thus  found,  add  the  logarithm  of  the  difference  of  latitude,  and 
the  sum,  rejecting  radius,  will  be  the  logarithm  of  the  distance. 

Note. — ^l^he  true  bearings  or  course  thus  found,  may  be  reduced  to  the 
magnetic,  or  compass  course,  if  necessary,  by  allowing  the  value  of  the 
variation  to  the  right  hand  thereof  if  it  be  westeriy  j  but»  to  the  left  hand, 
if  easterly : — this  being  the  converse  of  reducing  the  course  steered  by  com- 
pass to  the  true  course. 

And  this  rule  comprises  the  substance  of  that  nautical  operation,  which 
is  generally  termed  making  oat  a  day'a  work  at  sea. 


Digitized  by 


Google 


Example  1, 

A  ship  from  Cape  Espichell,  in  latitude  38925^  N.  and  longitude  9?  13' 
W.  bound  for  Porto  Santo,  in  ktitude  33?3^  N,  and  longitude  16?  17  •  W., 
by  i'eason  of  contrary  winds  was-  obliged  to  sail  upon  the  following  courses : 
viz.,  (with  the  larboard  tacks  aboard,)  W.  by  S.  56  miles  j  N.  W,  by  W, 
110  miles;  W.  N.  W.  95  miles;  (and  then  with  the  starboard  tacks 
aboard,)  S.  by  E.  i  K  50  miles  ;  S.  by  W.  i  W.  103  miles ;  and  S.  S.  W. 
116  miles,  when  she  was  found-  by  observation  to  be  in  latitude  34?  1 7  '*  N , 
and  longitude  13?42  •  W. ;  the  lee- way  on  each  of  the  courses  was  about 
half  a  pcHnt ;  the  variation  was  two  points  westerly  on  the  three  first 
courses,  and  If  point  on  the  three  last ;  required  the  true  course  and  dis« 
tance  made  good;  the  latitude  and  longitude  at  which  the  ship  arrived  by 
account ;  and  the  direct  course  and  distance  between  her  true  place,  by 
observation,  and  the  port  to  which  she  is  bound  ? 


TRA.VEKSK  TaBLB. 

Corrected 
Cottnee. 

Dis- 
tances, 

.  Difference  of  Latitude. 

Departure. 

N. 

S. 

E. 

W. 

S.W.b.W,4W. 
W.bvN.iN. 

• 

86 

110 

95 

50 

103 

116 

31.9 
9.3 

26.4 

37.0 
102,5. 
115.9 

33.6 

10.1 

5.7 

49.4 

105.8 

94.5 

41.2 
Diff.  lat. 

281,8 
41.2 

49.4 
Departure. 

249.2 
49.4 

240.6 

199.8 

To  find  Iho  Cdurse  made  good  i^ 

DqMrturd  =  .  .  .    .     199«  8  miles.    «    .    Logarithm  =  2. 300596 
Difference  of  lat.  =       240. 6  miles.     .     .     Logarithm  =  2.381296 

Course  made  ghoi  S.  39?42:25^'  W.  a    .    Log.  tang.  =  9. 919300 

"to  find  the  Distance  made  good  : — 

Course  made  good  »  S.  39?42^25?  W.  .    Log.  see.  s:  10. 1 13892 
.Difference  of  lat.  =      240. 6  miles.     .    .     Logarithm  =:  2.381296 


Distance  made  good  an  312«  7  miles. 


Logarithms  2.495188 


636  A   COMPENDIUM  OF   PJtACriCAL   NAVIGATION. 

Hence,  the  course  made  good  is  S.  39?42'.25r  W.  orS.  W.  i  S.  nearly, 
and  the  distance  313  miles  nearly. 

To  find  the  Latitude  and  Longitude  come  to  by  Account,  or  Dead 

Reckoning : — 

Latitude  sailed  from    38^25'  N 38?25^  N.M.pts.— 2500. 1  ms. 

Diff.  oflat.  made 
gocfJi  =  240. 6  ms.=4?  1  ^  S.     . 

Lat.cometobyacc.=s34?24^  N.Byob.=  34?17<  N.Nf.pts.=  2192.0  ms. 

Meridional  difference  of  latitude  by  observation  =  •     •    •    .    308. 1  ms. 

Meridional  difference  of  lat.  =      308. 1  miles.    Logarithm  =  2. 488692 
Course  made  good  =    .    .    S.  39M2'25r  W.    Log.  tang.  =i  9. 919300 


Diff.  of  long,  made  good  =    4?  16'  W.  s  255. 8  ms.  Log.  s    2. 407992 
Longitude  sailed  from  =        9. 13    W. 

Long,  come  tobyacct.  =    13?  29'  W.  .  . 

To  find  the  Course  and  Distance  from  the  Ship  to  her  intended  Port : — 

Lat. of  ship  by  ob.=34?17'  N.  Mer.  pts.  =  2192.0  Long.  =13?42:  W. 
Lat- of  Porto  Santo=33.    3    N.  Mer.  pU.  =  2103. 1  Long.  =16. 17    W. 

Diff.  oflat.  =    .    .    1?14'         Mer.  diff.  lat.  88.9  Diff.  long.  2?35^ 

=  74  miles.  =  155  miles. 

Difference  of  longitude  =    155  miles.    Logarithm  =  .     .    .    2.190332 
Mer.  diff.  latitude  =     .     88,9  miles.     Logarithms.     •     .     1.948902 


Courses    .    .    .    S.  60?9M9':' W.    Log.Ung.      .    .    .*    10.241430 

Course  =  .    .    .    .     S.  60?9M9r  W.  Log.  secant  =    •    .     10.303185 
Diff^srence  of  latitude  74  miles.     .     .    Logarithm  s     .    .       1.869232 

Distances    .    .    •     148. 7  miles.        Logarithms     .    .      2.  I724I7 

Hence,— ITie  course  made  good  is  S.  39?42'  25r  W.  or  &;.  W.  J  S.  nearly. 

Distance  made  good  s 313  miles. 

Latitude  come  to  by  account  =;     '•    •    .    34?24^  N. 

Latitude  by  observation  s 34?17- N. 

Longitude  come  to  by  account  s  .    ...     13?  29^  W. 
Longitude  by  observation  =     .     .     .     .     13?42'  W. 
Porto  Santo  bears  S.  60?9M9l'  W.  or  S.  W,  by  W.  J  W.  nearly. 
Distant.    .    .    .     149  miles. 


Digitized  by 


Google 


A    VQtAnfiViVM    %}¥    I'KACTIUAL.    FIAVIQATION* 


uo/ 


Note.'^lf  the  variation  be  one  point  and  three-quarters  west,  the  ship 
roust  steer  W.  b.  S.,  by  compass. 

Example  2. 

A  ship  from  Port  Royal,  Jamaica,  in  latitude  17^58^  N.,  and  longitude 
76^53'.  W.,  got  under  weigh  for  HayU,  St  Domingo,  in  latitude  18?30?N., 
and  longitude  69?49'W.,  and  sailed  upon  the  following  courses;  viz.,— 
S,  40  miles,  8.E.b.S.  97  miles,  N.b:E.  72  miles,  S.E.iS.  108  miles, 
N.b.E.|E.  114  miles,  S.E.  126  miles,  N.N.E.  86  miles;  and  then  by 
observation  was  found  to  be  in  latitude  16?55'N.,  and  longitude  72?30C 
W. ;  the  lee-way  on  each  of  those  courses  was  a  quarter  of  a  point  (the 
wind  being  between  S.E.  b.  E.  |  E.  and  E.  b.  N.  i  N.),  and  the  variation  of 
the  compass  half  a  point  easterly ;  required  the  true  course  and  distance 
made  good,  the  latitude  and  longitude  at  which  the  ship  arrived  by  account, 
with  the  direct  course  and  distance  between  her  true  place  by  observation 
and  the  port  to  which  she  is  bound  I 


Traverse  Table. 

Corrected 
Courses. 

Distances. 

Difference  of  Latitude. 

Departure. 

N.. 

s. 

E. 

w, 

S.iW. 

btSaE^XE. 

N.b.E.|E. 

s.s.e.|e. 

N.b.E.|E. 

S.E.iS.. 

N.N.E.iE. 

40 

97 

72 

108 

114 

126 

86 

69.8 
107.3 

■     77.7 

39.6 

87.7 

92.6 
101.2 

•  ■ 

41.5 
17.5 
55.5 
38.4 
75.1 
36.8 

5.9 

254.8 
Diff.ofLat. 

321.1 
254.8 

264.8 
5.9 

5.9 
Departure. 

66.3      ■ 

258.9  = 

To  find  the  Course  made  good  :— 


Departure  s     •    •    •     • . 
Difference  of  latitude  =  • 

Course  ss 


.    258.9milesvLog.  =  .    .    .   '2.413132 
.       66.3  miles    Log.  ^  .    .    .     1.821514 

S.75?38n0?E.  Log,  tangents  10.591618 


Digitized  b' 


GooQle 


638  A   OOMPXKOIUM  OP  PRACTICAL  NAVIOATION. 


To  find  the  Distance  made  good : — 

Course  made  good  =  .    ,    S.  75°38'.  IOC  E.    Log.  secant  =  10/605409 
DifTerence  of  latitude  =  .     .    66. 3  miles         Log.  =     .    .     1.821514 

Distance  = 267.3  miles         Log.  =     .    .    2.426923 

To  find  the  Latitude  and  Longitude  come  to  by  Account,  or  Dead 

Reckoning  :— 

Lat. sailed  from  =  17?58;N.    .    .     17?58'.N.  Mer.  pts=1096. 1  miles. 
Diif.  of  lat.  made 

good  66.3miIes=:I.   6  S. 


Lat.cometobyacc.l6?52?N,  Byobs.  16?55rN.  Mer.  pts=cl030. 1  miles. 
Meridional  difference  of  latitude^  by  observation,  =r     •    •    •    66. 0  miles 

Meridional  difference  of  latitude  =>  66  miles      Log.  =     .     .     1. 819544 
Course  made  good  =     .     .    S.  75 ?38<  lOr  E.  Log.  tangent^::  10. 5916 1 8 

Difference  of  long,  made  good84?  18^  E.»257. 7  miles    Log.B:2. 411 162 
Longitude  sailed  from  =     .     76. 53    W. 

Longitude  come  to  by  account=72?35 '  W. 

To  find  the  Course  and  Distance  from  the  Ship  to  her  intended  Pent  :— 

Lat.ofshipbyobs.l6^55'.N.        Merid.  pts=:  1030. 1  Long.  =  72?30fW. 
Lat.  ofHayti=  18.30  N.        Merid.ptsr=1129.8Long.  =  69.49  W. 

Diff.  of  latitude       1?35^  Mer.diff.lat.    99.7  Diff.  long.  2?4 11 

=  95  miles.  s  161  miles. 

Difference  of  longitude  —     .    .    .     161  miles    Log.  =       •    2.206826 
Meridional  difference  of  latitude  =  99. 7  miles    Log.  =       .     1. 998695 

Course  = N.  58n3C  55?  E.  Log.  tongrs  10.208131 

Course  :a'    .....    •  'N.  58?  13l55rE.  Log.secant^lO.  278617 

Difference  of  latitude  «=  .    .    .     .    95  miles       Log,  as       ,     1*977724 

.  _ 

Distance  :« 180.4  miles  ljOg.sa      .    2.256341 

Digitized  by  VjOOQ  IC 


Hence,--'nie  course  made  good  is  S,  75^38'.  lOrfi.,  or  E.b.S.  JS.  nearly. 

Distance  made  good  =a 267  j^  miles. 

Latitude  come  to,  by  account,  s=  •     16?52'.  north- 
Latitude  by  observation  s  • .     •     •     16^55^  north. 
Longitude  come  to,  by  account,  =    72?35'  west. 
Longitude,  by  observation,  ss      .    .  72?30C  west. 

The  true  course  from  the  ship  to  Hayti  is  N.58?i8<65TE.,  6t 
N.E.b.E.j(B.  nearly. 

The  course,  by  compass,  is  N.E.  ^  E.     • 

And  the  distance  1801  miles  nearly. 

Note.  —For  the  method  of  making  out  a  day's  work  by  inspectum,  see 
Problem  IX.,  page  249. 


OP  THE  LOG-BOOK. 

A  Log-Book  is  a  true  and  correct  register  of  all  the  various  transactions 
which  happen  on  board  of  a  ship,  whether  at  sea  or  in  harbour :.  such  as, 
coming  to  an  anchor,  getting  under  weigh,  loosing  or  furling  sails,  mOoring 
or  unmooring,  making  or  shortening  sail,  mustering  at  quarters  or  by 
divisions,  exercising  great  guns  and  small  arms,  &c.  &c.  &c.  This  book 
should  be  a  faithful  transcript  of  the  log-board. 

The  sea  day,  like  the  civil,  begins  at  midnight  in  the  Royal  Navy,  and 
ends  at  the  midnight  following :  it  is,  however,  divided  into  two  parts, 
each  consisting  of  12  hours.  The  first  12  hours,  or  those  contained 
between  midnight  and  noon,  are  denoted  by  a.m.,  which  signifies  ante 
meridiem,  or  iefore  mid -day;  and  the  other  12  hours,  or  those  from  noon 
to  midnight,  are  denoted  by  p.it.,  which  signifies  post  meridiem,  or  ajler 
mid-day.  The  reckoning,  however,  is  kept  from  nooft  to  noon,  the  same 
as  in  the  merchant  service. 

When  a  ship  is  bound  to  a  distant  port  or  place,  the  bearing  and 
distance  of  that  port  or  place  must  be  previously  computed,  by  Problem 
V^  page  628.  The  bearing  or  true  course,  thus  determined,  must  be 
reduced  to  the  compass  course,  by  applying  the  variation  to  the  fight  hand 
thereof  if  it  be  westerly,  but  to  the  left  hand  if  easterly : — (see  Problem  V., . 
page  496).  If  islands,  capes,  or  headlands  intervene,  it  will  be  necessary 
to  find  the  several  courses  and  distances  between  each  successively;  making 
proper  allowance  for  the  variation. 

At  the  time  of  leaving  the  land,  the  bearing  of  some  point  or  place  is  to 
be  carefully  observed,  whose  latitude  and  longitude  are  known ;  which, 
together  vrith  the  estimated  distance  of  the  ship  from  such  point  or  place, 
is  to  be  noted  down  on  the  log-board.    This  is  called  taking  a  departure^ 


640  A   COMPENDIUM   OF  PRACTICAL   NAVIGATION. 

As  the  distance  inferred  from  estimation  is  very  susceptible  of  error, 
particularly  in  hazy  weather,  or  when  that  dis,tance  is  considerable,  it  will 
be  advisable  to  make  use  of  the  following  method  in  taking  a  departure  ; 
viz.,  Let  the  bearing  of  some  well-known  place  be  observed,  and,  when  the 
ship  has  run  a  convenient  distance,  on  a  direct  course,  let  the  bearing  of 
the  same  well-known  place  be  again  observed;  then  there  will  be  a  triangle 
formed,  in  which  there  is  one  side  given :  that  is,  the  distance  sailed  between 
the  times  of  observation,  and  all  the  angles,  to  find  the  distance  between 
the  ship  and  the  place  observed.  This  may  be  done  by  Problem  I., 
Oblique  Sailing,  page  256  ;  or  it  may  be  very  readily  determined  by  means 
of  a  good  chart.  In  like  manner  may  a  departure  be  taken  from  a  light- 
house at  night. 

In  making  out  the  first  day's  work  after  leaving  the  land,  especial  care 
must  be  taken,  in  setting  down  the  bearing  and  distance  of  the  departure 
in  a  traverse  table,  to  make  use  of  the  opposite  point  of  the  compass  to 
.  that  bearing ;  and,  also,  to  make  due  allowance  for  the  variation.  Thus, 
if  the  object  from  which  the  departure  was  taken  bore  N.E.  b.  E.,  and  the 
variation  of  the  compass  be  2  points  westerly,  then  the  true  course  for 
the  traverse  Table  is  S.W.  b.  S. ;  abreast  of  which,  in  the  proper  column, 
is  to  be  placed  the  estimated  or  computed  dis^ce. 

The  course  steered,  is  indicated  by  the  compass ;  the  distance  sailed,  in 
a  given  time,  is  determined  by  the  log-line  and  the  half-minute  or  quarter- 
minute  glass.  In  His  Majesty's  Royal  Navy,  the  log  is  hove  once  in  every 
hour ;  and  so  it  is  on  board  ships  belonging  to  the  East  India  Company. 

The  several  courses  and  distances  sailed  during  the  interval  of  24  hours, 
or  from  noon  to  noon,  together  with  all  the  remarks  and  occurrences  that 
are  worthy  of  notice,  are  generally  marked  down  with  chalk  on  a  board, 
painted  black,  called  the  log-board.  This  board  is  usually  divided  into 
six  columns :  the  first  column  on  the  left  hand  contains  the  hours  from 
noon  to  noon,  viz., •from  noon  to  midnight,  and  then  from  midnight  to 
noon ;  the  second  and  third  columns  contain  the  knots  and  fathoms  suled 
every  hour;  the  fourth  contains  the  courses  steered;  the  fifth  the  winds; 
and  in  the  sixth  the  various  remarks  are  written,-^8uch  as,  the  state  of  the 
iveather,  the  sails  set  or  taken  in,  the  observations  for  ascertaining  the 
ship's  plate,  the  variation  of  the  compass,  and  whatever  else  may  be 
deemed  necessary.  The  log-board  is  transcribed  every  day  at  noon  (under 
the  direction  of  the  Master,)  into  the  log-book,  which  is  divided  into 
columns  exactly  in  the  same  manner. 

The  form  of  the  log-book  which  is  now  made  use  of  in  the  Royal  Navy, 
will  be  shown  presently. 

The  courses  steered  must  be  corrected  for  the  variation  of  the  compass, 
and  also  for  lee- way,  if  any.  .If  the  variation  be  westerly,  it  must  be 
allowed  to  the  left  hand  of  the  course  steered ;  but  if  easterly,  to  the  right 
hand  thereof,  in  order  to  obtain  the  true  course.— See  Problem  VI.,  page497» 


Digitized  by 


Google 


The  lee- way  is  to  be  allowed  to  the  right  hand  of  the  course  steered, 
if  the  larboard  tacks  be  on  board ;  btit  to  the  left  hand,  if  the  starboard 
tacks  be  on  board. 

The  variation  of  the  compass  should  be  determined  twice  a  day  (every 
moriiing  and  evening,)  if  possible.  The  method  of  doing  this  is  «hown  in 
the  several  problems  contained  between  pages  483  and  495. 

With  respect  to  the  lee-way,  its  nature  or  effect  may  be  thus  ex- 
plained:— 

When  a  ship  is  close-hauled,  and  the  wind  blowing  fresh,  that  part  of 
the  wind  which  acts  upon  the  hull  and  rigging,  together  with  a  consider* 
able  part  of  the  force  which  is  exerted  on  the  sails,  tends  to  drive  her 
immediately  from  the  direction  of»tlie  wind,  or,  as  it  is  .termed,  to  lee- 
wards But  since  the  bow  of  a  ^ip  exposes  less  surface  to  the  water  than 
her  side,  the  resistance  will  be  leiis  in  the  fii^t  case  than  in  the  second ; 
the  velocity,  therefore,  in  the  direction  of  her  head,  vrill,  in  most  cases, 
be  greater  than  in  the  direction  of  her  side ;  and  the  ship's  real  course  will 
be  between  those  two  directions.  Hence  the  angle  contained  between  the 
line  of  the  ship's  apparent  course  and  the  line  she  actually  describes 
through  the  water,  is  termed  the  angle  of  lee^way^  or,  simply,  the  lee-way. 

The  angle  oflee-wfiy  that  a  ship  makes  may  be  very  readily  determined 
in  the  following  manner ;  viz.,  Draw  a  semi-circle  on  the  taffrail,  with  its 
diameter  at  right  angles  to  the  ship's  keel,  and  its  circumference  divided 
itito  points  and  quarter-points ;  then  let  the  angle  be  observed  which  is 
contained  between  the  semi-diatneter  pointing  right  aft,  .or  parallel  to  the  . 
keel,  and  that  which  points  in  the  direction  of  the  wake,  and  it  will  be  the 
lee-way  required.  Or,  after  heaving  the  log,  if  the  line  (before  it  is  drawn 
in)  be  applied  to  the  centre  of  the  semi-circle,  the  points  and  quarter* 
po|nts  contained  between  ita  direction  and  the  fore  and  lift  radius  of  the 
aemi-cirde  will  be  the  lee-way,  as  before.         « 

Many  writers  on  navigation  have  given  rules  for  ascertaining  the  quan- 
tity of  lee- way  which  a  ship  makes,  independent  of  observation.  These 
are  as  follow  5  viz.,  . 

1.  When  a  ship  is  close-hauled,  has  all  her  sails  set,  the  water  smooth, 
with  a  light  breeze  of  wind,  she  is  then  supposed  to  make  little  or  no 
lee-way.-  .    . 

2.  Allow  one  point  when  the  top-gallant -sails  are  handed. 

3.  Allow  two  points  when  under  close-reefed  top-sails. 

4.  Allow  two  points  and  a  half  when  one  top-sail  is  handed. 

5.  Allow  three  points  and  a. half  when  both  or  the  three  top-saila  are 
handed. 

6.  Allow  four  points  wheii  the  fore-sail  or  fore->course  is  handed. 
7«  Allow  five  points  when  under  the  main^-sail  or  mfiin-course  only, 

2t 


642  A  courssDWU  o?  practical  vaticatiok. 


8.  Allow  six  pmnU  when  under  a  balaaoed  i 

9.  ADovr  seven  pomts  when  under  bare  poles. 
Ab  these  rules  depend  entirelv  upon  the  quantity  of  sail  set,  ' 

rtgkrd  to  the  model  at  the  stup,  or  to  the  nature  of  die  wajr  in 
may.  be  trimmed  for  sailing,  it  is  evident  that  they  are  far  ttom 
general,  and  that  they  are,  in  reality,  little  more  than  mere  probttUe  eoo- 
jcctures.  But  since  the  accuraey  of  aship's  reekoning  depends;,  in  some 
measure,  upon  the  truth  of  the  lee-way,  it  ought  to  be  dedoecdl,  at  aB 
times,  from  actual  observation,  as  above  directed;  and  then  its  Tahne  ahonid 
be  carefully  noted  down,  in  a  separate  orfumn,  on  the  log-board  :  ao  that 
all  concerned  may  be  thereby  enabled  to  Correct  the  course*  stecicd,  is 
making  out  their  days'  works  at  noon. 

In  very  strong  gales,  with  a  contrilry  wind  and  a  high  se%  it  is  not 
prudent  to  attempt  working  to  wmdward:  in  such  cases,  the  grand  object 
is,  to  avoid,  as  much  as  possible,  losing  ground,  or  being  driv 
Whh  this  intention,  it  is  customary  to  by  the  ship  to,  under  no 
than  may  be  barely  sufficient  to  check  that  violent  rolling  which  a 
otherwise  acquire,  to  the  eifdangering  of  her  masts,  yards,  and'  rigging. 
When  a  ship  is  brought  to,  the  helm  is  kept  about  three  parts  aleey  which 
brings  her  head  gradually  round  to  the  wind.  The  force  of  this  etenDCDt  hav- 
ing then  very  little  power  on  the  sails,  the  ship  consequently  loses  her  way 
through  the  water,  which  ctesmgtoactupon'the  rudder,  herheadfidls  off  from 
the  wind ;  the  sail  which  she  has  set  fiUsj  atad  gives  her  fresh  way  through 
the  water,  which,  acting  on  the  rudder,  brings  her  head  again  gradually 

.  round  to  the  wind ;  and  .thus  she  obtains  a  kind  of  vibratory  motion, 
coming  up  to  the  wind  and  falling  off  fiom  it  alternately. 

Ships  Iie*to  under  different  sails,  according  to  circumstancies ;  and  one 

*  vessel  will  lie^to  considerably  better  under  some  particular  sail  thAn  another. 
But,  in  general,  a  close-reefed  ihun- top-sail  is,  perhaps,  the  most  eligible 
sail  to  lie-to  under ;  because  of  its  being  nearly  over  the  centre  of  motion, 
and,  also,  because  of  its  elevated  position,  which  renders  it  far  less  suscep- 
tible of  being  becalmed  in  the  trough  of  the  sea  than' either  the  courses  or 
storm-stay-sails. 

When  a  ship  is  lying-to,  observe  the  points  of  the  compass  upon  which 
she  comes  up  and  falls  off,  and  take  the  middle  point  for  her  apparent 
course :  to  which  let  the  variation  and  the  lee-way  be  applied,  and  the  troe 
course  will  be  obtained.  Thus,  suppose  a  ship  lying-to  under  a  close- 
reefed  main-top-sail,  with  her  larboard  tacks  on  board,  comes  up  S.S.W., 
and  falls  off  to  S.W.b.  W. ;  then,  allowing  the  variation  to  be  1|  point 
west,  and  the  lee-way  to  be  21* points,  the  cburse  made  good  is  S.W.i  W.: 
for  the  middle  point  between  S,S.W.  andS.W.b.W.  is  SlW.iS.j  to 

which,  li  point  westerly  variation  being  allowed  to  the  left,  and  2j  points 
lee- way  to  the  right,  makes  the  true  course  S.W.iW. 


Digitized  by 


Google 


A  COMPXKDIUM  Or  PRACTICAL  NAVIOATIOX.  849 

The  setting  and  drift  of  currents,  ynth  tht  heave  and  drift  of  the  sea, 
should  be  set  down  as  courses  and  distances  upon  the  log-board:  these  are 
to  be  corrected  for  variation  only. 

The  xsompntation  made  from  the  several  corrected  cofirses^  and  their 
co/responding  distances,  is  called  a  day's  work;  and  the  ship's  place^ 
deduced  therefrom,  is  called  her  place  by  account,  or  dead  reckoning. 

If  the  course  and  distance  made  by  a  ship  could  be  correctly  aacertuned, 
by  means  of  the  compass  and  the  log,  nothing  more  would  be  necessary  in 
determining  her  true  place  at  sea ;  for  the  absolute  course  aiid' distance 
being  known,  the  latitude^  and  longitude  could  be  readily  computed,  by 
Problem  VL,  page  629.  But,  in  co9sequence  <i{  the  irregularities  to  which 
the  hcfkving  of  the  log  is  subject,  particularly  during  .the  night,  with  many 
unforeseen  and  itnavoidable  causes,  such  as  sudden  squalls,  imperfect 
compasses, .  unequal  care  in  the  helms-man,  inaccurate  allowances  for 
variation  and  lee^way,  &e.  &c.,  the  latitude  .and  longitude  of  the  ship,  as 
inferred  from  dead  reckoning,  will'very  seldom  agree  with  the  truth,  or 
with  those  immediately  deduced  from  celestial  observation.  In  conse- 
quence of  this  discrepancy,  Mveral.  writers  on  navigation  have  proposed  to 
apply  a  conjectural  correction  to  the  departure  or  meridian  distance,  in 
order  to  find  the  true  longitude.  Thus,  if  the  course  be  near  the  meridian, 
the  error  la  wholly  Attributed  to  the  distance,  sind  the  departure  is  to  be 
increased  or  diminished  accordingly;  if  it  be  nedr  a  parallel,  that  is,  near 
the  east  or  west  point  of  the  compass,  the  course  only  is  8Upp<»ed  to  be 
erroneous  I-  and  if  the  course  be  towards  the  middle  of  the  quadrant,  viz., 
near  four  points,  the  assumption  is  that  both  course  and.  distance  are  wrong« . 
These  corrections,  being  computed  and  applied  according  to  the  rules 
given  by  different  authors,  will  generally  place  the  ship  upon  different  sides 
of  her  meridian  by  account :' hence,  since  the  corrections  arising  from  these 
rules  are  evidently  founded  upon  a  vague  kind  of  guess-work,  they  ought 
to  be  absolutely  rejected. 

'When  the  latitude  by  account  differs  from  that  by  observation,  the  log« 
line  and  half-minute  glass  should  be  carefully  examined,  and,  if  found 
erroneous,  the  distance  sailed,  as  indicated  thereby,  should  be  corrected 
accordingly,  by  the  Problems  given  for  that  purpose,  between  pages  272  and 
276.  If  the  corrected  distance,  thus  found,  with  the  course,  does  not  pro- 
duce a  coincidence  in  the  latitudes  by  account  and  observation,  the  mariner 
should  then  consider  whether  the  variation  has  been  properly  determined  and 
allowed  upon  the  courses  steered  by  compass ;  if  not,  these  courses  are  to 
be  again  corrected;  but  no  other  alteration  whatever  should  be  made  in 
them.  If  the  latitudes  by  account  and  observation  be  still  found  to 
disagree,  the  navigJitor  should  next  Consider  whether  the  ship's  place  has 
been  affected  by  a  current  or  by  the  heave  of  the  sea,  and  allow  for  their 
course  and  drift  to  the  test  of  his  judgmenU    By  carefully  applying  thdse 

2t2 


Digitized  by 


Google 


644  A  COMPKNDIUM   OP  VRACTECAL  NAVIGATfON. 

corrections,  a  new  difference  of  latitude  and  departure,  and  a  new  course 
and  distance,  will  be  obtained ;  which  will,  in  general,  prodace  an  ap- 
proximation in  the  latitudes :  beyond  this,  no  alteration  whatever  should 
be  made  in  the  departure  with  the  view  of  finding  the  longitude  by 
account. 

However,  since  there  are  many  mariners  who,  from  long-establisked 
practice,  are  not  willing  to  depart  from  the  common  Aystem  of  correcting 
the  dead  reckoning  by  the  rules  laid  down  for  that  purpose  in  certain 
Epitomes  of  Navigation }  and  since,  these  rules  are  exceedingly  complicated, 
and  admit  of  a  variety  qf  cases j  the  following  general  rule  is  given  f©r  the 
use  and  guidance  of  such  persons,  which  reduces  those  various  cases  into 
one  very  concise  method,  and  thus  does  away  with  the  necessity  of  con- 
sulting several  complex  rules  before  the  desired  correctioif  can  be  obtained. 

A  general  Rule  for  correcting  the  Dead  Reckoning  :— 

Augment  the  distance  sailed  by  iwo-ihirds  of  the  difference  between  the 
latitude  by  account  and  that  by  observation,  when -the  observed  latitude  is 
before  or  ahead  of  that  by  account;  but  diminish  the  distance  sailed  in  the 
same  proportion,  wh^n  the  observed  latitude  is  -astern  or  behind  that  by 
account.     Then, 

Enter  the  general  Traverse  Table  with  this  corrected  djstance  and  the 
difference  of  latitude  by  observation,  and  find  the  corresponding  departure. 
Now,  with  the  departure,  thus  found,  in  a  latitude  column,  and  the  middle 
latitude  as  a  course,  find  the  correspjonding  distance,  anditwiH  be  the 
conected  difference  of  longitude. 

Example  1. 

Suppose  a  ship,  from  a  place  in  latitude  47?49<  N.  and  longitude  9?29^ 
W.,  sailed  S.  43?  W.  160  miles,  and  then  finds  her  latitude  by  account  to 
be  45'?54'  N.,  but  by  observation  her  true  latitude  is  46?39<  N.;  required 
the  longitude  come  to  by  account,  or  dead  reckoning  ? 

iSoInrion.— -The  difierehce  between  the  latitude  by  account  and  that  by 
observation,  is  15' miles;  the  two- thirds  of  which  is  10  miles.  Now,  this 
being  added  to  the  distance  sailed;  because  the  observed  latitude  is  before 
or  ahead  of  that  by  account,  makes  the  corrected  distance  =s  170  niiles  : 
with  this  corrected  distance  and  the  difference  of  latitude  by  observation, 
viz.,  2?  10^  or  130  miles,  the  corresponding  departure,  in  the  general 
Traverse  Table,  is  109. 3  miles.  Then,  with  this  departure,  in  a  latitude 
column,  and  the  middle  latitude  (between  the  latitude  sailed  fronf  and  that 
arrived  at  by  observation),  viz.,  46?44^  as  a  course,  the  difference  of  longi- 
tude corresponding  thereto,  in  a  distance  column  at  the  top  or  bottom  of 
the  page,  is  159  miles,  *or  2?39^  W. ;  which,  being  added  to  the  longitude 
left,  shows  the  longitude  at  which  the  ship  arrived  to  be  12 ?S^  west. 


Digitized  by 


Google 


Example  2. 

Suppose  a  ship  from  Perto  Santonin  latitude  33^3'  N.  and  longitude 
]6?17' W.^  sailed  N.  47?  E.  210  miles,  and  then  finds  her  latitude  by 
account  to  be  35"? 26^  N.,  but. by  observation  her  true  latitude  is  only 
35?8C  N.;  required  th^  longitude  come  to  by  accouiit,  or  dead  reckoning  ? 

Solution.-^The  diifference  between  the  latitude  by  account  and  that  by 
observation;  is  IS'  miles;  the  two- thirds  of  which  is  12  miles.  Now,  this 
being  subtracted  from  the  distance  saifed,  because  the  observed  latitude  is 
astern  or  behind  that  by  account,  makes  the  corrected  distance  =198 
miles:  with  this  corrected  distance  and  the  difference  of  latitude  by 
observation,  viz.,  2^5^  or  125  miles,  the  corresponding  departure,  in  the 
general  Traverse  Table,  is  153.5  miles.  Then,  with  this  departure,  in  a 
latitude  column,  and  the  middle  latitude  (between  the  latitude  sailed  from 
and  that  come  to  by  observation),  viz.,  34?5|t  as  a  course,  the  difference 
of  longitude  corresfiondtng  thereto,  in  a  distance  column  at  the  top  or 
bottom  of  the  page,  is  185  miles,  or  3?5f  E.;  which,  being  subtracted 
from  the  longitude  left,  shows  the  longitude  at  which  the  ship  arrived  to 
be  13?  12^  west. 

Remath.^-Alihongh  the  above  gener&l  rule  for  correcting  the  dead 
reckoning.  i3  the  most  simple,  and,  perhaps  the  most  accurate  of  any  that 
have  been  as  ye^  devised  for  that  purpose,  yet  the  author  has  frequently 
found,  on  making  the  land  after  a  long  voyage,  that  the  longitude  deduced 
therefrom  d^ered  several  degrees  Jroin  the  truth:  hence  it  is  evident, 
notwithstanding  the  easy  and  specious  feasibility  of  this:method,  that  the 
prudent  mariner  will  do  well  to  be  extremely  cautious  in  applying  it  to 
practice ;  nor  should  he  ever  place  any  manner  of  faith  in  the  longitude  so 
deduced,  particularly,  if  he  has  been  any  considerable  time  from  the  land. 
From  this  it  is  manifest  that  the.  navigator  should  determine  the  longitude 
of  his  ship,  as  often  as  possible,  both  1^  the  lunar  observations  and  l^y  a 
chronometer;  and  from  the  true  longitude,  thus  found,  the  reckoning  of 
this  element  is  to  be  carried  forward,  in  the  same  manner  as  that  of  the 
latitude,  from  the  last  observation.  A  separate  account,  however,  should 
be  kept  of  the  longitude  by  dead  reckoning :  such  account  i^.not  only  very 
satisfactory,  but  it  often  .proves  highly  useful  as  a  reference;  particularly 
in  comparing  the  computed  velocity  and  drift  of  a  current  with  those 
deduced  from  actual  experiment. 


The  following' is  the  form  a(  the  log-book  which  is  now  used  in  His 
Majesty's  Royal  Navy,  and  from  which  we  will  make  out  apracHca/  day's 
work. 


646 


Log'Baak  qfHU  Majesty's  Ship  • 


Wedneidayj  June  4th,  1823. 


H. 

10 
11 
13 

K.  ] 

?.  Ccunes. 

Winds. 

No.  of 
Sisals. 

S.E. 
S.E. 

A.M.    Moderate  breezes  and  basy  ireatber. 

At  4»30«»  employed  washing  decks. 

At  7*30**  unmoored  ship,  and  hnre  short  on  tfae  best 
bower. 
At  8  UO"  weighed  and  made  saU. 

At  noon  light  winds  and  hazy  weather. 

Coarse. 

Distance. 

Lat  by 
Account. 

Latitude  b} 
Obserr. 

Long,  hv 
Account. 

Ob«erv.    ^''™- 

Variation 
at  Noco. 

Bearinn  and 
Di.st.atN«on. 

34°20'S. 

• 

25o2d'W. 

cape  of  Gaud 

HopeK  b.S. 

(5  miles. 

r 

2 
3 

4 

6 

7 

9 

9 

10 

11 

12 

5  4 

5  4 

6  0 
6  2 
6  2 

6  2 

7  6 

8  0 

8  6 

9  0 
9  4 
»  6 

N.W.b.W. 
N,W.§W. 

N.W.<W. 

N.W. 

SJS. 
8.E. 

p.  M.    Moderate  breeses,  with  ihtclc  basj  wcatbcr. 

At  2*30"  shook  a  reef  nut  of  the  top.-sails^  and  set 
the  top-mast  and  fop-gaUaat  studding-sails.     • 

At  5*  heat  to  quarters,  and  exercised  great  raos 
and  small  sirms. 

At  8*  firesh  breexes  and  doady  weather. 

At  midnight  fresh  breeses  and  dear  weather. 

Log-Booi  qfjht  Majatjf'i  Ship  ■ 


't  Tkurtdoff^  June  5th,  1^3. 


H. 

K. 

9 

9 

9 

9 

9 

9 

10 

10 

9 

10 

9 

11 

9 

12 

9 

Course. 


N.69W. 


Courses, 


TTWT 


N.W.b.N. 


Distance 


214  miles. 


Winds. 


"SET 


SJS, 


S.E. 


No.  of 

Signals. 


Ramaika  and  Osdureaces, . 
▲.M.    ^resh  breezes  and  ciear  weather. 


Lat.  by 
Account. 


33o6'S. 


Latitude  by  Long,  by 


At  4  *  ane  clear  weather ;  emfdoyed  washing  decks. 
At  6* 30«  set  the  fewer  stikdding-sails* 
At  8*40»  in  lower  ttaddiog-sails. 

At  10*30*  mottmd  by  diidsipBi,  and  Inspcettd  the 

eople's  clothing. 

At  noon  fresh  breezes  and  fine  ^rlearnreather. 
Long,  by 


Observ. 


33?  4' 30*^5. 


Account. 


Lunar 
Obsenr. 


14<'24'E.  U^gr-R 


Lottg.by  Variation 


Chron. 


at  Noon. 


14«24'E.23<»10'W, 


Bearings  and 
Disk  at  Noon. 

jSt.  Helena 
N.4flP&l'W., 
dist.  lias  ms. 


Digitized  by 


Google 


Note. — ^The  departure  is  taken  from  the  Cape  of  Good  Hope ;  and  as 
this  place  bore,  at  "aoon,  E.  b.  S.  from  the  ship^  .distant  15  miles,  the  com- 
pass bearing  or  course  of  the  ship  from  the  Cape  was,  therefore,  W.  b:  N. 
NdW,  the  variation,  2i  points  west,  being  allowed  to  the  left-hand  of 
W.  b.  N.,  shows  the  true  bearing  or  course  to  be  W.  b.  S,  i  S.  The  •  other 
courses  are,  in  like  manner,  to  be  corrected  for  variation ;  but  since  the 
value  of  this  element  is  not  the  same  at  both  noons,  it  is  advisable  to  allow 
25?20<,  6r  2i  points  west,,  on  the  courses  in  the  first  12  hours,  of  between 
noon  and  midnight,  and  23^10%  or  2  points  west,  on  the  courses  in  the 
other  12  hours,  or  between  midnight  and  noon;  then,  these  corrected 
courses,  with  their  respective  distances,  being  inserted  in  a  Traverse  Table, 
after  the  following  manner,  the  difference  of  latitude  and  departure  corre- 
sponding thereto,  \iith  the  course  and  distance  made  good,  maybe  readily 
determined  by  calculation^  agreeably  to  the  rule  given  iii  Problem  VIIL, 
page  633 ;  or,  perhaps  more  readily,  by  the  general  Traverse  Table. — See 
Problem  IL,  page  t08.  '  . 


T&AVBBSB  TaBLS. 

Corrected 
Courses. 

Dtttances. 

■  DiflEsrence  of  Latitvde. 

Departure. 

N.. 

S.  . 

£.. 

w. 

W.b.9.iS. 

W.JN. 
W.b.N.iN. 
W.b.N.}N. 
W.b.N.fN. 

W.N.W. 
N.W.b.W. 

15 
11 

18 
22 

37    . 
48 
6d 

Diff.oflat. 

1.6 

4.4 
.  6.4 
12.5 
18.4 
37.8     , 

3. 6 

— 

14.6 
10.9 
17.5 
21.1 
d4.-8 
44.3 
56,5 

8l.l     . 
3,6 

3.6 

Departures 

199.7 

o.p 

77.5 

199.7 

To  find  the  Course  and  Distance  made  good  :— 


The  differeiMy  6f  latitude  77 '^i  <^d  the  departure  199. 7,  are  found  to 
sfgree  nearest  abreast  of  69  degrees,  and  under  or  over  214  miles  distance. 

Hence  the  course  made  good  is  N.  69?  W%  or  W,b,N,f  N.  nearly,  and 
the  distance  214  miles. 


Lat.  in  by  ace,  =    .  33?  6'.  lOrS.  By  ob.33?  4^30?  S.  Mr,  p8.=2104. 9 

•  ■^^— — — — — 

Meridionaldifferenceoflatitude^  by  observation^  =     •    .     ,     •  95.2 

[miles. 

Meridional  difference  of  latitude  =  95. 2  miles     Log.  =  .     .1.  978637 
Course  made  good  =     •     .     .    .     N.  69^  W.     Log.  tang.  =  10. 415823 


Diff.  of  long,  made  good  =  .    4°  8^        W.=248  ms.    Log.=2. 394460 
Long,  of  C.  of  Good  Hope  =  18?32:  IS^T  E. 


Long,  come  to  by  account  =  14?24!  15  ^  E. 

To  find  the  Coyrse  and  Distance  from  the  Ship  to  St.  Helena : 

Lat.ofshipbyobs.33?  4^  S.      Mer.pt8.  =  2104.3      Long.=  14?28^£. 
Lat.  of  St.  Helena  15.55    S.      Mer.pts.=    967-5       Long*=  5.44  W. 


Diff.oflat  =:   .     17?  5^    Mer.  diff.  lat,=  1136.8  Diff. long.  20?  12: 

=:  1029  miles.  =  1212  mUes. 

Difference  of  longitude  s    1212  miles.  Log.  =  .    •    .    3. 0S3503 

Merid.diff.  lat.  =      •    .     1136.8  miles.  Log.  =  •    .     .    3.055684 


Course  =  .     .    •    .    N.46?51CWr        Log.tang.  =  .     10.027819 

Courses.     ...    N.  46?5r.  W.  Log.  secant  =    10.165001 

Diff.  of  lat. «       ,    .    .     1029  miles.     .   Log.  ss  .     .•   .    3.012415 


Distance  =:     ....     1505  miles.        Log,  =  ,    .    .    3. 177416 

Hence,— The  course  made  good  is  N.  69?  W.  or  W.  by  N.  }  N.  nearly.* 
Distance  made  good  =     .    .     .     2 14*  miles. 
Latitude  come  to  by  account  '==      83?  6'  IQ":  S. 
Latitude  by  observation  =     .     .    33.   4.80'  S. 
Longitude  come  to  by  account  =:  14. 24. 15    E. 
Longitude  by  lunar  observation  :=  14. 28,   0    E. 


'     To  find  the  Latitude  and  Longitude  come  to  by  Account : — 

Lat.  Cape  GoqdHope34?23U0rS.    .  34?23M0r  S.  Mer.pts.=220a.  I       | 
Diff.  Lat.  77. 5  ms.  .     1. 17. 30  N. 


Digitized  by 


Google         ^ 


A   COMPENDIUM   OF  PRACTICAL  NAVIGATION.  D4Sl 

Longittide  by  chronometer  =  .    .14. 24.   0    E. 

Variation  at  noon 23. 10.  0    W.  * 

St.  Helena  bears  N.  46?5 1  ^  W.  or  N,  W.  J  W.  nearly,  inde- 
pendent  of  variation. 

Distsant  •    •    • 1503  miles. 


OF  THE  MEASURE  OP  A  KNOT  ON  THE  LOG  LINE. 

It  ha&beeQ  remarked,  page  272,  in  the  introduction  to  the  Problems  for 
correcting  the  distance  sailed  on  account  of  any  errors  that  may  be  dis-- 
coyered  in  the  log  line  and  half-minute  glass,  that  the  distance  between 
any  two  adjacent  knots  on  the  log  line  should  bear  the  same  proportion  to 
a  nautical  mile  that  half  a  minute  does  to  ai)  hour,  viz.  the.oue  hundred 
and  twentieth  part ;  that  a  nautical  mile  contains  6080  feet ;  and  that  this 
number  divided  by  120,  gives  the  true  measure  of  a  knot,  viz.  50  feet  a^d 
8  inches. — But,  sinc^  the  young-  navigator  may  be  desirous  of  being  ;nade 
acquainted  with  the  principles  upon  which  th|S  measure  has  been  deter- 
mined, the  following  considerations  are,  therefore^  submitted  to  his  atteii* 
tion ;  which,  besides  satisfying  him  in  that  particular,  may  do  something 
towards  giving  him  a  just  idea  of  the  true  figure  of  the  earth  ;^and,  with- 
out which  idea  he  can  liever  clearly  comprehcind  the  principles  upon  which 
the  art  of  navigation  is  founded. 

The  earth  is  a  planet,  and  the  next,  in  the  solar  system,  above  Venus.— 
Our  senses  assure  us  of  its  opacity ; — and  that  it  is  of  a  glo|:>ular  or  spheri* 
cal  figure  will  appear  evident  from  the  arguments  which  follow:—- 

A  lunar  eclipse  is  occasioned  by  the  moon's  passing  through  theiearth's 
shadow  ;  and  since  this  shadow,  when  projected  on  the  lunar  disc,  is  ob* 
seived  to  be  always  circular  in  every  different  position  of  the  earth,  it 
necessarily  follows  that  the  •earth,  which  casts  the  shadow,  must  be  sphe- 
rical, since  nothing  but  a  sqphere,  when  turned  in  various  positions  with 
respect  to  a  luminous  body,  can  project  a  circular  shadow. — ^Again, 

A  lunar  eclipse  is  observed  sooner  by  those  who  live  eastward  than  by 
those  who  live  westward ;  the  di^erence  of  time  being,  always  propor* 
tional  to  the  difference  of  longitude  between  the  plfu^es  of  observation. 

Now,  if  the  earth  were  an  extended  plane^  as  the  primitive  fathers  as- 
serted, the  eclipse  would  happen  at  the  same  instant  in  all  places :— but 
this  is  so  far  from  being  the  case,  that  the  inhabitants  of  Jamaica  will  not 


*  The  variation  of  the  compass  may  he  very,  readily  determined  at  noon  (sufficiently  c<)r* 
rectly  for  nautical  purposes,)  by  the  second  part  of  th«  Rule  to  Problem  IV.  page494«  read* 
ing  tun  instead  of  star  or  planet  .«*fiee  note  at  bottom  of  pa^  495« 


OW  A  GOIIPJEMDIDH  OF  PIUCTICAL  NAVIGATION. 

see  an  eclipse  of  the  moon  until  about  five  houn  after  it. takes  place  at 
Greenwich ;— therefore  the  figure  of  the  earth  mttst'be  spherical^  or  very 
nearly  so. 

If  the  earth  were  an  extended  plane^  the  meridian  zenith  distance  of  any 
one  fixed  star  would  be  the  same  in  all  parts  of  the  world  ;  because  the 
measure  of  the  earth's  diameter  bears  no  more  propdttion  to  the  immea- 
surable distance  of  the  nearest  fixed  star  than  an  indivisible  point' does  to 
the  diameter  of  the  earth.-r-But,  since  the  meridian  zenith  distance  of  the 
same  fixed  star  is  found  to  differ  with  the  latitude,  the  difference  in  the 
zenith  distance  being  always  proportional  to  the  intercepted  arch  of  the 
meridian ;  and  since  it  is  the  known  property  of  a  curve  that  the  arches  are 
proportional  to  their  correspondent  angles,  therefore  the  surface  of  the 
earth  and  sea  is  of  a  curvilinear  form.— Menqe  the  earth  must  be  of  a  sphe- 
rical figure. 

The  earth  has  been  circumnavigated  by  mbny  .persons,  at  diflerent 
periods,  who,  by  sailing  in  a  westerly  direction,  allowance  being  made  for 
promontories,  &c.  arrived  at  the  place  whence  they  sailed. — Henee,  the 
earth- must  be  either  of  a  cylindrical,  or  a  spherical  .figure  ;-^but  that  it  is 
not  of  a  cylindrical  figure  will  appear  obvious  by  considering  that  the  dif^ 
fer^nce  of  longitude  and  meridional  distance  between  two  places  would,  on 
the  cylindrical  hypothesis,  be  equal  ;*-whereas,  experience  and  actual  ob- 
servation, demonstrate  that  the  very  reverse  of  this  takes  place  ^-**tberefe^e 
the  earth  must  be  of  a  spherical  form  from  west  to  east. 

If  a  ship  in  north  latitude  sails  southerly,  the  north  polai'  star  will  be 
found  gradually  to  decrease  in  altitgde  till  the  vessel  reaches  the  Equator  ; 
at  which  place  the  star  virill  be  seen  immersed  in  the  horizoh.-**After  cross- 
ing the  Equator^  and  as  the  ship  advances  in  the  southern  hemisphere,  the 
stars  in  the  neighbourhood  of  the  south  celestial  pole  will  be  seen  gradu- 
ally emerging  from  the  southern/horizon,  and  increasing  in  altitude,  whilst 
those  about  the  north  celestis^  pole  will  be  entirely  lost  sight  of;  being 
hid  below  the  horizon  :~hence  the  earth  is  spherical  from  north  to  south  ; 
but  it  is  also  spherical  from  west  to  east,  as  appears  from  its  circumnavi- 
gation ;  therefore  the  figure  of  the  earth  is  that  of  a  sphere. 

When  two  distant  ships  are  approaching  each  other,  at  sea,  the  royals 
and  top-gallant  sails  only  of  each  are  visible  at  first;  the  lower  sails  and 
hulls 'being  concealed  by  the  convex  surface  of  the  waters — but  as  th^ 
draw  nearer  towards  each  other,  the  parts  that  Were  so  concealed  by  the 
convexity  of  the  sea's  surface,  will  be  seen  toilse  gradually  above  the  Im^* 
zon. — ^Now,  if  the  sea  were  an  extended  plftne,  the  hulls  or  bodies  af  the 
ships  would  be  the  first  parts  seen;  and  because  they  arc  the  largest,  they 
would,  evidently,  be  seen  at  the  greatest  distance ;  nor  would  the  small 
sails  near  the  masts'  heads  be  visible  until  the  ajiproacb  of  the  ships 
brought  them  considerably  nearer. 


Digitized  by 


Google 


In  making  the  land  the  most  elevated  parts  are  first  seen,  such  as  moun- 
tains, kc. ;  then  tops  of  light  houses  and  steeples,  and  shortly  afterwards  the 
coast,  or  beach : — this  plainly  demonstrates  that  the  surface  of  the  sea  is 
convex.  • 

The  sun  is  observed  sooner  at  rising  and  later  at  setting  by  a  person  at 
the  mast-head  of  a  ship  than  by  one  on  deck ;  and  so  is  the  moon  and  all 
other  celestial  objeets.-^These  phenomena  evidently  arise  from  the  sphe- 
rical figure  of  the  earth  ;  and  are,  therefore,  most  convincing  and  satisfac- 
tory proofs  of  Its  globularity. 

Again.— The  continual  presence  of  the  «un,  above  the  horizon,  during 
the  space  of  several  months  in  the  neighbourhood  of  one  terrestrial  pole,, 
while  at  a  place  equally  distant  from  the  other,  he  is  as  long  absent,  affords 
another  convinchig  proof  that  the  earth  is  of  a  spherical  figure. 

The  spherical  figure  of  the  earth  may  be  also-  inferred  from  the  method 
of  levelUngj  or  the  art  of  conveying  water  from  one  place  to  another ; — for, 
in  this  operation  it  fs  always  found  necessary  to  make  an  allowance 
between  the  true  and  the>  apparent  levels  on  account  of  the  rotundity  of 
the  earth ;  the  true  level  being  a  curve  line  which  falls  below  the  straight 
lirie  of  apparent  level  about  6  inches  in  1  mile ;  32  inches  in  2  miles  $  128 
inches  in  4  miles,  &c.,  the  curvature  always  augmenting  in  proportion  to 
the  square  of  the  distance.     See  Problem  X.,  between  pages  545  and  547. 

Finally,-' All  die  planets  arie  observed  to  be  of  a  spherical  figure }  and 
since  the  earth  is  a  planet,  subject  to  the  same  laws,  and  revolving  round 
the  sun  in  the  same  manner  as  the  other  planets,  it  must,  therefore,  by 
analogy,  be  also  spherical. ' 

The  irregularities  on  the  earth's  surface,  occasioned  by  mountains  and 
TalHes,  are  very  inconsiderable  compared  with  its  magnitude ;  and  take  off 
no  more  from  its  actual  rotundity  than  the  little  risings  on  the  coat  of  an 
orange  do  from  the  rotundity  of  that  fruit  2— for  the  highest  eminence  or 
mountain  bears  a  less  proportion  to  the  magnitude  of  the  earth  than  the 
smallest  grain' of  sand  does  tor  an  18-inch  globe. — ^Thus*, 

The  summit  of  Chimbora$o,  one  of  the  Andes  Mountains,  and  the  high- 
est in  the  known  world,  is  only  20280  feet  above  the  level  of  the  sea, 
or  not  quite  4  mtle^  in  perpendieular  height. — ^Now,  the  radius  of  the 
earth  is  30902200  feet>  and  that  of  an  18-inch  j^obe  9  inches  ;^hence,  by 
the  rule  of  proportion,  as  20902200  feet  :  20902200  feet  4-  20280  feet 
s  20922480*  feet  : ;  9  inches  to  9. 0087  inches  i  from  which  deduct  9 
inches  (the  radius  of  the  artificial  globe,)  and  the  remainder  0. 0087  is  the 
relative  elevation  of  Mountf  Chimbora^o  on  an  18'inch  globe ;  and  as  this 
is  scarcely  the  one  hundred  andjl/ieenth  part  of  an  inch,  it.  is,  therefore, 
considerably  less  than 'a  common  grain  of  sand. — Hence  it  is  evident 
that  the  highest  mountidns,  and  deepest  vallies,  take  little  or  nothing  from 
the  earth's  rotundity. 


0^^  A.  COMPENDIUM  OP  PRACTICAL  NAVIGATION. 

•  Although  when  speaking  of  the  earth  in  general  terms,  it  may  be  < 
dered  as  a  globe,  or  sphere ;  yet,  in  strictness  it  is  not  a  perfect  sphere,  but 
rather 'an  oblate  spheroid ;  which  is  a  solid  generated  by  the  re?oliitionof 
a  semi-ellipse  about  its  shorter  axis  or  diameter ;— and  actual  admeasure- 
ments, in  sundry  places,  have  clearly  proved  that  the  polar  axis,  or  diame- 
ter is  about  35  miles  less  thati  the  equatorial  diameter. — However,  »iice 
the  earth*  differs  so  very  little  from  a  globe  or  sphere,  it  may,  ^erefose,  be 
very  safely  considered  as  being  perfectly  spherical  in  all  nautical  ealcnlatioos 
whatever. 

The  spherical  figure  of  the  earth  being  (bus  satisfactorily  established,  its 
jnagnitude  may  be  determined  by  measuring  a  small  portion  of  a  meridian, 
and  observing  the  zenith  distances  of  one  or  more  stars  at  the  extreme  sta.- 
tions;  then,  the  difference,  between  the  zenith  distances  of  the  same  star 
gives  the  correspondent  celestial  arch.— :Now, 

As  the  celestial  arch,  thus  found,  is  to  the  measured  or  intercepted  por- 
tion of  the  qaeridran  ;  so  is  one  degree,  to  its  absolute  length  in  the  same 
measure  in  which  the  portion  of  the  meridian  was  {aken. 

In  this  manner  the  celestial  arch  of  one  degree  has  been  fovmd  to  con- 
tain 69. 093  English  miles ;  and  since  the  earth's  circomference,  like  that 
of  all  other  spheres,  contains  360  degrees;  thereforie  360  degrees  x  69.093 
miles  =  24873. 48,  is  the  true  measure  of  the  earth's  circumference  in 
English  miles. — Hence,  its  diameter  is  7917^  miles,  English  measure. 

Now,  since  the. nautical  arch  is,  in  every  respect,  equal  to  the  celestial 
arch,  the  length  of  a  degree  in  the  one  being  precisely  equal  to  the  length 
of  a  degree  in  the  other,  each. containing  60  geogri4>hical  miles j  and  since 
the  measure  of  a  degree  of  this  arch  in  English  miles,  is  69. 093,  or  364815 
English  feet;— therefore  364815  feet  •+-  60  miles  =  6080  feet;  whiefa, 
evidently,  is  the  true  length  of  a  nautical  mile,  expressed  in  English  mea- 
sure.— ^And,  if  6080  feet  be  divided  by  120  (tiie  numbev  of  half  minutes  in 
an  hour,)  the  quotient  50  feet  and  8  inches  will  be  the  true  measure  of  a 
knot.— And,  hence  the  principles  upon  which  the  measure  of  a  knot  upon 
the  log  line  has  been  determined. 

But,  because  it  is  safest  to  have  the  reckoning  a-head  of  the  ship,  48 
feet,  or  8  fathoms  are,  therefore^  commonly  allow^  between  every  two  ad- 
jacent knots  oil  the  log  line : — and  this  measure  is  to  correspond  to  a  g^ass 
running  30  seconds ;  or,  rather  29|  seconds,  so  as  to  make  up  for  any  time 
that  may  be  unnvoidably  lost  in  the  act  of  turning  the  gla 


jRemarL— The  instruments  made  use  of  for  measuring  angles  ats^  and 
for  ascertaining  the  latitude  and  longitude,  are  quadrants,  sextants,  and 
reflecting  circles.  Since,  however,  splu^e  cannot  be  afforded  in  this  weak 
for  giving  particular  descriptions  of  these  instruments,  and  the  man- 
ner of  adjusting  and  using  them;  the  reader  is,  therefore^  reqpecCfydly 


Digitized  by 


Google 


referred  to  an  ocular  examination  of  the  iustniments,  and  to  a  few 
explanatory  remarks  from  some  person  practically  acquainted  with  the 
various  uses  to  which  they  may,  or  can  be  applied. — A  few  hours  practical 
•instructipn  will  convey  more  real  information  to  a  person  on  these  sub- 
jects, than  if  he  were  to  spend  a  whole  year  in  poring  over  the  voluminous 
descriptions  which  have  been  puUished,  by  different  authors,  relative  to 
the  use  of  the  quadran^  and  sextant. 

Instead,  therefore,  qf  wasting  time  and  paper  with  descriptions  that  may 
be  well  omitted,  we  will  here  endeavour  to  describe  that  which  is  of  far 
more  importance  to  the  practical  navigator,  viz.    . 

The  true  Method  of  finding  the  Index  Error  of*  a  Sextant,  so  as  to 
guard  against  the  Errors  arising  from  the  Flexibility  and  the 
*    Friction  of  the  Index  Bar. 

The  customary  method  of  finding  the  index  error  of  a  quadrant  or  sex- 
tant (as  directed  by  writers  on  the  use  -of  these  instruments,)  is  by  measuring 
the  vertical  diameter  of  the  sun  to  the  right  and  left  of  0  on  the  arch,  with 
a  motion  of  the  index  in  contrary  directions  (that  is,  by  bringing  the  re- 
flected image  to  touch  the  lower  and  upper  limbs  of  the  direct  object  alter- 
nately), and  theh  taking  half  the  difference  of  those  measures  for  the  index 
error  of  the  instrument. — ^lliis  method,  it  must  be  observed^  is  very  far 
irom  being  correct ;  because  it  is  the  hocizontal  diameter  of  the  sun,  and 
not  Us  vertical  diameter,  that  should  be  n^easured;  for,  while  the  former 
remains  invariably  the  same,  the  latter  is  subject  to  continual  alterations 
owing  to  tlie  effects  of  atmospherical  refraction,  as  will  appear  evident 
by  an  inspection  of  the  last  column  of  Table  V.— Moreover,  since  the  index 
is  not  an  inflexible  bar,  and  since  it  doe»  not  turn  upon  its  centre  without 
suffering  some  slight '  degree  of  friction ;  it  is  therefore  evident  that  the 
measure  of  the  sun*s  diameter  taken  by  the  progressive  motion  of  the  index 
will,  in  most  cas^s,  be  more  than  the  truth.:  whilst  that  taken  by  the  con- 
trary or  retrogressive  motion  will,  in  general,  be  less  than  the  truth  :— 
hence,  the  index  error  established  upon  the  above  principles  must  frequently 
mislead  the  mariner  by  rendering  inaccurate  what,  otherwise,  might  be  a 
very  correct  observation. .  And  this  accounts  for  the  result  of  the  evening 
observations,  taken  on  shore  by  means  of  an  artificial  horizon,  so  very  sel-. 
dom  agreeing  with  the  result  of  those  taken  in  the  morning  ;  even  though 
all  imaginable  care  be  used,  and  though  the  observer  keeps  the  same  plane 
and  roof  of  the  horizon  directed  to  him  during  the  time  *of  both  obser- 
vations. 

Now,  to  guard  against  the  errors  arising  from  the  bending  and  the  fric- 
tion of  the  index  bar,  as  well  as  that  proceeding  from  the  contraction  of 
the  sun^s  vertical  diameter ;  let  the  following  observations  be  attended  to. 


in  finding  the  ind^  error  of  a  quadrant  or  soxtanty  and  the  coojmat  ciect 
of  thoie  errors  will  be  obviated. 

JFltr^.— To  find  the  Error  for  a  Progrestive  Motioii  of  the  Index  :-- 

Screw  the  inverting  telescope  into  its  place.  Slack  the  index.  Tun 
the  tangent  screw  backward  to  nearly  as  far  as  it  will  go.  Put  the  noniai 
of  the  index  about  1?151  to  the  right  of  0  on  the  arch,  and  then  fiwten 
the  index  sufficiently  tight  for. observation.-— Hold  the  sextant  so  that  its 
plane  may  be  parallel  to  the  horixontal  diameter  of  the  sun  :  direct  the 
sight  to  that  object,  and  turn  the  tangent  screw  forward  imtil  the  limbs  of 
the  sun  seen  by  reflection  and  direct  vision  make  a  perfect  contact. — ^Note 
down  the  angle  and  it  will  express  the  measure  of  the  sun's  diameter  to 
the  right  of  0  on  the  arch.^Direct  the  sight  ag^n  to  the  sun,  and  turn  die 
tangent  screw  gtill  forward  until  the  opposite  limbs  are  in  perfect  contact : 
note  down  the  angle,  and  it  will  be  the  measure  of  the  sun's  diameter  to 
the  left  of  0  on  the  arch.— Now,  if  both  measures  of  the  diameter  are  the 
same,  there  i^  no  error  in  the  angles  shown  by  the  progressive  motion  of  the 
index ;  but  if  those  measures  do  not  correspond,  half  their  difference  is  to 
be  taken  as  the  index  error  of  the  instrument,  which  error  will  be  additive 
when  the  diameter  measured  to  the  right  of  0  exceeds  that  measured  to 
the  left ;  otherwise,  subtractive.— Then,  this  error  is  to  be  considered  as  a 
constant  quantity  (so  long  as-  the  instrument  does  not  meet  vrith  any  aeci^ 
dent),  and  to  be  applied  to  allwcreasifig  angles,  either  of  altitude  or  dis- 
tance, which  may  be  taken  by  th^  progressive  motion  of  the  index* 

Jgainr^To  ^nd  the  Error  for  a  Retrogressive  Motion  of  the  Index  :-- 

Slack'  the  index.  Turn  the  tangent  screw  forward  to  nearly  as  &r  as 
it  will  go.  Put  the  nonius  of  the  index  about  1°.\5'  to  the  I^  of  0  on  the 
arch,  and  then  fasten  the  index  sufficieudy  tight  for  observation* — ^Hold  the 
sextant  as  before ;  direct  the  sight  to  the  sun,  and  turn  the  tangent  screw 
backward  until  the  limbs  of  the  sun  seen  by  reflection  and  direct  vision 
make  a  perfect  contact : — ^note  down  the  angle,  and  it  will  express  the  mea- 
sure of  the  sun's  diameter  to  the  left  of  0  on  the  ^rch*:— Direct  the  si^t 
again  to  the  sun,  and  turn  the  tangent  screw  still  backward  until  the  op* 
posite  limbs  are  in  perfect  contact ;  read  off  the  angle,  and  it  will  be  the 
the  measure  of  the  sun's  diameter  to  the  right  of  0  on  the  arch* — Now,  if 
both  measures  of  the  diameter  are  the  sam^  there  is  no  error  in  the  angles 
shown  by  the  retrogressive  motion  of  the  index :  but  if  those  measures  do 
not  correspond,  half  their  difference  is  to  be  taken  as  the  index  error  of  the 
instrument ;  which  error  will  be  luiditive  when  the  diameter  measured  to 
the  right  of  0  exceeds  that  measured  to  the  left ;  otherwise,  subCractive* 


Digitized  by 


Google 


Then,  this  error  is  to  be  considered  as  a  constant  quantity  (so  long  as  the 
instrument  does  not  meet  with  any  accident)^  and  to  be  applied  to  all  de- 
creanng  angla,  either  of  altitude  or  distance,  which  may  be  taken  by  the 
backward  or  retrogressiire  motion  of  the  index. 

Hence  it  is  very  probable  that  two  errors  may  be  established  for  the 
same  ii^trument }  the  one  for  increasing,  and  the  other  for  decreasing  an-> 
gles*  The  true  values  of  those  errors  should  be  noted  down  (for  the  future 
guidacce  of  the  observer,)  with  a  black-lead  pencil  on  the  inside  of  his  sex- 
tant case  in  the  following  manner,  viz.  i-^ 

Error  for  the.  forward  or  progressive  motion  of  the  index  OMO?  sub- 
tractive.  . 

Error  for  the  backward  or  retrogressive  motion  of  the  index  IMO* 
additive. 

Or. whatever  the  errors  may  be. 

And  thus  the  correct  values  T)f  the  index  error  will  be  properly  deter- 
mined, whilst  the  errors  arising  from  the  spring  and  the  friction  of  the 
bar,  together  with  that  proceeding  from  the  contraction  of  the  sun's  ver- 
tical diameter  will  be  all  safely  provided  against.  . 


OP  TAKING  THE  ALTITUDE  OP  A  CELESTIAL  OBJECT 
BY  MEANS  OP  AN  ARTIFICIAL  HORIZON. 

Since  the  generality  of  nautical  persons  do  not  appear  to  be  sufficiently 
acquainted  with  the  manner  of  applying  the  necessary  corrections  to  angles 
of  altitude  taken  by  means  of  an  artificial  horizon;  the  author  is,  there- 
fore, induced  to  make  a  few  observations'  touching  the  direct  application 
of  tliose  corrections ;  in  doing  which  some  hints  wiU  be  thrown  out  for  the 
guidance  of  young  observers,  relative  to  the  nature  and  use  of  the  astro*- 
nomical  instrument  now  under  consideration. 

Cf  the  Artyicial  Horizon. 

In  settling  the  positions  of  places  in-land  in  an  astronomical  manner, 
or- in  ascertaining  the  error  and  the  rate  of  a  chronometer  on  shore  where 
there  is  not  an  open  and  commanding  view  of  the  sea  horizon,  the  ob- 
server must,  ia  all  such  cases,  have  recourse  to  an  artificial  horizon  for  the 
purpose  of  taking  the  necessary  angles  of  altitude. 

Although  there  is  a  great  variety  of  artificial  horizons  now  extant,  yet, 
for  the  sake  of  conciseness,  I  shall  only  touch  upon  the  two  that  are  in  my 
own  posscssion*-^The  first  of  these  consists  of  a  plane  spetulum,  or  polished 


656  OF  TAKING  TMS  ALTITUDB  OF  A  -CBLESTIAL  Q^BCT. 

ptete  of  glass  (4  inches  long  by  3  inches  broad,)  fixed  in  a  brass  frame, 
and  standing  upon  three  adjusting  screws  :  by  means  of  these  and  a  spirit 
level,  placed  in  different  positions  on  its  surface^  it  may  be  made  perfectly 
parallel  to  the  plane  of  the  horizon  :  observing  that  the  adjusting  screws 
are  to  be  turned  until  the  air-bubble  rests  in  the  middle  of  the  spirit  level 
on  the  surface  of  the  speculum. — ^The  other  is  the  common,  or  quicksilver 
horizon  ;— this  simply  consists  of  a  small  wooden  trough,  about  half  an 
inch  deep,  3i  inches  long,  and  2^  inches  broad  ; — ^into  this  trough «  few 
pounds  of  mercury  or  quicksilver  are  poured ;  the  surface  of  which  as- 
sumes when  settled,  agreeably  to  the  nature  of  fluids,  an  exact  horizontal 
plane. — To  prevent  the  mercury  from  being  ruffled  or  sigitated  by  the*ac- 
tion  of  the  wind,  a  roof  is  placed  oyer  it,  in  which  are  fixed  two  plates  of 
glass,  the  two  sides  of  each  plate  being  ground  mathematically  plane  and 
parallel  to  one  another: — And,.of  all  artificial  horizons  an  instrument  of 
this  description  is  the  very  best  that.  cai\  be  employed  in  taking  the 
altitudes  of  the  heavenly  bodies. 

Of  the  Use  of  the  Artificial  Horizon;  that  «,  to  observe  the  Altifude  of 
the  Stm,  or  other  Celestial  Object,  with  a  Sextant,  and  an  Artificial 
Horizon. 

In  taking  the  altitude  of  the  sun,  or  other  luminary,  the  observer  is  to 
place  his  artificial  horizon  betwixt  him  and  the  object  selected  for  observa- 
tion ;  and  at  such  a  convenient  distance  as  tp  see  the  image  of  that  object 
reflected  from  the  middle  of  the  quicksilver  as  well  as  the  real  object  in  the 
heavens : — then,  having  screwed  the  plain  tube,,  or  the  natural  telescope 
of  the  sextant  into  its  place  in  the  socket ;  and  placed  one  or  two  of  the 
dark  screens,  according  to  tlie  brightness  of  the  sun,  to  intervene  on  each 
side  of  the  horizon  glass  ;  the  lower  limb  of  the  reflected  image  of  the 
sun,  as  seen  through  the  erect  or  natural  telescope,  is  to  be  brought  into 
contact  with  the  upper  limb  of  the  image  reflected  from  the  artificial  ho- 
rizon : — but,  if  the  altitude  of  the  upper  limb  of  the  object  be  irequired,  it 
must  be  brought  into  contact  ^lith  the  lower  limb  of  the  image  as  seen  in 
the  artificial  horizon. — Now,  the  angle  on  the  arch  of  the  sextant  being 
read  bff,  and  the  index  error,  if  any^  applied  to  it,  the  result  will  be  the 
double  of  the  sun's,  or  other  object's  altitude  above  the  horizontal  plane  : 
to  the  half  of  which,  if  the  object  be  the  sun,  let  the  semi- diameter,  re- 
fraction and  parallax  be  applied,  and  the  true  central  altitude  will  be 
obtained. 

Remarks. 

Since  neither  the  plain  tube,  nor  the  natural  or  erect  telescope  can  be 
depended  upon  in  taking  observations  when  rigorous  exactness  is  required; 


Digitized  by 


Google 


BY  MJEEANS   OF   AN   ARTIFICIAL   HORIZON.  657 

the  inverting  telescope  should^  therefore,  be  invariably  made  use  of  in  all 
cases  where  angles  of  altitude  are  to  be  measured  with  astronomical  preci- 
sion : — and  here,  jperhaps,  it  may  not^  be  unnecessary  to  state  that  when 
the  inverting  telescope  is  used,  the  lower  limb  of  the  sun,  or  moon,  will  ap- 
pear to  be  the  upper  limb,  and  conversely.*— Hence,  in  observing  the  alti- 
tude of  the  lower  limb  of  the  sun  or  moon,  the  apparent  upper  Umb  of  the 
object,  as  'seen  in  the  horizon  glass  through  the  inverting  telescope,  is  to 
be  brought  into  contact  with  the  lower  limb  of  the  image  in  the  artifidal 
ftomon.*— in  this  case  the  reflected  image  in  the  Rrtificial  horizon  will  ap- 
pear to  be  uppermost. — ^Again,  in  observing  the  altitude  of  the  upper  limb 
of  £he  sun  or  moon,  the  apparent  lower  Umb  of  the  object,  as  seen  in  the 
horizon  glass  of  the  sextant  though  the  inverting  telescope,  is  to  be  brought 
into  contact  with  the  upper  Umb  of  the  image  in  the  artificial  horizon  .—in 
this  case  the  reflected  image  in  the  artificial  horizon  will  appear  to  be  un- 
dermost. 

If  an  observer  be  placed  as  remote  from,  or  as  near  to,  an  artificial  hori- 
zon as  possible,  the  rays  of  light  passing  from  the  sun  or  other  celestial 
object  to  his  eyej  and  from  that  object  to  the  surface  of  the  artificial  hori- 
zon, will,  on  account  of  the  immense  distance  of  such  object  from  the 
earth,  be  physically  equal  and  parallel  in  every  respect  to  each  other  :— - 
hence,  it  is-  easy  to  perceive  that  it  is  immaterial  whether  the  artificial  hori- 
zon be  placed  high  or  low,  remote  or  near  with'  respect  to  the  observer, 
provided  he  can  but  ^ee  the  object's  reflected  image  Iherein. 

When  an  angle  of  altitude  is  taken  by  means  of  an  artificial  horizon, 
its  measure  on  the  limb  of  the  sextant  will  always  be  the  double  of  die 
true  value  thereof  above  the  horizontal  plane  ^7-this  will  appear  evident  by 
considering  that  if  a  person  places  himself  at  any  distance  before  a  plane 
mirror,  or  common  looking-glass,  his  reflected  image  will  appear  just  as  far 
behind  such  looking-glass  as  he  is  before  it : — and,  upon  this  simple  prin- 
ciple it  is  that  the  reflected  image  of  the  sun,  or  other  object,  will  appear 
to  be  as.  far  below  the  surface  of  die  artificial  horizon  as  the  real  object  is 
above  it  5— but  since  the  limb  of  the  real  object,  as  reflected  from  the  index 
glass  of  the  sextant,  is  to  be  brought  into  contact  with  that  of  the  image 
apparendy  reflected  below  the  surface  of  the  artificial  horizon,  it  is  there- 
fore manifest  that  die  contained  angle,  as  expressed  on  the  arch  of  the  sex- 
tant, must  be  equal  to  twice  the  measure  of  the.  observed  angle  of  altitude 
above  the  plane  of  the  horizon  :— and  from  this  we  may  readily  perceive 
that  angles  of  altitude  taken  in  the  above  manner  are  not  affected  by 
the  angle  of  horizontal  depression,  commonly  called  '^  the  dip  of  the 
horizon.''  ♦ 

•  See  page  387. 

2u 


Digitized  b' 


GooqIc 


Q^b    -         OF  TAKING  THB   ALTITUDB   OV  A   CBLBSTIAL  OBJECT, 

Now,  the  double  angle  of  altitude  being  thus  obtained,  the  true  altHnde 
of  the  object  is  to  be  deduced  therefrom  in  the  following  manner,  viz.  :-— 

l^r«f  •— To  apply  the  Corrections  when  the  Sun  is  observed  :^* 

Correct  the  observed  angle  for  the  index  error  of  the  sextant,  if  any  ; — to 
the  hsdf  of  which  apply  the  sun's  semi-diameter  by  addition  if  the  lower 
limb  be  observed,  but  by  subtraction  if  it  be  the  upper  limb,  and  the  sun's 
apparent  central  altitude  will  be  obtained ;  from  which  let  the  difference 
between  the  refraction  and  parallax  ^corresponding  thereto  be  subtracted, 
and  the  remainder  will  be  the  -sun's  true  central  altitude. 

Seeond^^To  apply  the  Corrections  when  the  Moon  is  observed  ^— 

Find  the  moon's  apparent  central  altitude  in  the  same  manner  ays  if  it 
were  the  sun  that  was  under  consideration  ;  observing  to  correct  her  eemi- 
diameter  by  the  augmentation  contained  in  Table  IV. ;— then,  to  the  ^- 
parent  altitude,  thus  found,  let  the  correction  in  Table  XVIIL  be  added|y 
and  the  sum  will  be  the  true  altitude  of  the  moon's  centre. 

l^d.— To  apply  the  Corrections  when  a  fixed  Star  is  observed : — 

Correct  the  observed  angle  for  the  index  errOr  of  the  sextant,  if  any; 
from  the  half  of  which  subtract  the  refraction  corresponding  dierelo^  and 
the  remainder  will  be*the. star's  true  altitude. 

EmmpU  U 
Let  the  measure  of  the  observed  angle  between  the  lower  limb  of  the 
sun  reflected  from  the  index  glass  of  a  sextant,  and  the  upper  Kmb  thereof 
reiected  from  an  artificial  horizon  be  103°  14^40^  5  the  index  error  of  the 
sextant  3U0T  additive,  and  the  sun's  semi«diameter  16C18?;  required 
the  true  altitnde  of  the  sun's  centre  } 

Measure  of  the  observed  angle  := ,    •     103?14C40T 

Index  error  = ,    .       +3.40 


Corrected  observed  angle  =  •    ...;....  103?  18^205 

The  half  of  which  is  the  correct  observed  altitude  of 

the  sun's  lower  limb  above  the  plane  of  the  hor«    x:  51?39'  107 

Sun's  semi-diameter  =•••••«    4    •    p    9  +   16*  18 

Apparent  altitude  of  the  sun's  centre  =   •    •    ,    •     •  5I?55^28T 


Refraction  answenng  to  ditto  =  0'A4^.\    ,.^ 

T*^    ti       1.  «  r^    ^    \  diflFerence  = 

P&rtdlax,  ditto  ditto       0.   5    > 


-      0.39 


Sun's  true  central  altitude  =  .    .    .    .    .    ,    .    .,    .    5K54M97 


Digitized  by 


Google 


BT  MEANS  OP  AN   ARTlflCIAL   HORIZON.  659 

Example  2. 

Let  the  measure  of  the  observed  angle  between  the  upper  limb  of  the 
moon  reflected  from  the  index  glass  of  a  sextant  and  the  lower  limb  there- 
of reflected  from  an  artificial  horizon  be  39^50^20?  ;  the  index  error  of  the 
sextant  l',50^  svibtractive,  the  moon's  semi-diameter  14^46^>  and  her 
horizontal  parallax  54' 13?;  required  the  true  altitude  of  the  moon's 
centre? 

Measure  of  the  observed  angle  IS •      39?50'.20'r 

Inde^i  error  ss •    ^    ••  •    •       -*     1.50 

'  ■ ■  ■    ^ 

Correctedobserved  angle  =  .,   ..,•••.,.      39?48'30i: 

The  half  of  which  is  the  correct  observed  altitude  of  the 

moon's  upper  limb  above  the  plane  of  the  horizon  ^     I9?54'  15T 
Moon's  semi-diameter  =:s  .     .     14'46?  )   «  i^   ei 

Augmentation  of  do,  1  ab.  I  v ,  =:  U.    5    j 

Apparent  altitude  of  the  moon's  centre  =      .     •    •    •     19?39^24T 
Correction  of  ditto,  Table  XVIIL  =   .    .    .      •     .    .     +48.25 

Moon's  true  central  altitude  =     .    .    *    .    .    •    •    .     20?27'49r 

Example  3. 

Let  the  measare  of  the  observed  angle  betweeiK  the  centre  of  a  fixed 
star  reflected  from*  the  index  glass  of  a  sextant  and  the  centre  thereof  re* 
fleeted  from  an  artificial  horizon  be  71?  16  C 10?,  and  the  index  error  of  the 
sextant  2 '.30?  subtractive ;  required  the  true  altitude  of  the  star  ? 

Measure  of  the  observed  angle  =s *    71?  16^  10? 

Index  error  as      ••••••••.•.'••     -*     2,30 

Corrected  observed  angle  « 71?13'40? 

The  half  of  which  is  the  correct  observed  altitude  of 

the  star's  centre  above  the. plane  of  the  koriaon  bi  ,     •  95. 36. 50 

llefraetion  conrespondlng  to  ditto  ^      ..••..-^     1. 19 

*■  **  III      ■ 

Tniesltitadeeftlit»tara. 95?S5;31? 

Note.'^'When  the  altitude  of  a  celesUai  object  exceeds. 60  degrees,  it 
cannot  be  taken  by  means  of  a  sextant  and  an  artificial  horizon ;  because, 

2u2 


Digitized  bv 


•  660  OF  TAKING  THJt   ALTITUDB  OF  A  CBLB8TIAL  OBJECT. 

in  this  case,  the  measure  of  the  double  angle  of  altitude  will  exceed  the 
limits  of  the  graduated  arch  of  the  former  instrument. 

Remarks. 

1.  In  observing  equal  altitudes  by  means  of  an  artificial  horizon,  or  in 
taking  a  continued  series  of  altitudes  for  the  purpose  of  determiniDg  the 
error  and  the  rate  of  a  chronometer ;  it  will  be  essentially  necessary  to 
keep  the  same  plane  of  the  glass  roof  of  the  horizon  towards  the  observer 
in  each  observation ;  so  that  in  the  event  of  there  being  any  trifling  defect 
in  the  parallelism  of  the  surfaces  of  the  two  plates  of  polished  glass,  which 
form  the  roof  of  the  horizon,  the  error  arising  therefrom  may  equally 
affect  each  observed  altitude.  To  make  certain  of  alwap  having  the  same 
side  of  the  roof  next  the  observer,  it  will  be  advisable  to  make  a  small 
mark  in  the  wooden  part  thereof':  then,  this  mark  being  kept  towards  the 
observer,  in  every  observation,  the  altitudes  will  thus  be  prevented  from 
being  unequally  afiected  by  any  want  of  parallelism  that  may  chance  to 

be  in  the  phme»  of  the  glass  part  of  the  roof. 

* 

2.— In  calm  weather  the  altitudes  may  be  taken  by  reflection  from  the 
quicksilver  without  making  use  of  the  glass  roof :— in  like  manner  they 
may  be  taken,  during  such  weather,  by  reflection  from  a  bason  of  water  ; 
or,  by  reflection  from  a  cup  of  tar,  treacle,  oil,  or  other  fluid  and  viscous 
substance. 

3.— Mariners  frequently  supply  themselves  with,  what  may  be  termed,  a 
home-made,  or  ship-built  artificial  horizon ;  the  quicksilver  in  which  they 
shelter  under  a  roof  formed  by  two  squares  of  the  thick  glass  with  which 
ships  are  usually  furnished :— this  is,  to  say  the  least  of  it,  a  poor  substi- 
tute :— it  is  vainly  endeavouring  to  accomplish  that,  by  memis  of  a  couple 
of  squares  of  common  glass,  which  can  scarcely  be  effected  by  the  most 
highly-finished  and  parallel  planes  that  can  possibly  be  produced,  by  the 
labour  and  ingenuity  of  the  most  eminent  optician,  or  mathematical  in- 
strument maker  ;-.indeed,  it  is  a  most  absurd  and  mistaken  contrivance  j 
and  a  pertinacity  in  its  use  betrays  an  evident  deficiency  of  usefiil  know- 
ledge on  the  part  Of  the  proprietor :— for,  since  the  surfaces  of  those 
squares  are  not  rendered  mathematically  accurate  by  being  ground 
perfectly  plane  and  parallel  to  one  another,  they  cannot  possibly  refract 
and  reflect  the  rays  of  light  in  an  exact  uniform  manner :— hence,  the 
angle  of  incidence  will  not  be  equal  to  the  angle  of  reflection;  and  thus 
the  angles  of  altitude  observed  in  such  a  mahe-sf^ft  and  defectioe  horizon 
must  and  will  be  always  erroneous. 


Digitized  by 


Google 


A  NSW  MBTHOD  OF  TINBINO  THB  XX>KOrnrDB«  661 

A  NEW  AND  CORRECT  METHOD  OF  FINDING  THE  LONGI- 
TUDE  ON  SHORE,  AND,  IN  SOME  CASES,  AT  SEA; 

Which,  besides  being  remarkably  simple,  will  be  found  equally  as  strict  as 
the  common  Method  by  the  Lunar  Distances,  and,  at  the  same  time, 
considerably  more  practicable  than  that  Method :  for  most  Mariners 
must  be  aware  of  how  extremely  difficult  it  is  to.  take  an  accurate  Lunar 
Observation  at  Sea,  particularly  when  the  Ship  rolls  rapidly,  or  pitches 
with  toy  degree  of  violence ;  but  few  find  any  difficulty  whatever  in 
measuring  the.  Altitude  of  a  Celestial  Object,  provided  the  Horizon  be 
sufficiently  clear;— and,  these  points  betag  premised,  we  will  now 
proceed  to  the  Solution  of  the  following 

Problem. 

Given  the  Latitude  of  a  Place  or  Ship,  the  observed  Altitude  of  the  Moon's 
well-defbied  Limb,  and  the  apparent  Time  of  Observation :  to  find  the 
Longitude  of  that  Place  or  Ship. 

RULB. 

To  the  apparent  tinie  of  observation  (always  reckoned  from  the  preced- 
ing nooii),  add  the  longitude  by  account,  in  time,  if  it  be  west,  or  subtract 
it  if  east,  and  the  sum  or  remainder  will  be  the  estimated  time  at  Green- 
wich ;  to  which  let  the  sun's  right  ascension,  at  the  noon  preceding  the 
Greenwich  time,  be  most  carefully  reduced ;  and  let  the  moon's  declinationj 
at  the  period  preceding  the  Greenwich  time,  be  also  carefully  reduced  to 
the  same  time. . 

To  the  sun's  reduced  right  ascension,  add  the  apparent  time  of  observa- 
tion (rigidly  determined) ;  and  the  sum  (abating  24  hours,  if  necessary,) 
will  be  the  right  ascension  of  the  meridian/  which  convert  into  motion  or 
degrees. 

Reduce  the  observed  altitude  of  that  part  of  the  moon's  well-defined 
limb  which  is  either  nearest  to  or  farthest  from  the  horizon,  according  as 
the  lower  or  upper  limb  may  be  observed,  to  the  true  central  altitude,  by 
Problem  XV.,  page  323.  Then,  with  the  moon's  true  central  altitude, 
thus  found,  her  corrected  declination,  and  the  given  latitude,  compute  her 
angufa&r  distance  from  the  meridian.  Now,  if  the  moon  were  in  the  eastern 
hemisphere  at  the  time  of  observation,  let  her  angular  distance  from  the 
meridian  be  added  to  the  right  ascension  of  the  meridian ;  and  the  sum 
(diminished  by  360  degrees,  should  it  exceed  this  quantity,)  will  be  the 
moon's  correct  right  ascension ;  but  if  she  were  in  the  western  hemisphere, 
her  angular  distance  is  to  be  subtracted  from  the  right  ascension  of  the 
meridian  (increased  by  360  degrees,  if  necessary) :  the  remainder  will  be 
the  moon's  correct  right  ascension  at  the  time  and  place  of  observatiofn. 


Digitized  by 


Google 


662 


A  HSW  ftfBTHO0  OP  riNpiNG  THS  UyKOmTOB. 


With  the  moon's  correct  right  ascension^  thus  found^  enter  page  VI.  of 
the  month  in  the  Nautical  Almanac^  opposite  to  the  given  day,  or  to  that 
which  immediately  precedes  or  follows  it,  and  take  out  the  next  less  and 
the  next  greater  tabular  right  ascensions ;  find  their  difference,  and  find^ 
also^  the  difference  between  the  computed  and  the  next  less  tabular  right 
ascension.  Now,  from  the  proportional  logarithm  of  the  last  difference, 
subtract  the  proportional  logarithm  of  the  first  difference :  the  remainder 
will  be  the  proportional  logarithm  of  a  portion  of  time,  which  being  added 
to  the  hour  standing  over  the  least  tabular  right  ascension^  the  ^m  will 
be  the  apparent  time  at  Greenwich;  the  difference  between  which  and  the 
apparent  time  of  observation,  will  be  the  required  longitude  in  dme; — east, 
if  the  time  at  ship  or  place  be  the  greatest;  otherwise,  west. 

Note.— The  above  rule  is  made  out  on  the  assumption  that  the  moon's 
place  in  right  ascension  and  declination  is  given  in  page  VI.  of  tlie  month 
in  the  Nautical  Almanac,  agreeably  to  the  form  exhibited  in  the  page  at 
the  end  of  this  problem.  It  is  evident  that  this  rule  may  be  very  consi- 
derably contracted,  but  the  author  gives  it  thus  in  detail  with  the  view  of 
making  himself  the  more  clearly  understood, — so  that  the  reader  may  not 
mistake  his  meaning. 

Examipk  1. 

September  8th,  1826^  tn  latitude  40?27'dO?  south,  and  lonptude,  by 
account,  30?40C  west,  at  IMQTl?-  apparent  time,  the  true  altitude  of 
the  moon's  centre,  ea^t  of  the  meridian,  was  31?55^23l';  required  the 
true  longitude  of  the  place  of  observation  ? 


lUutiraiUm. 


In  the  annexed  diagram 
(projected  stereographically 
upon  the  plane  of  the  meri- 
dian^) let  the  primitive  circle 
Z  H  N  O  represent  the  plane 
of  the  meridian  of  the.  place 
of  observation;  SP  the  earth's 
axis ;  £  Q  the  equator ;  H  O 
the  horizon;  and  ZN  the 
prime  vertical,  in  which  Z 
represents  the  zenith.  Let  the 
small  circle  a  a  represent  the 
parallel  6f  the  moon's  alU  tude, 
and  that  expressed  by  6  6  the 
parallel  of  her  declination. 


H 


^> 

\\ 

k^ 

/ 

X 

K      1 

\     / 

^y^ 

w 

\/ 

"^^ 

W 

K 

^ 

y 

N 


Digitized  by 


Google 


A  KBW  urnmoD  or  finding  tbs  lonoituds. 


663 


rPhe  intersection  of  those  small  circles  in  J>  shows  the  moon's  place  in  the 
Heavens  at  the  time  of  observation.    Draw  the  oblique  circles  S  ])  P  and 
Z  })  N.    Then,  in  the  oblique  angled  spherical  triangle  Z  ])  S,  there  are 
^ven  the  three  sides,  to  find  the  hour  angle  Z  S  ]) ;  viz.,  the  side  D  Z  :£■ 
t;he  co-altitude  or  zenith  distance  of  the  moon,  the  side  ])  S  ^  the  moon^s 
cHstance  from  the  elevated  pok  of  the  equator,  and  the  side  S  Z  ss  the  co- 
latitude  of  the  place;  to  find  the  moon's  distance  from  the  meridian 
expressed  by  the  angle  at  S  ;  which  angle  is. easily  determined  by  calcula- 
tion, and  )vhich  being  compared  with  the  right  ascension  of  the  meridian 
(by  addition -or  subtraction,  according  as  the  object  may  have  been  in  the 
eastern  or  western  hemisphere  at  the  time  of  observation,)  the  moon's 
correct  right  ascension  will  thus  be  obtained,  and,  hence,  the  apparent 
time  of  observation  at  Greenwich. 


Ckmputaiwn* 


Apparent  time  of  observation  ss    .     .     .    •    .     i?10?17' 
Longitude  by  account  80?40'  W.,  in  time  =  +2.   2. 40 


Estimated  time  at  Greenwich 


3M2T57: 


Sun's  reduced  right  ascension  «     11^  6?11'.2 
Apparent  time  of  observation   a      1.10.17 

Right  ascension  of  the  meridian  m  12n6?28\2  s  184?7'3? 

Moon's  reduced  declination  s  20?49'3r  south. 

Moon's  tnie  central  altitude  =k  3 1  ?55 '.  23f 

Moon's  south  polar  distance  =  69. 10.67     Log.  co-secant  =  0.029319 

Latitude  of  the  place  of  obs.  =  40. 27. 30    Log.  secant  =  .     0. 1 18685 

Sumsss     . 141^33^50? 


Half  sum  = 70M6^55r  Log.  co-sine  =       9.517413 

Remainder  =     .    .    .    •    .    38.51.32    Log.  sine  =       .    9.797547 


Sum=  19.462964 


Half  the  hour  angk=     .    .    32°36^24r  Log.  sine  =       .    9.731482 
i  '8  angular  dist.  east  of  mend.  65^2^  48? 


Digitized  b' 


GooQle 


DIM  A  NEW  MSTHOD  OP  FINDING  TBB  IjONGmUXB. 

Moon's  angular  dist. E.  of  merid.^?  12M8r 
Right  ascension  of  the  inerid.=  184.   7.   3 

J'sILA.attime&placcofob8.249?19:5iri|^-^,  q^  i;^  k_  ,  ^.^t 
rsRJLperNant.Al.atIIlhrs  249. 11.46  i^"'  ^^  ^^^^-J-^'? 
J's    ditto    ditto      atVIhr5251.   4.53   Jl>iff.l.53.  7  Rlog.  .201/ 

Portion  of  time  = 0M2?52!  Pn^.log.=  iTufiO 

Time  corresponding  to  least  tabular  R.A.=3.   0.   0 

Apparent  time  of  obsenr.  at  Greenwich  =  3M2?52! 
Apparent  time  at  the  place  of  obsenr.  =     1 .  10. 17 

Long,  of  the  place  of  obsenr.,  in  time  =     2?  2?35 !  =  30?38!45f  wtaL 

iVbte.— The  latitude  and  the  altitude  are  made  use  of  directly^  in  the 
above  calculation,  instead  of  their  complements,  so  that  the  hour  a&ele 
may  come  out  a  logarithmic  sine. 

Example  2. 
September  I4th,  1826,  in  latitude  47?12^20r  north,  and  loiigitnd«^  by 
account,  38?47'  east,  at  14*58r38!  apparent  time,  the  tnie  altitude  of 
the  moon's  centre,  west  of  the  meridian,  was  16?24'18r;  rapured  the 
true  longitude  of  the  place  of  observation  ? 

Apparent  time  of  observation  =      ....     1 4 1 58?38 ! 
Longitude  by  account  38M7 '  cast,  in  time  =  —  2. 35.   8 

Estimated  time  at  Greenwich  =       ....     12?23T30! 

Sun's  reduced  right  ascensions  11  ^297  8'. 3 
Apparent  time  of  observations  14.58.38  .0 

Right  ascension  of  the  mend.  ==  2? 27r46'.3  =  36?56^34if 

Moon's  reduced  declination  =  4?24'0f  south. 
Moon's  true  central  altitude  =  16?24 ^Sr 

Moon's  north  polar  distance  =  94. 24.   0    Log.  co-secant  =    0. 001282 
Lat.  of  the  place  of  observ.  =  47. 12. 20    Log.  secant  =     .0, 167S93 

Sum  = 158?  0^38r 


Half  sums 79?  0^9?  Lpg.  co-sine  =s    .    9.280393 

Remainders 62.36.    1     Log.  sine  =    .    .9.948324 


Sum  s  19.397892 


Half  the  hour  angles    •    .    29?59(53r  Log.  sine  s=    ,    .    JJ.  698946 


Digitized  by 


Google 


Pom.-  ■*-XV,|,       , 

^•»?  taken  „,f  eh   °'''''»^^o^  ' 

«»o»„.     '^^"^'^nt  time  of  oi 
^e  will  heft.    .  ' 

^^^  to  eve 
f"**"' centre  r^«^S*2f 


Digitized  by 


Google 

I 


6M  A  NSW  MBTHO0  Of  KNOINO  THB  IX>NGITUDB. 

Moon'8  tnie  central  altitude  =  17*41  UO^ 

Moon's  north  polar  distance  =  76.22.22     Log.  co-secant  =  0. 012401 

Lat  of  the  place  of  observ.  =  50. 4 S.  15     Log.  secant  ^     .    0. 199301 

Sumr= 144?52'.17^ 


Half  sum  = 72? 26 C  SirLog.  co-sine  «B  .    9.479683 

Remainder  = 54«  44. 28^  Log.  sine  =    .    .    9. 91 1984 

Sum:s  19.603371 


Half  the  hour  angle  =      .    .     39^8^  8r  Log.  sine  =  *   .    9.80I6JS5| 

J 's  angular  dist.  west  of  merid.  78?36(  16r 
Right  ascension  of  the  merid.  106.    8. 10§ 

])'8R.A.attime&placeofobs.=27?31'54^^  I  T\-irn«^i  /  jrt#ri>t  /m»^/« 
r.iUV.perNautAL.tXVIII.h.26.50.  9  |l>'ff.0?41.45irPJog.,6346 
Moon's  ditto  ditto  atXXI.h»28.  19. 16     JDiff.1.29.   7    P.log..8a53 

Portion'of time  =     . l?24r20:Prop.log.=.8298 

Time  corresponding  to  least  tabalar  R.  A.  =  18.    0.  0 

Apparent  time  of  observation  at  Greenwich3Ll9i24T20! 
Apparent  time  at  the  place  of  observation  b  19. 20.   0 

Longitude  of  the  place  of  observ.,  in  time  =    0^  4^20:  =  l?5'0r  west; 
which  is  the  correct  longitude  of  the  given  place* 

Remarks. 

In  finding  the  longitude  by  the  proposed  method  (which  is  evidently 
founded  on  the  most  natural  and  unerring  principles,)  the  moon's  ri^t 
ascension  becomes  the  principal  element  in  the  calculation;  aud  since  this 
must  be  deduced  from  her  true  central  altitude,  the  observed  altitude  of 
her  limb  must  be  taken  with  all  imaginable  care  :  for  which  purpose  the 
observation  should  necessarily  be  made  with  a  sextant  or  circular  instru- 
ment, and  the  inverting  telescope  ought  invariably  to  be  used,  particularly 
if  the  altitude  be  observed  at  night  *,  moreover,  \t  is  that  part  of  the  moon's 
round  or  well-defined  limb  (upper  or  lower,  as  the  case  may  be>}  which  is 
perpendicular  to  the  plane  of  the  horizon,  that  must  be  brought  donin^  to 
the  surface  of  the  water  or  sea. 

The  most  favourable  time  for  observing  th«  moon's  altitude,  ao  as  to 
obtain  the  longitude  to  the  greatest  possible  degree  of  exactness,  is  when 
her  change  of  altitude  is  the  quickest ;  and  this  always  happens  when  she 
is  in  or  near  tq  the  prime  vertical ;  that  is,  the  east  or  weH  point  of  the 


Digitized  by 


Google 


horizon.  The  altitude^  however^  should  not  be  less  than  5  degrees^  on 
account  of  the  uncertainty  of  the  atmospherical  refraction  near  the  horizon; 
nor  should  the  object's  distance  from  the  meridian  be  less  than  3  hours^  or 
45  degrees.  • 

Since  an  error  of  1  minute  of  a  degree  in  the  moon's  computed  right 
aaeension  will  have .  the  same  effect  upon  the  deduced  longitude  that  an 
error  of  the  same  value  has  in  the  computed  distance  by  the  method  of 
the  lunar  observations,*  it  is''  essentially  requisite  that  several  altitudes  of 
the  moon's  round  or  well-defined  limb  be  most  carefully  observed,  and  the 
corresponding  apparent  times  per  watch  noted  down  :  the  sums  of  these, 
divided  by  their  number,  will  give  the  mean  altitude  and  the  mean  corre- 
sponding apparent  time.;  which  should  be  depended  upon  in  preference  to 
any  single  observation*  If  there  be  an  assistant  observer  to  take  the  altitude 
of  the  sun,  a  planet,  or  a  fixed  star,  at  the  same  moment  that  the  principal 
observer  takea  the  altitude  of  the  moon's  well-defined  limb,  the  apparent 
timci  may  then*  be  deduced  from  the  altitude  of  such  object ;  and  thus  any 
imperceptible  irregularity  in  the  going  of  the  watch,  since  the  last  time  of 
ascertaining  its  error,  will  be  provided  against. 

In  a  very  rough  sea,  and  when  the  ship  rolls  or  pitches  considerably,  it 
wiH  be  most  highly  advisable  to  multiply  the  observations,  and  to  take  the 
mean  as  the  true  result. 

It  is  to  be  noted,  however,  that  the  sextant  must  be  properly  acyusted, 
07  the  value  of  its  index  error  very  carefully  determined,  by  the  method 
in  page  653 ;  the  instrument  must  then  be  held  in  a  direct  vertical  posi- 
tion, so  that  its  plane,  if  produced,  would  meet  with  an  imaginary  plumb- 
line  passing  throujfh  the  moon's  centre,  and  let  fall  from  that  gart  of 
her  well-defined  limb  which  is  exactly  perpendicular  to  the  plane  of  the 
horizon^  and  which  is  either  the  nearest  to  or  the  most  remote  from  the 
said  plane  s  any  deviation  from  this  position,  either  to  the  right  or  left, 
will  make  the  angle  of  altitude  something  more  than  the  truth. 

As  it  may  be  rather  difficult  to  make  a  perfect  contact  between  that  part 
of  the  moon's  well-defined  limb,  thus  indicated,  and  the  horizon  imme- 
diately under,  when  she  is  so  posited  in  the  heavens  that  an  imaginary 
straight  line  joining  the  cusps  of  her  horns  is  at  right  angles,  or  perpen- 
dicular to  the  plane  of  the  horizon,  (a  position  in  which  she  is  at  times 
during  the  two  or  three  days  preceding  and  following  her  conjunctive 
Sy^ygia,.  or  change,t)  it  will  therefore  be  advisable  to  take  her  altitude  at 

*  This  I  St  a  mean  rate,  will  b«  ftbout  27§  milet  in  places  under  the  equatpr ;  but  since 
the  error  decreases  in  proportion  to  the  co-sine  of  the  latitude,  it  will  only  amount  to  about 
17i  miles  in  tbe  parallel  of  Portsmouth. 

f  The  moon  is  never  exactly  in  this  position,  except  when  she  arrives  at  the  nona^esimal 
degrSe ;  that  it,  the  90th  degree  of  tbe  ecliptic  above  the  borison  i  and  then  she  is  too  near 
to  the  meridian  for  observation. 


668  A  NSW  MSTHOD  OF  FINDING  THB  LONGITUDE. 

some  convenient  moment  before  or  after  the  time  of  her  being  ia  this 
mifavourable  position ;  observing,  however,  that  the  moment  so  chosen  be 
sufficiently  far  from  the  period  of  her  passing  over  the  meridian  of  the 
given  place. 

It  will  appear  very  perceptible,  from  what  has  been  thus  adduced,  that 
the  proposed  method  of  finding  the  longitude  possesses  a  most  decided 
advantage  over  that  by  the  lunar  distances ;  because,  while  most  marinerB 
are  found  competent  to  take  a  very  correct  altitude  of  a  celestial  object, 
few  are  found  sufficiently  qualified  to  m^easure  the  angular  distance  between 
the  moon  and  sun,  or  a  fixed  star,  to  that  degree  of  precision  which  is  so 
indispensably  necessary  to  the  obtaining  of  the  true  longitude ;  particularly 
when  the  object  seen  by  reflection  is  to  the  left-hand  of,  and  considerably 
lower  than,  that  seen  by  direct  vision.  Besides,  there  is,  at  times,  a  very 
considerable  degree  of  uncertainty  attendant  on  the  admeasurement  of  a 
lunar  distance  from  the  sun  or  a  fixed  star ;  for  the  objects  approach  or 
recede  from  each  other  so  very  slowly,  that  the  eye  of  the  observer  is  very 
frequently  deceived,  unless  aided  by  a  high  magnifying  power ;  and  this 
cannot  always  be  used,  when  the  ship  suffers  a  violent  degree  of  agitation : 
hence  it  commonly  happens,  after  making  the  contact  between  the  limbs 
of  the  objects  apparently  perfect,  that,  on  directing  the  sight  to  them 
again,  their  limbs  will  appear  to  be  separated,  or,  perhaps,  entered  upon 
each  other ;  and  this  separation,  or  entering,  in  direct  opposition  to  the 
absolute  motion  of  the  moon.  But  if  the  horizon  be  clear,  and  the  moon 
at  a  proper  distance  from  the  meridia$i^  all  uncertainties  vanish  in  bringing 
her  well-defined  limb  in  contact  with  the  visible  horizon  expressed  by  the 
convex  surface  of  the  sea  j  because  she  then  rises  or  falls  so]  very  rapidly^ 
that  a  careful  observer  may  take  the  altitude  of  her  limb  to  the  sixth  part 
of  a  minute ;  and  this  is  a  degree  of  exactness  that  can  very  rarely,  if  at 
all,  be  introduced  into  the  measured  lunar  distances  taken  at  sea. 

Though  what  has  been  said  here  is  strictly  true  in  theory,  yet  it  may  often 
fail  in'practice,  owing  to  the  uncertainty  of  the  sea  horizon:  hence  it  is 
evident  that  the  method  in  question  is  onit/  properly  adapted  to  the  deter'- 
mination  of  the  hiigU^tde  of  places  on  shore,  where  the  altitudes  can*  be 
correctly  taken  by  mearu  of  an  artificial  horizon;  and,  certainly,  .with  this 
view,  it  will  be  found  to  be  one  of  the  very  best  methods  for  settling  the 
true  positions  of  places  inland  or  along  the  coast. 

Were  the  moon's  place  in  right  ascension  and  declination  computed  in 
the  Nautical  Almanac  agreeably  to  the  form  in  page  670,  it  would  very 
considerably  facilitate  the  finding  of  the  longitude  by  the  metiiod  now 
under  consideration ;  for  then  the  mariner  would  be  provided  with  all  the 
necessary  elements  that  enter  into  the.  calculation,  with  the  bare  exception 
of  the  moon's  correct  central  altitude  and  the  apparent  time ;  but  these,  it 
is  supposed,  his  own  diligence  will  always  furmsh.    Moreover,  the  loxigi« 


Digitized  by 


Google 


^ude  could  then  be  as  readily  inferred  from  an  altitude  of  the  moon's  limb 
and  the  corresponding  time  indicated  by  a  chronometer,  as  it  is  now  by 
that  of  the  sun  and  the  same  delicate  piece  of  mechanism. 

Note. — Page  670  shows  the  manner  in  which  the  moon's  right  ascension 
and  declination  should  be  computed  and  arranged,  so  as  to  answer  the 
intention  of  the  present  problem ;  these  right  ascensions  and  declinations 
may  be  readily  deduced  from  the  elements  given,  at  noon  and  midnight, 
in  page* VI.  of  the  month  in  the  Nauticfd  Almanac,  by  means  of  the 
problem  for  illustrating  the  use  of  Table  XVIL,  page  34.  Thus,  to  find 
the  moon's  right  ascension  at  III.,  VI.,  and  IX.  hours,  September  1st,  1826: 

First        Second  Mean 
Diff.  Diflf.    2dDiff. 


Moon'aR.A.atmidnt.,Aug.31st,142M3a2r  [5032'  4^ 

Moon's  do^at  noon,  Sept.  1st,  =  149.  15.  16  1  (  0. 38  . 

•  '^  J6. 32  42  fl'DiT 

Moon's  do.  at  midnt.,  Sept.  l8t,=  155;  47. 58  ^    *      '        i  1. 23  ^ 

^  >  6«  34.   5    ) 

Moon's  do.  at  noon,  Sept.  2d,  =  162.  22.  3  3 

The  variation  of  right  ascension  betwee*n  noon  and  midnight  of  the 
given  day,  is  6?32C42T ;  the  proportional  part  of  which  corresponding  to 

III.  hours,  is .     1^38: 10^  5 

Equation  of  second  difference  ==       •     .  ^  5.    6 


Proportional  part  corrected  =     •    •    •     1?38^  4^.9 
Moon's  R,  A.  at  noon,  1st  September,  =»  149.  1 5.  16 

Moon's  R.  A.  at  III.  hours  =  .    .     .      150?53 '.  20^.  9 

The  propor.  part  qf  6?32M2r  corresponding  to  VI.  hours,  is  3?16:2K.  0 
Equation  of  second  difference  =       .^    ......    •  —  ?•    5 

Proportional  part  corrected  = 3?16'13''.  5 

Moon's  R.  A.  at  noon,  1st  September,  = 149. 15.  16 

Moon's  R- A.  at  VI.  bourse 152?3^.29^5 

The  propor.  part  of  6?32M2f  corresponding  to  IX.  hours,  is  4?54^3ir.  5 
Equation  of  second  difference  = ^5.6 

Proporticmal  part  corrected  = 4?54^25''.9 

Moon's  R,  A.  at  noon,  1st  September,  s 149.15.16 


Moon's  R. A.  at  IX.  hours  = 154?  9'41\9 

The' moon's  declination  is  to  be  determined  in  the  same  manner  pre- 
cisely ;  observing,  however,  to  apply  the  corrected  proportional  part  by 
addition  or*subtraction,  according  as  the  declination  at  the  preceding  noon 
OT  midnight  is  increasing  or  decreasing. 


t& 


217. 
232. 
247. 
262, 
277. 


291 
305. 
318. 
330. 
342. 

6 

17. 
29. 
41. 


NoOD,orO*       III.* 


U9«l&'lb' 
162.22.  3 
175.38.  2 
189. 10.4:1 
203.   6.46 


19.26 
16.25 
18.36 
21.17 
7.59 


219.19.  6 
234.  8.38 
249.11.46 
264.13.20 
278. 56. 58 


25.14 
5.50 
9.  8 
39.44 
45.13 
14.27 
16.18 
58.48 
48.34 
50.19 


I.  9.53 
306.45.43 
319.44.36 
332.11.36 
344.  14.  34 


150»53'2r 
164.  0.53 
177.18.33 
190. 53. 52 
204.  53.   8 


356.   2.24 

7. 43.  .55 

19. 26. 57 

31.18.   4 

43.21.31 


VI.* 


I52«31'30' 
165.39.53 
178.59.22 
192.  .37. 22 
206. 39. 56 


IX.* 


221.  9.  8 
236.  1.  3 
251.  4.53 
266.  5.  4 
280. 45. 27 


222.59. 
237.  53. 
252.57. 
267.56. 
282. 33. 


294.53.56 
308.25.  2 
321. 19. 35 
333.43.  7 
345. 43. 42 


357.30.16 

9.11.35 

20.  55.  17 

32.  47. 45 

44.  .^2. 57 


154°  9' 
167. 19. 
180.40. 
194.21. 
208. 27. 


42* 

4 
28 
16 

8 


168.58.25 
182.21.51 
196.  5.32 
210.14.46 


224.50.17 
239. 46. 26 
254. 50.  56 
269.47.36 
284.20.54 


296.37. 
310.  3. 
322. 54. 
335. 14. 
347. 12. 


358.58. 
10.39. 
22.23. 
34. 17. 
46.24. 


XII.* 


157«26'  19" 
170.37.58 
184.   3.34 
197.50.13 
212.    2.50 


296. 20. 18 
311.41.57 
324.28.  6 
336.45.  4 
348.41.18 


0. 25. 46 
12.  7.  1 
23.52.26 
35. 47. 45 
47.56.30 


XV.*        XVIII.» 


2.^6.4l.2.i 
241.39.22 
256.43.48 
271.38.20 
f.   7.49 


\b^  A' AT" 
172.  17. 46 
185.  45.  37 
199.35.19 
213.  51.  18 


228.32.47 
243.  32. 22 
258. 36. 29 
273.28.38 
287.  54. 1 1 


300.  2.34 
313.19.33 
326.  1.39 
338.15.38 

350.    9.49 


3OI.44.T5 
314.56.37 
327.34.46 
339.45.43 

351.38.10 


XXI 

173.57.4: 

187.28. 
2f»1.20.5(> 
215.48.  lu 


i30.24.2: 
245.25.2: 
260.28,^.^ 
275.18.31 
289.39.59 


9113.25.20 
316.33.  t 

329.   7.38 
1.15.; 

6.23 


1.53.26 
13.34.50 
25.21.12 
37.18.  4 
49.28.39 


3.21.  4 
15.  2.44 
26.50.  9 
38.48.36 
51.  0.59 


I341I] 
J353. 


4.441.42 

16.30.43 
28.  19. 16 
40.19.21 
52.33.35 


8i.    2.   7 

77.42.53 

90.32L   4 

103.25.27 

116.19.45 


54. 
^. 
79. 
92. 
105. 

117, 
130. 
143. 
156. 
17.0. 


6.23 

36.37 

18.40 

8.35 

2.14 


55.39.25 
68.11.18 
80.54.S4 
93. 45. 10 
106.39.    I 


57.12.41 
69.46.  9 
82. 30. 34 
95.21.48 
108. 15. 49 


58.  46. 
71.21. 
84.  6. 
96.58. 
109.52. 


56.31 
50.42 
46.47 
49.38 
6.16 


119.33.17 
132. 27. 32 
145.24.  8 
158.28.17 
171.47.12 


121.10.  2 
134.  4.23 
147.  1.37 
160.  7.11 
173.28.31 


J22. 46. 
1135.41. 
i48.39. 
161.46. 
175.  10. 


60. 

72. 

85. 

98. 
IIL 
124. 
137. 
150. 
163. 
176. 


19.50 
56.23 
42.54 
35.11 
29.24 


61.53.44 

74.31.45 

87.  19. 13 

100.11.55 

113.  6.11 


,  63.27.50 
I  76.  7.15 
I  88.55.36 
1101.48. 40 
114.42.58 


23.34 
18.15 
16.58 
25.44 
52.14 


126.  0.20 
138.  55. 16 
151.54.51 
165.  5.24 
178.34.41 


127.37.  6 
140.32.21 
153.32.56 
•  166.45.23 
180.17.34 


129.13.54 
142.  9.32 
155.11.11 
168.25.40 
1S2.   0.52 


The  Moon's  Declination,  September,  1826. 


Noon,orO*       111.* 


70  10' 
2.16. 
2.50. 
7.50. 
12.25. 


16. 16. 

19.  6. 
20.42. 
21.   0, 

20.  0. 


0"N 

2n. 

17  8, 

39 
42 

26 

9 

5 
23 
32 


17.51, 
14.44. 
10.55. 

6.37. 

2.  7. 


5 

25 

4 

56  s. 

8  s. 


2.24. 

6.45. 
10.45, 
14. 15. 
17.    7, 


a7w 

18 
13 
13 
24 


19.15. 
20.32. 
20.55. 
20.21. 
18.49. 


16.22. 
1.3.   2. 

8.58. 

4.18. 

0.4.3. 


0 

23 
20 
18 

3 
"10 
41 

1 

2n 

.508. 


6«34'34"n 
1.38.  3n 
3.28.348. 
8.26.50 
12.57.21 


16.41.18 
19.22.26 
20.48.40 
20.57.  1 
19.47.56 


17.  5.  9 
19.37.32 
20.54.  6 
20.52.28 
19.34.16 


17.30.34 
14. 17.  46 
10.24.   8 
6.  4.32  s. 

1.32.  .58  s. 


2.57.53N. 

7.16.38 
11.13. 17 
14.38.i^8 
17.25.55 


19.27.31 
20.38.18 
20.  53. 49 
20.  13.  I 
18.3:1.45 


15.59..'>U 
12.34.23 
8.24.42 
3.41.  IOn. 
1.22.198. 


VI.* 


5058' 45"n 
0.59.56N 
4.   6.416. 
9.   2.33 
13.28.14 


17.27.59 
19.51.25 
20.58.23 
20.46.43 
19. 19.  .34 


17.   9.13 
13.50.30 
9.52.49 
5.30.59  8, 

0. 58. 50  s. 


3.31.    6n 
7.47.36 
11.40.50 
15.   2.    5 
17.43.44 


19.39.14 
20  43.21 
20.51.40 
20.  3.49 
18.17.35 


15. 3b.  58 
12.    5.25 
7. 50. 52 
3.   4.    On. 
2.  0.488. 


IX.» 


5»ir2'33"w 
0.21.4111 

4.44.388. 
9.37.48 
13.58.22 


16.47.   2 
13.22.37 
9.21.  8 
4.57.168 
0.24.45  8. 


4.   4.   6n. 

8.18.12 
12,  7.54 
15.24.36 
18.  0.48 


19.50.  9 
20.47.32 
20.48.54 
19.53.42 
18.   0.35 


15.13.10 
11.3.5.46 
7.16.32 
2.26.34N. 
2.  .39. 17  8, 


XII.* 


4045' 38"N 
0.16.418. 
5.22.258. 
10.12.34 
14.27.43 


49 
7 
31 
47 
49 


1 

8 

4 

24  s, 
19n. 
54n. 
27 
28 
29 
10 


16 
52 
31 
40 

45_ 

.35 
27 
41 

5lN. 

46  ft. 


XV.* 


4«  8'37"N 
0.55.   38, 
5.59.578. 
10.46.47 
14.56.14 


1».  10.  34 
20.15.25 
21.  1.19 
20.31.40 

^8^l7.  3 
i6.~  0. 12 

12.25.   6 
8.16.41 
3.49.25  s. 
0.43.  17n. 


5.   9.26N. 

9. 18. 18 
13.  0.30 
16.  7.43 
18.33.   0 


20.  9.33 
20.59.29 
20.40.49 
19.30.34 
17.23.53 


14.28.13 
10.34.29 
6.   6.22 
1.10.54N. 
.3.56.118. 


xvm.* 


XXI.* 


353F2PN    »i3tAS^s 
1.33.27  8.   2.11.51s. 


6.37.10  s. 
11.20.22 
15.23.52 


18.30.12 
20.25.30 
21.  1.  4 
20.22.35 

18.29.31 


18.48.44 
20.34.24 
21.  0.45 
20.12.  2 
18.10.41 


15.35.40 
11.55.35 
7.44.   2 
3.15.238. 
1.17.   8n 


6. 18. 41 N. 

9.47.42 
13.25.58 
16.28.17 
18.47.37 


20.18.  0 
20.58.38 
20.35.13 
19.17.35 
17.   4.10 


13.5*.  i 
10.  2.56 
5.30.40 
0.32.48N 
4.34.278. 


7.14,   4  s. 
11.53.21 
15.50.36 


15.10.24 
11.25.34 
7.11.   7 
2.41.17$. 
I.50.51N. 


6.13.38N. 

9.56.41 
13.50.52 
16.48.11 
19.  2.   I 


20.25.37 
L  54.36 
20.28.52 
19.  3.45 
16.43.35 

13. 30. 16 
9.30.46 
4.54.33 
0.  S.26S. 
5.1231>. 


Tbe  author  is  of  opmioB,-  that,  for  the  purpose  of  determinijig  the  true  longitude  of  plans  oq 
shore,  the  method  in  question  will  be  found  prefer»ble  to  any  otbWf  pwlicolarly  wbtn  Hk  altlMes 
are  taken  by  ineann  of  an  artificial  horizon. 


Digitized  by 


Google 


J.  u  jneauce  ine  luoon  9  nxgn^  JMMcetwvan  ona  x/rQimono/i  \q  any  gwen 
Time  under  a  known  Meridian. 

RuLB.— To  the  apparent  time  of  observatioa  (always  reckoned  from  the 
preceding  noon,)  add  the  longitude,  in  time,  if  it  be  west,  but  subtract  it 
if  east ;  the  sum,  or  remainder  will  be  the  time  at  Greenwich. 

With  this  time  enter  page  VL  of  the  month  in  the  Nautical  Almanac, 
opposite  to  the  given  day,  or  to  that  which  immediately  precedes  or  follows 
it,  and  take  out  the  right  ascensions  and  declinations  at  Uie  periods  directly 
preceding  aqd  following  the  said  time  $  find  their  difference,  and,  also,  the 
difference  between  the  said  or  Greenwich  time  and  the  preceding  period :  then. 

To  the  proportional  logarithm  of  the  last  difference,  add  the  propor- 
tional logarithm  of  the  first  difference,  and  the  sum  will  be  the  propor- 
tional, logarithm  of  a  correction  which  is  always  to.be  added  to  the  right 
ascension  at  thft  preceding  period ;  but»  to  be  applied  by  addition,  or  sub- 
traction to  the  declination  at  that  period,  according  as  it  may  be  increas- 
ing or  decreasing. — And,  thus,  the  right  ascension  and  declination  will  be 
obtained,  sufficiently  near  the  truth  for  all  nautical  operations,  indepen- 
dent of  interpolations,  or  second  differences. 

This  rule  is  founded  on  the  assumption  that  the  moon's  right  ascension  and 
declination  are  given  in  theNautical  Almanac  agreeably  to  the  form  in  page670. 

Examfie. 

Required  the  moon's  right  ascension  and  declination,  Sept.  18th,  1826^ 
^t  19^20?0!  apparent  time,  in  long.  1  ?5  ^  W.  of  the  meridian  of  Greenwich  ? 

Apparent  time 19*20?  0! 

Longitude  1?5^  west,  in  time  =     .     .    .     +     4. 20 

Greenwich  time 19*24720! 

To  find  the  Moon's  Right  A/M:ension  1 — 

Ap.timeatGr.=  19*24'r20'.  )  difference  =  .    .     1»24T20:  P.L.x=.3293 

D  's RA.Sep.l8,  at  IShours  5  =26?50^  9T.  j.^^    ^039^  7^  p. L.=. 3058 

]>  's  ditto  ditto  at  21  hours  s      28. 19. 16  )  .. . 

Correction  of  right  ascension  =  .   .    ..    .    .     .    0^4 1  US ^  P,  L.= .  6346 

R.A.  at  the  period  preced.  the  Greenwich  times  26.50.   9 

Moon'^  correct  R.A.  at  time  and  place  of  obs.  =  27?31  '541^ 

To  find  the  Moon's  Declination : — 
Ap.timeat  A.=  19*24?20!  1  difference  »  .     .     1*24*20!  P.L.=.S293 

D'8dec.Sep.l8,atl8hoursJ  =13?25!58rN.|^^Q024/54i^  PL-  8591 
D'sdo.        do.    at  21  hours  «  13.50.52  N.i 

Correction  of  declination  = +  0?ll!40^P.L.=  l.  1884 

Declln.  at.  period  preced.  the  Greenwich  times    13.  25. 58  north. 

Moon's  correct  dec.  at  time  and  place  of  obs.  s  IS'^S? "SST  north. 


vr    rixvj/tnv 


J<1J>     IjAllAUATASy 


Hence,  it  is  evident,  that  if  the  moon's  right  ascension  and  declination 
be  given  in  the  Nautical  Almanac  to  every  third  hour,  the  reduction  of 
those  elements,  to  a  given  time  under  a  known  meridian,  will  become  a 
more  simple  operation  than  the  reduction  of  the  sun's  right  ascension  and 
declination  to  a  given  time  and  place. 


SOLUTION  OF  PROBLEMS  relative  to  finding  the  Latitudes  and 
Longitudes,  Right  Ascensions  and  Declinations  of  the  Heavenly  Bodies, 
and  to  the  computing  of  the  Central  Distances  between  the  fdoon  and 
Sun,  a  fixed  Star,  or  a  Planet 

Problem  L 

Given  the  Bight  Ascension  and  Declinaiion  of  a  Celestial  Objed,  tojimd 
iU  LaHiude  and  Longitude. 

Example. 

The  apparent  right  ascension  of  a  Arietis,  August  1st.  18^5,  was 
1*57*23M76,  audits  declination  22938^4-'. 44  north;  required  its 
apparent  latitude  and  longitude,  the  obliquity  of  the  eclipdc  being 
23?27^42-'.875? 

a  Arietis,  right  ascen,  =  1*57^23',  176,  in  degrees  =  29?20'.47'.  64, 
and  north  polar  distance  =  67^21 155 ""•  56. 

Construction. 

Describe  the  pri- 
mitive circle  PESQ, 
with  the  chord  of 
60?  on  the  plane  of  ^ 
the  solstitial  colure : 
— draw  the  Equator 
EQ,  and,  at  right 
angles  thereto,'  the 
axis  PS;  in  which 
P,  represents  the  N. 
celestial  pole. 
Make  £  a,  Q  £  = 
22?38U-'.44,  the 
star's  north  declin. 
adon ;  and  with  the 
tangent  of  its  com- 
plement, viz.  67?21  C55''.  56,  describe  the  parallel  of  declination  a  £.— Take 


Digitized  by 


Google 


.  OF  THB  HBAVBNLY  BODIES.  673 

the  star's  right  ascension^  viz.,  29?20U7'^.  64^  in  the  compasses,  from  the 
scale  of  semi-tangents,  and  lay  it  off  on  the  Equator  from  <y>  to  R,  and  with 
the  secant  of  its  complement,  viz.  60?39' 12*'.  36,  describe  the  circle  of 
right  ascension  PRS;  the  intersection  of  which  with  the  parallel  circle 
a  by  at  9|c ,  will  be  the  apparent  place  of  the  star  in  the  heaveQs.-^Make 
P  ]V,  S  0=s  23?27  -42".  875  the  obliquity  of  the  ecliptic  :-.draw  the  polar 
line  NO,  and,  at  right  angles  thereto,  the  ecliptic  line  yf  <r  ®  ' — ^through 
the  intersecting  point  i^,  draw  the  circle  of  longitude  N  ^te  O,  cutting  the 
ecliptic  in  L; — then,  the  arc  t  L>  will  be  the  longitude  of  the  star,  and 
the  arc  L  9|e ,  its  latitude )  the  former  being  taken  in  the  compasses,  and 
applied  to  the  scale  of  semi-tangents,  will  be  found  to  measure  about  35{ 
degrees : — the  angle  FN*  (measured  by  the  arc  L  o  =  54}  degrees,) 
represents  the  co-longitude  of  the  star,  and  the  arc  N  ^  its  co-latitude; 
the  latter  being  reduced  to  the  primitive  circle  will  be  found  to  measure 
about  80  degrees. 

Now,  in  the  oblique  angled  spherical  triangle  FN  3|e,  given  the  side 
PN  =  23?27M2''.875,  the  obliquity  of  the  ecliptic ;  the  side  F  *  = 
67?21 '  55^.  56,  the  star's  north  polar  distance,  and  the  included  angle  N  F 
4c  ==  1 19?20<47^.  24 ;  tofind  the  sideN  i^  =  the  co-latitude  of  the  star, 
or  its  distance  from  the  north  pole  of  the  ecfiptic,  and  the  angle  FN  i|c  :±= 
its  co*longitude. 

Note, — ^The  circle  of  right  ascension  which  passes  through  yp,  viz.  PQS, 
is  always  equal  to,  or  expressed  by  270  degrees ;  and  that  which  passes 
through  npj  or  atv,  by  0,  or  360  degrees  ;  the  difference,  therefore,  between 
yf  and  nn,  or,  which  is  the  same,  between  Q  and  <r ,  Is  90  degrees ;  which 
being  added  to  the  arc  v  R,  29?20'47''.  64,  makes  the  whole  arc  Q  R= 
1 19?20'47''.  64,  which  is  the  true  measure  of  the  angle  R  F  Q ;  that  is, 
the  angle  N  P  :|c ,  comprehended  between  the  two  given  sides. 

Hence,  by  spherical  trigonometry.  Problem  IIL,  Remark  1,  page  203^ 

An.NP*=sll9?20M7''.64^-2=59?40^23^82.tw.L.sin.  19. 8721827-68 
Side  FN  =  obi.  of  the  ecliptic  =  23.27.42  .875  L.sin.  9.6000350.88 
SideP  :|c  =  star's  N. polar dis.=  67.21.55  .56      L.8in.  9.9651914.48 


Sum=  39.4374093.04 


Difference  of  the  two  sides  =  .    .  43?54n2^685  Hf.S.  19.7187046.52 

2x 

Digitized  by  VjOOQ  IC 


Diff.  of  the  two  8ide8=43?54'.  12".  685  Half  S.  of  logs.  19. 7187046. 52  + 


Half  difference  .    .    21?57'-  6''.3425  Log.  aine  »      9.5726693. 15 


Archss    ,    ,    .    ,    54?27'.23''.53      Log.tang.=s    10.1460353.37 


I 

\ 


Log,  sine  of  arch^  subtract  from  half  sum  of  logs.  =?  9. 9104508. 95  — 
Half  the  side  P  *  =  40»  I'.  14".  26  =  Log.  sine  =      9. 8082537. 57 

Die  whole  side  P  4^  sSO^'.  2 '.  28''.  52 ;  which  is  the  co^latitude  of  the gtren 
star,  or  its  distance  from  the  north  pole  of  the  ecliptic;  heuce  the 
latitude  of  a  Arietis  is  9?57 '31''.  48  north. 

Now,  in  the  oblique  angled  spherical  triangle  PN  4k  >  the  three  sides  aie 
given,  and  the  angle  P ;  to  find  the  angle  N  =::  the  co-longitude  of  the 
star.— Hence,  by  spherical  trigonometry,  Problem  I.,  page  198, 

As  the  side  N  *  =  .  80?  2128^^.52  Log.  co-secant  =  10.0065934.92 
Is  to  the  ang.NP  4k  »  119*20.47  .64  Log.sine«=  .  .  9.9403527.32 
SoistheudeP^s  »   67.21.55  .56  Log:  sines  •    .    9.9651914.48 

TotheanglePN  4c=::54?46nr.S8  Log.  sine  a  .  .  9.9121376.72 
The  angle  N,  thus  foiind»54?46;  1 1 ".  38,  which  is  the  co-longitude  of 
the  star,  and  which  is  measured  by  the  arc  of  the  ecliptic  L  o,  being  taken 
from  90  degrees ;  that  is,  from  <r  ®,  leaves  the  arc  cy»  L=35?13'48'.  62 ; 
which,  therefore,  is  the  apparent  .longitude  of  the  star  a  Arietis. — Hence, 
the  apparent  longitude  of  the  givenstar  is  1  'S't  ISMS'".  62j  and  its  apparent 
latitude  9?57'3r.48  north. 
Now,  from  the  above  Problem  we  obtain  the  following 

General  Rule 
for  computing  the  latitude  and  longitude  of  a  celestial  object,  vis» 

Find  the  difference  between  the  object's  right  ascension,  expressed  in  de- 
grees, and  270  degrees  3  ^ —  to  twice  the  logarithmic  sine  of  half  this  diff- 
erence, add  the  logarithmic  sines  of  the  object's  north  polar  distance,  and 
of  the  obliquity  of  the  ecliptic  :  from  half  the  sum  of  these  three  logarithms 
subtract  the  logarithmic  sine  of  half  the  difference  between  the  object's 
north  polar  distance  and  the  obliquity  of  the  ecliptic  ;  the  remainder  will 
be  the  logarithmic  tangent  of  an  arch,  the  logarithmic  sine  of  which  being 
subtracted  from  the  half  sum  of  the  three  logarithms  will  leave  the  logar- 
ithmic sine  of  an  arc,  which  being  doubled  will  give  the  object's  distance 
from  the  north  pole  of  the  ecliptic  )  the  difference  between  which  and  90 
degrees  will  be  the  latitude  of  the  object,  which  will  be  north  when  the 
ecliptic  polar  distance  is  the  least  5  otherwise,. south. 

•  In  all  caseSy  whenever  this  difference  exceeds  180  degreei,  it  must  be  subuacled  fimn 
360  degrees. 


Digitized  by 


Google 


OF  THB   HBAVSNLY  BODIES.  6/5 

To  find  the  Longitude  : — 

To  the  logarithmic  secant  of  the  object's  latitude^  add  the  logarithmic 
sine  of  the  difference  between  its  righl  ascension  and  270  degrees^*  and 
the  logarithmic  sine  of  its  north  polar  distance ;  the  sum  of  these  three 
logu-ithms,  abating  20  in  the  index^  will  be  the  logarithmic  sine  of  an  archi 
virhich  being  subtracted  from  90  degrees,  will  leave  the  object's  true  longi- 
tude when  its  right  ascensio|i  is  less  than  6  hours  or  90  degrees ;  but  which 
is  to  be  increased  by  6  signs  when  the  right  ascension  b  between  12  and 
18  hours^  that  is,  between  180  and  270  degrees. 

Again. — If  tlie  right  ascension  lies  between  6  and  12  hours;  that  is,  be- 
tween 90  and  180  degrees,  the  arch,  so  found,  is  to  be  augmented  by  3 
sig^ns ;  but  if  the  right  ascensioq  is  belweeii  18  and  24  hours,  viz.  between 
270  and  360  degrees,  it  is  to  be  augniented  by  9  signs ;  in  either  case 
the  true  longitude  of  the  object  will  be  obtained. 

Example  I. 

The  apparent  right  ascehsioh  of  Aldebaran,  August  Srd.,  1825,  was 
4^  25T56' .  1 15,and  its  declination  16?9'5''.35  north;  required  k» apparent 
latitude  and  longitude,  the  obliquity  of  the  ecliptic  being  23?27'42'[.  875? 
Aldeb's.RJV.=4*25T56M15=66?29^   1^.725 


Diff.  between  R.A.and  270?  =^  203?80'.58'^,  275 


And  360? -.a03?301 58^275=  156929C  1^725 


The  half  of  which  is  ==  .  .  .  78?14'*80".  862  Tw.L.»in.l9.d8l580 
Aldebaran's.N.  polar  distance  =f  73.50.  54  .650  Log.  sii).=:fl.fi8.25.U 
Obliquity  of  ecliptic  =  ...     23. 27. 42  .  875  Log.  sin.=9. 600035 


Sum=    39.564126 


Diff.  bet.  P.  dist.  and  ob.  of  eclip.=50?  23  H  P.  775  Half  S.=  19.  782068  + 

Half  ditto  =x .    25. 11.85  .887  Log.  sin.   9.629077 

Arch=: 54?53C2ir     Log.  teng.=  10. 152986 

Log.  fiine  of  drch,  subtract  from  .  ' 

balfsumoflogs.   =  .    .-......•,,     9.912775- 

Arc=    .     • 47?44<22'\5  Log.  sine  =  9.869288 

Twice  the  arc  =  .  .  .  .  .  95^28  f  45'.  0;  which  is  Aldebaran's  dis- 
tance from  the  north  pole  of  the  ecliptic  j  its  latitude,  therefore,  is 
5^28'.45r  south. 

tm^i^^m^^mmm  I'^p  m        m  n  i  n    man      ■  i  ■  ■  ■— *  ■'■;  n  ■  .         -       ;  -ii^       ■■■!  imi    n  i  h     i     ii     i    i  i  i  ^  ' 

*  See  Note,  page  674. 
2x2 


676  OF  FINDING  THE  LATJTUBBS^   &C. 

To  find  Aldebaran's  Longitude :— - 

Lat.  of  Aldebaran  =  .  .  .  .  5?28U5r  S.  Log.  8ec.= 10. 001989 
Diff.  bet\v.  R.  A.  and  270  deg.=  156.  29.  1  .  725  Log.  8ine=  9. 600982 
Aldebaran'fl  N.  polar  distance  =  73. 50. 54  •  65    Log.  sine  =  9. 9825 1 1 


Aldebaran's  co-longitude  =  .    .  22?38M1 ' .  6      l«g.  sine  =  9. 585482 


Aldebaran's  longitude  =s     .    .    67?2la8^4  5  or2!7?2ia8^4. 


Example  2 

The  apparent  right  ascension  of  Pollux,  September  3rd.,  1825,  was 
7*34r38'.74,  and  its  declination  28?26n9^.48  north;  required  it» 
apparent  latitude  and  longitude^  the  dbliquity  of  the  ecliptic  being 
23?27M2^875? 

Pollux's  R.A.=7*34r38'.  74=113?39M1^  1 


DifiF.bet.  R.A.and270?  s   .  156?20:l8^9 


The  half  of  which  is  =  ...  789 10^  9^.  45,  Tw.  L.S.  19. 981350 
Pollux's  N.  polar  distance  =  .  61 .  33. 40  .  520  L.  sin.  =  9. 944150 
Obliquity  of  ecliptic  s  .     .    .    23. 27. 42  .  875  L.  sine  =  9. 600035 


Sum  =  .  39. 525535 


Difr.bet.P.dis,andob.of  ecl.=     389  5(57^.645  Half  sum  19. 762767  i+ 


Half  dittos     ......     199  2C58^  822  Log.  sine  9. 513733  J 


Arch= 60935(4ir     L.  tang.  =  10.249034 


Log.  sine  of  arch,  subtract  from 

half  sum  of  Logs.  = 9.940102  — 


Arcs        .......    4l939^50^5Log.si^c=  9. 822665  i 


Twice  the  arc 839 1 9U P.  0;  which  is  Pollux's  distance 

from  the  north  pole  of  the  ecliptic : — hence,  its  latitude  is  6940'  19.' 
north. 


Digitized  by 


Google 


y 


To  find  Pollux's  Longitude  : — 

Latitude  of  Pollux  =  •  .  .  6^40^  19r  N.  Log.  secant  =  10.  00295 1 
Diff.  bet.  R.  A.  and  270  deg.=  156.  20.  18  .9  Log.  sine  =  .  .  9.  p03503 
Pollux's  north  polar  distances  61.33.40  .52  Log.  sine  =  •  .  9.944150 


Arch  =  . 0!20?48U4^2    Log,  sine  =..  9.  550604 

Add     .     ..,•..    3.   0.   0.   0  .0. 


Sum  = 3  !20'? 48  U4^  2;  which  is  the  true  longitude 

of  Politic.— Hence,  the  latitude  of  Pollux  is  6?40'19f  north,  and  its  lon- 
gitude 3 !  20?48 1 44 ''.  2,  as  required. 

In  like  manner  may  the  latitudes  and  longitudes  of  the  moon  and  planets, 
be  deduced  from  their  respective  right  ascensions  and  declinations. 


Problem  IL 

Qivmi  the  Latitude  and  LmgUude  of  a  Celesiial  Object ;  to  find  its  Right 
Jscension  and  Declination. 

Example.  . 

The  apparent  long,  of  a  Arietis,  August  1  st,.  1 825,  was!  1 1 5?  13  M8''.  62, 
and  its  latitude  9?57 -3^.48  norths  required  its  right  ascension  and  de- 
clination, the  obliquity  of  the  ecliptic  being  23?27'42''.  875  ? 

The  construction  of  this  Problem  is  exactly  like  that  of  the  preceding ; 
thus,  lay  the  longitude  of  the  given  star  off  on  the  ecliptic  line  from  on  to 
L,  and  draw  the  circle  of  longitude  N  L  O. — ^Make  yf  d,  ©  c  =  the  star's 
latitude,  and  draw  the  parallel  circle  cd;  the  intersection  of  which  with 
the  circle  of  longitude,  at  ^y  will  represent  the  apparent  place  of  the  star 
in  the  heavens.-— See  the  last  projection. 

Now,  in  the  oblique  angled  spherical  trifmgle  NP  a|e  ;  given  the  side 
PN  =  23?27U2''.875,  the  obliquity  of  the  ecliptic;  the  sideN  «  = 
80°2'28''.  52,  the  star's  distance  from  the  north  pole  of  the  ecliptic,  and 
the  included  angle  P  N  *  =  54?46'  1 1 ".  38,  the  complement  of  the. star's 
longitude  (measured  by  the  arc  L  ©)  ;^  to  find  the  side  P  *  =  the  star's 
north  polar  distance,  and  the  angle  N  P  *  (measured  by  the  arc  R  Q) ; 
the  difference  between  which  and  r  Q,  viz.  90  degrees,  expressed  by  the 
arc  Y  H  =  will  be  the  star's  right  ascension.  « 

Pence,  by  spherical  trigonometry,  Problem  IIL  Remark  1,  pi^e  203^ 


O/O  OP    M  Mill  mi   THS   KIUIIT   A9(;jinaiUPl9y   (»i;« 

Angle  PN*=      ..    .    .    54?46:il".38 


Half  ditto  5=  .  .  .  •  27?23^  5^69  Tw.L.sm.l9. 32545 14. 58 
Side  P  N  =  obliquity  of  the 

ecliptic  = 23.27.42  .  875  Log.sin.=9. 6000350. 88 

Side  N  :f:   =  star's  ecliptic 

polar  distance  =       •    .  80.   2.28  .250  Log.8in.= 9. 9934065. 08 


Sum  =s38. 9188930. 54 


Difference  of  the  two  sides  =56?34^45^  645  HalfS.  19.4594465.27  + 


Half  difference  =      ...  28^7 ^ 22*.  822  Log.  sin.  9. 675713/.  08 

Archtt     4 3l?17;22^56    Log.T.   9.7837328.19 

Logarithmic  sine  of  arch  = 9.7154718.60  — 

Half  the  side  P  *  =      .     .  33^40^57''.  76  Log.  sine  9. 7439746. 6? 

The  whole  side  P  ]|c  =  .  .  67^21  ^55^^.  52 ;  which  is  the  co-declination 
of  the  ^ven  star,  or  its  north  polar  distance;  hence^  the  declination  of  a 
Arietis  is  22?38U''.  48,  north. 

Now,  in  the  oblique  angled  sphisrical  triangle  N  P  4c  ;  the  three  sides 
are  given,  and  the  angle  N;  to  find  the  angle  P  =  the  arc  RQ;  or  the 
sum  of  the  starts  right  ascension  and  90  degrees. 

Hence,  by  spherical  trigonometry.  Problem  L,  page  198, 

As  the  side  P  %  s: .  .  67?2H55'';  52  Log.  co-sec.  «10. 0348085. 84 
IstotheanglePN*  =  54.46.11  .38  Log.  sine  «  .  9.9121S76.70 
So  is  the  side  N  )fe  =  .    80.  2. 26  .  52  Log.  sine  s    .  9. 9934065. 08 


To  the  sup.  of  at^.  NP^e  s60. 39. 12  .  38  Log.  sine  ±s  .  9. 9403527. 62 


Hence,  theang.  NPsjc  isal  19. 20M7'.  62  =:the  arc  RQ ;  from  which  take 
the  are  tQ,  90  degrees  5  and  the  remaining  arc  t  R  =:  29?20'47*'.  62,  is 
the  star's  right  ascension.— The  apparent  right  ascension  of  a  Arieds, 
on  the  given  day,  was  therefore,  29?20U7''.62,  and  its  dedination 
22?38M^  48  north. 

Now,  from  the  above  Problem  the  following  general  rule  is  dedwced  for 
computing  the  right  ascension,  and  declination  of  a  celestial-object^  vis. 
Find  the  difference  between  thei  ol^ect's  {on^tude  wd  three  signs  (e) 


Digitized  by 


Google 


ence^  add  the  logarithmic  sines  of  the  object's  distance  from  the  north 
pole  of  the  ecliptic,  and  of  the  obliquity  of  the  ecliptic  ;  from  half  the  sum 
of  these  three  logarithms  subtract  the  logarithmic  sine  pf  half  tlie  difference 
between  the  obliquity  of  the  ecliptic  and  the  pbject's  ecliptic  polar  distance^ 
and  the  remainder  Ivill  be  the  logarithmic  tang*  of  an  arch ;  the  log.  sine  of 
which  being  subtracted  from  the  half  sum  of  the  three  logarithms,  will  leave 
the  logarithmic  sine  of  an  arc^  the  double  of  which  will  be  the  object's  dis- 
tance from   the  north  pole  of  the  equator  :— now,  the  difference  between 
this  distance  and  90  degrees  will  be  the  declination  of  the  object ;  which 
will  b^  north  when  the  first  term  is  less  than  the  latter ;  otherwise  south. 
To  find  the  Right  Ascension  :— 
To  the  logarithmic  secant  of  the  object's  declination,  add  the  logarithm 
mic  sine  of  the  difference  between  its  longitude  and  90  degrees,  *  and  the 
logaTithmic  sine  of  its  distance  from  the  north  pole  of  the  ecliptic ;  the  sum 
of  these  three  logarithms;  abating  20  in  the  index,  will  be  the  logarithmic 
sine  of  an  arch,  which  being  subtracted  from  90  degrees  will  leave  the  ob- 
jeet'a  correct  right  ascension  when  its  longitude  is  less  than  3  signs  or  90 
degrees;  but  which  is  to  be  increased  by  180  degrees  when  the  longitude 
is  between  6  and  9  signs,  or  between  180  and  370  degrees.    Again,  if  the 
longitude  is  between  3  signs  and  6  signs,  that  is  between  90  and  180  de- 
grees, the  arch,  so  found,  is  to  be  augmented,  by.  90  d^roes;  but  if  the 
longitude  lies  between  9  and  12  signs,  vis.  between  270  and  360  degrees, 
it  is  to  be  augmented  by  270  degreei ;  in  eithier  case  the  correct  right 
ascension  of  the  object  wjll.be  obtained,  which  may  be  converted  into  time, 
if  necessary,  by  Problem  I.,  page  296. 

Example  K 
The  apparent  longitude  of  Aldebaran,  Aug.  3rd,1825,was  2*  7°21 :  18".  4 
audits  latitude  5? 28! 45?,  south;  recpiired  its  apparent  right  ascension 
and  declination,  the  obliquity  of  the  ecliptic  being  23'?27'42''.  875  ? 
Aldebaran's  longitude  =s  .    •     .    67?2la8^4 

Difference  to  90  degrees  »    ,    ,    22?38M1''.6 

The  halfof  which  is  =  .  .  .  Hei9'20\  8  TwJi.  sine  18.585974 
Aldebaran's  ecliptic  polar  dist.  es  95. 28, 45  . 0  Log.  sine  9. 998011 
Obliquity  of  ecliptic  =     .    .    »    23. 27. 42  .  875  Log.  sine  9. 600035 


Sum  =  38. 184020 


Difference  betw.  ob.  of  edip.  and 
star's  eclip.  polar  distance  =  .    72?  1 '  2".  125  Half  S.  19. 0920 10  + 

•  In  an  cues,  whenever  tbit  difference  exceeds  }89  deprees^  it  must  be  subtrscted  from 


JV      jrsi^«^jit^^«      «-« 


Difference  betw.  ob.  of  eclip.  and 

star's  eclip.  polar  distance  =  .     72?   1  \  2\  125  Half  S.  19.  092010  + 


Half  ditto  = 36?  0C31\062  Log  sine  9.  769309 

Arch  =     . 11.52.21.27     L.  tang.  9. 322701 

Log,  sine  of  arch,  subtract  from  ■ 

half  sum  of  logs.   = 9.313310- 


Arc= 36?55^27*'.3  Log.sine    9.778700 


Twice  the  arc= 73?50'.54*'.65    which  is  Aldebaran's 

.distance  from  the  north  pole  of  the  equator  \  hence  its  declination  is 
16?'9'.5%4  north. 


To  find  the  Right  Ascension  :— 

Declination  of  Aldebaran=  16?  9\  5\4  Log.  secant  =  10.017490 

Diff.  bet.  long,  and  90  deg.=:  22. 38. 4 1  .6  Log.  sine     =  9. 58548 1 

Aldebaran's  ecliptic  P.  dis.=:95.  28.45  . 0  Log.  sine    s  9. 99801 1 


Arch  5=      .....    23?30:58^.3  Log.sine    =      9.600982 


Aldebaran's  right  asc.  =   66?29'.   r.  7,or4i25?56M 


Example  2. 

The  apparent  longitude  of  Pollux,  Sep.  3rd,  1825,  was  3!20?48M4'.  2, 
and  its  latitude  6? 40^  19^  north;  required  its  apparent  right  ascension  and 
declination,  the  obliquity  of  the  ecliptic  being  23? 27 '42^.875  ? 

Pollux's  longitude  =     ....  110?48M4^2 


Difference  to  90  degrees  =    .    ,  .20?48<44^.2 


The  half  of  which  is  =  .  .  .  10?24^22M  Tw.L^in.l8. 513353 
Pollux's  ecliptic  polar  distance  =  83.19. 41  .0  Log.  sin.  9.997049 
Obliquity  of  the  ecliptic  =     .     .     23. 27. 42  .  875  L.  sine  9. 600035 

Sum  38.110637 

Difference  between  ob.  of  ecliptic  _.«...— ...^ 

and  sUr's  eclip.  pol.  distance  s  59?5K58'',  125  Hf,  S.  19.055318it 


Digitized  by 


Google 


OF  THE  UBAVBNLY  BODIES.  681 

Difference  between  ob.  of  ecliptic  *  •  * 

and  star's  eclip.  pol.  distance  =  59?51  ^58^.  125  Hf.  S.  19. 055318^+ 

Half  ditto  =       ...    .    .    ...    29?55^59^  062  L.  sin.  9.698090 


Arch= 12?49^25''.32  L.  tang.  9.3572281 

Log.  sine  of  arch,  subtract  from 
half  sum  of  Logs.  = 9.346258*- 

Arc=      .........    30M6^50^.03  Log. sin. 9. 709060 

TVicethearc=    ....     .    61^33^0'', 06  ;  which  is  Polhix^s  dis- 
tance from  the  north  pole  of  the  equator ;— hence,  its  declination  is 
28?26n9^.  94  north. 

To  find  the  Kght  Ascension  :-— 
Declination  of  Pollux  =  .    $8?26n9^.94  Log.  secant  =10.055850 
Diff^betw. Ion. and 90 deg.=20. 48. 44  .20  Log.  sine      =    9. 550603 
Pollux's  ecliptic  polar  dis.  =83. 19. 41  .  00  Log.  sine      =    9. 997049 


Arch= 23?39'.4ir        Log.  sine      =    9.603502 

Add     .......    90.   0.   0 


Right  ascension  of  PolInx=l  13?39M1  =  7*34r38^  .73 

Note. — In  the  same  manner  may  the  right  ascensions  and  declinations  of 
the  moon  and  planets  be  deduced  from  their  respective  latitudes  and  lon- 
gitudes. 

PROBLSM   in.    * 

Given  the  Latitudes  and  Longitudes  of  the  Moon  and  Sun,  Moon  and 

fixed  Star,  or.  Moon  and  Planet;  to  find  the  tnie  Central  Distance 

between  them. 

Note. — If  this  Problem  be  projected  stereographically  on  the  plane  of 
the  circle  of  longitude  passing  through  one  of  the  objects,  it  will  be  found, 
in  every  respect,  like  Problem  XXIV.,  page  273,  of  "  The  Young  Navi- 
gator's Guide  to  the  sidercfd  and  planetary  parts  of  Nautical  Astronomy  ;*' 
reading,  however,  difference  of  longitude  for  difference  of  right  ascension, 
and  ecliptic  polar  distances  for  polar  distances  : — henc^,  it  is  evident  that 
there  is  a  spherical  triangle  to  work  in,,  where  two  sides  and  the  included 
angle  are  given  to  find  the  third  side,  or  central  distance  between  the  ob- 
jects, ^d  which  may  be  computed  by  the  following 

General  Euk. 

.To  twice  the  logarithmic  sine  of  half  the  difference  of  longitude  between 
thf  two  objects,  add  the  logarithmic  co-sines  of  their  latitudes ;  from  half 


Digitized  by 


Google 


the  sum  of  these  three  logarithms  subtract  the  logaridimie  rioe  trf  half  the 
difference  or  half  the  sum  of  the  latitudes^  according  as  they  are  of  die  tame 
or  of  contrary  names ;  the  remainder  will  be  the  logarithmic  tangent  of  an 
arch,  the  logarithmic  sine  of  which  being  subtracted  from  the  half  sura  of 
the  three  logarithms,  will  leave  the  logarithmic  sine  of  half  the  true  central 
distance  between  the  two  given  objects. 

Example.  L 
September  3rd^  1825,  the  moon's  apparent  longitude,  at  noon^  was 
1 '.  16?  19^29r,  aud  her  latitude  2?33^S0?  north;  at  the  same  time  the  ap- 
parent longitude  of  Pollux  was  3f20?48^3S:,  and  its  latitude  6?40U9? 
north ;  required  the  true  central  distance  between  those  two  objects  ? 


Longitude  of  Pollux  s=  • 
Longitude  of  the  moqn  c 


U0'?48^38r 
46.19.29 


Difference  of  longitude  =        64?29'  9^ 


The  half  of  which  is  =  . 
Latitude  of  Pollux  =s 
Latitude  of  the  moon  s 


32?l4^34ir  Tw.  the  log.  sJn.l9. 454285 
.  6. 40. 19  N.  Log.  co.sine=9. 997049 
.  2. 33. 30  N.    Log,  co-sine=9. 999567 


Sum  =  39. 450901 


DilTerenee  of  latitude  .    .    #    4?  eU9?  Half  som     «    I9.7254ftO|  + 

Half  differences    ....    2?  3:24ir  Log.  sine    .      8. 5549771 

Arch=       86?  8air    Log.tang.=    11.170473 

Log.  sine  of  arch,   subtract  ■ 

from  half  sum  of  logs,  s         .    .    .    .    ^    .    .    «    .    9.999012- 


Half  true  distances      .    .    32?11^  4^  Log.  sine  =        9.726438| 

Hence,  the  true  central  distance  between  the  moon  and  Pollux,  at  the 
g^ven  time,  was  64?22!8T  •  which  corresponds  exactly  with  the  computed 
distance  in  the  Nautical  Almanac. 

Note.^^lt  is  evident  that  the  same  result  will  be  obtdned  by  mdiing 
use  of  the  right  aseensions  and  declinatlona  of  the  objects. 

JS^HnpIe  2. 
August  22,    1825,  the   moon's    apparent   longitude,   at    noon,    was 
8!  19?6'  15r, and  her  latitude 0?d^2K  north;  at  the  same  time  the  appa- 
rent longitude  of  Spica  Virginis  was  6121?24l32?,  and  its  latitude 
2^3:^5?  apttth )  rei|[aiitdtbe  tnie^entria  dtaUaeeUttmo  Hmc  ob|^t»^ 


Digitized  by 


Google 


uw  i;uMruimu  thjs  lukak  uiSTAjncjS9* 


ooo 


Longitude  of  the  moon  &=  •    259?  6^  IS'T 
Ifongitude  of  Spica  Virginks  201.  24. 32. 

Difference  of  longitude  =     •     57?41'43'r  this  divided 
.    by  2  gives     28?50'.51ir  Twice  L.  sin.=  19. 366962 
Lat.ofmoon=     0.  3.21  N.  Lo^.co-siue  10.000000 
IfatofSpicaVirg.  2.   2.23  S.  Log.co-sine    9.999725 


Sum  = 


.39.366687 


Sum  of  lata.  =  •  2?  5M6r  Half  sum  =  .19. 683343  i  . .  19.  683343  ^ 
Half  ditlosi 
Arch »     • 


1?  2^53r  Log.  sine  =  .     8.  2622374 
87?49M2^  Log.  tang. 


n.  421106  L.sin.  9. 99968S 


Half  true  dist.  =  28?5 1  ^37^  Log.  sine  = .9.  683655  J 

Hence,  the  true  central  distance  between  the  moon  and  Spica  Virgtnia 
is  57^43  H 4^ ;  which  is  1 1  more  than  that  given  in  the  Nautical  Almanac. 

Example  S. 

August.  4th,  1825,  the  moon's  apparent  longitude,  at  noon,  was 
0'.  14?l3C32r,  tod  hiSr  latitude  4?d8Uirnorth  ;  at  the  same  time  the  sun's 
longitude  was  4 !  1 1  ?4d '  46 ;  required  the  true  central  distance  7 

Nol^.— Since  the  sun  apparendy  moves  in  the  ecliptic,  he  has,  therefore, 
no  latihide. 

Mvkm's  longitude  =«  14?13^82? 
Sun's  tongitude  aa    131.43.46 

Difference  pf  long.=  1 1 7. 30. 1 4  this  divided 

iy  2  gives  5^?45 1  7'  Twice  the  L.  sines  m  863660 
Moon's  lat.  4. 38. 4 1  N.  Log.  co-sine  =  9. 99857 1 
Sun's  Ut.  S5  0.   0.   0       Log.  co-sine  s  .  lO.OOOOOO 


Sum  as  k 


Diff.oflat.r=4?38!4lr  Hdfium: 


Half  dittx>=  2?  19^2Qir  Log.  sine  a 
Arch  =     .  87?16'55r    Log.tang..= 
Hf.  req.  dis.  58^42:  lOjr  Log.  sine  = 


39.862431 

19.9312151 

S.  607688 


19.931215^ 


11. 323527iL.8in.9. 999511 


9,9317044 


Hence^  the  true  central  distance  between  moon  and  sun  is  1 17?24^2ir  j 
wW^h  corresponds  with  that  in  th^  Nawtjc^  Almwuc^ 


684  .  OP  couPvriVG  the  lunar  di^ancbs.  > 

Rem(7r%'.— Since  the  co-latitudes  of  the  sun  and  moon  and  the  compre- 
hended  angle  (expressed  by  their  difference  of  longitude,)  form  a  quadran- 
tal  spherical  triangle;  therefore  the  true  central  distance  between  these 
particular  objects  may  be  more  readily  determined  by  the  following  con- 
cise method  than  by  the  above  general  Rule,  viz. 

To  the  logarithmic  co-sine  of  the  difference  of  longitude,  add  the  logar- 
ithmic co-sine  of  the  moon's  latitude ;  the  sum  of  these  two  logarithms, 
abating  10  in  the  index,  will  be  the  logarithmic  co-sine  of  the  true  central 
distance  between  the  sun  and  moon. 

Example  1. 

August  6th,  1825,  the  moon's  longitude,  at  noon, was  l!d?0'34'y  and 
her  latitude  3?  23  ^20?  north;  at  the  same  time,  the  sun's  longitude  was 
4113?38!46T  ;  required  the  true  central  distance  ? 

Moon's  longitude  =    38^  0'34r 
Sun's  longitude  =  ..  133.38.46 


Difference  of  long.  =:    95?38n2r  Log.  co-sine  =  .    ,    .    .  8.992199 
Moon's  latitude  =  .    .    3.23,20     Log.  co-sine  =;=  .    .    .    .9.999240 

True  central  distance  =95?37'36r    Log.  co-sine  =  .  ..  ..   *.  8.991439 
which  is  precisely  the  same  as  that  given  in  the  Nautical  Almanac. 

Example  2. 

August  7th,  1825,  the  moon's  longitude,  at  noon,  was  l!20?4M2f,  and 
her  latitude  2?30^42?  north;  at  the  same  time  the  swi's  longitude  waa 
4 :  14?36 '  1 8^ ;  required  the  true  central  distance  ? 

Moon's  longitude  =   50?  4M2^        . 
Sun's  longitude  =  .  134.36. 18 

Difference  of  long.  =  84 ?3 1  ^  36r  Log  .  co-sine  =  ....    8. 979468 
Moon's  latitude  =    .    2. 30. 42     Log.  co-sine  =   •    .    .    .    9. 999583 

True  central  dist  =    84?31^55r  Log.  co-sine  =        ...    8.979051 

which  exactly  corresponds  with  the  computed  distance  in  the  Nautical 
Almanac. 

Example  3. 

Required  the  true  central  distance  between  the  moon  and  sun  at  noon, 
August  8th  ;  at  midnight,  August  8th;  at  noon,  August  9th^  and  at  ovd- 
night,  August  9th,  1825  ? 


Digitized  by 


Google 


Moon's  long,  noon  Aug.  8th  =    62?22^38T 
Sun's  longitude  ditto  =    •     .     135. 33. 5 1 


JJifTerence  of  longitude  =  .    .    73?  1 1 :  13f  Log.  co-sine  =     9. 461273 
^Moon's  latitude  =  .    .    •    .         1.30.   9    Log.  co-sine  =     9.999851 

Distance  at  noon,  Aug.  8th,         73?  1 1 '. ZV.  Log,  co-sine  s     9. 46 1 1 24 


A^ooii's  long.  mid.  Aug.  8th  =    68?38'20r 
Sun^s  longitude  ditto  s      .    *.  136.    2.38} 


Difference  of  longitude  = 
IVloon's  latitude  =s       .    • 


67?24n8jr  Log.  co-sine 
►    0. 57. 33       Log.  co-sine 


9. 584572 
9.  999939 


Distance  at  midnight,  Aug.  8th=67? 24 '.  3 K  Log.  co-sine  s     9. 5845 1 1 


Moon*s  long,  noon,  Aug!  9th  =3     74?59'  18r 
Sun's  longitude  ditto  =  .    .     •  136.31.26 

Difference  of  longitude  s  .    .      61?32^  8r     Log.  co-sine  =  9. 678166 
Moon's  latitude  = 0.  23. 49      Log.  co-sine  =  9.  999990 


Distance  at  noon,  Aug.  9th,  =       6ie32'  llr     Log,  co-sine  =  9.  678156 


Moon's  long,  at  mid.  Aug.  9th  =81 925 1 59^ 
Sun's  longitude  at  ditto  =     .     .  137*    0. 14 

Difference  of  longitude  =  .     .      55934M5r  Log.  co-sine  =  9.752346 
Moon's  latitude  =  •     .     ;    •    .    0.10.42    Log.  co-sine  =  9.999998 

Distance  at  midnight  Aug.  9th  =  55934'  16^  Log.  co-sine  =    9. 752344 
Now,  from  these  four  consecutive  lunar  distances,  the  distances  at  the 
intermediate  periods,  oi>  every  third  hour  may  be  readily  determined  in  the 
following  manner,  viz. 

Find'  the  proportional  parts  of  the  difference  at  the  middle  interval  be- 
tween the  four  distances  (that  is,  between  the  second  and  third  distances,) 
answering  to  3  hours,  6  hours,  and  9  hours :  correct  these  proportional 
parts  by  the  equation  of  second  differences  agreeably  to  the  rule  given,  for 
that  purpose,  in  page  34  ;-^then,  these  corrected  proportional  parts  being 
applied  to  the  second  lunar  distance  by  addition  or  subtraction,  according 
as  the  distances  are  increasing  or  decreasing,  the  sum  or  difference  will  be 
the  true  distances  at  the  given  periods  :^thus. 


OF  COMPUTING  THE  LUNAR  mSTANGBS* 


Aug.8,1825,di8.atN.  =73nH34 
Ditto  •  .  .  do.  at M.  =67.24.31 
Aug.  9»  •  .  do.atN.  =6L82.  )K: 
Ditto  .  •  .  do.atM.s=55.84. 16 


UtdUt  SddUff. 

5?47'  3^^ 


McwMCC 


c   c9  on   }0?5n7^i      , 


The  proportional  parts  of  5?52'20.T  (the  middle  first  difference)  anawer^ 
ing  to  the  intermediate  periods,  viz. 

To  3  hours  it  is  =s 1*?28'.  5^} 


Equation  of  second  difference 

Proportional  part  corrected,  a    . 
Distance  at  midnighty  Au^«  8th  = 

Distance  at  15  hours,  Aug.  8th  ss 


To  6  hours  it  is  =;=       •     .     •     • 
Equation  of  second  difference  = 

Proportional  part  corrected  =    . 
Distance  at  midnight  Aug.  8th  = 

Distance  at  18  hours  Aug.  8th  =s. 

And  to  9  hours  it  is  =    '  •     •     • 
Equation  of  second  difference  = 

Proportional  part  corrected  ss'  . 
Distance  at  midnight  Aug.  8th  = 

Distance  at  21  hours  Aug.  8th  =s  . 


-  31 


1?27'34 
67.24.31 

.6S?d6^57f 

2?56U0r 
•      -  41 

2?55i29r 
87.^4.31 

>  f.      I   II  ■■  I  I 

64°29C  2r 

4?24'15r 
-  31 

4^23:447 
67.24,31 

63?  0^477 


The  distances  for  the  intermediate  periods  corresponding  to  the  first 
and  to  the  last  12  hours;  that  is,  for  every  third  hour  between  the  first 
and  second  distances,  and  between  the  third  and  fourth  distance,  may  be 
also  very  readily  determined  by  means  of  the  Formulae  which  are  given  in 
page  1 17  of  the  Nautical  Almanac  for  1825. 


Digitized  by 


Google 


APPENDIX. 


Showing  the  direct  application  of  logarithms  to  the  solution  of  pro- 
blems connected  with  the  ctoctrine  of  compound  interest ;  which  deve- 
lops the  extraordinary  powers  of  logarithmical  arithmetic  more  than 
imy  other  department  of  science  which  has  been  touched  upon  in  this 
work. 


D^miion.— .Compound  Intbrest  is  that  which  is  deduced  not  only 
from  the* sum  of  money  lent  as  the  principal^  but  also  from  the  interest 
ariring  thereon ;  which  interest,  as  it  becomes  due  at  the  stated  times  of 
payment,  is  added^  or  supposed  to  be  added^  to  the  principal/ 

Although  it  is  illegal  to  lend  money  at  compound  interest,  yet  hi 
purchasing  annuities,  pensions,  or  leases  in  reversion,  it  is  usual  to  allow 
the  purchaser  compoutid  intel'est  for  the  use  of  his  ready  money. — ^And^ 
these  points  being  premised,  we  will  proceed  to  the  solution  of  the  most 
interesting  problems  relating  to  this  department  of  science] — ^for  which 
purpose  the  following  Tables  have  been  computed. 

Note. — The  rates,  or  .ratios,  o(  £  1  sterling  contained  in  these  tables 
were  computed  by  .the  rule  of  proportion  in  the  following  manner,  viz.— 
As  £  100  I  £3 : :  £  1  to  £.  0300  ; — hence,  the  amount  of  £  1  for  one  yedr  is 
£1.0300;  which,  therefore,  is  the  ratio;  and  so  on  for  the  rest.— The 
respective  numbers  annexed  to  these  ratios  are  expressed  by  the  common 
logarithms  corresponding  thereto  :-*-thus,  the  logarithm  of  1.0300  is 
0.0128372;  and  the  logarithm  of  .0300  is  8.4771213,  and  so  on  of 
others. — In  this  part  of  the  work  it  has  been  deemed  advisable  to  take 
out  the  logarithm^  to  seven  places  of  decimals  ;  though  for  ordinary 
purposes  six  places  of  decimals  will  be  found  amply  sufficient. 


Digitized  by 


Google 


688 


S 


9 

^ 


1^ 


a 


Pi      V 


i 


<^ 


I 


9 


<>   s   s   ^   :?   s   s 

•••       o        Ct       •••       t^       w       c« 

tS      to      '^      *^      £?      o      5P 


I 


8    i  S 

-    «  S 

t-4            <«V  <P 

Sl>.  Ok 

o  o 

00       CO  00 

»    I  i 

^  s  $ 


Si  ti 

s  ^ 

2  S 

00  00 

I  I 


lO         »^ 

00        00 

1    I 


a   2 


I  s 

l2 


2f  o  £2  2     a 

M  00  ^  O        00 

a  S3  S  2     ^ 

25  *^  S  2 

2  g  s  ^ 

e»  et  « 


8       eo      -^ 
cs      00      eo 


o  a»  e* 

^  s  § 

K.  O  0> 

CO  ^ 


I    r   I 


D       I       I       S 


90 

D 


.!*5  iA<c  r*  00  Q  -J  ©s 
^■<  -^  •*  ■  55  5s  S? 
o      o      o      O      ^     o 


5 L 


S     § 


5    5 

00        00  00 

I    I  I 

So  lO 

«o       *^  00 

s  s  s 


00 

I 


a    S 


! 


3  s 
s  s 


•n     Ok 

00      <o 


B     2     S     2     2     2     S 

s  s  i  i  s  s  s 


to     to 

s  s 


00         ^ 

s  s 


o 
D 


-<  ^  § 

00       00        A 

-  i  s 


o  o  o 

11  I  l' 

i  i  i 

^  .^  M 


II    I 


i  § 


o     o     -4     M     e«     CO 

9^  r^  ^m  m*  m^  9M 


I 

g  Si 
s;  9 


«ododooo      ooo 
q2' 


A        e«        CD        00        04 

S    S    S    S    B 
S    2    S:    a    » 


I 


i  1 1  i  I  i  I  i 


3 


CO  ^  IP 

p}  <M  CJ 

S  g]  »    ^ 

;;  S  Si    S 


I      R 


1 1 


I      Q 

£    S 

S    3 


I     I 


s  -s 


0  I 

SiA        O  lA 

ci       O  t>. 

s  s  s  s 


co«oef^^^'*»o*o« 


Digitized  by 


Google 


Problem  I. 
Given  the  Princgtaly  Bate  of  Interest,  and  Time;  to  find  the  Amount. 

RULB. 

Multiply  the  logarithm  of  the  ratio,  Table  A,  by  the  time,  or  number  of 
payments,  according  as  the  instalments  may  be  reckoned  in  years,  half 
years,  or  quarters  ;  to  the  product  of  those  two  numbers,  add  the  logar- 
ithm of  the  principal,  and  the  sum  will  be  the  logarithm  of  the  amountt 

l?emarfc.-«-The  logarithm  of  the  ratio,  contained  in  Table  A,  must  always 
be  taken  out  so  as  to  correspond  with  the  nature  of  the  instalments ;— - 
thus,  if  the  payments  are  to  be  made  yearly,  the  logarithm  is  to  be  taken 
out  of  the  first  column ;  if  half  yearly,  it  is  to  be  taken  out  of  the  second 
column  ;  but  if  quarterly,  it  must  be  taken  out  of  the  third  column.-— The 
same  is  to  be  observed  of  liable  B ; — and,  in  order  to  avoid  repetition,  it 
must  be  remembered  that  this  remark  is  applicable  to  the  various  cases  of 
compound  interest,  &c.  which  may  be  given. 

Example  L 

What  will  a  principal  of  £240000  sterling  amount  to  in  45  years,  by 
annual,  payments,  at  the  rate  of  5  per  cent,  per  annum,  compound 
interest? 

Kate,  5  per  cent. ;  ratio  =  1. 0500,  logarithm^  Table  A.  s   .0. 02  U  893 
Multiply  by  the  number  of  payments  =  .••••••  45 

1059465 
0847572 


Product  =    . 0.9535185 

Principal  =  £  240000,  the  logarithm  of  which  is  =  •     .     •    5 .  38021 1 2 

Amount,  or  improved  principal  =  .    £2i 56402       Log.  =;  6.3337297 

Example  2. 
What  will  a  principal  of  £240000  sterling  amount  to  in  45  years,  by 
half-yearly  payments,  at  the  rate  of  5  per  cent,  per  annum,  compound 
interest  ? 

Rate,  5  per  cent. ;  ratio  =  1. 02500;  logarithm,  Table  A,  =  0. 0107239 
Multiply  by  the  number  of  payments  =  45  years  x  2  =      .   .  90 

Products     .......: 0.9651510 

Principals  £240000,  the  logarithm  of  which  is  =      .    .    .    5.3802112 

Amount,  or  improved  principal,  =  £2214941        Log.  =    .    6. 3453622 

2  Y 


690  COMPOUND  IKTBRXST. 

Example  3. 

What  will  a  principal  of  £240000  sterling  amount  to  in  45  years,  by 
quarteriy  payments,  at  the  rate  of  5  per  cent,  per  annum,  compound 
interest  ? 

Rate,  5  per  cent. ;  ratio  =  1 .  012500 ;  log.,  Table  A,  =      .0. 0053950 
Multiply  by  the  number  of  payments  =  45  years  x  4  =  •    •  180 

4316000 
053950 


Products 6.9711000 

Principal  =  £240000,  the  log.  of  which  is  s 5.3802112 


Amount,  or  improved  principal,  =  £2245490        Log.  =  •    6.3513112 

Note. — ^The  above  examples  will  show  the  reader  the  great  evil  which  is 
attendant  upon  other  than  yearly  payments  of  interest.  Thus,  in  the 
present  instance,  while  thie  principalis  augmented  to  nine  times  its  original 
value,  the  excess  of  the  half-yearly  above  the  yearly  instalments  is  £58539  ; 
but  by  quarterly  instalments,  it  is  full  £89088 :  which  excess  is  evidently 
to  the  manifest  injury  of  the  borrower  or  debtor.  This  will  appear  still 
more  evident  from  the  following 

Example. 

Admitting  that  the  national  debt  of  Great  Britain  was  £206590000 
sterling  in  the  year  1786,  what  would  its  probable  amount  be  in  the  year 
1814  (being  a  lapse  of  28  years),  by  yearly,  half-yearly,  and  quarterly 
payments,  at  the  rate  of  5  per  cent,  per  annum,  compound  interest  ? 

Pirst,-*For  Yearly  Payments : — 

Rate,  5  per  cent ;  ratio  »  1 .  0500 ;  log.  Table  A,  ==      .    .    0^  021 1893 
Multiply  by  the  number  of  payments  = 25 

1695144 
0423786 


Product  = 0.5938004 

Amounf  of  debt  in  1786  =  £206590000,  the  log.  of  which  is=8. 3151093 

Amount  of  ditto  in  1814  ss  £809859533,  very  nearly     Log.»8. 9084097 

Digitized  by  VjOOQ IC 


COMPOUND  IKTSASST^  691 


Second,— For  Half-yearly  Payments  :— 

Rate,  5  per  cent. ;  ratio  =  1 .  02500  j  log..  Table  A,  =     «    .    0. 01 07239 . 
Multiply  by  the  number  of  payments  a  26  years  x  2  oa       «  56 

0648434 
0536195 


Products 0.6005884 

Amount  of  debt  in  1786  =  £206590000,  the  log.  of  wMch  i8»8. 8151098 

I  ■!■  Mil         ^ 

Amount  of  debt  in  1814  a  £823469720^  very  nearly      Log*a8. 9156477 


Third,— For  Quarterly  Payments  t-^ 

Rate,  5  per  cent. ;  ratio  =  1 .  01 2500  $  log.,  Table  A,  =       .0. 0053950 
Multiply  by  the  number  of  payments  =  28  years  x  4  =s       •  112 

0107900 
0593450 


Products     . ,0.6042400 

Amount  of  debt  in  1786=  £206590000,  the  log.  of  which  i8=8. 3151093 


Amount  of  ditto  in  1814  ^  £8305 18462,  very  nearly      Log.aS.  9193493 

Hence  it  appears,  that  if  the  national  debt  were  £206590000  in  the 
year  1786,  and  that  it  were  allowed  to  multiply  by  the  accumulation  of 
interest  upon  interest,  it  would  amount,  at  the  end  of  28  years,  rht^  in 
1814,  by  annual  payments,  to  the  sum  of  £809859533  3  by  half-yearly 
payments,  to  the  sum  of  £823469720;  but,  by  quarterly  payments,  to  the 
enormous  sum  of  £8305 18462  sterling  !  And  this  is  a  sum  of  such  mag- 
nitude, that  it  could  not  be  liquidated  by  all  the  gold  and  silver  now  in  , 
circulation  amongst  the  different  kingdoms,  states,  and  empires,  in  the 
known  world. 

We-  will  only  adduce  one  more  example  in  order  to  apprize  the  reader 
of  the  almost  incredible  manner  in  which  a  smn  of  money  may  be  imp? oveA 

by  the  accumulation  of  interest;  for  which  purpose,  let  us  suppose  that  six- 
pence sterling  is  the  sum  put  out  at  compound  interest,  that  the  rate  is  & 
per  cent,  per  annum,  payable  by  half-yearly  instalments,  and  that  the  time 
is  450  years. — Then, 

2y2 


Digitized  by 


Google 


Rate,  6  per  cent ;  ratio  =  1 .  0300 ;  log.,  Table  A,  =       .    .    0. 0128372 
Multiply  by  number  of  payments  =  450  years  x  2  ==      •    .  900 

Products .11.5534800 

Principal  =  6d.  sterling,  or  .  025  £,  the  Ic^.  of  which  is  =  .     8. 3979400 

Amount,  or  improved  principal,  =  £8941700000        Log.  =  9. 9514200 

Hence  it  is  evident,  that  sixpence  sterling  put  out  at  mterest  vpon  interest j 
agreeably  to  the  given  rate  and  time,  would  amount  to  the  amazing  sum 
of  eight  thousand  nine  hundred  and  forty-one  millions  and  seven  hundred 
thousand  pounds  sterling;  which  sum  could  not  be  made  up  by  all  the 
gold  and  silver  that  have  been  dug  out  of  the  bowels  of  the  earth  from  the 
creation  of  the  world  to  the  present  day ! 


Problem  II. - 

Given  the  Amount  or  improved  PrindpaU  Rate  of  Interest,  and  Ttme; 
to  find  the  original  PrindpaU 

Rdlk. 

Multiply  the  logarithm  of  the  ratio.  Table.  A,  by  the  time  or  number  of 
payments ;  subtract  the  product  from  the  logarithm  of  the  amount,  and  the 
remainder  will  be  the  logarithm  of  the  original  principal  or  sum  put  out  at 
interest.  ^ 

Example. 

What  principal  put  to  interest  for  31  years,  and  payable  half-yearly,  will 
amount  to  £29876  sterling,  at  5  per  cent,  per  annum,  compound  interest? 

Rate,  5  per  cent. ;  ratio  =  1 .  02500 ;  log..  Table  A>  =    ...     0, 0107239 
Multiply  by  number  of  payments  =  31  years  x  2  =  .    •    .  62 

0214478 
0643434 


Products 0.6648818 

Amount,  or  improved  principal,=:  £29876,  the  log.  of  which  is=4. 4753225 

Original  principal,  or  sum  put  to  interest,  =  £6463      Log.  =3. 8104407 


Digitized  by 


Google 


COMPOUND  INTEREST.  693 


Problem  •  IIIl 

Given  the  original  Principal  or  Sum  lent,  the  7«m^,  and  Hie  Amount  or 
improved  Principal;  to  find  the  Rate  of  Interest. 

'  Rule. 

From  the  logaiithtn  of  the  amount  or  improved  principal,  subtract  the 
logarithm  of  the  original  principal)  divide  the  remainder  by  the  number 
of  payments,  and  the  quotient  will  be  the  logarithm  of  the  ratio ;  with 
which  enter  the  proper  column  of  Table  A,  according  to  the  modes  of 
payment^  and  opposite  thereto,  in  the  left-h&nd  column^  will  be  found  the 
required  rate  of  interest. 

Example. 

At  what  rate  per  cent,  per  annum  will  £2360  amount  to  the  sum  of 
£4792  in  15  years,  the  payments  being  made  quarterly  ? 

Amount,  or  improved  principal,  =  £4792,  log.  of  which  is=:  3. 6805168 
Original  principal  or  sum  =  £2360,  the  log.  of  which  is  =  3.3729120 

Divide  by  the  number  of  payments  =  15  years  x  4  =     60)0.3076048 

The  ratio  to  which,  in  Table  A,  is  1. 01 1875,  =  quotient  =  0, 0051267. 5 
Hence,  the  rate  of  interest  is  4|  per  cent,  per  annum. 


Problem  IV. 

Gftt^  the  oriffnal  Principal  or.  Sum  lent,  the  Rate  oflnteregt,  and  the 
Amount  or  improved  Principal ;  to  find  the  Time. 

Rule. 
From  the  logarithm  of  the  improved  principal,  subtract  the  logarithm  of 
the  original  principal  |  divide  the  remainder  by  the  logarithm  of  the  ratio. 
Table  A,  and  the  quotient  will  be  the  time  or  number  of  payments  in  years,, 
halves,  or  quarters,*  as  the  case  may  be. 

Example, 

In  what  time  will  the  deposits  or  funds  in  the  savings'  banks,  which  are 
now  (June,  1826,)  estimated  at  £14500000  sterling,  amoutft,  by  half- 
yearly  payments,  to  the  sum  of  £107415024  sterling,  at  the  rate  of  4^  per 
cent  per  annum,  compound  interest  ? 


Digitized  b' 


Google 


Q94  AAnUlllAB     in    AKJUUlKSy 

Amount^  or  improved  pTincipaI,£l074l5024,  the  log,  of  which  is  8.  0310457 
Principal,  or  sum  funded  in  June,  1826,=:f  14500000      Log.=:7.  1613680 

Rate,  4|  per  cent. ;  ratio  =  1. 02250 ;  divide  by  log,  of  this, 

TableA,= '.    0.0096633)0.8696777 

=  90  half-yearly  payments :  hence,  the  required  time  is  45  years. 

From  the  above  result,  an  abstract  reaaoner  would  be  apt  to  imagine 
that  the  savings'  banks  are  more  of  individual  than  of  national  uUlity« 


ANNUITIES  IN  ARREARS,  AT  COMPOUND  INTEREST, 

Definition. — An  annuity  is  said  to  be  in  arrears  when  the  debtor  keeps 
it  in  his  hands  for  any  certain  time  after  the  term  or  period  of  payment 
becomes  due.  The  sum  of  all  the  single  payments,  together  with  the 
interest  due  upon  each  payment,  from  the  time  of  its  becoming  payable  to 
the  time  that  the  whole  is  paid  off,  is  called  the  amount  of  such  annuity. 


Problem  I. 

Given  an  Annuity ^  the  Tirtke  or  Number  ofPaymenU,  and  the  Rate  fer 
•    Cent,  per  Annum  ;  iofind  the  Amount. 

RULK. 

Multiply  the  logarithm  of  the  ratio,  Table  A,  by  the  number  of  pay- 
ments ;  find  the  natural  number  answering  to  the  product,  and  diminish  it 
by  the  integral  part  of  (he  ratio,  viz.,  I ;  then,  to  the  logarithm  of  this 
diminished  natural  number,  add  the  logarithm  of  the  annuity  (proportioned 
to  the  modes  of  payment)  :  from  the  sum  of  these  two  logarithms,  subtract 
the  logarithm  of  the  decimal  part  of  the  ratio  contained  in  Table  B;  and 
the  remainder  will  be  the  logarithm  of  the  amount  of  the  annuity. 

Note.'-U  the  payments  be  yearly,  take  the  logarithm  of  the  whole 
annuity ;  if  half-yearly,  take  the  logarithm  of  half  the  annuity ;  and  if 
quarteriy,  take  the  logarithm  of  the  one-fourth  part  of  the  annuity. 


Example 

B560  per  ai 
^^  ^  _r  cent,  com 
quarterly  payments  ? 


If  an  auteity  of  £560  per  annum  be  unpaid  for  9  years,  what  will  it 
amount  to,  at  4i  per  cent,  compound  interest,  by  yeariy,  half-yearly^  and 
arterlv  uavments  ? 


Digitized  by 


Google 


Rate,  4J  per  cent.  5  ratio=  1 .  0425 ; 

log,,  Table  A    =  0.0180761 
MuLbyno.ofpajTi.z:  9 

Product  =       .    .    0. 1626849  Nat.  no.=  1 .  4544033 
Subtract  the  integral  part  of  the  ratio,  viz.,    1 


I>imiRished  natural  number  rz       .     .     .    0.4544033  Log.=:9. 6574415 
Annuity  =  £560,  the  log.  of  which  i«  =: ^.7481880 

Sum  of  the  two  logarithms  =     ;....,.•..  12.4056295 
Decimal  part  of  the  ratio=.  0425  j  log.,  Table  B,  =  .    .     ,8. 6283889 

Amount  of  annuity  by  yearly  payments  =  £5987. 432     Log.=:3. 7772406 

Second, — For  Half-yearly  Payments  : — 

Rate,  4^  per  cent. ;  ratio  =  1 .  021 25  ; 

log..  Table  A,  ==  0,0091321 
Multiply  by  no.  of 

paym.::=9  years  X  2:=  18 

0730568 
0091321 


Produets      .    .    0.1643778  Nat^Bo^K  4600838 
Subtract  the  integral  part  of  the  ratio,  viz.,  1 


Diminished  natural  number  =:       ...    0. 4600838  Log.=9. 6628370 
Annuity,  £560 ;  one  half  of  which  is  £280 :  log.  of  this  =    .     2. 447 1580 

Sum  of  the  two  logarithTns  ^    .    , 12.1099950 

Decimal  pait  of  the  ratio  =  .  02125  5  log.,  Table  B,  —  .    .    8. 3273589 

Amount  of  the  annuity  by  half-yearly  paym.=£6062. 382  Log.  3. 7826361 

Thii;d,— For  Quarterly  Payments  :— 

Rate,  4i  per  cent. ;  ratio=:  1 .  010625  ; 

log..  Table  A,  =  0. 0045900 
Multiply  by  no.  of 

paym.::;:9yrsx4=  36 

0275400 
0137700 


Product  ;=      .    •    0, 1652400  Nat.  no.=  1.46?9855 


ANNUITIES   IN  ARRBARS, 

Product  =       .    •    0. 1652400  Nat.  no.r:  1 .  4629855 
Si^btract  the  integral  part  of  the  raiio^  viz.^  1 


Diminished  natural  number  =       .     .     •    0»  4629855  Log.=:9. 6655674 
Annuity,  £560 ;  oae  fourth  of  which  is  £140 :  log.  of  this  =     2. 1461280 

Sum  of  the  two  logarithms  == 11.8116954 

Decimal  part  of  the  ratio  =:  .010625 ;  log.,  Table  B,  =:  .     .     8. 0263289 

Amount  of  the  annuity  by  quarterly  payments=£6100. 515  Log.3. 7853665 


iVb^^.-^Instead  of  subtracting  the  logarithm  of  the  decimal  part  of  the 
ratio  (Table  B)  from  the  sum  of  the  logarithms  of  the  diminished  natural 
number  and  the  annuity,  its  arithmetical  complement  may  be  added  to 
these  two  logarithms ;  which,  perhaps,*  will  render  the  operation  a  little 
more  concise,  or,  at  least,  apparently  so.    Thus : — 

Diminished  natural  number  x:  0. 4629855  Log.  ==  •  .  9. 6655674 
Annuity,  £560i  one  fourth  ofwhich  is  £140  Log.  =  .  .  2.146^1280 
Decimal  part  of  ration  .  0 1 0625 ;  log.,  Table  B,  arith.  comp.  =  1 .  97367 1 1 

Amount  of  the  annuity  by  quarterly  paym.  =  £6100. 5 15  Log.  3. 7853665 


Problem  II, 

Ct!>en  the  Time  or  Number  of  Payments^  the  Rate  per  Cent,  per  Anrnm^ 
and  the  Amo\mt ;  to  find  the  Jnnvity. 

RUJJB, 

Multiply  the  logarithm  of  the  ratio  (Table  A)  by  the  number  of  pay- 
ments 5  find  the  natural  number  answering  to  the  product,  and  diminish  it 
by  the  integral  part  of  the  ratio.    Then, 

To  the  arithmetical  complement  of. the  logarithm  of  the  diminished 
natural  number,  add  the  logarithm  of  the  amount  and  the  logarithm  of  the 
decimal  part  of  the  ratio  (Table  B):  the  sum  of  these  three  logarithms  (abat- 
inglO  in  the  index,)  wJU  be  the  logarithm  of  the.  annuity  for  yearly  payments, 
but  of  half  the  annuity  for  half-yearly  payments,  or  of  one  fourth  thereof 
for  quarterly  payments. 


Digitized  by 


Google 


AT  COMPOUND  INTBRB8T.  697 

Example. 

A  certain  annuity,  at  the  rate  of  4i  per  cent,  per  annum,  amounted,  at 
the  end  of  9  years,  hy  annual  payments,  to  the  sum  of  £5987. 432;  by 
half-yearly  payments,  it  amounted  to  the  sum  of  £6062. 382 ;  but  by 
quarterly  payments,  to  the  sura  of  £6100. 5 15 ;  required  the  yearly  value  of 
that  annuity  ? 

First,— For  Annual  Payments  ;— 

Rate,  4i  per  cent. ;  ratio=  1. 0425 ; 

log..  Table  A,=  0.0180761 
Mult.byno.ofpaym.=  9 


Products     .    .    0.1626849  Nat.no.  1.4544033 
Subt.  the  integral  part  of  the  ratio,  viz.,  1 


Diminished  natural  number  =  .  •  0. 4544033  Log.  ar.co.0. 3425585 
Amount  of  annuity  by  annual  payments  ==  £5987. 432  Log.=3. 7772406 
Decimal  part  of  the  ratio  =: .  0425 ;  log.,  Table  B,  =      .    .    8. 6283889 


Yearly  value  of  the  annuity  =:  £560        Log.  =    .    .    .    .    2.  748 1 880 


Second,— For  Half-yearly  Payments  :— 


Rate,  4i  per  cent.;  ratio  =:  1 .  02125 ; 

log.,  Table  A,  =:  0. 009132 1 
Multbyno.ofpaym. 

=:9yearsx2=:  18 


0730568 
0091321 


Products    .    •    0.1643778  Nat.  no.  1.4600838 
Subt.  the  integral  part  of  the  ratio,  viz.,  I 


Diminished  natural  number  =:  .  ;  0. 4600838  Log.  ar.  co.  0. 337 1 630 
Amount  of  annuity  by  half-yearly  payments=£6062. 382  Log.3.  7826361 
Decimal  part  of  the  ratio  =  .  02125 ;  log..  Table  B,  =    •     .     8. 3273589 


Half  the  value  of  the  annuity  =     .    ,    ,    .    £280    Log.  =  2. 447 1580 
Yearly  value  of  the  annuity  =•••••    £560,  as  required. 


Digitized  by 


Google 


Third, — For  Quarterly  Payments : — 


Rate,  4^  per  cent ;  ratio  =  1. 010625 ; 

log.,  Table  A,  n  0. 0045900 
Mult,  by  no.  of  paym . 

=  9year8x4=  36 

00275400 
00137700 


Product  =  .  .  0. 1 652400  Nat.  no.  1 .  4629855  Lpg.  ar.  co.  0. 3344326 
Amount  of  annuity  by  quarterly  payments=  £6 100. 515  Log.=3.  7S5366S 
Decimal  part  of  the  ratio  =: ,  010625  ;  log.,  Table  B,  =z       ,8. 0263289 

One  fourth  the  value  of  the  annuity  =     *     .     .    £140  Log.=2. 1461 280 


Yearly  value  of  the  annuity  =t   •    •    •    •    .    •    £560,  as  required. 


.    Problem  III. 

Qiven  the  Jnnuity^  the  Rate  peg  Cent ^  per  Jnnum^  and  the  Amount^  tott& 
tlie  Modes  of  Payment;  to  find  the  Time,  or  Number  of  PaymeHts. 

To  the  logarithm  of  the  amount,  add  the  logarithm  of  the  decimal  part 
of  the  ratio  (Table  B) ;  find  the  natural  number  answering  to  the  sum  of 
these  two  logarithms,  and  increase  it  by  the  whole,  the  half^  or  the  fourth 
of  the  annuity,  according  as  the  instalments  may  be  annual,  half-yeariy,  or 
quarterly.  From  the  logarithm  of  this  increased  natural  number,  subtract 
the  logarithm  of  the  whole,  the  half,  or  the  fourth  of  the  annuity,  according 
to  the  manner  in  which  the  instalments  may  be  made  payable :  divide  the 
remainder  by  the  logarithm  of  the  ratio  (Table  A),  and  the  quotient  will 
express  the  time,  or  number  of  payments }  which  will  be  in  years,  if  those 
be  annual;  otherwise,  in  half  years  or  quarters,  as  the  case  .may  be.  The 
latter  expressions  iv e  to  be  divided  by  2  or  4,  to  find  the  time. 

Example. 

An  annuity  of  £560^  at  4^  per  cent,  per  annum,  amounted,  by  annual 
pa}'ments,  to  the  sum  of  £5987*  432 ;  by  half-yearly  payments,  to  the  sum 
of  £6062.382;  but,  by  quarterly  payments,  to  the  sum  of  £6100. 5 15; 
required  the  time  corresponding  to  each  mode  of  payment } 


.]itizedbyV^OO*^l^ 


AT  COMPOUND  INTBEBST.  •     699 

First,— For  Yearly  Payments : — 

Amount  byyearly  payments 

=£5987- 432  Log.  3. 7772406 
Dec.  part  of  the  ratio, 

Tab.B, .  0425  Log.8. 6283889 

Sum  of  the  two  Iogs.=:2. 4056295  Nat.  no.  254. 46585 
Whole  annuity  for  yearly  payments   =:  £560 

Increased  natural  number  =       ...     814. 46585  Log.2. 9108729 
Whole  annuity  =  £660,  the  log.  of  which  is  =     .     .     .     2. 748 1 880 


Rate,  4^  per  cent. ;  ratio=  1 .  0425 ;  divide  by  log.  of  this, 

Table  A,  = 0.0180761)0.1626849=9 

hence  the  time,  by  annual  payments,  is  9  years. 

Second, — For  Half-yearly  Payments  :— 

Amoui(t  by  half-yearly  payments 
=£6062. 382  Log.3. 7826361 
Dec.  part  of  the  ratio. 
Tab.  B, .  02125  Log.8. 3273589 

Sum  of  the  two  logs.  2. 1099950  Nat.no.l28. 82347 
Half  the  giTMi  annuity  s;  •    .    •    .    £2(90 

Increased  natural  number  =  .     .    .      408. 82347  Log.2. 61 15358 
Half  the  given  annuity  =  £280,  the  log.  of  which  is  =  2.4471580 

Rate,  H  per  cent. ;  ratio=  1 .  02 1 25  ;  divide  by  log.  of  this. 
Table  A,  = 0.0091321)0.1643778=18 

Now,  18  half-yearly  payments,  divided  by  2,  shovf  the  time  to  be  9 
years. 


Third, — ^Por  Quarterly  Paym^ts  ;— 

Amount  by  quarterly  payments 

=£6100. 515  Log.3.  7853665 
Dec.  part  of  the  ratio. 
Tab.  B,.  010625Log.8. 0263289 


Sum  of  (be  twologs.=  1. 81 16954  N«t.po.64, 81797 

/Google 


Digitized  by ' 


Sum  of  the  two  logs.r:  1 .  81 16954  Nat.no.  64. 81797 
One  fourth  of  the  given  ahnuity  =  «     £140 

Increased  natural  number  =  .     .     .       204. 81797.  Log.  2. 31 13680 
One  fourth  of  the  annuity  =  £140,  the  log.  of  which  is  =  2. 14612S0 

Rate,  H  per  cent ;  ratio  =:  1 .  010623 ;  divide  by  log.  of 
this.  Table  A,  = 0.0045900)0.1652400=36 

Now,  36  quarterly  payments,  divided  by  4^  give  9  years  5  which,  there- 
fore, is  the  required  time. 


PRESENT  WORTH  OP  ANNUITIES  IN  ARREARS,  AT 
COMPOUND  INTEREST. 

Definiiim.^^When  an  anittu/y,  to  be  entered  on  immediately,  is  sold 
for  ready  money,  the  price  which  ought  to  be  paid  for  it  is  called  its 
present  worthy 

Problem  I. 

Gioen  an  Jnnuiiy,  the  Time  qf  its  Continuance,  and  the  Rate  per  Cent, 
per  Annum  ;  tojtnd  the  present  Worth  of  thai  Anmtky. 

RULB.  . 

Multiply  the  logarithm  of  the  ratio  fTable  A)  by  the  number  of  payments; 
subtract  the  product  from  the  logarithm  of  tlie  whole,  the  half,  or  the  fourth 
of  the  annuity,  according  to  the  mode  of  payment :  find  the  natural  num- 
ber of  the  remainder,  and  subtract  it  from  the  whole,  the  half,  or  the  fourth 
of  the  ahnuity,  according  as  the  instalments  may  be  yearly,  half-yearly,  or 
quarterly ;  find  the  logarithm  of  this  difference,  from  which  let  the  loga- 
rithm of  the  decimal  part  of  the  ratio  (Table  B)  be  subtracted,  and  the 
remainder  will  be  the  lo^rithm  of  the  present  worUi  of  the  annuity. 

Example. 

An  annuity  of  £365  is  to  be  continued  7  years;  required  the  present  value 
thereof,  by  yearly,  half-yearly,  and  quarterly  payments ;  allowing  the  pur- 
chaser 5  per  cent,  per  annum,  compound  interest,  for  the  use  of  his  ready 
money? 


Digitized  by 


Google 


AT  COMPOUND   INTBRBST,  701 


First, — ^For  Yearly  Payments : — 


Rate,  5  per  cent. ;  ratio  s=  1. 0500 ; 
log.,  Table  A,  =      .    0.0211893 
Mult,  by  no.  of  payments=  7 

Products     ....    0.1483251 

Whole  annuity,£365Log.2. 5622929  .    .    £365 


Remainders     .    .    .     2.4139678  Nat.no.  259. 39869 


Difference  between  nat.  number  and  annuity=105. 60131  Log.  2. 0236693 
Decimal  part  of  the  ratio.  Table  B,  =  .  0500        Log.  =      .    8. 6989700 


Present  worth  of  given  annuity  by  yearly  paym.  £2112. 0267  Lg.3. 3246993 

Second, — ^For  Half-yearly  Payments  :-* 

Rate,  5  per  cent;  ratio  =  1. 02500; 

log.iTableA,=      .    0.0107239 
Multiply  by  number  of 

payments  =7x2=  14 


0428956 
0107239 


Product  =     ,    .    .    .    0. 1501346 
Halfannuity,£182. 5  Log.2. 2612629  .    .    £182.5 

Remainder  =     .    ...    2. 11 1 1283  Nat.no.  129. 16009 


Diff.  between  nat.  no.  and  half  the  annuity  =     53. 33991  Log.  1 .  7270523 
Decimal  part  of  the  ratio  =  .  02500        Log.,  Table  B,  =    .     8. 39/9400 

Present  worth  of  annuity  by  half-yearly  paym.  £21331 5966  Log.3. 3291 123 

Third, — For  Quarterly  Payments  : — 

Rate,  5  per  cent. ;  ratio  =1.01 2500 ; 

log.,  Table  A,  =      .0. 0053950 
Multiply  by  number  of 

payments=7yearsx4=  28 

0431600 
0107900 


Products     .    .    .    .    0.1510600 


Digitized  by 


Google 


Products     ....    0.1510600 
One4thofann.£91.25Lg.l. 9602329  .    .    £91.25 


Remainder  =     .    .     .     1 .  809 1 729  Nat.  no.  64. 442582 


Diff.betw.nat.  num.  and  one  4  th  of  annuity  =  26.807418  Log.  1.4282550 
Decimal  part  of  the  ratio  =  .012500        Log.^  Table  B,  s     8.0969100 

Present  worth  of  the  ann.  by  quarterly  paym.  £2144. 5936  Log.3. 3313450 


Pboulbm  II. 

Given  the  present  Worth  qfan  Annuity,  the  Time  qfUs  Cqntibiuaneef  emd 
the  Rate  of  Interest ;  to  find  the  yearly  Value  of  that  Jnmdty. 

Rule. 

,  Multiply  the  logarithm  of  the  ratio  (Table  A)  by  the  number  of  pay- 
ments ;  take  the  arithmetical  complement  of  the  product,  find  the  natural 
number  corresponding  thereto,  and  let  its  arithmetical  complement  or 
difference  to  zero  be  noted.  Then,  to  the  arithmetical  complement  of  die 
logarithm  of  this  difference  to  zero,  add  the  logarithm  of  the  purchase- 
money,  or  present  worth  of  the  annuity,  and  the  logarithm  of  the  decimal 
part  of  the  ratio  (Table  B) :  the  sum  of  these  three  logarithms  (abating  10 
in  the  index,)  will  be  the  logarithm  of  the  annuity,  or  of  its  half,  or  quarter, 
according  as^the  mode  of  payment  may  be  by  yearly,  half-yearly,  or  quar- 
terly instalments.  .     . 

Example, 

The  purchase-money,  or  present  worth  of  an  annuity,  which  is  to  con- 
tinue 7  years,  amounts,  by  yearly  payments  at  5  per  cent,  per  annum,  to 
the  sum  of  £2 112. 0267 ;  by  half-yeariy  payments,  to  the  sum  o]r£2133. 5966 ; 
but  by  quarterly  payments,  to  the  sum  of  £2144. 5986 }  required  the  yearly 
value  of  the  annuity,  agreeably  to  each  mode  of  payment  ? 

First,— For  Yearly  Payments  :— 

Rate,  5  per  cent. ;  ratio  =  1 .  0500 ; 

log..  Table  A,  =  0.0211893 
Mult,  by  no.  of  paym.  7 

Products    .    .    0.1483251 


AT  COIIMUNO  IKTBRI 
Product  =    .     .    0.1483251 


Arith.  comp,  s      9. 85 16749  Nat.no.  0. 7 106 

Ar4  CO.  of  nat.  niim«,  or  diff.  to  zero  s  0. 2893 
Purchase-money^  or  present  worth  of  the  annuitj 

payments^  a  £2112. 0267  .     Log.  =      . 
Decimal,  part  of  the  ralio^  Table  B,  a  .0500 

Yearly  value  of  the  annuity  as  £365        IiOg«  i 

Second, — For  Half-yearly  Pa  | 

Rate,  5  per  cent.  ^  ratio  =:  1 .  02500  5 

log..  Table  A,  =  0. 0107239 
Multiply  by  no.  of 

paym.=7yr8x2=i  14 

0428956 
0107239 


Products:     .    .    0^501346 


Arith,  comp.  =       9. 8498654  Nat.no.  0. 70! 

Ar.  CO.  of  nat.  num.,  or  diff.  to  zero,  =  0. 29 

Purchase- money,  or  present  worth  of  the  anm 

yearly  payments,  £2133. 5966        Log.  = 

Decimal  part  of  the  ratio.  Table  B,  =:  •  O250 

Half  the  annuity  =:     .    .    £182.5        Log 
Yearly  value  of  the  annuity=:£365,  as  require< 


Third,~For  Quarterly  Pa    I 

Rate  5  per  cent  3  ratio  =  1. 012500 ; 

log.,  Table  A.,  =  0. 0053950 
Multiply  by  no.  of 

paym.= 7yrs  x  4  =  28 

U431600 
0107900 


Products  .    .    •    0.151060 


Digitized  by 


Google 


Products     .    .    0,151060 


Arith.  comp.  =      9. 8489400  Nat.  no.  0. 7062200 


Ar.  CO.  of  nat.  num.,  or  diff.  to  zero^  =r  0/2937800  Log.  ar.  ^o.  0. 5319778 
Purchase  money  or  present  worth  of  the  annuity  by  quar- 
terly payments,  £  2144. 5936        Log.  =    ' 3.3313450 

Decimal  part  of  the  ratio,  Table^B.  =  .  012500      Log. .    .    8. 0969100 

One  fourth  of  the  annuity  :s  £   91.25         Log.  =    .    .    .     1.960232S 

Yearly  value  of  the  annuity =£365,  as  required. 


Probijim  III. 

Gioefi  an  AnnuiSy^  the  present  worth  of  that  Annuity,  and  the  Mate  per 
Cent,  per  Annum  ;  to  find  the  Time  of  Us  Continuance. 

RlTLB. 

To  the  logarithm  of  the  present  worth  of  the  annuity,  add  the  logarithm 
of  the  ratio^  Table  A.,  and  find  the  natural  number  corresponding  to  their 
sum  ;— augment  tlie  present  worth  of  the  annuity  by  the  yearly  value  of 
the  annuity^  its  half,  or  quarter,  according  to  the  modes  of  payment ;  and 
find  the  difference  between  it  and  the  natural  number  found  as  above  : — 
find  the  logarithm  of  this  difference,  and  subtract  it  from  the  logarithm  of 
the  given  annuity,  its  half  or  quarter,  as  the  case  may  be  :-^ivide  the  re- 
mainder by  the  logarithm  of  the  ratio.  Table  A,  and  the  quotient  will  be 
the  number  of  payments ;  which  will  be  in  years  if  those  payments  be  an- 
nual ;  otherwise  in  halves,  or  quarters  according  to  the  modes  of  instal- 
ment. 

Example. 

An  annuity  of  £365,  can  be  purchased  for  the  sum  of  £  21 12. 0267,  by 
annual  payments;  for  £2133.5966,  by  half-yearly  payments,  or  for 
£2144.5936,  by  quarteriy  payments  ;  the  purchaser  is  to  be  allowed  5 
per  cent,  per  annum  for  the  use  of  his  ready  money ;  required  the  time  of 
the  continuance  of  that  annuity  agreeably  to  each  mode  of  payment  ? 

First, — For  Yearly  Payments  :— 

Present  worth  of  the  ann.  =  £2112. 0267  Log.  =  3. 3246993 
Rate,  5  per  cent ;  ratio.  Tab.  A.  =  1 .  0500  Log.  =  0, 02 1 1 893 

Sum  of  the  two  logs.  =  3. 345886 


iQQgjk 


Sum  of  the  two  logs. =3. 3458886  Nat.N.=2217. 6031 
Present  worth  of  the  annuity  = 

£2112,0267  +  whole  ann.  £365  =  2477.0267 


DiflFerence=     ..•.,....      259. 4236  Log.=2. 4140095 

Given  annuity  £  365,  the  log.  of  which  is  =......     2. 5622929 

Rate  5  per  cent;  ratio  =3  1.  0500;  divide  , 

bylogarithmof  this.  Table  A.  =     .    .    .    0.0211893)     0.1482834 

^  7  payments; — hence  the  time,  or ' Continuance  of  the  annuity  is  7 

years. 

Sccond^Por  Half  Yearly  Payments  :— 
Present  worth  of  the  ann.  = 

JE  21S3. 5966  Log.=3. 3291 123 
Rate  5  per  cent ; 

ratio.  Tab.  A.  = 

1. 02500  Log.=0. 0107239 

Sum  of  the  two  logii.=3. 3398362  Nat.N.=2I86. 9367 
Present  worth  of  the  annuity  = 

£2133.6966  -h  half  ann.  £  182.5  =2316.0966 


Differences 129. 1599  Log.=  2. 1111277 

Halfgivenann.=£I82.5thelog.  ofwhichis        2.2612629 

Rate  5  per  cent;  ratio  =  1,02500;  divide  

by  logarithm  of  this.  Table  A.  =  ...  0. 0107239  )  0. 1501352 
=  14  payments;  and  14  -s-'  2  =  7  years,  is  the  time  or  continuance  of 
the  annuity,  as  required. 

Third,— For  Quarterly  Payments  :«^ 
Present  worth  of  the  ann.  = 

£2144. 5936  Log.=3. 3313450  . 

Rate  5  per  cent ;  ratio 
1. 012500 
Log.Tab.A.=  0.0053950 


Sum  of  the  two  Iog8,=3. 3367400  Nat.N.=217I .  4010 
Present  worth  of  the  annuity  = 

^  2144. 5936+one  fourth 

of  the  ann.  £91.25  =    ....    2235.8436 


Differences 64.4426  Log.=  1.8091730 

One  fourth  of  the  given  ann.  =  £  91. 25,  the  log.  of  which  is=:  1. 9602329 
Rate  5  per  cent ;  ratio  =  1 .  012500  :  divide  • 

by  logarithm  ofthis,  Table  A.  =  .  .  .  0.0053950)  0.1510599 
=  28  payments ;  now  28  -*-  4  =  7  years,  which,  therefore,  is  the  time 
required. 

22 


PRESENT  WORTH  OF  ANNUITIES  IN  REVERSION,  AT 
COMPOUND  INTEREST. 

• 

D^ithfi.'^An  annuity  is  said  to  be  in  retyermn  when  it  is  not  to  b^ 
entered  upon  until  some  particular  etent  has  happened^  or  until  some  oer* 
tain  period  has  elapsed  after  the  .time  of  its  sale  :-^the  ready  money  which 
should  be  paid  down  for  an  anntiity  of  this  dcscriptioh  is  called  its  present 

worth. 

pROmjftM   I. 

Given  an  Annuity  in  Reversion,  the  Time  of  iU  Continuance^  the  Period 
at  which  it  is  to  be  entered  upon,  with  tlw  Rate  of  Interest;  to  find  Us 
present  Worth. 

RULB. 

Find  the  logarithm  of  the  present  worth  of  the  annuity  agreeably  to  the 
time  of  its  continuance^  as  if  it  were  to  be  entered  upon  immediately,  by 
Problem  I.,  page  700  }  from  this  logarithm  subtraet  the  logarithm  of  the 
ratio.  Table  A.,  multiplied  by  the  time  which  is  to  elapse  before  the  pur- 
chaser enteni  tipon  the  annuity,  and  the  remainder  will  be  the  logarithm  of 
the  present  worth  of  the  annuity  in  reversion. 

Noiet^f  the  annuity  be  made  payaUe  by  half-yearly,  or  quarteriy  m- 
stalmentSi  the  ratio,  &c.  &c*  are  to  be  proportioned  accordingly,  the  same 
as  in  the  preceding  Problems. 

Example, 

What  is  the  present  worth  of  the  reversion  of  a  lease  of  £475  per  an- 
num, by  yearly  payments,  to  continue  25  years,  but  not  to  be  entered  upon 
till  the  end  of  7  years  after  the  time  of  sale ;  allowing  the  purchaser  5|  per 
cent,  for  present  payment  ? 

Rate  5f  per  cent. ;  ratio,  Tab«  A.  &&  1. 0550  Log.a    «    .    .0. 0232525 
Multiply  by  number  of  payments  =•...••••.••   25 

1162625 
0465050 

Product  == 0.5813125 

Annuity,  or  lease  =  £  475, Logarithm  =     2. 6766936 

Natural  Number  sea      124.5607     .     .    •    .Logarithms     2.0953811 
Differences   .    .      350.4393 


and  annuity^  or  lease  =  .  .  350.4393  Log.  =  2.5446128 
Decimal  part  of  the  ratio,  Tab.  B=.  0550  Log.  =  8. 7403627 
Logarithm  of  the  present  worth  of  the  lease,  sup-         

posing  it  were  to  be  entered  upon  immediately  s=3. 8042501ss:£6371«6 
Rate  5^  per  eent. ;  ratio,  Tab.  A.ss  1. 0550  Log.a 

Ol  0232525  X  7  years  =     ......    .0.1627675 

Present  worth  of  the  rev.  «  £4380.0848    Log.sS.  6414826 

Note. — The  latter  part  of  the  operation  may  be  abridged  in  the  follow- 
ing manner,  viz  : — 

To  the  logarithm  of  the  difference  between  the  natural  number  and  the 
gjyen  annuity  or  lease,  add  the  arithmetical  complement  of  the  logarithm  of 
the  decimal  part  of  the  ratio.  Table  B.,  and  the  arithmetical  complement 
of  the  product  of  the  logarithm  of  the  ratio,  Table  A.,  by  the  time  which  is 
to  elapse  before  the  purchaser  enters  upon  the  annuity;  the  sum  of  these 
three  logarithms,  abating  10  in  the  index,  will  be  the  logarithm  of  the  pre- 
sent worth  of  the  annuity  or  lease  in  reversion. 

Thus.  Diff.  betw.nat.  numb,  and  ann.  or  lease=350. 4393  L.=2. 5446128 
Pec.  part  of  ratio.  Tab.  B.=.  0550  Log.  arith.  comp.=  1.  2596373 
Ratio,Tab. A.=l .  0550  L,=0. 0232525  x  7  ys.  Ar.  com.  9. 8372325 

Present  worth  of  the  reversion  =£4380. 0848    Log.s  3«  64 14826 


PaoBLEM  II. 
Given  the  present  Worth  of  an  Annuity  in  Rev'ersion,  the  time  of  its  con* 
tinuancej  and  the  Period  at  which  it  ia  to  be  entered  upon,  with  tlie 
Rate  of  Interest ;  to  find  the  yearly  value  of  the  Annuity. 

RuL?. 
Multiply  the  logarithm  of  the  ratio.  Table  A.,  by  the  number  of  pay- 
ments or  time  of  continuance  :  take  the  arithmetical  complement  of  the 
product,  and  find  the  natural  number  corresponding  thereto,  and  let  its 
arithmetical  complement,  or  difference  to  zero  be  noted  :-^theii,  to  the 
arithmetical  complement  of  the  logarithm  of  this  difference  to  zero,  add  the 
logarithm  of  the  decimal  part  of  the  ratio.  Table  B.,  the  product  of  the  log" 
arithm  of  the  r&tio.  Table  A.  by  the  time  that  is  to  elapse  before  entering 
upon  the  reversion,  and  the  logarithm  of  the  present  worth  of  the  reversion ; 
the  sum  of  these  four  logarithms,  abating  10  in  the  index,  will  be  the  log- 
arithm of  the  yearly  value  of  the  annuity. 

Note. — If  the  annuity  be  made  payable  by  half-yearly,  or  quarterly  in- 
stalments, the  ratio,  &q.  &c,  are  to  be  proportioned  accordingly,  the  same 
as  in  the  preceding  Problems.  - 

2z  2 


Example. 

What  is  the  yearly  value  of  an  annuity  or  lease,  to  be  entered  upou  7 
years  hence,  and  then  to  continue  18  years,  that  is  25  years  continuance, 
and  which  may  be  bought  for  the  sum  of  £  4380. 0648,  ready  money, 
allowing  the  purchaser  5i  per  cent,  compound  interest  ? 

Rate5§  percent;  ratio,Tab.  A.  =  1.0550  Log.  =    .    .    .0.0235525 
Multiply  by  number  of  payments  = 25 

1162625 
0465050 

Products •    •    0-5813125 

Arithmetical  complement  = 9.4186875 

The  natural  number  answering  to  arithmetical  complement, 
considered  as  a  Logarithm,  is  =0. 2622331 

Difference  to  zero  =^  -  .  .  0. 7377669  Log.  ar.  comp.  0. 13^0808 
Dec.  part  of  the  ratio.  Tab.  B.  =  .  0550  Logarithm=  8. 7403627 
Ratio,  Table  A.=l.  0550  Log.=  0. 0232525  x  7  years  =  0. 1627675 
Present  worth  of  the  reversion  =  £4380.0848  Logarithm  =  3.6414826 

Yearly  value  of  the  annuity  or  lease  =  £  475       Logaritlim  =  2. 6766936 


Problem  III. 

Gftt?^  an  Annrnty  in  Reversio?},  its  present  Wofth^  the  Period  at  txMch  it 
is  to  be  entered  ttpon,  and  the  Rate  of  Interest ;  to  find  the  Time  of  its 
Continuance. 

RULB. 

To  the  logarithm  of  the  present  worth  of  the  reversion,  add  the  product 
of  the  logarithm  of  the  ratio,  Table  A.,  by  the  time  that  is  to  elapse  before 
entering  upon  the  annuity,  and  the  logarithm  of  the  decimal  part  of  the 
ratio,  Table  B  ;  the  sum  of  these  three  logarithms,  abating  10  in  the  index, 
will  be  the  logarithm  of  a  natural  number.  Take  the  diflference  between 
the  natural  number,  thus  found,  and  the  yearly,  half-yearly,  or  quarterly 
vijue  of  the  annuity,  according  to  the  nature  of  the  instalments ;  subtract 
the  logarithm  of  this  difference  from  the  logarithm  of  the  yeariy,  half- 
yearly,  or  quarterly  value  of  the  annuity,  as  the  case  may  be :  divide  the 
remainder  by  the  logarithm  of  the  ratio.  Table  A.,  and  the  quotient  will  be 
the  whole  time  of  the  continuance  of  the  given  annuity  or  lease ;  which 
will  be  in  years,  hdves,  or  quarters,  according  to  the  mode  of  payment. 


mmgm 


An  annuity  or  lease  worth  £  475  per  annum  (to  be  entered  upon  at  the 
end  of  7  years  after  the  time  of  sale,)   may  be  purchased  for  the  sum  of 
^4380.0848;    required  the  time  of  the  continuance  of  that  annuity, 
allowing  the  purchaser  5^  per  cent,  for  the  use  of  his  ready  money  } 

Present  worth  of  the  annuity  or  lease=£4380. 0848  Log.  =  3. 6414826 
Ratio,  Tab.  A.=  1. 0550  Log.=0. 023252&X 7  yrs.    (Rate 

Si  per  cent.)  = 0. 1627675 

Decimal  part  of  the  ratio,  Tab.  B.  =  •  0550  Logarithm  =     8. 7403627 


JNTatural  number  = 


350. 4393     .  Logarithm  =     2. 5446128 


Yearly  value  of  given  ann.s  •  475. 


Logarithm  =     2. 6766936 


Difference  = 124.5607       Logarithms     2.0953811 

Rate  5  i  percent;  ratio.  Table  A.=  1 .  0550 ;  divide  by 

logarithm  of  this  = 0.0232525)    0*5813125  = 

'  25  payments ; — hence  the  tinie»  or  continuance  of  the  annuity  is  25 
years,  as  required* 

JN'ote, — Should  it  be  required  to  find  the  time  that  should  elapse  between 
the  perio(l3  of  purchasing  and  entering  upon  the  annuity  or  lease,  it  may 
be  very  readily  determined  by  an  indirect  solution  of  Problem  I.,  page 
706  ; — as  thus  : 

From  the  logarithm  of  the  present  worth  of  the  amiuity  determined  as 
if  it  were  to  be  entered  upon  immediately,  subtract  the  logarithm  of  the 
present  worth  thereof  in  reversion : — divide  the  remainder  by  the  logarithm 
of  the  ratio,  Table  A.,  and  the  quotient  will  be  the  time  required ; — ^this^ 
it  is  presumedi  is  so  very  obvious  as  not  to  require  the  illustration  of  an 
exannple. 


PRESENT  WORTH  OF  FREEHOLD  ESTATES,  OR  PERPETUAL 

ANNUITIES, 

At  an  Assigned  Rate  per  Cent.  Compound  Interest,  to  be  entered  on 

immediately. 

DefinUion.^Freehold  Estates,  or  perpetual  jinnuitiessignify  any  interest 
of  money,  rents,  pensions,  grants,  &c.  payable  yearly,  half  yearly,  or  quar- 
terly, and  to  continue  for  ever:— in  buying  these,  the  purchaser  is  allowed 
a  certain  per  centage  for  his  ready  money ;  which  money  is  called  the  pre^ 
sent  worth  of  the  perpetual  annuity,  to  be  entered  on  immediately. 


Paoblbm  L 

Given  the  Yearly  Bent  qf  a  freeliold  Estate ^  or  perpetual  Annuity,  Ofid 
the  Rate  per  Cent.;  to  find  the  present  fVarth  thereof. 

RULB. 

From  the  logarithm  of  the  yearly,  half-yearly,  or  quarterly  ralue  of  the 

f  given  perpetual  annuity,  according  to  the  mode  of  payment  (the  index 

being  increased  by  10),  subtract  the .  logarithm  of  the  decimal  part  of  the 

ratio  corresponding  to  such  payment.  Table  B.,  and  the  remainder  will  be 

the  logarithm  of  the  present  worth  of  the  given  perpetual  annuity,  &c« 

Example. 

What  is  the  present  worth  of  a  freehold  estate,  or  perpetual  annuity  of 
£360,  per  annum,  payable  half  yearly,  allowing  the  purchaser  4|  per  cent, 
compound  interest,  for  his  ready  money  ? 

Given  perpetual  annuity=£360 ;  half  of  which  is  £180  Log.s2. 2552725 
Rate,  4}  per  cent.;  dec.  part  of  ratio.  Tab.  B^.  02375  Log.^8«  3756636 

Present  worth  of  the  given  freehold  estate=£7578. 93  Log.=3. 8796089 


Pr6bl|im  II. 

Gtioen  the  present  Jf^orth  of  a  fteehM  EsMe,  or  perpetual  jhmuUyj  and 
the  Bate  per  Cent.;  to  find. the  yearhf  Value  of  that  Estate,  or  per* 
petual  Annuity. 

Rule, 

To  the  logarithm  of  the  present  worth  of  the  given  perpetual  annuity, 
add  the  logarithm  of  the  decimal  part  of  the  ratio  (Table  B),  according  to 
the  mode  of  payment ;  and  the  sum  will  be  the  logarithm  of  the  yearfy 
value  of  the  annuity,  or  of  its  half  or  quarter,  as  the  case  nay  be. 

Example. 

If  a  freehold  estate,  or  perpetual  annuity,  be  bought  for  £7578. 93,  what 
ought  the  yearly  value  thereof  to  be,  allowing  the  pureheaer  4}  per  cent, 
compound  interest,  for  his  ready  money ;  the  rent  being  payable  half- 
yearly  ? 

Present  worth  of  the  given  perpetual  annuity=£7578. 93  Log.  3. 87960S9 
Rate,  4f  per  cent. ;  decimal  part  of  the  ratio.  Table  B,  = 

•  02375  (half-yearly  payments)         Log.  =:   .    *    •    .    •    8.3756636 


Half-yearly  value  of  the  annuity  s  £180        Log.=  .    .    .    2.2552725 
Yearly  rent  of  the  freehold  e8tate=:£360^  as  required. 


OR  FERPBTUAL  ANN! 


Problbm  III 


Oilmen  4he  yearly  Value  of  a  freehold  Etta 
the  presefit  tVorth  thereof;  to  Jim 

.    RULR. 

From  the  logarithm  of  the  yearly,  half-y 
given  perpetual  annuity,  according  to  the 
being  increased  by  10),  subtract  the  logari 
that  annuity,  and  the  remainder  will  be  the 
of  the  ratio :  with  thia  enter  Table  B.,  and  th 
thereto  will  be  found  standing  abreast  there 
Table  A.j  which  will  be  the  ratte  per  cent,  re 

Example. 

If  a  freehold  estate,  or  perpetual  annuity 
half-yearly,  be  sold  for  £  7576. 93,  what  i: 
interest,  allowed  to  the  purchaser  ? 

Yearly  Value  of  the  annuity,  £860,  half  of  wl 

Present  worth  of  ditto  ss  £7578. 93.  the  lo 

Rate,  4|  per  cent..  Table  A,  =  decimal  pan 

Table  B,  «  .02375        Log.  a     ,    . 


PRESENT  WORTH  OF  FREEHOLD  E 
ANNUITIES  IN  REA 

At  an  assigned  Rate  per  Cent.,  ' 

D^fmiion^^Freehold  estates,  or  perpefuc 
any  interest  of  moneys  rents,  pensions,  gra 
yearly,  or  quarterly,  and  to  continue  for  e 
entered  upon  until  some  particular  event 
specified  time  has  elapsed  after  the  time  o( 
purchaser  holds  the  freehold,  &c.,  after  he 
the  reversion ;  and  the  money  which  he 
called  its' present  worth. 


Digitized  by 


Google 


712  PRBSBNT  WORTH   OF  FRSEHOLD  BSTATBS, 

Problem  I. 

Given  the  yearly  Rent  of  a  freehold  EsiatCy  or  perpetual  Amnaiy^  the 
Time  at  which  it  is  to  be  entered  upon,  and  the  RaJLe  per  Cent. ;  to  find 
the  present  Worth  of  the  Reversion  of  that  Estate. 

Ruu. 

To  the  product  of  the  logarithm  of  the  ratio  (Table  A)  by  the  time  that 
is  to  elapse  before  entering  upon  the  reversion,  add  the  logarithm  of  the 
decimal  part  of  the  Tatio  (Table  B) :  subtract  the  sum  of  these  two  loga- 
rithms from  the  logarithm  of  the  yearly  value  of  the  given  perpetual 
annuity  (the  index  being  increased  by  10),  and  the  remupder  will  be  the 
logarithm  of  the  present  worth  of  that  perpetual  annuity,  or  freehold 
estate. 

Example. 

The  reversion  of  a  freehold  estate^  or  perpetual  annuity  of  £490  per 
annum,  to  be  entered  upon  7  years  hence,  is  to  be  sold ;  what  is  its  present 
worth,  allowing  the  purchaser  5  per  cent*,  compound  interest,  for  his  ready 
money  ? 

Rate,  5  per  cent. ;  ratio.  Table  A,  =  1 .  0500        Log.  =     •    0. 021 1893 
Mult,  by  given  time,  viz.,  the  time  before  entering  upon  the  revers.  7 

Products 0.1483251 

Decimal  part  of  the  ratio.  Table  B,  =  .  0500        Log.  =    .    8. 6989700 


Sum  of  the  two  logarithms 8.8472951 

Yearly  value  of  the  estate,  or  perpetual  annuity,=:£490    Log.=2. 6901961 

Present  worth  of  the  reversion  r^  £6964. 6774        Log.  =  •    3. 8429010 


Problbm  IL 

Given  the  present  Worth  of  a  freehold  Estate^  or  perpetual  Annuity  in 
Reversion^  the  Time  at  which  it  is  to  be  entered  upon,  and  the  Rate  per 
Cent.;  to  find  its  yearly  Value. 

RULB. 

To  the  product  of  the  logarithm  of  the  ratio  (Table  A)  by  the  time  that 
is  to  elapse  before  entering  upon  the  reversion,  add  the  logarithm  of  the 
decimal  part  of  the  ratio  (Table  B),  and  the  logarithm  of  the  present  worth 
of  the  reversion  of  the  given  perpetual  annuity :  the  sum  of  these  three 
logarithms  (abating  10  in  the  index^}  will  be  the  logarithm  of  the  }'early 
value  of  that  perpetual  annuity. 


Digitized  by 


Google 


If  a  freehold  estate,  or  perpetual  annuity,  to  be  entered  upon  7  years 
hence,  be  sold  for  £  6964, 6774,  what  is  the  yearly  value  thereof,  allowing 
tlie  purchaser  5  per  cent.,  compound  interest,  for  his  ready  money  ? 

Rate,  5  per  cent. ;  ratio.  Table  A,  =s  ] .  0500        Log.  =      .    0. 02 1 1 893 
JVf  ult.  by  given  time,  viz.,  the  time  before  entering  upon  the  revers.  7 

Product  = 0.1483251 

Decimal  part  of  the  ratio.  Table  B,  =  .  0500        Log.  =     .     8. 6989700 
Present  worth  of  the  reversion=jE;6964. 6774        Log.  =     .    3. 8429010 

Yearly  value  of  the  given  perpetual  annuity  ^  £490        Log.=2. 6901961 


Problbm  IIL 

Given  the  yearly  Value  of  a  freehold  Estate^  or  perpetual  Amuity  in 
Reversion,  ilie  Rate  per  Cent,,  and  its  present  Worth;  to  find  the  Time 
that  must  elapse  before  entering  upon  the  Reversion, 

Rule. 

To  the  logarithm  of  the  present  worth  of  the  reversion,  add  the  loga- 
rithm pf  the  decimal  part  of  the  ratio  (Table  B) ;  subtract  the  sum  of 
these  two  logarithms  from  the  logarithm  of  the  yearly  value  of  the  given 
perpetual  annuity  :  now,  the  remainder  being  divided  by  the  logarithm  of 
the  ratio  (Table  A),  the  quotient  will  express  the  time  that  must  elapse 
before  entering  upon  the  reversion  of  that  perpetual  annuity. 

Example, 

If  a  freehold  estate,  or  perpetual  annuity  of  £490  per  annum,  be  sold  for 
£6964. 6774,  in  what  time  hence  will  the  purchaser  be.  entitled  to  enter 
thereon,  allowing  him  5  per  cent.,  compound  interest^  for  the  use  of  his 
ready  money  ? 

Pres.  wprth  of  the  revers.  of  given  annuity=6964. 6774  Log.  3. 8429010 
Rate,  5  per  cent ;  dec.  part  of  ratio,  Table  B,    0. 500  Log.  8. 6989/00 

Sum  of  the  two  logarithms  = 2.5418710 

Yearly  value  of  the  given  perpetual  annuity    £  490     Log.  =  2. 6901 96 1 

Rate,  5  per  cent  j  ratio,  Table  A,=  1 .  0500;  divide  by  log,  

of  this  == .    0.0211893)0.1483251=7 

.    Hence,  the  purchaser  will  be  entitled  to  enter  upon  the  reversion  at  the 
end  of  7  years. 


PftOBLBM. 

To  find  in  hcuo  many  Years  any  Principal  or  Sum  of  Money  wiU  dauUe 
itself,  at  compound  Interest,  by  yearly,  half-yearly,  or  quarterfy 
Payments. 

Let  the  logarithm  of  the  ratio  (Table  A)  be  considered  as  the  decimal 
part  of  a  natural  number  \^  fiod  the  logarithip  corr^ponding  thereto^  and 
subtract  it  from  the  constant  logarithm  9*  4786093  :  the  remainder  will  be 
the  logarithm  pf  the  time  in  which  a  given  sum  of  money  will  double  iUelf 
at  any  proposed  rate  of  interest  within  the  limits  of  Table  A :  if  the  pay- 
ments of  interest  be  annual^  the  time  will  be  expressed  in  years ;  odierwise, 
in  half  years^  or  quarters  of  years^  as  the  case  may  be. 

Note.  —The  constant  logarithm  is  thus  determined :— Let  the  drnMe  of 
any  given  sum  of  money  be  represented  by  the  number  2,  the  logarithm  of 
which  is  0. 3010300;  consider  this  as  the  decimal  part  of  a  natural  num- 
ber: then^  the  logarithm  corresponding  thereto  is  9.47860983  which, 
therefore,  becomes  a  constant  expression  for  all  modes  of  payment  and 
ratos  of  interest. 

Example* 

Required  the  number  of  years  in  which  any  given  sum  of  money  will 
double  itself,  at  compound  interest,  by  yearly^  half-yearly,  and  quarterly 
payments ;  the  rate  being  5  per  cent,  per  annum  ? 

First,— For  Yearly  Payments  :— 

Constant  log.  = 9.4786098 

Rate,  5  per  cent ;  log*,  Tsj^lt  A,  S7  0»  021 1893 ;  consider  this 
as  the  decimal  part  of  a  natural  number,  the  log»  of  which  \s^%»  3261 167 

Number  of  years,  as  required,  =  14. 2067         Log.  ss    .    .     1. 15249S1 

Second, — For  Half*yearly  Payments  :— 

Constant  log.  =3 9.4786098 

Rate,  5  per  cent.;  log.  Table  A,  =  0. 0107239 ;  consider  this  as 

the  decimal  part  of  a  natural  number,  the  log.  of  which  is  =     8. 0303528 

Time,  in  half  years,  cs     .    .    .    28. 07094        Log.  s:     .     h  4482570 

Number  of  years,  as  required,  =  14.03547;  which,  therefore,  is  the 
time  in  which  a  sum  of  money  will  double  itself,  at  5  per  cent*  ccnnpoimd 
interest,  by  half-yearly  payments. 


Digitized  by 


crop* 


OR  PBRPETUAL  ANVUmX3  IN 


Third,— For  Quarterly  Pa 

Constant  log,  =       .    •    • 

Rate,  5  per  cent.;  log., Table  A,=0. 005395( 
the  decimal  part  of  a  natural  number,  the  log 

Time,  in  quarters  of  years,  ps  55.  79797 


Number  of  years;  as  required,  13^  94949 ;  whi 
which  a  sum  of  money  will  double  itsel 
ioterest^  by  quarterly  payments* 

It  is  after  this  manner  that  the  followin 
excepting,  however,  the  last  column,  or  that 
merely  expressed  by  the  quotient  of  £100,  d 
cent. :  thus,  £100  -+-  £5  =  20  years ;  which, 
a  given  sum  of  money  will  double  itself,  at  5 
interest. 

A  Tablb, 

Exhibiting  tlie  Time  ia  which  any  Sum  of 
several  given  Rates  per  Cent,  per  Annum, 
at  Simple  Interest. 


Rates 
Crat. 

Time,  for  compouod  Interes 

Yearly 
Payineots. 

Half.yearly 
Payments. 

£ 
3 

H 
3i 

4 
44 

4i 
4| 
5 

f 
? 

8 

9 

10 

.Years. 
23. 4498 
21.  6723 
20. 1488 
18. 8284 
17.  6730 
16. 6535 
15.7473 
14.9365 
14.2067 
13. 5464 
12.  9461 
12.3981 
11.8957 
11.0067 
10. 2448 
9. 0065 
8.0432 
7.2725 

Years. 

23.2779 
21. 5003 
19.  9770 
18. 6567 
17.5013 
16.4820 
15.5759 
14.  7652 
14.0355 
13.3753 
12. 7752 
12.2273 
11.7249 
10.8361 
10.0744 
8. 8365 
7. 8736 
7.  1033 

By  the  above  table  it  is  evident,  that  if  £ 
compound  interest,  at  the  rate  of  5  per  ce 
itself,  by  yearly  payments  of  the  interest, 


Digitized  by 


Google 


716  COMPOUND   INTERBST. 

payments^  in  14  years  j  and  by  quarterly  payments,  in  13^  years ;  whilst 
at  simple  interest,  it  will  not  double  itself  in  less  than  20  years.  Hence, 
if  the  given  sum  be  £1250,  it  will  amount,  in  13-^^  years,  by  quarterly 
payments,  to  the  sum  of  £2500 ;  in  21 -fc  years,  to  the  sum  of  £5000 ;  in 
41-/V  years,  to  the  sum  of  £10000 ;  in  55/^  years,  to  the  sum  of  £20000  ; 
and  so  on  in  geometrical  progression :  while  at  simple  interest,  the  same 
sum  would  only  amount,  in  the  same  space  of  time,  viz.,  55-^  years,  to  the 
sum  of  £4737.  7s.  6d. 

jRcmarfc.— The  preceding  problems  and  examples  contun  all  that  is 
essentially  necessary  to  be  known,  independently  of  theory,  in  the  doctrine 
of  compound  interest.  The  author  is  not  aware  that  the  direct  loga- 
rithmical  solutions  of  the  various  complex  cases  connected  with  this 
subject  have  been  given  by  any  other  writer  :  many,  indeed,  have  published 
theorems  for  this  purpose ;  but  these  theorems  (such  as  those  given  by 
the  late  ingenious  Dr.  Maskelyne,  in  his  very  learned  Introduction  to 
Taylor's  Logarithms,  under  the  head  Compound  Interest,)  are  expressed  in 
such  a  scientific  manner  as  to  be  of  little  use,  in  a  mere  practical  point 
of  view ;  being  much  better  adapted  for  employing  the  minds  of  the  cu- 
rious in  mathematical  researches  and  investigations,  than  for  abridging 
the  labour  attendant  on  arithmetical  computations. 


Digitized  by 


Google 


THE  USE  OF  THE  GENERAL  VICTUALLING  TABLE, 

Contained  in  Vol.  II.,  Page  661. 

As  this  Table  contains  the  exact  daily  proportion  of  sea  provisions  for 
any  given  number  of  men  within  the  ordinary  limits  of  victualling,  it  will 
"be  found  of  considerable  utility  to  the  Pursers  of  the  Royal  Navy,  in 
closing  their  annual  accounts,  and  in  completing  the  ship's  provisions  to 
any  specified  time :  it  will  also  be  of  great  use  to  officers  serving  as 
Commanders  and  Pursers ;-— and,  perhaps,  to  those  gentlemen  in  the 
Victualling  Department  of  His  Majesty's  service  who  are  employed  in  the 
examining  and  auditing  of  the  Naval  Victualling  accounts  of  the  above- 
mentioned  officers* 

Remarks. 

1 . — ^As  the  size  of  the  page  would  not  admit  of  separate  columns  being 
employed  for  the  salt  beef  and  salt  pork,  and  as  the  allowance  of  each  of 
these  species  is  precisely  the  same  ;  one  column  only  has  been  introduced 
into  each  page  of  the  Table  on  account  of  those  articles  of  victualling  5 — 
which  column  contains  the  exact  proportion  of  each. — Hence,  in  taking  out 
the  proportions  from  this  column,  corresponding  to  any  given  number  of 
men,  care  must  be  taken  to  put  down  such  proportions  ixvice ;  that  is,  first 
for  salt  beef,  and  then  for  salt  pork ;  or,  otherwise,  to  double  those  propor- 
tionsy  at  once,  for  salt  meat  generally. 

2. — In  like  manner,  as  the  size  of  the  page  would  not  admit  of  separate 
columns  being  employed  for  bread  and  beer,  and  for  oatmeal  and  vinegar ; 
it  will  be  necessary,  in  taking  out  the  proportions  of  those  species,  corres- 
ponding to  any  given  number  of  men,  to  put  down  as  many  gallons  of  beer 
as  the  gecoJid  column  expresses  pounds  of  bread ;  and  as  many  gallons  of 
vinegar  as  the  last  column  expresses  gallons  of  oatmeal. 

The  following  problems,  will  illustrate  the  principal  uses  to  which  this 
Table  may  be  applied. 

PaofiLEM  I.  ^ 

Given  the  Number  of  Men  victualled  for  one  Day,  to  find  tlie  corresponding 
Proportion  of  each  Species  of  Provisions. 

Rule* 

If  the  given  number  can  be  found  in  the  left-hand  column  of  the  table, 
the  corresponding  proportion  of  each  species  of  provisions  will  be  found 
abreast  of  it  in  the  same  horizontal  line  j  but  if  it  cannot  be  exactly  found, 
which  in  general  will  be  the  case,  write  down  any  two  or  more  of  the 


tabular  numbers  that  will  make  up  the  given  one,  opposite  to  which  pat 
down  the  corresponding  quantities  of  provisions :  then,  the  sums  of  these 
quantities  will  be  the  true  proportion  of  each  species  of  provisions. 

Example. 

Let  the  number  of  men  victualled  for  one  day  be  45685 ;  reqiured  the 
true  proportion  of  each  species  of  provisions  corresponding  thereto  ? 

Men.  Bread.    Beet'   Salt  Meat.     Flour.     Peaae.      Sagar.      Coeoa.     Tea.    OatmatL      ViMgar. 

45000  ^vc  46600  45000  33750.0  16875.0  1406.2  4218.12  2S12.8  703.2  401.6.4  401. 6L   4 

600  do.      600      600      450. 0      225. 0      18. 6        56. 4      37. 8      9. 6    6. 2. 12      5. 2. 12 

85  do.        85        85      63.12      31.14      2.5^      7.15}    5.5      1.5(0.6.1      0.6.1 


45685  give  45685  45685  34263.12  17131.14  1427.5^  4282.15}  2855.5  713.13^  407.73  407. 7.3 

Note. — In  making  out  the  Purser's  annual  victualling  account,  the  pro- 
portions of  salt  beef  and  pork,  as  given  in  the  table,  are  to  be  doubled,  or 
thrown  into  one  sum  under  the  head  of  salt  meat,  as  above. 

If  there  be  miy  fresh  meat  issued  during  the  period  of  the  account,  sub- 
tract the  amount  thereof  from  the  number  victualled,  and  then  take  out 
the  proportion  of  salt  meat,  flour,  and  pease^  corresponding  to  the  remain- 
der :  thus,  suppose  the  quantity  of  fresh  meat  issued  to  be  22238  pounds. 

No.  vie.  for  one  day  is  45685 
Lbs. fr. meat  issued^:  22238 

— — —  Salt  meat.    Flour.    Pease. 

Remainder  =       .     23447.  Now, 23000 gives  17250.0  8625.   0  718.6 

400  do.      300.0    150.   0     12.4 
47  do.         35.4       17.10      1.3| 

23447  gives  17585. 4  8792.10  732. 5i 
See  Pursers'  Instructions  (Appendix),  No.  21,  page  11 7* 


Problem  II. 

Given  the  Complement  of  Men,  and  the  Number  of  Days  for  which  they 
are  to  be  victualled;  to  find  the  Proportion  of  each  Spedes  of  Pro- 
visions. 

Rule. 

Multiply  the  complement  of  men  by  the  given  number  of  days  for  which 
they  are  to  be  provisioned,  and  the  product  will  be  the  number  to  be 


Digitized  by 


erwgr 


directed  in  the  last  problem* 

Example. 

Let  the  complement  of  a  ship  be  275  men,  and  the  time  for  which  they 
a.re  to  be  victualled  4  lunar  months  or  112  days;  required  the  propprtion 
of  each  species  of  sea  provisions  ? 

Given  complement  of  mens  275 
M ultlply  by  time,  in  days^ss     1 1 2 

Product  =       •     .    «    •    30800,  which  tsjthe  number  to  be  victualled  for 

one  day. 

M«B.        Bread.  Spitits.  Salt  Beef.  Salt  pAfk.  Flour.  Peaia.    Sugar.   Cncoa.     Tea.      Oatmeal.     Vinega' 

3OOOO^ve30000  937.4    11250    11250   11250  937.4   2812.8    1875  468.12  267.6.12  267.6.12 

800  do.      800     25.0        300        300       300    25.0       76.0.     50       12.8      7.1.  2f      7.1.2 


30800  g^ive  30800  962.4    11550     11550  11550  962.4  2887.8   1925     481.4    275.0.0     275.0.0 

And  these  are  the  exact  proportions  of  the  different  species  of  provisions 
for  the  given  complement  and  time.     * 

RenfKirks. 

1.  Since  the  salt  beef  is  generally  cut  up  in  8  lbs.  pieces,  and  the  pork 
in  4  lbs.  pieces,  the  pounds  of  salt  beef  are  to  be  divided  by  8,  and  the 
pounds  of  salt  pork  by  4  :  the  respective  quotients  will  be  the  number  of 
pieces  of  each  species.  Thus,  in  the  above  case,  tlie  number  of  pieces  of 
salt  beef  is  1443^,  and  of  pork  2887^. 

2.  It  being  customary  to  substitute  a  proportion  of  raisins  and  suet  for  a 
part  of  the  flour,  a  deduction  is  to  be  made  from  the  full  allowance  of 
the  latter,  on  account  of  such  quantities  of  those  substitutes  as  it  may  be 
deemed  advisable  to  demand  from  the  victualling  stores ;  observing  that 
one  pound  of  raisins  is  equal  to  one  pound  of  flour,  and  that  half  a  pound 
of  currants  or  half  a  pqund  of  suet  is  to  be  considered  as  being  equal  to  one 
pound  of  raisins  or  one  pound  of  floun 

3.  As  tobacco  and  soap  are  directed  to  be  issued  to  the  ship's  company, 
in  the  proportion  of  two  pounds  of  the  former  and  qhc  pound  of  the  latter 
per  man  per  lunar  month,  the  Purser  Is  to  include  those  articles  in  his 
demand  for  provisions  from  the  Victualling  Agents ;  but  it  is  to  be  ob- 
served, that  he  must  only  demand  as  much  tobaccco  and  soap  as  will 
answer  for  the  two-thirds  of  the  complement,  agreeably  to  the  time  for 
which  the  ship  is  ordered  to  be  provisioned. 


If  the  complement  of  men  be  multiplied  by  2,  and  the  product  divided 
by  3,  the  quotient  will  be  the  two-thirds  of  the  complement :  this,  being 
multiplied  by  the  number  of  months  for  which  the  ship  is  ordered  to  be 
victualled^  will  give  the  number  of  pounds  of  soap ;  the  double  of  which 
will  be  the  number  of  pounds  of  tobacco.  Thus^  in  the  above  case^  where 
the  complement  is  275  men^  and  the  time  4  lunar  months ; — 

Now,  275  Baen  x  2-^3=  183  J,  which  is  the  f  of  the  complement. 
Multiply  by  no.  of  months  =     4 

Product ;     733^  =  pounds  of  soap. 

Double  of  ditto  •     •    •    •  146^f  s  pounds  of  tobacco. 


I'HB  END   OF  VOLUME  T. 


Printed  by  MUU,  Jowctt,aad  MUlt,  Bolt-coait,  Flect-ttrcet 


Digitized  by 


Google 


ERRATA 

Page  141,  Note,  line  4,  for  "kt,  50?  Si 

Page  463,  line  6  from  the  bottom,^  '^ii 

2. 5L  I ''  ready  moon's  corrected  ri(; 


Digitized  by 


Google 


Just  publisfiedf  tlie  Second  EdUiony  in  royal  8vo.y  price  lis.  boards. 


OF 


To  the  Sidereal  and  Planetary  parts  of  NAUTICAL  ASTRONOMY : 
being  the  theory  and  practice  of.  finding  the  LATITUDE,  the  LON- 
GITUDE, and  the  Variation  of  the  Compass  by  the  FIXED  STARS 
and  PLANETS.  To  which  is  prefixed  the  description  and  use  of  the 
NEfF  CELESTIAL  PLANISPHERE. 

By  THOMAS  KERIGAN.  R.  N. 

LONDON  '.  —  PRINTED  FOR  BALDWIN    AND   CRADOCK. 


Digitized  by 


Google 


Digitized  by 


Google 


.  •    k 


•I  ■■ 


k  •»•    .• 


.if 


Digitized 


by  Google 


Digitized  by 


Google 


Digitized  by 


Google 


Digitized  by 


Google 


It 


"-^^^-t^B, 


7/^7 


Digitized  by  VjOOQIC 


2-^/*/J 


THE  UNIVERSITY  OF  CAL 


oogle 


~d^30 


mr 


■■'-^-^947/ 


?^o 


n> 


^^-PB^,,^ 


^7 


Digitized  by  LjOOQIC